Deepak Marwah Medicine Mcqs-Compressed - Compressed

You might also like

Download as pdf or txt
Download as pdf or txt
You are on page 1of 265

First Medicine Book with Harrison's 19th References

Covering 5000 MCQ’s of all recent exams upto JIPMER & AIIMS May 2015
and covering MOST IMPORTANT CHANGES OF Harrison’s 19 / e for PGMEE Aspirants

MARWAH’s
Internal Medicine
MCQ’s
References from the latest edition of Harrison’s 19/e 2015 & 18/e
• Cardiology
Deepak Marwah • Neurology
• Endocrinology
• Respiratory system
• Hepatology
• GIT
• Rheumatology
• Kidney
• Hematology
Special features • Fluid and
• More than 5000 Q’s arranged Topic wise with Authentic Answers & electrolytes
updated from Harrison’s 19th Edition.
• Latest NBE / DNB Pattern Questions covered ffom 2012 to 2015 • Nutrition
• Most Recent Questions covered from JIPMER & AIIMS May 2015
• Other Questions from AIPGMEE , AIIMS, PGI, JIPMER , UPSC & Other • Tumors
states from 2000 to 2014
• Most Important / Recent Updates covered along with Important Tables & • Genetics
I Flowcharts based on Harrison’s 19th edition.
I • Best book for revising Medicine subject as a Last Minute Revision • Infection SPsT
CBS
miiiiiiiiiimmmiiiiiiiiimimiiiimimmiii TAB LE OF CO NT EN TS |ltlllllllllIlllltlH!lllHl!lll< llillllllllUlilllllll

1. Cardiology 1 - 34

2. Neurology 35 - 72

3. Endocrinology 73 - 100

4. Respiratory System TO1 - 126

5. Hepatology 127 - 146

6. GIT 147 - 163

7 . Rheumatology 164 - 185

8. Kidney 186 - 205

9. Hematology 206 - 228

.
10 Fluid and Electrolytes 229 - 237

11. Nutrition 238 - 242

.
12 Tumors 243 - 250

.
13 Genetics 251 - 254

14. Infection 255 - 264


CHPkPTER Cardiology
1
CARDIOLOGY UPDATES
> Sodium nitroprusside is no longer the treatment of choice in acute hypertensive emergency which is treated with combination of
clevidipine or nicardipine plus labetalol or esmolol.

> Beta blockers are not the ideal first line agents for management of hypertension because of lack of efficacy in primary prevention of
Ml and it has proved to be inferior compared to other drugs in prevention of stroke .

Most Recent Q’s 2014- 15 6. Standard ECG leads are unable to detect?
[JIPMERJune 2015]
1. Best hypertensive drug in pulmonary hypertension? a. LVF
b. RBBB
[AIMS May 2015]
c. Right ventricular infarct
a . Bosentan b. Amlodipine d. Arrhythmia Ref: Harrison 19th 1458f
c. B blocker d . Diuretics
7. Most common cause of death in acute MI is:
Ref: Harrison 19“ ' p 1659
[ NBE Pattern 2014]
2. Diuretic which can be given in mild to moderate hy¬ a. VF b. AF
pertension? [AIIMS May 2015] c. Stroke d . Left ventricular rupture
a. Loop diuretic Ref: Harrison 19 th p 1596
b . Thiazide 8. Most common cause of endocarditis in IV drug abus ¬
c. Osmotic diuretic ers is: [NBE Pattern 2014]
d . Potassium sparing diuretic a. Coagulase negative staph
Ref: CMDT 2015 p 441 b. Staph aureus
c. Enterococci
3. Which of the following is not a part of Duke Criteria for d. Streptococci Ref: Harrison 19th p 823
infective endocarditis? [ AIIMS May 2015]
9. A 20 year old female presented to casualty with histo¬
a. Splenomegaly ry of sudden onset of chest tightness and palpitation.
b. Fever > 100.4 Celsius An ECG was taken, with is shown below. What could be
c. Blood culture positive the probable diagnosis? [ NBE Pattern 2014]
d. IV Drug abuse a. AF b. STEMi
,
Ref: Harrison 19 h p 819 c. VT d. PSVT
Ref: Harrison 19 th p 1483 f, 1484
4. A girl comes with symptoms of Sydenhams chorea and
acute rheumatic fever is suspected . Other major criteria 10. All are characteristics of Cardiac tamponade excepts.
of Rheumatic fever are absent. No evidence of sore throat. [NBE Pattern 2014 ]
Best investigation to prove rheumatic etiology is? a . Pulsus paradoxus
[ AIIMS May 2015] b. Kussmaul's sign
c. Prominent x descent in J VP
a. ASLS b. ASLO
d. Right ventricular diastolic collapse on echo
c. Throat culture d . Blood culture
Ref: Harrison 19th p 1463t
Ref: Harrison 19 th p 2152
11. Use of Digoxin is contraindicated in WPW syndrome.
5. Upon ECG lead II , III, and aVF are not normal . Which of Why? [ NBE Pattern 2014]
the following vessel is blocked? [JIPMER June 2015] a. It can precipitate asystole
a . Left coronary artery b. It can precipitate ventricular fibrillation
b. Left anterior descending c. It causes complete AV block
c. Right coronary artery d. They are more susceptible to digoxin toxicity
d. Right circumflex artery Ref: Harrison 19“ ' 1598t Ref: Harrison 19th p 1481

Ans. 1. a. Bosentan 2. b. Thiazide 3. a . Splenomegaly 4 . b . ASLO


5. b. Left anterior... 6. c. Right ventricular 7. a . VF 8 . b. Staph aureus
9. d. PSVT 10. b. Kussmaul’s... 11. b. It can precipitate ventricular fibrillation
Marwah s Internal Medicine MCQs (Based on Harrison's 19th )

.
12 What is the treatment given for the condition in ECG 21. Pulmonary apoplexy is seen in which of the following
given below: [ NBE Pattern 2014] conditions? [DNB Pattern 2014]
a. Adrenaline b. Atropine a. MR b AS .
c. Amiodarone d. Adenosine c. MR d. MS
Ref: Harrison 19th p 1770, 1469 Ref: Harrison 19th p 1443
I! 13. Inferior Rib notching is seen in: [ NBE Pattern 2014] 22 . Auenbrugger's sign is seen in? [DNB Pattern 2014]
.
a Coarctation of aorta
a. Pericardial effusion
i
"’ b. Marfan syndrome
b. Pericardial tamponade
.
c Rheumatoid arthritis
c. Pericarditis
d. Osteogenesis imperfecta
d. Pericardial tumors Ref Harrison 19th p 474
Ref: Harrison 19th p 1620
' .
14 Which sign is a pansystolic mumur on deep inspira 23 . Ewart’s sign is seen in? [DNB Pattern 2014]
.
¬

tion? a Pericardial effusion


[ DNB Pattern 2014]
a. Carvallo's sign b. Muller's sign b. Pericardial tamponade
c. Corrigan's sign d. Duroziez sign c. Pericarditis
d. Pericardial tumors Ref: Harrison 19th p 474
Ref: Harrison 19th p 1548
.
15 Pulsus alternans is seen in? [DNB Pattern 2014] 24. Pulsus paradoxus is seen in? [DNB Pattern 2014]
a. Severe cardiac failure a. Cardiac tamponade
b. Aortic regurgitation b. Mitral regurgitation
c. c. Aortic regurgitation

Cardiolgy
HOCM
d. Mitral stenosis Ref: Harrison 19th p 1446 d. Mitral stenosis
16 . Pulsus bisferiensis seen in? [DNB Pattern 2014] Ref: Harrison 19th p 1573
a. Aortic regurgitation
b. HOCM CARDIOLOCV UPDATES
c. CHF
> Choice of Anti_ Hypertension Drugs
d. Cardiac tamponade
Ref: Harrison 19th p 1562 Age <55 years | Age > 55 years

17 . Pulsus parvus et tardus is seen in? [DNB Pattern 2014]


a. Severe AS b. Severe MS Stepl ACE! | CCB
c. Severe AR d. Severe MR
Ref: Harrison 19th p 1445 Step 2 ACEI+CCB
18 . Carvallo's sign is seen in? [ DNB Pattern 2014]
a. Tricuspid regurgitation
Step 3 ACEI+CCB+DIURETICS
b. Mitral Stenosis
c. Aortic Stenosis
d. Aortic regurgitation Step 4 Resistant hypertension ( MC Cause is Non-
Compliance)
19. Mid diastolic murmur is seen in which of the follow¬
[DNB Pattern 2014] A +C+D+ Consider further
ing?
a. AS b. MS Diuretic or alpha-or

c. TR d. MR beta-blocker

Ref: Harrison 19th p 1443


20 . Duroziez's sign is seen in [DNB Pattern 2014] 25 . Read the following ECG. It demonstrates which of the
a. Mitral Stenosis following? [ DNB Pattern 2014]
a. SVT
b. Mitral Regurgitation
b. WPW syndrome
c. Tricuspid Stenosis
d. Aortic Regurgitation
.
c Atrial fibrillation
d. Ventricular fibrillation
Ref: Harrison 19th p 1641
Ref: Harrison 19th p 1498

Ans. 12. b. Atropine 13 . a. Coarctation of aorta 14.a. Carvallo’s sign 15. a . Severe cardiac failure
16. b. HOCM 17. a. Severe AS 18. a. Tricuspid regurg... 19. b. MS
20. d. Aortic Regurg... 21. d. MS 22. a. Pericardial effu.. 23 . a . Pericardial effusion
24. a. Cardiac tam... 25. d. Ventricular fibrillation
Cardiology

26. What is the Median survival time in cardiac amyloido ¬

CARDIOLOGY UPDATES
sis? [DNB Pattern 2014]
a. 1- 3 months > Hypertensive emergency definition
b. 3 - 6 months
BP must be reduced in one hour
c. 6 -12 months
It Includes any of the above
d. 12- 18 months Ref: Harrison 19th p 723 1. Hypertensive encephalopathy (headache+ irritability+
27. During cardiac imaging the phase of minimum mo ¬ confusion altered sensorium)
tion of heart is: (AI 2010 ) 2. Hypertensive nephropathy (Hematuria+ proteinuria
a. Late systole and progressive kidney damage)
b. Mid systole 3. Intracranial haemorrhage
.
c Late diastole 4 . Aortic dissection
d. Mid diastole 5. Eclampsia
6. Pulmonary edema
Ref: Harrison’s Advances in Cardiology by Braunwald (2002/ 227) 7. Unstable angina
8. Ml
Murmurs & Heart Sounds
32. Y steep decent is seen in: ( Rajasthan 2009]
28. Treatment of choice for ST segment elevated myocar ¬ a. Constrictive pericarditis
dial infraction (STEM1): [JIPMER 2014] b. Cardiac tamponade

Cardiolgy
a. Thrombolysis with altepase c. RVH
b. tPA with streptokinase d. TR Ref: Harrison 19th p, 1443
c. Primary percutaneous coronary intervention
d. Low molecular weight heparin 33. All of the following murmurs may be heard in patients
with aortic regurgitation except: ( AIIMS Nov 02 ]
Ref: Harrison's 18th Pg: 2039 Harrison 19th p 289e -3
a. High-pitched decrescendo diastolic murmur.
29. Loud pulmonary component of second heart sound
b. Soft, low pitched mid distolic rumbling murmur.
heard in: (PCI June 04)
c. Mid-systolic ejection flow murmur
a. Pulmonary hypertension
d. Pansystolic murmur.
b. TOF
c. Eissenmenger's syndrome Ref: Harrison’s 18/ e pl 944, Harrison 19th p 51 e-6
d. Pulmonary stenosis 34. A young female presents with history of dyspnoea on
e. AS exertion. On examination, she has wide, fixed split S2
Ref: Harrison's 18/ e p2076, 1923 , Harrison 19th p 51 e-l with ejection systolic murmur (III/ VI) in left second
intercostal space. Her EKG shows left axis deviation.
30. What is false in relation to Carey Coombs Murmur ?
The most probable diagnosis is:
a. Delayed Diastolic Murmur (AIIMS Nov 06)
( AIIMS May 04] (AIIMS May 03]
b. Seen in Rheumatic Fever
a. Total anomalous pulmonary venous drainage.
c. Associated with AR b. Tricuspid atresia.
d. Low Pitched Murmur c. Ostium primum atrial septal defect.
Ref: Internet , Harrison 19th p 51e-6 d. Ventricular septal defect with pulmonary arterial
31. Closed mitral valvotomy is contraindicated in: hypertension.
(UP 2011) Ref: Harrison's 18/ e pl 921, 1922; Harrison 19th p 1520
a. Associated with tricuspid regurgitation (TR) 35 . The murmur of hypertrophic obstructive cardiomy ¬

b. Associated with severe pulmonary arterial hyper ¬

opathy is decreased in which of the following:


tension [PAH) (AIIMS Nov 2000]
c. Calcified valve a. Supine position b. Standing position
d. Left atrial thrombus c. Valsalva maneuver d. Amyl nitrate inhalation
Ref: Harrison’s 18/ e pl 932, 1933, Harrison 19th p 1542f Ref: Harrison's 18/ e pi 969, Harrison 19th p 51e- l
Ans. 26. c. 6-12 months 27. d. Mid diastole 28. a. Thrombolysis 29 . a . Pulmonary hypertension
30. c. Associated 31. a. Associated with 32 . d. TR 33. d. Pansystolic murmur
34. c. Ostium primum 35. a. Supine position
th
Marwah's Internal Medicine MCQs ( Based on Harrison’s 19 )

I > Treatment of choice for hypertensive emergency

Myocardial ischemia Nicardipine and esmolol

Acute kidney injury Fenolodopam


Aortic dissection Esmolol plus nicardipine
Acute pulmonary edema ( systolic dysfunction) Nicardipine plus NTG and loop diuretic
Acute pulmonary edema ( diastolic dysfunction) Esmolol plus low dose NTG and loop diuretic
Acute ischemic stroke SBP> 180-200mmHg Nicardipine
Intracerebral haemorrhage SBP> 140-160 mmHg Nicardipine
Hyperadrenergic states ( cocaine use) Nicardipine and Benzodiazepine
Pre-eclampsia/ eclampsia Labetalol

36. During cardiac imaging the phase of minimum mo ¬ 41. Reverse split S2 is seen in : (NBE Pattern 2014-15)
tion of heart is: (AI 2010] a. Aortic stenosis b. Mitral stenosis
a. Late systole b. Mid systole c. Pulmonary artery hypertension
c. d. Mid diastole d. Pulmonic stenosis

Cardiolgy
Late diastole
Ref: Internet, Harrison 19th p 270e- l Ref : Harrison's 18th
p 1939 Harrison 19th p 1531
37 . Right murmur that decreases in intensity with inspi ¬ 42 . All are true about Physiological murmur except:
ration (JIPMER 2014) a. Only Diastolic murmur (NBE Pattern 2014-15)
a. Pulmonary ejection click in pulmonic stenosis b. Midsystolic murmur
b. Tricuspid stenosis c. Present in child with anemia
c. Tricuspid regurgitation d. Not audible without stethoscope
d. Pulmonic regurgitation
Ref: Harrison 17th ed ch. 220 Harrison 19th p 1448
Ref: Braunwald 8th edn p 106, Harrison 19th p 1450 43. Loud P 2 is found in? (NBE Pattern 2014-15)
38. P.S.T is: (NBE Pattern 2014-15) a. Pulmonary HTN b. MS
a. Phenol sulfotransferase deficiency c. MR d. Aortic incompetence
b. Post traumatic stress disorder
Ref: H18th ed. p 2046 Harrison 19th p 1655
c. Protein S deficiency
44. Systolic murmur is associated with?
d. Paroxysmal supraventricular tachycardia
(NBE Pattern 2014-15)
Ref: Net Source: Pubmed a. Ejection click b. Opening snap
39. Wrong about continuous murmur: c. S4 d. Pericardial knock
(NBE Pattern 2014-15)
Ref: Harrison 19th p 1448
a. Seen with coarctation of aorta
b. Peaks at S 2 45. Severity of mitral stenosis is assessed by:
c. Heard both in systole and diastole a. SI (NBE Pattern 2014-15)
d. Increase on squatting b. Loudness of opening snap

I 40.
Ref: HI 8th ed , p 1829 ch. 227, Harrison 19th p 1449
Which is not a high pitched heart sound :
c.
d.
Intensity of murmur
Duration of murmur
Ref: Harrison's, 18th ch. 237 Harrison 19th p 1540

I
a. Mid systolic click (NBE Pattern 2014-15)
b. Pericardial shudder 46. All of the following heart sounds occur shortly after
c. Opening snap S 2 except: (AI 2003)
d. Tumor plop sound a. Opening snap b. Pericardial knock
c. Ejection click d. Tumor plop

I Ref: HI 8th ed., ch. 227, Harrison 19th p 1447

Ans. 36. d . Mid diastole 37. a. Pulmonary ejection


40. d. Tumor plop sound 41. a . Aortic stenosi
Ref: Harrison 18th/ p 1826, 1827 Harrison 19th p 1447
38. a. Phenol sulfotransferase 39. d. Increase
42. a. Only Diastolic murmur 43. a . Pulmonary HTN
44. a. Ejection click 45. d. Duration of murmur 46.c. Ejection click
Cardiology

c. Heparin d. PTCA
CARDIOLOGY UPDATES
Ref H18/ e p2021 17/ e pl 537, Harrison 19th p 1494

> Hypertensive urgency Definition 52. Raised serum level of lipoprotein- (a) is a predictor of:
BP must be reduced in few hours a. Cirrhosis of liver (AI 03) II
Asymptomatic severe hypertension SBP > 220/125mmHg b. Rheumatic arthritis
Optic disc edema c. Atherosclerosis d. Cervical cancer
Progressive target organ complications Ref: Harrison's 18/ e pi 987, Harrison 19th p 291 e-4 |
Severe peri-operative hypertension
53. Which of the following increases the susceptibility to j
Parenteral drugs not used.
coronary artery disease: (AI 03) I
a. Type V hyperlipoproteinaemia
47. Which of the following murmurs increase with Val ¬
b. Von willebrands disease
salva maneuver? (AllMS Nov 2010) c. Nephrotic syndrome
a. MR b. VSD d. Systemic lupus erythematosus
c. AS d. HOCM
Ref Harrison 19th p 249
Ref: Harrison 18th/ 1830 Harrison 19th p 1450
48. 54. Which of the following statements about Atheroscle ¬

Continuous murmur is found in all, EXCEPT:


a. rosis is true: (ARMS May 2011)
Mitral stenosis with mitral regurgitation
b. Patent ductus arteriosus (AllMS May 93) a. Intake of Unsaturated Fatty Acids is associated with
c. Rupture of sinus of Valsalva decreased risk

Cardiolgy
d. Systemic Arterio Venous (AV) fistula b. Extent of lesions in veins is similar as that in arteries
Ref: Hurst's 12th /290 Harrison 19th p 1449 c. Thoracic Aorta is more commonly involved than
abdominal aorta
d. Hypercholesterolemia alone does not increase the
risk of atherosclerosis per se
49. Reversible myocardial ischemia is best diagnosed by: Ref : Internet, Harrison 19th p 291 e- 4
[J1PMER 2014]
a. MUGAscna b. Thallium scan 55. All of the following arteries are common sites of oc¬
c. PET scan d. Coronary angiogram clusion by a except: (ARMS May 2005)
a. Left anterior descending
Ref: Harrison's 18th. Pg: 1844, Harrison 19th p 1440
b. Right coronary artery
50. In an established case of coronary artery disease c. Circumflex coronary artery
(Post MI) which of the following drug will not prolong
[JIPMER 2014] d. Marginal artery
life expectancy?
a. ACE inhibitors b. Statin Ref: Robbin's 8/ e p551, Harrison 19th p 1642
c. Aspirin d. Nitrate
Reperfusion is believed to restore contractile func ¬

Ref: CMDT 2013, pg. 370, Harrison 19th p 1439 tion of: (ARMS May 2011 )
51. A 30-year-old male presents with severe pain chest, a. Stunned Myocardium
breathlessness, hypotension and ECG shows ST eleva ¬
b. Hibernating Myocardium
tion in V 3, V4, V5 and V6 leads. He will be best treated c. Ischemic non- viable myocardium
with: (DP PGMEE 2009) d. Non ischemic viable myocardium
a. Streptokinase b. t- PA
Ref: Harrison 19th p 1604
CARDIOLOGY UPDATES
> Malignant Hypertension
Hypertensive Encephalopathy or nephropathy with hypertensive retinopathy
Progressive kidney damage ensues
> Acute ischemic stroke
Treat only if SBP> 180-200 and should be reduced by 10-15%
If thrombolysis is indicated then BP < 185/110 during and 24 hours after treatment

Ans. 47 . d . HOCM 48. a . Mitral stenosis with 49. c. PET scan 50. d. Nitrate
51. d. PTCA 52. c. Atherosclerosis 53. c. Nephrotic syndrome 54. a. Intake of Unsaturated Fatty
55. d. Marginal artery 56. b. Hibernating Myocardium
i
L Marwah'

57 .
s Internal Medicine MCQs ( Based on Harrison's 19 th
)

Which of the following drugs has been linked with in ¬ 64. Which is best for plaque morphology?
creased cardiac mortality: (Al 2012 ) a. CCTA [ NEET Pattern 2015-16)
a. Rofecoxib b. Metoprolol b. MRI
c. Losartan d. Nicorandil c. CMR

3
Ref: Goodman Gilman 11 / e p684, 686 d. 1VUS Ref: CMDT 2015, p 354
58 . A myocardial infarct showing early granulation tissue 65. All are true regarding universal definition of MI ex¬
has most likely occurred: (Al 02 ) cept?
a. 1hours old b. 24 hours old a. Sudden death is seen in MI (A1IMS May 2012)
c. 1week old d. 1month old b. New regional wall motion with increased biochemi ¬

Ref: Robbin’s 8/ e p550; Table 12-5, Harrison 19th p 1598t cal marker is MI
c. 3 x Troponin elevation is required for CABG related MI
j 59. Non ischaemic chest pain is caused by:
d. Re -infarction can be diagnosed if elevation in
(Rajasthan 2009)
a. Bleomycin b. Vincristinum troponin level by 5 to 7 % on serial sampling
c. Cyclophosphamide d. Cisplatunum Ref: Harrison 19th p 1602 Table 295.2
Ref: Harrison's 18/ e pl 362, Harrison 19th p 103e-13 66. A patient suffered a transmural myocardial infarc-
60. Best test for viable myocardium after MI? .
tion He was started on streptokinase and warfarin
(JIPMER 2014) .Which of the following finding on further examina ¬

a. Thallium scan b. MUGA scan tion is an indication for stopping thrombolytics ?


c. MDCT d. Stress Echocardiography (ARMS May 2013)
a. Mobitz type II block
Ref: Braunwald 8th edn table 16-1, CMDT 2015 p 354

Cardiolgy
b. Pericardial fluid collection on Echo
61. Which drug prolongs life in patient of chronic stable c. Deep vein thrombosis in the leg
angina? (JIPMER 2014) d. Pericardial friction rub
a. Aspirin b. b-Blocker
c. Diuretic d. ACE inhibitor Ref Harrison's 18th ed. p-2028 Harrison 19th p 1610
Ref: Braunwald 8th edn ch 54, CMDT 2015 p 356 67 . A young male patient presents with LDL 600 mg/

62. A patient with anterior wall Ml was thrombolysed


.
dl, triglycerides 140 mg/ dl What would be the most
likely finding on physical examination?
.
within 6 hours with STK On day 3 he had fever with (AIIMS May 2012 )
.
chills and platelet count of 60,000 Which of the fol ¬
a. Tendon xanthoma
lowing is responsible for this picture? b. Lipemia retinalis
(NEET Pattern 2015-16)
c. Palmar xanthoma
a. Aspirin b. Ranolazine d. Tuberous xanthoma
c. STK d. Clopidogrel
Ref Harrison's 18th ed., table 356 -3, Harrison 19th p 2440
Ref: Harrison 18th edn p 990 ch 118, Harrison 19th p 745
63. ROSE questionnaire is used for?
68 . Thrombolytics can be given in treatment of acute
myocardial infarction, if patient comes within:
(NEET Pattern 2015-16)
a. 3 hrs (ARMS May 2013)
a. Alcohol addiction b. 6 hrs
b. Sex addiction c. 12 hrs
c. Angina assessment
d. 24 hrs
d. Deep Vein thrombosis Assessment
Ref: Braunwald 8th edn Ch 57 Ref Harrison's 18th ed. Ch. 245 P-2027
CARDIOLOGY UPDATES
> In Hemorrhagic stroke target Mean arterial pressure of < 130 mm Hg

> Pulmonary Artery Hypertension Treatment Update


Initiate warfarin in idiopathic PAH with diuretics + oxygen + digoxin
Salt restricted diet
CCB or sildenafil
Poor response to vasodilators use BOSENTAN
Good response to vasodilators use Epoprostenol/ inhaled iloprost/ subcutaneous trepopostinil

Ans . 57. a. Rofecoxib 58. c. 1 week old 59 . a . Bleomycin 60 . a . Thallium scan


61. b. b-Blocker 62. d. Clopidogrel 63. c. Angina assessment 64 . a. CCTAy
65. c. 3 x Troponin elev.. 66. b. Pericardial fluid.... 67. a. Tendon xanthoma 68. c. 12 hrs
Cardiology

CARDIOLOGV UPDATES
> Systolic Heart Failure Management Update

E
Diuretics + ACEI+ beta blocker
If still NYHA class ll-IV add Spironolactone
If still NYHA class ll-IV and LVEF< 35% add ivabradine
If still NYHA class ll-IV and LVEF< 35% with QRS> 120msec add Cardiac resynchronization therapy
If still NYHA class ll-IV then put LVAD=/eft ventricular assist device

69. Which of the following is not a contraindication to 75 . A patient of acute Myocardial infarction with ECG
thrombolysis? ( NBE Pattern 2014-15] showing ST segment elevation has severe hypoten ¬

a. BP>180/110 mm Hg sion. Immediate management of the patient is:


b. Diabetic retinal flame shaped hemorrhage (NBE Pattern 2014-15)
c. History of previous cerebral bleed a. Intra-aortic balloon pump
d. Aortic dissection b. PCI with angioplasty
Ref: Harrison's 18th ed., Harrison 19th p 1605 c. Thrombolysis
d. Intravenous fluids
70. Hypertension patient has presented with BP of
220 /130 mm Hg in the emergency with headache but Ref: H 18th 2234 ch. 272 Harrison 19th p 1762
.
no CNS deficit What is the goal BP for this patient? 76. Wellen's syndrome is seen with:

Cardiolgy
(NBE Pattern 2014-15) (NBE Pattern 2014 -15) j
a. 200/150 b. 180 /110 a. Stable angina b. Unstable angina
c. 160/90 d. 140/90 c. Prinzmetal angina d. Ludwig angina
Ref: European Society of Cardiology Guidelines 2015 Ref: Harrison's 18th ed. Ch-228 p 1836-37
71. Dose of streptokinase to be used in MI is? 77 . Half- life of alteplase? (NBE Pattern 2014-15)
a. 0.15 Million units (NBE Pattern 2014-15) a. 3 min b. 6 min
b. 1.5 Million units c. 9 min d. 12 min
c. 15 Million units Ref: Harrison's, p 1002, ch. 118. Harrison 19th p 759
d. 150 Million units
Harrison -18 th
ed. Ch -118 p 1001 Harrison 19th p 1605
78 . Persistent ST segment elevation 24 hours after treat¬
Ref :
.
ment for MI with P.CIis due to
72 . Dose of reteplase for management of MI is? (NBE Pattern 2014-15)
a. 5 IU ( NBE Pattern 2014 -15) a. Left ventricular aneurysm
b. 10 IU b. Impending cardiac rupture
c. 15 IU d. 50 IU c. Dressier syndrome
Ref: Harrison's 18th ed., ch. 118 p 1003 Harrison 19th p 759 d. Coronary artery dissection
.
73 Incorrect about Broken heart syndrome: Ref : Harrison's 18th ed., p 2034, Harrison 19th p 1610
a. Catecholamine toxicity (NBE Pattern 2014-15) 79. Most common arrhythmia seen after reperfusion strat ¬

b. ST elevation egy in MI? (NBE Pattern 2014-15)


c. Apical ballooning a. A.I.V.R b. V.T
d. Dobutamine for cardiogenic dysfunction c. V.fibrillation d. P.S.V.T
Ref: Harrisonl 8th ed., ch. 238 p 1964 Harrison 19th p 1565 Ref : Accelerated Idioventricular Rhythm ( AIVR); H 19th p 1609
74 . Most common cause of sudden death (< 24 hours) in 80 . Correct sequence of ECG changes in acute Ml is?
MI is? (NBE Pattern 2014-15)
(NBE Pattern 2014-15)
a. Cardiogenic shock
a. T inversion, ST elevation, Q wave
b. Ventricular fibrillation b. ST elevation, T inversion, Q wave
c. Mobitz 1heart block
c. ST elevation, Q wave, T inversion
d. Cardiac rupture
d. Q wave, ST elevation, T inversion
Ref: H 18th p 2032 ch. 245, Harrison 19th p 1602 Ref: Harrison’s 18th ed. Ch -228 p 1837 Harrison 19th p 1456
Ans. 69. b. Diabetic retinal... 70. c. 160/90 71. b. 1.5 Million units 72. b. 10 IU
73. d. Dobutamine 74. b. Ventricular fibrillation 75. b. PCI with angioplasty 76. a. Stable angina
77. b. 6 min 78. a. Left ventricular aneurysm.79. a . A .I.V.R 80. b. ST elevation,
Marwah's Internal Medicine MCQs ( Based on Harrison's 19th )

81. False positive troponin I can be seen in? 87. Most common cause of Secondary hypertension is?
a. DC shock ( NBE Pattern 2014-15) a . Renovascular disease ( NBE Pattern 2014 -15]
b. Pulmonary embolism b. Pheochromocytoma
c. Chronic liver failure c. Renal parenchymal disease
d . Renal failure d. Hyperthyroidism
Ref : Cardiac intensive care 2nd edition, pg. 199 Ref: ch. 247: Harrison 18th ed. p 2048 Harrison 19 th p 1617
82 . A hypertensive patient is brought with a BP of 220 88. An athlete presents with chest pain, which of the fol ¬
/130 mmHg to the casualty. CT scan shows an infarct. lowing tests would be most sensitive and specific for
What will be the primary BP of the patient to start diagnosis of myocardial infarction?
thrombolysis? ( AlPGJan. 2012]
a. 200 /150 ( NBE Pattern 2014-15) b. Creatine kinase - MB
a . Troponin I and T
b. 180 /110
c. Leukocytosis d. C-reactive protein
c. 165/120
d. 140 /90 Ref: Ch 245, P 2028 Harrison 19 th p 2562 Ref: Harrison’s ; 18 ed., ch 245 p 2023 Harrison 19 th p 1600
83 . Most common cause of Resistant hypertension is? 89. All are true regarding Prinzmetal's Variant Angina
a. Non-compliance of patient ( NBE Pattern 2014-15) ( PVA) except? ( Bihar PG 2014]
b. Obstructive sleep apnea a. Detection of transient elevation of ST segment with
c. Pheochromocytoma rest pain
d. Renovascular disease Ref: Harrison 19th p 1626 b. Silent ischemia
84. CAD is related to decreased levels of which HDL: c. Focal spasm is most common in left coronary

Cardiolgy
a. HDL1 ( NBE Pattern 2014 -15) artery
b. HDL 2 d . Coronary angiography which demonstrates tran ¬

c. HDL 3 d. HDL 4 sient coronary spasm is the diagnostic hallmark


Ref: Disorders of Lipo- protein Metabolism, ch. 356 Ref Harrison's 18th ed., ch. 243 Harrison 19 th p 1598
85 . All are associated with increased risk of coronary 90. Which of the following is associated with atheroscle ¬
events except: ( NBE Pattern 2014 -15) rosis? ( NBE Pattern 2014-15]
a. High sensitivity CRP a. Chlamydia trachomatis
b. High Agatson score b. Chlamydia pneumonae
c. Intravascular IVUS showing lumen reduction c. Chlamydia psittaci
d . Ability to complete stage 3 of Bruce protocol d . Chlamydia gingivalis
treadmill Ref Harrison 17th ed ch. 169 Harrison 19 th p 1173
Ref: Harrison 18th, ch 234 p 2004 Harrison 19 th p 1582
91. Usual duration of chest pain in chronic stable Angina
86. In myocardial infarction, the correct sequence of in ¬ is? ( NBE Pattern 2014-15]
crease in enzyme levels is: ( NBE Pattern 2014 -15) a . 1-3 minutes b. 5-10 minutes
a. CPK, AST, LDH b. CPK, LDH , AST
c. 15 minutes d . 30 minutes
c. AST, CPK, LDH d . LDH , CPK, AST
Ref: Net source: Pubmed Ref: Harrison 19 th p 1580

CARDIOLOGY UPDATES
> ARRYTHMIAS

Arrhythmia ECG criteria Drug of choice


Atrial flutter .
Saw tooth waves in lead II, lll aVF Ibutilide for rhythm control
Esmolol of rate control
Multifocal atrial tachycardia Varying >3 different P wave morphologies Verapamil
SUSTAINED Ventricular Fast wide QRS complex on ECG > 30 Amiodarone followed by ICD due
tachycardia seconds. to tendency to recur following
structural damage to heart .

Ans. 81. c. Chronic liver.. 82. b. 180/110 83. a. Non-compliance 84 . b. HDL 2


85. d . Ability to com... 86. a . CPK , AST, LDH 87. c. Renal parenchyma 88. a. Troponin I and T
89. c. Focal spasm... 90 . b. Chlamydia pneumonae 91. b. 5-10 minutes
8
Cardiology

CARDIOLOCV UPDATES

> CHA2 DS2 -VASC RISK ASSESSMENT for ATRIAL FIBRILLATION

1 Risk factors
C- congestive heart failure
H- hypertension
A - age > 75 y
D - diabetes mellitus
1
1
Points

2
1
0
1
2
3
-
CHA 2 DS 2 VASC Score Estimated Annual Stroke Rate

0
1.3%
2.2%
3.2%
4.0%
E
S - stroke or TIA, embolus 2 4
V - vascular disease 1 5 6.7%
A - age 65-75 y 1 6-9 >9%

-
Sex female 1

92 . Pregnant lady with Hypertension with diabetes mel - 97. ST elevation is seen in all of the following conditions
litus requires which drug to control her BP? except: ( AIIMS 2002)
a. ACE inhibitors ( NBE Pattern 2014-15) a. Myocardial infarction

Cardiolgy
b. Beta blocker b. Coronary artery spasm
c. ARB d. Diuretics c. Constrictive pericarditis
Ref: Harrison's, ch. 247, Hypetensive Vascular Disease p 2056 d. Ventricular aneurysm Ref Harrison 19 th p 1456

93. Unstable angina is characterized by? 98. Exercise testing is absolutely contraindicated in which
( NBE Pattern 2014 -15) one of the following: ( Al 2003)
a. Decrescendo pattern of symptoms a. One week following myocardial infarction
b. Crescendo pain with ECG findings b. Unstable angina
c. ST segment elevation c. Critical aortic stenosis
d . Normal cardiac biomarkers d . Peripheral vascular disease
Ref: Harrison’s 18th ed ch. 244 p 2015 Harrison 19 th 1594 Ref Harrison 18th/ p 2004, Harrison 19 th p 1532

94. Which is incorrect about Dressier syndrome? 99. Aortic regurgitation does NOT occur in:
a. Post MI pericarditis ( NBE Pattern 2014-15) a. Acute MI ( AI1MS Sept 96 )
b. Post MI pleuritis b. Marfan's syndrome
c. Autoimmune c. Rheumatic heart disease
d. Treatment with steroids is necessary d . Infective endocarditis
Ref: Harrison's, 18th Ch. 239 p 1974 Harrison 19 th p 1610 Ref: Harrison 18th/ p 1930, 1942,1943, Harrison 19 th p 1535
100. All of the following are risk factors for atherosclerosis
95 . Dressler's syndrome is characterized by?
a. Onset within 72 hours ( NBE Pattern 2014 -15) except : ( Al 06 )
b. Treatment of choice is steroids a. Increased waist - hip ratio
c. Pericardial effusion b. Hyperhomocysteinemia
d. Angina c. Decreased fibrinogen levels
d. Decreased HDL levels
Ref Harrison's, 18th ed., ch. 239 p 1975 Harrison 19 thpl 610
Ref Harrison 18th/ p 1987, Harrison 19th p 1587 -88
96 . Most common cause of hypertension is?
a. Renal A. Stenosis ( NBE Pattern 2014 -15) 101. Which of the following is the best marker to predict
b. Essential HTN future cardiac events ( DNB June 2012)
c. Pheochromocytoma a. hs CRP b. Homocystine
d . Chronic Glomerulonephritis c. Interleuken -6 d. LDL
Ref Harrison 19 th p 1616 Ref: Braunwald’s 8th/ 1013, 1014, Harrison 19 th p 1582

Ans. 92. b. Beta blocker 93. b. Crescendo pain wit 94. d. Treatment with steroids 95. c. Pericardial effusion
96. b. Essential HTN 97. c. Constrictive pericarditis 98. c. Critical aortic stenosis 99. a. Acute Ml
.
100 . c. Decreased fibr ... 101. a. hs CRP
Marwah s Internal Medicine MCQs (Based on Harrison's 19th)

102 . Most important predictor of coronary artery disease 108. All of the following drugs are used in the management
is: (A1IMS May 09) of acute myocardial infarction, except: ( AIIMS May 04 )
a. VLDL b. LDL a. Tissue Plasminogen activator
MHII c. Chylomicron .
d LDL/HDL b. Intravenous beta blockers
Ref: Braunwald 8th /1008; Essential Cardiology 2nd/ 6; c. Acetylsalicylic acid
.
103 Most common site of myocardial infarction is: (AI 98) d. Calcium channel blockers
a. Anterior wall of left ventricle Ref: Harrison 18th / p 2026, Harrison 19th p 1603-4
b. Posterior wall of left ventricle .
109 Streptokinase and Urokinase are contraindicated in:
c. Posterior wall of right ventricle a. Intracranial malignancy (AI 2010)
d. Inferior wall of left ventricle b. Pulmonary Embolism
Ref: Robbins 6th/557 c. AV fistula
104. All of the following arteries are common sites of oc ¬ d. Thrombophlebitis
clusion except: (AIIMS May 2005) Ref: Braunwald’s 8th/1237, Harrison 19th p 1605
a. Left anterior descending .
110 A patient with acute inferior wall myocardial infarc ¬

b. Right coronary artery .


tion has developed shock Which of the following is
c. Circumflex coronary artery the most likely cause of shock: (AIIMS May 04)
d. Marginal artery a. Cardiac rupture
b. Interventricular septal perforation
Ref: Robbin's 7th/ 577-578; Snell’s 7th/ l 17-119 c. Papillary muscle rupture
j .
105 In stable angina : (AIIMS Nov 03) d. Right ventricular infarction
a. CK-MB is elevated
Ref: Harrison 17th /154, Harrison 19th p 1608

Cardiolgy
b. Troponin I is elevated
c. Myoglobin is elevated .
111 A new systolic murmur after Acute myocardial infarc ¬
d. The levels of cardiac markers remain unchanged tion may be due to all of the following except:
a. Complete heart block ( AIIMS May 1993)
Ref: Ch_ 243, Harrison 18th Harrison 19th p 1582 b. Rupture of Interventricular septum
106. A patient presents 12 hours following a Myocardial c. Papillary muscle dysfunction
infarction. Test of choice : (AIIMS June 98) d. Ischemic cardiomyopathy
a. Lactate dehydrogenase Ref: CMDT 2008/ 323
b. Cardiac troponins .
112 A hypertensive diabetic is having proteinuria, antihy¬
c. Creatinine phosphokinase pertensive of choice is: (AIIMS May 95)
d. Myoglobin a. Propanolol b. Clonidine
Ref H18th/ p 2016, 2023, 2024 4, Harrison 19th p 1600 c. Enalaprilat d. Alpha methyldopa
107. A patient presents with acute anterior wall infarction Ref: Harrison 19 th p 1623
and hyotension. Which will be the immediate treat ¬ .
113 All of the following statements regarding subendo ¬

ment modality for this patient: (A12007) cardial infarction are true, except: (AI 06)
a .Intra aortic balloon counter pulsation a. These are multifocal in nature
b. Anticoagulation b. These often result from hypotension or shock
c. Thrombolytic therapy c. Epicarditis is not seen
d. Primary angioplasty d. These may result in aneurysm
Ref Harrison 18th / p 2027, Harrison 19th p 1762 Ref: Robbins 7th/ 575

CARDIOLOGY UPDATES

>

Target

Class
specific

Bleeding
oral anticoagul ants for

Dabigatran
Antithrombin
stroke prevention

risk Less intracranial bleeding


in patients with nonvalvula

Rivaroxaban
Factor Xa inhibitor
Less intracranial bleeding
r atrial fibrillation.

Apixaban
Factor Xa inhibitor
Substantially lower risk of
compared to warfarin incidence of gastrointes tinal major bleeding less intracranial
Higher incidence of gastrointestinal Higher
bleeding
bleeding
bleeding

Ans.102. d. LDL/HDL 103 .a. Anterior wall of left 104. d . Marginal artery 105. d. The levels of cardiac
106. b. Cardiac troponins107. d . Primary angioplasty 108 . d. Calcium channel 109. a. Intracranial
110. d. Right ventricular 111. a . Complete heart 112. c. Enalaprilat 113 . d. These may result in
Cardiology

:l »] r »] MT«vJ » rJ »7 i 4
^
> Rheumatic Mitral Stenosis Grading

Very severe MS
MVA 1.0 cm2
10 mm Hg
TYz 220 msec

Rheumatic Fever
T
MVA 1.5 cm2
Severe MS

Pfnean >5 mm H9
1 5 0 msec
MVA > 1.5 cm2
P
TVi < 150
Progressive MS

5 mm Hg
msec E
120. A 22 year female comes with complains of dyspnea
114. True about acute rheumatic fever: [ Manipal 2014 ] and palpitations since 4 years shows verrucous veg ¬
a . Group A streptococci has been linked to it epidemio- etations of mitral valve. The condition is due to:
logically ( DNB 2013)
b. Usually associated with history of sore throat a. Libman Sachs endocarditis
c. All are sensitive to pencillin b. Rheumatic endocarditis
d . All of the above Ref: Harrison 19th p 2152 c. Non bacterial endocarditis
115. In cases of Streptococcal pharyngitis, how early should d . Infective endocarditis
the treatment be initiated to effectively prevent Rheu¬
Ref: Harrison' s 18/ e p422, 1053, Harrison 19th p 2136

Cardiolgy
matic fever? [J 1PMER 2014]
a. 7 days b. 8 days 121. Not common in RHD? ( AIIMS Nov. 14)
c. 9 days d. 10 days a. MS b. AS
Ref: Nelson Textbook of Pediatrics, 19th edn. and Harrison' s Prin ¬
c. PS d. TS
ciples of Internal Medicine, 18th edn, Harrison 19 th p 231 Ref: OP Ghai 8th edn p 436
116. A 10 -year-old boy, Pappu, died of acute rheumatic fever. Ref: Harrison' s 18th edn ch 332, p 2752, Harrison 19th p 2150
All the following can be expected at autopsy except:
( AI 02) 122 . Which of the following is not seen in mitral stenosis
a. Ashoff nodules ( AIIMS Nov. 14 )
b. Rupture of chordae tendinae a. Loud SI
c. Me Callum patch b. Opening snap
d. Fibrinous pericarditis c. Mid diastolic murmur
Ref: Harrison's 18/ e p2754, Harrison 19th p 2152 d. S 3
117 . True about Rheumatic fever: ( PCI Dec 03 ) Ref: Harrison' s 18th edn ch 237 p 1930, Harrison 19th p 1540
a. Chorea is aggravated during pregnancy
123. Sequel of rheumatic heart disease in a 5 year old child
b. Chorea and arthritis co-existing
c. Subcutaneous nodules are tender is? ( NBE Pattern 2014-15)
d. Erythema multiforme seen a. Mitral regurgitation
Ref: Harrison's 18/ e p2754, 17/ e p2094, Harrison 19th p 2152 b. Mitral stenosis
c. Tricuspid stenosis
118. Which of the following is least likely to be associated
with Infective Endocarditis: ( AI 2012) d. Tricuspid regurgitation
a. Small ASD
c. Mild MR
b. Small VSD
d. Mild MS
-
Ref : Harrison' s 18th ed., , ch. 322 2753 Harrison 19th p 2150 51
124. A patient has got a history of hypersensitivity to peni ¬
Ref: Harrison's 18/ e pl 863, 17/ e pl 922 , Harrison 19th p 821
cillin. What is the drug that can be used for rheumatic
119. Which of the following have most friable vegetation: fever prophylaxis in such a patient.
a. Infective endocarditis (Al 2010)
b. Libman Sack's endocarditis ( AIIMS May 2013)
c. Rheumatic heart disease a. Sulfisoxazole b. Sulfasalazine
d . SLE c. Streptomycin d. Sulfadiazine
Ref: Harrison's 18/ e pl 053, Harrison 19th p 821 Ref: AHA Guidelines 2010 and Nelson ch. 438
Ans . 114. d . All of the above 115. c . 9 days 116 . b. Rupture of chordae 117 . a . Chorea is aggravated
118 . a . Small ASD 119 . a . Infective endocarditis 120 . a . Libman Sachs 121 . c . PS
122. d . S 3 123. a . Mitral regurgitation 124. d . Sulfadiazine
B
Marwah's Internal Medicine MCQs (Based on Harrison's 19th)

125. Which of the following is a minor criteria for diag ¬


Valvular Heart Disease
nosis of Rheumatic Fever (RF) according to modified
Jones criteria? (AI 2007 )
a. ASO titre 131. All are seen in Mitral stenosis except: [ Manipal 2014]

Q
a. LVH
b. Past History of Rheumatic Fever
c. Fever b. Left atrial dilation
d. Subcutaneous nodules c. Pulmonary hypertension
d. Atrial fibrillation Ref: Harrison 19th p 1551
Ref: Harrison 16th / 1978, Harrison 19th p 2150
132. A 55 - year - old hypertensive patient has a standing BP
126. Characteristic feature of Rheumatic carditis is :
190 /105 and sitting BP - 180 /100. He also has irregu ¬

(AI 99)
larly irregular rhythm, double apical impulse and bi ¬

a. Pericarditis b. Endocarditis
lateral basal crepitations but no murmurs. Heart rate
c. Myocarditis d. Pancarditis
cannot be determined. What is the likely cause?
Ref: Harrison 18th/ 2754, Harrison 19th p 2151 [ JIPMER 2014 ]
127. Diagnostic feature in rheumatic heart disease is : a. Left atrial myoma
(AI 97) b. Mitral regurgitation
a. Aschoff's nodule c. Cor pulmonale
b. Me Callman patch d. Left ventricular hypertrophy
c .
Adeno Carcinoma Ref. Harrison's 18th edn. Harrison 19th p 1817 , 1626 -1627

d. Alveolar cell Carcinoma


133. The severity of mitral stenosis is assessed by:
Ref: Harrison 18th/2754, Harrison 19th p 2151 (Rajasthan 2009)

Cardiolgy 128. True about Erythema Marginatum in Acute Rheumat¬ a. Left atrial enlargement
ic fever is: (PCI Dec 01) b. Loudness of SI
a. Pruiritic c. Loudness of opening snap
b. Commonly involves face d. A 2-0S gap
c. Common manifestation of Acute Rheumatic fever Ref: Harrison's 18/ e p3606, Harrison 19th p 1551
d. Usually associated with carditis
134. The severity of mitral regurgitation is decided by all
Ref: Harrison 18th/ p 2754, Harrison 19th p 2151 of the following clinical findings except:
129. Earliest valvular lesion in the case of acute rheumatic (AIIMS May 03)
fever is: (AlIMS May 94) a. Presence of mid-diastolic murmur across mitral valve.
a. Mitral regurgitation (MR) b. Wide split second heart sound.
b. Aortic Regurgitation (AR) c. Presence of left ventricular S3 gallop.
c. Mitral stenosis (MS) d. Intensity of systolic murmur across mitral valve
d. Aortic Stenosis (AS)
Harrison 18th / p 2753, Harrison 19th p 2151 Ref: Harrison’s 18/ e pl 935, Harrison 19th p 1551
Ref:
135 . All of the following are true for mitral valve prolapse,
130. Site of lesion in endocarditis of RHD is: (PG1 Dec- 97)
except: (AI 06)
a. Along line of closure of valves
b. Both sides of valves a. Transmission may be as an autosomal dominant
c. Valve cusps trait
d. Free margin of valves b. Majority of the case present with features of mitral
Ref: Robbins 7th/ 597 regurgitation
c. The valve leaflets characteristically show myxoma
wm
¬

rfT:l >l [ » l « » I W J H t l »7r


I d.
tous
The disease is one of the common cardiovascular
manifestations of Marfan’s Syndrome
> MITRAL REGURGITATION GRADING
Ref: Harrison’s 18/ e pi 937, Harrison 19th p 837
Severe MR Progressive MR 136. A five year old child presents with Left ventricular
Vena Contracts >0.7 cm Vena Contracta <0.7 cm hypertrophy and central cyanosis. What is the most
R Vol> 60cc R Vol <60 cc probable diagnosis? (AIIMS Nov 2000)
RF>50% RF <50% a. Tricuspid atresia
b . Eisenmenger syndrome

Ans.125. c. Fever 126 d. Pancarditis 127 a. Aschoff ’s nodule 128 d . Usually associated with..
129. a . Mitral regurgitation 130 a. Along line of closure 131 a. LVH 132 a. Left atrial myoma
133. d. A2-OS gap 134 d. Intensity of systolic 135 b. Majority of the case .. 136 a. Tricuspid atresia
Cardiology

J : 1 »][•] 11 *j»7 i j
^
> Abnormal Aortic Valve With Reduced Systolic Opening

0
Vmax 4ms Vmax 3 m/ s-3.9 m/s Pmean 20-39 mm Hg
^ ^ ^
mean 0 mm Hg Vmax <3 m/ s Pmean <30 mm Hg
The indications for Aortic valve replacement in the asymptomatic patient of aortic stenosis have been redefined as
Relative indications for which surgery can be considered include an abnormal response to treadmill exercise; rapid progression of AS ,
especially when urgent access to medical care might be compromised;
Very severe AS defined by an aortic valve jet velocity >5 meters/s or mean gradient >60 mmHg and low operative risk
Excessive LV hypertrophy in the absence of systemic hypertension.
Exercise testing can be safely performed in the asymptomatic patient, as many as one-third of whom will show signs of functional impairment .

c. Tetrology of Fallot 142. A lady presents with grade-III dyspnea. She has severe
d. Total anomalous pulmonary venous drainage mitral stenosis with atrial fibrillation. Along with in ¬

Ref: Harrison's 18/ e pl 927, Harrison 19th p 1527 creased ventricular rate, clots in the left atrium are |
seen. Which of the following should not be done?
137. Which of the heart valves has the shortest life?
a . Starr- Edward valve (J and K 2012 )
(AUMS Nov. 2012) 1
b . Bjork-Shiley valve a. Diltiazem to reduce the heart rate
b. Warfarin therapy

Cardiolgy
c. Biological valve
d. St.Jude valve c. Open mitral commissurotomy and removal of clot
d. Cardioversion with percutaneous balloon valvoto-
Ref: Harrison's 18/ e pi 830, Harrison 19th p 1162 my
138. A patient presents with syncope, dyspnea & angina,
what is the possible diagnosis? (AIIMS Nov. 14) Ref Harrison's 18th ed. P-1932 Harrison 19th p 1542
a. Mitral stenosis b. Aortic stenosis 143. Criteria for mitral Valvotomy includes all except?
c. Pulmonic stenosis d . Tricuspid stenosis a. Significant symptoms (NBE Pattern 2014-15)
Ref Harrison's 18th, p ch 237: 1940, Harrison 19th p 1530 b. Isolated mitral stenosis
c. Mobile non calcified valve
139. In Mitral Stenosis double atrial shadow is due to
( NEET Pattern 2015-16) d. Left atrial thrombus
enlargement of?
a. Right atrium b. Left atrium Ref Harrison 19th p 1542
c. Both atria d. Left auricle 144. Pulsatile liver is seen with (NBE Pattern 2014-15)
Ref: Grainger Radiology, 4th p 684 Harrison 19th p 1540 a. Tricuspid regurgitation
140. In pregnancy which valvular disease is most danger ¬ b. Tricuspid stenosis
ous? (NEET Pattern 2015-16) c. Dilated cardiomyopathy
a. MR b. MS d . Pulmonic stenosis
c. AS d . AR Ref : Harrison's 18 th
p 1948 Harrison 19th p 1548
Ref: Current diagnosis and Treatment, 2 nd edn, ch 32, 145. Incorrect about chronic aortic regurgitation is
141. A person with aortic stenosis could perform an exer ¬
a. Chest pain (NBE Pattern 2014-15)
cise for 11 minutes in Bruce protocol. Exercise had to b. Wide pulse pressure
be stopped due to fatigue. He had a peak systolic gra ¬
c. Quincke's sign d . Late systolic murmur
dient of 60 mm Hg across the aortic valve at rest. The
Ref : Harrison's 18 th
ed., ch 237 p 1944 Harrison 19th p 1536
best management for him would be
(AUMS May 2012) 146. Dysnea, syncope and angina pectoris occur most com ¬

a. Medical management monly in? (NBE Pattern 2014-15)


b. Aortic valve balloon dilation to prevent worsening a. MS b. AS
c. Aortic valve replacement c. MR d . AR
d. Coronary angiography Ref: Harrison 19th p 1531
Ref CMDT 2015, p 339, Harrison 19th p 1533,
Ans. 137. c. Biological valve 138. b. Aortic stenosis 139 . b. Left atrium 140. c. AS
141. c. Aortic valve rep... 142. d. Cardioversion with ... 143. d. Left atrial thrombus 144. a. Tricuspid regurgitation
145. d. Late systolic murmur 146. b. AS

I
Marwah's Internal Medicine MCQs ( Based on Harrison s 19th)
'

CARDIOLOGY UPDATES
> The 2013 ACC /AHA Guideline on the Treatment of Blood
Cholesterol to Reduce Atherosclerotic Cardiovascular risk in adults

Q
suggests statin therapy in four populations: patients with
(1) Clinical atherosclerotic disease
(2) LDL cholesterol > 190 mg/dL
(3) Diabetes who are aged 40-75 years
(4) an estimated 10-year atherosclotic risk of > 7.5% aged 40-75 years.
Importantly, the updated guidelines no longer recommend treating to a target LDL cholesterol, an approach that has never been
shown to be effective in randomized trials. Patients in these categories should be treated with moderate or high intensity statin,
with high intensity statin for the higher risk populations.
High-, moderate-, and low-intensity statin therapy (Used in the RCTs reviewed by expert panel).

-
High Intensity Statin Therapy -
Moderate Intensity Statin Therapy -
Low Intensity Statin Therapy
Daily dose lowers LDL-C on average Daily dose lowers LDL-C on average by Daily dose lowers LDC-C on average by
by approximately > 50% approximately 30% to < 50% < 30%
Atorvastatin (403)-80 mg Atorvastatin 10 (20) mg Simvastatin 10 mg
Rosuvastatin 20 (40) mg Rosuvastatin (5) 10 mg Pravastatin 10-20 mg
Simvastatin 20-40 mg Lovastatin 20 mg

Cardiolgy
Pravastatin 40 (80) mg Fluvastatin 20-40 mg
Lovastatin 40 mg Pitavastatin 1 mg
Fluvastatin XL 80 mg
Fluvastatin 40 mg twice daily
Pitavastatin 2-4 mg

.
147 A patient recovered from right hemi-paresis On ex . ¬ .
150 A person with mitral regurgitation and atrial fibrilla ¬

amination patient was found to have aortic valve le ¬ tion presents with syncope. On examination the per ¬

.
sion and is in sinus rhythm What will you do to pre ¬ son has a heart rate of 55. What is the most probable
(AIIMS Nov. 2013) cause? (NBE Pattern 2014-15)
vent further stroke?
a. Digitalis toxicity
a. Anti-platelet only
b. Incomplete heart block
b. Anti-coagulant only
c. Stroke
c. Both anti-platelet and anticoagulant d. Subarachnoid Hemorrhage
d. Low molecular weight heparin subcutaneously.
Ref: H 18th p 1935 Harrison 19th p 309-310
Ref: Harrison 19th p 2566, Table 446 -3
.
151 Most common valvular lesion seen with carcinoid syn ¬

.
148 Blood pressure is difficult to measure in a patient drome is? (NBE Pattern 2014-15)
w|tj : , (NBE Pattern 2014-15) a. Tricuspid stenosis and pulmonic stenosis
a. Mitral stenosis b. Tricuspid insufficiency and pulmonic stenosis
b. Aortic stenosis c. Mitral stenosis and aortic stenosis
c. Complete heart block d. Mitral insufficiency and aortic stenosis
d. Atrial fibrillation Ref : Harrison's 18 th
ed., ch 237 Harrison 19th p 564
Ref: Harrison 19th p 1486 .
152 Extremely bad prognosis is seen in which heart dis ¬

. The treatment of acute pulmonary oedema includes ease in pregnancy? (NBE Pattern 2014-15)
149
(AIPGJan. 2012) a. Repaired TOF
all of the following except?
b. Bicuspid aortic valve
a. Morphine b. Digoxin
c. ASD
c. Frusemide d. Positive ventilation
d. Pulmonary artery hypertension
Harrison, ch. 272 p 2236 - 37 Harrison 19th p 1761
Ref

149 b. Digoxin 150 a. Digitalis toxicity


Ans. 147. -
a. Anti platelet only 148. d. Atrial fibrillation
151. .
b. Tricuspid insu.. 152. d. Pulmonary artery...
Cardiology

153. Area of mitral orifice in adults is: (AIIMS Dec 94) Pericardial Disease
a . 6-8 cm 2 b. 2 -5 cm 2
c. 4-6 cm 2 d . 1-4 cm 2
161. Typical features of Acute pericarditis includes:
Ref: Braunwald 6 th/ 378,1643,1673 Harrison 19 th p 1539 oandK 2012 )
154. Severity of mitral stenosis is assessed by: a. Chest pain identical to that of myocardial infraction I
( AI 1998) & ( .AIIMS May 03) b. A friction rub that is best heard in the axilla in |
a. Loud opening snap midexpiration
b. Length of murmur c. ST elevation on the ECG with upward concavity
c. Loud SI d . Elevation of the serum creatine kinase
d. Splitting S 2 Ref: Harrison's 18/ e pl 971, Harrison 19 th p 1573
Ref: Harrison 18th/ p 1930, 1931, Harrison 19 th p 1540 162 . Haemorrhagic pericarditis occurs in all of the follow ¬

155. All of the following statements about mitral valve pro ¬ ing conditions except: ( AIIMS May 03) j
lapse are true except: ( AIIMS Nov 93) a. Transmural myocardial infarction
a. It is more common in females b. Dissecting aneurysm of aorta
b. Most patients are symptomatic c. Metastatic disease of pericardium
c. It has a benign clinical course d . Constrictive pericarditis
d. Transient cerebral ischemic is a know complication Ref Harrison's 18/ epl 972, 1976, Harrison 19 th p 1576
Ref Harrison 18th/ p 1937, Harrison 19 th p 1546 163. During ventricular pressure pulses square root wave
( PGI Dec 2000)

Cardiolgy
156. Calcification of the aortic valve is seen in : (AI 95) is seen in:
a. Aortic stenosis b. Aortic regurgitation a. ASD b. MVPS
c. Marfan's syndrome d . Hurler’s syndrome c. Dilated cardiomyopathy
Ref: Harrison’ s 15th / 1350, Harrison 19 th p 1531 d. Constrictive pericarditis
Ref: Harrison' s 18/ e pl 972, 1976 , Harrison 19 th p 1576
157 . LVH is commonly seen with : (AI 1999 )
a. Pure mitral stenosis 164. All of the following pulmonary symptoms and non -
b . ASD with fossa-ovalis pulmonary causes are correctly matched except
c. Aortic incompetance a. Tachypnea - Acidosis ( AIIMS Nov. 14 )
d. Carcinoid syndrome Ref Harrison 19 th p 1535 b. Wheezing- congestive heart failure
c. Cyanosis-anxiety
158. Obstruction in pulmonary stenosis may occur at the d. Chest pain- pericarditis
following sites : ( AI 97 )
Ref Harrison’ s 18th edn ch 227, Harrison 19 th p 1442
a. Supravalvular b. Valvular
c. Subvalvular d . All of the above 165 . Restrictive and constrictive pericarditis occurs to ¬
Ref Harrison 19 th p 1549 gether in:
a. Radiation
( NBE Pattern 2014 15)
b. Adriamycin
-
159. Hepatomegaly with liver pulsation indicates: c. Amyloidosis
(AI 2009 )
d . Post cardiotomy syndrome
a. TR
Ref Harrison p 196,7 Harrison 19th p 1577
b. MR
c. Pulmonary hypertension CARDIOLOGY UPDATES
d. MS
Ref Harrison 18th/ p 1948, Harrison 19 th p 1548 > Non Infective Causes of Myocarditis
Granulomatous inflammatory disease
160 . In a patient with myocardial infarction the valvular Sarcoidosis
lesion commonly seen is: (AIIMS May 02) Giant cell myocarditis
a . Aortic stenosis Eosinophilic myocarditis
b. Mitral regurgitation Polymyositis, dermatomyositis
c. Aortic regurgitation Collagen vascular disease
d. Septal defect Peripartum cardiomyopathy
Ref Harrison 18th / p 2022, Harrison 19 th p 1543 Transplant rejection

Ans. 153. c. 4-6 cm2 154. b. Length of murmur 155. b. Most patients... 156. a . Aortic stenosis
157. c . Aortic incom... 158 . d. All of the above 159. a . TR 160. b. Mitral regurgitation
161. c. ST elevation... 162. d. Constrictive pericarditis 163. d. Constrictive pericar... 164. c. Cyanosis-anxiety
165. a. Radiation
th
Marwah's Internal Medicine MCQs ( Based on Harrison's 19 )

1166. Electrical alterans is seen in? ( NBE Pattern 2014-15) Cardiomyopathy


a. Cardiac tamponade
b. Constrictive pericarditis 171. ECG finding suggestive of Left ventricular hypertro ¬

c. Severe LVF phy? (Maharashtra 2011)


d. Severe RVF a. RaVL + SV3 >28 mm
b. SV1 + (RV5 or RV6) >35 mm
Ref: Harrison's 18th p 1975 Harrison 19th p 1459
c. Evidence of Left atrial hypertrophy
H 167. A 50 year old lady presents with cold intolerance, con
stipation, hoarseness of voice. On Chest X-Ray cardio-
¬

d. All of the above


Ref: Harrison's 18/ e pi 834, 1835, Harrison 19th p 1494
megaly is present. Investigation to diagnose the con¬
172. Which is going to best declare the case as that of inter ¬

dition: (AllMS May 2013)


atrial septal defect with other cardiac abnormalities?
a. Coronary angiography
( Rajasthan 2009)
b. Lt ventriculography a. Elevated pressure in left atrium
c. Rt.ventriculography b. Elevated pressure in right atrium
d. Echocardiography c. Elevated P 02 in pulmonary artery
Ref: Harrison's 18th P-2920 Harrison 19th p 27Oe, 2291 d. Systolic murmur Ref: Harrison 19th p, 1729
168. Beck's Triad is seen in: [A12010) 173. The most common cause of tricuspid regurgitation is
secondary to: (AI 03)
a. Constrictive Pericarditis
a. Rheumatoid heart disease
b. Cardiac tamponade
b. Dilatation of right ventricle

Cardiolgy
c. Right Ventricular Myocardial Infarction (RVMI) c. Coronary artery disease
d. Restrictive Cardiomyopathy d. Endocarditis due to intravenous drug abuse
Ref: Harrison 18th / p 1972 Harrison 19th p 1573 Ref: Harrison's 18/ e pi 948, Harrison 19th p 1548
169. Electric alternans is seen in: ( AI 1995) 174. The 9 month old child of a diabetic mother presents
a. Cardiac tamponade with tachypnea and hepatomegaly. Echocardiogra ¬

b. Restrictive cardiomyopathy phy of the heart showed normal cardiac morphology


c. Constructive pericarditis with asymmetric septal hypertrophy. Which of the
following will you give to treat this child:
d. Right Ventricular MI (RVMI)
(AllMS Nov 2000)
Ref: Harrison 18th / p 1839, 1975, Harrison 19th p 1573 a. Digoxin b. Frusemide
170. All of the following statements about chronic constric ¬ c. Propranolol d. Isoptin
tive pericarditis are true except (PGI Dec 99) Ref: Harrison's 18/ e pl 969, 1970, Harrison 19th p 2296
a . Commonest cause in India is 'Idiopathic' 175. Not a Single gene disorder? (JlMPER-2014)
b. Kussmaul's sign is present a. Mitral Valve Prolapse
c . Ascitis is not in proportion to edema b. Hypertrophic obstructive Cardiomyopathy
d. Right ventricular End Diastolic pressure is raised c. Dilated Cardiomyopathy
d. Arrhythmogenic Right Ventricular Cardiomyopathy
Ref: H 18th / p 1975, 1976 H/119, Harrison 19th p 1575 - 76
Ref: Harrison 18th edn ch 238 Harrison 19th p 1553

CARDIOLOCV UPDATES

> Prognostic Markers For Hypertrophic Obstructive Cardiomyopathy


History of cardiac arrest or spontaneous sustained ventricular tachycardia
Syncope
Family history of sudden cardiac death
Spontaneous nonsustained ventricular tachycardia
LV thickness > 30 mm
Abnormal blood pressure response to exercise

Ans. 166. a . Cardiac... 167. d. Echocardiography 168. b. Cardiac tamponade 169. c. Constructive pericarditis
170. a. Commonest... 171 . d. All of the above 172. b. Elevated pressure in right atrium
173. b. Dilatation of rig... 174. c. Propranolol 175. d. Arrhythmogenic Right ...
Cardiology

176. 25 year old footballer is elbowed in the chest by the 183. In case of sudden death in a young football player, the
rival defender during ball possession . Following the first clinical suspicion would rest on which of the fol ¬
chest trauma the player collapses and dies. The most lowing differentials? ( NBE Pattern 2014-15)
probable cause of death is: ( NBE Pattern 2014-15) a . Arrthymogenic right ventricular dysplasia
a . HOCM b. Commotio cordis b. Takotsubo cardiomyopathy
c. Hemothorax d . Aortic transaction c. Atrial septal defect
Ref: Harrison’s 18th p 1981, Harrison 19 th p 289 e - 3 d. Eisenmenger complex
177. Cardiomyopathy does not occur in Ref: ch. 233 p 1897 Harrison 19 th p 1564
( NBE Pattern 2014-15) 184. Drug contraindicated in HOCM is?
a. Duchenne muscular dystrophy ( NBE Pattern 2014 -15 )
b. Alkaptonuria c. Pompe disease a. Verapamil
d. Fabry’s disease b. Propranolol
Ref: Harrison 18th. p 1957; Harrison 19 th p 1558 c. Digoxin
d. None of the above
178. RCM is caused by all except: ( NBE Pattern 2014-15)
Ref: Harrison 19th p 1569
a. Fatty infiltration of myocardium
b. Amyloidosis 185. Which Cardiomyopathy is caused by chronic alcohol ¬
c. Daunorubicin ism? ( NBE Pattern 2014 -15)
d. Carcinoid syndrome a. Dilated
Ref: Harrison' s: 18th p 1959 Harrison 19th p 1565 b. Hypertrophic

Cardiolgy
c. Atrophy
179. Banana shaped left ventricle is seen in?
d. Restrictive
a. HOCM ( NBE Pattern 2014 15) - Ref: Harrison's, 18th p 1952 Harrison 19th p 1558
b. DCM
c. RCM 186. All are true about hypertrophic cardiomyopathy, ex¬
d. Takotsubo cardiomyopathy cept : ( All MS 1996 )
Ref Harrison 19th p 1568 a. Systolic dysfunction
b. Concentric hypertrophy
180. Which of following is a treatable cause of RCM? c. Diastolic dysfunction
a. Fabry's disease ( NBE Pattern 2014 -15) d . Double apical impulse Ref: Harrison 18th / p 1968
b. Amyloidosis
c. Endomyocardial fibroelastosis Arrythmia
d . Hypereosinophilic syndrome
Ref: Harrison's, ch. 238 p 1965 Harrison 19th p 1565 187 . A person with mitral regurgitation and atrial fibrilla ¬
tion presents with syncope. On examination the per ¬
181. Alprostadil is contraindicated in:
( NBE Pattern 2014-15) son has a heart rate of 55 . What is the most probable
cause ? ( AI 2007 )
a. Tricuspid atresia
a. Digitalis toxicity
b. Transposition of great arteries b. Incomplete heart block
c. Tetralogy of Fallot c. Stroke
d. Total anomalous pulmonary venous connection d . Subarachnoid Haemorrhage
(TAPVC)
Ref Goodman & Gilman 11/ e p889, Harrison 19 th p 203
Ref Cloherty Manual of Neonatal Care: 7th ed., ch. 41
188. The most common reentrant tachycardia associated
182 . Incorrect about restrictive cardiomyopathy: with WPW syndrome is: ( PGI 2012 )
a . Kussmual's sign ( NBE Pattern 2014-15) a. Orthodromic AV reentry
b. Pulsatile liver b. Antidromic AV reentry
c. Pedal edema c. Rapidly conducting AF
d . Dip and spike configuration in ventricular systolic d . None
pressure. Ref: Harrison 18/ e p! 889, Harrison 19 th p 1481
Ref: Harrison 18th p 1975; Harrison 19 th p 1573

Ans . 176. b. Commotio cor. .. 177. b. Alkaptonuria 178. a . Fatty infiltration of ... 179. a. HOCM
180. a . Fabry ’s disease 181. d . Total anomalous... 182. d. Dip and spike confi. .. 183. a. Arrthymogenic right...
184. c. Digoxin 185. a. Dilated 186. a . Systolic dysfunction 187. a . Digitalis toxicity
188. a. Orthodromic AV reentry
Marwah’s Internal Medicine MCQs ( Based on Harrison's 19,h )

189. 40 years Lady intra -operatively develops HR = 220 196. Most common mechanism of arrhythmia?
bpm and Blood pressure of 70/ 40, ECG shows qRS a. Re -entry -
( NBE Pattern 2014 15)
complex =0.11 second. Best management is? b. Early after depolarization

Q
(JIPMER 2014 ) c. Late after depolarization
a. Adenosine 6mg/6mg/12 mg d. Automaticity
b. Amiodarone
Ref Harrison's, 18th p 1863: Mechanism of Arrhythmia
c. DC cardioversion
d. Esmolol 197. Most common beningn cardiac rhythm is?
( NBE Pattern 2014-15)
Ref: Harrison 18th p 1892, Harrison 19th p 1483
a. Atrial premature contraction
190. Most common arrhythmia in ICU patients:
b. Atrial fibrillation
a. Atrial flutter ( NEET Pattern 2015-16 ) c. Ventricular premature contraction
b. Atrial fibrillation
d. Ventricular tachycardia
c. PSVT
Ref Harrison's, 18th erf, ch. 233, Tachy-arrythmias p 1880
d. NPAT
198. LBBB is seen with all except: (AIIMS May 2013)
Ref: Harrison 18th p 1880, p 121, Harrison 19th p 1486
a. Acute MI b. Ashmann syndrome
I 191. Most common cause of heart block in infants is? c. Hypokalemia d. Hyperkalemia
a - SLE ( NEET Pattern 2015-16 ) Ref: Harrison 19th p 1455
b. Surgery for congenital heart disease
c. Viral myocarditis 199. What is not correct about LBBB? ( NBE Pattern 2014-15)

Cardiolgy
d. Rheumatic fever a . Can occur after MI
Ref: Net source : PubMed. p 458, O.P. Gahai 8th edn b. ST segment elevation is seen
c. A-V dissociation
192. Patient with ICD collapses, which 1CD imaging modal ¬ d . Cardiomyopathy is associated
ity is best suited for these patients? (AIIMS Nov. 2012)
Ref: Harrison's, ch. 228 p 1835 Harrison 19th p 1454
a. Chest X-ray b. MRI
c. CT d. USG 200. A patient develops sudden palpitations with HR= 150/
Ref: Harrison's, 18th ch 233 Harrison 19 th p 1499 min, rythm regular. What could be the cause?
a. PSVT ( NBE Pattern 2014-15)
193. An 18 year old boy is asymptomatic. On ECG he has a b. Sinus tachycardia
short PR interval with delta waves. Which of the fol ¬ c. Ventricular tachycardia
lowing is not routinely required for these patients? d. Atrial flutter with block
a. Hotter monitoring (ARMS May 2013)
Ref: H 18th , p 1888 Harrison 19th p 1479
b. Treadmill test
c. Reassurance 201. Atrial fibrillation may occur in all the following condi ¬
d. Beta blockers tions, except- ( NBE Pattern 2014-15)
Ref: Harrison 18th ed., p 1890, Harrison 19th p 1483 a. Mitral stenosis
b. Hypothyroidism
194. What is incorrect about Brugada syndrome? c. Dilated cardiomyopathy
a. SCN 5A defect ( NBE Pattern 2014-15)
d. Mitral regurgitation
b. Asymptomatic ST segment elevation
Ref: Harrison’s 18th p 1881; Harrison 19th p 1486
c. Sudden death
d. Pacemaker is treatment of choice 202. Which of the following arrhythmia is most commonly
Ref: Harrison 18th p 1899, Harrison 19th p 1497- 98 associated with alcohol binge in the alcoholics?
a. Ventricular fibrillations ( NBE Pattern 2014 -15)
195. According to AHA 2010, drug not used in CPR is? b. Ventricular premature contractions
a. Adrenaline ( ARMS Nov. 2013)
c. Atrial flutter
b. Vasopressin c. Atropine
d. Atrial fibrillation
d. Amiodarone
Ref: Harrison's 18th ed. Ch -233 p 1881 Harrison 19th p 1486
Ref: AHA 2010 guidelines; Harrison 19th p 1770

Ans . 189. c. DC cardioversion 190. b. Atrial fibrillation


.
191. b. Surgery for con ...
195. c. Atropine 196. a. Re-entry
-
192. a. Chest X ray...
193. b. Treadmill test 194. d . Pacemaker is treat..
197. a. Atrial premature...198. c. Hypokalemia 199. c. A-V dissociation 200. a. PSVT
201. a. Mitral stenosis 202. d . Atrial fibrillation
Cardiology

203 . Which of the following statements about premature 209. Which of the following is a Sino atrial disease?
ventricular beat is false? ( NBE Pattern 2014 -15 ) ( NBE Pattern 2014-15)
a . Sequential depolarization of ventricles a. Atrial ectopics b. Ventricular ectopic
b. Wide, Bizarre, Notched qRS complexes c. Sinus arrest d. A-V block
c. Prevalence decreases with age Ref: Harrison 19 th p 1468
d. Palpitations is a common presenting feature
210. Frog sign is seen in: [ NBE Pattern 2014-15]
Ref: Harrison’ s 18th ed., Ch -228 p 1890 Harrison 19 th p 1492
a . AVNRT
204. Which of the following is the most common heart block b. Diabetic nephropathy
in neonatal lupus [ NBE Pattern 2014-15) c. Medullary sponge kidney
a. 1st degree heart block d . Budd Chiari syndrome
b. Mobitz 2A heart block Ref: Harrison's, 18th p 1888 Harrison 19 th p 1479
c. Mobitz 2 B heart block
d. 3rd degree heart block 211. All of the following are true about Atrial fibrillation,
except: ( NBE Pattern 2014-15)
Ref: H 18th p 2734 Harrison 19 th p 2133
a. Risk of thromboembolism
205 . A 78 - year - old male with hypertension (controlled on b. Digoxin for treatment
anti - hypertensive drugs) presents with new onset of c. Cardioversion followed by anticoagulation
mild left hemiparesis and the finding of atrial fibril ¬ d. Ectopic in pulmonary vein
lation . Which of the following must be done? Ref Harrison's 18th p 1882; Harrison 19 th p 1486 -87
( NBE Pattern 2014 -15)

Cardiolgy
212 . Congenital long QT syndrome causes death due to?
a. Close observation b. Permanent pacemaker ( NBE Pattern 2014 - 15)
c. Aspirin d . Warfarin
a. Complete heart block
Ref : HI 8th ed., p 1882; Harrison 19th p 1488 b. Polymorphic ventricular tachycardia
206. In the ICU, a patient suddenly becomes unresponsive, c. Acute myocardial infarction
pulseless, and hypotensive, with cardiac monitor in ¬ d. Recurrent supraventricular tachycardia
dicating ventricular tachycardia. The first therapeu¬ Ref: Harrison's 18th p 1897; Harrison 19 th p 1497
tic step among the following should be: 213. All are true about WPW syndrome except?
( NBE Pattern 2014 -15) ( NBE Pattern 2014-15)
a. Amiodarone 300 mg IV push a. More common in female
b. Lidocaine 1.5 mg / kg IV push b. Delta wave in ECG
c. Defibrillation at 200 joules biphasic c. HIS bundle study is done to diagnosis
d. Defibrillation at 360 joules uniphasic d . Can occur in a normal heart .
Ref : Harrison’s 18th ed., ch 273 p 2244 Harrison 19 th pi 769 Ref: Harrison's 18th p 1889; Harrison 19 th p 1481
207 . Canon 'a' wave is seen in? ( NBE Pattern 2014 -15) 214. Broad complex tachycardia, due to ventricular tachy¬
a. Junctional rhythm cardia is suggested by all except?
b. Atrial fibrillation ( NBE Pattern 2014- 15)
c. Atrial flutter a. Fusion Beats
d. Ventricular fibrillation b. A V dissociation
Ref: Harrison’s, 18th p 1823 Harrison 19 th p 1444 c. Capture Beats
d . Termination of tachycardia by carotid Sinus massage
208. Which is incorrect about a pacemaker:
Ref H 18th p 1888 Harrison 19 th p 1492, 1494
( NBE Pattern 2014-15)
a. Deployed below skin of the chest 215 . Drug of choice in maintenance therapy in P.S.V.T is:
b. Pacing leads lie in the right atrium and right ventricle ( NBE Pattern 2014-15)
c. Treatment of choice in Mobitz 1 heart Block a. Amiodarone b. Lignocaine
d . Biventricular pacing is useful for dilated cardiomy ¬ c. Verapamil d. Adenosine
opathy Ref: Harrison’ s 18th Ch -233 Harrison 19 th p 1484
Ref: Harrison’ s 18th pg. 1895 Harrison 19 th p 1475

Ans . 203. c . Prevalence dec... 204. d. 3rd degree heart block ... 205. d. Warfarin 206. c. Defibrillation at 200 jou ...
207. a . Junctional rhy... 208. c. Treatment of choice ... 209. c. Sinus arrest 210. a. AVNRT
211. c. Cardioversion ... 212. b. Polymorphic ventri... 213. a . More common in ... .
214 d. Termination of tachysage...
215. c. Verapamil *
Marwah's Internal Medicine MCQs ( Based on Harrison's 19th )

I 216. A chronic alcoholic develops palpitations suddenly


ECG
after alcohol binge. Which of the following arrythmia
is most commonly associated with alcohol binge in
223. Epsilon waves in ECG are seen in:
the alcoholics? ( AIIMS Nov 01 )
a. Ventricular fibrillations ( NBE Pattern 2014 -15)
b. Verticular premature contractions a . Arrythmogenic right ventricular dysplasia
c. Atrial flutter b. Takotsubocardiomyopathy
c. Hyperthermia
d . Atrial fibrillation
d . Restrictive cardiomyopathy
Ref: Harrison 18th/ p 1881 Harrison 19 th p 1486
Ref: Harrison's 18th, pg. 1896 Harrison 19 th p 1564
I 217. Drug of choice in PSVT is : (Al 1998) 224. QT prolongation is seen in all, except:
a. Amiodarone a. Hypothermia ( NBE Pattern 2014-15)
b. Lignocaine
b. Digitalis toxicity
c. Quinidine c. Hypocalcemia
d . Adenosine Ref Harrison 19 th p 1483 d. Romano -ward syndrome
218. Feature of Torsade de pointes is: (AIIMSDec 97 ) Ref: Harrison' s 18th, p 1838 Harrison 19 th p 1457
a. Wide qRS complex 225. QT interval is shortened in: ( NBE Pattern 2014 -15)
b. Short qRS complex a. Hypocalcemia
c. Prolonged QTc interval b. Hypokalemia
J d. Short QTc interval Ref:Harrison 18th ed, Ch 233 c. Hypercalcemia
0 219. Congenital long QT syndrome can lead to-: d. Digoxin
U
1 a. Complete heart block (A11MS May 2003) Ref: Harrison's Ch -233 p 1839 Harrison 19 th p 1457
U
1
b. Polymorphic ventricular tachycardia 226. ST elevation is seen in all of the following conditions
c. Acute myocardial infarction ( NBE Pattern 2014 -15)
U d. Recurrent supraventricular tachycardia
except:
a. Myocardial infarction

VJCI Ref: Harrison 18th / p 1891, Harrison 19 th p 1491


220. All of the following are features of Mobitz Type I block,
b. Coronary artery spasm
c. Constrictive pericarditis
d . Ventricular aneurysm
except: ( AI - 1992 )
Ref: H 18th Table 228.1 p 1836 Harrison 19th p 1457
a. Constant PR interval
b. Normal qRS morphology 227. Inverted T waves are seen in? ( NBE Pattern 2014 -15)
c. Regular Atrial Rhythm a. Hyperkalemia b. Hyperthermia
d . Atrial rate > ventricular rate c. Wellen syndrome d . Coronary syndrome X
Ref: Harrison 18th, Ch - 232, edn. Harrison 19 th p 1472 Ref: Harrison' s 18th ed -Ch -228 p 1837
221. Constant PR interval is seen in: ( PGI - 1997 ) 228. The following ECG findings are seen in Hypokalemia:
a. First degree block ( NBE Pattern 2014-15)
b. Second degree - Mobiz type 1 block a. Increased PR interval with ST depression
c. Second degree - Mobitz type II block b. Increased PR interval with peaked T wave
d. Third degree block c. Prolonged QT interval with T wave inversion
Ref: Harrison 18th edn, Ch - 232, Harrison 19 th p 1472259 . d. Decreased QT interval with ST depression
Ref: Harrison's Ch -45 Harrison 19 th p 1457 and p 310
222. A young patient sustains cardiac arrest, in the medical
ward. Immediate defibrillation is advised when the ECG 229. Which ECG finding is most likely to be seen at time of
shows: cardiac arrest (UPSC 2013)
a. Ventricular Tachycardia ( Al 1992) a. Ventricular fibrillation
b. Asystole b. Ventricular tachycardia
c. Electromechanical dissociation c. Atrial fibrillation
d . Paroxysmal atrial tachycardia
d . Persistant Bradyarrhythmia
Ref: Harrison 18th/ p 2239, 2244, Harrison 19 th p 1769 Ref: H 18th ed. , p 2239 Harrison 19th pi 768

Ans . 216. .
d . Atrial fibrill.. 217. d . Adenosine 218. c. Prolonged QTc ... 219. b . Polymorphic ventri ...
220. a . Constant PR... 221 . a . First degree block 222. a. Ventricular Tachy... 223. a . Arrythmogenic right...
224. b. Digitalis toxicity 225. c. Hypercalcemia 226. c. Constrictive pericar... 227. c . Wellen syndrome
228. a. Increased PR... 229. a . Ventricular fib...
Cardiology

230. Which of the following may occur due to hyper - kale - 237 Feature of Torsades de pointes is:
( NBE Pattern 2014-15)
( NBE Pattern 2014 -15)

b
mia:
a. Prolonged PR interval a. Hypermagnesemia
b. Prolonged qRS interval b. Short qRS complex
c. Ventricular fibrillation c. Prolonged QTc interval
d. All of above d. PQ segment
Ref: Harrison's 18"' ed., ch. 228 p 1838 Harrison 19th p 1457 Ref: Harrison ' s 18 th
edp 1838 Harrison 19th p 1497
231. All of the following electrocardiographic findings may 238. True about Torsades de pointes ? ( NBE Pattern 2014 - 15)
represent manifestations of digitalis intoxication, ex¬ a. ST segment depression b . QTc prolongatio n
cept: ( NBE Pattern 2014 -15) d . PQ segment elevation
c. Narrow qRS Complex
a. Ventricular Bigeminy th p 1497
b. Junctional tachycardia Ref: Harrison ' s 18 “ ed. Ch -233 p 1898 Harrison 19
c. Bidirectional tachycardia the office with a his ¬
239. A 40-year-old male presents to
d. Multifocal Atrial tachycardia tory of palpitations that last for a few seconds
and
Ref: Harrison's, 18th p 1886 Harrison 19th p 1485 occur two or three times a week . There are no other
232. Which of the following is cause of RBBB? symptoms. ECG shows a rare single unifocal prema j -
ture ventricular contraction. The most likely cause
of
( NBE Pattern 2014-15)
;

a. It can occur in a normal person this finding is: ( NBE Pattern 2014 -15 )
b. Pulmonary embolism a. Underlying coronary artery disease
c. Corpulmonale b. Valvular heart disease

Cardiolgy
d. All of the above c. Hypertension
Ref: H 18th p 1835 Harrison 19th p 1454 55 - d. Idiopathic
233. Alternating RBBB with Left anterior hemiblock is seen Ref: Harrison's 18th ed., p 1890 Harrison 19th p 1492
in? ( NBE Pattern 2014 -15) [ NBE Pattern 2014-15)
240. Osborn wave is seen in?
a. 1st degree heart block
a . WPW syndrome type I
b. Complete heart block
b. Hypercalcemia
c. Mobitz type II block
d. Bifascicular block c. Cardiac tamponade
Ref: H 18th p 1835 Harrison 19th p 1455 d. Athlete Ref: Harrison 19th p 1457

234. Which is the following is the commonest ECG finding 241. 12 year old wheel chair bound boy with scoliosis has
in pulmonary embolism? ( NBE Pattern 2014 -15) presented with Dyspnea. ECG shows deep QS waves in
a . Sinus Tachycardia V 2, V3 will tall R waves in V5, V6. Probable diagnosis:
b. Inverted T wave in Lead III ( NBE Pattern 2014-15)
c. S1Q3T 3 a. Amyotrophic lateral sclerosis
d. Pathological Q b. Duchenne muscular dystrophy
Ref: Harrison' s 18th ed-Ch -228 p 1834 Harrison 19th p 1633 c. Pott's paraplegia
d. Myotonic dystrophy
235. Left axis deviation is seen in all except?
( NBE Pattern 2014- 15) Ref : H18ch ed., p 1963 Harrison 19th p 1558, 462 e 5 -
a. Left anterior hemi block 242. Treatment of asymptomatic bradycardia is:
b. Inferior wall MI
c. ASD [Septum secundum]
a. No treatment is required. ( NBE Pattern 2014 15 )
b. Give atropine
-
d. Right pneumothorax
c. Isoprenaline
Ref: Harrison's, 18th ed. p 1834 Harrison 19th p 1453
d. Cardiac pacing
236. All of the following features can differentiate between Ref: Harrison’ s 18th ed., Ch -232 p 1870 Harrison 19th p 1469
-
ventricular tachycardia and supraventri cular tachy¬
243. All are ECG changes in hypokalemia , except:
cardia except: ( NBE Pattern 2014-15)
a. qRS < 0.12 seconds a. U wave ( NBE Pattern 2014-15)
b. Ventricular rate > 160/ min b. ST segment sagging
c. Variable first heart sound c. T-wave flattening or inversion
d. Relieved by carotid sinus massage d . QT interval shortening
Ref: H 18th ed., p 1889, 1892; Harrison 19th p 1484, 1494 Ref: H 18th ed. Ch -45; Harrison 19th p 307
Ans . 230. d . All of above 231. .
d . Multifocal Atrial.. 232. d . All of the above 233. d . Bifascicular block
234. .
a . Sinus Tachyc .. 235. c. ASD (Septum se. .. 236. b . Ventricular rate... 237. c. Prolonged QTc interval
238. b. QTc prolon... 239. d . Idiopathic 240. b. Hypercalcemia 241 . b. Duchenne muscular...
242. .
a. No treatment is .. 243. d . QT interval shortening
th )
Marwah's Internal Medicine MCQ s ( Base d on Harri son's 19

244. A patient with angina, exertional syncope and Left 252 . What is diagnostic of fresh myocardial infarction in
ventricular hypertrophy is diagnosed as aortic ste ¬ ECG : ( AllMS May 95)
nosis. What is the predicted life span of this patient ? a. QT interval prolongation
( NBE Pattern 2014-15) b. P mitrale
a. 1 year b. 2 years c. ST segment elevation
c. 3 years d. 4 years d. ST segment depression
Ref : Harrison's 18th ed., ch 237 p 1940 Harrison 19th p 1532 Ref: Harrison 18th/ p 1395 Harrison 19 th p 1456
245. Low voltage ECG is seen in? ( NBE Pattern 2014-15) 253. QT interval is shortened in: ( AI 95)
a. Hypothyroidism b. Hyperthyroidism a. Hypocalcemia b. Hypophosphatemia
c. Diabetes d . Addison's disease c. Hypercalcemia d. Hypernatremia
Ref: Harrison's 18th p 1833 Harrison 19th p 1457 Ref: Harrison 18th/ p 1836 Harrison 19th p 1457
.
246 PR interval is reduced in? ( NBE Pattern 2014-15) 254. All of the following may occur due to hyperkalemia,
a .
Wenckebach phenomenon except: ( AI 06 )
b. WPW syndrome a. Prolonged PR interval
c. Hypothyroidism b. Prolonged qRS interval
d. Complete heart block c. Prolonged QT interval
Ref: Harrison' s 18th p 1889 Harrison 19 th p 1481 d. Ventricular asystole
247. Low voltage in ECG indicates? ( NBE Pattern 2014-15) Ref: Harrison 18th/ p 1838 Harrison 19th p 310
a . Pulmonary embolism
255. In left sided massive pneumothorax, ECG shows all ,

Cardiolgy
b. Cor pulmonale except- : ( AIIMS May 94)
c. Infective endocarditis
Ref: Harrison 19th p 1457 a. Left axis deviation
d. Pericardial effusion
b. Absent R wave
248. All of the following are the electrocardiographic fea ¬ c. Pathological Q waves
tures of Hyperkalemia, except: d. Precordial T wave inversion
( NBE Pattern 2014-15) Ref: Harrison 19th p 1455
a . Prolonged PR interval b. Prolonged QT interval
c. Sine wave patterns d. Loss of P waves 256. In a patient with wide -complex tachycardia, the pres ¬
Ref: Harrison's 18th ed., Ch. 45 p 1930 Harrison 19 th p 310 ence of all of the following in the ECG indicates ven ¬
tricular tachycardia except ( AIIMS Nov 03)
249. In LVH, SV1 + RV6 is more than mm?
a . Atrioventricular dissociation
a. 25 ( NBE Pattern 2014 -15)
b. Fusion beats
b. 30
c. Typical right bundle branch block
c. 35
d. Capture beats Ref: Harrison 19 th p 1489
d. 40 Ref: Harrison 19 th p 1454
257 . All of the following are features of Premature Ventric ¬
250. In left sided massive pneumothorax, ECG shows all, ( AI 1993)
( NBE Pattern 2014 -15) ular Complexes, Except:
except:
a. Left axis deviation a. Wide qRS complex
b. Absent R wave b. Absent P wave
c. Pathological Q waves c. Complete compensatory pause
d . Precordial T wave inversion d . Prolonged PR interval
Ref : Harris on' s 18th
ed., Ch. 227 p 1834 Harrison 19th p 1453
RefBraunwald 8th / 893-898 Harrison 19th p 1489
251. In a patient with wide- complex tachycardia, the pres ¬
ence of all of the following in the ECG indicates ven ¬ 258. What is diagnostic of fresh myocardial infarction in
( NBE Pattern 2014 -15) ECG: ( AIIMS May 95)
tricular tachycardia except?
a . Atrio-ventricular dissociation a. QT interval prolongation
b. Fusion beats b. P mitrale
c. Typical right bundle branch block c. ST segment elevation
d. Capture beats d. ST segment depression
Ref : Harrison' s 18th ed., p 1835 Harrison 19th p 1494 Ref: Harrison 18th / p 2023, Harrison 19th p 1600

Ans. 244. c. 3 years 245. a . Hypothyroidism .


246 b. WPW syndrome 247. d . Pericardial effusion
248. b. Prolonged QT ... 249. c. 35 .
250 a . Left axis deviation 251 . c. Typical right bundle...
252. .
c. ST segment .. 253. c. Hypercalcemia .
254 c. Prolonged QT... 255. a. Left axis deviation
256. c. Typical right... 257. d . Prolonged PR interval .
258 c. ST segment elevation
Cardiology

259. All of the following conditions are more commonly 266. Which of the following is not a major Framingham cri ¬
associated with LBBB pattern than RBBB Pattern on teria in CHF: ( AIIMS Nov 2008)
ECG, Except: ( AIIMS 2013) a. Cardiomegaly
a. Acute MI b. Aortic valve disease

Q
b. Paroxysmal nocturnal dyspnea
c. Lev Disease d. Ashmann Phenomenon c. S3 gallop
Ref: Harrison 19 th p 1455 d . Hepatomegaly Ref: Harrison 19 th p 1514
260. Brugada Syndrome is associated with: ( ARMS 2013) 267 . Not a marker of heart failure? ( ARMS Nov. 14 )
a. Left Bundle Branch Block b. Sirtuin
a . CRP
b. Left Anterior Fascicular block
c. Left Posterior Fascicular block c. BNP d. Troponin
d. Right Bundle Branch Block Ref: H 18th pi 906, Harrison 19 th p 1505
Ref: Harrison 18th/ 1898, 1899, Harrison 19 th p 1497 268. Inability to carry out physical activity without discom¬
fort comes under? ( ARMS Nov. 14 )
CHF a. NYHA1 b. NYHA 2
c. NYHA3 d. NYHA 4
261. C.V. P (Central Venous Pressure) and pulmonary wedge
Ref H 18th table 234-2, p 1818, Harrison 19th p 1440
pressure give an accurate assessment of all the fol ¬
lowing except: ( DP PGMEE 2010) 269. A patient with CHF with LVEF<40% should be given?
a. Tissue perfusion ( NEET Pattern 2015-16 )
b. Volume depletion a. ACEI + beta blocker b. ACEI + furosemide

Cardiolgy
c. Volume overload c. ACEI + CCB d. ACEI + ARB
d. Myocardial function
Ref: Harrison 18th p 1909 Ch 234, Harrison 19 th p 1511
Ref: Bailey 24/ e p69, Harrison 19th p 1443
270. Kerley B line are seen at: ( NBE Pattern 2014-15)
262. Typical JVP finding in cardiac tamponade: a. Apex
( PGI June 2000 )
b. Cardiophrenic angle
a. Absent 'Y' descent b. Prominant 'a' wave
c. Absent 'a' wave d. Prominent 'Y' wave c. Lung fissure
d. Pleural surface
Ref: Harrison's 18/ pl 972, Harrison 19 th p 1443
Ref Sutton Textbook of Radiology, 7th ed., p 131,145
263 . Sudden Cardiac Death (SCD ) can occur in the follow¬
ing disease, except: (AP 2012 ) 271. Cheyne stokes breathing is seen in:
a. Prolonged Q-T syndrome ( NBE Pattern 2014-15)
b. W- P-W syndrome a. Intracranial hypotension
c. Ventricular septal defect b. Congestive heart failure
d. Hypertrophic cardiomyopathy c. Left atrial myoxma d. Pickwinian syndrome
Ref: Harrison's 18/ e p2239, 2241, Harrison 19th p 1767 Ref: Harrison 18th p 1904 Harrison 19 th p 1504
264. Most common type of ASD is: ( DP PGMEE 2010) 272 . In a patient with chronic congestive cardiac failure,
a. Ostium primum all of the following drugs prolong survival except :
b. Ostium secundum a. Metoprolol ( AIIMS May 04 )
c. Sinusvenosus type
b. Carvedilol
d. Endocardial cushion defect
c. Enalapril
Ref Harrison's 18/ e pi 921, Harrison 19 th p 2721
d . Digoxin
265. Cardiac output is decreased in all except: Ref H 18th/ p 1906 , 1908, 1909, Harrison 19 th p 1512
(J and K 2010 )
a. Dilated cardiomyopathy 273. All of the following statements about digitalis are true
b. CCF due to mitral regurgitation in rheumatic heart except : ( A11997 )
diseases a . Excretion is mainly renal
c. Pericardial effusion b. Oral absorption is good
d. Pulmonary edema due to circulatory congestion in c. Actively metabolized in liver
acute glomerulonephritis d . Lipid soluble Ref KDT 5th/ 461, 462
Ref Harrison's 18/ e p2218, Harrison 19 th p 1462

Ans . 259. d . Ashmann ... 260. d. Right Bundle Branch... 261. a. Tissue perfusion 262. a . Absent *Y’ descent
263. c. Ventricular... 264. c. Sinusvenosus type 265. d. Pulmonary edema .. . 266. d . Hepatomegaly
267. b. Sirtuin 268. d. NYHA 4 269. a. ACEI + beta blocker 270. d. Pleural surface
271. b. Congestive... 272. d. Digoxin 273. c. Actively metabolized in liver
,
Marwah's Internal Medicine MCQs ( Based on Harrison's 19 h )

274. Treatment of digoxin over dose includes administra ¬ 281. Neonate with cyanosis, heart failure and systolic mur ¬
tion of all of the following except : mur is suffering from: ( NBE Pattern 2014 -15)
( AI 1997 ) a. TOF b. VSD
a. Potassium b. Lignocaine c. TGA d . Rheumatic fever

Q c. Phenytoin

Cor pulmonale except :


a. Kerley B lines
d. Hemodialysis
Ref: Harrison 19th p 473e 9
275. All the following are radiological features of Chronic

b. Prominent lower lobe vessels


( .AI 1996 )
-
cyanotic spell?
a. Phenylephrine
c. Calcium chloride
Ref: Harrison 19th p 1527
282. Which of the following is not given in treatment of
(AllMS Nov. 2013)
b. Propranolol
d. Soda bicarbonate
Ref: Nelson Textbook of Pediatrics, ch. 430
c. Pleural effusion
d. Cardiomegaly 283. Ductus arteriosus complete closure occurs at how
Ref: Harrison 18th/ p 1914 many weeks in a term baby? ( NBE Pattern 2014-15)
a. 1 week b. 2 week
Congenital Heart Disease c. 3 week d. 4 week
Ref: O.P. Ghai, 7th ed., pg. 395
276. The commonest mode of inheritance of congenital 284. Partial anomalous pulmonary venous connection is
heart disease is (AI 2002 ) associated with which of the following defects?
a. Autosomal dominant a. Sinus venosus ASD ( NBE Pattern 2014-15)
b. Autosomal recessive b. Ostium primum ASD

Cardiolgy
c. Sex linked dominant c. Endocardial cushion defect
d. Multifactorial d. Tricuspid atresia.
Ref H 18/ p1920, Harrison 19th p 270e 24 25 - - Ref: Nelson, 18th ed., ch. 256
277. True statement about ductus arteriosus is: (AI 2000) 285. Reversal of shunt is not possible in natural history of:
a. It undergoes anatomic closure within 24 hrs of birth a. ASD (AIIMS Nov. 2012)
b. Forms the ligamentum venosum in later life b. VSD
c. It is induced to close by high levels of prostaglandins c. TOF
d . May cause a machinery murmur by its patency. d. PDA
Ref: H 18/ e pl 923-24, Harrison 19th p 1524 f Ref: TOF: OP Ghai, 7th ed. pg 408 Harrison 19th p 1526
218. Turner syndrome is associated with? 286. A preterm baby with Patent Ductus Arteriosus. All are
( NEET Pattern 2015-16 ) true except: (ARMS may 2013)
a. Coarctation of aorta a. Narrow pulse pressure
b. Aortic dissection b. Necrotizing enterocolitis
c. Aortic regurgitation c. Continuous murmur
d. Pulmonic stenosis d . Congestive heart failure
Ref: Nelson 18th edn Ch 587: Harrison 19th p 1524 Ref: Harrison 19th p 1523

279. A Preterm baby with PDA will have all except: 287 . The commonest mode of inheritance of congenital
a. C02 washout ( AIIMS Nov. 2012 ) heart disease is (AI 2002)
b. Pulmonary hemorrhage a. Autosomal dominant
c. Necrotising Enterocolitis b. Autosomal recessive
d. Bounding pulses c. Sex linked dominant
d. Multifactorial
Ref: Nelson Pediatrics ch. 101.4 Harrison 19th p 1523
Ref: Harrison 18th/ p 1920, Harrison 19th p 1520
280. All are causes of sudden death in infant except:
288. All can cause recurrent pulmonary infection except:
a. Romano ward syndrome
b. Aortic stenosis
( NBE Pattern 2014 15)- a. VSD ( ARMSSep 96 )
c. Hypoplastic left heart syndrome b. Recurrent LVF
c. TOF
d . Kawasaki disease
d. ASD
Ref: Nelson, ch. 436, table 436.1; Harrison 19th p 2192 Ref: Ghai 6 th/ 406 - 409; Harrison 19th p 1526

Ans. 274. d . Hemodialysis 275. b. Prominent lower... 276. d. Multifactorial 277. d. May cause a machinery.. .
278. a. Coarctation of ... 279. a . COz washout 280. d. Kawasaki disease 281. c. TGA
282. c. Calcium chloride 283. d. 4 week 284. a. Sinus venosus ASD 285. c. TOF
286 . a. Narrow pulse. .. 287. d. Multifactorial 288. c. TOF
Cardiology

289. Essential criteria for TOF includes all except : 296. A 1-month-old boy is referred for failure to thrive. On
a. Valvular stenosis (AIMS Nov 07) examination, he shows feature of congestive failure.
b. Infundibular stenosis The femoral pulses are feeble as compared to bran ¬

c. Over riding of aorta chial pulses. The most likely clinical diagnosis is:

Q
(AI 2006)
d. RVH Ref: Harrison 19th p 1526 a. Congenital aortic stenosis
290. Which of the following is a component of Pentalogy of b. Coarctation of aorta
Fallot: (AI 2007) c. Patent ductus arteriosus
a. Atrial Septal Defect [ASD) d. Congenital aortoiliac disease
b. Patent Ductus Arteriosus [PDA] Ref Ghai 6th/ 419, Harrison 19th p 1525
c . Coarctation of Aorta (COA) 297. A Ten year old boy presents to the pediatric emer ¬
d. Transposition of Great Arteries (TGA) gency unit with seizures. Blood pressure in the upper
Ref Stedman's Medical Dictionary 28th/ l 452. extremity is measured as 200/140 mm Hg. Femoral
pulses were not palpable. The most likely diagnosis
291. In which of the following a 'Coeur en Sabot' shape of (AI 2010)
amongst the following is:
the heart is seen: (AI 2004) a. Takayasu Aortoarteritis
a. Tricuspid atresia b. Renal parenchymal disease
b. Ventricular septal defect .
c Grandmal seiures
c. Transposition of great arteries d. Coarctation of Aorta Ref: Harrison 19th p 1525
d. Tetralogy of Fallot 298. Which condition is most commonly associated with =
Ref: Ghai 5th / 303, Harrison 19th p 1527 coarctation of aorta? (AI 2008] j

Cardiolgy
a. PDA b. Bicuspid aortic valve
292. Potts shunt is : (AI 2001)
c. Aortic stenosis d. VSD
a. Right subclavian artery to right pulmonary artery
b. Descending aorta to left pulmonary artery Ref: Harrison 18th/ p 1925, Harrison 19th p 1525.
c . Left subclavian to left pulmonary artery 299. Coarctation of aorta is associated with all, except:
d. Ascending aorta to right pulmonary artery a. Turner's syndrome (AIIMS June 98)
e. Left Venticular Hypertrophy (LVH) b. Bicuspid aortic valve
c. Pulmonary stenosis
Ref: Sabiston 15th/ 2023-4, 0.R Ghai p 422 8th edn d. Atresia of aortic arch
293. All of the following are true about ASD except: Ref: Harrison 18th/ p 1925, Harrison 19th p 1525
a. Right atrial hypertrophy (AI 2001) 300. The most common type of total anomalous pulmonary
b. Left atrial hypertrophy venous connection is: (AI 2005)
c. Right ventricular hypertrophy a. Supracardiac b. Infracardiac.
d. Pulmonary hypertension c . Mixed. d. Cardiac.
Ref: Harrison 18th/ p 1921, 1922, Harrison 19th p 1521 Ref: Schwartz 8th/ 622, 0. P Ghai 8th edn p 427
ASD is associated with RA and RV hypertrophy but not 301. The heart lesion not found in Congenital Rubella in¬
LA hypertrophy. fection is: (AI 1995)
a. ASD b. VSD
294. MC cause of death in adult with PDA is : (PCI Dec 99) c. PDA d. PS
a. CCF
Ref: Ch 236, Harrison 18th edn, 0. P Ghai 8th edn p 401
b. Infective endocarditis
302. Infective endocarditis is least common in:
c. Rupture
(AIIMS Nov 93)
d. Embolism Ref Harrison 19th pi 523 a . Mitral stenosis b. Aortic stenosis
.
295 All of the following are characteristic features of Tri ¬
c. VSD d. ASD
cuspid Atresia Except: (AIIMS Dec 98) Ref: API 8th/ 500
a. Left Axis deviation 303. Least common site for vegetation is: (AIIMS May 94)
b. Right ventricular hypoplasia a . Aortic Stenosis (AS)
c. Pulmonary vascularity is diminished b. Mitral Stenosis (MS)
d. Splitting of S 2 c. Mitral Regurgitation (MR)
Ref Ghai 6th/ 408, 410, 414 - 415, Harrison 19th p 1527 d. Atrial Septal Defect (ASD) Ref: API 8th/ 500
Ans . 289 . a. Valvular stenosis 290 . a . Atrial Septal... 291 . d. Tetralogy of Fallot 292. b. Descending aorta to left...
293. b. Left atrial hy.. 294. a. CCF 295. d. Splitting of S 2 296. b. Coarctation of aorta
297 . d. Coarctation... 298. b. Bicuspid aortic valve 299. c. Pulmonary stenosis 300. a. Supracardiac
301. a . ASD 302. d. ASD 303. d. Atrial Septal Defect ( ASD )
Marwah's Internal Medicine MCQs ( Based on Harrison's 19th )

312. Thrombolytics are contraindicated in which condition:


Miscellaneous [ Manipal ]
a. CNS malignancy
304. The patients having acute cardiac failure do not show b. A-V fistula
edema, because: [ Manipal 2014] c. Pulmonary embolism
a. The plasma oncotic pressure is high d. Thrombophlebitis Ref KD Tripathi 5th pg 468
b. There is renal compensation
c. Increased cardiac output 313. Carotid sinus massage leads to: [ Manipal ]
d. There is a fall in the systemic capillary hydrostatic a. Bradypnea
pressure Ref: George Mathew pg 297 b. Reflex bradycardia
c. Reflex tachycardia
t 305. Type of pulse in "atrial fibrillation” : [ Manipal 2014]
d. Hyperpnea Ref Kumar and Clark pg 741
a. Regularly regular
b. Irregularly regular 314. Not a feature of ventricular tachycardia of aberrant
c. Regularly irregular conduction: [ Manipal ]
d. Irregularly irregular a. Wide QRS > 0.14 sees
Ref: P] Mehta pg 302 Harrison 19 th p 1483 b. Carotid massage causes a 2 : 1 block
1 306. Dissecting aneurism is a feature of: c. Oesophageal leads show AV dissociation
[ Manipal 2014]
a. Bloom's syndrome D. Deep S-wave Ref: Kumar & Clark Pg 746
b. Marfan’s syndrome 315. True about "Digoxin toxicity” : [ Manipal]
c. Balient syndrome a . QT interval is shortened
d. None of the above

Cardiolgy
b. Ventricular bigeminy
Ref Harrison 15th pg 1432 Harrison 19th p 2512 c. Atrial flutter
d. Mobitz II degree heart block
I 307 . Drugs used in CCF are all except: [ Manipal 2014]
a . Diuretics Ref Kumar and Clark pg 762; Harrison 19th p 42t
b. Vasodilators 316. Digoxin is useful in: [ Manipal]
c. Domapmine / Dobutamine a. Atrial ectopics
d . Amiodarone b. Flutter with 2 :1 block
Ref: KD Tripathi 5th Pg. 467 Harrison 19 th p 2029 c. Ventricular tachyarrhythmia
308. Causes of non-pitting oedema: [ Manipal 2014] a. Ventricular bigeminy
a . CCF b. Renal diseases Ref KD Tripathi 5th pg 466 Harrison 19 th p 42 t
c. Filariasis d. Cirrhosis [ Manipal ]
317 . Not a left to right shunt:
Ref PJ Mehta Pg 21 Harrison 19 th p 1418t a. PDA b. VSD
309. Features of "TIETZE's SYNDROME": [Manipal] c. ASD d. TGA
a . Costochondritis with swelling Ref Harshmohan pg 275
b. Costochondritis without swelling
318. On routine examination of a 25 - year - old man, an ejec ¬
c. Rheumatoid arthritis
d . Ankylosing spondylitis
tion systolic murmur with musical quality is heard.
What is the characteristic of this murmur?
Ref Harrison 19 th p 2246
[ DNB Pattern ]
310. Maintenance digoxin : [ Manipal] a . Best heard in lower left and mid-sternal border
a. 0.25 mg/ day b. Best heard when patient is lying in left lateral posi ¬
b. 2.5 micro gram / day tion
c. 25 mg / day c. Less intense with excitement and fever
d . 250 mg/ day d. None of above
Ref Harrison 19th p 42t Ref Harrison 19th p 1447
311. Aschoff nodules is seen in: [Manipal] 319. In CPR, all the following drugs may be administered
a. Rheumatoid arthritis by endotrachial route except: [ DNB Pattern]
b. Rheumatic heart disease a. Adrenaline b. Atropine
c. Wilson's disease
c. Lignocaine d. Sodium bicarbonate
d. Fabrey’s disease
Ref Harrison 19 th p 1769
Ref Harshmohan pg 295 Harrison 19 th p 2151

Ans . 304. d. There is a fall in ... 305. d . Irregularly irregular 306. b. Marfan ’s syndrome 307. d. Amiodarone
308. c. Filariasis 309. a . Costochondritis with... 310. b. 2.5 micro gram /day 311 . b. Rheumatic heart disease
312. a. CNS malignancy 313. b. Reflex bradycardia 314. b. Carotid massage. .. 315. b. Ventricular bigeminy
316. b. Flutter with ... 317. d. TGA 318. a. Best heard in lower... 319. d . Sodium bicarbonate
Cardiology

320. Paradoxical splitting of S2 is seen in all except: 329. Patient with HR = 40, ECG: absent P waves with normal
a. Right bundle branch block [ Manipal ] QRS complexes, has: [ DNB Pattern ]
b. Left bundle branch block a. Atrial fibrillation

B
c. Hypertension b. Incomplete AV block
d. Aortic stenosis Ref: Harrison 19th p 1447 c. Ectopic pacemaker
321. Most common cause of sudden death in post MI pa ¬ d. Sinus arrhythmia Ref: Harrison 19th p 1489
tients: [ Manipal ] 330. Risk factors for coronary artery disease are all except:
a. Cardiac aneurisms b. Arrhythmias [ DNB Pattern ]
c. LVF d. Myocardial rupture b. High IDL
a . High VLDL
Ref: Harrison 19th p 1597 c. High LDL d. Low HDL
322 . Canon waves are seen in: [ Manipal ] Ref: Park 17th pg 275 Harrison 19th p 1587
a. Constrictive pericarditis by. [ Manipal ]
331. Cardiac hypertrophy is diagnosed
b. Complete heart block
c. TR a. Chest X-ray
d. Atrial fibrillation Ref: Harrison 19th p 1483 b. Cardiac biopsy
c. Left ventricular thickness
323. Cause of decreased cardiac output is: [ Manipal ]
a. Weighing the heart
a. Decreased venous return
Ref: Harrison 19th p 1817
b. Decreased end diastolic volume
c. Increased sympathetic stimulation 332. Drug used in ventricular arrhythmia due to WPW syn
¬

d. Decreased vagal tone drome: [ Manipal]

Cardiolgy
Ref Chatterjee -Physiology pg 227 a. Amiodarone b. Digoxin
324. Not a feature of Fallot's tetralogy: [ Manipal ] c. Lignocaine d . Verapamil
a. Left ventricular hypertrophy Ref KD Tripathi 5th pg 481 Harrison 19th p 1481
b. Boot shaped heart 333 . All of the following drugs are indicated in immediate
c. VSD post MI period except: [ DNB Pattern]
d. Overriding of arch of aorta b. Morphine
a. CCBs
Ref Harrison 19th p 1527 d . Aspirin
c. Enalapril
325. Drug contraindicated in acute myocardial infraction: Ref: Harrison's 18th pg 2028 Harrison 19th p 105, 106 t
[DNB Pattern]
a. Nifedipine b. ACE inhibitors 334. Nocturnal angina and diaphoresis is a predominant
c. Methanol toxicity a. Aspirin symptom in: [ DNB Pattern ]
Ref: KD Tripathi 5th pg 497 Harrison 19th p 1589 a. Chronic AR b . Chronic MR
c. Acute AR d . Acute MR
326 . Post Operative hypertension can be due to all except: Ref: Harrison' s 18th pg 1942 Harrison 19 th p 1579
a. Pain [DNB Pattern]
b. Fear 335. Angina equivalent is a term given to: [ DNB Pattern]
c. Hypovolemia a. Breathlessness b. Sweating
d. Hypothermia Ref Harrison 19th p 201f c. Chest pain d. Palpitations
327 . About I.V. drug users all are true except: [DNB Pattern] Ref: Harrison 19th p 1530
a. Contaminated drug causes of endocarditis 336. Most common source of systemic emboli is: [Manipal]
b. Tricuspid valve is commonly affected a. Varicose veins
c. Staph aureus is causative organism b . Mural thrombi
d. Contaminated needles pass the infection c. Cardiac thrombi
Ref Harrison 19th p 822t
d. Pulmonary embolism
328. Common causative organism of 'Acute bacterial endo ¬ Ref: Kumar & Robbin, pg 91 Harrison 19th p 1980, 1980t
carditis': [ Manipal ]
a. Staph epidermidis 337 . Giant 'A' wave is in all except: [ DNB Pattern ]
b. Staph aureus a. Complete heart block
c. Streptococcus viridans b. Tricuspid stenosis
a. Strepto coccus pyogens c. Pulmonary hypertension
Ref: Kumar and Clark pg 794 Harrison 19th p 8221 d . MR Ref Harrison 19th p 1543

Ans. 320. b. Left bundle... 321 . b. Arrhythmias 322. d . Atrial fibrillation 323. a. Decreased venous return
324. a. Left ventricular... 325. .
a Nifedipine 326. c. Hypovolemia 327. a. Contaminated drug...
328. b. Staph aureus 329. a . Atrial fibrillation 330. b. High IDL 331. c. Left ventricular thickness
332. a. Amiodarone 333. a . CCBs 334. a . Chronic AR 335. c. Chest pain
336. b . Mumbthrombi 337. d . MR
Marwah's Internal Medicine MCQs ( Based on Harrison s
th
' 19 )

338. In HOCM, all are risk factors for sudden death except: 344. The most common type of total anomalous pulmo ¬
a. Left ventricle thickness > 30 mm [ Manipal ] nary venous connection is: ( A! 2005)
b. Mitral regurgitation a. Supracardiac b. Infracardiac.
c. Syncope c. Mixed. d. Cardiac.
d. History of non -sustained VT 345. Pulmonary edema associated with normal PCWP is
Ref: Harrison 19 th p 1568 observed . Which of these is not a cause: ( Al 01 )
A 41- year-old patient is diagnosed with infective en ¬
a. High altitude
docarditis. Which of the following has good progno ¬ b. Cocaine overdose
sis? [JIPMER 2014] c. Post cardiopulmonary bypass
d. Bilateral renal artery stenosis
a. Prosthetic valve edocarditis
Ref: Harrison 19 th p 1618
b. IV drug abuse
c. Staphylococcus aureus 346. Tuberoeruptive xanthomas are characteristic of fa ¬
d. Streptococcus viridians milial forms of: ( MP PG 2009 )
Ref: Harrison' s 18th Pg. 1062, Harrison 19 th p 821 a. Hypercholesterolemia
b. Dysbetalipoproteinemia
340. Only right sided cardiac sound which decreases in in¬ c. Defective apoB- 100
tensity with inspiration is: [JIPMER 2014 ] d . Sitosterolemia
a. Pulmonary ejection sound Ref: Harrison's 18/ e p3149, 17/ e p2419, Harrison 19th p 2440 t
b. Right ventricular S3
c. Systolic murmur of PS 347 . Tangier's disease is associated with ( AP 2012 )
a. High LDL b. HighTG

Cardiolgy
d. Systolic murmur of TR
c. High HDL d. Low HDL
Ref: At textbook of cardiovascular Medicine, 10th edn. Pg: 103
Ref: Harrison's 18/ e p3154, Harrison 19 th p 1443
341. True statement reading diagnosis of vegetations in
348. Pulmonary hypertension may occur in all of the fol ¬
infective endocarditis using transthroacic echocar¬ ( AIIMS Nov 06 )
lowing conditions except:
diography (TTE) is: [JIPMER 2014 ] a . Toxic oil syndrome
a. Has more than 90% sensitivity b. Progressive systemic sclerosis
b. Less than 2 mm vegetations is detected c. Sickle cell anemia
c. Transthoracic echocardiography equals transesopa - d . Argemone mexicana poisoning
geal echocardipgrapy in sensitivity Ref: Harrison's 18/ e p2082, Harrison 19 th p 1657
d . Absence of vegetations does not rule out infective
endocarditis 349. Rapid X descent unlikely in: ( PGI Dec 99 )
Ref . Harrison 19 th p 816 , 821
a . Constrictive pericarditis
b. Cardiac tamponade
342 . All of the following are features of early atropinisa - c. RVMI
tion except: ( Kerala PG 10) d. Restrictive cardiomyopathy
a. Blurring of vision b. Absence of sweat Ref: Harrison's 18/ e pi 975, Harrison 19 th p 1443
c. Drying of mouth d . Bradycardia
350. Which of the following about vasovagal syncope is
Ref: Goodman and Gillman, Chap-7, Harrison 19 th p 278e -3f false? ( Kerela PG 2008)
343. A 4 1 / 2 - year - old girl always had to wear warm socks a. Not mediated by bezold jarisch reflex
even is summer season . On physical examination, it b. Beta blocker can be useful in treatment
was noticed that she had high blood pressure and her c. Reversed by salt loading test
femoral pulse was weak as compared to radial and d . Dipyridamole may be useful in treatment
carotid pulse . A chest radiograph showed remarkable Ref: Harrison's 16 / e, pl 27, 130, Harrison 19 th p 1470
notching of ribs along with their lower borders. This 351. All of the following statement about atrial myxomas
was due to: ( AllMS Nov 02) are true, except: ( DNB 2011 )
a. Femoral artery thrombosis. a. Most common site is Left Atrium
b. Coarctation of aorta. b. Most common in young individuals
c. Raynaud's disease. c. Distant metastasis are rare
d. Takayasu 's arteritis. d . Most myxomas are familial
Ref: Harrison's 18/ e pl 925; Harrison 19 th p 1525 Ref: Harrison's 18/ e pi 979, 17 / e p

Ans. 338. b. Mitral regurg... 339. d. Streptococcus viri... 340. a. Pulmonary ejection ... 341. d . Absence of vegetations...
342. d . Bradycardia 343. b. Coarctation of aorta. 344. a. Supracardiac .
345. d . Bilateral renal artery..
346. b. Dysbetalipopro... 347. d. Low HDL 348. d . Argemone mexicana... 349. c. RVMI
350. a . Not mediated... 351. d. Most myxomas are familial
Cardiology

352. Which is true about measurement of BP with syphg- 359. Beyond which critical value Shock Index [Heart rate /
manometer versus intra-arterial pressure measure ¬ BP] in pregnancy is considered abnormal?
ments: ( A12001 ) ( AI 1MS Nov. 2012 )

D
a . Less than intravascular pressure a. 0.9-1.1 b. 0.5 0-.7
b. More than intravascular pressure c. 0.3-05 d. 0.7-0.9
c. Equal to intravascular pressure Ref: Net source: Pubmed.
d. Depends upon blood flow
360. Investigation of choice for aortic dissection with hy¬
Ref: Harrison's 18/ epl 824, Harrison 19 th p 1612 ( NBE Pattern 2014-15)
potension is ?
353 . Most common complication of cardiac catheterization: a. CT scan
( DNB 2013) b. Technetium 99 scan
a. Arrhythmia c. MR1
b. Hypertension d. T.E.E.
c. Vascular bleeding Ref: Harrison’s 18th p 2064; Harrison 19 th p 1641
d. Contrast reaction
361. All form boundaries of triangle of auscultation except?
Ref: Harrison's 18/ e pl 853- 1854
-
( NBE Pattern 2014 15)
354. Judkins technique used for: ( DNB 2013) a. Trapezius
a. Central venous line placement b. Latissmusdorsi
b. Coronary arteriography c. Scapula
c. Renal angiography d . Rhomboid major Ref: P) Mehta Clinical Methods

Cardiolgy
d . Chest tube insertion
362. Most common aortic branch involved in Takayasu Ar ¬
Ref : Harrison's 18/ e pl 854, 2005 teritis is? ( NBE Pattern 2014-15)
355. Most common cause of Dilated cardiomyopathy is: a . Left subclavian artery
a. Alcohol ( DNB 2013) b. Common carotid artery
b. Viral infection c. Abdominal aorta
c. Pregnancy d. Renal artery Ref: Harrison 19th p 2189
d. Metabolic disease 363. Therapeutic hypothermia is of benefit in preventing
Ref: Harrison's 18/ e pl 961, Harrison 19 th p 1556 neurological complications in ( NBE Pattern 2014 -15)
356. DRESS syndrome is associated with all except: a. Sepsis b. Poly-trauma
a. Eosinophilia ( NBE Pattern 2014-15) c. Cardiac arrest d. Ischemic stroke
b. Myocarditis Ref Harrison's, 18th p 2258-59
c. Endocarditis 364. Patient complains of dizziness. Best treatment for this
d. Encephalitis patient is? ( NBE Pattern 2014-15)
Ref: Harrison’s 18th p 435 Harrison 19 th p 382 a. Atropine b. Isoprenaline
357 . P.S.Tis: ( NBE Pattern 2014-15) c. Adrenaline d. Pacemaker
a . Phenol sulfotransferase deficiency Ref Harrison’s 18th ed. p 1874; Harrison 19th p 1474
b. Post traumatic stress disorder 365. Who invented the stethoscope: ( NBE Pattern 2014-15)
c. Protein S deficiency a. Rene Laenneac b. Leewenhelok
d . Paroxysmal supraventricular tachycardia c. Joseph Littmann d . William Osier
Ref: Net Source: Pubmed
Ref: Medical Encyclopedia
358. Impotence and loss of libido in hemochromatosis is 366. Cafe coronary term was coined by:
due to? ( NBE Pattern 2014-15) ( NBE Pattern 2014-15)
a. Iron deposition in heart a. Roger Hausen
b. iron deposition in pituitary b. William Osier
c. Iron deposition in Liver and increased estrogen c. Christian Barnard
d. Iron deposition in the testis d . Jean Marie Charcot
Ref: Harrison's, ch. 357 p 3157, Harrison 19 th p 2514 Ref: Medical Encyclopedia
Ans . 352. b . More than intra... 353. c. Vascular bleeding 354. b. Coronary arterio... 355. a. Alcohol
356. c. Endocarditis 357. a. Phenol sulfo... 358. -
b. Iron deposition in ... 359. a. 0.9 1.1
360. d . T. E.E . 361. d . Rhomboid major 362. a. Left subclavian artery 363. c. Cardiac arrest
364. a. Atropine 365. a . Rene Laenneac 366. a . Roger Hausen
th
Marwah 's Internal Medicine MCQ s ( Base d on Harr ison 's 19 )

367. While at the ward round, you see an elderly lady at¬ 374. What is not true about infective endocarditis?
tendant slump to the floor. Going to her aid , you no¬ a .
Hematuria ( NBE Pattern 2014-15)
tice her to be unresponsive and apneic. Your first step b. Rose spots seen
c. Splinter hemorrhages seen
in Adult Basic Life Support (CPR ) should be the fol ¬
d. Hemiplegia
lowing?
Ref: Harrison 18 th ch. 124 Harrison 19th p 817
a. Check for a carotid pulse ( NBE Pattern 2014 15) -
b. Assess breathing 375. Most common cause of infective endocarditis is?
c. Determine responsiveness a. Staphylococcus aureus ( NBE Pattern 2014 -15)
d. Institute chest compression b. Streptococcus viridians
c. Streptococcus pyogenes
Ref: Harrison' s 18"' p 2244 Harrison 19th p 1768 Ref : Harrison 19 th p 816
d. Streptococcus mutilan
368. Dissecting aneurysm is seen ml( NBE Pattern 2014 15) - 376. Dicrotic pulse is seen in? ( NBE Pattern 2014 -15)
a . Takayasu disease b. Atherosclerosis a. HOCM b. DCM
c. Syphilis aortitis d . Marfan syndrome c. RCM d. Left ventricular failure
Ref: Harrison' s, 18th ch. 248 p 2060 Harrison 19th p 1641 Ref : Harrison’s 18th ed ., ch 227 p 1824 - 25
I 369. Displacement of cardiac apex to left and downwards
indicates? ( NBE Pattern 2014 15) - 377 . Which of the following conditions causes both supe ¬
rior as well as inferior notching of the ribs?
a. Right ventricular hypertrophy ( NBE Pattern 2014 -15)
b . Left ventricular hypertrophy a . Coarctation of aorta b. Hyperparathyroidism
c. Right atrial hypertrophy c. Interrupted aortic arch d. Blalock Taussig shunt

Cardiolgy
d. Left atrial hypertrophy
Ref : Principles of cardiovascular radiology, pg. 86, ch. 7
Ref: Harrison 19 th p 1531
378. Sinus Bradycardia is defined as heart rate of ?
370. Osier's nodes are seen in? ( NBE Pattern 2014 15) - a . Less than 40 / min ( NBE Pattern 2014 -15)
a . Rheumatoid arthritis b. Less than 50 / min
b. Rheumatic heart disease c. Less than 60 / min
c. Subacute bacterial endocarditis d . Less than 70 / min
d . Typhoid Ref: Harrison’ s, 17th ed., ch. 225 Harrison 19 th p 1470
371. Pulsus alterans is seen in? ( NBE Pattern 2014 -15) 379. Most common presentation of cardiac lupus?
a . Anterior wall MI a. Myocarditis ( NBE Pattern 2014-15)
b. Bronchial asthma b. Pericarditis
c. Critical aortic stenosis c. Aortic regurgitation
d . Constrictive pericarditis d . Libman Sacks endocarditis
Ref: Harrison 19 th p 1446 Ref: Harrison' s 18th ed., ch. 319 p 2729 Harrison 19 th p 2129
380. Infective endocarditis where lifelong treatment is re ¬
372 . Most common cause of unilateral pedal edema? ( NBE Pattern 2014-15)
quired:
a. Pregnancy ( NBE Pattern 2014-15)
a. Aspergillus endocarditis
b. Lymphedema b. Libman sacks endocarditis
c. Venous insufficiency c. Fusarium solani
d. Milroy disease d . Enterococci
Ref: Harrison's, 18th ch. 249 p 2073 Harrison 19 th p 1651 Ref: Current diagnosis and treatment in Cardiology 2nd edition, ch 29
373 . A lady on anti hypertensive medication comes with 381. 60 year old female patient with cardiac prosthetic
hemiparesis and speech difficulty for 2.5 hours. BP is valve has serum creatinine of 3 mg % with pyo- ne-
180 / 100. What is the best treatment for this patient? phrosis. Investiga tion of choi ce for deter minin g pros ¬

( AllMS Nov. 2013) thetic valve damage is? ( NBE Pattern 2014-15)
a. Aggressive Reduction of BP a . Blood culture
b. Modest Lowering of BP b. Cinefluorography
c. Thromobolysis with tissue plasminogen activator c. Over penentrated CXR
d. Aspirin and Clopidogrel loading dose.
d . T.E.E.
Ref: Harrison s 18
' th Ch. 229 [ table 229.1 ] Harr ison 19 th p 820
Ref: American stroke association 2010 guidelines

d . Marfan syndrome 369 . b. Left ventricular. . . 370 . c . Suba cute bact erial . ..
Ans . 367 . d . Institute chest... 368 .
Mod est Lowe ring 374 . b . Rose spots seen
371 . a . Anterior wall Ml 372. c. Veno us insuff icienc y 373 . b . . . .
/ min
377 . b . Hype rpar athyr oidis m 378 . c . Less than 60
375. a . Staphylococcus ... 376 . b. DCM
379 . b. Pericarditis 380 . a . Aspergillus endocarditis 381 . d . T. E . E.
Cardiology

382. Raised Intra-abdominal pressure to consider abdomi ¬ c. ASD (Atrial septal defect)
nal compartment syndrome is? ( NBE Pattern 2014-15) d. Coarctation of aorta
a . 0-12 mm Hg b. >12 mm Hg Ref Harrison 18th/ p 1826 Harrison 19 th p 1447
c. > 20 mm Hg d . > 30 mm Hg
Ref: Schwartz textbook of surgery 9th ed., P 316 391. Fixed splitting of S 2 may be seen in all except:
a. Pulmonary embolism ( PG1 June 95)
383. Pulsus paradoxus is seen in all except: ( AIMS June 98)
b. PS
a . IPPV b. COPD c. ASD
c. Cardiac Temponade d . Constrictive pericarditis
d . LBBB
Ref Harrison 18th/ p 1824, 1825 Harrison 19 th p 1446
Ref Hurst 12th /268: 'Cardiology' 3rd/ ed Harrison 19thpl 447
384. Pulsus alternans occurs in : ( PGl-)une 98)
a . Constrictive pericarditis 392 . Third heart sound is seen in all except : ( AIMS Dec 98)
b. Viral myocarditis a. Athletes
c. Hypokalemia b. Mitral stenosis
d . MI c. Constrictive pericarditis
Ref Harrison 18th / p 1825 Harrison 19 th p 1446 d. LVF
Ref Harrison 18th/ p 1827 Harrison 19 th p 1448
385. Water Hammer pulse is seen in: (AIMS May 07 )
a. Aortic stenosis 393. True about third heart sound is: ( PCI Dec- 98 )
b. Aortic regurgitation a. Absent in Chr. Constrictive pericarditis
c. Aortic stenosis and Aortic regurgitation b. Absent in aortic aneurysm

Cardiolgy
d. Mitral regurgitation c. Absent in MS
Ref Harrison 18th/ p 1943, 1944 H Harrison 19 th p 1445 d . Normal physiologically in athletes
386. Giant 'a’ waves in JVP occur in all except: ( Ai 1996 ) Ref: Harrison 18th/ p 1827 Harrison 19 th p 1448
a. Junctional rhythm 394. S4 is seen in all of the following, Except:
b. Pulmonary hypertension a. Thyrotoxicosis ( AIMS May 94)
c. Tricuspid regurgitation b. Acute Ml
d. Complete heart block
c. Atrial fibrillation
Ref Harrison 18th/ p 1823 Harrison 19 th p 1444 d . Hypertrophic cardiomyopathy
387. C wave in JVP indicates: ( AI 2009) Ref Harrison 18th/ p 1827 Harrison 19 th p 1448
a . Atrial contraction
395. All of the following sounds are diastolic sounds, except:
b. Bulging of tricuspid valve
c. Ventricle systole ( PGI - Dec-06 )
d . Rapid ventricular filling a. S3 b. S 4
Ref Harrison 18th/ p 1823 Harrison 19 th p 1444 c. Opening snap d. Ejection click
Ref Harrison 18th/ p 1827 Harrison 19 th p 1447
388. All of the following are true about SI , except:
a Lower frequency than S 2 (AI 1991) 396. Continuous murmur is present in
b. Caused by closure of mitral valve a. PDA ( PGI June 06 )
c. Heard at the end of ventricular systole b. AS with AR
d. Better heard with diaphragm of stethoscope c. Shunt between pulmonary & subclavian arteiy
Ref Harrison 19 th p 1447 d. VSD with AR
389. First heart sound is soft in all, except: ( AIMS Dec 95) Ref Harrison 18th/ p 1829 Harrison 19 th p 1449
a. Short PR interval b. Ventricular septal defect 397 . The drug of choice in patients with Wolff- Parkinson -
c. Mitral regurgitation d. Calcified valve White syndrome with atrial fibrillation is:
Ref Harrison 18th/ p 1827 Harrison 19 th p 1447 ( AIMS Nov 03)
390. Wide split S 2 occurs in: ( AIMS May 93) a. Digitalis b. Procainamide
a. VSD (ventricular septal defect) c. Verapamil d. Adenosine
b. Mitral stenosis Ref: Hurst 12th/ 981, Harrison 19 th p 1483

Ans. 382. b. >12 mm Hg 383. a. IPPV 384. d . Ml 385. b. Aortic regurgitation


386. c. Tricuspid reg ... 387. b. Bulging of tricuspid... 388. c. Heard at the end of ... 389. a. Short PR interval
390. c. ASD ( Atrial... 391. d . LBBB 392. b. Mitral stenosis 393. c. Absent in MS
394. c. Atrial fibrillation 395. d. Ejection click 396. a. PDA 397. b. Procainamide
th )
Marwah's Internal Medicine MCQs ( Based on Harrison's 19

equ ency abla tion is don e for : ( AIIM S Jun e 98) 405. Agent of first choice in an acute attack of Prinzmetal's
398. Radiofr ( AI 1995)
angina is:
a. Ventricular tachycardia a. Diltiazem b. Nitrates
b. PSVT c. Propranolol d. Verapamil
i WPW Ref Harrison 19 th p 1598
d . Atrial tachycardia
406. Which test is performed to detect reversible myocar ¬
P Ref: Harrison 18th / p 1889, 1890, Harrison 19 th p 1483
dial ischemia ? ( AllMS May 03 )
I 399. Left atrial filling pressure closely approximates - a. Coronary angiography.
j ( AIIMS May 93 ) b. MUGA scan .
a. Pulmonary capillary wedge pressure c. Thallium scan.
b. Central venous pressure d. Resting echocardiography.
c. Intrapleural pressure Ref: Harrison 18th/ p 2024, Harrison 19 th p 1585
d. Intracranial pressure 407 . Pul sus par ado xus is a cha rac teri stic feat ure of :
Ref : Hurst 12 th / 494~ Harrison 19 th p 1461 a. Con stric tive peri card itis ( AIIM S Dec 92 )
b. Cardiac Tamponade
j 400. Pulmonary edema associated with normal PCWP is c. Hypertrophic obstructive cardiomyopathy
observed . Which of these is not a cause: ( Al 01 )
d . Restrictive cardiomyopathy
a . High altitude Ref Harrison 18th/ p 1975
b. Cocaine overdose
408. Rapid Reduction of blood pressure is indicated in :
c. Post cardiopulmonary bypass ( AI 1991 )
a . Acute Aortic Dissection

aCrdilogy
d . Bilateral renal artery stenosis b. Hypertensive Encephalopathy
Ref Harrison 18th edn, Ch - 247, Harrison 19th p 1617 c. Intracerebral haemorrhage
d . All of the above
401. A 26 yr old asymptomatic woman is found to have arr -
thymias and a systolic murmur associated with mid- Ref Braunwald 8th/ 1045, Harrison 19 th p 1627 - 1641
systolic clicks; which investigation would you use: 409. In Accelerated HTN what is metabolic defect:
a. Electrophysiological testing (AI 2001 ) ( PG1 June 2000)
b. Tc scan a . Normal non - ionic metabolic acidosis
c. Echocardiography b. Ionic gap met acidosis
c. Hypomagnesemia d . Metabolic alkalosis
d . Angiography
Ref: Harrison 19 th p 322
Ref Harrison 18th/ p 1937, Harrison 19 th p 1546
like ly to be as 410. Most common cause of renal artery stenosis in young
402 . Ang ina pect oris and Syn cope are mos t ¬
( AIIMS Dec 97)
adults in India is:
sociated with: ( AI 1994 )
a. Atherosclerosis
a . Mitral stenosis b. Aortic stenosis b. Non specific aorto-arteritis
c. Mitral regurgitation d . Tricuspid stenosis c. Fibro muscular dysplasia
Ref: Harrison 18th/ p 1939, Harrison 19 th p 1531 d. None of the above
Ref Disease of kidney and urinary tract 8th/ 1279
403 .Which of the following dietary interventions has shown
to reduce mor talit y in pati ents with coro nar y hea rt dis ¬ 411. Pulmonary hypertension in COPD is due to all, except:
' ( AIMS May 07 ) a. Hypoxia ( PGI Dec 97 )
ease. -
b. Pulmonary vasoconstriction
a. High Fibre diet c. High lung volume
b. Steral Esters d. Bronchoconstriction
c. Potassium supplements Ref Harrison 18th/ p 2081, Harrison 19 th p 1505
d . Omega 3 polysaturated fatty acids dis
412. AH the
of follo win g fact ors pre disp ose to Aor tic ¬

404. Which of the followi ng is the pre ferr ed mar ker for de ¬
( PGI June 02)
section, except:
tecting Acute STEMI in Atheletes: ( AI 2012) a . Systemic hypertension
a. CK-MB b. Troponin T/ l b. Coarctation of aorta
c. C- Reactive Protein d. LDH c. 1st trimester pregnancy
Ref: Harrison 18th/ 2023, 2024; Harrison 19th p 1600 d. Takayasu's arteritis Ref Harrison 19th p 1641

Ans. 398. c. WPW 399. a . Pulmonary capillary... 400. .


d . Bilateral renal . . 401. c. Echocardiography
402. b. Aortic stenosis 403. d. Omega 3 polysatur... 404. b. Troponin T/ l 405. b. Nitrates
406. c. Thallium scan 407. b. Cardiac Tamponade 408. d . All of the above 409. d . Metabolic alkalosis
410. .
b. Non specific. . 411. d. Bronchoconstriction 412. c. 1st trimester pregnancy
Cardiology

413. True regarding cardio pulmonary resuscitation is- 418. Least conduction velocity is seen in: ( PGIDec 98)
( AIIMS Dec 97) a. AV node
a . Most common presentation in ECG is asystole b. Purkinje fibres
b. Compression to ventilation ratio is 5:1

E
c. Bundle of HIS
c. Adrenaline is given if cardioversion fails d. Ventricular myocardial fibres
d . Calcium gluconate is given immediately Ref Ganong 22nd/ 547- 549; Harrison 19 th p 1451
Ref: Harrison 18th/ p 2244 Harrison 19 th pi 769
419. During the cardiac cycle the opening of the aortic
414. Mitral valve vegetations do not usually embolise to : valve takes place at the : ( A104 )
a - Lung
b. Liver
( AllMS Nov 2001] a . Beginning of systole
b. End of isovolumetric contraction
c. Spleen c. End of diastole
d. Brain d . End of diastasis
Ref Harrison 18th Ch 124, Harrison 19 th p 816 Ref: Guyton 10th / 99, Harrison 19 th p 1447
415. Most common heart valve involved in IV drug user is: 420. At the end of isometric relaxation phase ( AI 2000 )
a . Mitral valve ( AllMS Feb 97) a. Atrioventricular valves open
b. Aortic valve b. Atrioventricular valves close
c. Pulmonary valve c. Corresponds to peak of "C" wave in JVP
d . Tricuspid valve d. Corresponds to T wave in ECG
Ref Harrison 18th/ p 1055 Harrison 19th p 816 Ref: Guyton 10th/ 99

Cardiolgy
416. A patient with a prosthetic heart valve develops endo ¬ 421. Mean arterial pressure is calculated as:
carditis eight months after valve replacement. Most ( AllMS Nov 06 )
likely organism responsible is: ( AllMS Nov 2010) a. (SBP + 2 DBP) /3 b. (DBP + 2SBP) / 3
a. Staphylococcus Aureus c. (SBP + 3DBP) / 2 d. (DBP + 3SBP) / 2
b . Staphylococcus Epidermidis Ref: Chaudhary 5th / 244
c. Streptococcus Viridans 422 . Most common cause of aortic aneurysm is: ( AI 98)
d. HACEK group a. Syphilis b. Marfan’s syndrome
Ref: Harrison's 17th/ 790 Harrison 19 th p 817 c. Atherosclerosis d. Congenital
417 . Vegetations on undersurface of A. V. valves are found Ref Harrison 14th/ 1394, Harrison 19th p 1639
in : ( AI 2001 )
a. Acute Rheumatic corditis
b. Limban Sack's endocarditis
c. Non thrombotic bacterial endocarditis
d . Chronic rheumatic carditis
Ref: Harrison 18th, Ch 124, Harrison 19 th p 817

Ans. 413. c. Adrenaline is... 414. a. Lung 415. d . Tricuspid valve 416. b. Staphylococcus Epi...
417. b . Limban Sack’s... 418. a. AV node 419. b. End of isovolumet... 420. a . Atrioventricular valves
421 . a . (SBP + 2 DBP ) /3 422. c. Atherosclerosis
NOTE
CHAP TER Neurology
2
NEUROLOCV UPDATES
> Drug of Choice for Selection of Antiepileptic Drugs

Generalized-Onset Tonic-Clonic Focal Typical Atypical Absence,


Absence Myoclonic , Atonic
First-Line
Lamotrigine Lamotrigine Valproic acid Valproic acid
Valproic acid Carbamazepine Ethosuximide Lamotrigine
Oxcarbazepine Lamotrigine Topiramate
Phenytoin
Levetiracetam

Most Recent Q’s 2014- 15 5. First Cranial Nerve involved in increased ICP?
(JIPMER June 2015)
1. A middle aged man comes with RTA and bleeding a. 4th nerve b. 6th nerve
from the scalp. He is unconscious. A card in his pocket c. 9th nerve d. 10th nerve
reveals that he is a known diabetic on Climipiride + Ref: Harrison 19 th p 1792
Metformin 2 tablets twice daily. What should be the 6. Cafe au lait macules are the hallmark of:
next step? ( MIMS May 2015) ( NBE Pattern 2014 )
a. Send blood for tests, start iv glucose and send to CT a. Neurofibromatosis
b. Start normal saline and send to CT b. Von Hippel Lindau syndrome
c. Dextrose solution, CT scan c. Sturage weber syndrome
d. Airway, CT scan, Blood Sugar if < 70 start dextrose d. Angiokeratoma
Ref: Harrison 19th pi 775
Ref Harrison 19th p 604 -605
2. Hypertensive hemorrhage is most commonly seen in? 7. In right sided lesions of the frontal lobe eyes are devi ¬
( AUMS May 2015)
ated to: ( NBE Pattern 2014)
a. Basal ganglia b. Thalamus
a. Right b. Left
c. Brain stem d. Cerebrum
c. Upwards d. Downwards
Ref: Harrison 18th P 2582-83,
8. Friedreich's ataxia is the most common form of inher¬
3. A man comes with aphasia. He is unable to name things
ited ataxias and accounts for almost 50% of the cases.
and repetition is poor. However comprehension, fluen ¬
Which of the following gene mutation causes Fried¬
cy and understanding written words isunaffected. He
(AIIMS May 2015) reich ataxia? ( NBE Pattern 2014 )
is probably suffering from?
a. Frataxin b. Merlin
a. Anomic aphasia
c. Fibrillin d. Menin
b. Transcortical sensory aphasia
c. Conduction aphasia Ref Harrison 19th p 2630
d. Broca aphasia Ref Harrison 19 th pi 77, 2573
9. Pantothenate kinase associated neurodegeneration
4. Less in CSF when compared to plasma is all except. is another term for: ( NBE Pattern 2014)
(JIPMER June 2015) a. Fahr’s disease
a. Glucose b. Chloride b. Hallervorden -Spatz disease
c. Protein d. Calcium c. Machado-Joseph disease
Ref Harrison 19th p 149, 443e -3t d. Leigh disease Ref Harrison 19th p 2611

Ans. 1. d. Airway, CT... 2. a . Basal ganglia 3. c. Conduction aphasia 4. b. Chloride


5. b. 6th nerve 6. a . Neurofibromatosis 7. a. Right 8. a. Frataxin
9. b.Hallervorden-Spatz disease
th
Marwah's Internal Medicine MCQs ( Based on Harrison's 19 )

NEUROLOGY UPDATES

> Most Common Cause of Seizures

Perinatal hypoxia and ischemia

l
Neonates (<1 month)
Intracranial hemorrhage
Metabolic disturbances (hypoglycemia, hypocalcemia, hypomagnesemia, pyridoxine deficiency
Infants and children Febrile seizures
(>1 month and <12 years) Metabolic, degenerative, primary epilepsy syndromes
CNS infection
Developmental disorders
Adolescents (12-18 years) Trauma
Genetic disorders
Infection
Illicit drug use
Brain tumor
Young adults (18-35 years) Trauma
Alcohol withdrawal
Illicit drug use

Neurolgy
Brain tumor
Autoantibodies
Older adults (>35 years) Cerebrovascular disease
Brain tumor
Alcohol withdrawal
Metabolic disorders (uremia, hepatic failure, electrolyte abnormalities, hypoglycemia,
hyperglycemia
Autoantibodies

10. Wernickes encephalopathy consists of: 13. Which of the following is a false statement about Mill ¬
(NBE Pattern 2014) er fisher syndrome? (NBE Pattern 2014)
a. Mental confusion b. Ophthalmoplegia a. Ophthalmoplegia b. Ataxia
c. Cerebellar ataxia d. All of the above c. Areflexia d. Squint
Ref: Harrison 19th p 1734 Ref: Harrison 19th p 989
11. Which among the following statements about steroid 14. Dawson's fingers are seen in: (NBE Pattern 2014 )
myopathy is false? (NBE Pattern 2014) a. Multiple sclerosis b. Nueromyelitis optica
a. Highest incidence is with prednisolone c. Acute disseminating encephalomyelitis (ADEM)
b. Females are more susceptible d. Gullain Barre Syndrome
c. High dose steroids can cause acute onset quadriple- Ref Harrison 19th p 2661
gia 15. Anti - GDI antibodies are seen in which variant of
d. It is classically a proximal muscle weakness with GBS? (DNB Pattern 2014)
sparing of facial muscles and sphincters a. AIDP b. AMAN
Ref: Harrison 19th p 2130t c. AMSAN d. MFS
12. Which of the following is associated with fetal hy¬ Ref Harrison 19th p 2694
drops? (NBE Pattern 2014) 16. All are treatable diseases of spinal cord EXCEPT:
a. Alpha-thalassemia b. Beta-thalassemia (DNB Pattern 2014)
. d. Hereditary spheocytosis .
.-
c Sickle cell anemia a ALS b Multiple sclerosis
Ref: Harrison 19th p 638, 1196 c Syringomyelia d. A-V malformation
Ref: Harrison 19th p 2657
Ans . 10. d. All of the above .
11. a. Highest incidence is. . 12. a. Alpha -thalassemia 13. d. Squint
.
14 a. Multiple sclerosis 15. b. AMAN 16. a. ALS
Neurology

17. Which part of the brain is most affected in deep coma? 22. Person suffers seizure on waiting for train. Has band
(DNB Pattern 2014) showing him to be epileptic and the meds he takes.
a. Brain stem b. Locus ceruleus Meds are in his pocket. So what should you do?
c. Frontal lobe d. RAS (AUMS Nov.14) M
Ref: Harrison 19th p 1774 a. Take person away from train make sure he does not
18. Pseudo tumor cerebri is caused by? fall on tracks, meanwhile call for medical help and mmA
transfer to hospital
( DNB Pattern 2014)
a. Vitamin A excess b. Vitamin D excess b. Take person away from train stuff handkerchief M
c. in his mouth hold his hands and feet till seizure |
Vitamin E excess d. Vitamin B1excess
subsides then transfer to hospital ^
Ref Harrison 19th p c. Take person away from train make him lie down
19. All of the following can cause neuropathy with Pre ¬
hold his leg up give the meds and water then transfer
dominant motor involvement except (Manipal) to hospital
a. Acute inflammatory demyelinating polyneuropathy d. Take person away from train give meds and water
b. Acute intermittent porphyria then transfer to hospital
c. Lead intoxication
d. Arsenic intoxication
Ref : American epilepsy society guidelines 2014
Ref Harrison 19th p 2608 (NEETPattern 2015-16)
23. DOC of GTCS in pregnancy?
Epilepsy a. Lamotrigine b. CBZ
c. Levetiracetam d. Valproate
20. Which of the following drugs is not used in Juvenile Ref : p 329-330, Adam and Victor, principles of neurology, 8th

Neurolgy
Myoclonic Epilepsy (JME): (A12010) 24. A 72- year old man with normal renal functions pres ¬

a. Topiramate b. Zonisamide ents with new onset focal seizures. Which of the fol ¬

c. Carbamezapine d. Valproate lowing is the best drug to manage the patient?


Ref Harrison’s 18/ e p3262 Table 369.8, Harrison 19th p 2544 (AIPG 2012)
a. Sodium valproate
21. A 15 year old boy with epilepsy on treatment with
b. Oxcarbazepine
combination of valproate and phenytoin has good
c. Levetriacetam
control of seizures. Levels of both drugs are in the
d. Pregabalin
therapeutic range. All of the following adverse effects
can be attributed to valproate except: (Al 2004) Ref: Harrison’s 18th Ch. 369, Harrison 19th p 2552
a. Weight gain of 5 kg 25. In EEC type of wave seen in metabolic encephalopathy:
b. Serum alanine aminotransaminase 150 IU/L (NBE Pattern 2014 /15)
c. Rise in serum ammonia level by 20pg/ dL a. Alpha b. Beta
d. Lymphadenopathy c. Gamma d. Delta
Ref Harrison’s 18/ e p3263 Table 369.9, Harrison 19th p 408 Ref Handbook of stroke, 2nd pg 99
NEUROLOGY UPDATES
> Characteristics of the Mesial Temporal Lobe Epilepsy Syndrome

Clinical observations Laboratory Studies


Aura common Unilateral or bilateral anterior temporal spikes on EEG
Behavioral arrest/stare Hypometabolism on interictal PET
Complex automatisms Hypoperfusion on interictal SPECT
Unilateral posturing Material-specific memory deficits on intracranial amobarbital
Postictal disorientation (Wada) test
MRI Findings
Small hippocampus with increased signal on T2-weighted sequences
Small temporal lobe
Enlarged temporal horn

Ans. 17. d. RAS 18 . a. Vitamin A excess 19. d. Arsenic intoxication 20. c. Carbamezapine
21. d. Lymphadenopathy 22. a. Take person away... 23. a . Lamotrigine 24. b. Oxcarbazepine
25. d. Delta
Marwah's Internal Medicine MCQs ( Based on Harrison's 19th)

26. In Juvenile myoclonic epilepsy ( JME), most common 31. 7 year old girl is easily distracted in class and exhibits
presentation is? ( AIIMS May 2013) poor scholastic performance. Seizures are precipitat ¬

a. GTCS during sleep ed by hyperventilation. Diagnosis is?


b. GTCS on awake state

3
( NBE Pattern 2014/ 15)
c. Myoclonus a. Myoclonic seizures b. Absence seizures
d. Absence seizures c. Atonic seizures d. Myotonia
Ref: Harrison’s 18th P-3253, Harrison 19th p 2544 Ref: Nelson, Pg 593.4, Harrison 19th p 2543
27 . A 30 - year old man complains of falling asleep at work 32. Most effective management in medically intractable
frequently, which he attributes to disturbed sleep seizures? ( AIIMS May 2013)
at night. He also gives h/ o falls while partying with a. Ketotic diet
friends. Which of the following problems he might be b. Vagus nerve stimulation
facing? c. Deep brain stimulation
(AIIMS Nov 2012)
a. Paralysis during sleep-wake transition with halluci ¬ d. Surgery Ref: Harrison 19th p 2557
nations 33. Most common electrolyte abnormality causing sei ¬
b. Snoring and witnessed apneas zures in hospitalized patients is:
( NBE Pattern 2014/ 15)
c. Leg problems while going off to sleep a. Hyponatremia b. Hypernatremia
d. Generalized seizures in the wake state c. Hypokalemia d. Hyperkalemia
( Ref: Harrison's 18th Ch. 27, Harrison 19th p 189 Ref: Harrison’s 18th ch. 45, Harrison 19th p 298
34. Incorrect about Lafora’s disease is:

Neurolgy
28. New anti -epileptic drug for Lennox Gastaut syndrome
is: ( NBE Pattern 2014/ 15)
(AIIMS Nov 2012)
a. Lacosamide a. Myoclonus
b. Vigabatrin
b. Autosomal recessive
c. Rufinamide d. Zonisamide
c. Diagnosis by DNA sequencing
( Ref: Harrison’s 18th Ch. 369, Harrison 19th p 25551 d. Treatment with glucocerebrosidase infusions
129. A pregnant lady is a known case of juvenile myoclonic Ref: Harrison’ s 18th ch.369, Harrison 19 th p 2545t
epilepsy and is receiving sodium valproate. Which of
35. EEG showing < 3 Hz polyspike activity is seen in:
the following drugs is best suited after valproate for
( NBE Pattern 2014/ 15)
management of this patient. (AIIMS May 2013) a. Lennox Gastaut syndrome
a. Phenytoin b. Carbamazepine b. Absence seizures
c. Lamotrigine d. Lacosamide c. Juvenile myoclonic epilepsy
( Ref: H 18th table 368- 9 and 369- 9, Harrison 19th p 2559 d. GTCS
30. Which can differentiate between seizures and syncope? Ref: Harrison's 18th ch. 369, Harrison 19 th p 2544
(AIIMS May 2013)
a. Recovery from unconsciousness 36. Hysprrythmia on EEG is seen in: ( NBE Pattern 2014/ 15)
b. Injury due to fall a. Infantile tremor syndrome
c. Urinary incontinence b. Petit mal epilepsy
d. Tongue bite c. Rolandic epilepsy
( Ref: Harrison 18th table 399 -7., Harrison 19th p2551 d. Infantile spasm
Ref: Nelson, 18th Ch: 593.2

NEUROLOGY UPDATES
> Administration of Intravenous Recombinant Tissue Plasminogen Activator (RTPA ) for Acute Ischemic Stroke ( AIS )

Indication
Clinical diagnosis of stroke
Onset of symptoms to time of drug administration <4.5h
CT scan showing no hemorrhage or edema of > 1/3 of the MCA territory

Ans. 26. c. Myoclonus 27. a. Paralysis during... 28. c. Rufinamide 29. c. Lamotrigine
30. a. Recovery from... 31. b. Absence seizures 32. d. Surgery 33. a. Hyponatremia
I8 34. d. Treatment with .. . 35. a. Lennox Gastaut... 36. d. Infantile spasm
Neurology

NEUROLOCV UPDATES
> Contraindication for Reteplase for Acute Ischemic Stroke

E3
Contraindication
Sustained BP >185/110 mmHg despite treatment
Platelets <100, 000; HCT <25%
iw i /o ,; glucose
yiuoubt; <50 or >400
ur ^ mg//dL
H\JU rng UL
Use of heparin within 48 h and prolonged PTT, or elevated INR
.Rapidly improving symptoms
impiuviuy oyilipiuillb
Prior stroke or head injury within 3 months; prior intracranial hemorrhage
Major surgery in preceding 14 days
Minor stroke symptoms
Gastrointestinal bleeding in preceding 21 days
Recent myocardial infarction
Coma or stupor

37. Drug of choice in neonatal seizures: 43. 6 year old child with abnormal twitching of the face
( NBE Pattern 2014/ 15) during sleep noticed by mother. EEG shows spike over
a. Phenobarbitone b. Phenytoin Create - temporal area. Diagnosis:
c. Pentobarbital d. Topiramate ( NBE Pattern 2014/ 15)

Neurolgy
a. GTCS b. Rolandic epilepsy
Ref: O.P. Ghai pg. 527, 7th edn
c. Absence seizure d. Subtle seizure
38. Atypical febrile seizures are associated with: Ref: Nelson, 18th Ch. 593.2
( NBE Pattern 2014/ 15)
a. Complex partial seizures 44. A 29- year old, 4-month pregnant primigravida has a j
b. No post ictal deficit h/ o juvenile myoclonic epilepsy. She has been regu- j
c. Neuro-degeneration larly taking sodium valproate and now seeking an |
d. Raised 1CT Ref: Nelson, 18th Ch. 593.1 opinion for her antiepileptic regimen. What would J
39. Not seen in tuberous sclerosis ( AIIMS NOV 2014)
you suggest her? (AIIMS Nov. 2012) j
a. Immediately taper valproate and start lamotrigine
a. Giant cell Astrocytoma b. Continue valproate with monitoring of drug level
b. White matter migration lines
c. Switch to carbamazepine
c. Sub-ependymal nodules
Ref Harrison 19th p 604 d. Add lamotrigine to valproate
d. Ependymoma
Ref: H 18th Ch.369.Pg no.3269„ Harrison 19th p 2559
40. Pyknolepsy is seen in: ( NBE Pattern 2014/ 15)
a. Lennox Gastaut syndrome 45. All of the following are features of juvenile Myoclonic
epilepsy, except ( AIIMS May 04)
b. Absence seizure
a. Myoclonus on awakening
c. Narcolepsy
Ref CMDT 2013, Pg 968 b. Generalized tonic-clonic seizures
d. Adrenoleukodystrophy
c. Automatism
41. Best drug for photosensitive epilepsy: d . Absence seizures
( NBE Pattern 2014/ 15)
Ref : Harrison 18th/ p 3253, Harrison 19th p 2543
a. Valproate b. Topiramate
c. Ethosuximide d. Zonisamide 46. Myoclonic seizure typically seen in: ( PGIDec 98)
a. SSPE b. Cerebellar lesion
Ref Epilepsy comprehensive textbook volume 3, 2nd edn p 2561
c. Pontine lesion d. Thalamic lesion
42. Periodic epileptiform discharges in EEG are seen in: Ref : Harrison 18th/ p 3428, Harrison 19th p 2543
a g.S.P.E ( NBE Pattern 2014/ 15)
47 . Commonest type of seizure in newborn: ( Al 08)
b. Herpes simplex encephalitis
a. Clonic b. Tonic
c. Subdural effusion c. Subtle d. Myoclonic
d. Status epilepticus Ref Nelson, 18th ch. 275
Ref : Manual of Neonatal Care by Cloherty 6th/ 484

38. a. Complex partial... 39. d. Ependymoma 40. b. Absence seizure


Ans. 37. a. Phenobarbitone
42. a. S.S.P.E 43. b. Rolandic epilepsy 44. b. Continue valproate..
41. a . Valproate
45. c. Automatism 46. a. SSPE 47. c. Subtle
Marwah's Internal Medicine MCQs ( Based on Harrison's 19th )

NEUROLOGY UPDATES c. Hereditary sensory motor neuropathy ( HSMN )


d.

a
Multifocal motor neuropathy
Risk of Stroke Following Transient Ischemic Attack : The
Ref: H 18/ e p3473; Table 385-1, Harrison 19th p 2695t
>
ABCD2 score 51. Not seen in tuberous sclerosis (AIIMS Nov. 14)
a. Giant cell Astrocytoma
Clinical Factor Score b. White matter migration lines
A: Age S60 years 1 c. Sub-ependymal nodules
B: SBP > 140 mmHg or 1 d. Ependymoma Ref : Harrison 19th p 604
DBP >90 mmHg 52 . All of the following are true about von Hippel Lindau
C: Clinical symptoms 2 syndrome except? ( NBE Pattern 2014/ 15)
Unilateral weakness 1 a. Multiple tumours are uncommon
Speech disturbance b. Hemangio-pericytomas are seen in the cranio-spinal
without weakness 2 axis
D: Duration 1 c. Supra -tentorial lesions are seen
>60 min 1 d. Tumors of Schwann cells are seen
10-59 min
Ref: Harrison's Wed, pg 3078 andMedscape 2342
D: Diabetes (oral
medications or insulin ) 53. 48 year old woman with history of seizures has pre ¬
sented with gross haematuria and left flank pain. Ab ¬
Total Score Sum Each Category dominal CT scan reveals left perinephric hematoma

Neurolgy
2
ABCD Score Total 3-Month Rate of Stroke (%) with 3cm angiomyolipoma along with multiple right
0 0 renal angiomyolipomas measuring from 1.5 to 6.5 cm.
1 2 The most likely diagnosis is: (AIIMS Nov 2012)
2 3 a. Tuberous sclerosis
3 3 b. Von Hippel Landau syndrome
4 8 c. Familial angiolipomatosis
5 12 d. ADPKD Ref: Nelson ch. 596.2, Harrison 19 th p 604
6 17 54. Tuberculosis sclerosis is associated with all except:
7 22 a. Ash leaf macule (APPG 2014)
b. Shagreen patch
c. Schwannoma d. Adenoma sebaceum
48. A symmetric high -voltage , triphasic slow wave pat¬
Ref: Ch. 596.2 Nelson Textbook of Pediatrics, Harrison 19th p 604
tern is seen on EEG in the following : ( AIIMS May’ 06 )
a. Hepatic encephalopathy 55. A child presented to the casualty with seizures. On ex¬
b. Uremic encephalopathy amination oval hypo - pigmented macules were noted
c. Hypoxic encephalopathy on the trunk, along with sub- normal IQ. Probable di ¬
d . Hypercarbic encephalopathy agnosis of the child is: (AIIMS Nov 2012)
Ref.: Harrison 18th/ p 2601 a. Neurofibromatosis b. Sturge Weber
c. Tuberous sclerosis d. Incontinentia Pigmenti
Neurocutaneous Disorders Ref: O.P. Ghai 7th Pg. No. 564, Harrison 19th p 604
56. An 8 year old boy has mental retardation. On exami ¬
49. All of the following neurophysiological defects are likely nation he is found to have a well defined kidney lump.
to result from right lobe involvement, except: CT scan shows a well - defined hypo-echoic lesion in
a. Visuospatial defects (AIIMS May 08) the kidney and multiple lesions in the liver showing
b. Anosognosia density of - 50 to - 80 Hounsfield units. Probable diag¬
c. Dyscalculi nosis is: (AIIMS Nov 2012)
d. Spatial dysgraphia a. Autosomal recessive polycystic kidney
50. Conduction velocity of nerve is reduced in all of the b. Tuberous sclerosis
following conditions, except: c. Von Hippel Landau
(AI 2012)
a. Acute motor axonal neuropathy (AMAN ) d. Paraganglioma
b. Acute inflammatory demyelinating neuropathy (AIDP) Ref: ch. 596.2 Nelson 18th edn, Harrison 19th p 604

Ans. 48. a . Hepatic ence... 49. c. Dyscalculi 50. a. Acute motor axonal...
52. a. Multiple tumours... 53, a. Tuberous sclerosis 51. d. Ependymoma
54. c. Schwannoma 55. c. Tuberous sclerosis
56. b. Tuberous sclerosis
Neurology

57. The diagnosis of a patient presenting with Seizures,


63. Not essential for brain death: ( NEET Pattern 2015-16 )
Mental retardation and Sebaceous adenoma is:
a. Negative Apnea test
a. Hypothyroidism ( AI 1995) b. Loss of brain stem function
b. Tuberous sclerosis

u
c. Loss of pupillary reflex
c. Toxoplasmosis d. Loss of deep tendon reflexes
d. Down's syndrome
Ref : Harrison 18th edn, Ch 274: Harrison 19th p 1775, 1776
Ref: Nelson 18th/2483, 2484, Harrison 19th p 604
64. Which of the following is a sign of brainstem death?
58. Metabolic encephalopathy presents with : a. Doll’s eye reflex ( AIMS Nov 2012)
a. Broca's aphasia ( AIMSJune 99 ) b. Fixed non-reactive pupil
b. Anomic aphasia c. Horner’s pupil
c. Transcortical sensory aphasia d. Positive vestibulo cochlear reflex
d. Transcortical motor aphasia Ref H 18/ e, pg 2250, Harrison 19th p 1776
Ref : Harrison 19th p 178 65. Pseudotumor Cerebri is seen in? ( AIIMS May 2013)
a. Obese women in the age group 20- 40 yrs
Raised ICT b. Obese males 20-40 yrs
c. Thin females 50-60 yrs
59. A 22 - year old obese female experiences right eye dip¬ d. Thin males 50-60 yrs
lopia. She had increased weight gain in last year and Ref Harrison’s 18th P- 233, Harrison 19th p 203
her current BMI is 35. Fundus examination reveals
66. In Glasgow coma scale, withdrawal to pain comes
papilledema on the right side. Most likely cause for
( AIMS Nov 2012)

Neurlogy
under which score?
her symptoms is: (JIPMAR 2014 ) a. M 2
a. Craniopharyngioma b. M 3
b. Idiopathic intracranial hypertension c. M 4
c. Optic neuritis d. M 5
d. Sagittal thromboembolism Ref Harrison 18th edn Ch 267: Harrison 19th pi 777
Ref : Harrison 19th p 1615, 1627, 2582
60. A 24-year -old man falls on the ground when he is
struck in the right temple by a baseball . While being
NEUROLOGY UPDATES
driven to the hospital, he lapses into coma. He is unre ¬
sponsive with the dilated right pupil when he reaches
the emergency department. The most important step > The ICH Score
in initial management is: (AI 2002 )
Clinical or Imaging Factor Point Score
a. Craniotomy
b. CT scan of the head Age
<80 years 0
c. X-ray of the skull and cervical spine s80 years 1
d. Doppler ultrasound examination of the neck
Hematoma Volume
Ref: Harrison's 18/ e p3379, Harrison 19th p 184
<30 cc 0
61. Cushing's triad has all except? ( AIMS Nov. 14) >30 cc 1
a. Bradycardia Intraventricular Hemorrhage Present
b. Hypotension No 0
c. Abnormal breathing Yes 1
d. Posturing Infratentorial Origin of Hemorrhage
Ref : Greenburg Handbook of Neurosurgery, 7th Ch 27, p 868, No 0
62. Increased ICT is shown by? -
( NEET Pattern 2015 16 ) Yes 1
a. Miosis Glasgow Coma Scale Score
b. Systemic hypotension 13-15 0
c. Tachycardia 5-12 1
d. Reduction in GCS -
34 2
Ref : Harrison’s 18th edn, Ch 367, Harrison 19th pi 779

Ans. 57. b. Tuberous... 58. b. Anomic aphasia 59. b. Idiopathic... 60. a. Craniotomy
62. d. Reduction in GCS 63. d. Loss of deep tendon 64. b. Fixed non-reactive pupil
61. d. Posturing
65. a . Obese women in... 66. c. M4
Marwah's Internal Medicine MCQs ( Based on Harrison's 19th )

NEUROLOGY UPDATES

a > Cranial Nerve Syndromes

Site Cranial Nerves Usual Cause

Internal auditory meatus VII , VIII Tumors of petrous bone (dermoids, etc.);
infectious processes; acoustic neuroma
Pontocerebellar angle V, VII , VIII , AND sometimes IX Acoustic neuroma ; meningioma
Jugular foramen ..
IX X XI Tumors and aneurysms
Posterior retroparotid space IX, X, XI , XII AND Horner’s syndrome Tumors of parotid gland , carotid
body, lymph nodes; metastatic tumor;
tuberculous adenitis
Sphenoid fissure (superior orbital ) III , IV, first division V, VI Invasive tumors of sphenoid bone;
aneurysms
Lateral wall of cavernous sinus III , IV, first division V, VI , often with Infection , thrombosis, aneurysm , or fistula
proptosis of cavernous sinus; invasive tumors
from sinuses and sella turcica ; benign
granuloma responsive to glucocorticoids

Neurolgy
67. Cerebral perfusion pressure to be maintained in road 72. A waves in ICP monitoring is due to:
traffic accident case is: ( NBE Pattern 2014/15) a. Atrial contraction ( NBE Pattern 2014/ 15)
a. 30-50 mm Hg b. 50-70 mm Hg b. Cheyne stokes breathing
c. 70-90 mm Hg d. >90 mm Hg c. BP fluctuations
Ref: Harrison's 18th ch:275, Harrison 19th p 1780 d. Brain herniation
68. DOC for idiopathic intra -cranial hypertension: (I. I.H ) Ref: Handbook of Neurosurgery, Greenberg, Pg :653
a. Acetazolamide ( NBE Pattern 2014/ 15) 73. You are a doctor in C.H.C. when a patient of acute head
b. Glycerol injury comes with worsening of CCS leading to coma.
c. Mannitol Patient has unilateral dilatation of pupil and hemiple ¬
d. Dexamethasone gia. You decide to do burr hole at? ( AIIMS NOV 2014)
Ref: Walsh and Hoyt Clinical Neuro-ophthalmology P. 285-86 a. Refer to higher centre if not known which side bleed
is present
69 . Pseudo-Tumour Cerebri is caused by all except:
b. If no localizing signs, hole in left temporal (domi
a. Vitamin A toxicity
¬
( NBE Pattern 2014/ 15)
nant lobe]
b. PCM toxicity
c. Burr hole in middle
c. Sudden stoppage of steroids
d. Burr hole contralateral to the dilated pupil
d . Obesity
Ref: Greenfield handbook of neurosurgery, Ch 27, p 864-65
Ref: Walsh and Hoyt Clinical Neuro-ophthalmology Pg. 280
74. Dilated ventricles with normal CSF pressure is seen in:
70 . In case of head injury in children intracranial pres¬
a. Hydrocephalus ( NBE Pattern 2014/ 15)
sure to be maintained below: (NBE Pattern 2014/15)
b. 10 mm Hg b. Pseudotumour cerebri
a. 5 mm Hg
c. Hydrocephalus ex-vacuo
c. 20 mm Hg d. 30 mm Hg
d. Normal variant Ref: OP. Ghai, 549, 7th ed
Ref: Nelson Table 602
75. Triad of normal - pressure hydrocephalus includes :
71. Transtentorial uncal herniation causes all except: a. Tremor, aphasia dementia
a. Ipsilateral dilated pupils ( NBE Pattern 2014/ 15) (AI 1999)
b. Ataxia, aphasia, gait disorder
b. Ipsilateral hemiplegia
c. Gait disorder, urinary incontinence, dementia
c. Cheyne stokes respiration
d. Gait disorder, urinary incontinence, lower cranial
d. Withdrawal reflex
nerve palsy
Ref: Harrison's 18th ch. 274, Harrison 19 th p 1772
Ref : Harrison 18th/ p 3313, Harrison 19 th p 2606 07 -
Ans. 67. c. 70-90 mm Hg 68. a . Acetazolamide 69. b. PCM toxicity 70. a. 5 mm Hg
71. d. Withdrawal 72. d. Brain herniation 73. b. If no localizing signs... 74. c. Hydrocephalus ex-vacuo
75. c. Gait disorder, urinary...
Neurology

76. All the following are features of Pseudotumor Cerebri Headache


except : (A11996)
a. Normal-sized ventricles on CT scan
81. A 22 - year- old man presents with history of headache
b. Increased protein in CSF

E3
for 6 months which is mainly on frontal region oc¬
c. Papilledema casionally associated with nausea. He has been tak¬
d. Absence of focal neurological deficit
ing paracetamol 3g per day, hydroxycodeine 50 mg 3
Ref.: Harrison 18th/ p 126, 233, Harrison 19th p 203 times a day and aspirin 300 mg 3 times a day for head ¬
77. In a patient with head injury, damage in the brain is aches, but only with temporary relief from symptoms.
aggravated by (AI 2010) No focal signs on neurological examination. He also
a. Hyperglycemia b. Hypothermia has history of depression and is on treatment for 2
c. Hypocapnia d. Serum osmolality year with paroxetine now. What is the diagnosis?
a. Cluster headache (JIPMER 2014 )
Ref : Harrison 18th/ p 2257, 2258, Harrison 19th pi 780
b. Migraine
78. An elderly female presented with history ofprogressive c. Depression associated headache
right-sided weakness and speech difficulty. She gives a d. NSAIDs oversue /Analgesic abuse headache
history of a fall in her bathroom two months back. The Ref : Harrison's 18th Pg. 125, Harrison 19 th p 2594
most likely clinical diagnosis is : ( Al 91 )
a. Progressive supranuclear palsy 82 . A 65yr old lady underwent mastectomy. 6 months
b. Left cerebral tumor later she developed headache with pain at temple re ¬
c. Left sided stroke gion with ESR 55mm fall in 1 st hour. Diagnosis?
= (JIPMER 2014)

Neurolgy
d. Left chronic subdural haematoma
Ref : Ch - 370 Harrison 18th, Harrison 19th p 1772
a. Giant cell arteritis b. Meningeal metastasis
c. Tension headache d. P.A.N
79. The earliest manifestations of increased intracranial Ref : Harrison 18th Ch 326 and 379, edn, Harrison 19th p 2188
pressure following head injury is : ( AI 2005)
83. Sumatriptan is contraindicated in all except:
a. Ipsilateral papillary dilatation ( NBE Pattern 2014/ 15)
a. Basiliar migraine
b. Contralateral papillary dilatation
c. Altered mental status b. Ischemic heart disease
d. Hemiparesis c. Pregnancy
d. Ergot alkaloids in last one week
Ref : Harrison 18th / p 2257, Harrison 19 th p 1779
Ref: Harrison's 18th Ch.14, Harrison 19 th p 2591
80. Criteria for Brainstem death includes: (Al 2012)
a. Positive Doll’s eye Reflex 84. All of the following cause metabolic headache except:
a. Hypoxia ( NBE Pattern 2014/ 15)
b. Absent pupillary light reflex and dilated pupils
b. Hypercapnia
c. Pin-point pupils
d . Positive vestibulo -ocular reflex c. Hyperglycemia
d. Hyponatremia
Ref : Harrison 18th/ 2252, Harrison 19th p 1774 -75
Ref: Harrison's 18th Ch:39, Harrison 19th p 118

NEUROLOGY UPDATES
> Diagnostic Criteria for Neuromyelitis Optica

Required
.
1 Optic neuritis
2. Acute transverse myelitis
Supportive ( 2 of 3 criteria required):
over 3 more vertebral segments
1. Longitudinally extensive cord lesion extending
2. Brian magnetic resonance imaging normal or not meeting criteria for multiple sclerosis
-
3. Aquaporin 4 seropositivity

Ans. 76. b. Increased protein... 77. a. Hyperglycemia 78. d. Left chronic sub... 79. c. Altered mental status
80. c. Pin-point pupils 81. d. NSAIDs oversue/ Ana... 82. a. Giant ceil arteritis 83. d. Ergot alkaloids in last...
84. c. Hyperglycemia
th
Marwah's Internal Medicine MCQs ( Based on Harrison's 19 )

|: dl -
:/•] Mini 11 J »7:1141
Demyelination
> Electrophysiologic Features: Axonal Degeneration Versus Segmental

3 CMAP amplitude
Distal latency
Conduction velocity
Conduction block
Temporal dispersion
Axonal Degeneration
Decreased
Normal
Normal
Absent
Absent
Normal
Prolonged
Slow
Present
Present
Normal or absent Prolonged or absent
F wave
Normal or absent Prolonged or absent
FI reflex
Needle EMG Mi
Spontaneous activity
Fibrillations Present Absent
Fasciculations Present Absent
Motor unit potentials
Decreased

Neurolgy
Recruitment Decreased
Morphology Long duration/ polyphasic Normal

85. A 70 year old retired Military person with good pre ¬ 88. A young female presents with severe headache and
vious medical record complains of bi - temporal head ¬ neck stiffness of abrupt onset. She says, she has never
ache which is decreased in lying down position. He had such severe headache before. She also complains
states that he gets relief by giving pressure over bi¬ of associated nausea and photophobia. Likely diagno ¬
lateral temples. The patient also complains of loss of sis is:
appetite with feeling feverish. The most probable di ¬ a. Subarachnoid hemorrhage (SAH] ( AIIMSMay 09)
agnosis is: (AllMS Nov 2012) b. Migraine
a. Chronic tension headache c. Viral Encephalitis
b. Temporal arteritis d . Hydrocephalus
c. Migraine Ref : Harrison’s 17th/ l 727, Harrison 19th pi 784
d. Fibromyalgia 89. Sudden excruciating headache is most characteristic of
Ref: CMDT 2011 pg. 819 and PubMed, Harrison 19th p 2188 a. SAH ( PGI Dec 01 )
86 . Cluster headache is characterized by all, except: b. Aneurysmal bleeding
a. Affects predominantly females. (AI 2005) c. Epilepsy
b. Unilateral headache. d. Intracerebral hemorrhage
c. Onset typically in 20-50 years of life. e. Hysteria
d. Associated with conjunctival congestion. 90. A 25 -year-old male had severe headache followed by
Ref : Harrison 18th/ p 122, Harrison 19th p 2595 unconsciousness. CSF tap revealed red blood cells.
87. What is drug of choice for acute attack of migraine - Most likely diagnosis is: (AIIMS Dec 94 )
a. Methysergide a. Subdural hematoma
(AIlMSMay 95)
b. Caffeine b. Embolism
c. Amitryptiline c. Tumour
d. Sumatriptan d. Ruptured aneurysm
Ref : Harrison 18th/ p 117, 118, 119, Harrison 19th p 2591 Ref : Harrison 19th pi 785

Ans. 85. b. Temporal arteritis 86. a. Affects predo... 87. d. Sumatriptan


89. a. b SAH & Aneury... 88. a. Subarachnoid hemorrhage
90. d. Ruptured aneurysm
Neurology

Brain Tumor 96. Brain tumour with worst prognosis in children:


a. Cerebellar astrocytoma ( NBE Pattern 2014/15)

a
91.
,
Most common site of subependymal astrocytoma (giant
ce 1): (AIMS Nov 07)
b. Brainstem glioma
c. Craniophyrangioma
a. Trigone of lateral ventricle d. Pineal body tumour Ref OP. Ghai, 545, 7th ed
b. Foramen of Munro
c. 97. Most common intracranial tumour is: (AIMS May 94)
Temporal horn of lateral ventricle
d. 4th ventricle a. Meningioma b. Glioma
Ref Harrison 19th p 599 c. Craniopharyngioma d . Metastasis
92. Hemangioblastoma associated with VHL are most
commonly seen in:
Ref : Harrison 19th p 598
(DNB 2012 )
a . Cerebellum b. Liver 98. Which of the following is the most common type of
c. Kidney d . Pancreas Glial tumors? (AI 06 )
Ref: Harrison's 18/ e p2360, Harrison 19th p 2239 a . Astrocytomas b. Medulloblastomas
c. Neurofibromas d . Ependymomas
93. Mass in brain managed by chemo and radiation. After
2 months, she develops again vomiting and headache. Ref : Harrisons 16th/ 2453, Harrison 19th p 599
What is the investigation of choice to detect the mass? 99. Most common site of sub ependymal astrocytoma (gi ¬

a . Contrast enhanced MR1 (JIPMER 2014) ant cell) (AIMS Nov 07)
b. PET Scan a . Trigone of lateral ventricle
c. MRI b. Foramen of Munro
d. CT scan
c. Temporal horn of lateral ventricle

Neurolgy
Ref : Imaging in clinical oncology 5th edn, p 140, d. 4th ventricle
94. Most common cancer showing Ieptomeningeal metas
Ref..-Neurology in clinical practice 4th /1428;
¬

tasis to brain: (Bihar PG 2014)


a. Lung cancer b. Malignant melanoma 100. All of the following tumors may be malignant except:
c. Breast cancer d . Prostate cancer a . Glioma (AI 1997)
Ref: Harrison's 18th ch. 379, Harrison 19th p 605 b. Astrocytoma
95. A neurosurgeon in parent teacher meeting saw a child c. Hemangioblastoma
with precocious puberty anduncontrollable laughing. d. Ependymoma Ref : Chandrasoma Taylor 2 nd / 933
He suggested the parent of child to get a MRI done to 101. Which of the following brain tumors does not spread
evaluate for the possibility of: (AIMS Nov 2012) via CSF? (AI 2004)
a . Hypothalamic Hamartoma a . Germ cell tumors b. Medulloblastoma
b. Pineo-germinoma c. CNS Lymphoma d . Craniopharyngioma
c. Pituitary adenoma
d. Craniophrangioma Ref: Nelson 18th ch. 563.2 Ref : Harrison 19th p 603

NEUROLOGY UPDATES
> Important Neuropathy

Lead Encephalopathy; motor neuropathy (often resembles radial Axonal degeneration of motor axons
neuropathy with wrist and finger drop); autonomic neuropathy;
bluish-black discoloration of gums
Mercury Abdominal pain and nephrotic syndrome ; encephalopathy; Axonal degeneration; degeneration of dorsal
ataxia ; paresthesias root ganglia , calcarine, and cerebellar cortex
Thallium Encephalopathy; painful sensory symptoms; mild loss of Axonal degeneration
vibration; distal generalized weakness may also develop;
autonomic neuropathy; alopecia
Arsenic Abdominal discomfort , burning pain, and paresthesias; Axonal degeneration
generalized weakness; autonomic insufficiency ; can resemble
GBS

Ans . 91 . b. Foramen of Munro 92. a . Cerebellum 93 . b. PET Scan 94. c . Breast cancer
95. a. Hypothalamic Ha ... 96. b . Brainstem glioma 97 . b. Glioma 98. a . Astrocytomas
99 . b. Foramen of Munro 100. c. Hemangioblastoma 101. d. Craniopharyngioma
Marwah's Internal Medicine MCQs ( Based on Harrison's 19th)

NEUROLOGY UPDATES

> Brighton Criteria for Diagnosis of Guillain-Barre Syndrome (GBS ) and Miller Fisher Syndrome

3 Clinical case definitions for diagnosis of GBS


1.
2.
3.

4.
5.
Bilateral AND flaccid weakness of the limbs
Decreased or absent deep Rendon reflexes in weak limbs
Monophasic illness pattern and interval between onset and nadir of weakness between 12 h and 28 days and subsequent
clinical plateau
Electrophysiologic findings consistent with GBS
Cytoalbuminologic dissociation (i.e. elevation of CSF protein level above laboratory normal value AND CSF total white cell
count <50 cells/pL
6. Absence of an identified alternative diagnosis for weakness
Clinical case definitions for diagnosis of Miller Fisher syndrome
1. Bilateral ophthalmoparesis and bilateral reduced or absent tendon reflexes, and ataxia
2. Absence of limb weakness
3. Monophasic illness pattern and interval between onset and nadir of weakness between 12 h and 28 days and subsequent
clinical plateau
4. Cytoalbuminologic dissociation (i.e., elevation of cerebrospinal protein above the laboratory normal and total CSF white

Neurolgy
cell count <50 cells/pL
5. Nerve conduction studies are normal, OR indicate involvement of sensory nerves only
6. No alterations in consciousness or corticospinal tract signs

.
102 All the following are true of Craniopharyngioma 106. Brain tumor is associated with A /E: (PGI Dec 99)
except ( AI 1994 ) a. Tuberous sclerosis
a. Derived from Rathke's pouch b. Von Hippel Landau syndrome
b. Contains epithelial cells c. Neurofibromatosis
c. Present in sellar or infra-sellar location d. Sturge Weber syndrome
d. Causes visual disturbances
Ref : Harrison 18th/ p 2360, 3389, 3390, OP Ghai p 586
Ref.: Harrison 18th / p 2883, Harrison 19th p 603
.
103 Neurofibromatosis I is most commonly associated Alzheimer & Dementia
with: (AIIMS Nov 07)
a. Brain stem gliomas 107. Which of the following areas of brain is most resistant
b. Optic nerve glioma to Neurofibrillary tangles in Alzheimer’s disease:
c. Sub ependymal pilocytic astrocytoma (Al 2012)
d. Glioblastoma multiforme a. Entorhinal cortex
b. Hippocampus / temporal lobe
Ref : Rudolph's Pediatrics 21st/ 2399 c. Lateral geniculate body
.
104 CNS tumor seen in Von Hippel Lindau syndrome is: d. Visual association area
a. Meningioma (PGIDec 99) Ref: Harrison’s 18/ e p3306, Harrison 19th p 2598
b. Cerebellar hemangioblastoma
c. CNS lymphoma .
108 Which of the following is a cause of reversible demen ¬

d. Glioma tia? ( A1200S )


Ref : Harrison 18th / p 2360 a. Subacute combined degeneration.
.
105 All of the following tumors are malignant except: b. Picks disease.
a. Glioma (Al 1997) c. Creutzfeldt - Jakob disease.
b. Astrocytoma d. Alzheimer’s disease.
c. Hemangioblastoma
d. Ependymoma Ref : Chandrasoma Taylor 2nd / 933 Ref: Harrison’s 18/ e p3302, Harrison 19th p 1263/170

Ans. 102. c. Present in sellar... 103. b. Optic nerve glioma 104. b. Cerebellar hemangio... 105. c. Hemangioblastoma
106. d. Sturge Weber... 107. c. Lateral geniculate body 108. a. Subacute combined degeneration.
Neurology

109. Pick's body in pick’s disease is: (AI 2008) 116. With ageing, a slight decrease in cognitive impairment
a. Tau protein b. Alpha synuclein is seen due to increase in level of: ( ARMS May 2010 )
c. Beta synuclein d. A amyloid a. Homocysteine b. Taurine
Ref: Harrison’ s 18/ e p3311, Harrison 19th p 175 c. Methionine d. Cysteine
110. Not seen in Alzheimer’s Disease? Ref CMDT 2012 / pg. 1620, Harrison 19 th p 434e
( NEET Pattern 2015-16 ) 117. Lobes affected in Alzhiemers: ( ARMS May 2013)
a. Cerebral atrophy a. Frontal and temporal lobe
b. Ventricular shrinkage b. Temporal and parietal lobe
c. Sun downing c. Parietal and occipital lobe
d. Apraxia d. Parietal and frontal lobe
Ref : Harrison’s 18th edn, Ch 371, Harrison 19th p 2598- 99 Ref: Harrison's 18th Ch. 371, Harrison 19th p 2597 - 98
111. Anosmia is early clinical feature of ? 118. Reversible dementia is seen in ( NBE Pattern 2014/ 15)

a. Alzheimer b. Parkinson’s disease


-
( NEET Pattern 2015 16 ) a. Wilson b. Huntington disease
c. Alzhiemer s' d. Myxedema
c. Huntington’s chorea d. All of the above
Ref Harrison’s 18th Ch. 371, Harrison 19 th pi 71
Ref : Ch 12, table 12.1, Adam and Victor Principles of
neurology 9th edn, 119. Retrograde amnesia is seen with:
( NBE Pattern 2014/ 15)
112. All are seen in Alzheimer s disease except:

a. Post traumatic head injury
( AlIMS Nov. 2011 ) b. Temporal lobe epilepsy

Neurolgy
a. Aphasia b. Acalculia c. Drug addiction
c. Apraxia d. Agnosia d. Lennox gastaut syndrome
Ref H 18th. P3305, pg 1177, Harrison 19 th p 2598- 99 Ref: Harrison's 18th ch. 28,
113. Neurofibrillary tangles are seen in case of Alzheim ¬
er’s disease. Which of the following areas of brain is 120. Which of the following is predominantly involved in
resistant to neurofibrillary tangle formation? Alzheimer’s dementia: ( ARMS May 08)
(AIIMS Nov 2012 ) a . Frontal cortex
a. Area of visual association b. Temporo - Parietal cortex
b. Cuneal gyrus of area VI c. Fronto- Parietal cortex
c. Entorhinal cortex d. Fronto-Temporal cortex
d. Lateral geniculate nucleus Ref : Robbins 7th/ 138, Harrison 19 th p 2599
Ref: Harrison's; 18/ e, pg 3305, Harrison 19th p 2598 121. A chromosomal anomaly associated with Alzheimer's
114. All of the following are true about Alzheimer’s disease dementia is: (Al 2001 )
except: ( ARMS Nov 2012) a. Trisomy 18 b. Patau syndrome.
a. Number of Senile plaques correlate with age c. Trisomy 21 d . Turner’s syndrome
b. Underlying tau proteins suggest neurodegeneration . Ref : Harrison 18th/ p 3307, Harrison 19th p 2600 -01
c. Number of neurofibrillary tangles is associated with
122. Which of the following is not seen in early onset Al ¬
the severity of dementia zheimer’s Disease: (ARMS Nov 2011 )
d. Extracellular inclusions can be found in the absence a. Aphasia
of intracellular inclusions b. Apraxia
Ref: Adams & Victor Principles of Neurology 8th Ch. 39 c. Acalculia
d. Agnosia Ref : Harrison 19th p 2598- 99
115 . Regarding Alzheimers disease(AD), Small vessel dis¬
ease (SVD) and Cerebral amyloid angiopathy (CAA) 123. The following is not a feature of Alzheimer's disease:
which of the following is false? (ARMS Nov 2011 ) a. Neurofibrillary tangles ( AI 2004 )
a. SVD is not related to AD b. Senile (neuritic) plaques
b. CAA can be seen in patients with AD c. Amyloid Angiopathy
c. SVD is related to AD d. Lewy bodies
d. CAA is associated with SVD Ref .- Harrison 19 th p 2600
Ref Harrison's 18th P-3309, 3296, 3276;

Ans. 109. a. Tau protein 110. b. Ventricular shrinkage 111. d. All of the above 112. b. Acalculia
113. d. Lateral genicul... 114. d. Extracellular... 115. c. SVD is related to AD 116. a. Homocysteine
117. b. Temporal and... 118. d. Myxedema 119. b. Temporal lobe... 120. b. Temporo-Parietal cortex
121. c. Trisomy 21 122. c. Acalculia 123. d. Lewy bodies
th
Marwah's Internal Medicine MCQs ( Based on Harrison's 19 )

Parkinsonism 131. Wheel chair sign is seen in:


a. Parkinsonism ( NEET/ DNB Pattern Questions)
b. Duchenne muscular dystrophy
124. Transcranial Magnetic Stimulation of which part of the c. Becker’s dystrophy
brain has been shown to reduce frequency of symptoms d . Thomsen disease Ref: Harrison 18th edn, ch. 372
( AI 2012)
' '

in Parkinsonism:
a. Striatus 132. Levo-dopa is contraindicated in: ( NBEPattern 2014/ 15)
b. Globus pallidus externus a. Malignant melanoma
c. Subthalamic nucleus b. Multiple system atrophy
d. Putamen c. Shy dragger syndrome
Ref: Harrison’s 18/ e p3325, Harrison 19th p 2633 d . Olivopontocerebellar atrophy
Ref: 250 Cases in Clinical Medicine, RR Baliga,
125. Element causing Parkinsonism. ( NBEPattern 2014/ 15 )
a. Manganese b. Copper 133. In Parkinsonism what is not present: (AIIMS May 94)
c. Iron d. Lead a. Tremors at rest b. Past pointing
.
Ref: Harrison's 18th ed Table 372-2: Harrison 19 th p 2610 c. Akinesia d . Rigidity
Ref : H 18th/ p 3317, 3318, Harrison 19th p Table 449 - 1, p 2609
126. Which of the following disease causes loss of cortical
neurons? 134. An elderly man presents with features of dementia,
a. Spino-cerebellar ataxia ( AIIMS Nov. 2012 ) ataxia, difficulty in downward gaze and a history of
b. Progressive Supranuclear gaze palsy frequent falls. Likely diagnosis is:
c. Cortico-basilar degeneration (AI 2001 ) (AIIMS May 01 )

Neurolgy
d. Multiple system atrophy a. Parkinson’s disease
Ref: Harrison’s, 18th pgno.3341., Harrison 19th p 2605 b. Progressive supranuclear gaze palsy
c. Alzheimers disease
127. In a patient of Parkinson’s disease with intractable
d. None of the above
tremors, the most p Referred site of DBS (Deep Brain
Stimulation) is: (AIIMS Nov. 2012 ) Ref : Harrison 2559; 18th/ p 3311, 3312, Harrison 19th p 2612
a. Globus Pallidus externa 135. Extrapyramidal symptoms are seen in all EXCEPT:
b. Putamen a. Paralysis agitans ( AIIMS Nov 2000 )
c. Subthalamic nuclei b. Carbon monoxide poisoning
d. Substantia nigra c. Cerebrovascular accident
Ref: Harrison's; 18/ e pg 372, Harrison 19th p 2616 d . Multiple sclerosis
128. A 50 year old lady presents with slowness of move ¬ Ref : Harrison 18th/ p 3318, Harrison 19 th p 2662 -63
ment for 2 years, rigidity and vertical square wave
jerks. What is the most probable diagnosis? Infections of CNS
( AIIMS Nov 2012 )
a. Parkinson’s disease 136. Infection with the following may cause convulsions:
b. Lewy body dementia ( Manipal)
c. Progressive supranuclear paralysis a. Naegleria fowled b. Entamoeba histolytica
d. Multiple system atrophy c. Trachornonas d. Loa loa
Ref: Harrison’s 18th P-3319, Harrison 19th p 2612 Ref: Harrison 19 th p 1367
129. Coarse tremors in tongue are seen in all except: 137. A young male develops fever, followed by headache,
a. Parkinsonism ( AIIMS Nov 2013) confusional state, focal seizures and a right hemipare -
b. Alcohol sis. The MRI performed shows bilateral frontotempo ¬
c. Thyrotoxicosis d. General paresis ral hyperintense lesion. The most likely diagnosis is:
130. Iron deposition in basal ganglia is seen in: ( Manipal )
a. Halloverden Spatz disease ( NBE Pattern 2014/ 15) a. Acute pyogenic meningitis
b. Hemochromatosis b. Herpes simplex encephalitis
c. Hemosiderosis c. Neurocysticercosis
d. All of above d. Carcinomatous meningitis
Ref: Harrison’ s 18th ch:372, Harrison 19th p 2611, 2626

Ans. 124. c. Subthalamic nuc... 125. a. Manganese 126. c. Cortico-basilar deg... 127. c. Subthalamic nuclei
128. c. Progressive... 129. c. Thyrotoxicosis 130. a. Halloverden Spatz... 131. a . Parkinsonism
132. a. Malignant mela... 133. b. Past pointing 134. b. Progressive supra ... 135. d. Multiple sclerosis
136. a. Naegleria fowleri 137. b. Herpes simplex encephalitis
Neurology

138. Burst supression pattern on EEG is typically seen in: 146. A patient presented with headache, vomiting. CT scan
a. Anoxic encephalopathy ( DNB 2012) showed presence of brain abscess. Culture and anti ¬
b. Absence seizures
biotic sensitivity result is awaited. Which antibiotic
c. SSPE

B
can be given empirically in this patient till getting the
d. Herpes simplex encephalitis
report? ( NBE Pattern 2014/ 15)
Ref: Harrison 19th pi 735 a. Penicillin G
139. Neurological complications of meningitis include all b. Ceftriaxone + Metronidazole
of the following, except: (AIIMS Nov 02) c. Amikacin
a. Seizures d. Gentamicin
b. Increased intracranial pressure. Ref: H ’s 18th P-3430, 1026, Harrison 19 th p 1097 - 98
c. Cerebral hamartoma. 147. A patient with Tubercular meningitis was taking ATT
d. Subdural effusions. regularly. At end of 1 month of regular intake of drugs
Ref: Harrison 15/ e p2464, 2481; Harrison 19th p 889 deterioration in sensorium is noted in condition of
140. Which of the following statements about Prions is true: the patient. Which of the following investigations is
a. They are infections proteins not required on emergency evaluation?
( A12008)
b. They are made upof bacteria and virus a. MRI (AIIMS Nov 2012)
c. They have rich nuclear material b. NCCT
d. They can be cultured in cell free media c. CSF examination
d. Liver function tests
Ref: Harrison's 18/ e p3441, Harrison 19 th p 453e-2
Ref: OP Ghai 7th ed
141. Aseptic meningitis caused by? ( NEET Pattern 2015 16 ) -

Neurolgy
148. Which of following is correct about prions?
a. Indomethacin b. Ibuprofen (AIIMS Nov 2010)
a. Long incubation period
c. Aspirin d. Icatibant
b. Destroyed by autoclaving at 121°C
Ref : Dubois hand book of Lupus , p 1150, 7th edn, c. Nucleic acid present
142. Herpes zoster infection can lead to? d. Immunogenic
( NEET Pattern 2015 16 ) - Ref: Harrison's 18th ch. 383, Harrison 19 th p 453e- l
a. Frontal lobe infarction 149. A Bengali woman had fever for 2 days and has irrele ¬
b. Parietal lobe infarction vant talking. She is complaining of weakness in arms.
c. Temporal lobe infarction Which of the following drugs can be given in ER to her,
d. Occipital neuralgia pending investigations? ( AIIMS May 2013)
Ref : Essentials of Pain Medicine,3rd edn, p 290 a. Mannitol
143. All are true about Ramsay hunt syndrome except? b. Acyclovir
a. Facial palsy ( NEET Pattern 2015-16 ) c. Penicillin
b. Vesicles in meatus d. Amphotericin B
c. Vertigo Ref: Harrison’s, ch. 381, Harrison 19th p 896
d. Palatal myoclonus 150. In HIV infection all are affected except:
Ref : Harrison 18th edn, Ch 376.2, Harrison 19 th p 2648 (AIIMS Nov 2012 )
a. Cingulate gyrus
144. All are true about CNS leukemia except:
( NBE Pattern 2014/ 15) b. Caudate nucleus
a. CNS irradiation is given
methotrexate is given c. Globus pallidus
b. Intrathecal
d. Cerebral white matter
c. Seen with acute myeloid leukemia
d. Single blast in CSF is sufficient for diagnosis Ref: Harrison’s 18th ch. 189, Harrison 19th p 1264 -65
Ref: OP. Ghai, 583, 7th ed, OP Ghai 8th 602-603 151. Which is not correct about Bell’s Palsy?
( AIIMS Nov 2012)
145. Fever blisters can occur due to:( NBE Pattern 2014/15)
a. Steroid is mandatory
a. HHV6 b. Unilateral facial weakness
b. Varicella c. Urgent surgical decompression
c. Primary HSV 1 infection d. HSV 2 is not the cause
d. Reactivation of HSV-1 Ref: Harrison's ch. 376., Harrison 19th p 2647
Ref: Harrison's 18th P-1454, Harrison 19th p 1177

Ans 138 a Anoxic encephal .. . 139. c . Cerebral hamartoma 140 . a . They are infections... 141 . b. Ibuprofen
142 d Occipital neu .. . 143 . d . Palatal myoclonus 144. c . Seen with acute .. . 145 . d . Reactivation of HSV-1

^
"

146 b Ceftriaxone . .. 147 . c . CSF examination 148. a . Long incubation ... 149 . b. Acyclovir
150. a . Cingulate gyrus 151 . c . Urgent surgical decompression
th
Marwah's Internal Medicine MCQs ( Based on Harrison's 19 )

.
152 Locomotor ataxia, a late manifestation of syphilis due 158. Periodic lateralized slowing is seen in:
to parenchymatous involvement of the spinal cord is a. Herpes simplex encephalitis

a
called: ( APPG 2014) b. Mesial temporal sclerosis ( NBE Pattern 2014/ 15)
a. General paralysis of insane c. SSPE
b. Tabes dorsalis d. Variant Creutzfelt Jacob disease
c. Meningovascular syphilis Ref Harrison's 18th Ch:381, Harrison 19th p 895
d. Syphilitic amyotrophy 159 Treatment for Neurocysticerosis in children:
.
Ref: Harrison’s, ch 169, Harrison 19th p 1136, 1138 ( NBE Pattern 2014/15)
a. Albendazole 15 mg / /
kg day for 1 week
153. A 30 year old patient is having high grade fever with
altered sensorium. On third day the patient develops b. Albendazole 15 mg / kg / day for 2 week
seizures and has Nuchal rigidity. The CSF examination c. Albendazole 15 mg / kg / day for 3 week
of the patient shows presence of 300 cells / cu.mm, d. Albendazole 15 mg / kg / day for 4 week
protein 70 mg%, glucose 54mg% (BLOOD GLUCOSE = Ref: OP. Ghai, 725, 7th ed
95mg%) with polymorph 65% and the rest being
160. Disease spread by cannibalism:
lymphocytes. The most probable diagnosis of the pa ¬ ( NBE Pattern 2014/15)
tient is: ( AllMS Nov 2012) b.
a. Nipah virus Nocardiosis
a. Viral meningitis
c. Kuru
b. Tubercular meningitis
d. Madras motor neurone disease
c. Pyogenic meningitis
d. Cerebral malaria. Ref Harrison's 18th Ch: 383, Harrison 19th p 455e

Neurolgy
Ref:Harrison’s 18th , Ch. 381 Harrison 19th p 888 161. CSF glucose level is: ( NBE Pattern 2014/ 15)
154. Which of the following enzymes can be detected in a. Half the plasma glucose
CSF: ( AllMS Nov 2012) b. 2 / 3 plasma glucose
c. 1/3 plasma glucose
a. GGT and ALT
d. Same as plasma glucose
b. CK- MBandALP
Ref: Harrison's 18th ch: 381, Harrison 19th p 888
c. LDH and CK
d. Deaminase and superoxide dismutase 162. A person whose CSF grows Streptococcus pneumoniae
i
Ref: TB of Medical Biochemistry by Chatterjee: 8th pg. 730 on culture is most likely to show? (AlPG 2011 )
.
155 Eosinophilic meningitis is seen with all except: a. Pleocytosis, high protein, reduced sugar
a. Coccidiomycosis ( NBE Pattern 2014/15) b. Pleocytosis, high protein, high sugar
b. Cryptococcal meningitis c. Lymphocytosis, high protein, normal sugar
c. Lepto-meningeal metastasis d. Lymphocytosis, low protein, normal sugar
d. Helminthic infections Ref Harrison's 18/ e, pg 3414 Harrison 19th p 888
Ref: Ch. 602.3 Eosinophilic meningitis Nelson 18th 163. In Pneumococcal meningitis the empirical treatment
156. Ipsilateral 3rd nerve palsy with ipsilateral hemiplegia given is: ( NBE Pattern 2014/ 15)
is seen with: ( NBE Pattern 2014/ 15) a. Penicillin G b. Doxycycline
a. Uncal herniation c. Tetracycline
d. Vancomycin + ceftriaxone
b. Cingulate herniation
c. Millard Gubler syndrome -
Ref: Harrison’ s 18th P 1157, 2158
d. Weber syndrome 164. Which of the following is the classical CSF finding
Ref: Harrison’s 18th ch:274, Harrison 19th pi 772 seen in TBM? (4/ 2007)
a. Increased protein, decreased sugar, increased
157. Not useful for diagnosis of S.S.P.E
lymphocytes
( NBE Pattern 2014/ 15) b. Increased protein, sugar and lymphocytes
a. EEG c. Decreased protein, increased sugar and lympho ¬
b. Brain biopsy
cytes
c. Antibody to measles in C.S.F d. Increased sugar, protein and neutrophils
d. Antibody to measles in blood
Ref : Harrison 18th/ p 3436, Harrison 19th p 888
Ref: Nelson 18th ed, Ch 243

Ans. 152. b. Tabes dorsalis 153. c. Pyogenic meningitis 154. c. LDH and CK 155. b. Cryptococcal meningitis
156. a. Uncal herniation 157. d. Antibody to measles.. 158. a. Herpes simplex... 159. d. Albendazole 15mg / kg...
160. c. Kuru 161. b. 2/3 plasma glucose 162. a. Pleocytosis, high . 163. d. Vancomycin + ceftriaxone
164. a. Increased pro...
Neurology

165 . Characteristic finding in CT in a TB case is : 172. The drug of choice in Herpes Simplex Encephalitis is:
a. Exudate seen in basal cistern ( AI 2001 ) ( AI 1994)
b. Hydrocephalus is non communicating a. Acyclovir b. Zidovudine
c. Calcification commonly seen in umbellium c. Amantadine d. Vidarabine
d. Ventriculitis is a common finding
Ref : Harrison 18th/ p 3425, 3425, Harrison 19th p 896
Ref: Cranial MR1 & CT, Lee Rao & Zimmer Man - 4th/ 479
173. Which of the following is the most common location
166 . Pneumococcal meningitis is associated with the fol ¬ of intracranial neurocysticercosis : (AIIMS Nov. 05)
lowing CSF findings: (Al 2012) a. Brain parenchyma b. Subarachnoid space
a. Pleocytosis with low protein and low sugar
c. Spinal cord d . Orbit
b. Pleocytosis with high protein and low sugar
c. Lymphocytosis with low protein and low sugar Ref : H 18th/ p 3431, Harrison 19th p 1431, Table 260 1 -
d. Lymphocytosis with high protein and low sugar 174. 1Which of the following statements about Prions is
Ref : Harrison 18th/ 3414, Harrison 19th p 888 true: ( AI 2008)
167 . The drug of choice in Cryptococcal Meningitis is: a. They are infectious proteins
a. Pentostatin b. They are made up of bacteria and virus
(AI 1995)
b. Amphotericin B c. They have rich nuclear material
c. Clotrimazole d. They can be cultured in cell free media
d. Zidovudine Ref : Ananthnarayanan 7th/ 567; Jawetz 24th/ 581.
Ref : Harrison 18th/ p 1650, Harrison 19th p 899 175. Which one of the following is not a Prion associated
disease: (AI 2005)
168. Subdural empyema is most commonly caused by:

Neurolgy
a. H influenza (AI 2000) a. Scrapie
b. Staphylococcus aureus b. Kuru
c .Streptococcus pneumoniae c. Creutzfeldt-Jakob disease
d. E. Coli d. Alzheimer’s disease
Ref : Harrison 18th/ p 3432, Harrison 19th p 903 Ref : Harrison 18th/ p 3442 (Table: 383.2
169 . Which of the following viruses is not a common cause 176. All of the following statements about Creutzfeldt-
of viral encephalitis? (AI 2004) Jakob disease are true, except: ( AIIMS May 04 )
a. Herpes simplex virus type 2 a. It is a neurodegenerative disease
b. Japanese encephalitis virus b. It is caused by infectious proteins
c. Nipah virus c. Myoclonus is rarely seen
d. Cytomegalovirus d. Brain biopsy is specific for diagnosis
Ref : Harrison 18th/ p 3419, 3420, Harrison 19th p 890 Ref: Harrison 18th/ p 3444, 3445, 3447
170. Which of the following is the most common cause of Neuropathy & Myopathy
meningoencephalitis in children: (AI 2010 )
a. Mumps
177. Duchenne Muscular Dystrophy is a disease of:
b. Arbovirus
a. Neuromuscular junction ( Manipal )
c. HSV
Ref : Nelson' s 18th/ 2521 b. Sarcolemmal proteins
d. Enterovirus
c. Defective gene for dystrophin
171. A young male develops fever, followed by headache, d. Disuse atrophy due to muscle weakness
confusional state, focal seizures and right hemiparesis. Ref: Harrison 19th p 462e 5 -
The MRI performed shows bilateral frontotemporal
hyper intense lesion . The most likely diagnosis is: 178. All of the following are true about Weber’s syndrome,
( AI 2004) except: ( DNB 09, 12)
a. Acute pyogenic meningitis a. Ipsilateral occulomotor nerve palsy
b. Herpes simplex encephalitis b. Diplopia
c. Neurocysticercosis c. Contralateral hemiplagia
d. Carcinomatous meningitis d. Ipsilateral facial nerve palsy
Ref : Harrison 18th / p 3422, 3423, Harrison 19th p 893, 895 -
Ref: Harrison's 18/ e p3287, Harrison 19th p 441e 38f

Ans. 165. a. Exudate seen in... 166. b. Pleocytosis with high... 167. b. Amphotericin B 168. c. Streptococcus pneumoniae
169. a. Herpes simplex ... 170. d. Enterovirus 171. b. Herpes simplex... 172. a. Acyclovir
173. a. Brain parenchyma 174. a. They are infectious... 175. d. Alzheimer ’s disease 176. c. Myoclonus is rarely seen
177. c. Defective gene... 178. d. Ipsilateral facial nerve palsy
's 19th )
Marwah's Internal Medicine MCQs ( Based on Harrison

(AI 2007) 188. Protein defective in congenital muscular dystrophy:


179. Benedict’s syndrome, all are true except: ( NBE Pattern 2014/ 15)
a. Merosin
a. Contralateral tremor b. Dystrophin
b. 3rd nerve palsy
c. Laminin
c. Involvement of the penetrating branch of the basilar Ref: OP. Ghai, 566, 7th ed

I
d. Sarcoglycan
artery
189. A 14- year-old girl presents with quadriparesis facial
,
d. Lesion at the level of the pons h
palsy, winging of scapula and ptosis. There is / o sim ¬
Ref: Harrison’s 18/ e p238, Harrison 19 th p 462 less severe . Her
ilar illness in father and brother but
Millard Gubler syndrome includes the following ex¬ CPK level is also raised (500 IU / L) . She is probably suf ¬
cept: (AI 2007) fering from? ( AIPG 2011 )
a. 5th nerve palsy a. Emery- Dreifuss muscular dystrophy
b. 6th nerve palsy b. Becker muscular dystrophy
c. 7th nerve palsy c. Limb girdle dystrophy
d. Contralateral hemiparesis d. Scapulofaciohumeral dystrophy
Ref: Harrison’ s 18/ e p239, Harrison 19th p 209 Ref H ; 18/ e, pg 3498, Harrison 19 th p 462e, 2707
181. Pontine Stroke is associated with all except: (AI 2007 ) 190. Conduction velocity of nerve is NOT affected in which
a. Bilateral pin point pupil of the following? ( NBE Pattern 2014/ 15)
b. Pyrexia a. Leprosy
c. Vagal palsy b. Motor neuron disease
d. Quadriparesis c. Hereditary neuropathy
Ref: Harrison's 18/ e p3290, Harrison 19th p 2579f d. A.I.D.P
182. Beevor's sign is seen in: (AIIMS May 09 ) Ref H 18/ e, pg 3347, Harrison 19th p 2632 33 -
a. Abdominal muscle b. Facial muscle 191. Small fibre neuropathy is seen in? (AIPG 2010)
c. Respiratory muscle d. Hand muscle a. HIV
Ref: Harrison’ s 18/ e p3367, Harrison 19th p 2651 b. Vitamin B12 deficiency
c. AIP
183. Lesions of the lateral cerebellum cause all of the fol ¬ d. Cisplatin toxicity
lowing, except: ( AI 2010)
Ref Harrison 19th p 2684 -85
a. Incoordination b. Intention tremor
c. Resting tremor d. Ataxia 192. Which of the following is Not true about polymyositis?
Ref: Harrison’s 18/ e pl 93, Harrison 19th p 2672 a. Limb girdle weakness (AIIMS Nov 2012)
b. Ophthalmoplegia
184. Lesion in which of the following structure leads to c. Para -neoplastic syndrome
Kluver- Bucy syndrome: (AIIMS May 04 ) d. Spontaneous discharge in EMG.
a. Amygdala

1
b. Hippocampus
Ref Harrison’s 18th Ch. 388, Harrison 19 th p 2194
c. Hypothalamus d. Temporal lobe
Ref: Guyton 10/ e p687 193. A 10 year old child with large leg muscles and low
levels of CPK- MM is suggestive of diagnosis:
l 85' Dystrophic gene mutation leads to: (AIIMS May 03) ( AIIMS Nov 2012)
a. Myasthenia gravis
if b. Motor neuron disease
c. Poliomyelitis
d. Duchenne muscular dystrophy
a. Duchenne muscular dystrophy
b. Beckers muscular dystrophy
c. Congenital myopathy
d. Hereditary sensori-motor neuropathy
Ref: Harrison’s 18/ e p3492, Harrison 19th p 462e -5 Ref Harrison 18th edn, Ch. 387: Harrison 19th p 595
186. High Steppage Gait is seen in? ( NEET Pattern 2015-16 ) 194. A 60 year old female is having proximal muscle weak¬
a. Foot drop b. Frontal lobe stroke ness with increased serum creatinine kinase. The
c. Tabes dorsalis d. Leprosy probable diagnosis is: ( NBE Pattern 2014/ 15)
Ref : Harrison's 18th edn, Ch 377, Harrison 19th p 1136 a. Polymyositis
187. Arsenic poisoning causes? ( NEETPattern 2015-16 ) b. Dermatomyositis
a. Polyneuritis b. Mononeuritis multiplex c. Inclusion body myositis
c. Radiculopathy d. Myelopathy d. Limb girdle muscle dystrophy
Ref : Harrison's 18th edn, Ch 384, Harrison 19 th p 2689 Ref: Harrison’s 18th ch:388, Harrison 19th p 2194 - 96

Ans. 179. d. Lesion at the... 180. a. 5th nerve palsy 181. c. Vagal palsy 182. a . Abdominal muscle
183. c. Resting tremor 184. a. Amygdala 185. d. Duchenne muscular... 186. c. Tabes dorsalis
*§|
j 187. a . Polyneuritis 188. a. Merosin 189. d. Scapulofaciohumeral . . .190. b. Motor neuron disease
191. a . HIV 192. b. Ophthalmoplegia 193. a. Duchenne muscular... 194. a. Polymyositis
Neurology

203. Dystrophic gene mutation leads to : ( ARMS May 03 )


195. Palpable nerves are seen in: ( NBE Pattern 2014/ 15)
a. Charcot Marie Tooth disease a. Myasthenia gravis
b. Diabetes mellitus b. Motor neuron disease
c. Neurosyphilis c. Poliomyelitis
d. Myotonic dystrophy Ref: Harrison's 18th ch:384 d. Duchenne’s muscular dystrophy
Ref : Harrison 18th/ p 3492, Harrison 19th p 462e I
196. Which of the following predominant motor involve ¬
ment? ( NBE Pattern 2014/ 15) 204. In Duchenne’ s muscular dystrophy, which muscle
is I
a. Lead poisoning b. Arsenic poisoning not involved : ( ARMS Feb 97 ) %
c. Kashin Beck disease d. Keshan disease a. Gastrocnemius
Ref: H 18th Ch:384, Harrison 19th p 2688 b. Vastus medialis
c. Brachioradialis
197 . Limb girdle muscle dystrophy includes all of the fol ¬ d. Infraspinatus
lowing groups of disorder except: (AIPG 2011) Ref : Ghai 6th/ 548, Harrison 19 th p 462 e
a. Sarcoglycanopathy b. Dystrophinopathy
c. Dysferinopathy d. Calpainopathy 205. All are Congenital Myopathies, except:
a. Central- core Myopathy ( ARMS May 2011 )
Ref: H 18th Ch.387.Pg. no.3495„ Harrison 19 th p 462 e
b. Nemaline Myopathy
198. Fasciculation is seen in: ( PGIDec 03) c. Z band Myopathy
a. UMN type of lesion b. LMN type of lesions d. Centro-nuclear Myopathy Ref : Robbins 7th/ 1340 I
c. Myoneural junction d. Peripheral neuropathy 206. Which of the following antibodies is specific for myo- j

Neurolgy
Ref : Harrison 18th/ p 182, Harrison 19th p 438e sitis: ( ARMS Nov 08) I
199. All are true about peripheral - neuropathy except : a. Anti - Jo-1
a. Glove and stocking anaesthesia (AI 1999 ) b. Anti - Scl-70
b. Proximal muscle weakness c. Anti - Sm
c. Nerve-conduction deficit d. Anti- ku Ref : Harrison 19 th p 2196
d. Decreased Reflexes 207 . Which is NOT a feature of polymyositis?
Ref : Harrisons 18th/ p 3448, Harrison 19 th p 2675 a.Pharyngeal muscle involvement (AIIMS- 97) j
b.Gottron’s rash
200. All of the following are feature of autonomic neuropa ¬
( A1IMS May 08 )
c.Proximal muscle involvement
thy, Except:
d.Pain in limbs
a. Resting Tachycardia
Ref : Harrison, 17th/ 2696, Harrison 19 th p 2195
b. Silent Myocardial Infarction
c. Orthostatic Hypotension 208. All of the following are feature of dermatomyositis,
d. Bradycardia Except: (ARMS Nov 09 )
a. Salmon Patch
Ref : Morgan’s 4th/ 804, 805; 'Stoelting’sAnesthesia’ 5th/ 375 b. Gottron’s patch
201. All of the following are predominant motor neuropa ¬ c. Mechanic finger
thy except: (AIIMS May' 06 ) d. Periungual telengiectasias
a. Acute inflammatory demylinating poly radiculo- Ref : Harrisons 17th/ 2696, Harrison 19 th p 2195
neuropathy
b. Porphyric neuropathy CVA / Stroke
c. Lead intoxication
d. Arsenic intoxication Ref : Harrison 19 th p 2688 209. A patient presents with sudden ataxia, headache, vom¬
iting and drowsiness. What is the diagnosis?
202. Pure motor paralysis is seen in : (AI 2000)
a. Polio (JIPMER 2014 )
a. Acute cerebellar hemorrhage
b. Guillain Barre syndrome
b. Acute subarachnoid hemorrhage
c. Diabetes mellitus
c. Transient ischemic attack
d. Sub-Acute Combined Degeneration
d. Herpes simplex encephalitis
Ref ; Harrison 18th/ p 3347, Harrison 19th p 1290- 91
Ref : Harrisons 18th Ch. 370

Ans. 195. a. Charcot Marie... 196. a. Lead poisoning 197. b. Dystrophinopathy 198. b. LMN type of lesions
199. b. Proximal mus... 200. d. Bradycardia 201. d. Arsenic intoxication 202. a. Polio
203. d. Duchenne’s... 204. b. d Vastus medialis... 205. c. Z band Myopathy 206. a. Anti - Jo-1
207. b. Gottron’s rash 208. a. Salmon Patch 209. a. Acute cerebellar hemorrhage
s 19 )
Marwah's Internal Medicine MCQs ( Based on Harrison

severe headache
216. A hypertensive patient presents with
210. A 67 year- old male patient presents with sudden stiffness but no Focal
and vomiting. He has got neck
onset diplopia, right sided facial nerve palsy involving probable diag ¬
Neurological Deficit. What is the most
both upper and lower part of the face and left sided
hemiparesis. Most probable site of the lesion: nosis?
(JIPMER 2014) a. Intra -cerebral bleed ( NBE Pattern 2014/ 15)
b. Subarachnoid haemorrhage
fjM a. Below midbrain b. Right pons
HB C. Right mid brain d. Left lateral medulla c. Meningitis
Hi e. Left frontal d. Meningo-encephalitis
Ref.: Davidson, 20th Box 20.49, Harrison 19th p 157, 1152
-
Ref: Harrison' s 18th P 2261, Harrison 19th pi 785

I 211. Right internal carotid artery stenosis leads to all 217 . Aneurysm of Posterior communicating artery will
except: (J 1PMER 2014 ) compress which cranial nerve? ( NBE Pattern 2014/ 15)
a. Dysphasia a. Trochlear
b. C / L hemiparesis b. Trigeminal
c. C/ L hemisensory loss c. Optic
d. Drop attacks d. Occulomotor
Ref : H 18th edn, Harrison 19 th p 2571 -
Ref: Harrison's 18th P 2261, Harrison 19th pi 785

I 212. Which test is not useful in a patient with history of


syncopal attack? ( AIIMS Nov 06 ) 218. A 65 yr old male fainted in the bathroom. His relatives
informed that his stool was black in colour. He is

Neurolgy
a. Electrophysiological testing
b. Tilt table testing hypertensive and has got history of coronary artery
c. PET Scan disease. He was on long term treatment with atenolol ,
d. Holter monitoring aspirin and sorbitrate. His BP = 80 / 50 and HR 150 /
min. Most probable diagnosis: (ARMS May 2011 )
Ref: Harrison's 18/ e pi 77, Harrison 19th p 147
a. Myocardial infarction with cardiogenic shock
213. A 45 years old hypertensive male presented with sud ¬ b. Cerebrovascular accident
den onset severe headache, vomiting and neck stiff ¬ c. Gastric ulcer and bleed
ness. On examination he didn’ t have any focal neuro¬ d. Septic shock
logical deficit. His CT scan showed blood in the Sylvain Ref: Harrison’s 18th P-2444, Harrison 19 th pi 778, 1918
fissure. The probable diagnosis is: (AIIMS May 03)
a. Meningitis b. Ruptured aneurysm 219. Gait apraxia is seen in: ( ARMS Nov 2011 )
c. Hypertensive bleed d. Stroke a. ACA
Ref: H 18/ e p2261, 2262 Harrison 19 th p 1637 b. MCA
c. PCA
214. In Balint syndrome all are seen except? d. Posterior choroidal artery
( NEET Pattern 2015-16 )
Ref: Harrison's 18th Ch. 24, Harrison 19 th p 163
a. Opsoclonus b. Optic ataxia
c. Simultagnosia d. Ocular apraxia 220. A 16 year old girl has recurrent abdominal pain. On
Ref : Harrison 18th Ch 370: Harrison 19 th p 2575 work up USG abdomen and LFT was normal . She has
215. A patient presents with subarachnoid haemorrhage. now developed sudden onset bilateral loss of vision ,
NCCT reveals blood in the fourth ventricle. The bleed ¬ pupils were bilaterally responding to light, with CT
ing is most likely to occur from an aneurysm of which head and MRI head being normal . Diagnosis of the
of the following arteries? (NBE Pattern 2014/ 15) patient is: ( ARMS Nov 2013)
a. Anterior communicating artery a. Bilateral optic neuritis
b. Basilar top region b. PCA infarct
c. AICA c. Malingering
d. PICA d. Devic’s disease
Ref: Handbook of Neurosurgery, Greenberg, Pg. 2601 Ref: Harrison 19th p 2656 and p 196, 202

Ans. 210. b. Right pons 211. d. Drop attacks 212. c. PET Scan
214. a. Opsoclonus 213. b. Ruptured aneurysm
215. d. PICA 216. b. Subarachnoid haem... 217. d. Occulomotor
218. c. Gastric ulcer... 219. a . ACA 220. c. Malingering
Neurology

221. A 26 year old healthy female got pregnant for the first 226. Cranial nerves involved in Millard Gubler syndrome
time and a LSCS was done for fetal distress. Mild hy¬ are: ( NBE Pattern 2014/ 15)
pertension was present during pregnancy. 2 days after a. 3rd and 4th nerve
b. 4th and 5th nerve

B
delivery she had headache and seizures. CT shows 2X3
cm Para -sagittal Lesion. Urinalysis shows no protein ¬ c. 6th and 7th nerve
uria. Diagnosis is: (AIIMS Nov 2013) d. 7th and 8th nerve
a. Eclampsia Ref Harrison’ s 18th ch. 28, Harrison 19th p 209
b. Hypertensive l.C.H. 227. Which of the following is protective in stroke?
c. Saggital sinus thrombosis a. Hypocapnia ( NBE Pattern 2014/ 15)
d. Pituitary apoplexy Ref: William’ s 23rd pg 1169 b. Hypercapnia
c. Hyperglycaemia
222. A patient developed sudden severe headache 2 hours d. Hypocalcemia
ago and became unconscious. On regaining con ¬
sciousness he developed photophobia and neck rigid ¬
Ref: Harrison’s 18th Ch:370, Oxford Textbook of stroke, p 130
ity. What is the next line of management? 228. A diabetes mellitus patient with BP of 220 /130 mm
(AIIMS Nov 2013) Hg is brought to the casualty in coma. CT scan shows
a. NCCT a large infarct. What is the target BP of this patient to
b. IV antibiotics initiate thrombolysis? ( NBE Pattern 2014/ 15)
c. CSF a. Less than 200/130
d. MRI b. Less than 180 /110
Ref CMDT 2013 pg. 984-985, Harrison 19th p 1785 c. Less than 160 /100

Neurolgy
d. Less than 140 /90
223. A patient has developed confusion and keeps bumping
into objects. He can speak fluently and on examination Ref J.N.C 8 guidelines, Harrison 19th p 1627
patient has inability to differentiate between fingers, 229. Hypertensive Patient presents with one day history of I
cannot write. On MRI T 2 images show cortical and sub headache, Nausea, vomiting and difficulty in walking.
cortical lesions: ( AIIMS May 2013) Diagnosis is? ( NBE Pattern 2014/ 15) |
a. Gerstman syndrome a. Extradural haemorrhage
b. Anton syndrome b. Intra- parenchymal haemorrhage
c. Millard Gubler syndrome -
c. Sub dural haemorrhage
d. Locked in syndrome
Ref: Harrison’s 18th ch. 370, Harrison 19th p 179, 2573
-
d. Sub arachnoid haemorrhage
Ref: Harrison’s 18th Ch:370, Harrison 19th p 2582
224. A middle aged hypertensive male develops sudden 230. Cerebral infarct is earliest detected by:
onset unconsciousness, with nuchal rigidity. Rest of
a. Diffusion weighted MRI
the neurological examination is within normal limits.
( NBE Pattern 2014/15)
( AIIMS May 2013) b. P.E.T scan
Diagnosis is?
c. MRI scan
a. SAH
d. CT scan Ref: Harrison 19th p 2580
-
b. Intra parenchymal bleed
c. Extra -dural hemorrhage 231. What is incorrect about Charcot neurological triad:
d. Sub dural haemorrhage. a. Scanning speech ( NBE Pattern 2014/ 15)
Ref: Harrison's 18th ch. 275, Harrison 19th pi 785 b. Intentional tremor
c. Nystagmus
225. A third cranial nerve palsy associated with pupillary d. Rigidity
dilation, loss of ipsilateral light Reflex, focal pain be ¬
hind the eye may occur with? ( APPG 2014) Ref Oxford Illustrated companion to medicine, p 647
a. Aneurysm in the cavernous sinus 232. All are true about Broca's aphasia except:
b. Expanding supraclinoid carotid aneurysm a. Non fluent aphasia ( NBE Pattern 2014/ 15)
c. Expanding aneurysm at the junction of posterior b. Damage to posterior part of inferior frontal Gyrus
communicating and internal carotid artery c. Repetition is preserved
d. Expanding anterior cerebral artery aneurysm d. Syntax is preserved
Ref: Harrison 19th pi 785 Ref: Harrison’s 18th Ch:26, Harrison 19th pi 77t, 178

Ans. 221. c. Saggital sinus... 222. a. NCCT 223. a. Gerstman syndrome 224. a. SAH
225. c. Expanding ane... 226. c. 6th and 7th nerve 227. a. Hypocapnia 228. b. Less than 180/110
-
229. b. Intra parenchy... 230. a. Diffusion weighted MRI 231. d. Rigidity 232. d. Syntax is preserved
Marwah's Internal Medicine MCQs ( Based on Harrison s 19* )

233. Christmas tree appearance of urinary bladder is seen 241. Hemianopia, cortical blindness, amnesia and thalam ¬
in; ( NBE Pattern 2014/ 15) ic pain are associated with the occlusion of:
a. Neurogenic bladder ( NBE Pattern 2014/ 15)
b. Stress incontinence a. Anterior cerebral artery
c. Autonomous bladder b. Middle cerebral artery
d. Enuresis c. Posterior cerebral artery
Ref: Clinical Pediatric Nephrology, 2nd ed Pg 96 d. Basilar artery

^ 234. Death after Rupture of Berry Aneurysm is due to:


a. Re -bleeding
b. Cerebral ischemia
-
c. Intra ventricular hemorrhage
( NBE Pattern 2014/ 15)
Ref: Harrison’s, 18th Ch: 370, Harrison 19th p 2575
242. Ipsilateral 3rd nerve palsy with Contralateral hemi ¬
plegia is known as:
a. Millard Gubler syndrome
( NBE Pattern 2014/ 15)
d. Myocardial infarction
b. Weber's syndrome
Ref: Harrison’s 18th Ch:275, Harrison 19thpi 785
c. Foville syndrome
235. Patient complains of dysphagia for solids and liquids.
The level of lesion is at: d. Benedicts syndrome
( NBE Pattern 2014/ 15)
a. Cortical level b. Brainstem damage Ref: Harrison’s, 18th Ch: 370, 2575
c. Cranial nerve palsy d. Esophagus 243. Characteristic features of a lesion in the lateral part of
Ref: Harrison' s 18th ch. 38, Harrison 19th p 257 the medulla include all except: ( NBE Pattern 2014/ 15)
236. Duret haemorrhage is seen in: ( NBEPattern 2014/ 15) a. Ipsilateral Horner’s syndrome
a. Brain b. Adrenal gland

Neurolgy
b. Contralateral loss of proprioception to the body and
c. Lungs d. Liver limbs
Ref: Neuro- Imaging, Roy Riascos c. Nystagmus
237 . The most common cause of embolic stroke: d. Dysphagia
( NBE Pattern 2014/ 15) Ref: Harrison’ s, 18th ch: 370, Harrison 19 th p 2576
a. Non rheumatic atrial fibrillation
b. Carotid artery atherosclerosis 244. Most common site of cerebral infarction is in the
c. Paradoxical embolism territory of: ( NBE Pattern 2014/ 15)
d. LV aneurysm Ref: H 18th ch. 370, 18th ed, 2564 a. Anterior cerebral artery
238. Lacunar stroke true is: b. Middle cerebral artery
( NBE Pattern 2014/ 15)
a. Female on OCP c. Posterior cerebral artery
b. Male with HTN d. Posterior inferior cerebellar artery
c. Young male with AV malformation Ref: Harrison's 18th ch. 370, Harrison 19th p 2572
d. Young male with mycotic aneurysm
Ref: Harrison's 18th ch. 370, Harrison 19th p 2565 245. Pyrexia, pin point pupils and unconsciousness is
239. Thrombosis of the Superior branch of middle cerebral
characteristic of: ( NBE Pattern 2014/15)
a. Brain stem lesion
artery leads to: ( NBE Pattern 2014/ 15)
a. Motor aphasia b. Cerebellar lesion
b. Urinary retention c. Thalamic lesions
c. Bitemporal hemianopia d. Internal capsule lesions
d. Grasp Reflex Ref: Harrison's, 18th ch: 370, Harrison 19th p 1774
Ref: Harrison’s 18th ch: 370, Harrison 19th p 2573 246. Lateral medullary syndrome is due to thrombosis of:
240. Presence of hemiplegia with homonymous hemiano ¬
( NBE Pattern 2014/ 15)
pia with gaze to same side is seen with occlusion of a. Posterior inferior cerebellar artery
which brain vessel?
b. Anterior inferior cerebellar artery
a. Middle cerebral artery ( NBE Pattern 2014/15) c. Superior cerebellar artery
b. Basilar artery
c. Anterior cerebral artery d. Posterior communicating branch of middle cerebral
d. Internal cartoid artery artery
Ref: Harrison’s, 18th ch: 370, Harrison 19 th p 2573 Ref: Harrison's, 18th ch: 370, Harrison 19th p 2576

Ans. 233. a. Neurogenic bla... 234. b. Cerebral ischemia 235. d. Esophagus 236. a . Brain
237. a. Non rheumatic... 238. b. Male with HTN 239. a . Motor aphasia
241. c. Posterior cere... 240. a. Middle cerebral artery
242. b. Weber ’s syndrome 243. b. Contralateral loss...
245. a . Brain stem lesion 244. b. Middle cerebral artery
246. a. Posterior inferior...
Neurology

247. Lateral medullary syndrome is associated with: 254. Least common site for Berry Aneurysm:
a . Dissociative anesthesia ( NBE Pattern 2014/ 15) a. Vertebral artery ( NBE Pattern 2014/ 15)
b. Dense hemianesthesia b. Basilar artery

a
c. Crossed hemianesthesia c. Junction of anterior cerebral artery and internal
d. No sensory deficit is seen cartoid artery
Ref: Harrison’s, 18th ch: 370, Harrison 19th p 2576 d. Posterior cerebral artery
Ref: Neuro-Imaging, Roy Riascos, Pg 1061
248. Hemiplegia is most often caused by thrombosis of:
a. Anterior cerebral artery 255. Hemiparesis is not seen with: ( NBE Pattern 2014/ 15)
( NBE Pattern 2014/ 15)
a. Vertebral artery stroke
b. Middle cerebral artery
b. Middle cerebral artery stroke
c. Posterior cerebral artery c. Posterior cerebral artery stroke
d. Basiliar artery d. Weber syndrome
Ref: Harrison’s 18th ch: 370, Harrison 19th p 2572 73 - Ref: Harrison's, 18th ch: 370, Harrison 19th p 2576 -77
249. Which of the following is not a usual feature of right 256. In a patient with ruptured cerebral aneurysm, cause
middle cerebral artery territory infarct? of delayed neurological symptoms are, all except:
( NBE Pattern 2014/ 15) a. Enlargement of aneurysm ( NBE Pattern 2014/ 15)
a. Aphasia b. Hemiparesis b. Rebleed
c. Spasm d. Hydrocephalus
c. Facial weakness d. Dysarthria
Ref Harrison’s, 18th ch: 370, Harrison 19th p 2573 -
Ref CMDT 2013, Pg 984 85, Harrison 19th pi 785
257. The most common cause of Intraparenchymal bleed

Neurolgy
250. Cranial nerve most commonly compressed by intra ¬ is: ( NBE Pattern 2014/ 15)
cranial aneurysm: ( NBE Pattern 2014/ 15) a. Thrombocytopenia b. Diabetes
a. Oculomotor c. Hypertension d. Berry aneurysm
b. Facial Ref CMDT 2013, Pg 997, Harrison 19th p 2582
c. Optic
258. Elderly man complains ofthree episodes of visual loss ,
d. Trigeminal
in right eye over 20 minutes. Blood vessel involved is:
Ref Harrison’s, 18th ch: 370, Harrison 19th p 1785 a. Anterior cerebral artery ( NBE Pattern 2014/ 15)
251. Thrombosis of Anterior cerebral Artery, distal to the b. Middle cerebral artery
communicating branch leads to ( NBE Pattern 2014/ 15) c. Internal carotid artery
d. Basiliar artery Ref: CMDT 2013, Pg 977
a. Contralateral Hemiparesis
b. Ipsilateral hemiparesis 259. Which is true about CADASIL? ( NBE Pattern 2014/15)
c. Incontinence a. Monogenic stroke syndrome
d. Seizures b. White matter changes
c. Onset is usually in the fourth or fifth decade of life
Ref: Harrison's, 18th ch: 370, Harrison 19th p 2573 d. All of the above
252. Anterior inferior cerebellar arterial occlusion can Ref: Harrison's 18th Pg -3279, Harrison 19 th p 2568
cause: (NBE Pattern 2014/15)
260. The investigation to be performed in patient of SAH
a. Contralateral lower leg weakness with normal CT scan? ( NBE Pattern 2014/ 15)
b. Urinary retention a. Contrast enhanced CT
c. Hemianopia b. Three tube test
d. Hemianaesthesia of opposite half of the face c. MRI
Ref Harrison’s, 18th ch: 370, Harrison 19 th p 2576 d. Gadolinium enhanced MRI
Ref: CMDT 2013, Pg 985-86
253. Which of the following is not involved in lateral med ¬
ullary syndrome? ( NBE Pattern 2014/ 15) 261. Following are features of ischemia in anterior choroi ¬
a. Sympathetic tract dal artery territory except: ( NBE Pattern 2014/ 15)
b. IX, X, XI cranial nerves a. Hemiparesis b. Hemisensory loss
c. XIIth cranial nerve c. Homonymous hemianopia
d. Predominant involvement of the anterior limb of
d. Spinothalamic tract
internal capsule
Ref Harrison’s, 18th p 370, Harrison 19th p 2576 Ref: Harrison’s 18th ch: 370, Harrison 19th p 2573 74 -
Ans. 247. c. Crossed hemi... 248. b. Middle cerebral artery 249. a. Aphasia 250. a . Oculomotor
251. c. Incontinence 252. d. Hemianaesthesia... 253. c. Xllth cranial nerve 254. a. Vertebral artery
255. a . Vertebral artery... 256. a . Enlargement of ane... 257. c. Hypertension 258. c. Internal carotid artery
259. d. All of the above 260. b. Three tube test 261. d. Predominant...
on Harrison's 19'")
Marwah's Internal Medicine MCQs ( Based
haemorrhage is:
all except: 269. Most common cause of intracranial
262. Pontine Stroke is associated with a. Sub arachnoid haemorrhage ( AllMS Nov 98)
a. Bilateral pin point pupil ( NBE Pattern 2014/ 15)
b. Intracerebral hemorrhage
b. Pyrexia
c. Vagal palsy c. Subdural haemorrhage
d. Quadriparesis d. Extradural haemorrhage
fjm Ref: CMDT 2013, Pg 977, Harrison 19 th pi 775 Ref : Harrison 18th/ p 3294, Harrison 19th p 2582
syn- 270. Which of the following is the most
common location
263 Which cranial nerve is involved in locked in
( A11994, 2003)
Wm drome?
a. 7 b. 9
( PCI May 2012 ) of hypertensive
a. Pons
hemorrhage ?

c. 10 d. 12 b. Thalamus
e. All of the above c. Putamen / external capsule
Ref: Harrison's 18th ch. 370, Harrison 19th p 2578 d. Subcortical white matter
264. Posterior cerebral artery occlusions leads to loss of Ref : Harrison 18th/ p 3294, Harrison 19th p 2582
memory due to involvement of:( NBE Pattern 2014/ 15)
a. Superior temporal gyrus 271. Commonest cause of subarachnoid haemorrhage is:
b. Supra marginal gyrus a . Rupture of circle of Willis aneurysm ( AI 1998)
c. Angular gyrus b. Rupture or vertebral artery aneurysm
d. Hippocampus c. Rupture of venecomitants of corpus striatum
Ref Harrison’s, 18th ch: 370, Harrison 19th p 2575 d. Rupture of dural sinusis

Neurolgy
265. History of transient ischemic attack, excludes: Ref : Harrison 18th/ 2262, Harrison 19th p 1784
a. Amaurosis fugax ( NBE Pattern 2014/ 15) ( AIIMS May 2011 )
272. Berry aneurysm is caused by:
b. Weak shoulder shrugging
c. Asymmetrical mouth retraction a. Degeneration of internal elastic lamina
d. Seizures b. Degeneration of tunica media
Ref Harrison’s 18th Ch: 370, Harrison 19 th p 2568 c. Degeneration of muscular layer
d. Degeneration of external elastic lamina
266. Features of posterior inferior cerebellar artery throm ¬
Ref : Harrison 19th p 1785
bosis include all of the following except:
( NBE Pattern 2014/ 15) 273. Which is least common site of berry aneurysm:
a. Sudden onset of severe vertigo a. Basilar artery ( AIIMS Dec 95)
b. Acute cerebellar S/S with Nystagmus to the side of b. Vertebral artery
lesion the involvement c. Anterior cerebral artery
c. Palsy of 10,11,12 nerves d. Posterior cerebral artery
d. Horner’s syndrome
Ref: Harrison’ s, 18th ch. 370, Harrison 19 th p 2576 Ref : 18th/ 2262, Harrison 19thpi 784
274. Which of the following is the most common cause of
267. 80 year diabetic presents with right sided face, arm
and leg weakness. Sensation, speech and comprehen ¬ late neurological deterioration in a case of cerebro ¬
sion are intact. Blood vessel involved is: vascular accident : . (AIIMS Nov. 2000)
( NBE Pattern 2014/ 15) a. Rebleeding
a. Anterior cerebral artery b. Vasospasm
b. Lenticulostriate artery c. Embolism
c. Internal carotid artery d. Hydrocephalus
d. Basiliar artery
Ref: Harrison 18th/ p 2262, Harrison 19th p 1785
Ref CMDT 2013, Pg 979, Harrison 19 th p 2572
268. Most common cause of cerebrovascular accident is : 275. ‘Duret Hemorrhages’ are seen in : ( AIIMS May 08)
a. Embolism ( AI1MS 96 ) a. Brain
b. Arterial thrombosis b. Kidney
c. Venous thrombosis c. Heart
d. Haemorrhage d. Lung
Ref: Harrison 18th/ 3270, 3271, Harrison 19 th p 2565
Ans. 262. c. Vagal
- .palsy 263. e. All of the above
266.
-
c. Palsy of 10,11 ,12... 267.
264. d. Hippocampus 265. d. Seizures
b. Lenticulostriate artery 268. b. Arterial thrombosis
270. c. Putamen/... 269. b. Intracerebral hemorrhage
271. a . Rupture of circle... 272. a. Degeneration of inter...273. b. Vertebral artery
274. b. Vasospasm 275. a . Brain
Neurology

276. A 45 year old male patient presented in the casual ¬ 283. Third nerve palsy with pupillary sparing is seen in all
ty with two hours history of sudden onset of severe

a
except: (AIIMS May 95)
headache associated with nausea and vomiting on a. Hypertension
clinical examination the patient had necks stiffness b. Aneurysm of posterior communicating artery
and right sided ptosis. Rest of the neurological exami ¬ c. Diabetes mellitus
nation was normal. What is the clinical diagnosis : d. Craniopharyngioma
a. Hypertensive brain haemorrhage ( A1IMS Nov 03) Ref : Harrison 18th/ p 238, Harrison 19th p 1785
b. Migraine
c. Aneurysmal subarachnoid haemorrhage 284. Most common nerve involved in intracranial aneu¬
d. Arteriovenous malformation haemorrhage rysm is: ( AI 1998)
a. Trochlear b. VII
Ref.: Harrison 19th p 1784
c. VIII d. Occulomotor
277 . All of the following are features of temporal lobe le ¬ Ref : Harrison 18th/ p 2262, Harrison 19th p 2983
sions except: ( ARMS Dec 94 )
a. Wernicke’s Aphasia 285. Lacunar infarcts are caused by : ( AI 2001 )
b. Contralateral homonymous superior quadran- a. Lipohyalinosis of penetrating arteries
tanopia b. Middle carotid artery involvement
c. Auditory hallucinations c. Emboli to anterior circulation
d. Hemineglect d. None of the above
Ref : Davidson's 20th/ 1152; Harrison 19 th p 180 Ref : Harrison 18th/ p 3276, Harrison 19th p 2565

278. Hemiplegia is commonly associated with infarction of 286 . Investigation of choice for screening of proximal in ¬
ternal carotid artery stenosis is : (ARMS June 2000)

Neurolgy
the area of distribution of the: (AI 1994)
a. Anterior cerebral artery a. Doppler flow USG
b. Middle cerebral artery b. CT substraction angiography
c. Posterior cerebral artery c. MRI
d. Anterior communicating artery d. Angiography ( DSA)
Ref: Harrison 18th/ p 3284, 3285, Harrison 19th p 156 Ref: Harrison 15th/ 2237, Harrison 19th p 2581
287 . Wallenberg’s syndrome does not involve which cra ¬
279. Which of the following is not a usual feature of right
nial nerve: ( PGI June 99)
middle cerebral artery territory infarct:
a. IX b. X
( ARMS Nov 02)
b. Hemiparesis
c. XI d. XII
a. Aphasia
c. Facial weakness d. Dysarthria Ref : Harrison 18th/ p 3288, 3289, Harrison 19 th p 2576
Ref: Ch 370, Harrison 18th Harrison 19 th p 2573 288. Lateral medullary syndrome (Wallenberg syndrome )
is characterized by all, Except: ( ARMS June 97)
280. Which of the following sites is not involved in a poste ¬ a. Giddiness
rior cerebral artery infarct: ( AI 2011 )
b. Dysphagia
a. Midbrain b. Thalamus
c. Crossed hemianaesthesia
c. Temporal lobe d. Anterior Cortex
d . Horner’s syndrome is rare
Ref : Harrison 18th/ p 3286, 3287, Harrison 19th p 2575 Ref: Harrison 18th/ p 3288, Harrison 19th p 2576
281. Memory impairment occurs in embolism of posterior 289. Pontine Stroke is associated with all except :
cerebral artery because of damage to : ( ARMS Dec 94) a. Bilateral pin point pupil ( AI 2007)
a. Hippocampal gyrus b. Pyrexia
b. Superior temporal gyrus c. Vagal palsy
c. P Refrontal gyrus d. Quadriparesis Ref : , Harrison 19 th p 1775
d. Angulargyrus
Ref: Harrison 18th/ p 3287, Harrison 19 th p 2575 290. The only thrombolytic agent approved for the treat¬
ment of acute ischemic stroke is: ( ARMS Nov 03)
282. All of the following Neurophysiological defects are a. Tissue Plasminogen activator
likely to result from right lobe involvement, except: b. Streptokinase
a. Visuospatial defects (ARMS May 08) c. Urokinase
b. Anosognosia
c. Dyscalculi
-
d. Pro urokinase
Ref : Harrison 18th/ p 3272, 3273, Harrison 19 th p 2652
d . Spatial Dysgraphia
Ans. 276. c. Aneurysmal sub... 277. d. Hemineglect 278. b. Middle cerebral artery 279. a. Aphasia
280. d . Anterior Cortex 281. a. Hippocampal gyrus 282. c. Dyscalculi 283. d . Craniopharyngioma
284. d . Occulomotor 285. a . Lipohyalinosis of pen .. 286. a . Doppler flow USG 287. d. XII
288. d . Horner’s syn ... 289. c. Vagal palsy 290. a . Tissue Plasminogen activator
s 19 )
Marwah's Internal Medicine MCQs ( Based on Harrison

ophthalmo 297 . A Thirty five year old female has proximal weakness
291. A patient presents with unilateral painful
¬

an enlargement of cavern ¬ of muscles, ptosis and easy fatigability. The most sen ¬
plegia. Imaging revealed ( AI 2011 )
side . The likely diagnosis is: sitive test to suggest the diagnosis is:
ous sinus on the affected a . Muscle Biopsy
(AllMS May 08)
b. CPK levels
a. Gradenigo syndrome
b. Covernous sinus thrombosis c. Edrophonium test
-
c. Tolosa Hunt Syndrome d. Single fiber EMG
d. Orbital Pseudotumor .-
Ref Harrison 18th/ p 3481, Harrison 19 th p 2702
(AI 1999 ) regarding Eaton
292. Prosopagnosia" is characterised by :
" 298. Whch one of the following is correct
a . Inability to read Lambert syndrome - ( AIIMS Nov 04)
b. Inability to identify faces a. It commonly affects the ocular muscle
c. Inability to write b. Neostigmine is the drug of choice for this syndrome
d. Inability to speak Ref.: Harrison 19th p 182 c. Repeated electrical stimulation enhances muscle
power in it.
M . Gravis
d. It is commonly associated with adenocarcinoma of
lung
293. Cranial Nerve 8 palsy causes all except? Ref : H 18th ed, Ch 386, Harrison 19th p 2703
( NEET Pattern 2015-16 )
a. Gag Reflex b. Vertigo 299. Thymoma is associated with: (AI 2000)
c. Motion sickness d. Tinnitus a. Myasthenia gravis
b. Scleroderma

Neurolgy
Ref: Harrison’s 18th Ch 367 and e42, Harrison 19th p 2538
c. Oesophageal atrasia
1 294. Ice pack test is done for: ( NBE Pattern 2014/ 15)
d . Hyper-gammaglobulinemia
a. M. Gravis
Ref : Harrison 18th/ p 3480, 3483, Harrison 19th p 2701
b. Multiple system atrophy
c. Hyperparathyroidism 300. Which of the following is not associated with Thymo ¬

d. Hypokalemic periodic paralysis mas: ( AI 2001 )


Ref: Neurological signs by A] Earner, 2013 a. Red cell aplasia
b. Myasthenia gravis
295. 137. Miller Fisher test is used for: c. Hypergammaglobulinemia
( NBE Pattern 2014/ 15) d. Compression of the superior mediastinum
a. Evaluating cognition in normal pressure hydroceph ¬
Ref : Harrison 19 th p 2701
alus
b. Severity of paralysis in Gullian Barre syndrome 301. In myasthenia gravis, correct statement regarding
c. Grading muscle involvement in ALS thymectomy is : ( AI 2001 )
d. Measuring disability in multiple sclerosis a. Should be done in all cases
Ref: Neurology Review, Andrew Tarulli, 2011 b. Should be done in cases with ocular involvement
only
296 . A 35 year old man presents with complaints of pto ¬ c. Not required if controlled by medical management
sis, with difficulty in chewing and occasionally swal ¬ d. Should be done only in cases that are associated
lowing. On examination there is a asymetrical ptosis, with thymoma
lateral arm abduction time of 60 seconds. Repetitive
Ref : Harrison 18th/ p 3483, 3484, Harrison 19 th p 2704
nerve stimulation test shows decremental response.
EMG shows myopathic response. Anti-Ach receptor
antibodies are negative. Probable diagnosis is:
Channelopathy
(AIIMS May 2013) 302. All of the following are associated with ‘Episodic’
.
a. Ocular M Gravis weakness, except: ( AI 2012 )
b. Generalised M. Gravis a. Channelopathy
c. Since Anti-Ach receptor antibodies are negative b. Lambert- eaton syndrome
consider other diagnosis
c. Hyperphosphatemia
d. Chronic Progressive external opthalmoplegia
d. Hyperkalemia
Ref: Harrison's 18th ch.386, Harrison 19th p 2702
Ref: Harrison's 18/ e p! 85, Harrison 19th p 2460

Ans. 291 . c. Tolosa - Hunt ... 292. b. Inability to identify... 293. a . Gag Reflex 294. a . M . Gravis
295. a . Evaluating cog ... 296 . b. Generalised M . Gravis 297 . d . Single fiber EMG 298 . c . Repeated electrical . . .
Y YT 299 - a - Myasthenia gravis 300 . c . Hypergamma . . . 301 . a . Should be done .. . 302. c . Hyperphosphatemia
Neurology

303. Which of the following is a Channelopathy?


310. Which is true about hydrancephaly:
a. Ataxia Telangiectasia
b. Frederich Ataxia
( NEET Pattern 2015 161 - ( NBE Pattern 2014/ 15)
c. Spinocerebellar ataxia a. Sun setting sign
d. Anderson Tawil Syndrome b. Normal size ventricles

l
c. Mental retardation
Refi: Harrison 18th edn, Ch 387, Harrison 19th p 462e -
18 d. Absent cerebral hemispheres
304. Episodic muscle weakness can be caused by all of the Refi Nelson 18th edn ch. 592.7
following except: (MPG 201 n
a. Hypercalcemia 311. Which of the following is a neuronal migration defect?
b. Channelopathies ( NBE Pattern 2014/ 15)
c. Lambert- Eaton syndrome a. Gaucher disease b. Schizencephaly
d. Hyper-phosphatemia c. SSPE d. Sphingolipidosis
Ref: H 18/ e, pg 3100, 3482, 3504, Harrison 19 th p 444e, 2703 Refi Ch: 592.7, Nelson, 18th ed

305. SCN4A defect is seen in: ( NBE Pattern 2014/ 15 ) Disease of Spinal Cord
a. Brugada syndrome
b. Hyper-kalemic periodic paralysis 312. Patient was treated for Mantle cell Hodgkin lymphoma
c. Lambert Eaton syndrome with radiation. After 6 months, he develops an electric
d. Para-myotonia congenital shock like pain along the spine on flexing his neck.
Ref: CMDT 2013, Pg 1036, Harrison 19 th p 444e What is the diagnosis? (JIPMAR 2014)
a. Cervical arthritis b. Lhermitte sign

Neurolgy
306. All of the following are neurologic channelopathies, c. Uthoff sign d. Spinal cord compression
except
Ref: Harrison 19th p 708
a. Hypokalemic periodic paralysis ( AIIMS May 04)
b. Episodic ataxias 313. All of the following are true about Brown Sequard
c. Familial hemiplegic migraine Syndrome, except: (AIIMS Nov 2011 )
d. Huntington's disease a. Ipsilateral Pyramidal Tract Features
Ref : Harrison 18th/ p 3225 Table: 366.1, Harrison 19th p 2621 b. Contralateral Spinothalamic Tract Features
c. Contralateral Posterior Column Features
Pediatric Neurology d. Ipsilateral Plantar Extensor
Ref: Harrison’s 18/ e p3367, Harrison 19th p 156, 161, 2652
307 . A high maternal level of alfa foeto protein is noted, in 314. Spinal shock is characterized by (select three best
most cases of: ( Manipal ) options): ( PGIJune 03)
a. Spina bifida a. Spasticity b. Wasting
b. Gaucher’s disease c. Sensory loss d. Urinary retention
c. Tay-Sach’s disease e. Areflexia
d. Down’s syndrome Refi HI 8/ e p3366-3367, Harrison 19th p 2653, 2653 f
Refi SARP Tumors pg 7 Harrison 19th p 118
315. H reflex is used for? ( AIIMS Nov. 14)
308. Small posterior fossa seen in? ( NEETPattern 2015 16) - a. SI radiculopathy b. L5radiculopathy
a.Arnold chiari malformation c. L4 radiculopathy d. L3 radiculopathy
b. Dandy walker Refi: Harrison 18th E 45
c. Medulloblastoma
d.Schizencephaly Ref : Nelson 18th edn, Ch 592: 316. All of the following features are seen in myelopathies
except? (AIIMS Nov. 2012)
309. SSPE is not diagnosed by? ( NEETPattern 2015 16 ) - a. Facial sensory impairment
a. EEG b. Brisk jaw jerks
b. Antibodies to measles in CSF
c. Brisk pectoral jerks
c. Antibodies to measles in blood
d. Urgency and incontinence of micturition
d. Antigen in brain biopsy
Refi: Ch 243: Nelson 18th edn, Ref. De Jong Neurological examination Pg No.l 94,201& Pg no.474

Ans. 303. d. Anderson Tawil... 304. a. Hypercalcemia 305. b. Hyper-kalemic... 306. d. Huntington’s disease
307. a. Spina bifida 308. a. Arnold chiari... 309. c. Antibodies to meas... 310. d. Absent cerebral...
311. b. Schizencephaly 312. b. Lhermitte sign 313. c. Contralateral Post... 314. c, d, e Sensory loss...
315. a. S1 radiculopathy 316. b. Brisk jaw jerks
's 19*h )
Marwah's Internal Medicine MCQs ( Based on Harrison
cause spinal
in India 325‘ All of the following metastatic tumours
317. Most common type of spinocerebellar ataxia cord compression except : ( NBE Pattern 2014/ 15)
( AllMS Nov 2012)
a. Lung cancer b. Breast cancer
a. SCA1 b. SCA2 c. Lymphoma d. Meningioma
c. SCA3 d. SCA4 Ref: Harrison's 18th Ch:377, Harrison 19th p 2653
Ref: Ch. 373: Harrison's 18th Harrison 19th p 2628 seen in:
326 Dissociative sensory loss is not
( NBE Pattern 2014/ 15)
318. Most common cranial nerve involvement causing op- a. Syringomelia
II thalmoparesis in Gullian Barre syndrome is due to: b. Cauda equina syndrome
SS ( NBE Pattern 2014/ 15) c. Diabetes mellitus
a. 6th nerve b. 7th nerve d. Damage to spino-thalamic pathways
c. 8th nerve d. 9th nerve Ref: Harrison's 18th ch.377, Harrison 19th p 2651
Ref: Harrison’s 18th Ch. 385, Harrison 19 th p 2697 327' Sub-acute combined degeneration of spinal cord is
Seen in: ( NBE Pattern 2014/ 15)
319. In cervical syringomyelia all are seen except:
a. Burning sensation in hands ( AllMS Nov 2012) a. Thiamin deficiency
b. Hypertrophy of abductor pollicis brevis b. Tuberculosis of spine
c. Plantar extensor c. Addisonian pernicious anemia
d. Absent biceps reflex. d. Industrial toxin damage to spinal cord
Ref 18th Ch. 377, Harrison 19th p 2658 Ref: Kumar & Clark: Clinical Medicine, 2012, Pg 816
320. Most common slowly growing vascular tumor of spinal 328. Dose of methylprednisolone following spine injury:
cord, cerebellum and brain is: (AllMS Nov 2012) a. 30 mg/ kg within 3 hrs of injury

Neurolgy
a. Hemangioblastoma b. Pilocytic astrocytoma ( NBE Pattern 2014/ 15)
c. Meningioma d. Medulloblastoma b. 45 mg/ kg within 6 hrs of injury
Ref: Harrison' s 18th ch. 343, Harrison 19th p 2654 c. 50 mg/ kg within 9 hrs of injury
I 321. Most common extra -dural tumour of the spine is: d. 60 mg/ kg within 12 hrs of injury
a. Neurofibroma ( Bihar PG 2014 ) Ref CMDT 2013, Pg 1018
b. Glioma 329. Hypotension in Acute Spinal Cord injury is due to:
c. Meningioma a. Loss of Sympathetic tone ( NBE Pattern 2014/ 151
d. Metastasis b. Loss of Parasympathetic tone
Ref: Harrison's ch. 377, Harrison 19 th p 2654 c. Vasovagal Attack
322. Onion skin pattern of sensory loss in face is seen: d. Orthostatic Hypotension
a. Syringomyelia ( NBE Pattern 2014/ 15) Ref: CMDT 2013, Pg 1018
b. Diabetes mellitus
330. The cause of systemic secondary insult to injured
c. Amyloidotic polyneuropathy
brain include all except: ( NBE Pattern 2014/ 15)
d. Leprosy
a. Hypocapnia b. Hypoxaemia
Ref: Ch. 377: Harrison's 18th , Harrison 19 th p 2658 c. Hypotension d. Hypothermia
323. Dissociative sensory loss is seen in all except:
Ref: Harrison’s 18th ch: 275, Harrison 19 th p 1781
( NBE Pattern 2014/ 15)
a. Anterior spinal artery occlusion 331. Hexanucleotide repeats are seen in:
b. Leprosy ( NBE Pattern 2014/ 15
c. Multiple system atrophy a.
Motor neuron disease
d. Hydromelia b.
Spinocerebellar ataxia
c.
Huntington chorea
Ref: 250 cases in clinical medicine pg. 70-71 d.
Spinomuscular atrophy
324. Not seen in miller fisher syndrome is: Ref: Harrison's 18th ch: 374, Harrison 19th p 2633
a. Anti -GQl antibodies ( NBE Pattern 2014/ 15) 332. Which of the following sensation are transmitted by
b. Ataxia
the Dorsal Tract/ Posterior column: ( Al 2008)
c. Cranial nerve palsy
a. Fine touch b. Pain
d. Postural hypotension c. Temperature d. All of the above
Ref: Harrison 19 th p 2698
-
Ref..Ganong 22nd / 138; Guyton llth/ 588

Ans. 317. b. SCA2


321. d. Metastasis 322. a . Svrinomrivplia w C' Mult e system
abd...
,
b' Hypertrophy of 320. a. Hemangioblastoma

B 325.
329.
d . Meningioma
a. Loss of Sym... Svoo? ? nl
326 b CanHa « ' Syndrome
330. d . Hypothermia
he mia
M
P ..
327. C. Addisonian per...
. '
331. a . Motor neuron disease
324.
328.
332.
d. Postural hypotension
a . 30 mg/ kg whhin 3 hrs..
a. Fine touch
Neurology

333. A ventrolateral cordotomy is performed to produce 340. Spastic paraplegia is caused by all , except: ( AI 2009 )
relief of pain from the right leg. It is effective because a. Vitamin B12 deficiency
it interrupts the: b. Cervical spondylosis
a. Left Dorsal Column ^
2012)
c. Lead poisoning
b. Left Lateral Spinothalamic tract
c. Right Lateral Spinothalamic tract
d. Right Corticospinal tract
334. Type of sensation lost on same side in Brown sequard
syndrome is:
a. Pain
b. Touch
c. Proprioception
( AllMS Nov 93)
d. Motor neuron disease
Ref : Neurology by Anish Bahra/ 68; Oxford H o f T M 2nd/ 436
341. Anterior spinal Artery thrombosis is characterized by
all, except:
a. Loss of pain & touch
b. Loss of vibration sense
c. Loss of power in lower limb
( PGI Dec 2000 ) E
d. Sphincter dysfunction
d. Temperature
Ref : Harrison 19th p 2652
Ref.: Harrison 18th/ p 191, 3367, Harrison 19th p 2652
342. Most common drug used in spasticity in patients with
335. The following are components of Brown Sequard syn ¬ spinal cord injury is: ( NBE Pattern )
drome except : ( 4 / 2007) b. Tinazidine
a. Diazepam
a. Ipsilateral extensor plantar response
c. Salicylates d. Baclofen
b. Ipsilateral pyramidal tract involvement
c. Contralateral spinothalamic tract involvement Ref : Harrison 19th p 2672
d. Contralateral posterior column involvement 343. 65 year old man presents with anaemia and posterior

Neurolgy
-
Ref .Harrison 18th/ 191, 3367, Harrison 19th p 2652 column dysfunction, the likely cause is:
a. Brdeficit (AIIMS May 95)
336. Which of the following is not a feature of extramedul ¬
b. B12-deficit
lary tumour: ( AI 2008 )
c. SSPE
a. Early Corticospinal signs and paralysis
d. Multiple sclerosis
b. Root pain or midline Back -pain
c. Abnormal CSF Ref : Harrison 18th/ p 3374, Harrison 19th p 2659
d. Sacral sparing 344. Features of syringomyelia include all of the following,
Ref : Harrison 18th/ p 336, Harrison 19th p 2651 -52 except : ( PGI 09 )
a. Dissociative sensory loss
337. Clinical features of Conus Medullaris syndrome in ¬ b. Bilateral involvement
clude all of the following Except: ( A12008) c. Segmental sensory loss
a. Plantar Extensor d. Wasting of small muscles of hand
b. Absent knee & ankle jerks e. Ascending weakness
c. Sacral anesthesia Ref : Ch - 387 Harrison 18th, Harrison 19th p 2658
d. Lower sacral & coccygeal involvement
345. All of the following are true about Guillain Barre Syn ¬
Ref : Principles of Surgical patient care 2nd / 501, De Jong's Neu¬
rological examination 6th/ 578 drome (GBS), Except: ( Al 2010)
a. Ascending paralysis
338. Hypotension in Acute Spinal Cord Injury is due to : b. Flaccid paralysis
a. Loss of Sympathetic tone (AIIMS Nov. 06 ) c. Sensory level
b. Loss of Parasympathetic tone d. Albumino-Cytological Dissociation
c. Vasovagal Attack Ref : H 18th/ p 3473, 3474, 3475, Harrison 19th p 2694
d. Orthostatic Hypotension
346. Conduction velocity of nerve is reduced in all of the
Ref : Harrison 18th/ p 3366, 3367, Harrison 19 th p 2651
following conditions, except: (AI 2012)
339 . Which of the following signs is not suggestive of a a. Acute Motor Axonal Neuropathy (AMAN )
cervical spinal cord injury : ( AIIMS Nov. 05)
b. Acute Inflammatory Demyelinating Neuropathy
a. Flaccidity (AIDP)
b. Increased rectal sphincter tone c. Hereditary Sensory Motor Neuropathy ( HSMNj
c. Diaphragmatic breathing d. Multifocal Motor Neuropathy
d. Priapism
_ Ref : Harrison 18th/ 3452, 3473, Harrison 19th p 2695
Ref : Ch E-42, section-l H 18th/ edn, Harrison 19 th p 438e

Ans. 333. b. Left Lateral... 334. c. Proprioception 335. d. Contralateral post... 336. d. Sacral sparing
337. b. Absent knee... 338. a . Loss of Sympathetic... 339. b. Increased rectal... 340. c. Lead poisoning
341. b. Loss of vibration...342. d. Baclofen 343. b. B12-deficit 344. e. Ascending weakness
345. c. Sensory level 346. a. Acute Motor Axonal...
Marwah's Internal Medicine MCQs ( Based on Harrison's 19th )

347. Charcot’s Joint includes all of the following except : 354. A farmer with long history depression on antidepres ¬
a. Syringomyelia .
(AIIMS Nov 06 ) sants, consumed unknown substance and presented
b. Leprosy with bradycardia, hypersalivation, small pupils. What
c. Diabetes is the most probable diagnosis? (JIPMER 2014 )

1
d. Arthrogryposis Multiplex Congenita a. OP poisoning b. Atropine
Ref.: Harrison 18th/ p 2856 T: 336.2, Harrison 19th p 2243 c. TCA d. Paracetamol poisoning
Ref : H 18th Harrison 19th p 267, 473-e
Multiple Sclerosis 355. Which of the following drugs is not recommended for
the treatment of Multiple Sclerosis: ( AIIMS Nov 2011 )
348. Immunological abnormality seen in ataxia telangiec¬ a. Interferon p- la
tasia: ( Manipal ) b. Interferon p- lb
a. Decreased serum IgA c. Glatiramer acetate
b. Decreased IgM d. Mycophenolate
c. Elevated levels of IgE -
Ref: Harrison’s 18/ e p3402, Harrison 19th p 441e 27 f
d. Increased IgG
356. Which of the following clinical features of demyelin -
Ref: Harrison 19th p 451e-3t / 2109 ating myelopathy least likely suggests a progression
349. All are the features of absence seizures except: to multiple sclerosis? (AIIMS Nov. 2012)
a. Usually seen in childhood ( Manipal ) a. Complete cord transection
-
b. 3 Hz spike wave in EEG b. Bilateral visual loss
c. Postictal confusion c. Absence of oligoclonal bands

Neurolgy
d. Precipitation by hyperventilation d. Poor prognosis
Ref: Harrison 19th p 2543 Ref: HI 8th Ch. 380 Harrison 19th p 2665
357. Drug that causes maximum reduction in appearance
350. DOC for a / c migraine attack of moderate intensity:
of new lesions and change in disease severity in mul ¬
a. Paracetamol ( Manipal)
tiple sclerosis is: ( NBE Pattern 2014/ 15)
b. Paracetamol + Ergotamines + Sumatriptan +
a. Natalizumab b. Glatiramer
Domperidone
c. Interferon beta la d. Interferon beta lb
c. Paracetamol + Sumatryptan
Ref: Harrison’ s 18th ch. 380, Harrison 19th p 2669
d. Ergotamines + Sumatriptan + Domperidone
Ref: KD Tripathi 5th pg 152 Harrison 19th p 2593 358. 30 year old female presents with complaints of grad ¬
ual onset weakness of legs for one month with reduc
|
| 351. DOC for Alzheimer’s dementia: ( Manipal ) tion in visual acuity and urinary incontinence for past
a. Valproic acid few days. Contrast MRI shows periventricular lesions.
b. Clozapine Which of the following drugs is not used in these pa ¬
c. Acetyl cholinesterase inhibitors tients? ( NBE Pattern 2014/ 15)
d. Cinnerizine a. Fingolimod b. Beta interferon
Ref: Kumar & Clark 1265 Harrison 19th p 2598 c. Glatiramer acetate d. Mitotane
352. In Werdnig Hoffman’s disease is a disease of: Ref: Harrison's 18th ch:380, Harrison 19th p 2669
a. Posterior horn cell ( Manipal ) 359. Pulfrich effect is seen in: ( NBE Pattern 2014/ 15)
b. Anterior horn cell a. Multiple sclerosis b. C.I.D.P
c. Grey mater c. C.R.P.S d. Red green color blindness
d. Motor endplate Ref: Harrison's 18th ch. 380
Ref: Kumar & Clark pg 1209 Harrison 19th p 2636 360. Which of the following is used in the treatment of
353. Features of ’ BALINT SYNDROME ”. ( Manipal ) Multiple Sclerosis: (AIIMS Nov. 2006 )
a. Oculomotor apraxia a. Interferon Alpha
b. Simulanagnosia b. Interferon Beta
c. Optic ataxia c. Infliximab
d. All the above d. Interferon gamma
Ref: CMDT 2015, pg - 980 Ref : Harrison 18th/ p 3403, 3404, Harrison 19th p 2269

Ans. 347. d. Arthrogryposis... 348. a. Decreased serum IgA 349. c. Postictal confusion 350. b. Parace...
351. c. Acetyl choline... 352. d. Motor endplate 353. d. All the above 354. a . OP poisoning
355. b. Interferon p-1b 356. c. Absence of oligo... 357. a. Natalizumab 358. d. Mitotane
359. a. Multiple sclerosis 360. b. Interferon Beta
Neurology

Miscellaneous 369. Subdural hemorrhage is secondary to the rupture of:


a. Middle meningeal artery ( Manipal )
361. A 55 - year- old patient cannot copy a simple drawing. b. Dural venous sinus
Which area of his brain is most likely affected: c. Bridging veins
A. Non dominant frontal lobe ( Manipal ) d. Middle meningeal vein
b. Non dominant parietal lobe
C. Non dominant temporal lobe
Ref: Bailey 24th pg 598 Harrison 19 p 457 e-3f
th
II
d. Non dominant occipital lobe 370. Features of Huntington’s chorea are all except:
a. Autosomal dominant ( Manipal )
Ref: Harrison 18th Ch. 370
362. Treatment modalities used in the management of bed ¬ b. Caudate nucleus calcification in CT
wetting include all the following except: ( Manipal ) c. Inheritance increases after 50 years
a. Daytime training d. Males and females are equally affected
b. Restriction of fluid intake at night Ref: Harrison 19th p 2621
C. Imipramine 371. Duchenne's muscular dystrophy ( Manipal )
d. Tolnaftate Ref: O.P. Ghai p. 468, 504 a. Neuromuscular junction
363 . Most common presenting feature of ALZHEIMER’S dis¬ b. Sarcolemmal proteins
ease: ( Manipal ) c. Muscle contractile proteins
a. Change in personality d. Disuse atrophy due to muscle weakness
b. Difficulty in learning new information
c. Loss of intellectual skills
Ref: Harrison 19th p 462e-5 j

Neurolgy
d. Loss of memory Ref: Harrison 19th p 2598 372. Reduced blink reflex occurs in all except: ( Manipal ) j
( Manipal ) a. Alcohol intoxication
364. Lesion in tuberous sclerosis:
a. Adenoma malignum b. Parkinsonism
b. Adenoma sebaceum c. Progressive supranuclear palsy
c. Cafe au lait spots d. Myasthenia gravis Ref: Harrison 19th p 2701
d. Axillary freckling Ref: Harrison 19th p 604 373. Features of tentorial herniation include: ( Manipal )
365. In multiple sclerosis, CSF level of protein is: ( Manipal ) a. Vomiting
a. Increased b. Decreased b. Deterioration of consciousness
c. Depleted d. Normal c. Dilated pupil
Ref: Harrison 19th p 441 e - 27 f d. All the above Ref: Harrison 19th p 1772
366. About Parkinsonism which of the following is false? 374. In benign intracranial hypertension: ( Manipal)
( Manipal ) a. There is a restriction of upgaze
a. Most of the patients have head tremors at rest b. Normal ventricles is found in 50% of cases
b. Primitive reflexes are released c. Brain scan is important in young women to excluded
c. Cog wheel rigidity is present sagittal sinus thrombosis
d . Festinant gait is there Ref: Harrison 19th p 2611 d. Optic nerve fenestration should be performed early
367 . CSF analysis in a person suffering from fever shows before the vision is affectd.
increased lymphocytes, slightly elevated proteins and Ref: Harrison 19th p 1615, 1627, 2582
normal glucose levels. The person is suffering from:
a. Pyogenic meningitis ( Manipal ) 375. All are seen in alkaptonuria except: ( Manipal )
b. Tuberculous meningitis a. Cardiac disease b. Ochronosis
c. Syphilitic meningitis c. Arthritis d. Seizures
d. Viral meningitis Ref: H 18th pg 3219 Harrison 19th p 434e - 2 t, 434e - 4 5
Ref: George Mathews pg 189 Harrison 19 p 888
th
376. Lacunar infarcts are caused by: (AI 2001)
368. Confabulations and early memory loss, is a feature of: a. Lipohyalinosis of penetrating arteries
a . Wernicke’s encephalopathy ( Manipal )
b. Middle carotid artery involvement
b. Korsakoff ’s psychosis c. Emboli to anterior circulation
c. Huntington chorea d. None of the above
d. Alzheimer's disease
Ref: H 18/ e p3276, Harrison 19 th p 441 e - l 7 f
Ref: Harrison 19 p 1783/ 2607 / 2724
th

Ans. 361. b. Non dominant... 362. d . Tolnaftate 363. d . Loss of memory 364. b. Adenoma sebaceum
365. d . Normal 366. a. Most of the patients... 367. d . Viral meningitis 368. b. Korsakoff ’s psychosis
369. c. Bridging veins 370. c. Inheritance increases... 371. c. Muscle contractile... 372. d. Myasthenia gravis
373. d. All the above 374. c. Brain scan is impor... 375. d. Seizures 376. a. Lipohyalinosis...
Marwah's Internal Medicine MCQs ( Based on Harrison's 19th )

377. The earliest manifestations of increased intracranial 384. Feature (s) of Duchenne muscular dystrophy includes:
pressure following head injury is: (AI 2005J a. Defective gene is dystrophin ( PGI May 2013)
a. Ipsilateral papillary dilatation b. Ragged red fibres
b. Contralateral papillary dilatation c. Autosomal recessive mode of inheritance

0 c. Altered mental status


d. Hemiparesis
Ref: H 18/ e p2257, Harrison 19th p 457 e-l
378. A patient presents with unilateral painful ophthalmo¬
plegia. Imaging revealed an enlargement of cavern ¬
ous sinus on the affected side. The likely diagnosis is:
a. Gradenigo syndrome (AlIMSMay 08)
d. Anticentromere antibody present
e. Serum CK levels are raised
Ref: H 18/ e p3491 -3495, Harrison 19th p 462 e-5
385. True about ankylosing spondylitis: ( PGI May 2013)
a. Presents with backache and joint stiffness
b. Associated with HLAB27
c. Inflammation of ligament and tendon at attachment
to bones
b. Covemous sinus thrombosis d. Foot joint involvement
c. Tolosa- Hunt Syndrome e. Spine involvement
d. Orbital Pseudotumor -
Ref H 18/ e p2774 2775, Harrison 19 th p 2169, 2171
Ref Harrison 19th p 207 / 1352 386. A 7 year old girl’s, parents gave history of fever for
379. Neurotransmitter in striatal pathway is: ( DNB 2013J which she was treated with Paracetamol following
a. Glutamine which the fever subsided. Later she developed sei ¬
b. Glycine zures and altered sensorium. The urine examination
c. Serotonin revealed oxalate crystals on microscopy, Blood anion

Neurolgy
d. Dopamine gap and osmolality were increased. The diagnosis is:
Ref: Harrison’s 18/ e p3227
a. Lactic acidosis (AIIMS May 2002)
1 380. Koenen’s tumor is seen in: ( DNB 2013) b. Ethylene glycol poisoning
a. Neurofibromatosis c. Renal tubular acidosis
b. Tuberous sclerosis d. Paracetamol poisoning
c. Struge Weber syndrome Ref: H 18/ e p367, Harrison 19 th p 1858/ 64e- 9
d. Tuberculosis Ref: Harrison’s 18/ e p2360, 3384 387. A newly posted junior doctor had difficulty in finding
I 381. A 26 year old female presents with severe headache out base deficit/ excess for blood in a given patient. An
to emergency department. She is having similar experienced senior resident advised a quick method
episodes for the past 6 months. All the features favour to determine acid base composition of blood based on
a diagnosis of cluster headache in this patient except: pC02. Which of the following is the likely method he
suggested to predict acid base composition of blood?
a. Periodic pain
a. Red ford nomogram (AI 2004)
b. Conjunctival injection b. DuBio’s nomogram
c. Bilateral photophobia c. Goldman constant field equation
d. No response to oral sumatriptan d. Siggard -Andersen nomogram
Ref: H 18/ e 2012; pi 12, 122, Harrison 19th p 2594 -2595 Ref: Ganong 19/ e p703 fig 39.8
382. Single limb paralysis due to lesion in: 388. The initial fluid of choice of the treatment of hyper-
a. Area 4 ( NEET Pattern Question ) natremic dehydration is: ( MP PG 2009 )
b. Thalamus a. Normal saline
c. Brain stem b. N / 2 saline
d. Internal capsule Ref : Harrison 19th p 462 c. N / 4 saline
d. D / 4 saline+ 5% dexturose
383. Amaurosis fugax false is: ( NEET Pattern Question)
Ref: Harrison’s 18/ e p350-351; Harrison 19th p 304
a. Sudden loss of vision
b. Etiology is hemorrage from retinal artery 389. Best way to differentiate starvation acidosis and dia
c. Transient betic acidosis is: ( Kerala PG 09)
d. Hollenhorst plaque is seen a. History of obesity b. Leucocytosis
c. Hyperchloremia d. Bicarbonate level
Ref: Harrison’s 18/ e p230, 1307t
Ref: Lipplincott 3/ e p338, Harrison 19th p 320

Ans. 377. c. Altered mental... 378. c. Tolosa-Hunt Syndrome 379. d. Dopamine 380. b. Tuberous sclerosis
381. c. Bilateral photo... 382. a. Area 4 383. b. Etiology is hemo... 384. a . Defective gene...
385. e. Spine involve... 386. b. Ethylene glycol poi... 387. d. Siggard-Andersen... 388. a. Normal saline
389. d. Bicarbonate level
Neurology

390. Which finding favours diagnosis of MR rather than MS: 398. Left lobe of the brain is responsible for:
a. Loud SI ( PGI May 2013) a. Enjoying music ( NBE Pattern 2014/ 15)
b. Third heart sound b. Spatial orientation
c. Tapping apex c. Fine motor movement
d. Displaced apex beat d. Processing of speech Ref: Harrison's 18th P-202
e. Left ventricular hypertrophy
Ref: H 18/ epl 934, 1935, 1929, 1930, Harrison 19th p 270e- 18 399. Blink reflex is used for diagnosis of: ( AllMS Nov 2012)
a. Mid pontine lesions
391. Wernicke’s encephalopathy is not seen in? b. Neuromuscular transmission
(JIPMER 2014) c. Axonal neuropathy
a. Thalamus b. Mammillary body d. Motor neuron disease
C. Hippocampus d. Periventricular white matter Ref: Walsh and Hoyt’s Clinical Neuro Opthalmology Vol. 1 pg 1254
Ref : H 18th edn, Ch 275: Harrison 19th p 1783
400. True about critical illness myoneuropathy?
392. Good syndrome is?
a. Thymoma with immunodeficiency
( NEETPattern 2015 16 ) - (AllMS Nov 2013)
mechanical ventilation
a. Diaphragm atrophy due to
b. Thymoma with M. Gravis b. Cranial nerves are more commonly involved than
c. Thymoma with serum sickness peripheral nerves
d. Thymoma with pure red cell aplasia c. Peripheral nerves histology shows demyelination
Ref : Washington Manual of oncology, 2nd edn, p 172 d. Injury is completely irreversible.
393. DOC for Tourette syndrome? ( NEETPattern 2015-16 ) Ref: Harrison 19th p 2684

Neurolgy
a. Haloperidol
401. A chronic alcoholic for 15 years did not consume alco¬
b. Valproate
hol for religious reasons. On the first day he developed
c. B complex
nausea and vomiting and on second day seizures.
d. Clonidine Ref: Harrison 18th edn, Ch 372
What is the most likely treatment for this patient
394. Hung - up reflexes are seen in? ( NEET Pattern 2015-16 ) (AllMS Nov 2013)
a. Chorea b. Atheotosis a. Diazepam b. Sodium valproate
c. Cerebral palsy d. Cerebellar palsy c. Phenytoin d. Carbamazepine
Ref : Ch 591, Nelson 18th edn, OP Ghai 8th ed p 579 Ref: Harrison 19th p 2727
395. The triad of strabismus, diplopia and ptosis are caused 402. Cases of fatal hepatic toxicity have been reported
due to damage of which nerves: ( NBE Pattern 2014/15) requiring periodic monitoring of liver function tests
a. Oculomotor with: ( Bihar PG 2014)
b. Trochlear a. Tolcapone b. Entacapone
c. Abducent c. Rasagiline d. Ropinirole
d. Cervical sympathetic truk Ref: Harrison’ s 18th ch. 372, Harrison 19th p 2615
Ref: H 18/ e, pg 238, Harrison 19th p 2537 403. A 5 year old child presents with loss of vision and
396. A patient presents with ataxia. On examination ankle axial proptosis. Pupillary examination shows relative
and knee jerks are absent. Plantars are extensor. The afferent papillary defect. Probable diagnosis is?
diagnosis would be: ( AIPG 2010) a. Optic nerve glioma (APPG 2014 )
a. Friedreich’s ataxia b. Optic nerve sheath meningioma
b. SACD c. Retinoblastoma
c. Vitamin Bn deficiency d. Optic disc melanocytoma
d. Tabes Dorsalis Ref: Nelson Textbook of Pediatrics, ch. 620: Disorders of Vision
Ref H 18/ e, pg 3343, Harrison 19th p 2629 -2630 404. Inability to perform rapid alternating movements is
397 . Lateral Corticospinal injury is associated with all called: (APPG 2014 )
except: (AllMS Nov. 2012) a. Dysdiadochokinesia
a. Loss of superficial abdominal reflexes b. Dysarthria
b. Loss of gross movements of fingers and hands c. Dyssynergia
c. Babinski sign positive d. Dystonia
d. Clasp knife rigidity Ref Physicon by Sanoop KS Ref: Harrison 18th edn, Ch 373: Harrison 19th p 2626

Ans . 390 . b. Third heart sound 391 . c . Hippocampus 392. a . Thymoma with imm ... 393 . d . Clonidine
394. a . Chorea 395 . a . Oculomotor 396 . a . Friedreich ’s ataxia 397 . b. Loss of gross move...
398 . d . Processing ... 399 . a . Mid pontine lesions 400. c . Peripheral nerves... 401 . a . Diazepam
402. a . Tolcapone 403. a . Optic nerve glioma 404. a . Dysdiadochokinesia
th
Marwah's Internal Medicine MCQs ( Based on Harrison s 19 )

405. In Huntington’s Chorea the causative mutation in the 413. In which of the following vomiting is not associated?
protein Huntingtin is a: ( APPG 2014) a. Hypocalcemia ( NBE Pattern 2014/ 15)
a. Point mutation b. Hyponatremia
MH b. Gene deletion c. Diabetes Mellitus
c. Frame shift mutation d. Hypercapnia
WJM d. Trinucleotide repeat expansion Ref: H 18th Ch.39, Harrison 19th p 300, 2422, 2559
Ref: Harrison’s, ch. 372, Harrison 19th p 2621 414. Candle wax dripping sign is seen in:
406. The aqueous pressure is higher than intracranial pres¬
( NBE Pattern 2014/ 15 )
sure by about: ( Bihar PG 2014) a. Sarcoidosis b. SLE
a.1 to 5 mm Hg c HIV d. Rheumatoid arthritis
b. 6 to 10 mm Hg Ref Harrison’s 18th Ch:329, Harrison 19th p 2207
c. 15 to 18 mm Hg
415. Cause of death in Rett syndrome is:
d. More than 20 mm Hg ( NBE Pattern 2014 / 15 )
‘i 407 . In Wernicke’s encephalopathy amnestic defect is corre ¬ a. Arrhythmia b. Pneumonia
lated most closely with inflammation and necrosis of: c. Hypoglycaemia d. Birth asphyxia
a. Mamillary bodies (APPG 2014) Ref: Nelson 18th ch. 29.4
b. Midline cerebellum
c. Peri-acqueductal gray matter 416. Which is not correct regarding chronic fatigue syn ¬
d. Dorso-medial thalamus drome? ( NBE Pattern 2014/ 15 )
Ref: Harrison's 18th , ch. 275: Harrison 19th p 1783 a. Non tender lymph nodes

Neurolgy
b. Fatigue> 6 months
408. Which of the following does not require a lumbar c. Impairment of recent memory and intelligence
puncture in children? ( AIIMS Nov 2012) d. Myalgia /arthralgia
a. ALL
Ref: Harrison’s 18th ch:389, Harrison 19th p 464el
b. HL
c. NHL 417. Werner syndrome is characterized by all except:
d. AML Ref: 495.2 Nelson Acute leukemia and 496.2 a. Premature ageing ( NBE Pattern 2014/ 15)
b. Bird like facies
409. Inverse stretch reflex: ( Bihar PG 2014) c. Premature atherosclerosis
a. Is a monosynaptic reflex d. Pituitary adenoma
b. Is a bi-synaptic reflex
Ref Nelson 18th ch:90
c. Is a polysynaptic reflex
d. Has lower threshold than stretch reflex 418. Drug of choice for Hyper-ammonemia in urea cycle
Ref Harrison 18th edn, ch. 367 disorders is: ( NBE Pattern 2014/ 15)
a. Benzoic acid b. Sodium phenylbutyrate
410. Bilateral proptosis in children is seen in: c. Cysteamine d. lactulose
a. Neurofibromatosis ( AIIMS Nov 2012 )
Ref: Nelson 18th ed, Ch:85.11: Urea cycle defects
b. Leukemia
c. Cavernous hemangioma .
419 Kluver Bucy Syndrome has? ( NBE Pattern 2014/ 15)
d. Malignant fibrous Histiocytoma a. Hypersomnia b. Mood changes
Ref Nelson ch. 632/ Tumors of the Orbit
c. Hypersexuality d. Aggression
Ref: Olivier dulac, Harvery B. Sarnat, p 236
411. Ocular bobbing is seen in: ( NBE Pattern 2014/ 15)
a. Damage to M.L.F 420. All are true about Wolman Syndrome except:
b. Severe refractive error a. Autosomal recessive ( NBE Pattern 2014/ 15)
c. Pontine lesions b. Defect in cholestol ester transfer protein
c. Adrenal calcification
d. Cerebellum damage
d. Failure to thrive Ref Nelson 18th ch:86.4
Ref: Harrison's 18th Ch.11 -14, Harrison 19th p 1775
421. Pendular knee jerks are due to defect of:
412. Steroid are used in: ( NBE Pattern 2014/ 15) a. Ischemic stroke ( NBE Pattern 2014/ 15)
a. Severe typhoid b. Cerebral malaria b. Neocerebellum
c. E. coli septicemia d. H. influenzae meningitis c. Paleocerebellum
Ref: ch. 602, Nelson Textbook of Pediatrics 18th edn d. Archicerebellum

Ans. 405. d . Trinucleotide... 406. a. 1 to 5 mm Hg -


407. d. Dorso medial thala ... 408. b. HL
-
409. b. Is a bi synaptic... 410.
413. d . Hypercapnia 414.
b. Leukemia
a. Sarcoidosis
411. c. Pontine lesions 412. d . H. influenzae meningitis
415. b. Pneumonia 416. a. Non tender lymph nodes
417. d. Pituitary adenoma 418. b. Sodium phenylbutyrate 419. c. Hypersexuality 420. b. Defect in ch...
421. b. Neocerebellum
Neurology

422. Micturition reflex Centre is seen in: 430. A 25 year old man complains of syncope on shaving
( NBE Pattern 2014/ 15) without any sweating palpitations. Diagnosis is:
a. Barrington nucleus a T1 A ( NBE Pattern 2014/ 15)

B
b. Para -median pontine reticular pathway b. Carotid sinus hypersensitivity
c. Pulvinar c. Trigeminal neuralgia
d. Peri-ventricular nucleus d. Glossophyrangeal neuralgia
Ref: Autonomic Neurology, Wolfgang singes, Pg 8 Ref: CMDT 2013, Pg 977
423. Severe Hyperphosphatemia is not found associated 431. Muscle tone increasing on patient relaxing is seen in:
with? ( NEET/ DNB Pattern Questions) ( NBE Pattern 2014/ 15)
a. Hypocalcemia a. Paratonia b. Myotonia
b. Hypercalcemia c. Spasticity -
d Rigidity
c. Tetany Ref: CMDT 2013, Pg 997, Harrison 19th p 2574
d. QT prolongation ( NBE Pattern 2014/ 15)
432. Incorrect about Mannitol is:
Ref: HI 8th ch. 353, Harrison 19th p 313, 2469 Increases cerebral oxygen delivery
a.
424. Steroids are contraindicated in all except? b. Prolonged use leads to damage to blood brain bar¬
[ NBE Pattern 2014/ 15) rier
a. DM b. TB c. Exacerbates cerebral edema
c. Peptic ulcer d. Brain tumor d. Osmotic properties take 3 hours to develop
Ref: Harrison’s 18th ch 379, Harrison 19th p 598 Ref Goodman gilman 12th pg 748
425. All the following are complications of subarachnoid 433. Urine urge occurs at what bladder capacity?

Neurolgy
block except: ( NBE Pattern 2014/ 15) a. 50 ml ( NBEPattern 2014/ 15)
a. Post dural puncture headache b. 150 ml
b. Backache c. 250 ml
c. Hypertension d. 400 ml Ref: Harrison 19 th p 1778
d. Bloody tap 434. Which cranial nerve is involved in Tic Douloureux?
Ref Lee’s Synopsis of Anaesthesia, 12th/ e; Pg : 684-689 a. V ( NBE Pattern 2014/ 15)
426. During ultrasonography if the shape of cerebellum b. VI
shows “ banana sign" it suggests: c. VIII
( NBE Pattern 2014/15) d. IX
a. Anencephaly b. Hydrocephaly Ref: Harrison’s 18th ch: 372, Harrison 19th p 2623
c. Spina bifida d. Neoplasm 435. Argyl robertson pupil is seen in all except:
Ref: Neural tube defects: Diego E Wysyzinki, 2006 Pg 646 a. Chronic alcoholism ( NBE Pattern 2014/ 15)
427. Diencephalic pupils are due to damage to: b. Encephalitis
a. Superior colliculus ( NBE Pattern 2014/ 15) c. Diabetes mellitus
b. Lateral geniculate body d. Hypertension
c. Optic pathway Ref: 250 Cases in Clinical Medicine, pg. 38-39
d. Hypothalamus 436. Incorrect about Reye syndrome:
Ref Harrison's, 18th ch: 370 ( NBE Pattern 2014/ 15)
428. Dorello's canal carries: ( NBE Pattern 2014/ 15) a. Prothrombin time is increased
a. 3rd nerve b. 4th nerve b. Liver enzymes are elevated
c. 5th nerve d. 6th nerve c. Neuroglucopenia
Ref: Neuro-Oncology, Pg 136, 2011 ed d. Liver necrosis Ref OP. Ghai, 543, 7th ed
429. Drug used in amyotrophic lateral sclerosis: 437. Incorrect about cerebellar disease is:
a. Riluzole ( NBE Pattern 2014/ 15) ( NBE Pattern 2014/ 15)
b. Pramipexole a. Rhomberg sign
c. Rotigotine b. Rebound phenomenon
d. Ropirinole c. Dysaidokinesia
Ref: H 18th Ch: 374, Harrison 19 th p 2635 d. Dysmetria
Ref: Harrison's 18th ch. 373, Harrison 19th p 2539

Ans. 422. a. Barrington nucl... 423. b. Hypercalcemia 424. d . Brain tumor 425. d . Bloody tap
426. b. Hydrocephaly 427. d. Hypothalamus 428. d . 6th nerve 429. a . Riluzole
430. b. Carotid sinus... 431. a . Paratonia 432. d . Osmotic properties.. 433. b. 150 ml
434. a. V 435. d . Hypertension 436. d . Liver necrosis 437. a. Rhomberg sign
Marwah's Internal Medicine MCQs ( Based on Harrison s 19 )

438. Pin point pupils are due to damage to: 445. All are features of pyramidal tract lesion, Except .
a. Involuntary movement (AIIMS Sept 96 ]
a. Edinger Westphal nucleus ( NBE Pattern 2014/ 15) 's sign
b . Positive Babinski
b. Superior colliculus
c. Spasticity
c. Lateral geniculate body
d. Increased deep tendon reflexes
d. Descending sympathetic pathways
Ref : Harrison 18th/ p 181, 182, Harrison 19th p 438e
Ref: Harrison’s, 18th ch: 370, Harrison 19thpi 774
446. Which is pathgnomomc /or motor neuron disease
439. Therapeutic hypothermia is of benefit in preventing a. Fasciculation ( AIIMS Dec 95 )
neurological complications in: ( NBE Pattern 2014/15) b. Bladder, bowel involvement
a. Sepsis b. Poly-trauma c. Pseudohypertrophy
c. Cardiac arrest d. Ischemic stroke d. Sensory loss in patchy manner
Ref Harrison's 18th ch. 275 , Harrison 19 th p 1781 Ref: H 18th/ p 182, 3345, 3346, 3347, Harrison 19th p 2632
440. Inverse stretch reflex: ( Bihar PG 2014) 447 . Amyotrophic lateral sclerosis involve: ( PGI June 03)
a. Is a monosynaptic reflex a. Anterior horn cell b. Posterior horn cell
b. Is a bi-synaptic reflex c. Dorsal root ganglia d. Ventral root ganglia
c. Is a polysynaptic reflex e. Myoneural junction
d. Has lower threshold than stretch reflex Ref : Harrison 18th/ p 3345, Harrison 19 th p 438e
Ref Harrison 18th edn, Ch. 367: (AIIMS May 94 )
448. Motor neuron disease, TRUE is :
I 441. A female has episodic, recurrent headache in left hemi - a. Sensory involvement

Neurolgy
cranium with nausea and parasthesia on right upper b. Ocular motility is spared
and lower limbs is most probably suffering from : c. Involvement of anterior and lateral columns of
a. Migraine (AIIMS June 2000) spinal cord
b. Glossopharyngeal neuralgia d. Intellectual improvement
c. Herpes zoster infection of trigeminal Nerve Ref : Harrison 18th/ p 3346 -3347, Harrison 19th p 2632
d. Brain tumour 449. Beevor’s Sign is seen in: (AIIMS May 09 )
Ref : Harrison 18th/ p 114, 115, Harrison 19th p 2590 a. Abdominal muscle b. Facial muscle
442. A female aged 30, presents with episodic throbbing c. Respiratory muscle d. Hand muscle
headache for past 4 yrs. It usually involves one half of Ref : Harrison 18th / p 3367, Harrison 19 th p 2651
the face and is associated with nausea and vomiting. 450. All of the following are features of Friedreich’s Ataxia
There is no aura.Most likely diagnosis is: (AI 2001 ) Except: (AI 1992 )
a. Migraine a. Prograssive Ataxia is the most common presentation
b. Cluster headache b. Cardiomyopathy is a common association
c. Angle closure glaucoma c. Diabetes mellitus may be associated
d. Temporal arteritis d. Extensor plantar with brisk lower limb reflexes.
-
Ref .Harrison 18th/ p 114, 115, Harrison 19 th p 2590 Ref : Harrison 18th / p 2570, 3343, Harrison 19th p 2629
443. A 35 year old Lady Malti has unilateral headache, nau¬ 451. Pyramidal tract involvement with absent ankle jerk is
sea, vomiting and visual blurring. The diagnosis is: seen in : ( AIIMS May 01 )
a . Cluster headache (AIIMS June 99) a. Friedreich’s ataxia
b. Glaucoma b. Subacute combined degeneration of the spinal cord
c. Subarachnoid haemorrhage c. Lathyrism
d. Posterior fossa cyst. d. Tabes dorsalis
Ref : Harrison 18th/ p 114, Harrison 19th p 205 Ref : Harrison 18th/ p 3343, Harrison 19th p 2629
444. All of the following are features of Pseudobulbar pal¬ 452. Earliest presentation of Friedreich’s ataxia is-
sy, except: (AI 1991) a. Ataxia ( AIIMS Nov 93)
a. Dysarthria b. Dysphagia b. Seizures
c. Emotional lability d. Hypoactive jaw jerk c. Optic atrophy
-
Ref..Clinical Neurology 6 th/ 40, Harrison 19th p 438e d. Stuttering
Ref : Harrison 18th/ p 3343, Harrison 19th p 2629
Ans. 438. d . Descending sym... 439. c. Cardiac arrest
442. a . Migraine 443. b. Glaucoma
440.
444.
-
b. Is a bi synaptic reflex 441. a. Migraine
d . Hypoactive jaw jerk 445. a . Involuntary movement
446. a . Fasciculation 447. a. Anterior horn cell 448. b. Ocular motility... 449. a. Abdominal muscle
450. d . Extensor... 451. a. Friedreich ’s ataxia 452. a. Ataxia
Neurology

453. Absence seizures are seen in: (A11MS Dec 98) 461. Normal pressure Hydrocephalus is characterized by
a. Grand mal epilepsy b. Myoclonic epilepsy all except: (Al 1996) (Al 1997)
c. Petitmal epilepsy d. Hyperkinetic child a. Aphasia b. Dementia
Ref.: Harrison 18th / p 3251, 3252, Harrison 19th p 2543 c. Ataxia d. Urinary incontinence
454. Absence seizures are characterized on EEG by:
a. 3 Hz spike & wave
b. 1-2 Hz spike & wave
c. Generalized poly spikes
d. Hypsarrythmia
(Al 2003)

Ref : Harrison 18th/ p 3252, Harrison 19th p 2543


Ref : Harrison 18th/ p 3313, Harrison 19th p 2606
462. A 45- year-old man presents with history of frequent
falls. He has difficulty in looking down also. What is
the most probable diagnosis :
a. Normal pressure hydrocephalus
(ARMS Nov. 2000)
Q
b. Parkinson’s disease
455. The term post traumatic epilepsy refers to seizures c. Alzheimer’s disease
occurring (AIIMS Nov 02) d. Progressive supranuclear palsy
a. Within moments of head injury.
Ref : Harrison 18th/ p 3311, 3312, Harrison 19th p 2612
b. Within 7 days of head injury.
c. Within several weeks to months after head injury. 463. The CSF findings in TB meningitis are all the following
d. Many years after head injury except: CAl 1995)
a. Raised protein b. Low sugar
Ref : Textbook of Traumatic Brain Injury (2008/ 309) c. Low chloride d. High RBC count
456. All of the following drugs are used for managing sta ¬

Ref : Harrison's 18th/ 3414


tus epilepticus except : (ARMS May 06)

Neurolgy
a. Phenytoin 464. After a minor head injury a young patient was unable r

b. Diazepam to close his left eye and had drooling of saliva from left
c. Thiopentone sodium angle of mouth. He is suffering from:
d. Carbamzepine a. VIIth nerve injury (ARMS May 03) 1
b. Vth nerve injury
Ref : Harrison 18th/ p 3262, Harrison 19th p 2558
c. IIIrd nerve injury
457. The drug of choice for absence seizure (ARMS Nov. 06) d. Combined VIIth and IIIrd nerve injury
a. Valproate b. Gabapentin
c. Carbamezapine d. Phenytoin Ref : Harrison 19th p 2647
Ref : H 18th / p 3262, Harrison 19th p Table 445-8.2552 465. TRUE regarding upper motor neuron VHth nerve pa ¬

ralysis is: (ARMS Dec 95)


458. Transcranial Magnetic Stimulation of which part of the
brain has been shown to reduce frequency of symp ¬ a. Ipsilateral upper face paresis
toms in Parkinsonism: (Al 2012) b. Ipsilateral lower face paresis
a. Striatus c. Contralateral upper face paresis
b. Globus Pallidus Externus d. Contralateral lower face paresis
c. Subthalamic nucleus Ref: Macleods 10th/ 206, Harrison 19th p 2647
d. Putamen
466. Which of the following is not a component of Glasgow
Ref : H18th/ 3321, 3324, 3325, Harrison 19th p 2616 Coma Scale? (Al 2006)
459. Drug of choice in phenothiazine-induced Dystonia is: a. Eye opening b. Motor response
a. Diphenhydramine (Al 1996)
c. Pupil size d. Verbal response
b. Metoclopramide
c. Trifluperamide Ref : Fuller; Bailey Love 24th/ 601
d. Benztropine Refi KDT 5th/ 397 467. A head injured patient, who opens eyes to painful
460. All are true about Huntington’s disease except: stimulus, is confused and localizes to pain. What is
a. Chorea (AI 2001) the Glasgow coma score : (AIIMS Nov. 05)
b. Behavioral disturbance a. 7
c. Early onset of memory loss b. 9
d. Cog-wheel rigidity. c. 11
Ref : Harrison 18th/ p 3330, Harrison 19th p 2621 d. 13 Ref : Harrison 18th/ p 3381 (Table: 378.1 )

Ans. 453. c. Petitmal epilepsy 454 . a. 3 Hz spike & wave 455. c. Within several weeks 456. d. Carbamzepine
457. a . Valproate 458. c. Subthalamic nucleus 459. d. Benztropine 460. c. Early onset of memory...
461. a . Aphasia 462. d. Progressive supra ... 463. c. Low chloride 464. a. Vllth nerve injury
465. d. Contralateral low...466. c. Pupil size 467. c. 11
s 19th)
Marwah's Internal Medicine MCQs ( Based on Harrison'

474. Lesion in inferior frontal gyrus causes: (AllMS Dec 97)


.
468 Non-noxious stimuli perceived as pain is termed
ARMS May
as:
08) a. Defect in articulation
a. Allodynia (
b. Hyperalgesia b. Incomprehension of written language
c. Hyperesthesia c. Incomprehension of spoken language
d. Hyperpathia Ref.: Harrison 19th p 2643 d. Motor aphasia Ref : Harrison 19th pi 77
dlJM .
469 All of the following clinical findings are seen in Horn-
er’s syndrome, Except: (AllMS May 2011)
.
475 All are feature
a.
of Wernicke
Cogwheel rigidity (
’s encephalopathy
AllMS Feb 97 ) (
, Except:
AllMS Dec 95)
B L|
| a. Miosis b. Alteration in mental function
— b. Anhidrosis c. Ophthalmoplegia
c. Heterochromia of Iris d.
Ataxia
d. Apparent Exophthalmos
Ref : Harrison; 18th/ p 3313, Harrison 19th pi 782
Ref : Harrison 19th p 2651 .
476 Which of the following is not involved in Wernicke’s
.
470 Lesions of the lateral cerebellum cause all of the fol¬ Korsakoff psychosis: (AI 2012)
lowing, Except (AI 2010)
a. Mamillary body b. Thalamus
a. Incoordination b. Intention
c. Resting tremor d. Ataxia
tremor
c. Periventricular Grey matter
d.
Hippocampus
Ref : Harrison 18th Ch 373, Harrison 19th p 2626
Ref : Harrison 18th/ 2260; Harrison 19th pi 783
471. All the following are features of cerebellar disease 477. Memory impairment is most likely to occur in:
except: (AI 1996)
a. Down’s Syndrome (AllMS Nov 99)
a. Intention tremors b. Past pointing
b. Alkaptonuria c. Attention deficit disorder

Neurolgy
c. Hypertonia d. Ataxia
d. Conduct disorder
Ref : Guyton 10 th / , 656, Harrison 19th p 2626
655
.
472 Lesion in which of the following structure leads to
Ref : Harrison 18th/ p 3307, Harrison 19th p 2600
Kluver -Bucy syndrome: (AIIMSMay 04) 478. CSF glucose level is (ARMS June 2000)
a. Amygdala b. Hippocampus a. Half the plasma glucose
c. Hypothalamus d. Temporal lobe b. 2/ 3 plasma glucose
Ref : Guyton 10th/ 687 c. 1/ 3 plasma glucose
d. Same as plasma glucose
473. EEG shows: (NBE Pattern 2014/15)
a. Artifact Ref.: Harrison 19th p 888
b. Polyspike
c. Bi- frontal spikes
NEUROLOGY UPDATES
d. Generalized spike and slow wave
Ref: Harrison's 18th ch. e45, | Disorders Associated with Neuropathic Joint Disease |
Tabes dorsalis Congenital indifference to pain
Syringomyelia Leprosy
Meningomyelocele Peroneal muscular atrophy
Diabetes mellitus Amyloidosis

Neoplastic Meningitis
1 . Intrathecal chemotherapy, usually methotrexate , cytarabine, or thiotepa , is delivered by lumbar puncture or by an intraventricular
reservoir.
2 . An elevated CSF protein level is nearly always presents (except in HTLV- 1-associated adult T cell leukemia ).
3. Leptomeningeal seeding is frequent in patients under-going resection of brain metastases or receiving stereotactic radiotherapy for
brain metastases or receiving stereotactic radiotherapy for brain metastases.
4. Neoplastic meningitis can also lead to intracranial hypertension and hydrocephalus. Placement of a ventriculoperitoneal shunt may
effectively palliate symptoms in these patients.
5. Melanoma, breast and lung cancer, lymphoma (including AIDS-associated), and acute leukemia are the most common causes.
6. MRI findings suggestive of neoplastic meningitis include leptomeningeal, subependymal , dural , or cranial nerve enhancement :
superficial cerebral lesions; intradural nodules: and communicating hydrocephalus.

H|HAns. 468. a. Allodynia 469. d. Apparent Exophthalmos 470. c. Resting tremor 471. c Hypertonia
H|fl 472. a. Amygdala
476. d. Hippocampus
473. d. Generalized spike... 474. d. Motor aphasia 475. a. Cogwheel rigidity
477. a. Down ’s Syndrome 478. b. 2/ 3 plasma glucose
CH AP TE R Endocrinology

3
ENDOCRINOLOGY UPDATES
Diabetes Mellitus
diagno sed type 1 diabete s mellitu s .
> Diagnostic sensitivity and specificity of autoimmune markers in patients with newly

Sensitivity Specificity

ICA antibody 44-100% 96 %

70-90% 99%
Glutamic acid decarboxylase (GAD65)
99%
-
Tyrosine phosphatase ( IA 2 ) 50-70%
Zine transporter 8 (ZnT8) 50-70% 99%

Insulin ( IAA) 40-70% 99%

Most Recent Q's of 2014 - 2015 5. Which of the following is an example of high anion
gap acidosis? [ NBE Pattern 2014]
a. Renal tubular acidosis
1. A young lady with symptoms of hyperthyroidism with b. Diarrhea
elevated T4, has TSH levels 8.5. Further examination c. Ureterosigmoidostomy
reveals bitemporal hemianopia. Next step of d . Diabetic Ketoacidosis
management? [AUMS May 2015]
Ref: Harrison 19 th p 64 e - 9
a. Start anti - thyroid drugs, And do urgent MR 1 brain
b. Start beta blockers 6. Which of the following indicates malignancy in
c. Conservative management pheochromocytoma? [ NBE Pattern 2014 ]
d. Start anti -thyroid drugs and wait for symptoms to a. Abnormal mitotic figures
resolve b. Tumor necrosis
Ref: Harrison 19 th T 405 -7 p 2293 & 2296 c. Spindle cell morphology
d. Metastasis
2. In Galactorrhea amenorrhea syndromes, which is the
Ref : Harrison 19 th p 1613
investigation you should advise [apart from serum
prolactin]? [AUMS May 2015] 7. S1ADH is a known complication of TB meningitis.
a . TSH Which among the following is not true regarding
b. LH S1ADH? [ NBE Pattern 2014]
c. Urinary ketosteroids a. Decreased plasma osmolality
d. HCG Ref: Harrison 19 th p 2267 b. 1 lypervolemia
3 . IOC for insulinoma: [JIPMER 2015] c. Dilutional hyponatremia
a. MRI b. CECT d. Both a and b
c. Radionuclide scan d . EUS Ref: Harrison 19 th p 1259
Ref: Harrison 19th p 2338 8. In hemochromatosis, loss of libido and atrophy of
4. Patient with thyrotoxicosis. Eyeball protrud ed . testis occurs due to: [ NBE Pattern 2014]
Thyroid test done reveals? [JIPMER 2015] a. Deposition of iron in testis
a. T3 increased, T4 increased , TSH normal b. Deposition of iron in pituitary
b. T3 decreased, T4 decreased, TSH increases c. Deposition of iron in the thalamus
c. T3 increased, T4 increased, TSH decreased d. Reduced blood flow to testis
d . T3 decreased, T4 normal, TSH decreased Ref : Harrison 19 th p 2517
Ref: Harrison 19 th p 2294

Ans. 1 . a . Start anti -thyroid ... 2. a . TSH 3. d . EUS 4. d . T3 decreased , T4 normal . .


5 . d . Diabetic Ketoacid .. 6 . d . Metastasis 7 . b. Hypervolemia 8 . b . Deposition of iron in .. .
Marwah's Internal Medicine MCQs Based <h
)
( on Harriso n's 19

9. Drug used in both type 1 and type 2 Diabetes meliitus 10. Which of the following is not associated with
Diabete
is? [ DNB Pattern 2014] meliitus? (DNB Pattern __
a. Metformin a. Cushing's syndrome
b. Pramlintide b. Acromegaly

E3
.
c Coleselvam c. Hypothyroidism
.
d Panconeline Ref: Harrison 19th p 2409 d. Pheochromocytoma
Ref: Harrison 19th p 2405

ENDOCRINOLOGY UPDATES

> The Algorithm for the treatment of type 2 diabetes based on the recommendations of the consensus panel of the American
Diabetes Associate/ European Association for the Study of Diabetes.

Weight loss + exercise + metformin If HbA1c target not reached after - 3 months
Metformin + another agent If HbA1c target not reached after - 3 months
Metformin + two other agents If HbA1c target not reached after - 3 months

Endocrilgy
Metformin + more complex insulin regimen ± other noninsulin agent

Six main classes of agents


Metformin
Sulfonylureas (includes nateglinide, repaglinide)
Pioglitazone
GLP-1 receptor agonists
DPP-4 inhibitors
Insulins

> Factors in Therapeutic Decision (Single agent or combination)

Efficacy

-
DPP 4 inhibitors are of moderate efficacy
All other agents are of high efficacy

Hypoglycemic Risk
Sulfonylureas and insulins have increased risk of hypoglycemia

Effect on weight
Metformin and DPP-4 inhibitors are weight neutral
GLP-1 receptor agonists promote weight loss
Sulfonylureas, insulins, and pioglitazone are associated with weight gain

Major Side Effects


Metformin can cause lactic acidosis
Pioglitazone is associated with fluid retention, fracture risk, and possibly bladder cancer
GLP-1 receptor agonists are associated with nausea and vomiting and possibly pancreatitis
DPP-4 inhibitors may be associated with pancreatitis risk

Ans. 9, b. Pramlintide 10. c. Hypothyroidism


Endocrinology

Diabetes Mellitus c. Chromium


d. Zinc
11. Ideal investigation for adrenal insufficiency is:
a. ACTH stimulation
-
Ref . Harrison 18thedn, table 74.2 Harrison 19th p 96e-10
[ Manipal] 18. Not associated with diabetes mellitus:
b. Cortisol level estimation
c. Dexamethasone test a. Cushing syndrome [NEET Pattern 2015- 16 ] \
d. Urinary steroid estimation b. Acromegaly
Ref: Harrison 19th p 2323
c. Hypothyroidism
d. Phaeochromocytoma
12. An elderly male present with polyuria, polydipsia
and weight loss for 3 months. Which of the following Ref.: Harrison 18th,, Ch 344, p 2968, Harrison 19th p 2399
parameter is diagnostic of diabetes mellitus in this 19. Most common oral infection in diabetes mellitus?
patient? [JIPMER 2014] a. Candida [NEET Pattern 2015-16 ]
a. Fasting blood sugar 117 mg% b. Aspergillus
b. Fasting blood sugar 137 mg% c. Streptococcus
c. 2 hours blood sugar after OGT 180 mg% d. Staphylococcus
d . Urine dipstic + ++ [3+]
Ref . Harrison'sl 8th, Harrison 19 th p2399
-
Ref..Harrison 18th, Ch 344 Harrison 19th p 2429
20. GTT post 1 hour sugar for gestational diabetes is >
13. In the management of diabetic ketoacidosis: mg %?

Endocrilgy
a. Intracellular water deficit is best restored using half a. 140 b. 150
strength saline [0.45% saline] [J and K 2012] d . 200
c. 180
b. Potassium should be given even before checking the
serum potassium concentration Ref: William's obstetrics 24th edn, table 52.4
c. Bicarbonate infusion is often only necessary in 21. Aldose reductase inhibitor drugs are useful in?
severe acidosis pH < 7.0 a. Cataract [NEET Pattern 2015-16 ]
d. 5% dextrose solution should be avoided unless b. Diabetes mellitus
hypoglycaemia supervenes c. Hereditary fructose intolerance
Ref: Harrison's8/ e p2976, Harrison 19th p 318 d. Essential fructosuria
14. Which type diabetes is HLA associated: [AI 2002 ] Ref : Diabetic neuropathy clinical management p 319, 2nd edn
a. Type I diabetes 22 . Foot Ulcers in diabetes are due to all except?
b. Type II diabetes a. Decreased immunity [NEET Pattern 2015-16 ]
c. Malnutrition related type disease b. Neuropathy
d. Pregnancy related type diabetes
c. Microangiopathy
Ref: Harrison’s 18/ e p2973, , Harrison 19th p 2426 d . Macroangiopathy
15. Which of the following test is useful to distinguish Ref : Harrison's 18th, Ch 344: p 2987, Harrison 19th p 2428
between insulinoma and sulfonylurea related
hypoglycemia: [A1IMS May 08] 23. An obese lady with BMI = 35. FBG is normal and PPBG
a. Antibody to Insulin is slightly elevat ed . Ideal management is?
b. Plasma - C - peptide level a. Exercise [JIPMER 2014 ]
c. Plasma Insulin level b. Insulin
d . Insulin - Glucose ratio c. Metformin
Ref: Harrison' s 18/ e p3067, Harrison 19th p 569 d. Diet control
16. Antiepileptic of choice in Post diabetic peripheral Ref : Harrison 18th, ch 77, Harrison 19th p 2395
neuropathy: [DNB 2013] 24. Cause of death in diabetic ketoacidosis in children?
a. Pregabalin b . Amitryptylene a. Cerebral edema [ NBE Pattern 2014-15]
c. Carbamazepine d . Duloxetine b. Hypokalemia
Ref: Harrison' s 18/ e p2984, Harrison 19 th p 2548 c. Infection
17. Glucose intolerance is caused by deficiency of ? d. Acidosis
a. Selenium
b. Magnesium
[J1PMER 2014] .
Ref: Harrison's 18th ed. ch. 344 p 2978 79 Harrison 19th p 2418

Ans . 11 . a . ACTH stimulation 12. b. Fasting blood sugar... 13. c. Bicarbonate infusion... 14. a . Type I diabetes
15. None 16. a . Pregabalin ; d. Duloxetine 17. c. Chromium 18. c. Hypothyroidism
19. a. Candida 20. c. 180 21 . b. Diabetes mellitus 22. a. Decreased immunity
23. d . Diet control 24. a. Cerebral edema
Marwah's Internal Medicine MCQs ( Based on Harrison's 19th )

ENDOCRINOLOGY UPDATES
Laboratory diagnosis of coma in diabetic patients.

3 Related to diabetes
Hypoglycemia
Diabetic ketoacidosis
Hyperglycemic hyperosmolar state
coma
Urine Glucose

0
+++ +
++++
Acetone

0 or +
+ ++ +

0
Plasma Glucose

Low
High
High
Bicarbonate

Normal
Low
Normal or
slightly low
Acetone

0
+ ++ +

Lactic acidosis 0 or + 0 or + Normal or low or Low 0 or +


high

, 25. According to ADA guidelines, the diagnosis of diabetes 30. Diabetes is diagnosed when: [ NBE Pattern 2014 - 15]
is made when the fasting blood glucose is more than: a. The level of fasting glucose is > 100 mg / dL and that
a . 126 mg/ dl [ NBE Pattern 2014-15] of postprandial glucose is > 140 mg / dL

Endocrilgy
b . 100 mg/ dl b. The level of fasting glucose is > 126 mg/ dL and that
c. 140 mg/ dl of postprandial glucose is > 199 mg / dL
d. 200 mg/ dl c. The level of plasma insulin is >6 lU / dL
Ref: Harrison's 18"' ed. ch. 344, Diabetes Mellitusp 2968, d . The HbAlc level is > 5.5%
Harrison 19th p 2399 Ref: Harrison's 18" ed. ch. 344. pg. 2970. Table 344 -2. Harrison
26. Mauriac’s syndrome is characterized by all except: 19th p 2399
a . Diabetes [NBE Pattern 2014-15] 31. The glucose lowering effect is least and delayed by
b. Obesity several weeks with the following oral hypoglycaemic
c. Dwarfism
d. Cardiomegaly
agents
a. Insulin secretogogues
-
[ NBE Pattern 2014 15 ]

Ref: William's Textbook of Endocrinology. 12"' ed. P- 976: Normal b. DPP - IV inhibitors c. Biguanides
and Aberrant Growth] d. Alpha-Glucosidase inhibitors
27 . Which of the following is used in management of Ref: Harrison's 18th ed. ch. 344, Diabetes Mellitus p 2999,
diabetes? [ NBE Pattern 2014 - 15] Harrison 19th p 2416
a. Bromocriptine b. Octreotide 32. A diabetes mellitus patient presents with fungal
c. Prednisolone d . Pegvisomant -
infection of sinuses and peri orbital region with
Ref: Harrison's 18th ed. ch. 342, 346 , 351, Diagnosis and manage¬ significant visual impairment. Best treatment among
ment of Type 2 diabetes mellitus R.Henry, p 154 following is? [AllMS Nov 2012]
28.
a. Amphotericin B b. Itraconazole
All are short acting insulin, except:
c. Ketoconazole
a. Lispro [ NBE Pattern 2014-15]
d. Broad spectrum antibiotics
b. Aspart
c. Glulisine Ref: Harrison’s 18th ed. ch.205, Harrison 19th p 2429
d . Detemir 33. Consider 2 patients with Atherosclerosis, one is
Ref : Harrison’s 18" ed. ch. 344 p 2993, Harrison 19th p Table p -
diabetic and other is non diabetic. When compared
-
418 4, p 2411 -
to non diabetic, diabetic patient has 100 times
increased risk of: [AllMS Nov 2011 ]
29 . Chances of blindness in diabetic patient as compared
a. Myocardial infarction
to non -diabetic patient is? [ NBE Pattern 2014- 15]
b. Stroke
a. 5 times b. 10 times c. Lower limb ischemia
c. 15 times d. 25 times d. Vertebra basilar insufficiency
[ Ref. Harrison's 18" ed. ch. 344 p 2981, Harrison 19th p 2424 Ref: Harrison's 18th ed. P-2985, 2986, Harrison 19th p 2428

Ans. 25. a. 126 mg /dl 26. d. Cardiomegaly 27. a. Bromocriptine 28. d. Detemir
29. d. 25 times 30. b. The level of tasting glu... 31. c. Biguanides 32. a. Amphotericin B
33. c. Lower limb ischemia
Endocrinology

34 . Which is not seen in diabetic ketoacidosis: 39. Which one of the following statements about diabetes
a. Normal serum potassium [ NBE Pattern 2014-15] is not correct? [NBE Pattern 2014-15]
b. Plasma osmolality 380 mosm a. Insulin may be given subcutaneously in patient of
c . Urine Rothera test positive diabetic ketoacidosis
d. Urine Benedicts test positive b. Insulin antibodies are a hallmark for diagnosis of I
Ref: Harrison’s 18 ed. ch. 344 p 2977, Harrison 19th p 2416
“ insulin resistance
c. During hypoglycaemic episodes patient may I
35. Diabetes mellitus is present in ail except:
complain of difficulty in vision
a. Hemochromatosis [ NBE Pattern 2014-15] d. Positive test for ketone bodies in urine even after
b. Ataxia telengeictasia
patients of diabetic ketosis are treated.
c. Friedreich ataxia
d. Myotonic dystrophy Ref Harrison's 18 .
th ed ch 344 p 2968, Harrison 19th p 2404

Ref Harrison's 18 ' ed. ch. 344 p 2969, Harrison 19th p 2399 40. An obese patient presented in casualty with random
" blood sugar 400 mg%, urine sugar +++ and ketones j
36. Secondary diabetes may be noted in:
1+. Drug useful in management will be:
a. Acromegaly [ NBE Pattern 2014-15] a. Glibenclamide [NBE Pattern 2014-15] j
b. Addison's disease
b. Troglitazsone
c. Haemosiderosis
c. Insulin
d. Glucagonoma
.
d Metformin
Ref: Harrison's 18th ed. ch. 344 p 2969, Harrison 19th

Endocrilgy
Ref: Harrison's 18 . .
th ed ch 344 p 2977-78, Harrison 19th p 2419
Table 417-1, 2399
37. Diabetic ketoacidosis is associated with all except:
41. Retinopathy is most likely to be seen with:
a. IDDM of 5 years duration [NBE Pattern 2014-15]
a. t Utilization of glucose [ NBE Pattern 2014-15]
b. NIDDM of 8 years duration
b. t In protein catabolism
c. Gestational diabetes
c . t Anion gap
d. Juvenile diabetes started before puberty
d. Lipolysis
th
ed ch. 344 p 2976, Harrison 19th p 2418 Ref: Harrison's 18th ed. ch. 344 p 2981-82, Harrison 19th p 2424
Ref: Harrison' s 18
42. Insulin resistance is not seen in:[NBE Pattern 2014-15]
38. The following statements concerning diabetic keto ¬

a. Type 1.5 DM
acidosis are correct except: [ NBE Pattern 2014 -15 ]
b. Lipodystrophy
a. Abdominal pain c. Werner's syndrome
b. Low BUN d. Ataxic telangiectasia
c. Dehydration is out of proportion to the severity of
Ref: Harrison 's 18th
ed. ch. 344, Practical management of
vomiting
type 1 diabetes by steven edelman p 31
d. Low- dose insulin therapy is the treatment of choice:
Ref: Harrison' s 18 th
ed. ch. 344 p 2976, Harrison 19th p 2418

ENDOCRINOLOGY UPDATES
> Diagnostic criteria for insulinoma after a 72-hour fast.
Laboratory Test Result

Plasma glucose < 45 mg/dL

Plasma insulin (RIA) > 6 microunit/mL

Plasma insulin (ICMA ) 3 microunit/mL

Plasma C-peptide > 200 pmol/L (0.2 nmol/L)

Plasma proinsulin > 5 pmol/L

Beta-hydroxybutyrate 2.7 mmol/L


Sulfonylurea screen (including repaglinide and nateglinide) Negative

Ans. 34 . b. Plasma osmolal... 35. b. Ataxia telengeictasia 36. b. Addison’s disease 37. a. TUtilization of glucose
38. b. Low BUN 39. b. Insulin antibodies are a... 40. c. Insulin 41. a . IDDM of 5 years duration
42. a. Type 1.5 DM
Marwah's Internal Medicine MCQs (Based on Harrison's 19th)

ENDOCRINOLOGY UPDATES
> Treatment Goals for Adults with Diabetes

3
Index Goal

Glycemic control
HbA1C <7.0%
Preprandial capillary plasma glucose peak postprandial capillary plasma glucose 80-130 mg/dl _
<180 mg/dL

Blood pressure <140/90 mmHg


Lipids'
Low-density lipoprotein <100 mg/dL
High-density lipoprotein >40 mg/dL in men
>50 mg/dL in women
Triglycerides <150 mg/dL

43 . Hypoglycemic unawareness is because of: c. Insulin does not need change in ESRD
a.

Edocrinlgy
Shifting of oral hypoglycemics to insulin d. Add inhaled insulin to conventional administration
b. Insulin resistance [ NBE Pattern 2014-15]
Ref: Harrison's 18th ed. ch. 344, Harrison 19th p 2426
c. Autonomic neuropathy
48 . The most effective correction of acidosis in diabetic
d. Necrobiosis lipoidica
ketoacidosis is by: [ NBE Pattern 2014 -15 j
Ref: Harrison's 18* ed. ch. 344 p 2984, Harrison 19thp 2426 a. l.V. bicarbonate
44 . Oral anti- diabetic drug of choice in renal failure is: b. I.V. saline
a . Glyburide [ NBE Pattern 2014-15] .
c I.V. insulin
b. Chlorporamide d. Oral bicarbonate
c. Glipizide Ref: Harrison's 18th
ed. ch. 344,Harrison 19th p 2419
d. Metformin
49. In carcinoid syndrome, the part of heart mostl
Ref: Harrison's 18* ed. ch.344 p 2995, Harrison 19thp 2413 affected is: [ NBE Pattern 2014- 15
45 . A patient with DM of 4 years duration presents with a. Inflow tract of RV
dizziness and HR 52 /min, Probable cause is: b. Inflow tract of LV
a. Hypoglycaemia [NBE Pattern 2014-15] .
c Mural endocardium
b. Inferior wall MI d. Pericardium
c. Sick-sinus syndrome Ref: Harrison's 18* ed. ch. 350, Harrison 19th p 564
d. Autonomic dysunction
50 . Hypomagnesaemia presents with all except:
Ref: Harrison's 18* ed. ch. 344, Harrison 19th p 2427 a. Symptoms same as hypocalcaemia [NBE Pattern 20141 '
.
46 What is correct in diabetic ketoacidosis? b. Development of torsades de pointes
a. Low serum potassium [NBE Pattern 2014-15] c. Potentiates hypocalcaemia
b. Increased anion gap d. Seen in diabetic ketoacidosis
c . Metabolic alkalosis Ref: Harrison' s 18* ed. Harrison 19th Table 423-4 2462
d. Respiratory acidosis .
51 The complication of diabetes which cannot be
Ref: Harrison ’s 18* ed. ch .
344, Harrison 19th p 2418
prevented by strict control of blood sugar is:
.
47 Dose of insulin in diabetic nephropathy: a. Amyotrophy [ NBE Pattern 2014
a. Insulin dose should be increased in patient with b. Nerve conductivity
ESRD [NBE Pattern 2014-15] c. Fluorescein dye leak
b. Insulin dose should be decreased in patients with d. Microalbuminuria
ESRD Ref: Harrison's 18* ed. ch. 344, Harrison 19 th p 2423

Ans. 43. c. Autonomic neuro.. 44. c. Glipizide 45. b. Inferior wall Ml 46. b. Increased anion gap
ketoac .. .
47. b. Insulin dose shou.. 48. c. I.V. insulin 49. a. Inflow tract of RV 50. d. Seen in diabetic
51. c. Fluorescein dye leak
Endocrinology

52. Incorrect about gestational diabetes mellitus? c. Symptoms of Diabetes plus random blood glucose of [
a. Congenital malformations [NBE Pattern 2014-15] 190 mg/ dl
b. Metformin used d. Glycosylated Haemoglobin [HbAlC] > 6.5%
c. Prolonged labour Ref : Harrisons 18th/ 2970, Harrison 19th p 2399 I
d. Large for date baby
Ref: Harrison's 18th ed. ch. 344, GDM, Harrison 19thp 2400 59. The characteristic and common presentation of I
diabetic neuropathy is: [ PGI Dec 98] ||
53. Microalbuminuria refers to urinary albumin a. Amyotrophy
excretion rate of: [ NBE Pattern 2014-15] b. Mononeuropathy
a . 30-300 mg/ 24 hour c. Symmetrical sensory neuropathy
b. 400-600 mg / 24 hour d. Autonomic neuropathy
c. 700-900 mg / 24 hour Ref :Harrison 18th/ p 2984, Harrison 19th p 2426
d . > 100 mg/ 24 hour
60. Hypoglycemia is seen in [AIIMS May 01 ]
Ref: Harrison’ s 18th ed. ch. 344, Harrison 19thp 2425
a. Acromegaly b. Cushing's syndrome
54. Diabetes mellitus patient presents with HbAlC of c. Hyperthyroidism d. Hypopituotarism
9.6%. All improve with tight glycemic control except:
a. Neuropathy [NBE Pattern 2014-15]
-
Ref .Harrison 18th/ p 3007, Harrison 19th p 2257
b. Nephropathy Aldosterone Related Disorders
c. Retinopathy

Endocrilgy
d . Peripheral vascular disease
61. Which of the following is not seen in Secondary
Ref: Harrison' s 18th ed. ch. 344, Harrison 19th p 2423 Adrenal insufficiency? [ NEET Pattern 2015 -16 )
55. Necrobiosis lipoidica is seen in :[ NBE Pattern 2014-15] a. Pigmentation b. Postural hypotension
a. Diabetes insipidus c. Hypoglycemia d. Lassitude
b. Lyme disease Ref :Harrison 18th edn, Table 342.9, Harrison 19th p 2324
c. Diabetes mellitus 62. The most common cause of malignant adrenal mass
d . Simmonds disease is: ( .NBE Pattern 2014-15)
Ref: Harrison's 18th ed. ch. 344, Harrison 19th p 2429 a. Adrenocortical carcinoma
56. Which type diabetes is HLA associated: [AI 2002] b. Malignant Phaeochromocytoma
a. Type I diabetes c. Lymphoma
b. Type II diabetes d. Metastasis from another solid tissue tumor
c. Malnutrition related type disease -
Ref. Harrison's 18th ed. Harrison 19th p Table 406 5, 2321
d . Pregnancy related type diabetes 63. Primary hyperaldosteronism can be diagnosed by all
of the following criteria except: [ NBE Pattern 2014 -15)
Ref : Harrison 18th/ p 2973, Harrison 19th p 2403
a. Diastolic hypertension without edema
57. Which of the following is not a test for diabetes b. Metabolic acidosis present
mellitus [AIIMS Nov 2010]
c. Low plasma rennin activity that is not stimulated by
a. Fasting blood glucose volume depletion
b. Random blood glucose
d . Hyperaldosteronism which is not suppressed by
c. D-Xylose test volume expansion.
d. Oral Glucose tolerance test
Ref : Harrison's 17th/ 2275, 2277, Harrison 19th p 2410
.
Ref: Harrison' s 18th ed. ch 342.p 2950 Harrison 19th p 2319
64. Which of the following is the most common cause of
58. Which of the following findings can establish a Addison's disease in india? [ NBE Pattern 2014-15)
diagnosis of Diabetes Mellitus: [AIIMS Nov 2011]
a . Post-partum pituitary insufficiency
a. Fasting plasma glucose lOOmg/ dl and 2 hour b. Tuberculosis
prandial glucose 140 mg/ dl c. HIV
b. Fasting plasma glucose 125 mg/ dl and 2 hour d. Autoimmune adrenal insufficiency
postprandial glucose 199 mg/ dl
Ref: Harrison's 18th ed. ch. 342. pg. 2955, Harrison 19th p 2324

Ans. 52. -
a . Congenital malfo.. 53. a. 30 300 mg /24 hour 54. d. Peripheral vascular... 55. c. Diabetes mellitus
56.
60 .
a. Type I diabetes
d. Hypopituotarism
-
57. c. D Xylose test
61. a. Pigmentation
58. d. Glycosylated Haem... 59. c. Symmetrical sensory...
62. d . Metastasis from ano... 63. b. Metabolic acidosis present
64. b. Tuberculosis
Marwah’s Internal Medicine MCQs (Based on Harrison's 19th)

65. Adrenal reserve is best tested by means of infusion 73. All are features of Bartters syndrome, EXCEPT:-
with: (NBE Pattern 2014-15) a. Polyuria (AllMS Dec 95)
a. Glucocorticoids .
b ACTH b. Metabolic alkalosis
c. Hypothyroidism d. Metyrapone c. Periodic paralysis
d.
3
Hypertension
Ref: Harrison’s 18th ed. ch. 339, 342, Harrison 19th p 2310
66. Conn' s syndrome is characterized by all except: Ref. Harrison 19th p 295 , 306
a. Polyuria (NBE Pattern 2014-15) 74. Most common cause of adrenal insufficiency in India
b. Polydypsia is (AIIMS May 94 )
c. Weakness a. Autoimmune .
b Surgery
c. Steroid withdrawal d. Tuberculosis
d. Anasarca
Ref: Harrison's 18th ed. ch. 342, Harrison 19th p 2319 Ref: Harrison 19th p 2324
In Addison’s disease, most diagnostic test is:
67. Decreased plasma renin activity is seen in:
a. Serum Na+, K+, rennin (PCI Dec 97)
a. Primary hyperaldosteronismfJVBl? Pattern 2014-15)
b. Serum Na+, K+, saline suppression
b. Barter syndrome c. Serum creatinine / urea ratio
c. Pregnancy induced Hypertension d. ACTH stimulation test
d. Secondary aldosteronism Ref : Harrison 18th/ 2957, Harrison 19th p 2325
Ref: Harrison’s 18th
ed. ch. 342 p 2949, Harrison 19th p 2318

fcnaori gy
Cushing Syndrome
68. Incorrect about Addison’s disease is?
a. Hypoglycemia (NBE Pattern 2014-15) All of the following are true about Cushing's syndrome,
b. Hypokalemia [ PGI 08]
except:
.
c Loss of axillary and pubic hair
a. Red striae
d. Salt craving b. Increased adrenaline
Ref: Harrison’s 18th ed. ch. 342, Harrison 19th p 2324 c. Proximal muscle weakness
69. In Conn' s syndrome, all the following are seen, except: d. Edema
a. Hypokalemia (NBE Pattern 2014 -15) Ref: Harrison's 18/ e p2948, Harrison 19th p 2272
b. Hypernatremia 77. Obesity is seen in all except: [ NBE Pattern 2014-15]
c. Hypertension a. Cushing syndrome
d. Edema b. Pickwinian syndrome
c. Prader willi syndrome
Ref: Harrison's 18th ed. ch. 342, Harrison 19th p 2319 d. Sipple syndrome
70. Hypertension with hypokalemia is seen in following
except: (NBE Pattern 2014-15) Ref: Harrison 19th p 2340
78. Cushing syndrome is commonly caused by:
a. Cushing syndrome
a. Adrenal adenoma [ NBE Pattern 2014 - 15]
b. Liddle syndrome
c. End stage renal disease
b. Adrenal hyperplasia
c. Ectopic adrenal hormone production
d. Primary hyperaldosteronism
d. Adrenal carcinoma
Ref: Harrison’s 18th ed. ch. 280, Harrison 19th p 1817 ’ 18 th
ed. ch. 342 -1,
Ref: Harrison s
71. Conn' s syndrome is associated with all, except : 79 . Most common cause of Cushing' s syndrome is:
a. Hypertension (AI 2002 )
a. Pituitary adenoma [NBE Pattern 2014 -15 ]
b. Muscle weakness b. Adrenal adenoma
c. Hypokalemia
d. Edema c. Ectopic ACTH

Ref.:Harrison 18th/ p 2950, Harrison 19th p 2319 d. Iatrogenic steroids


Ref: Harrison’s 18th
ed. ch. 342 p 2897, Harrison 19th p 2314
72. Increased Aldosterone leads to all except: (AI 2009)
a. Hypernatremia b. Hypokalemia
c. Hypertension d. Metabolic acidosis
Ref : Harrison 18th/ p 2950, Harrison 19th p 2319
Ans. 65. b. ACTH 66. d. Anasarca 67 . a . Primary hyperaldoste.. 68 . b. Hypokalemia
69. d. Edema 70. c. End stage renal disease 71. d. Edema 72. d. Metabolic acidosis
73. d. Hypertension 74. d. Tuberculosis 75. d. ACTH stimulation test 76. b. Increased adrenaline
BO 77. d. Sipple syndrome 78. c. Ectopic adrenal... 79. d. Iatrogenic steroids
Endocrinology

80. Not seen in Cushing's syndrome: A/fi£


/ Pattern 2014-15] 86 . All of the following are true about Cushings syndrome,
a. Hypoglycemia b. Hypertension
c. Frank psychosis d. Hypokalemia excePl [PG108]
a. Red striae
Ref: Harrison’s IV ed. ch. 342 p 2897, Harrison 19
th p 2315 b. Increased adrenaline
81. A 28-year- old lady has put on weight 10 c. Proximal muscle weakness
[ kg over a
period of 3 years] and has oligomenorrhoea d. Edema
followed
by amenorrhoea for 8 months. The
blood pressure is Ref.:Harrison’s 18th/ 2945, 2946 , 2947,
160 / 100 mm of Hg . Which of the following
is the most Harrison 19th Table 406 -2, p 2315
appropriate investigation?
[AIPG 2011 ] 87 . A patient with cushinoid features presents with
a. Serum electrolytes
hemoptysis; he shows no response to dexamethasone
b. Plasma cortisol suppression test; most likely diagnosis here is:
c. Plasma testosterone and ultrasound a . Adrenal hyperplasia [AI 2001 ]
d. T3,T4 and TSH b. Adrenal adenoma
c. Ca lung with ectopic ACTH production.
Ref: Harrison' s 18‘" ed ch. 342, 339, Harrison 19th p 2315
d. Pituitary microadenoma
82 . A patient presents with hemoptysis and Cushingoid
Ref : Harrison 18th/ p 2946, 2947, Harrison 19th p 2315
features with a lack of dexamethasone suppression ,
the likely reason could be: 88. A chronic smoker presented with mild haemoptysis.
[ NBE Pattern 2014-15]

Endocrilgy
He also gave a history of hypertension and obesity.
a. Adrenal hyperplasia
Lab data showed raised ACTH levels, which were not
b. Adrenal adenoma suppressed by high dose dexamethasone. The cause
c. CA lung for the Cushing's syndrome in the patient is :
d. Ectopic ACTH production a. MEN I [AllMS May 02]
b. Pituitary adenoma
Ref: Harrison’s IV ed. p 2495 Table 342-1, Harrison 19th p 2315
c. Adrenal cortical adenoma
83. True about Cushing's syndrome is all except: d. Ectopic ACTH secreting tumor
[ NBE Pattern 2014 -15] Ref : Harrison 18th/ p 2946, 2947, Harrison 19 th p 2315
a. Association with the MEN 1 syndrome
89. Nelson's syndrome is most likely seen after:
b. Bronchial carcinoid causes cushing syndrome
a . Hypophysectomy [AllMS May 05]
c. Hypokalemia b. Adrenalectomy
d. Associated with coronary accidents c. Thyroidectomy
Ref Harrison' s IV ed. ch. 342 d . Orchidectomy
Ref .Basic Clinical Endocrinology [ Lange] 7th/ l 62; Harrison
84. Pseudo - Cushing syndrome is seen in :
[ NBE Pattern 2014 -15] 18th/ p 2899, Harrison 19th p 2273
a. Chronic alcoholism
b. Incidentaloma Disorders Related to Sex Steroids
c. Adrenal carcinoma
d. Nelson syndrome
90 . 40 year Male presents with primary infertility.
Ref Harrison's 18"' ed. ch. 342, Harrison 19th p 2316 Testis is present with azoospermia and absent vas
85. A common cause of Cushing's syndrome is: deferens [AI1MS Nov 14].
a. Cancer producing ectopic ACTH a. CFTR mutation
b. Pituitary adenoma [AIIMS May 93] b. , A.I.S
c. Adrenal tuberculosis c. Mullerian dysgenesis
d . None of the above d . Congenital adrenal hyperplasia
-
Ref .Harrison 18th/ p 2945 Harrison 19th p 2314 Ref : Nelson 18th and ch 259 Harrison 18th, Ch 400,
Harrison 19th p 1697

Ans . 80 . a. Hypoglycemia 81. b. Plasma cortisol 82. d. Ectopic ACTH produc.. 83. a. Association with the..
84. a . Chronic alcohol.. 85. b. Pituitary adenoma 86. b. Increased adrenaline 87. c. Ca lung with ectopic ACTH
88. d . Ectopic ACTH se.. 89. b. Adrenalectomy 90. a. CFTR mutation
th
Marwah s Internal Medicin
' e MCQs ( Based on Harriso n's 19 )

91. Normal height with absent pubertal features? 98. The diagnosis of a patient presenting with familial
a. Kallman syndrome [AllMS Nov. 14 ] Polyostosis, Precocious puberty and Pigmentation
b. Pure gonadal dysgenesis >s: [A1199S]
c. Testicular feminising syndrome a. Tuberous sclerosis

3 d. Turner syndrome b. McCune Albright syndrome


c. Klinefelter syndrome
Ref.: NELSON text book of Pediatrics 19th pno:l 941, 1944
d. SLE
92. Best for management of hypoglycemia in a girl child Ref : Harrison 18th/ p 3142, Harrison 19th p 2632
with Congenital adrenal hyperplasia ?
a . Betamethasone [ NBE Pattern 2014-15] Adrenal Medulla Disorders
b. Beclomethasone c. Budesonide
d. Hydrocortisone
.
Ref Harrison's 18th ed ch. 342, Harrison 19th p 2328
99. -
A 30 year -old male complains of loss of erection ;
he has low testosterone and high prolactin level in
93. -
Congenital 17 hydroxylase deficiency leads to blood; What is the likely diagnosis: [A12001]
hypertension due to accumulation of ? a. Pituitary adenoma
a . Deoxycorticosterone [ NBE Pattern 2014-15] b. Testicular failure
b. Cortisol c. Craniopharyngioma
c. 17 hydroxy pregnediol d. Cushing's syndrome
d. 17 hydroxy progesterone Ref: Harrison’ s 18/ e p2889, Harrison 19th p 2274

Edcrinolgy
,
Ref: Harrison’ s 18 h ed. ch. 342 Harrison 19th p 2328 100. All of the following are features ofpheochromocytoma
94. Salt losing type of adreno-genital syndromes is except: [Al 2002 ]
associated with: [ NBE Pattern 2014-15] a. Hypertensive paroxysm
a. Hypoglycemia b. Hypernatremia b. Headache
c. Hypertension d. Hypokalemia c. Orthostatic hypotension
[Ref : Nelson 18th ed., ch. 577, Harrison 19th p 2328 d. Wheezing
95. Gonadectomy is advised in: [ NBE Pattern 2014-15] Ref: Harrison's 18/ e p2963 Table 343-1, Harrison 19th p 2330
a. Kallman's syndrome 101. The predominant symptom / sign of pheochromo -
b . Testicular feminization syndrome cytoma is: [ DNB 2012 ]
c. Turner’s syndrome a. Sweating
d. Sexual precocity b. Weight loss
,
Ref: Harrison's 18 h ed. ch. 349, Harrison 19th p 2354 c. Orthostatic hypotension
96. A child presents with ambiguous genitalia without d . Episodic hypertension
hyperpigmentation and normal blood pressure, 2.5 Ref: Harrison’s 18/ e p2962, Harrison 19th p 2330
cm phallus with no opening at its tip, labia develop ed . 102. A 36 year old female with symptoms of
Gonads are not seen in inguinal region and Mullerian hyperparathyroidism, pituitary adenomas, islet cell
structures are present on USG. The most probable tumor with cutaneous angiofibromas. What is the
diagnosis is: [ NBE Pattern 2014-15] diagnosis? [AIIMS Nov 07]
a. AIS a. MEN 1 b. MEN 2 A
b. Maternal virilising tumor c. MEN 2 B d. MEN 2C
c. CAH Ref: Harrison's 18/ e p3073, Harrison 19th p 2339
d . 5-alpha -reductase deficiency 103. True about Sipple syndrome [MEN type 2]:
Ref: Harrison's 18th ed. pg.3054, Harrison 19th p 2362, 2366 a. MCT, Pheochromomcytoma, Mucocutaneous
97. A combination of gynaecomastia, decreased serum neuromas
testosterone and LH in a male patient is seen in: b. MCT, Pheochromocytoma, Parathyroid adenomas
a. Testicular failure [ AIIMS Dec 97] c. MCT, Pheochromocytoma, Panceratic tumours
b. Sertoli cell tumor d. MCT, Pheochromocytoma, Diabetes
c. Gonadotrophins Ref: Harrison' s 18/ e p3075, Harrison 19th p 2307
d. Androgen resistant state
Ref : Harrison 19th p 567, 2367

Ans. 91 . a. Kallman syndrome, b. Pure gonadal dysgenesis 92. d . Hydrocortisone 93. a. Deoxycorticosterone
94. a . Hypoglycemia 95. b. Testicular feminization ... 96 . b. Maternal virilising . 97 . b. Sertoli cell tumor
98 . b. McCune Albright. .. 99. a . Pituitary adenoma 100. d . Wheezing 101 . d . Episodic hypertension
102. a . MEN 1 103. a . MCT, Pheochromomcytoma ,
Endocrinology

104. Dose of clonidine in suppression test done in c. Episodic hypertension


pheochromocytoma is: [ DNB 2013] d. Weight gain

s
a . 0.3 mg b. 10 mg
c. 100 mg d. 200 mg
.
Ref: Harrison's 18th ed ch. 343 p 2963, Harrison 19th p 2329
Ref: Harrison' s 18/ e p2963, Harrison 19th p 1613 112. The residual form of phaeochromocytoma is treated
by: (NBE Pattern 2014-15)
105. Drug for management of hypertension
in a. Strontium
Phaeochromocytoma ? [ NEET Pattern 2015-16 ] b. Phosphorus
a . Phenoxybenzamine b. Phentolamine
c. Cobalt-60 d. MIBG
c. Labetalol d . Esmolol
Ref : Harrison's 18th, Ch 343, p 2963, Harrison 19th p 2331 Ref: Harrison's 18th ed. ch. 343 p 2964, Harrison 19th p 2331
106. In a case of Phaeochromocytoma, the diagnostic test 113. All the following drugs are used in pheochromocytoma
best avoided is: except: (NBE Pattern 2014- 15)
[ Bihar PC 2014]
a. MRI scan b. Urinary Metanephrines a. Prazosin
c. MIBG scan b. Atenolol
d. FNAC
c. Nitroprusside
.
Ref: Harrison s 18th ed. ch 343, phaeochromocytoma,
d. Metyrosine
Harrison 19th p 2330
107. Drug of choice for prenatal treatment of CAH due to Ref: Harrison' s 18th ed. ch. 343, Harrison 19th p 2330
21 alpha hydroxylase deficiency: 114. All are clinical features of pheochromocytoma, except:

Endocrilgy
a. Dexamethasone [ NBE Pattern 2014 -15] a. Increased hematocrit (AIIMSFeb 97 )
b. Betamethasone b. Orthostatic hypotension
c. Prednisolone c. Low cortisol level
d. Hydrocortisone d. Impaired glucose tolerance
.
Ref: Nelson Textbook of Pediatrics, 18th ed. Ch 577 - Congenital Ref : Harrison 18th/ p 2962, 2963, Harrison 19th p 2330
Adrenal Hyperplasia and Related Disorders
Disorders of Thyroid Gland
.
108 Which of the following is not found in
pheochromocytoma? [ NBE Pattern 2014 -15] 115. A 55 years old male patient underwent
a. Episodic hypertension
cholecystectomy for gall stone calculus. During
b. Postural hypotension
surgery the patient's pulse was irregularly irregular,
c. Increased hematocrit
d. Hypocalcemia =
160 / min, BP 80 / 50 mm of Hg, temp, 40°c. On
examination a swelling in the neck was found. Most
Ref Harrison's 18th ed. ch. 343 Harrison 19th p 2329 likely diagnosis is: [ Manipal ]
.
109 A patient with pheochromocytoma would secrete a. Thyroid storm b. Myocardial infarction
which of the following in a higher concentration? c. Pancreatitis d. SVT
a. Norepinephrine (NBE Pattern 2014 -15) Ref: Harrison 19th p 2297
b. Epinephrine
116. Treatment for childhood hypothyroidism is with:
c. Dopamine d . VMA
a. T4 [ Manipal ]
.
Ref: Harrison's 18th ed ch. 343, NANETS, Harrison 19th p 2329
b. T3
110. Pheochromocytoma is associated with: c. Levothyroxine
a . Vitiligo (NBE Pattern 2014 -15) d. TSH Ref Harrison 19th p 2289
b. Cafe -au-lait spots 117. The occurence of hyperthyroidism following
c. Ash leaf amelanotic macusles
administration of supplemental iodine to subjects
d. Acanthosis Nigricans
with endemic iodine deficiency goiter is known as:
Ref: Harrison's 18th ed. ch. 343 p 2964, Harrison 19th p 2331
a. Jod- Basdow effect [ Manipal]
111. In a patient with pheochromocytoma, all the following b. Wolff-Chaikoff effect
are seen except: (NBE Pattern 2014- 15) c. Thyrotoxicosis factitia
a. Diarrhoea d. De quervain's thyroiditis
b. Orthostatic hypotension Ref: Harrison 19th p 2300

Ans. 104. a. 0.3 mg 105. a. Phenoxybenzamine 106. d. FNAC .


107. a Dexamethasone
108. d . Hypocalcemia 109. a. Norepinephrine 110. b. Cafe au lait spots
- - 111. d. Weight gain
112. d. MIBG 113. b. Atenolol 114. c. Low cortisol level 115. a. Thyroid storm
116. c. Levothyroxine 117. a. Jod -Basdow effect
,
's 19 h )
Marwah's Internal Medicine MCQs ( Based on Harrison

118. The best marker to diagnose thyroid related disorders 124. All of the following are true about amiadaro
[ Manipal ] induced thyroid dysfunction except? iA! 2

a
is:
a. T3 a. Hyperthyroidism is common in iodine defici
cnt
b. T4 areas
c. TSH b. Hypothyroidism is more common in men
d. Thyroglobulin c. Amiodarone inhibits deiodinase activity
d. Amiodaraone therapy is associated with initial
119. An adolescent girl presented symmetrical, red, reduction of serum T4 levels
tender swellings in shin and arthralgia. X- ray reveals
Ref: Harrison' s 18/ e p2929, Harrison 19thp 229i
hilar and paratracheal lymph node. She is clinically
suspected to have sarcoidosis. Next step in the 125. Decreased radio iodine uptake is / are seen in [select
diagnosis is: []IPMER 2014] two options]: [ PCI Dec 2000]
a. Mediastinal lymph node biopsy a. Toxic multinodular goiter
b. 24 hours urine calcium estimation b. Grave's disease
c. CT thorax c. Subacute thyroiditis
d. Follow up with 3 monthly CXR d. Factitious thyroiditis
Ref: Harrison 19th p 2205 Ref: Harrison's 18/ e p2917, Harrison 19th p 2299
120. Which of the following antibodies is involved in 126. Pancreatitis, pituitary tumor and pheochromocytoma
the tissue destructive process associated with may be associated with: [ Al 2004, 2005]

Endcriolgy
hypothyroidism in Hashimoto's and atrophic a. Medullary carcinoma of the thyroid
thyroiditis? [Corned K 2010] b. Papillary carcinoma of the thyroid
a. Thyroperoxidase antibody c. Anaplastic carcinoma of the thyroid
b. Thyroglobulin antibody d. Follicular carcinoma of the thyroid
c. TSH receptor antibody Ref: Harrison’ s 18/ e p3072, 3073, Harrison 19th p 2342
d. Thyroid stimulating antibody
127. Which of the following is associated with
Ref: Harrison’s 18/ e p2920 Davidson's 20 / e p757, Bailey 25/e
hypothyroidism in sub Himalayan region : [ DNB 2013]
p776, Harrison 19th p 441e-29 f
a. Cu
121. Hypothyroidism may be caused by: [DNB] b. Fe
a. Lithium c. Zn
b. Hematochromatosis d . Se
c. Scleroderma Ref: Harrison's 18/ e p604, Harrison 19 th p 2290
d. All of the above 128. Proptosis is not seen in? [ NEET Pattern 2015- lb ]
Ref: Harrison's 18/ e p2918 Table 341 -4, Harrison 19th p 2289
a. Grave's disease
122 . Which of the following is not associated with b. Sarcoidosis
hypothyroidism: [ DNB]
c. Pituitary adenoma
a. Low T3
d . Myxoedema
b. High TSH
c. High Triglycerides Ref : Harrison's 18th, Ch 341, p 2920, 2923,
d. Low cholesterol Harrison 19th p 207, 2294
Ref: Harrison’s 18/ e p2920, Harrison 19th p 2289 129. The drug used in the management of medullary
carcinoma thyroid is: [ Bihar PC 20141
123. The lab investigation of patient shows |T 3, J T4, and
.

[TSH . It cannot be : [A12000] a. Cabozantinib


a. Primary hypothyroidism b. Rituximab
b. Pan -hypopituitarism c. Tenofovir
c. Liver disease d. Anakinra
d. None of the above
.
Ref Harrison's 18th ed. Ch 351 disorders affecting multiple endo ¬

Ref: Harrison's 18/ e p2916, 2921, Harrison 19th p 2289 crine systems, Harrison 19th p 2342

Ans. 118. c. TSH 119. a. Mediastinal lymph .. 120. a . Thyroperoxidase... 121. d. All of the above
122. d. Low cholesterol 123. a. Primary hypothyroidism 124. b. Hypothyroidism is.. .
125. c. Subacute thyroiditis, d. Factitious thyroiditis 126. a . Medullary carcinoma...127. d. Se
84 128. d. Myxoedema 129. a. Cabozantinib
Endocrinology

130. All are true about Hashimoto encephalopathy except: c. TSH level is not a good marker for diagnosis
a. Myoclonus [NBE Pattern 2014 - 15]

a
d. Before starting treatment checkout for adrenal
b. Seizures insufficiency
c. Steroid responsive encephalopathy Ref Harrison's 18th ed. ' Principles of Internal Medicine'; pg
d. EEG is normal 2901 -2902, Harrison 19th p 2288
Ref: Harrison’s Iff 1' ed.ch. 341 p 2920, Harrison 19th p 2291 137. Thyroid storm is seen in: [NBE Pattern 2014-15]
131. DOC for Hashimoto encephalopathy: a. Thyroid surgery
a. Steroids [ NBE Pattern 2014-15] b. Neonatal thyrotoxicosis
b. Prophythiouracil c. Peri-operative infection
c. 1-131 d. All of the above
d. Liothyronine infusion Ref Harrison 19th p 2265

.
Ref: Harrison's 18th ed. ch 341, Harrison 19th p 2291 138. Hypothyroidism is associated with the following
132. Common neurological manifestations of thyrotoxico¬ clinical problems, except: [NBE Pattern 2014-15]
sis include all except: a. Menorrhagia b. Early abortions
a. Hyper reflexia [NBE Pattern 2014-15] d. Thromboembolism
c. Galactorrhoea
b. Muscle wasting
Ref Harrison' s 18th ed.ch. 341 p 2920, Harrison 19th p 2290
c. Chorea
139. Pemberton sign is seen in ?

Edcrinolgy
d. Proximal myopathy without fasciculations [ NBEPattern 2014-15]
Ref: Harrison's 18th ed. ch. 341 p 2919, Harrison 19th p 2294 a. Retrosternal goiter b. Grave opthalmopathy
133. A 17- year old girl who was evaluated for short height c. Thyroid crisis d. Addisonian crisis
was found to have an enlarged pituitary gland. Ref Harrison's 18th ed. ch. 341 pg 2916, Harrison 19th p 2301
Her T4 was low and TSH was increas ed. Which of the 140. Which of the following are not related to Myx-edema?
following is the most likely diagnosis? a. Coronary atherosclerosis [APPG 2014]
a. Pituitary adenoma [ NBE Pattern 2014-15]
b. Type III hyperlipoproteinaemia
b. TSH -secreting pituitary tumour
c. Thyroid target receptor insensivity c. Massive Pericardial effusion
d. Primary hypothyroidism d . Absence of pulsus paradoxus
Ref: Harrison's 18th ed. p 2916, Harrison 19th p 2287, 2292 Ref: Harrison’ s 18th ed.ch. 341 pg 2920-21, Harrison 19th p 2291
134. The third generation TSH detection tests can detect 141. TSH cannot be used for monitoring response to
TSH at a minimum level of: [ NBE Pattern 2014 - 15] treatment in: [ NBE Pattern 2014-15]
a. 0.4 mU / L b. 0.04 mU / L a. Primary hypothyroidism
c. 0.004 mU / L d. 0.0004 mU / L b. Secondary hypothyrodism
Ref 18th ed, pg. 2917, Harrison 19th p 2288 c. Thyroprivic hypothyroidism
135. The occurrence of hyperthyroidism following d. Iodine deficiency
administra - tion of supplemental iodine to subjects in Ref. Harrison’s 18th ed.ch. 341 pg 2918, Harrison 19th p 2288
endemic area of iodine deficiency is due to? 142. Cardiac manifestations of Grave's disease would
a. Wolf-Chaikoff effect [ NBE Pattern 2014-15]
include all of the following except?
b. Jod- Basedow effect a. Wide pulse pressure [ NBE Pattern 2014-15]
c. Pemberton effect b. Atrial fibrillation
d. Graves' effect c. Pleuropericardial scratch
th ed. ' Principles of Internal Medicine'
Ref: p 2930 Harrison’s 18
d . Aortic insufficiency
ch. 341 Harrison 19th p 2300
Ref Harrison’ s 18th ed.ch. 341 p 2923, Harrison 19th p 2294
136. All are true about Central hypothyroidism except?
a. Craniopharyngioma [AIPG 2012]
b. Treatment is done according to TSH level in plasma

Ans. 130. d . EEG is normal 131. a. Steroids 132. c. Chorea 133. d . Primary hypothyroidism
134. c. 0.004 mU / L -
135. b. Jod Basedow effect 136. b. Treatment is done...
140. b. Type III hyperlipop...
137. d . All of the above
141. b. Secondary hypothyrodism
.
138. d. Thromboemboli. 139. a. Retrosternal goiter
142. d . Aortic insufficiency
th )
Marwah's Internal Medicine MCQs ( Based on Harrison s 19
'

143. Replacement dose of Thyroxine per day is? a. Lid retraction


a. lOOmcg [ NBE Pattern 2014-15] b. Frequent blinking
b. 200 mcg c. Poor convergence
c. 300 mcg d. Upper lid "lad" on down gaze
, .
ed ch. 341 p 2924, Harrison 19thp
I'
h
d. 400 meg Ref Harrison' s 18 2294
Ref: Harrison's 18th ed.ch. 341, Harrison 19th p 2292 151. Which is not seen in subacute thyroditidis:
a. Raised T4 levels [ NBE Pattern 2014-15]
M 144. Characteristic of Graves disease are A / E:
b. Raised ESR
a. Hyperthyroidism [ NBE Pattern 2014-15]
b. Pretibial myxoedema c. Pain
c. Atrial fibrillation d. High Radio iodine uptake
d. Ophthalmopathy Ref Harrison’s 18th ed. ch. 341 p 2917, Harrison 19th p 2299
Ref: Harrison' s 18th ed.ch. 341, Pg. 2922, Harrison 19th p 2294 152. Radioiodine uptake in endemic goitre is:
145. Drugs used in thyroid crisis are A / E: a. Normal -
[NBE Pattern 2014 15]
a. Propranolol [NBE Pattern 2014-15] b. Increased
b. Carbimazole c. Decreased d. Erratic
Ref Harrison's 18 ’ ed. ch. 341 p 2917, Harrison 19th p 2288
c. Iodine “
153. Hung up ankle jerk is seen in? [ NBE Pattern 2014-15]
d . Corticosteroids

Edcrinolgy
Ref Harrison's 18 ' ed. ch. 341 pg. 2927, Harrison 19th p 2297 a. Hypothyroidism b. Thyrotoxicosis
“ c. Sipple syndrome d. Wermer syndrome
146. Most common presentation of sick euthyoid state:
a . Low T 3 with normal T4 [ NBE Pattern 2014-15] Ref Harrison ' s 18th
ed. ch. 341, Harrison 19th p 2291
b. Low T 3 with low T4 154. Which one of the following features may not be seen
in hypothyroidism? [NBE Pattern 2014 -15]
c.Low T3 with highT4
a. Cold intolerance b. Deafness
d. High T3 with high T 4
c. Pericardial effusion d. Pretibial myxoedema
Ref Harrison's 18A ed.ch. 341 p 2915, Harrison 19th p 2299
Ref Harrison's 18th ed. ch. 341 p 2923, Harrison 19th p 2291
.
147 In myxoedema which is not correct:
[NBE Pattern 2014-15] 155. Goitrous hypothyroidism commonly occurs in all of
a. Slow pulse
the following except? [ NBE Pattern 2014 - 15]
b. Hypertension
a. Hashimoto's thyroiditis
c. Hypotension
b. Dyshormonogenesis [Pendred syndrome]
d. Dry skin
c. Thyroprivic hypothyroidism
Ref Harrison’s 18th ed. ch. 341 pg 2919, Harrison 19th p 2291
d. iodine deficiency
148. Thyroid carcinoma with pulsatile vascular skeletal
Ref Harrison's 18 ed. ch. 341, p 2918, Harrison 19th p 2290
metastasis is seen in: [ NBE Pattern 2014- 15] ’ “
a. Papillary b. Follicular 156. Presentation of hypothyroidism is?
c. Medullary d. Anaplastic a. Pretibial myxedema [NBE Pattern 201
b. Hirusutism
Ref Harrison' s 18th
ed. ch. 341 pg 2936, Harrison 19th p 2305
c. Easily brusiable skin
149. Myxoedema coma is treated with: d. Galactorrhoea
a. Hydrocortisone [ NBE Pattern 2014-15]
b. Liothyronine
.
Ref Harrison's 18th ed ch. 341 p 2920, Harrison 19th p 2291
157. Gestational hyperthyroidism occurs due to:
c. Levothyroxine a. Beta HCG from placenta [ NBE Pattern 20
d. All of the above b. Transplacental TSH transfer
Ref Harrison’s 18th ed. ch. 341, pg 2922 Harrison 19th p 2293 c. T.P.O antibody
150. Which one of the following clinical signs is not seen in
d. Anti-thyroglobulin antibody
ophthalmic Grave's disease? [ NBE Pattern 2014- 15] Ref: Harrison's 18th ed. ch. 341 Thyroid function in pregnancy,
Harrison 19th p 2300
Ans. 143. a. 100mcg 144. c. Atrial fibrillation 145. b. Carbimazole 146. a. Low T3 with normal T4
147. c. Hypotension 148. b. Follicular 149. d . All of the above 150. b. Frequent blinking
151. d. High Radio iodi... 152. b. Increased 153. a. Hypothyroidism 154. d. Pretibial myxoedema
155. a . Hashimoto’s thyr...156. d. Galactorrhoea 157. a . Beta HCG from placenta
Endocrinology

165. All the following are true of DeQuervan's Thyroiditis


except

s
[AI 1996 ]
a. TSH b. Free T4 a . Pain
c. T3 d. Free T 3 b. Increased ESR
Ref: Harrison's 18'* ed. Thyroid Gland, c. Increased radioactive iodine uptake
Harrison 19th p 2288 d. Fever
159. Which is true about Thyrotoxic Periodic
paralysis. Ref : Harrison 18th/ p 2928, Harrison 19 th 2298
a . Sodium channel defect [ NBE Pattern 2014-15]
b. Hypokalemic periodic paralysis 166. All cause weight gain, except : [AIIMS Sept 96 ]
c. Precipitated by fasting a. Diabetes mellitus b. Cushing's syndrome
d. Associated with myxedema coma c. Hypothyroidism d. Insulin secreting tumour
Ref: Harrison's 18th ed. ch. 383, Harrison 19th p 444 Ref : Harrison 18th/ p 2963, 233
e
160. The most common differential diagnosis Disorders of Parathyroid Gland & Calcium
of
hyperthyroidism in a young female is: Metabolism
a. Hysteria [NBE Pattern 2014-15]
b. Essential tremor
167. A 10 -year- old boy has a fracture of femur. Biochemical
c. Anxiety neurosis evaluation revealed Hb 11. 5 gm / dl and ESR 18 mm 1
d. Parkinsonism hour, serum calcium 12.8 Mg/ dl, serum phosphorus

Endocrilgy
Ref: Harrison's 18th ed. ch. 341, Harrison 19 th p 2294 2.3 mg/ dl , alkaline phosphatase 28 KA units and
blood urea 321mg/ dl . Which of the following is the
161. Which of the following conditions is associated with most probable diagnosis in his case? [ Manipal]
Hypothyroidism: [AI 2011 ] a. Nutritional rickets
a. Hoshimoto's Thyroiditis
b. Renal rickets
b. Grave's Disease
c. Toxic Multinodular Goiter c. Hyperparathyroidism
d. Struma ovary d. Skeletal dysplasia
Ref .Harrison 18th/ p 2918, 2919, Harrison 19th p 2290 Ref Davidson pg 702 Harrison 19 th p 1815

162 . All of the following are features of thyrotoxicosis, 168. Secondary hyperparathyroidism is seen in all of the
EXCEPT- [AIIMSJune 99 ] following, except: [AIIMS Nov 2010]
a. Diastolic murmur a. Rickets b . Osteomalacia
b. Soft non ejection systolic murmur c. Osteoporosis d. Renal failure
c. Irregularly, irregular pulse Ref: Harrison 18th/ p362, Harrison 19 th p 426 e - 4
d. Scratching sound in systole
169. AH of the following statements about
Ref : Harrison 18th/ p 2923, Harrison 19th p 2294 pseudohypoparathyroidism are true, except:[AI 2010]
163. All of the following are associated with Thyroid storm, a. I Serum PTH
except [A12002] b. I Serum calcium
a . Surgery for thyroiditis c. |Serum phosphate
b. Surgery for thyrotoxicosis d. Albright's hereditary osteodystrophy
c. Stressful illness in thyrotoxicosis Ref: Harrison's 18/ e p3117, Harrison 19 th p 430, 2485
d . 1131 therapy for thyrotoxicosis
170. Which of the following statements about
Ref : Bailey and Love 24th/ 796; Harrison 18th/ p 2927,
Harrison 19th p 2297 Pseudohypoprathyroidism is true: [AI 2011 ]
a. Caused by 'gain of function' inherited mutation in
164. Most common cause of Thyroiditis is: [AI 2000]
Gsa subunit
a . Reidl's Thyroiditis b. Decreased formation of cyclic GMP is observed
b. Subacute Thyroditis c. Decreased formation of Inositol triphosphate is
c. Hashimoto's Thyroiditis
observed
d . Viral Thyroiditis
d. Decreased formation of c-AMP is observed
Ref :Harrison 18th/ p 2929, Harrison 19 th p 2290
Ref: Harrison's 18/ e p3117, Harrison 19 th p 2485

Ans. 158. a . TSH 159. b. Hypokalemic periodic... 160. c. Anxiety neurosis 161. a. Hoshimoto’s Thyroiditis
162. a. Diastolic murmur 163. a . Surgery for thyroiditis 164. c. Hashimoto’s Thyro... 165. c. Increased radioactive
166. a. Diabetes mellitus 167. c. Hyperparathyroidism 168. c. Osteoporosis 169. a. I Serum PTH
87
170 . d . Decreased formation of c-AMP is observed
Marwah's Internal Medicine MCQs ( Based on Harrison's 19th )

171. Most common cause of hypercalcemic crisis is: 178. Which ion channel is affected in hypokalemic periodic
a. Carcinoma breast [ AllMS May 93] paralysis: [ DNB 20131
b. Parathyroid hyperplasia a. K* b. Na*
c. Parathyroid adenoma c. CP d . Ca2*
Qjl d. Paget's disease Ref: Harrison's 18/ e p3504, Harrison 19 th p 307
R|j| Ref: Harrison's 18/ e p3106, 3107, Harrison 19 th p 2481 179. Excess of calcium intake leads to: [ARMS Nov. 14 j
II 172 . Which of the following is not a feature of a. Cardiomyopathy b. Osteoporosis
hypercalcemia: [AI 06 ] c. Osteomalacia d. Milk alkali syndrome
a. Diarrhoea b. Polyuria Ref.: Harrison 18th p 3110, ch 353-6, Harrison 19 th p 2479
c. Depression d . Vomiting 180. A middle aged female has a pathological fracture of
Ref Harrison’ s 18/ e p3100, Harrison 19 th p 2481 clavicle, ribs and X- ray shows periosteal resorption of
2 nd and 3 rd metatarsals and phalanges. Most probable
| 173. Treatment of hypercalcemia [select two options]:
cause is? [ARMS Nov. 14 ]
a. Calcitonin [ PGI Dec 04]
b. Gallium nitrate a. Hyper-parathyroidism b. Hypo- parathyroidism
c. Renal Osteodystrophy d. Osteomalacia
c. Orthophosphate
d . Thyroxin Ref.:Harrison 18th p 3100, ch 353, Harrison 19th p 2472
Ref Harrison's 18/ e p3113, Harrison 19 th p 2480 181. Features of hyperparathyroidism are all except?

Endocrilgy
174. A 75 year old lady with fracture neck of femur a. Solitary adenoma [ NEET Pattern 2015- 16]
presents with two days history of altered sensorium b. Malignant
and decreased urinary output. Serum calcium is 15.5 c. Thyroid malignancy
mg/ dL, Urea is 140 mg/ dL, Creatinine is 2 mg / dL. All d . Parathyroid hyperplasia
of the following are useful for immediate management Ref : Harrison 18th, Ch 353: pg 2936, Harrison 19th p 2472
of hypercalcemia, except: [ARMS Nov 2010] 182. Which of the following statements is not true?
a. Normal saline b. Furosemide
[ARMS May 2013]
c. Dialysis d . Bisphosphonates
a. Parathyroid hormone-related protein is responsible
Ref Harrison's 18/ e p3112, Harrison 19 th p 2481 for causing hypercalcemia in cancer patients.
175 . Hypophophatemia is seen in: [ PGI Dec 02] b. The unionized fraction of calcium in the plasma is an
a. Pseudohypoparathyroidism important determinant of PTH secretion.
b. CRF c. Mg2* lninfluences PTH secretion in the same direction
c. Rickets as Ca2*, but is a less potent secret-agogue.
d . Hyperparathyroidism d . Ca 2* influences PTH secretion by acting on a calcium
e. Respiratory acidosis sensor G-protein coupled receptor located in the
Ref Harrison’s 18/ e p3087, Harrison 19 th p 1815 parathyroid gland .
Ref Ganong's Review of Medical Physiology’; 22/ e, pg 392
176. Which of the following is not seen in Vitamin D
deficiency: [PGI Dec 01 ] 183. All of the following statements are true except:
a. Increased alkaline phosphate [ NBE Pattern 2014 - 15 ]
b. Decreased phosphate in urine
a. 25-a-hydroxylation of vitamin D occurs in the liver
c. Hypophosphatemia
b. 1-a-hydroxylation of vitamin D occurs in the kidney
d . Decreased serum calcium
c. In the absence of sunlight, the daily requirement of
Ref Harrison’s 18/ e p3094, Harrison 19 th p 2464
Vitamin D is 400-600 1 U
177 . The most common cause of severe hypercalcemia is: d. William's syndrome is characterized by precocious
a. Vitamin D toxicity [Corned K 2010 ] puberty, mental retardation and obesity
b. Sarcoidosis Ref: Harrison' s 18 ' ed. ch. 353, Harrison 19th p 2477
c. Chronic renal failure “
d. Malignancy
Ref Harrison's 18/ e p3099, Harrison 19 th p 2481

Ans. 171 . a . Carcinoma breast 172. a . Diarrhoea 173. b. Gallium nitrate 174. d. Bisphosphonates
175. c. Rickets , d . Hyperparathyroidism 176 . b. Decreased phospha . . 177 . d. Malignancy
178 . d . Ca 2+ 179. d . Milk alkali syndrome 180 . a . Hyper -parathyroidism 181 . c. Thyroid malignancy
88 182. b. The unionized ... 183. d . William’s syndrome is characterized by precocious puberty .. .
Endocrinology

184. 70 year female is on treatment with Alendronate


191. Low serum alkaline phosphatase is seen with:
for severe osteoporosis. Now she complains
of pain a . Hypoparathyroidism [ NBE Pattern 2014 -15]
in right thigh. What is the next investigation
to be b. Hypophosphatasia
performed : [AIMS Nov 20121 c. Hyperparathyroidism
a. DEXA scan
d. Pseudohypoparathyrodism
b. X-ray
c. Serum vitamin D levels Ref: Harrison's 18th ed. ch. 355, 333, Harrison 19th p237, 426e-l
d. Serum alkaline phosphate levels
192. Hypercalcemic crisis is seen in all except:
Ref: Harrison 19th p 2501 a. Metastatic carcinoma breast [ NBE Pattern 2014-15]
185. Arrow headed finger on X-ray is suggestive of: b. Hyperparathyroidism
a. Acromegaly c. Pancreatitis
[ NBE Pattern 2014-15]
b. Hyperparathyroidism d. Hodgkin's lymphoma
c. Down syndrome -
Ref: Harrison's 18th ed. ch. 353, Harrison 19th p 2470, Table 424 1
d. Sarcoidosis .
193 Hyperphosphataemia is seen in:[ NBE Pattern 2014-15]
Ref: Harrison’ s 18th ed. ch. 353. Disorder ofPathyroid Gland a. CRF
b. Prolonged phosphate intake
186. Hypercalcaemia is caused by all except:
c. Pseudo-pseudo-hypoparathyroidism
a. Multiple myeloma [ NBE Pattern 2014- 15] d. Pseudo- hypoparathyrodism
b. Hyperparathyroidism

Endocrilgy
c. Sarcoidosis Ref: Harrison's 18th ed. ch. 353, Harrison 19th p 2461
d. Hypothyroidism 194. True about primary hyperparathyroidism is:
Ref: Harrison's 18th ed. ch. 353, Harrison 19th p 2470 a. Hypotension [ NBE Pattern 2014 -15]
b. Recurrent abortion
187. Nephro-calcinosis is a feature of :[ NBEPattern 2014 -15]
c. Neuropsychiatric changes
a. Primary hyperparathyroidism
d. Gallstone
b. Medullary cystic kidney
c. Vitamin C intoxication Ref: Harrison's 18th ed. ch. 353, Harrison 19th p 2470
d. Pseudo-hypoparathyroidism 195. Most common cause of hyperparathyroidism:
Ref: Harrison's 18thed. ch. 353, Harrison 19th p 2472 a. Iatrogenic [AIIMS Nov 93]
b. Medullary carcinoma thyroid
188. Tufting of terminal phalanges is seen in:
c. Parathyroid adenoma
a. Hypo-parathyroidism [ NBE Pattern 2014 -15]
d. Parathyroid hyperplasia
b. Hyperparathyroidism
c. Hyperthyroidism Ref : Ch. 343 Harrison's 18th / ed, Harrison 19th p 2471-2472
d. Gallstone 196. Subtle presentation of hyperparathyroidism is:
Ref: Harrison's 18th ed. ch. 353, Harrison 19th p 2472 a. Psychiatric manifestation [AIIMS Dec 97]
189. Magnesium deficiency is seen in all except? b. Nephrocalcinosis
a . Hypercalcaemia [ NBE Pattern 2014 -15] c. Abdominal pain
b. Tetany d. Asymptomatic hypercalcemia
c. Starvation Asymptomatic hypercalcemia Ref :Harrison 18th/ 3102,
d. Neurological abnormalities Harrison 19th p 2472
.
Ref: Harrison's 18th ed. ch 352, Harrison 19th p 2462 197. Which of the following is associated with secondary
190. Investigation of choice in parathyroid pathology is: hyperparathyroidism [AIIMS Dec 94 ]
a. CT scan [NBE Pattern 2014-15] a. Parathyroid adenoma
b. Gallium scan b. Marked hypercalcemia
c. Thallium scan c. Chronic renal failure
d. Technetium - 99 sestamibi scan d. Parathyroidectomy relieves the symptoms
Ref Harrison's 18th ed. ch. 353, Harrison 19th p 2474 Ref.:Harrison 18th/ p 3109, Harrison 19th p 2478

Ans. 184. b. X-ray 185. b. Hyperparathyroidism 186. d . Hypothyroidism 187. a. Primary hyperpara ...
188. b. Hyperparathyr... 189. a. Hypercalcaemia 190. d. Technetium- 99 s... 191. b. Hypophosphatasia
192. c. Pancreatitis 193. -
c. Pseudo-pseudo hypop... 194. c. Neuropsychiatric... 195. c. Parathyroid adenoma
89
196. d. Asymptomatic.. . 197. c. Chronic renal failure
th )
Marwah's Interna l Medici ne MCQs ( Based on Harriso n's 19

198. A 45 year old man, known case of chronic renal failure c. Etidronate
develops rugger jersy spine. The probable cause is : d. Rizol
a. Aluminium intoxication [AI 2000] Ref : Harrison 19th Table 424-4, p 2 in
0

B
b. Secondary hyperparathyroidism 205. All are used in treatment of hypercalcemia, except;
c. Osteoporosis a. Phosphate [A11MS May 93]
d. Osteomalacia
b. Mithramycin
[ Ref. Harrison 18th / p 3109, Harrison 19th p 2478 c. Vitamin D in high dose
199. Tufting of the terminal phalanges is seen in : [AI 1995] d . Any of the above
a. Hypoparathyroidism Ref : Harrison 18th/ p 3113, 3108, Harrison 19th p 2480
b. Hyperparathyroidism
206. All of the following agents may be used in the
c. Hyperthyroidism management of Chronic Hypocalcemia, except:
d. Hypothyroidism [AI 2012]
a. Etidronate
Ref.: Harrison 18th / p 3102, 3103, Harrison 19th p 2478 b. Thiazides
200. Low calcium and high phosphate is seen in: [AI 2010] c. Elemental calcium
a. Hyperparathyroidism d. Vitamin D analogs
b. Hypoparathyroidism Ref .-Greenspan’s Endocrinology 8 th / 316; Harrison 19th p 2488
c. Hyperthyroidism
207. Hypocalcemia with hyperphosphatemia are seen in:

Edcrinlogy
d. Hypothyroidism [ PGl Dec 2000]
a . CRF
Ref.: Harrisons 17 th/ 2391 , Harrison 19th p 2483 b. Pseudohypoparathyroidism
201. A patient presents with low serum calcium, high c. Vit. D deficiency
phosphorus and elevated PTH. Which of the following d. Magnesium deficiency
investigations is least contributory to establish a 208. All of the following are seen in Ricket’ s except:
diagnosis: [AI 2011] a. Bow legs [AIIMS May 03]
a. Vitamin D levels b. Gunstock deformity
b. Serum creatinine levels c. Pot belly
c. Cyclic AMP response to PTH d . Cratio tabes
d. Urine myoglobin Ref : OP Ghai 6 th / 128- 129; Nelson 17th / 186 , 188,
Ref : Harrison 19th p 2478, 2486, Table 424 - 6 Harrison 19th p 2465
209. Which is not true of hypocalcaemia: [AI 1995]
202. Hypercalcemia is associated with all except: [AI 2009]
a. Can occur in hypoparathyroidism
a. Hyperparathyroidism b. Latent tetany is seen
b. Sarcoidosis c. Prolonged QT interval
c. Milk alkali syndrome d. Inverse relation with Mg + + levels
d. Celiac disease Ref: Harrison 18th / 361, 3113, 3114, 3115, Harrison 19th p 2457
Ref . Harrison 18th/ p 360, 3099,
210. During a routine check up, a 67 - year -old man is found
Harrison 19 th p Table 424 - 1 , p 2470 to have a level of serum alkaline phosphatase three
203. Hypercalcemia is seen in all except: [ARMS May 93] times the upper limit of normal . Serum calcium and
a. Acute pancreatitis phosphorus concentrations and liver function test
b. Hypervitaminosis D results are normal . He is asymptomatic. The most
c. Addison's disease likely diagnosis is :
[ AIIMS Nov 99 ,
d . Hyperparathyroidism a. Metastatic bone disease
b. Primary hyperparathyroidism
Ref : Harrison 18th / ed., Ch 343, Harrison 19th p 2470 c. Paget’ s disease of bone
204. Treatment of hypercalcemia includes, all, except: d. Osteomalacia
th p 426 e
a. Gallium nitrate [AIIMS Dec 97] Ref: Harrison 18 th/ 3137, 3138, Harrison 19
b. Plicamycin

Ans. 198. b. Secondary hy... 199. c . Hyperthyroidism 200. b. Hypoparathyroidism


201 . a . Vitamin D levels , b.Serum creatinine levels 202. d . Celiac disease 203. a . Acute pancreatitis

D 207 a . CRF, b . PseudohypoP


204. d . Rizol 205. c . Vitamin D in high dose 206. a . Etidronate .
208. b. Gunstock defor. . 209. d . Inverse relation with . . 210 . c . Paget’s disease of bone
Endocrinology

211- Alkaline phosphatase is elevated in all, EXCEPT:


a. Rickets 216. Mainstay of treatment of nephrogenic diabetes
[ AllMSDec 97 ] insipidus is:
b. Osteomalacia [ DNB]
c. Hypoparathyroidism a .Desmopressin
d. Hypophosphatemia b. Thiazide/Amiloride diureties and salt restriction
Ref.:Chandrasoma Taylor 3rd 965
c .Desmopressin and salt restriction
/ d. Vasopressin and salt restriction
212. Osteoporosis may be seen in all except: [A11998] Ref: Conn’s Current Therapy [ Elsevier] 2012 /693; Harrison’s
a. Hyperparathyroidism
18 / e p 2906, Harrison 19th p 304
b. Hypoparathyroidism
c. Thyrotoxicosis 217. Polyuria with low fixed specific gravity urine is seen
d . Heparin administration in? [ NBE Pattern 2014 -15] I
-
Ref .Harrison 18th/ p 3122-3124, Harrison
19th p 2490
a. Diabetes mellitus
213 . Osteoporosis is seen in : b. Diabetes insipidus
[A! 1994] c. Chronic glomerulonephritis
a. Thyrotoxicosis
b. Cushing’s disease d. Potomania
c Menopause Ref: Harrison's 18th ed. ch . 344, 283, Harrison 19th p 2276, 1811
d . All of the above
218. Central diabetes insipidus is characterized by:
-
Ref .Harrison 18th/ p 3122-3124, Harrison 19th p 2490
[ NBE Pattern 2014-15]
a. Low plasma and low urine osmolality

Endocrilgy
Diabetes Insipidus
b. High plasma and high urine osmolality
214. A 33 year old lady presents with polydipsia and c. Low plasma and high urine osmolality
polyuria.Her symptoms started soon after a road d. Low urine and high plasma osmolality
trafic accident 6 months ago. The blood pressure Ref: Harrison's 18th ed. pg. 2909, Harrison 19th p 2276-77
is 120 / 80 mm Hg with no postural drop. The daily 219. Pituitary diabetes insipidus is improved by:
urinary output is 6 -8 liters. Investigation showed , Na a. Water restriction [ NBE Pattern 2014-15]
130 mEq / L, K. 3.5 mEq /L, urea 15 mg/ dL, sugar 65 - b. Lithium
mg / dL. The plasma osmolality is 268 mosmol / L c. Chlorpropamide
and urine osmolality 45 mosmol / L. The most likely
d. Chlorthiazide
diagnosis is: [AIIMS Nov 02]
a . Central diabetes insipidus. Ref: Harrison's 18th ed. ch. 344 pg. 2100, Harrison 19 th p 2278
b. Nephrogenic diabetes insipidus.
c. Resolving acute tubular necrosis. S.LA.D.H.
d . Psychogenic polydipsia.
220. Dilutional hyponatremia is seen in?
Ref: Harrison 18th/ p2904, Harrison 19 th p 2276, 2279 a. Addison's disease [NBE Pattern 2014-15]
215. A patient following head injury was admitted in b. Vincristine
intensive care ward with signs of raised intracranial c. Diuretic therapy
pressure. He was put on ventilator and started on d. Craniphyrangioma
intravenous fluids and diuretics. Twenty four hours
Ref: Goodman and Gillman ch. 25 Regulation of Renal function
later his urine output was 3.5 litres, serum sodium
156 mEq /L and urine osmolality of 316 mOsm / kg. 221. All of the following are true about SIADH except?
The most likely diagnosis based on these parameters [All MS Nov.2011 ]
js: [AIIMS Nov 04] a. Vaptans are approved by FDA for its treatment
a. High output due to diuretics b. Water- loading test can be used for diagnosis of the
b. Diabetes insipidus condition
c. Too much infusion of normal saline c. Urine sodium is usually low or normal in these
d. Cerebral salt retaining syndrome patients
Ref: Previous question, Harrison 19 th p 332 d. Serum sodium may be as low as 125 mEq / L in these
patients
Ref: H 18th ed. Pg. 2909 Harrison 19th p 2416 Table 404 -3, 2282

Ans. 211. c. Hypoparathyroi... 212. b. Hypoparathyroidism 213. d. All of the above 214. d. Psychogenic polydipsia.
215. a. High output due.. 216. b. Thiazide/ Amiloride... 217. c. Chronic glomerulo... 218. d . Low urine and...
219. c. Chlorpropamide 220. b. Vincristine 221. c. Urine sodium is usually low or..
th
Marwah's Internal Medicine MCQs ( Based on Harrison's 19 )

222 . All are true regarding ADH action except? 227. The syndrome of inappropriate ADH secretion is
a. Postoperative secretion is more characterized by the following: [ NBE Pattern 2014 -iS ]
b. ADH secretion occurs when plasma osmolality is a. Hyponatremia and urine sodium excretion > 20
low mEq / L
-
5 c. Acts on DCT
d . Neuro-secretion
[ NBE Pattern 2014 15]

Ref: H 18th ed. pg. 2902, 2903, 2906, Harrison 19th p 2276 -77
b. Hypernatremia and urine sodium excretion > 20

c.
mEq / L
Hyponatremia and hyperkalemia
223. Consider the following statements: In nephrogenic d. Hypernatremia and hyperkalemia
diabetes insipidus the patient is likely to have: Ref: Harrison's 18 ' ed. ch. 100 Harrison 19th p 2282

i. High vasopressin level [NBE Pattern 2014-15 ] 228. Inappropriate ADH secretion is seen in all except:
ii. Poor or no response to desmopressin
a. Head injury [ NBE Pattern 2014-15]
iii. High plasma osmolality.
b. Oat Cell carcinoma of Lung
iv. Dilutional hyponatremia
c. Acute encephalitis
Which of these statements are correct? d. Chromophobe adenoma
a. i, ii and iii
Ref: Harrison’s 18 ed. ch. 100 pg. 2102, Harrison 19th p 2281

b. ii, iii and iv
229. SIADH secretion is seen in all except : [A11994]
c. i and iv

tnaocri gy
a. Lung abscess
d . i, ii, iii and iv b. Interstitial Nephritis
Ref: Harrison's 18“ ed. pg . 2909, Harrison 19th p 2278, fig 404.4 c. Vinka alkaloids
224. Urine osmolality in diabetes insipidus is: d. Bronchial adenoma
a. <150 mosm / L [ NBE Pattern 2014-15] Ref : Harrison 18th/ p 2908, Harrison 19th p 2281
b. < 300 mosm / L
230. All are true regarding SIADH except: [ARMS Nov 93]
c. 600 mosm / L a. Increased level of ADH
d. 900 mosm / L b. Urine hyposmolar
Ref: Harrison' s 18th ed. ch. 344, 340, Harrison 19th p 2278 c. Hyponatremia
d. Adequate hydration status
225. True about SIADH is all except: [ NBE Pattern 2014-15] Ref : Harrison 18th/ p 2910, Harrison 19th p 2282
a. Hyponatraemia
231. Which of the following is the drug of choice for the
b. Urine hyposmolar
treatment of inappropriate anti - diuretic hormone
c. Increased ADH secretion: [AI 1MS Nov 05]
d. Adequate hydration status a. Frusemide
Ref: Harrison's 18“ ed. ch. 340 p 2909, Harrison 19 th p 2278 b. Hydrochlorothiazide
226. Syndrome of Inappropriate secretion of Anti- Diuretic c. Spironolactone
hormone [SIADH] may be seen in the following except: d. Demeclocycline
a. Use of vincristine [ NBE Pattern 2014 - 15] Ref.: Harrison 18th/ p 2910, Harrison 19th p 2282
b. Oat cell carcinoma of lung 232 . Which drug is not used in SIADH : [Al 1996]
c. Porphyria -acute attack a. Fludrocortisone
b. Demeclocycline
d . Primary pulmonary emphysema
c. Desmopressin
Ref: H 18th ed. p 2908, Harrison 19th Table 404-2, p 2281 d. Hypertonic saline
Ref : Harrison 18th/ p 2910, Harrison 19th p 2282

Ans. 222. b. ADH secretion ... 223. a . i , ii and iii 224. b. < 300 mosm / L
225. b. Urine hyposmolar 226 . d . Primary pulmonary. . . 227. a . Hyponatremia and ... 228 . d . Chromophobe adenoma
229. b. Interstitial Nephr.. 230. b. Urine hyposmolar 231 . d . Demeclocycline 232. c. Desmopressin
Endocrinology

233. All of the following conditions are known to


cause Which of these statements are correct?
diabetes insipidus, except : [AIIMS May 04] &
[Al 2005] a. i, ii & iv b. ii, iii & iv
a. Multiple sclerosis b. Head injury
c Histiocytosis c. i, iii & iv d. i, ii & iii
d. Viral encephalitis
Ref.:Harrison 18th/ p 2905, Harrison 19th p 2281 Ref: Harrison's 18th ed. ch. 339 p 2892, Harrison 19th p 2269
240. All of the following are features of acromegaly except?
Growth Hormone Defects a. Glucose intolerance [ NBE Pattern 2014 -15]
b. Non-suppression of growth hormone by glucose
234. All of the following are associated with ingestion
gigantism/
acromegaly, except: [DNB] c. Raised level of plasma somatomedin C
a. Mental Retardation d. Low serum phosphate
b. Hyperhydrosis
c. Visceromegly Ref: Harrison's 18th ed. p 2893, Harrison 19th p 2269
d. Impaired glucose tolerance 241. Paradoxical growth hormone response to TRH is seen
Ref: Harrison s 18/ e p2894; Williams Textbook of in all except? [AIPG 2010]
Endocrinology, a. Malnutrition b. Anorexia nervosa
Harrison 19 th p 2271
c. Prolactinoma d. Acromegaly
235. Confirmatory investigation for Acromegaly is:
a. Insulin induced GH suppression Ref: Harrison's 18th ed. ch. 339, 340, 350
[PGI June 01 ]
b. Glucose induced GH suppression 242. Heel pad thickness is useful in: [ NBE Pattern 2014 -15]

Endocrilgy
c. Random GH assay a. Hypothyroidism b. Acromegaly
d. IGF - I level
c. PEM d. All of the above
Ref: Harrison 18/ e p2894, Harrison 19 th p 2271
Ref: Harrison's 18th ed. ch. 339 p 2894, Harrison 19th p 2269
236 . Laron dwarfism is due to? [ NEETPattern 2015-16 ]
a. GH deficiency Carcinoid Syndrome
b . GHRH deficiency
c. GH receptor resistance 243 . True regarding actions of "ANP": [ Manipal]
a. Decreased BP
d. IGF-1 deficiency
b. Decreased GFR
Ref : Nelson 18th, Ch 558, Harrison 19 th p 2258 c. Increased sodium reabsorption
237. Which is NOT a side effect of GH administration? d. Increased absorption plasma renin activity
a . Gynecomastia [ NBE Pattern 2014 -15] Ref: Kumar & Clark pg 1065
b. Hypoglycemia 244. All are true about mitral valve prolapse except:
c. Slipped capital femoral epiphysis [ Manipal]
d . Pseudotumor cerebri a. Has mid or late systolic click with late systolic
Ref: Harrison's 18th ed. pg. 2894, Harrison 19th p 2260 murmur increased by squatting
b. Propranolol can be used for atypical chest pains
238 . Acromegaly is characterized by all except: [AIPG 2011 ]
c. Echo shows systolic displacement of mitral valve
a. Diabetes leaflets into left atrium
b. Muscular hypertrophy d. Increased incidence seen in family
c. Enlarged nasal sinuses Ref: Harrison 19 th p 1546
d. Increased heel pad thickness 245. Carcinoid Tumor of lung originates from?
Ref: Harrison's 18th ed. ch. 339 pg. 2894, Harrison 19th p 2269 a . Type 2 Alveolar cell [JIPMER 2014]
239 . Consider the following statements about acromegaly b. Clara cell
i. Impaired glucose tolerance [ NBE Pattern 2014-15] c. Mucus [Goblet] cell
d. Kulchitsky cell
ii. Galactorrhoea
iii. Hypertension Ref : Harrison 18th p 3062, ch 350, Harrison 19th p 563
iv. Suppression of growth hormone with glucose

Ans . 233. a . Multiple sclerosis 234. a. Mental Retardation 235. b. Glucose induced ... 236. c. GH receptor resistance
237. b. Hypoglycemia 238. b. Muscular hypertrophy 239. d. i, ii & iii 240 . d. Low serum phosphate
241. a . Malnutrition 242. b. Acromegaly 243. b. Decreased GFR 244. a. Has mid or late systolic...
245. d . Kulchitsky cell
Marwah’s Internal Medicine MCQs (Based on Harrison's 19th)

.
246 Carcinoid syndrome produces valvular disease .
253 A young woman comes with secondary
primarily of the: [ NBE Pattern 2014-15] and galactorrhoea . amenorrhoea
MR1 shows a tumor of < io
a. Venous valve mm
b.
Tricuspid valve .
diameter in the pituitary fossa Treatment is:
.

13
a Hormonal therapy for withdrawal bleeding
c. Mitral valve
b. Chemotherapy [ NBE Pattern 2014-15]
d. Aortic valve
c .
Bromocriptine
Ref: Harrison's 18* ed. ch. 350, Endocrine, Harrison 19th p 564 d. Surgery
.
247 Carcinoid syndrome is associated with all except: Ref: Harrison's lffh ed. ch. 339 p 2888, Harrison 19th p 2267
a .
Flushing of skin [NBE Pattern 2014-15] 254. Hirsutism is caused by all, Except:
b. Skin lesions like pellagra [Ai 2009 ]
a. Cushing's syndrome
c .
VMA in urine
b. Hyperthyroidism
d. Bronchospasm
c. Hyperprolactinemia
[ Ref : Harrison’s 18* ed, Ch 350, Harrison 19th p 564 d. Acromegaly

Prolactin and Gynaecomastia Ref : Harrison 18th / p 381, Harrison 19th p 331
Anterior Pituitary Disease
248. Estimation of the following hormones is useful while
investigating a case of gynaecomastia except: 255. In Type V hyperlipidemia there is excess of: [ Manipall

Endocrilgy
a. Testosterone [ Manipal] a. HDL b. LDL
b. Prolactin
c. IDL d. Chylomicrons
c. Estradiol
d. Luteinising hormone Ref: Harrison 19th p 2242
Ref Harrison 19th p 2366 .
256 Testicular feminising syndrome means: [ Manipal ]
a. 45 xy b. 47 xy
.
249 Not a Cause of Gynaecomastia? [NEET Pattern 2015-16]
c. 46 xy d. 47 xy
a. Hypothyroidism b. Kallman
c. obesity d. Klinefelter syndrome
Ref: Kumar and Clark pg 1030
257. Drug used in ventricular arrhythmia due to WPVV
[ Ref : Harrison 18th, Ch 346, Harrison 19th p 2366
syndrome: [Manipalj
250. All of the following causes hyperprolactinemia except: a. Amiodarone b. Digoxin
a. Methyldopa [NBE Pattern 2014-15] c. Lignocaine d. Verapamil
b. Phenothiazines Ref Harrison 19th p 1481
c. Bromocriptine .
258 POEMS syndrome is associated with all except:
d. Metoclopramide a. Osteolytic multiple myeloma [ Manipall
b. Hypertrichosis
Ref Harrison 19th Table 403-5 2266,2268 c .
Papilledema
251. Investigation to be done for hyperprolactinaemia? d. Amenorrhea
a. Estradiol estimation [NBE Pattern 2014-15] Ref : Harrison's 18th, pg 3080; Harrison 19th p 718
b. LH estimation 259. Asthenospermia means: [ Manipall
c. Diabetic status a. Immotile sperms
b. Absent sperms
d. Thyroid status
c. Dead sperms
Ref: Harrison's IS* ed. ch. 339 p 2888-89, Harrison 19th p 2267 d. Dysmorphic sperms
252. Prolactinoma presents with: [ NBE Pattern 2014-15] Ref Harrison's 18th Harrison 19th p 2387
a. Inferior quadrantopia b. Superior quadrantopia 260. Abnormalities of bone metabolism is associated will
. '
c Priapism d. Failure of lactation excess of which vitamins [ select two best options ]:
Harrison's Iff 1' ed. ch. 339, Harrison 19th p 2262 a. Vitamin A [PGlJunt
Ref b. Thiamine c. Vitamin B12
d. Vitamin D e. Tocoferol
Ref Harrison's 18/ e p3099, Harrison 19th p 27661
Ans. 246. b. Tricuspid valve 247. c. VMA in urine 248. b. Prolactin 249. a. Hypothyroidism
250. c. Bromocriptine 251. d. Thyroid status 252. b. Superior quadrant.. 253. c. Bromocriptine
94
254. b.Hyperthyroidism 255. d. Chylomicrons
258. a. Osteolytic multiple myeloma
.
256 a. 45 xy 257. a. Amiodarone
259. a. Immotile sperms 260. d. Vitamin D
I
Endocrinology

261. Pinch purpura around eyelids is one of the most 268. Which drug is essential in Sheehan's syndrome:
common cutaneous finding in : [ NEET Pattern Question ] a. Estrogen [A11996 ]
a. Fabry disease b. Cortisone
b. Porphyria cutanea tarda
c. Thyroxin
c. Primary systemic amyloidosis
d . Growth hormone
d . Secondary syst emic amyloidosis
Ref: Harrison’ s 18/ ep418, 947, Harrison 19thp 365 Ref : Harrison 18th/ p 2880, 2882, Harrison 19th p 2257

262. First drug to be started in Sheehan's syndrome is? Miscellaneous


a. Gonadotropins [NBE Pattern 2014-15]
b. Oestrogen
c. Thyroxine
-
269. A 28 years old lady has put on weight [10 kg over a I
d. Corticosteroids
period of 3 years], and has oligomenorrhoea followed j
by amenorrhea for 8 months. The blood pressure is I
Ref: Harrison's 18th ed. ch. 339 p 2879, Harrison 19th p 2257 160 /100 mm of Hg. Which of the following is the most 1
263. Which is the first hormone to fall in the blood in appropriate investigation? [ Manipal] j
Sheehan syndrome: [ NBE Pattern 2014 -15] a. Thyroid profile
a. GH b. ACTH b. Plasma cortisol
c. prolactin d. TSH c. Plasma testosterone and ultrasound evaluation of f
Ref: Harrison's 18th ed. ch. 334, 340 william' s endocrinology 12th pelvis
ed. pg. 246. Post- pattern pituitary infarction, d. All of the above

Endocrilgy
Harrison 19th p 2265 Ref: Davidson 17th pg 723
264. The following are recognized features of 270. All are true regarding idiopathic edema of women
panhypopituitarism except: [ NBE Pattern 2014 -15] except: [ Manipal]
a. Increased insulin sensitivity a. It is due to estrogen mediated sodium retention
b. Pigmentation of the mucous membranes b. It is not related to menstrual cycles
c. There is increased water retention in upright
c. Low serum thyroxine and TSH levels
position
d . Loss of secondary sex characters d. ACE inhibitors can be useful in some cases
Ref: Harrison's 18 ed. ch. 342 p 2956, Harrison 19th p 2324
“ 271. In pseudo- precoccous- puberty there is: [ Manipal]
265. Levodopa test is used to detect: [ NBE Pattern 2014-15] a. Virulization + /Spermatogenesis
a. LH b. ACTH b. Spermatogenesis + / Virulization +
c. FSH d . GH reserve c. Spermatogenesis + / Virulization
[ Ref : Pediatric endocrinology Mark a. Sparling, table 10.9, 4th ed
d.

Spermatogenesis / Virulization
272. In vitro DNA amplification is done by: [ Manipal]
266. A 32 -year -old female patient did not visit the a. Blotting technique
physician for the last 6 year when she had given birth b. Recombinant technique
to her last daughter, which was her third child. She has c. Electrophoresis
currently come with complaints of vaginal pruritus. d. Polymerase chain reaction
She also has history of cold intolerance and repeated
Ref: Kumar & Clark pp 168, 169 Harrison 19 th p 1247
skin infections. On examination, the skin is dry and
coarse. The pubic and axillary hair is absent. All the 273. Features of hypocalcemic states are all except:
following can be given for her treatment except: a . Decreased calcium absorption [ Manipal ]
a. Prednisolone [AIPG 2010] b. Increased calcium absorption form the gut
b. Thyroid hormone c. Increased calcium mobilization from the bones.
c. Ethinyl estradiol d. Increased PTH levels
d. Insulin Ref : Harrison 19th p 2257 Ref: Harrison 19th p 315
267. All are associated with pituitary apoplexy except: .
274 Associated feature of gonodal dysgenesic: [ Manipal]
a. Hyperthyroidism [AIIMS May 07] a. Short stature
b. Diabetes mellitus b. ILL developed testis c. Gynaecomatia
c. Sickle cell anemia d. Hypertension d. Pheochromocytoma
Ref : Harrison 18th/ p 2879, Harrison 19th p 2257 Ref: Harrison 19th p2352

Ans . 261. c. Primary systemic.. 262. d . Corticosteroids 263. a. GH 264. b. Pigmentation of the...
265. d . GH reserve 266. d . Insulin 267. a. Hyperthyroidism 268. b. Cortisone
269. d . All of the above 270. a. It is due to estrogen... 271. a. Virulization + /Sper... 272. d. Polymerase chain reaction
273. Decreased calcium absorption 274. a. Short stature
Marwah's Internal Medicine MCQs ( Based on Harrison's 19th )

275. A patient develops pre breakfast hyperglycaemia. 283. DOC for Alzimer’s dementia: [ Manipalj
Next step of management is: [ Martipal] a. Valproic acid
a. Start oral hypoglycaemic in morning b. Clozapine
b. Monitor overnight blood glucose levels c. Acetyl cholinesterase inhibitors
c. Start a dose of short acting insulin at bedtime d. Cinnerizine
E 41 d. Start early morning insulin therapy
Ref: Kumar & Clark 1265 Harrison 19th p 259a
Ref: Harrison 19th p 2406
284. True about obesity? [ NEETPattern 2015-isj
|276. To make a diagnosis of SIADH which of the following
| a. Seen mostly in females
criteria should be met? [ Mcmipal]
a. Persistently serum osmolality < 280 mosm b. Prevalence decrease upto 40 years of age
b. Serum potassium 6 mmol /1 c. No genetic predisposition
c. Presents with noeturia d. Smoking is a risk factor
d. Collapse of right apical lobe on X-ray
Ref : Harrison’s 18th, Ch 77, Harrison 19thp 2392
Ref: Kumar & Clark pg 1059/ Harrison 19th p 1259 285. Purtschner’s retinopathy is seen in all except:
277. A man with gynecomastia was found to have 47 XXV. a. Fat embolism [ NBE Pattern 2014 -15 ]
Related finding must include: [ Manipal ] b. Pancreatitis
a. Decreased levels of LH and FSH
b. Estrogen excess c. Chest trauma
c. Azoospermia d. Unilateral carotid artery occlusion

Endocrilgy
d. HCG excess Ref Harrison's 18th ed. ch.313, Harrison 19th p 40e -4
Ref Harrison 19th p 2353 286. Which is not seen in Allgrove syndrome:
278. Feature of pheochromocytoma: [ Manipal ] a. ACTH excess leading to Cushing syndrome
a. Increased urine free catecholamines b. Achalasia [ NBE Pattern 2014 - 15 ]
b. Increased urine metanephrines c. Alacrimia
c. Increased urinary VMA d. Hyperpigmentation
d. All of the above Ref Harrison 19th p 2330 Ref William' s Textbook of Endocinology 12"1 ed. P-841 -842- En ¬

279. Treatment of choice in Thyrotoxicosis: [ Manipal ] docrinology offetus


a . Propranolol 287. Pinna calcification is seen in all except?
b. Anti thyroid drugs a. Gout [AUMS Nov 2012 ]
c. Surgery b. Onchrnosis
d. Propranolol followed by surgery c. Frost bite
Ref Harrison 19th p 2294 d. Addisons disease
280. Hypercalcemia is seen in all except: [ Manipal ] .
Ref Harrison' s 18thCh 54 Skin Manifestations of Internal Dis ¬

a. Myxedema .
ease and www nejm.org / doi/ full/ 10.1056/ NE] Micml 000193
b. Hyperparathyroidism 288. A patient has amenorrhea with hypothalamic lesion .
c. Multiple myeloma The diagnosis is most likely to be?
d. CaLung Ref: Harrison 19th p 610 a. Kallman's syndrome [ NBE Pattern 2014 - 15 ]
281. Polydipsia, polyuria, increased is due to deficiency of: b. Asherman’s syndrome
a. Vasopressin [ Manipal] c. Stein Leventhal syndrome
b. Insulin d. Sheehans syndrome
c. Decreased ADH
d. Decreased insulin ’“ .
Ref Harrison’s 18 ed ch. 341 p 2878, Harrison 19th p 2256
289. Most common tumour of pituitary is:
Ref: Harrison 19th p 295
a. Prolactinoma [ NBE Pattern 2014 15 !
282. Resetting of pressure natriuresis is brought by: b. GH secreting adenoma
a. High sodium excretion with Normal BP c. ACTH secreting adenoma
b. Low sodium excretion with high BP [ Manipal ] d. TSH secreting adenoma
c. Low sodium excretion with low BP Ref Harrison's 1 e d. ch. 339 p 2882 Harrison 19th p 2266
d . High sodium excretion with high BP

Ans. 275. b. Monitor overn.. 276. a. Persistently serum 277. c. Azoospermia 278. d. All of the above
279. b. Anti thyroid drugs 280. a. Myxedema 281. a. Vasopressin 282. d. High sodium excretion
283. c. Acetyl cholines.. 284. d. Smoking is a risk factor 285. d. Unilateral carotid.
.. 286. a. ACTH excess leading to.
287. a . Gout 288- -
a Kallman’s syndrome 289. a . Prolactinoma
Endocrinology

290. Age related deterioration of cognitive function is


due 297. An obese lady aged 45 years was brought to
to increase in following:
[ A1 PG 20121 emergency in a semi -comatose condition. The

a
a. Homocystiene b. Cystiene
c. Taurine laboratory investigation showed K + [5.8 m mol / L]; Na
d . Methionine
+ [136 m mol / L]; blood pH [7.1], HC03 [12 m mol / L]
Ref: McCaddon et al 2006: omocysteine and cognitive
impair went Ketone bodies [350 mg / dl ]. Probable blood glucose is
291. The most common presentation ? [ AIPG 2011 ]
of cranio ¬
pharyngioma: a . < 45 mg/ dl b. < 120 mg / dl
a. Headache
[ NBE Pattern 2014 15] - c. > 180 mg/ dl d. < 75 mg / dl
b. Visual field defects
Ref Harrison's 18th ed. ch. 344, Harrison 19th p 2419
c. Endocrinal disturbance
d . Cardiac disturbance 298. Hypomagnesemia is seen in all except:
a. Gitelman syndrome [ NBE Pattern 2014 - 15]
Rtf: Harrison’s 18th ed. pg. 2081, Harrison 19thp 2264
b. Hungry bone syndrome
292. Bony clival erosion with intra -cranial calcification is c. Paget disease
seen in: [ NBE Pattern 2014 -15] d . Prolonged thiazide therapy
a. Craniopharyngioma Ref: Harrison' s 18th ed. ch. 352, Harrison 19th p 2462
b. Medulloblastoma
299. Patient with arthritis, skin hyperpigmentation and
c. Papilloma of the choroids plexus
hypogonadism is diagnosed to have:
d . Sella chordoma
a. Hemochromatosis [ NBE Pattern 2014-15]

Endocrilgy
Ref: Harrison's 18th ed. ch. 339 p 2883, Harrison 19th p 2264 b. SLE
293. Which of the following is under anterior pituitary c. Ectopic ACTH secreting tumor of lung
control? d. Wilson's disease
a. Fluid and electrolyte balance /M? ZT Pattern 2014-15] Ref Harrison's 18th ed. ch. 357, Harrison 19th p 2516
b. Control of blood pressure 300. 8 days old male child presents with pale stools,
c. Muscle activity turmeric colored urine, yellow sclera, H / O
d. Gonad function consanguineous marriage. On 3rd day after birth
developed fever and sepsis which was treated with
Ref: Harrison's 18th ed. ch. 339, Harrison 19th p 2358 chloramphenicol broad spectrum antibiotic.
-
294. All are true regarding cranio pharyngioma except: Investigation of choice to be done is assays of:
a. Derived from Rathke's pouch [NBE Pattern 2014-15] a. (3-glucosidase [NBE Pattern 2014 -15]
b. Contains epithelial cells b. p-galactosidase
c. GALT
c. Present in temporal or parietal lobes
d. Alpha -glucosidase
d . Causes visual disturbances
Ref: Harrison' s 18th ed. ch. 362
Ref Harrison's 18th ed.chpter 339, p 2883, Harrison 19th p 2264
301. A short stature patient with narrowed foramen
295. Reversible Dementia is a feature of ? magnum and rhizomelic limbs is seen in ?
a. Hyperparathyroidism [NBE Pattern 2014-15] a. Achondroplasia [NBE Pattern 2014-15]
b. Hypothyroidism b. Laron dwarf
c. Hyperthyroidism c. Hypothyroidism
d. Cushing's disease d . Morquio disease
.
Ref: William's Endo pg. 410 Harrison’ s 18th
ed. ch. 371, 341 Ref Harrison's 18th ed. ch. 353, 346 , Harrison 19th p 426e-7
296. Necrobiosis lipoidica diabeticorum is most marked 302. All are features of primary hemochromatosis
on: [NBE Pattern 2014-15] except:
a. Forearms a . Chorea [ NBE Pattern 2014 - 15/
b. Face b. Diabetes
c. Pre-tibial c. Arthritis
d . Sole of foot d. Skin pigmentation
Ref Harrison's 18th ed. ch. 344 p 2988, Harrison 19th p 2429 Ref: Harrison' s 18th ed. ch. 357, Harrison 19th p 2512

Ans. 290 . a . Homocystiene 291. a . Headache 292. d. Sella chordoma 293. d. Gonad function
294. c. Present in temp.. 295. b. Hypothyroidism -
296. c. Pre tibial
300. c. GALT
297. c. > 180 mg / dl
301. a . Achondroplasia
298. c. Paget disease 299. a . Hemochromatosis
302. a . Chorea , b. Diabetes
th
Marwah's Internal Medicine MCQs ( Based on Harrison's 19 )

303. Clinical features of pancreatic cholera A / E: 311. Pancreatitis, pituitary tumor and pheochromocyto
a. Hypokalemia [ A1PG 2010] may be associated with: [M 2004, 2005]
b. Achlorhydria a. Medullary carcinoma of the thyroid
c. Hypocalcemia b. Papillary carcinoma of the thyroid

S3
d. Glucose intolerance c. Anaplastic carcinoma of the thyroid
d. Follicular carcinoma of the thyroid
Ref: Harrison's 18th
ed. ch. 350, Harrison 19th p 570
Ref : Harrison 18th / p 3072, 3073, Harrison 19th p 2340
304. In Hemochromatosis iron deposition is found feature b / w Ectopic
differentiating
312. The ACTH
commonly at all sites except: [ NBE Pattern 2014-15]
secretion and Cushing syndrome is: [ AIIMS Dec 981
a. Heart b. Joints a. Hypokalemic alkalosis
c. Testes d. Pancreas b. Clinical features of cushing syndrome
Ref: Harrison's 18“ ed. ch. 357, Harrison 19th p 2512 c.
Hyperpigmentation
d. Hypertension
305 . All the following statements about Wilson’s disease
are true except: [NBE Pattern 2014-15] Ref.:Harrison 18th / p 2945, 2946, 2947, Harrison 19thp 23 ls
a. It is an autosomal recessive disorder 313. Which of the following is the earliest manifestation of
b. Serum ceruloplasmin level is < 20 mcg/ dl Cushing ‘s syndrome? [ Al 2004 j
c. Urinary copper excretion is < 100 mcg/ day a. Loss of diurnal variation
d. Zinc acetate is effective as maintenance therapy b. Increased ACTH
c. Increased plasma Cortisol
.
th ed ch. 360, Harrison 19th p 2519

Edcrinolgy
Ref: Harrison's 18
d. Increased urinary metabolites of Cortisol
306. "Hour -glass" shape of the chest and "tri-radiate pelvis" Ref : Harrison 18th / p 2946, Harrison 19th p 2316
are seen radio logically in: [NBE Pattern 2014-15]
a. Thyrotoxicosis .
314 In primary adrenal hyperplasia, incorrect is:
a. Hypokalemia [ ARMS May 94 j
b. Myxedema
b. Hyponatremia
c . Osteomalacia
c .
Hypernatremia
d. Hyperthyroidism
d. Water retention
Ref Oxford
: text book of rheumatolo gy 4th
ed. pg 1254, by R.A watts
Ref : Harrison 18th, Ch - 342, Harrison 19th p 2319
307. Increased urinary hydroxyproline is seen in?
a. Paget's disease [NBE Pattern 2014-15]
.
315 Addison's disease is characterized by all except :

b. Ehler Danlos syndrome


a .
Hyperglycemia [ Al 19931
c. Hypo-parathyroidism
.
b Hypotension
c .
Hyperkalemia .
d Hyponatremia
d. Osteoporosis
's 18th ed. ch. 355, Harrison 19th p 426 -e Ref :Harrison 18th / p 2956, 2957, Harrison 19th p 2324
Ref: Harrison
.
316 Regarding Addisonian pigmentation, all are tr
308. Which of these is not an antihypertensive medication: EXCEPT:
a. Clinidipine [NBE Pattern 2014-15] a. Involves moles and scars [ARMS May Oil
b. Chlorthalidone b. Involves Palmer creases
c. Canagliflozin c .
Does not involve oral mucosa
d. Captopril d. Decreased fibrosis
Ref: Harrison's IS ”
1
ed. ch. 344, Harrison 19th p 2415 Harrison 19 th p 2324
Ref:

309. Drug of choice for an attack of periodic paralysis with .


317 A 29 years old person is known diabetic on oi il .
calcium channel defect is: [NBE Pattern 2014-15] .
hypoglycemic agents since 3 years He has lost weight
a. ACTH b. Potassium chloride and never had DKa His grand father is diabetic but Ins
.
c. Calcium chloride d. Adrenaline father is nondiabetic. Which is the likely diagnosi
[A! 20091
Ref: Harrison's 18* ed. ch. 387, Harrison 19th p 444e
a.
MODY b. DM type I
310. Dancing carotid sign is seen in: [ NBE Pattern 2014-15]
b. Papillary Ca
c.
DM type II .
d Pancreatic diabetes
a. Thyrotoxicosis and Diabe &
c. Follicular Ca d. Hashimoto’s disease Ref.:CMDT 2007/1222; 'Clinical Endocrinology : .
Mellitus' by Sachdev & Gupta lst/ 916; ‘Hand book of Diabetes
[Ref : Bedside cardiology A. Sarkar,1" ed. pg. 85 th p 2404 , 2'
Williams 3rd / 72, Harrison 19

Ans. 303. c. Hypocalcemia 304. c. Testes 305. c. Urinary copper excre.. 306. c. Osteomalacia

m
307. a. Paget’s disease 308. c. Canagliflozin 309. b. Potassium chloride 310. a. Thyrotoxicosis
«> II.
311 a Medullary carci..
a.. mcuuiioij 312. a. Hypokalemic aihdiuao
- alkalosis OIO .
313. a. Loss of diurnal varia.. 314. b_ . Hyponatremia
31
315 R. Hyperglycemia
a. Hvnernlvcem
a ia C. Does nnt
316. c invnluo nrai
not involve mucosa
oral mumcn 317 . a. MODY , c.
u/mv e nM t
DM type II
Endocrinology

318. All of the following are true about Type I DM, Except
: b. Rhinocerebral mucormycosis
[ NEET / DNB Pattern Questions] c. Emphesematous pyelonephritis
a. Family history is present in 90% of cases
b. Antibodies against p cells d. Emphysematous appendicitis
c. Prone to Diabetic Ketoacidosis [ DKA] Ref : Harrisons 16 th / 186, 189, 1720, Harrison 19th p 2429
d. Insulin is required for management
of DKA
319. A 40 year old male patient is suffering from type
diabetes mellitus and hypertension.
Which of the
following antihypertensive drugs should not be
in such patients:
a. Lisinopril
c. Losartan d . Trandolopril
II

used
[MIMS Nov 03]
b. Hydrochlorthiazide
325. Recombinant human insulin is made by
a. CDNA from any eukaryote cell [ARMS June 2000]
b. Genome of any eukaryote
c. CDNA of pancreatic cell
d . Genome of pancreatic cell
Ref : Harrison 18th/ p 2993, Harrison 19th p 2411
0
326. Hypoglycemia is a recognized feature of all of the
Ref.: Harrison 19th p 2426 following conditions, except : [AI 2002]
320 . Impaired glucose tolerance on an oral GTT is indicated a. Uremia
by ; [ PGl Dec 02] b. Acromegaly
a. Fasting plasma sugar > 126 mg/ dl c. Addison’s disease
b. Random blood sugar > 200 mg/ dl d. Hepatocellular failure
c. Fasting blood sugar < 90 mg/dl Ref.: Harrison 18th/ p 3003, Harrison 19th p 2269
d. Fasting blood sugar < 140 mg/ dl; two hours after

Endocrilgy
glucose load > 200 mg / dl 327. Which of the following test is useful to distinguish
e. 2 hrs after glucose load 140- 200 mg/ dl; fasting between Insulinoma and Sulfonylurea related
blood sugar < 126 mg / dl hypoglycemia:
a. Antibody to Insulin [AIIMS May 08]
Ref : Harrison 19 th p 2399 b. Plasma - C- peptide level
321. HbAlC level in blood explains: [ A12004] d. Plasma Insulin level
a. Acute rise of sugar d. Insulin- Glucose ratio
b. Long terms status of blood sugar Ref :Harrison's 17 th/ 2354, 2355, Harrison 19th p 2434
c. Hepatorenal syndrome
328. A 35 year old man presents with vomitings and
d. Chronic pancreatitis confusion. On examination Na + 120 m mol/ L, K + 4.2
Ref : Harrison 18th/ p 2992, m mol / L, Uric acid 2 mg/ dl. Patient is not edematous.
322 . An obese patient presented in casualty in an The diagnosis is: [AIIMS Nov 99]
unconscious state . His blood sugar measured 400 a. Cerebral toxoplasmosis with SIADH
mg%, urine tested positive for sugar and ketones; b. Hepatic failure
drug most useful in management is: /A / 2001, AIIMS 98]
c. Severe dehydration
b. Troglitazone d. Congestive heart failure
a. Glibenclamide
c. Insulin d . Chlorpropamide Ref : Harrison 19 th p 2282
Ref : Harrison 18th/ p 2978, Harrison 19th p 2419 329. 'Hurthle cells' are seen in [AI 1995]
a. Agranulomatous Thyroiditis
323 . Treatment of 42 year old obese man with blood
b. Hashimoto's Thyroiditis
glucose 450 mg, urine albumin 2 +,sugar 4+, ketone 1+
[ARMS June 98] c. Papillary carcinoma of the thyroid
is
b. Glibenclamide d. Thyroglossal cyst
a. Insulin
c. Glipizide d . Metformin Ref : Bailey 24 th/ 803, 804, Harrison 19th p 2298
Ref : Harrison 18h/ ed„Ch344, Harrison 19th p 2419 330. The most common presentation of endemic goiter is
a. Hypothyroid [AI 1996 ]
324. Life threatening complications of diabetes mellitus b. Diffuse goiter
are all except: [ARMS May 07]
c. Hyperthyroid
a. Malignant otitis externa d. Solitary nodule
Ref : Bailey & Love 22nd / 511 ; Robbins 6 th / 1138

Ans . 318 . a . Family history. . 319 . b. Hydrochlorthiazide 320 . e . 2 hrs after glucose ... 321 . b . Long terms status of ...
322. c . Insulin 323. a . Insulin 324. d . Emphysematous app.. 325 . c. CDNA of pancreatic cell
326 . b. Acromegaly 327 . None 328 . a. Cerebral toxoplas .. 329 . a. Agranulomatous..
99
330 . b. Diffuse goiter
Marwah s Internal Medicine MCQs ( Based on Harrison's 19th )

331. Needle biopsy of solitary thyroid nodule in a young c. Peptic ulceration, achlorhydria, non beta cell
woman with palpable cervical lymph nodes on the d.
tumour
Peptic ulceration , achlorhydria, beta cell tumour
same sides demonstrates amyloid in stroma of lesion. Ref : Harrison 18th/ p 2455, Harrison 19th p 2336
Likely diagnosis is [Al 2002]
a. Medullary carcinoma thyroid 337. Which of the following tests is not used in the
b. Follicular carcinoma thyroid diagnosis of insulinoma [Al 2011 ]
c. Thyroid adenoma a. Fasting blood glucose
d. Multi nodular goitre b. Xylose test
c. C- peptide levels
Ref.: Bailey 23rd/ 731, 732;, Harrison 19th p 2305
d. Insulin / Glucose Ratio
332. Screening method for medullary carcinoma thyroid is: Ref : Harrison 18th/ p 3066, 3067, Harrison 19th p 2434
a. Serum calcitonin [Al 1997]
b. S. calcium 338. Carcinoid tumour is most common in [ AIIMS May 04]
a. Esophagus b. Stomach
b. S. alkaline phosphate
c. Jejunum d. Appendix
d. S. acid phosphatase
Ref : Harrison 18th/ p 3058, Harrison 19th p 56 4
Ref.: Bailey 24th/ 801, 802, Harrison 19th p 2305 06 - 339. Carcinoid syndrome produces valvular disease
333. A 26 year woman presents with a palpable thyroid [AIIMS May 04]
primarily of the
nodule, and needle biopsy demonstrates amyloid b. Tricuspid valve
a. Venous valves
in the stroma of the lesion . A cervical lymph node is

Endcriolgy
c. Mitral valve d. Aortic valve
palpable on the same side as the lesion , the preferred
treatment should be : Ref : Harrison 18th/ p 3062, Harrison 19th p 564
a. Removal of the involved node, the isthmus, and the 340. Obesity is not a feature of : [AIIMS May 95]
enlarged lymph node [Al 2002 ] a. Hypothyroidism b. Adrenal insufficiency
b. Removal of the involved lobe, the isthmus, a portion c. Hypogonadism d. Cushing's syndrome
of the opposite lobe, and the enlarged lymph node. Ref : Harrison's 18th/ ed, Ch 342, Harrison 19th p 2324
c. Total thyroidectomy and modified neck dissection
341. Endocrinal causes of Carpal Tunnel syndrome include
on the side of the enlarged lymph node.
all of the following, except [ Al 2009]
d. Total thyroidectomy and irradiation of the cervical
a. Diabetes Mellitus b. Hypothyroidism
lymph nodes.
c. Acromegaly d. Addison's disease
Ref : Ch.341 Harrison 18th/ ed, Harrison 19th p 2306
9 Ref : CMDT 2008/ 716, Harrison 19th p 2221
334. Pheochromocytoma predominantly secretes 342. Prader -Willi syndrome is associated with an increase
a. Epinephrine [AIIMS Nov 01 ] in which of the following hormones:
b. Norepinephrine a. Ghrelin [ NBE Pattern 2014 - 15 ]
c. Dopamine b. GH
d. DOPA c. FSH
Ref : Harrison 18th/ p 2963, Harrison 19th p 2330 d. LH
335 . All of the following are features of Ref: Prader willi syndrome: ch. 44 Nelson 1811 ed
phaeochromocytoma except : [Al 2002] 343. All are causes of osteoporosis, except: [ Al 20001
a. Hypertensive paroxysm a . Thyrotoxicosis
b. Headache b. Hypothyroidism
c. Orthostatic hypotension c. Chronic heparin therapy
d . Wheezing d. Old age
Ref : Harrison 18th/ 2962, 2963, Harrison 19th p 2330 Ref: Harrison's 18/ e p3124, Harrison 19th p 2497
336. The triad originally described by Zollinger Ellison
syndrome is characterized by: [Al 2002]
a. Peptic ulceration, gastric hypersecretion, non beta
cell tumour
b. Peptic ulceration, gastric hypersecretion, beta cell
tumour

Ans. 331 . a . Medullary card .. 332. a . Serum calcitonin 333. c. Total thyroidectomy and . .
334. b. Norepinephrine 335. d . Wheezing 336 . a . Peptic ulceration , ga ... 337 . b. Xylose test
338 . d . Appendix 339. b. Tricuspid valve 340. b. Adrenal insufficiency 341 . d. Addison 's disease
100
342. a . Ghrelin 343. b. Hypothyroidism
CHAPTER Respiratory System
4
Most Recent Q’s 2014- 15 b. Wegeners granulomatosis
c. Emphysema
1- Which of the following do not usually cause reduction d. Biastomycosis Ref: Harrison 19 th p 511
in Diffusion Lung Capacity of Carbon Monoxide? 7. DLCO is a very sensitive test for interstitial lung
a. Emphysema [AllMS May 2015] diseases, in which it is decreases. Which among the
b. Asthma
c. Pulmonary vascular obstruction following conditions cause an increase in DLCO?
d. Interstitial Lung disease a. Pulmonary fibrosis [ NBE Pattern 2014 ]
b. Pulmonary artery hypertension
Ref.: Harrison 19 th page 306 e - Sf
c. Alveolar hemorrhage
2. ABG of a patient with decreased pH , increased pC02 d. ILD Ref: Harrison 19 th p 306 e-5 f
and high bicarbonate . Diagnosis: /JIPMER 2015 ]
a . Respiratoiy alkalosis, compensated 8. The BODE index, a simple multidimensional grading
b. Respiratory acidosis, compensated system , is better than the FEV1 at predicting the risk
c. Respiratory acidosis not fully compensated of death from any cause and from respiratory causes
d. Metabolic alkalosis, uncompensated among patients with COPd . Which of the following is
Ref : Harrison 19th 306 e -7, 1663 not the component of BODE index?
a. BMI [ NBE Pattern 2014]
3. Lobectomy done in case of bronchogenic Carcinoma, b. Oxygen requirement
What will be the effect on spirometry? [ JIPMER 2015]
c. Dyspnoea
a. Increased residual volume
d. Exercise performance
b. Increased vital capacity
c. Increased dead space ventilation Ref: Harrison 19 th p 1701
d. Increased closing volume 9. Multi drug resistant tuberculosis is resistance to?
Ref: Harrison 19 th 514, 516 a. 1NH & Pyrizinamide [ DNB Pattern 2014]
4. A 45 year old male, smoker, presented with complaints b. INH & Rifampicin
of progressive weakness, anorexia and weight loss c. Rifampicin & Pyrizinamide
for 2 months. He also gives h / o low grade fever and d . Resistance to all Ref: Harrison 19 th p 205 e -2 t
cough. The chest radiograph of the patient is given 10. Upper lobe bronchiectasis is seen in which disease?
below. What is the most probable diagnosis? a. Cystic fibrosis [ DNB Pattern 2014 ]
a. Miliary TB [ NBE Pattern 2014 ]
b. Aspergilloma
b. Consolidation
c. HIV
c. Ca lung
Ref: Harrison 19 th p 1112 d . Bronchogenic carcinoma
d . Pleural effusion
Ref: Harrison 19 th p 1669
5. All are true about Klebsiella pneumonia except:
a . It is also known as Pfeiffer's bacillus 11. Neuromuscular disorder is related to which type of
b. Pneumonia caused by klebsiella has clear upper lung cancer? [ DNB Pattern 2014 ]
lobe predilection [NBE Pattern 2014] a. Adeno carcinoma
c. Associated with "red current jelly" sputum b. Squamous cell carcinoma
d. Can cause pneumatocoeles c. Small cell carcinoma
Ref: Harrison 19 th p 933 d. Bronchoavelolar carcinoma Ref Harrison 19 th p 511
6. Any lung cavity with a wall thickness more than or 12 . C02 level in kPa above which brain stem death is
equal to 4 mm should raise suspicion of Ca Lung. confirmed? [ DNB Pattern 2014]
Squamous - cell carcinoma is the most common a. 1 b. 3
histological type of lung cancer to cavitate [82 %]. c. 5 d. 6.5
Other causes of thick walled cavity include all except Ref Harrison 19 th p 442 e- 2
a. Lung abscess [NBE Pattern 2014]

Ans . 1. b . Asthma 2. b. Respiratory acidosis.. 3. c. Increased dead space.. 4. a . Miliary TB


5. a . It is also known as.. 6 . c. Emphysema 7. c. Alveolar hemorrhage 8. b . Oxygen requirement
9 . b. INH & Rifampicin 10 . a. Cystic fibrosis 11. c. Small cell carcinoma 12. d. 6.5
Marwah's Internal Medicine MCQs ( Based on Harrison's 19th
)

13. Farmer’s lung is caused due to exposure to? 20. The histologic hallmark of Langerhan’s cells is:
a. Penicillium casei [ DNB Pattern 2014] a. Dendritic cell processes NBEPattern 2014- ir

a
b. Thermoactinomyces sacchari b. Giant mitochondria
c. Cladosporium species c. Birbeck granules
d. Micropolyspora faeni d. Eosinophilic granules
Ref: Harrison 19th p 1024 Ref Harrison's 18th ed. ch. 56 p. 441, Harrison 19th p 135e -
36
14. Side effects of hyperbaric oxygen therapy are all 21 . A patient presented with a 5 cm cavity of upper lobe
EXCEPT: [DNB Pattern 2014 ] of lung; choice of diagnosis will be:
a. Absorption atelectasis a. CT scan [ NBE Pattern 2014 -iSj
b. Increased pulmonary compliance b. Percutaneous aspiration
c. Decreased vital capacity c. Bronchography
d. Endothelial damage d. Bronchoalveolar lavage and biopsy
Ref Harrison 19th p 447e 4t - RefCMDT 2014 p. 278.
22. A patient presented with hemoptysis and persistent
Pulmonary TB -
cough. The chest X ray of the patient was normal. The
next best investigation is? [NBE Pattern 2014 -15 ]
15.

System
Confirmatory test for Pulmonary tuberculosis: a. Helical CT
a. Sputum for AFB [ Manipal 2014} b. High resolution CT
b. Tuberculin test
c. Bronchoscopy
c. X-ray chest
d . Angiography Ref Harrison ' s 18th
ed fig. 34 - 2.
d. Gram stain Ref Harrison 19th p 1107
16. 23. Tuberculous pleural effusion is characterized by A / L
Diagnostic features of allergic bronchopulmonary

Respiratoy
a. Haemorrhage [ NBE Pattern 2014 1
aspergillosis [ABPA] include all of the following
b. LDH more than 60%
except: [A12003]
c. Protein is increased
a. Changing pulmonary infiltrates d. Mesothelial cells
b. Peripheral eosinophilia
Ref Harrison's 18th ed. ch. 263, Harrison 19th p 1109 -10, 1718
c. Serum precipitins against Aspergillosis fumigants
d . Occurrence in patients with old cavitatory lesions. 24. Miliary shadow on X-ray is seen in all except:
Ref Harrison's 18/ e p2120, 17/ e pl 610, Harrison 19th p 1695 a. Tuberculosis [ NBEPattern 2014 - 15 !
b. Loeffler’s pneumonia
17. Systemic Miliary TB spreads via? c. klebsiella
a. Artery [NEET Pattern 2015-16 ] d. Varicella pneumonia
b. Vein RefCMDT-2014 p. 271, Harrison 19th p 1108- 1112, 1184
c. Bronchus
25. Miliary mottling of lung is seen in all except:
d. Lymphatic
a. Silicosis [ NBE Pattern 2014 - 15
Ref Robbins 9th edition ch 13., Harrison 19th p 1112 b. Aspergillosis
18. Patient diagnosed with HIV and Tuberculosis . How to c. Haemosiderosis
start ATT and c -A . R .T? [ NEET Pattern 2015 -16 ] d. Tuberculosis RefCMDT 2014 p. 1537.
a. Start ATT first 26. Tubular breathing is seen in: [ NBE Pattern 2014 - 15]
b. Start cART first a. Pleural effusion b. Consolidation
c. Start both simultaneously c. Pleurisy d. Tuberculous cavity
d. Start cART only
, Harrison 19 th p 1112 RefCMDT 2014 ch. 9, Harrison 19th p 950 - 51
Ref Harrison' s 18 th edn, ch 189
27. Cavitatory lesions in lung are seen in: [ Al 201 ( 5
19. Pseudo-hemoptysis is seen mostly with?
[NEET Pattern 2015-16 ] a. Primary pulmonary Tuberculosis
a . Streptococcus
b. E.coli
b. Staphylococcal pneumonia
c. Serratia Marcescens c. Preumoconiosis
d. R.S.V d. Interstitial Lung disease
Ref: john Hopkins internal medicine board review, box 22 -3, 4th Ref : Robbins 7th/ ed 383, 385; Oxford Hand book of Medicine
edn, Harrison 19th p 245-46 and 1661 6 th/ ed 174; Harrison 19th p 805-806

Ans. 13. b. Thermoactinomy .. 14. b. Increased pulmonary c... 15. a. Sputum for AFB 16. d. Occurrence in patients
.
.
17. a Artery 18. a. Start ATT first 19. c. Serratia Marcescens 20. c. Birbeck granules
102
.
21. a CT scan 22. b. High resolution CT 23. d . Mesothelial cells 24. c. klebsiella
25. b. Aspergillosis 26. b. Consolidation 27. b. Staphylococcal pneumonia
Respiratory System

28. False-negative tuberculin test is seen in all except: 35. 40 year old patient with history of prolonged exposure
a. After 4-6 weeks of measles attack [AI 1996 ] to Aspergillus presents with repeated episodes of
b. Immunodeficiency state
c. Miliary tuberculosis
breathlessness. Chest X ray shows diffuse pulmonary
-
infiltrates. Skin hypersensitivity test is positive for
I
Ei
d. Atypical mycobacterial infection
Aspergillus antigen Peripheral blood picture shows
Ref.: Ananthnarayan 6 th / 333; Park 18th/ 150, normal eosinophil count and serum IgE levels are
Harrison 19th p 1131, 1114, 1121 normal. The most likely diagnosis is:
29 . A man presents with fever, wt loss and cough; Mantoux a. Allergic bronchial Asthma
reads an induration of 17 x 19 mm; Sputum cytology b. Allergic Bronchopulmonary Aspergillosis [ABPA]
is negative for AFB . Most likely diagnosis is ; [ A12001 ] c. Extrinsic Allergic Alveolitis
a. Pulm tuberculosis b. Fungal infection d. Invasive Pulmonary Aspergillosis
c. Viral infection d. Pneumonia Ref: Harrison’s 18/ e p2120, Harrison 19th p 1681
Ref.: CMDT 2005/ 254; 2009/ 246 , Harrison 19th p 1113
36. Devi, a 28 year female, has diarrhea , confusion , high
30. Tuberculous pleural effusion is characterised by all of grade fever with bilateral pneumonitis. The diagnosis
the following features except effusion ; [AI 1997] is: [AI 2000]
a . Harmorrhagic effusion a . Legionella

Respiratoy
b. Pleural fluid LDH more than 60 % that of serum b. Neisseria meningitis
LDH . c. Streptococcus pneumoniae
c. Increased deaminase d. H . influenza
d. Increased mesothelial cells
Ref: Harrison's 18/ e pl 237, Harrison 19th p 767t
Ref : Harrison 18th/ p 2178, 2179, Harrison 19th p 1109 -10
A patient presents with sudden onset of
31. Pleural fluid having low glucose is seen in all , except - breathlessness after a subclavian vein cannulation.
a . Tuberculosis [AIIMS May 93] On examination, breath sounds are absent while the
b . Empyema chest is hyperresonant on percussion on one side.
c. Mesothelioma

System
Most likely cause is: [AI 2012]
d . Rheumatoid arthritis a. Iatrogenic pneumothorax
-
Ref : Harrison 18th/ p 2179, Harrison 19th p 1109 10, 1716 b. Subclavian vein air embolus
32 . All are cavitating lesions in the lungs, except - c. Malposition of cannula
a . Caplan’s syndrome [AIIMS May 94 ] d . Cardiac arrhythmia
b. Hamartoma Ref: Harrison's 18/ e p3212, Harrison 19th p 2292
c. Wegner's granuloma 38. Which of the following is true regarding Idiopathic
d. Squamous cell carcinoma
Non specific interstitial pneumonia ? [AIIMS Nov. 2014 ]
Ref : Sutton 7th/ 138,140,123, Harrison 19th p 515 a . Honey combing on CT
33 . Pneumonia Most common cause for community b. Male predominant
acquired pneumonia: [ Manipal 2014} c. Elderly age
a . Staphylococcus b. H influenzae d . Good prognosis
b. Pseudomonas c. Streptococcus Ref:Harrison 18th edn, ch 261, Harrison 19 th p 1712, 1255
Ref: Harrison 19th p 233
39. Which is correct about pneumonia ?
34. Which is not true about aspergillosis: a. Bronchophonia [ NEET Pattern2015- 16 ]
a. Aspergillus Niger is the cause of fungal otitis externa b. Decreased vocal fremitus
b. It is highly contagious c. Shifting of trachea
c. Aspergilloma is common in preexisting TB, or cystic d . Amphoric breathing
disease
Ref:Harrison 18th edn, ch 251, Harrison 19th p 803-805
d. Aspergillus fumigatus is a cause of bronchial
Aspergillosis
Ref: H 18/ e pi 655, 17/ e pl 257, 1256, Harrison 19th pi 681

Ans. 28. d . Atypical mycoba... 29. a . Pulm tuberculosis 30. d . Increased mesothe. 31. None> A : None> Tuberculosis
32. b. Hamartoma 33. c. Streptococcus 34. b. It is highly contagious 35. b. Allergic Bronchopulmo...
36. a. Legionella 37. a . Iatrogenic pneumothorax 38. d . Good prognosis 39. a. Bronchophonia
103
Marwah's Internal Medicine MCQs ( Based on Harrison's 19th)

I Unipolar flagellate organism that causes pneumonia?


a. Pseudomonas
b. Mycoplasma
[NEET Pattern 2015-16]
47. Hamman's crunch sign is seen in:
a.
b.
Hamman rich syndrome
Aortic aneurysm
[NBE Pattern 2014-15 ]

c. Aeromonas c . Pneumo- Mediastinum


d. Klebsiella pneumonia d. Pneumothorax
RefiHarrison 18th edn, Ch 132, Harrison 19th p 1044- 45 Ref Harriso n's 17th
ed. p. 1569, Harriso n 19th pi 720

Treatment of spontaneous pneumothorax is: 48. Which is not correct about aspiration pneumonia?
a. 1PPV [ NBE Pattern 2014-15] a. 5 -15% of community acquired pneumonia is due to
b. Closed drainage aspiration pneumonia [AIIMS May 2013]
c. Simple needle aspiration b .
Anaerobic infection is most common
d . Thoractomy c. Superior segment of apex of lung is commonly
involved in recumbent position
Ref:CMDT 2014 p. 313, Harrison 19th p 1719
d. 20- 25ml of fluid less than pH 2.5 is required for
42. Pneumothorax occurs in all except: aspiration pneumonia
a. Langhans cell histiocytosis [NBE Pattern 2014-15] Ref FISHER Textbook of Pulmonology.
b. Marfan’s syndrome 49. Consider the following statements:

System c.
d.
Assisted ventilation
Bronchopulmonary aspergillosis
Ref:Harrison's 18th ed. ch. 144 p. 944, Harrison 19th p 1346 -47
43. Hypersensitivity pneumonitis is associated with:
The features of severe asthma include
1. Central cyanosis
2. Disturbance of consciousness
3. Pulsus paradoxus
[NBE Pattern 2014-15]

4. Heart rate less than 60 / minute

Respiratoy
a. Hilar lymphadenopathy [ NBE Pattern 2014 -15]
Which of these statements are correct?
b. Raised IgE
a. 2, 3 and 4 b. 1, 3 and 4
c. Raised serum precipitins
c. 1, 2 and 3 .
d 1, 2 and 4
d. Increased eosinophils
Ref.:Harrison 19th p 1697 - 98
RefCMDT 2014 p. 310, Harrison 19th p 1681 .
50 Breath sounds are decreased in following except:
44. Round pneumonia is seen with [ NBE Pattern 2014-15] a. Lobar pneumonia [ NBE Pattern 2014-15]
a. Streptococcal pneumonia b. Pneumothorax
b. Kerosene oil aspiration c. Pleural effusion
c. Lung cancer d. Atelectasis
d. Mendelson syndrome Lobar pneumonia : RefCMDT 2014 p. 1422 .
RefCMDT 2014 p. 270, Harrison 19th p 951 .
51 In tension pneumothorax first line of management is:
45. Which statement is true regarding pneumothorax: a- CXR [ AIIMS May 2013]
a. Absent breath sounds [ NBE Pattern 2014-15] b. Needle in 2nd intercostal space
c. Emergency thoracotomy
b.Increased percussion note
d. ICD tube in 5th interocostal space mid axillary line
c .Always needs chest tube insertion
RefSabiston Textbook of Surgery 18 ed., Harrison 19th p
d. Tracheal tug “1719, 1750
Ref Harrison's 18th ed. ch. 144, Sec. 9 p. 944, Harrison 19th p
1719, 1743 52. In Pneumonia severity index in adults,most important
46. The most common cause of spontaneous
factor is? [NBE Pattern 2014-15]
a' b. Congestive heart failure
pneumothorax is:
c. Hyponatremia d. Hypothermia
a. Tuberculosis [ NBE Pattern 2014-15]
b. Rupture of a sub-pleural bleb Ref Harrison’s IB ed. p. 2134, Harrison 19th p 806
"
c. Bronchogenic carcinoma
d. Bronchial adenoma
.
RefCMDT 2014 p 316, Harrison 19th p 1719
Ans . 40 . a. Pseud. 41. c. Simple needle aspiration 42. d.
44 . a. Streptococcal pne.. 45. a. Absent breath sounds
Bronchopulmonary... 43. c. Raised serum precipitins
46. b. Rupture of a sub-pleu... 47.
48 . c. Superior segment.. 49. c. 1, 2 and 3 c . Pneumo-Mediastinum
104 50. a. Lobar pneumonia 51. b. Needle in 2nd inter...
52. a . Age
Respiratory System

53. Causes of hypersensitivity pneumonitis is? 60. AH of the following features are seen in the viral
a. Silicosis [ NBE Pattern 2014 -15] pneumonia except : [AI 2005]
b. Farmer's lung a. Presence of interstitial inflammation .
c. Anthracosis b. Predominance of alveolar exudates.

1
d. Asbestosis c. Bronchiolitis.
Ref:CMDT 2014 p. 310, Harrison 19th p 1681 d . Multinucleate giant cells in the bronchiolar wall.
Ref : Robbins 7th/ 751, Harrison 19 th p 1207 - 10
54. X- ray finding of Staphylococcus pneumonia are A / E:
a . Involvement of lobar lobes [ NBE Pattern 2014-15] All of the following statements about Pneumocystis
b. Hilar lymphadenopathy Jiroveci are true Except: [AI 2008]
c. Empyema a. Usually associated with CMV infection
b. May be associated with Pneumatocele
d. Visible air bronchogram
c. Usually diagnosed by sputum examination
RefCMDT 2014 p. 270, Harrison 19 th p 805 - 06 d. Causes disease only in the immunocompromised
55. Caplan Syndrome is Pneumoconiosis with host
a. Lymphadenopathy Ref : Washington manual of Pulmonary Medicine [2006]/ 104 ;Jawetz
b. Congestive Cardiac Failure 24th/ 648,649, Harrison 19thp 372e, 756, 1358
c. Rheumatoid Arthritis 62 . Drug of choice for Mycoplasma pneumoniae is :

Respiratoy
d. HIV a. Penicillin [AIIMS Dec 94, May 94]
Ref : Harrison's 18th/ 2739-2125, Harrison 19 th p 1690, 2138 b. Tetracycline
c. Cefuroxime
56. Diagnostic features of Allergic Bronchopulmonary
d. Erythromycin
Aspergillosis [ABPA] include all of the following
Ref : Harrison 18th/ p 1419, Harrison 19 th p 1163 -64
except:
a. Changing pulmonary infiltrates 63 . Commonest sign of aspiration pneumonitis is :
[AI 2003]
b. Peripheral eosinophilia a. Cyanosis [AIIMS Dec 94]
c. Serum precipitins against Aspergillosis fumigants b. Tachypnea
c. Crepitations

System
d. Occurrence in patients with old cavitatory lesions.
d. Rhonchi
Ref : Harrison 18th/ p 2120, Harrison 19 th p 1346 - 47
Ref : Clinical / Anesthesiology 3rd/ 250, Harrison 19 th p 1098
57 . Devi , a 28 year female, has diarrhoea , confusion, high
grade fever with bilateral pneumonitis. The diagnosis 64. While inserting a central venous catheter, a patient I
is: develops respiratory distress . The most likely cause |
a. Legionella [AI 2000] is: [AI 2002] j
b. Neisseria meningitis a. Hemothorax b. Pneumothorax
c. Streptococcus pneumoniae c. Pleural effusion d. Hypovolumia
d. H. influenza Ref : Harrison 19 th p 1719
Ref : Harrison 18th/ 1237, 1238; Harrison 19th p 1014 - 16 65. In left sided massive pneumothorax, ECG shows
all,
except: [AIIMS May 94]
58 . Fever persisting even after treatment of pneumonia ,
a. Left axis deviation b. Absent R wave
likely diagnosis is: [AIIMSJune 97] c. Peaked P wave
a. Empyema b. Fungal lesion d. Precordial T wave inversion
c. Carcinoma bronchus d. Any of the above
Ref : Harrison 18th edn, Ch 233, Harrison 19 th pi 719, 1743
2Ref : Harrison 18th/ p 2136, Harrison 19 th p 135 - 138
66 . Hypersensitivity Pneumonitis is classically des
59 . Most common cause of empyema is: [AIIMS Dec 97] cribed
a. Bronchopleural fistula as a: [AI 2009]
b. Tubercular pneumonia a. Type I hypersensitivity reaction
c. Bacterial pneumonia b. Tye II hypersensitivity reaction
d. Pleurisy c. Type III [ Immune complex] Hypersensitivity
Ref : Harrison 18th/ p 2179, Harrison 19th p 968, 1098- 99 d. Type IV [Cell mediated] Hypersensitivity
Ref : Harrison 18th/ p 2116, Harrison 19 th p 1681
Ans. 53. b . Farmer ’s lung 54 . b . Hilar lymphadenopathy 55 . c . Rheumatoid Arthritis
56 . d . Occurrence in patients . . .
57 . a . Legionella 58 .
61 . a . Usually associated . . 62.
d . Any of the above
d . Erythromycin
59 . c . Bacterial pneumonia 60 . b. Predominance of alveolar..
63 . b . Tachypnea 64. b . Pneumothorax
I
65 . a . Left axis deviation 66 . c . Type III [Immune complex] Hypersensitivity, d .Type IV [Cell mediated]. . . 105
Marwah's Internal Medicine MCQs ( Based on Harrison’s 19th )

I 67.
Asthma

In a patient P 02 is 85 mmHg, PC02 - 50 mmHg, pH is


c. VP mismatch with dead space ventilation
d. Faulty pulse oxy- meter
RefRogers handbook of Pediatric intensive Care - fourth
edition

P
page no : 183. And Ch 143: nelson 18th
7.2 and HC03 is 32 meq / 1 is suffering from : edn
72 . One of the following is not an indicator of the severity
a . Respiratory acidosis with compensatory metabolic
of asthma [NBE Pattern 2014 -is]
alkalosis [AIIMSJune 2000]
a. Use of accessory muscles
b. Respiratory acidosis with compensatory metabolic
acidosis b. Pulsus paradoxus
c. Metabolic acidosis c. Cyanosis
d . Metabolic alkalosis d. Systolic BP
RefCMDT 2014 p. 293, Harrison 19 th p 1675
Ref: H 18/ e p363, 17/ e pi 590, 288, 289, Harrison 19 th p 322
73 . ABPA complicates: [ NBE Pattern 2014 -15]
I 68. A 10 year old boy with short stature presents with a. Cystic fibrosis
polyuria and polydipsia . Laboratory values reveal a b. TB cavity
pH of 7.4; HC03 of 17; Na + 140; pC02 32; K + 4.9; Cl - c. Bronchiectasis
112 . Likely acid base abnormality is:

System
d . Pneumoconiosis
a. Metabolic Alkalosis [AIIMS Nov 2010 ] RefCMDT 2014 p. 266, Harrison 19 th p 1346 -47
b. Non -Anion gap Metabolic Acidosis
c. Anion gap Metabolic Acidosis 74. Consider the following statements: [UPSC 2010]
d . Respiratory Acidosis Life threatening features of acute -severe asthma in
Ref: Harrison' s 18/ e p363, 365, Harrison 19 th p 1808 children include?

Respiratoy
1. Altered sensorium
69 . In a patient with bronchial asthma silent chest
2. Pulsus paradoxus
signifies:
3. Audible wheeze in both inspiration and expiration
a. Good Prognosis [DNB 2010]
4. Oxygen saturation 92 -95%
b. Bad Prognosis
c. Grave Prognosis Which of these statements is/ are correct?
d. Not a Prognostic sign a . 1 only
b. 2 and 4
Ref: Harrison's 18/ e p2109, Harrison 19 th p 1676
c. 1 and 2
d. 1 , 2 , 3 and 4
70. FEV1 / FVC is decreased in : [ DNB 2013] RefCMDT 2014 p. 244, Nelson ch 143 and British Asthma guide ¬

a. Asthma lines 2013., Harrison 19 th p 1679


b. Kyphosis
75. Bronchial asthma is associated with raised levels of:
c. Scoliosis
a . Leukotrienes [ NBE Pattern 2014 -15]
d. Fibrosis
b. PGI 2
Ref: Harrison's 18/ e p2092, Harrison 19 th p 1676
c. PGE 2
71. Child known case of bronchial asthma comes with d. Thromboxane
respiratory rate 48/ min, cannot speak 2 words, Ref:CMDT 2014 p. 243, Harrison 19th p 1669
occasional wheeze and oxygen saturation of 95%.
76. Aspirin sensitive asthma is associated with:
You give 3 doses of salbutamol nebulisation then he
a. Extrinsic asthma [ NBE Pattern 2014 -15]

i started to speak a sentence but saturation falls to


85%. Cause is?
a. Bronchomalacia
[AIIMS Nov. 2014]
b. Usually associated with urticaria
c. Associated with nasal polyp
d. Obesity

I A ns.
b. Right to left shunt

67. a. Respiratory a... 68. b. Non -Anion gap... 69. c. Grave Prognosis
,
RefCMDT 2014 p. 252 Harrison

70 . a. Asthma
19 th pi 680

71. c. VP mismatch ... 72. d. Systolic BP 73. a . Cystic fibrosis 74. a. 1 only
106 75. a. Leukotrienes 76. c. Associated with nasal polyp
Respiratory System

77. A pediatric asthmatic patient presents with a severe 82. In severe bronchial asthma, true is:
attack of acute wheezing and breathlessness and a. Inspiratory and expiratory rhonchi with reduced
drowsiness. Arterial blood gas analysis done after one sounds [ NBE Pattern 2014 -15]
hour of treatment with oxygen, nebulised salbutamol b. Hyper- resonant chest
and intravenous cortiocosteroids show pH 7.26, Pa
02 c. Increased fremitus and absent breath sounds
of 60 mm Hg and PaC02 of 60 mm Hg. The next step in d. Decreased fremitus and crackles
treatment should be: Ref:CMDT 2014 p. 244, Harrison 19th p 1675, 1679
a. Mechanical ventilation [AIPG 2011 ] 83. All of the following are useful for treating acute
b. Intravenous bicarbonate
bronchial asthma in children except:
c. Intravenous salbutamol
a. 100% Oxygen [ NBE Pattern 2014-15]
d. Increase in rate of oxygen delivery
b. Hydrocortisone function
Ref.:143, Harrison 19th p 807 - 809
c. IV aminophylline
78. Consider the following statement:
d. Sodium cromoglycate inhalation
Early onset extrinsic episodic asthma is characterized
Ref.CMDT 2014 p. 252, Harrison 19th p 1679
by [ NBE Pattern 2014 -15]
1. Family history of eczema or rhinitis 84. The following are interstitial lung disease except:
-

Respiratoy
2. Development of an early and late asthmatic reaction a. Sarcoidosis [ NBE Pattern 2014 15]
mediated by mast cells b. Fibrosing alveolitis
3. T lymphocytes that release cytokine like c. Bronchial asthma
interleukin-4 d. Pneumoconiosis
Which of these statements are correct? RefiCMDT 2014 p. 292, Harrison 19th p 1708 - 10
a. 1,2 and 3 b. 1 and 2 85. Commonly used route of administration for
c. 2 and 3 d . 1 and 3 Omalizumab in asthma in: [NBE Pattern 2014-15]
Ref:CMDT 2014 ch. 9 p. 240 -242, Harrison 19th p 1679 -80 a. Subcutaneous
79. b. Inhalationa!

System
Obstructive sleep apnea is defined as
number of apnea events/ hour? [ NBE Pattern 2014 -15] c. Intradermal
a. 2 b. 3 d . Intramuscular
c. 4 d. 5 .
Ref:CMDT-2014 p 252, Harrison 19th p 1678
RefCMDT 2014 p. 318, Harrison 19th pi 724-25 86. Bronchial asthma patient on artificial ventilation
80. The results of the pulmonary functions tests shown requires: [NBE Pattern 2014- 15]
below, the best diagnosis is : [ NBE Pattern 2014 -15] a. A low Inspiratory Flow
b. An equal IE ratio of 1:1
I Parameters Actual Predicted
c. An inverse ration ventilation
FEVl [L] 1.2 3.5-4.3
d. An IE ratio 1:2.5
FVC [L] 4.1 4.6-S.4
FEV1/ FVC [%] 29 72-80 .-
Ref CMDT 2014 p. 242,
PEF [L / min] 80 440 -540 87. Asthma is precipitated by A/ E: [ NBE Pattern 2014 15]-
DL CO 120% 100 % a. Aspirin b. bj Blocker
a. Asthma b. Asbestosis c. Suxamethonium d. Gold
c. ARDS d. Silicosis Ref:CMDT 2014 p. 243, Harrison 19th p 1680
Ref:CMDT 2014 p. 258, Harrison 19th p 1679
88. Curschmann's Spirals in sputum is seen in:
81. Best for treatment of Exercise induced asthma? a. Tuberculosis cavity [ NBE Pattern 2014-15]
a. Montelukast [NBE Pattern 2014-15] b. Asthma
b. Salbutamol c. Bronchitis
c. Ipratropium d. Bronchiectasis
d. Low dose Inhaled corticosteroids Ref:CMDT 2014 p. 244, Harrison 19th p 1675-76
“ .
Ref:Harrison's 18 ed ch. 254, Harrison 19th p 1671

Ans. .
77. a. Mechanical ventil. 78. a . 1,2 and 3 79. d . 5
~
80 a Asthma
81. d. Low dose Inhaled.. 82. a. Inspiratory and expiratory 83. d . Sodium cromogly..
'

85. a . Subcutaneous 86. d . An IE ratio 1:2.5


. 84 c. Bronchial asthma
87. a. Aspirin 88. b. Asthma
L Marwah 's Internal Medicine MCQs ( Based on Harrison's 19th )

All of the following statements about intrinsic allergic 95. Characteristic ECG finding of pulmonary
embolism:
Asthma are true, Except:
a. Nasal polyp
b. Normal lgE
[PGI 2009]
b - W .
a. Sinus tachycardia

c. T wave inversion
[NBE Pattern 2014-151

c. Family history Positive d . Epsilon waves


d. More aggressive RefCMDT 2014 ch. 9, Harrison's 18th ed. ch. 298
e. Allergic in nature Harrison 19th p 1633, 101
Ref.: Harrison 18th / p 2102, Harrison 19 th p 1669 96 . All are seen in massive pulmonary embolism except?
Aspirin- sensitive asthma is associated with: [A11995] a . Inter-ventricular septum deviation
a. Obesity b. Urticaria b. FallofSBP [NBE Pattern 2014-15]
c. Nasal polyp d. Extrinsic asthma c. Pulmonary plethora
Ref : Harrison 18th/ p 2102, 2115, Harrison 19 th p 1680 d. Elevated JVP
.
RefCMDT 2014 p 297, Harrison 19th p 1631 -33
91. Universal finding in Asthma is [ PGI June 02]
a. Hypoxia 97 . The point which distinguishes ARDS from cardiogenic
b. Hypercarbia pulmonary edema is: [NBE Pattern 2014-15]

System
c. Respiratory acidosis a. Normal P02
d. Metabolic Acidosis b. Normal pulmonary arterial pressure
Ref : Harrison 18th/ 2109; Harrison 19 th p 1669 -70 c. Normal arterial alveolar gradient
d. Normal PC 02
92. All are used in bronchial asthma, except :
RefCMDT 2014 p. 407, Harrison 19 th pi 763
a. Salbutamol [AUMSJune 97]

Respiratoy
b. Morphine 98. Most diagnostic of pulmonary emboli is:
c. Aminophyiline a . CTA [ NBE Pattern 2014-15]
d. Steroid b. V/ P scan
Ref : Harrison 19 th p 1669-70 c. HRCT
d. D- Dimer Assay
Pulmonary Embolism RefCMDT 2014 p. 299, Harrison 19 th p 1632 -34

93 . Ventilation perfusion scanning to be done in a 99 . Pulmonary embolism causes all except:


pregnant patient. All are the following are ture a . Bradycardia [ NBE Pattern 2014 -15]
regarding V/ P scan except: [JIPMER 2014] b. Decreased cardiac output
a. Normal scan rules out clinically significant c. Arterial hypoxaemia
pulmonary thromoesmbolism d . Acute right ventricular strain
b. VP scan contraindicated in pregnancy RefCMDT 2014 p. 257, Harrison 19 th p 1631 -32
c. An abnormal ventilation and perfusion is usually
100. Most common source of pulmonary embolism is:
due to conditions other than PTE.
d . Two or more segmental perfusion defects with
a. Atherosclerosis [NBE Pattern 2014-15]
b. Fracture fixation
normal ventilation has high probability of PTE
c. Pelvic surgery
Ref. Harrison's Textbook of Internal Medicine, 17th edn. Ch -247,
d . Cardiothoracic surgery
Fishman's Textbook of Pulmonary Diseases and Disorders, 4th
RefCMDT 2014 p. 297, Harrison 19th p 1631 -32
edn. Pg. 548-552. Harrison 19 th p 45
94. All of the following conditions may predispose to 101. All of the following are true of pulmonary embolism
pulmonary embolism except: [AI 2003]
except: [ NBE Pattern 2014 -15]
a. Protein S deficiency a. Sudden onset of pleuritic pain and haemoptysis and
b. Malignancy hypotension
c. Obesity b. ECG shows evidence of acute left ventricular stress
d . Progesterone therapy c. Blood LDH and SGOT levels are raised
Ref : Harrison's 18/ e p2170,2171, Harrison 19 th p 1631 d . Isotope perfusion ventilation scan is diagnostic
RefCMDT 2014 p. 300, Harrison 19 th p 1633, 101

I Ans. 89. c. Family history Positive, e. Allergic in nature


92. b. Morphine
.
90. c Nasal polyp 91. a. Hypoxia
93. b. VP scan contraindicat... 94. d . Progesterone therapy 95. b S1Q3T3
96. c. Pulmonary pletho.. 97. b. Normal pulmonary...
.
108
100. c. Pelvic surgery
98. a CTA . .
99. a Bradycardia
101. b. ECG shows evidence of ...
Respiratory System

«
102. Pulmonary embolism is seen in all except: b. Radioisotope perfusion pulmonary scintigraphy
a. Fanconi anemia [ NBE Pattern 2014 -15] c. EKG
b. Paroxysmal nocturnal haemoglobinuria d. Venography
c. Oral contraception
Ref : Harrison 18th/ p 2173, 2174,
d . Old age
Harrison 19th p 1634, 1667 -68
RefCMDT 2014 p. 297, Harrison 19 th p 1631 - 32
103. Most common source of pulmonary embolism is: 111. Bilateral Rhonchii may be seen in all of the following
a. Large vein thrombosis [ NBE Pattern 2014-15] Except [ PGI June 08]
b. Deep vein thrombosis a. Pulmonary Edema
c. Superficial venous thrombosis b. Bronchiectasis
d . Saggital sinus thrombosis c. Pulmonary Embolism
RefCMDT 2014 p. 297, Harrison 19 th p 1631 - 32 d . Emphysema
104. Air embolism is diagnosed by: [ NBE Pattern 2014 -15] Ref : Harrison's 17th/ 1584, Harrison 19 th p 1631
a . jEnd tidal C02 b. jEnd tidal N 2
c. Doppler study d. Ultrasound COPD
RefCMDT 2014 ch. 9, Harrison 19 th p
105. Most common symptoms of pulmonary embolism: 112 . A man working in coal mining factory for 16 years

Respiratoy
a. Chest pain develops symptoms of progressively worsening
[ NBE Pattern 2014 -15]
b. Dyspnea breathlessness and cough with expectoration . A
c. Haemoptysis spirometry was performed and his values were as
d. Cough follows FEV1- 1.4 1/min, FVC: 2.8 1/min and with
RefCMDT 2014 p. 297, Harrison 19 th p 1632 - 33 FEV1 / FVC ration of 50. What could be the cause?
a. Silicosis [JIPMER 2014]
106. The dome of the diaphragm is elevated in: b. Hypersensitivity pneumonitis
a. Typhoid fever [ NBE Pattern 2014 -15] c. COPD
b. Pulmonary infarction d . Idiopathic pulmonary fibrosis
c. Emphysema
Ref : Harrison's Textbook of Internal Medicine, 17th edn. Ch-250,

System
d. Cirrhosis
Fishman's Textbook of Pulmonary Diseases and Disorders, 4th
RefCMDT 2014 ch. 9, Harrison 19 th p 1632
edn. Pg. 968- 976, Harrison 19 th p 1676
.
107 All are true about pulmonary embolism, except :
113 . Caplan Syndrome is Pneumoconiosis with:
a. Chest pain is the most common symptom[AIIMS May 94]
b . Most commonly presents within 2 weeks
a. Lymphadenopathy
c. More is the survival time, more is the chance of b. Congestive Cardiac Failure
recovery c. Rheumatoid Arthritis
d. Arises from leg veins d. HIV
Ref : Harrison 18th/ p 2172, Harrison 19th p 1631 -32 Ref Harrison's 18/ e p2739, Harrison 19 th p 1690
108. All of the following conditions may predispose to 114. False about obstructive sleep apnea . [AIIMS Nov. 2014 ]
pulmonary embolism except: [ AI 2003] a. It affects more women
a. Protein S deficiency b. Malignancy b. Associated with hypertension
c. Obesity d. Progesterone therapy c. Day time sleepiness
Ref : Bailey 23rd/ 263; Harrison 19 th p 1631 -32 d. > 5 episodes / hour
109. In acute pulmonary embolism, the most frequent ECG Ref:Harrison 18th edn, ch 264, Harrison 19 th p 1723 - 1725
finding is : [AIIMS May' 06] 115. Finger in glove sign is seen in? [ NEET Pattern 2015-16 ]
a. SaQ3T3 pattern b. P. pulmonale a . Pulmonary alveolar Proteinosis
c. Sinus tachycardia d. Right axis deviation b. Pneumocystis Carinii
Ref : Harrison 18th/ p 2172, Harrison 19 th p 1633 c. Tuberculosis
110 . The most definitive method of diagnosing pulmonary
d. Bronchocele
embolism is : RefCIinical imaging Atlas, 5th edn, page 39.
[AIIMS Nov. 05]
a. Pulmonary arteriography

Ans. 102. a. Fanconi anemia 103.


...
106. b. Pulmonary infar 107.
110. a . Pulmonary arteri... 111.
.
114. a. It affects more.. 115.
a. Large vein thrombosis 104. c. Doppler study

c. Pulmonary Embolism
d . Bronchocele
112. c. COPD
105. b. Dyspnea
a. Chest pain is the most 108. d . Progesterone therapy 109. c. Sinus tachycardia
113. c. Rheumatoid Arthritis
J
1 M
Marwah's Internal Medicine MCQs ( Based on Harrison's 19 th
)

116. Central bronchiectasis is seen with ? 122. On an ABG, pH of 7.2, p02 of 46, pC of 80 are
02 indicative
a. Cystic Adenomatoid Malformation k [NBE Pattern 2014 -15]
b. Cystic fibrosis [NEET Pattern 2015-16 ] °
a. Acute exacerbation of COPD
c. Broncho carcinoma b. Adult respiratoiy distress syndrome
d. Tuberculosis c. Chronic bronchitis
1 Ref:Harrison 18th edn, ch 258, Harrison 19th p 1694- 95
117. Alveolar hypoventilation is observed in:
a. Gullian -Barre syndrome [ NBE Pattern 2014-15]
d. Acute asthma
RefCMDT 2014 p. 259, Harrison 19th p 1676 -77

b. Acute asthma
123. Which of the following is not true in obstructive lung
c. Bronchiectasis
disease? [NBE Pattern 2014-15]
a. FEV1J.
d. CREST syndrome
b. TLC|
RefCMDT 2014 p. 318, Harrison 19th p 2694 95 - c. FVC J.
118. A 60 year old man is being treated for pulmonary d. Reduced timed vital capacity
emphysema. He is admitted with laboured breathing Ref:CMDT-2014 ch. 9, Harrison 19th p 306 e
at rest with marked use of accessory muscles. Arterial
124. Which of the following is most likely to be associated
blood gas analysis revealed: [ Bihar PG. 2014] with bronchiectasis of the main bronchi:

System pH
PaC02
Pa 02
HCO 2
Possible diagnosis
7.33
64 mmHg
50 mmHg
43 mEq / L
a. Allergic bronchopulmonary aspergillosis
b. Endobronchial tuberculosis [ NBE Pattern 2014 -15]
c. Measles
d. Chronic bronchitis
RefCMDT-2014 p. 264-265, Harrison 19th p 1695

Respiratoy
a. Compensated metabolic alkalosis
125. Best method for detecting minimal bronchiectasis is:
b. Chronic compensated respiratory acidosis
c. Acute respiratory acidosis a. Bronchogram [ NBE Pattern 2014-15]
d. Hyperventilation is the main factor b. CT scan
Ref Acidosis and Alkalosis, Harrison's textbook of internal medi¬ c. Radionuclide lung scan
-
cine, ch. 47, Harrison 19th p 322 323 d. Chest X-ray
RefCMDT 2014 p. 265, Harrison 19th p 308e; 1694
119. Which product of cigarette smoke is responsible for
126. Most likely precursor to bronchiectasis is:
CAD? [ NBE Pattern 2014 15]
a. Nicotine a. Tuberculosis [ NBE Pattern 2014 -15]
b. Tar b. Carcinoma
" c. Bronchial adenoma
c. Polycyclic aromatic hydrocarbons
d. Benzene d. Necrotising pneumonia
-
RefCMDT 2014 p. 772, Harrison 19th p 663, 678 RefCMDT 2014 p. 265, Harrison 19th p 1694 95 -
iif
|
%|
120. For diagnosis of obstructive airway disease, which of
the following measurement is usual:
a. Vital capacity [ NBE Pattern 2014 -15]
127. Subcutaneous Emphysema may be found in the
following conditions:
a. Tracheostomy
[NBE Pattern 2014-15]
b. Hemilich maneuver
en c. Chest injury d. All of the above
b. Timed vital capacity
c. Tidal volume [ Ref : www.nlm.nih.gov-google, Harrison 19th p 1703-04
d. Blood gas analysis 128. The most common cause of non progressive -
RefCMDT 2014 p. 258, Harrison 19th p 306e-2 pnuemothorax is: [NBE Pattern 2014 -15]

I 121. All are complications of bronchiectasis except:


a.
b.
Cerebral abscess
Lung abscess
[NBE Pattern 2014 -15]
a . Rupture of subpleural bleb
b. Tuberculosis
c. Trauma
d. Bronchiectasis

I c.
d.
Amyloidosis
Bronchogenic carnicoma
RefCMDT 2014 p. 265, Harrison 19th p 1696
Ref:Harrison's 18th ed. ch. 144 p. 944, Harrison 19th p 1719 -20

ft Ans. 116.
120.
124.
b. Cystic fibrosis . -
117 a. Gullian Barre syndrome
b. Timed vital cap.. 121. d. Bronchogenic carnicoma
a. Allergic bronch... 125. b. CT scan
118. b. Chronic comp... 119. d. Benzene
122. a . Acute exacerba... 123. b. TLC|
...
126. d. Necrotising pn 127. d. All of the above
110
I 128. a. Rupture of subpleural bleb
Respiratory System

129. All are seen in emphysema except:


a. Decreased vital capacity
b. Hyperinflation
[NBE Pattern 2014-15]
b.
c.
d.
Decreased timed vital capacity
Increased residual volume
Increased diffusion capacity
I
c. Rhonchi RefCMDT 2014 p. 259, Harrison 19th p 306 e
d . Reduced DLco
137. Bronchiectasis is most common in:
Ref Harrison's 17th ed. p. 1550, Harrison 19th p 1664, 1701 a. Right middle lobe [ NBE Pattern 2014-15]
130. The most common cause of bronchiolitis is: b. Right upper lobe
a. Respiratory syncytial virus c. Left lower lobe
b. Adenovirus [NBE Pattern 2014-15] d. Left upper lobe
c. Herpes virus Ref:http:// www.ncbi.nIm.nih.gov/ pmc/ articIes/ PMC3005318;
d. Influenza virus CMDT 2014 p. 265, Harrison 19th p 1694- 95
RefiCMDT 2014 p. 269, Harrison 19th p 1205-06
138. Persistent coarse crepitations in the chest is
131. Emphysema presents with all except: diagnostic of: [NBE Pattern 2014-15]
a. Cyanosis [NBE Pattern 2014-15] a. Pulmonary TB
b. Barrel shaped chest b. Pulmonary oedema
c. Associated with smoking c. Cavity lesion

Respiratoy
d. Type 1 respiratory failure d. Bronchiectasis
Ref:CMDT-2014 p. 260, Harrison 19th p 249, 1740 -41 RefCMDT 2014 p. 265, Harrison 19th p 1694 - 95
132. Parents of a child with bronchiectasis may give a past
139. All are true in definition of chronic bronchitis except:
history of: [ NBE Pattern 2014 -15]
a. Cough > 2 months [NBE Pattern 2014-15]
a. Chickenpox
b. Bronchorrea
b. Mumps
c. Hoover sign
c. Whooping cough
d . Haemoptysis
d. Typhoid
RefCMDT 2014 p. 259, Harrison 19th p 1701 -04

System
RefiCMDT 2014 p. 265, Harrison 19th p 1694- 95
140. The complication least likely to occur in a case of
133. Investigation of choice to distinguish between COPD
with emphysema and bronchial asthma is?
chronic bronchitis is: [ NBE Pattern 2014- 15]
a. Allergy test to pollens [NBEPattern 2014-15] a. Pulmonary hypertension
b. Spontaneous Pneumothorax
b. Non reversible air flow obstruction
c. Respiratory acidosis
c. Chest X- ray
d. Amyloidosis H
d. Arterial blood gas analysis
RefCMDT 2014 p. 259, Harrison 19th p 1701 -04
Ref:CMDT 2014 p. 259, Harrison 19th p 1676
134. Not a CT finding in bronchiectasis: 141. IOC for Bronchiectasis: [NBE Pattern 2014-15] |
a. Tree in bud appearance [ NBE Pattern 2014-15] a. HRCT scan
b. Crazy paving appearance b. Spiral CT
c. Signet ring appearance c. Bronchoscopy
d. Traction bronchiectasis with lung fibrosis d. Pulmonary angiography
RefCMDT-2014 p. 265, Harrison 19th p 1694- 95
RefCMDT 2014 p. 265, Harrison 19th p 308e; 1694 95 -
135. If FEV1 is 1.3 L, FVC is 3.1 L in an adult man, the 142. Blood gas measurements of a patient shows the
pattern is suggestive of: [ NBE Pattern 2014-15] following values pH 7.2, pC02 80 mm Hg, p02 46 mm
a. Normal lung function Hg. Which of the following could be the most probable
b. Restrictive lung disease diagnosis: [AIIMS Nov 2000]
c. Obstructive lung disease a. Acute asthma
d. None of the above RefCMDT 2014 pg. 258. b. Acute exacerbation of COPD
136. False about emphysema is: [ NBE Pattern 2014 -15] c. ARDS d. Severe pneumonia
a . Decreased FEV1 Ref : Harrison 18th edn., Harrison 19th p 1701-04

Ans. 129. c. Rhonchi 130. a. Respiratory syncyt.. 131. a. Cyanosis 132. c. Whooping cough
133. b. Non reversible.. 134. b . Crazy paving appea.. 135. c. Obstructive lung , 136. d. Increased diffusion
137. c. Left lower lobe 138. d. Bronchiectasis 139. d. Haemoptysis 140 . b. Spontaneous Pneumotho..
141. a . HRCT scan 142. b. Acute exacerbation of COPD
th )
Marwah 's Internal Medicine MCQ s ( Based on Harri son 's 19

Bronchiectasis is most common in which lobe: 148. Ramesh 40 yrs male patient presenting with polyurh
Right upper lobe [AllMS May 95] pain abdomen, nausea, vomiting, altered

a
senso rium
Right middle lobe was found to have bronchogenic carcinoma.
The
electrolyte abnormality seen in him would be:
Left upper lobe
a. Hypokalemia [AIIMS May 02]
Left lower lobe
b. Hyperkalemia
Ref.: API Medicine 6 th/ 236 , API 7th / 306, c. Hypocalcaemia
Harrison 19th p 1694 95 - d. Hypercalcemia
144. Which of the following is NOT a complication of Ref: Harrison's 18/ e p3100, 743, Harrison 19th p 314
bronchiectasis : [AIIMS Sept 96, Al 1998] 149. 65 year old man presented with hemoptysis and stage
a . Lung abscess 3 clubbing. The probable diagnosis of the patient is:
b. Lung cancer a. Non -small cell lung Cancer [NBE Pattern 2014 -15]
c. Amylodosis b. Small cell cancer of lung
d . Empyema c. Tuberculosis
Ref : API 8th / 374, 375; Harrison 19th p 1694 95 - d . Sarcoidosis
145. 260 . Pulmonary Compliance is decreased in all of the
Ref:CMDT-2014 p. 1598, Harrison 19th p 519 21 -

System following conditions, Except:


a. Pulmonary Congestion
b. COPD
c. Decreased Surfactant
d. Pulmonary Fibrosis
[AI 2011 ] 150. Bilateral malignant pleural effusion is most often
seen in :
a. Ca breast
c. Mesothelioma
b. Ca-lung
d. Lymphoma
RefHarrison's 18th ed. ch. 263, Harrison 19th p 1717; 510 -12

Respiratoy
Ref : Text below; Harrison 18th Ch 260, 151. Bronchoalveloar lavage is beneficial in the evaluation
Harrison 19th pi 701 -1704 of: [NBE Pattern 2014 -15]
a . Squamous cell cancer lung
Lung Cancer b. Bronchiectasis
c. Bronchopleural fistula
d. Pulmonary alveolar Proteinosis
146 . Which of the following statements about small cell
RefCMDT 2014 p. 296, Harrison 19th pi 714
carcinoma is true? [AI 2009]
a. Bone metastasis is uncommon 152 . Differential diagnosis of a solitary pulmonary nodule
b . Peripheral in location includes all of the following except:
c. Chemosensitive tumor a. Neurofibroma [NBE Pattern 2014 -15]
d. Paraneoplastic syndrome with J. PTH is common b. Hamartoma
c. Tuberculoma
Ref: Harrison's 18/ e p751, 17/ e p551, 552, 553, 554,
d. Bronchial adenoma RefHarrison's 18th ed. p. 748.
Harrison 19th p 375e -2
153 . Smoking causes all cancers except:
147 . All of the following statements about Non Small Cell a. Liver cancer [ NBE Pattern 2014 -15]
Carcinoma of Lung [ NSCCL] are true, except: [AI 2012] b. Oral cancer
a. Contralateral mediastinal nodes are a c. Kidney cancer
contraindication to surgical resection d. Bladder caner
b. Single Agent Chemotherapy is preferred for patient
> 70 years with advanced disease -
Ref:CMDT 2014 p. 772.
c. Squamous Cell Carcinoma is the most common 154. Least common cause of clubbing is:
NSCCL amongst Asian population a. Adenocarcinoma [ NBE Pattern 2014 -15]
d . Gefitinib is most effective for female smokers with b Squamous cell CA
adenocarcinoma on histology c. Small cell CA
Ref: Harrison's 18/ e p751, Harrison 19th p 519 d. Mesothelioma
Ref:CMDT-2014 p. 1595, Harrison 19th p 506 10

Ans. 143. d. Left lower lobe 144. b. Lung cancer


147. d . Gefitinib is m.. 145. b. COPD 146. c. Chemosensitive tumor
148. d . Hypercalcemia 149. a. Non -small cell lung .. 150. c. Mesothelioma
151 d . Pulmonary alveo.. 152. a.
Neurofibroma 153. a. Liver cancer 154. c. Small cell CA
Respiratory System

155. Most common bronchogenic carcinoma is


a . Small cell carcinoma
b.
[AIIMSJune 2000]
Squamous cell carcinoma
162. A 60 year old man is suspected of having bronchogenic
Ca: TB has been ruled out in this pt. What should be
the next investigation- [AI 01 ]
%
c. Mixed cell carcinoma a . CT guided FNAC
d. Adenocarcinoma b. Bronchoscopy and biopsy
Ref.: Harrison 18th/ p 738, Harrison 19th p 507 -08 c. Sputum cytology
156. All of the following statements about small cell d. X- Ray chest
carcinomas are true, Except [PGI -June - 06 ] -
Ref : Harrison 19th p 511 12
a. Commonest Malignancy of lung
b. Associated with paraneoplastic syndrome
163. Serum ACE may be raised in all of the following except :
a . Sarcoidosis [AI 2005]
c. Cause SVC obstruction
d. Chemosensitive b. Silicosis
e. Commonly metastasize to brain c. Berylliosis
Ref : Harrison 18th/ p 738, Harrison 19th p 507-08 d . Bronchogenic carcinoma
Ref : Harrison 18th/ p 2810, Harrison 19th p 506; 511 -12,
157. In a chronic smoker, a highly malignant aggressive
1688, 2206
and metastatic lung carcinoma is :
a. Squamous cell Carcinoma
[AIIMS May 01 ]
164. Ramesh 40 yrs male patient presenting with polyuria, so
b. Small cell Carcinoma
c. Adenocarcinoma
pain abdomen, nausea, vomiting, altered sensorium
was found to have bronchogenic carcinoma. The
^3
Q/J

d. Large cell carcinoma


Ref : Harrison 18th/ p 738, Harrison 19th p 507 -08
electrolyte abnormality seen in him would be: ^mg
a. Hypokalemia [AIIMS May 02]
158. Following hormonal levels are increased in small cell b. Hyperkalemia
carcinoma of lung except : [AI 1997] c. Hypocalcaemia d. Hypercalcemia
a. ACTH
b. Growth hormone
Ref : Harrison 18th/ p 3100, 743, Harrison 19th p 511-12 •3
c. ANF
d . AVP
165. A 60 year old male presented to the emergency with
breathlessness, facial swelling and dilated veins on
%C/5
Ref : Harrison 18th edn/ p 738, 739, 743, Harrison 19th p 507 -08 the chest wall. The probable diagnosis is? o
a. Thymoma
159. Type of lung carcinoma producing superior vena cava
syndrome: [AIIMSJune 97] b. Lung cancer 3
a. Squamous cell carcinoma c. Hodgkin's lymphoma
b. Adenocarcinoma d . Superior vena caval obstruction
c. Small cell carcinoma Ref : Ch 276 Harrison's 18th, Harrison 19th p 1788, 608 -09
d . Anaplastic carcinoma
Ref : Schwartz 8th 562, Harrison 19th p 507 -08, 608-09 ARDS
160. Clubbing is least common in: [AIIMS Dec 97]
166. Most common cause of lung abscess?
a. Squamous cell carcinoma
b. Adenocarcinoma [ NEET Pattern 2015-16 ]
a . Staph aureus b.
Oral anaerobes
c. Small cell carcinoma of lung c. Klebsiella d. Tuberculosis
d. Mesothelioma Ref:Harrison’ s Iff " edn, Ch 258, Harrison 19th p 813, 1098
Ref : Harrison 18th/ p 743, Harrison 19th p 250, 507-08
167. Treatment of choice in cardiogenic shock with pump
161. A patient presents with secondaries to the adrenals. failure is: [NBE Pattern 2014-15]
The most common site of primary is : [AI 2000] a . Dobutamine
a - Lung b. Kidney b. Intracardiac adrenaline
c.
c. Breast d. Stomach Digoxin
Ref : Harrison 19th p 511 d. Intraaortic balloon pumping
-
Ref.CMDT 2014 p. 487, Harrison 19th p 1759 60 -
Ans . 155. d . Adenocarcinoma 156 . a . Commonest Maligna
.. 157 . b. Small cell Carcinoma 158 . b. Growth hormone
159 . c . Small cell care ... 160. c. Small cell carcinoma...
161 . a . Lung 162. b. Bronchoscopy and biopsy
163. d . Bronchogenic .. 164. d . Hypercalcemia 165. d . Superior vena caval ... 166 . b. Oral anaerobes
167 . a . Dobutamine

I
! Marwah 's Internal Medicine MCQs

168. The following are features of adult respiratory


distress syndrome except: [NBE Pattern 2014-15]
th
( Based on Harrison's 19 )

C.Weight reduction and diet control


d. Mandibular reposition surgery

a
a. Hypoxia Ref:Harrison's ch. 265: sleep apnea/ CMDTp 319
b. Hypocapnia Harrison 19th p 1727
c. Low protein Pulmonary edema
d. Stiff lungs RefiCMDT 2014 p. 322. 175. Not true obstructive sleep apnoea:
a. Nocturnal asphyxia [NBE Pattern 2014-15]
Wedge pressure in ARDS is usually:
a. Markedly increased [ NBE Pattern 2014-15]
b. Alcoholism is a cofactor
b. Moderately increased c. Prone to hypertension
c. Normal d . Overnight oximetiy is diagnostic to replace poly ¬

d . Decreased somnography
RefiCMDT 2014 p. 322, Harrison 19th p 1736 - 37 Ref:CMDT 2014 p. 319, Harrison 19 th p 1724 -25

170. All of the following are well recognized predisposing 176. In type II respiratory failure there is:
factors for adult respiratory distress syndrome, a. Low p02 and Low pC 02 [NBE Pattern 2014 -15]
except: [NBE Pattern 2014-15] b. Low p 02 and High pC02
a. Multiple blood transfusions c. Normal p02 and High pC 02
d. Low p 02 and Normal pC 02
£ b. Septicemia
c. Status asthmaticus Ref:Harrison's 18th ed. ch. 16 p. 83, Harrison 19th p 1736 -40
M d . Toxic gas inhalation
177 . Obstructive sleep apnoea may result in all of the
Ref:Harrison's 18thed. ch. 15 p. 82. following except: [NBE Pattern 2014-15]
V5 171. Type II respiratory failure is seen in: a. Systemic hypertension
a. Chronic bronchitis with cor- pulmonale b. Pulmonary hypertension
b. Chronic renal failure c. Cardiac arrhythmia
2 c. Adult respiratory distress syndrome
d. Pulmonary alveolar proteinosis
d. Impotence
Ref:CMDT 2014 p. 319, Harrison 19th p 1724 -25
£ Ref:Harrison's 18th ed. ch. 16 p. 83, Harrison 19th p 1736 -38
178. Which one of the following is common to all forms of
CU 172. True about adult respiratory distress syndrome shock? [ NBE Pattern 2014-15]
Q except: a . Decrease in tissue perfusion
a. pa02/ FiOz ratio < 200 [ NBE Pattern 2014 -15] b. Decrease in left atrial pressure
b. Normal PCWP [< 12 mm Hg] c. Decrease in cardiac output
c. Hypoxia d. Decrease in right atrial pressure
d. Low protein pulmonary edema RefiCMDT 2014 p. 485, Harrison 19 th p 1729-31
Ref: Harrison's 18th ed. ch. 15 p. 81, Harrison 19th p 1736 - 40
179. Respiratory failure type I consists of:
173. Type 3 respiratory failure occurs due to: a. Low Pa 02, normal or low Pa C02
a. Post-operative atelectasis [NBE Pattern 2014-15] b. Raised PaC02, low Pa 02 [NBE Pattern 2014-15]
b. Kyphoscoliosis c. Normal Pa02, low P 02
c. Flail tchest d. Normal Pa 02 and PaC02- high
d . Pulmonary fibrosis Ref:Harrison's 18th ed. ch. 16 p. 83, Harrison 19 th p 1736 -38
RefHarrison 18th edn, ch 267, Harrison 19 th p 1736 - 40
180. All are true about hyaline membrane disease except:
174. 40 year old smoker, obese, hypertension patient is a. Wide spread atelectasis [ NBE Pattern 2014-15]
having loud snoring. On sleep study patient had > 5 b. Absence of air bronchograms
episodes of apnea per hour of sleep at night. After c. X- ray shows ground glass mottling
control of BP and quitting smoking what is the next d. High maternal sugar inhibits surfactant production
best management for improvement of symptoms of
Ref:Harrison's 18th ed. ch. 267.
the patient?
a. C.P.A.P [ NBE Pattern 2014-15]
b. Uvulopalatoplasty

Ans. 168. c. Low protein Pul... 169. c. Normal 170. c. Status asthmaticus 171. a. Chronic bronchitis with ..
172. d. Low protein pu... 173. a. Post-operative... 174. a. C.P.A.P 175. d. Overnight oximetry is...
114
176. b. Low p02 and . .. 177. c. Cardiac arrhythmia 178. a. Decrease in tissue... 179. a. Low Pa 02, normal or low
180. b. Absence of air bronchograms
Respiratory System

181. All of the following are true about type 1 respiratory


failure except:
a. Decreased Pa 02
[ NBE Pattern 2014 -15]
b. High PC 02
c. FEV1 below 50%
d. Very low p02 RefCMDT 2014 p. 260.
I
Ej
b. Decreased PaC 02 189 . In restrictive lung disease: [AIPG 2010]
c. Normal PaC02 a . FEV1/ FVC decreased, compliance normal
d. Normal A-a gradient
b. FEV1/ FVC increased, compliance increased
Ref : Harrison's 18th ed. ch . 16 p. 83, Harrison 19th p 1736 -38
c. FEV1/ FVC decreased, compliance increased
182. In treatment of shock, parameter used to guide d. FEV1/ FVC increased, compliance decreased
treatment is? [NBE Pattern 2014 15] - .
RefCMDT 2014 ch 9, Harrison's 18th ed. ch 258.
a . CVP 190. The typical feature of interstitial lung disease is:
b. Urine output a. End Inspiratory rales [ NBE Pattern 2014 -15]
c. Reflex b. Expiratory rales
d. Eye opening c. Inspiratory rhonchi
RefCMDT 2014 p. 486, Harrison 19 th p 1748 d . Expiratory rhonchi
183 . Correct about ARDS is: [ NBE Pattern 2014 -15] RefCMDT 2014 p. 292, Harrison 19th p 1709 - 10
a. Low tidal volume ventilation 191. Acute respiratory failure does not occur with:
b. High lung compliance a . Porphyria [ NBE Pattern 2014- 15] 73
c. Low protein pulmonary edema b. Myasthenia gravis n
C/l
d. High pulmonary capillary pressure
Ref:Harrison's 18th ed. ch. 267, Harrison 19th p 1736
184. All are correct regarding ARDS except?:
c. Polio
d. Lead poisoning
RefCMDT 2014 p. 319, Harrison 19 th p 2607-08; 472e
--
M

W
1
192. The Epworth scale is used for assessing:
•-
a. Stiff lungs [ AIPG 2010] a. Body mass index [ NBE Pattern 2014 -15] o
b. Hypoxaemia without hypercapnea b. Vital capacity in post-operative patients
J
c. Increase in pulmonary capillary wedge pressure c. Sleep apnea
d. Intrapulmonary right to left shunt d. Risk of embolism in perioperative patient / cs
RefCMDT 2014 ch. 9 p. 322, Harrison 19 th p 1736 -37
<
RefHarrison' s 18th ed. p. 2184, Harrison 19 th p 1725 c/s <
185. Best ventilator strategy for ARDS is? 193. Following pulmonary changes are seen in restrictive
a. C.P.A.P
b. High frequency jet ventilation
[NBE Pattern 2014 -15] lung disease except:
a. TFEV1/ FVC
[ NBE Pattern 2014 -15]
b. J.TLC
3
c. Assisted control mechanical ventilation c. |RV d. tVC
d . Synchronized intermittent mandatory ventilation RefCMDT 2014 p. 258.
Ref:Harrison's 18th ed. ch. 267/ 268, Harrison 19 th p 1736 - 38
194. In type - II respiratory failure, there is : [A1IMS Nov 02]
186 . Duration of apnea in obstructive sleep apnea is a. Low p02 and low pC02
a. <10 sec [ NBE Pattern 2014 -15] b. Low p 02 and high pC
b. > 20sec 02
c. Normal p02 and high pC02 .
c. > 30 sec d . Low p 02 and normal pC .
d. > 60 sec 02
Ref.: Harrison 18th/ p 2200, Harrison 19th pi 736 38
RefCMDT 2014 p. 318, Harrison 19 th pi 724 -
195 . A patient presents with following parameters pH
187 . ARDS includes all except: [ NBE Pattern 2014 -15] 7.5, p C02 30 mmHg, p 102 mmHg and HC 16
a. Hypoxia 02 03
meq / 1. Which of the following correctly describes the
b. Hypercapnia
compensatory mechanism : [ A12010]
c. Non cardiogenic pulmonary edema
d. Normal P.C.W.P a. Respiratory Alkalosis
b. Metabolic Alkalosis
RefCMDT-2014 p. 322, Harrison 19 th p 1736 -37
c. Respiratory Acidosis
188. Restrictive lung disease is associated with
all except? d. Metabolic Acidosis
a. High residual lung volume [ NBE Pattern 2014
-15] Ref.: Oxford Hand book of Medicine 5th/ 684, Harrison 19th p 317

Ans . 181 . d . Normal A -a .. 182. a . CVP 183. a . Low tidal volume. . . 184 . c. Increase in pulmonary...
185. a . C. P.A. P 186 . a . < 10 sec
189. d. FEV1 /FVC incr... 187. b. Hypercapnia 188. a. High residual lung volume
190 . a . End Inspiratory rales 191 . d . Lead poisoning 192. c. Sleep apnea
193. fVC 194. b . Low p02 and high pC
02 195 . d . Metabolic Acidosis
th
Marwah's Internal Medicine MCQs ( Based on Harriso n s 19 )

196 . Arterial blood gas analysis of a patient reveals pH 7.2 , 202 . A patient with bilateral hilar lymphadenopathy on
HCO3 36mmol / L, pC02 60mm of Hg. The abnormality CXR was diagnosed as Sarcoidosis. What is the next
is . [ AIIMSJune 99] & [AI 1996 ] step? [JIPMER 2014 ]
a. Respiratory acidosis with compensatory metabolic a. CT thorax
alkalosis b. Lymph Node Biopsy
b. Respiratory acidosis
c. Serum ACE
c. Respiratory alkalosis with compensatory metabolic
d . Gallium scan
acidosis
d. Respiratory acidosis with compensatory metabolic RefFig. 329 -8, Harrison 18a edn, Harrison 19 th p 2209-2210
acidosis 203. Light criteria for pleural effusion are all except?
Ref.: Harrison 18th / p 363, 364, Harrison 19 th p 322 [ NEET Pattern 2015-16]
a. Effusion protein /serum protein ratio greater than
197 . Features of restrictive lung disease is [AIIMSJune 98] 0.5
a FEV1/ FVC decreases and compliance decreases b. Effusion lactate dehydrogenase [LDH] /serum LDH
b. FEV1/ FVC increases and compliance increases ratio greater than 0.6
c. FEV1/FVC decreases and compliance increases c. Effusion LDH level greater than two-thirds the
d. FEV1/ FVC increases and compliance decreases upper limit of the laboratory’s reference range of

sytem
Ref : Harrison 17th/ 1588. serum LDH
d . Effusion sugar is less than 2 / 3 rd of blood sugar
198. Decreased maximum mid - expiratory flow rate
indicates obstruction in : [A! 1994] & [AIIMS May 95] RefHarrison's 18th edn, ch 263, Harrison 19 th p 1716 - 17
a. Small airways 204. Incorrect regarding chylous pleural effusion?
b. Trachea a. Most common cause is trauma
c. Large airways b. Milky white fluid [ NBE Pattern 2014-15]

Respiratoy
d. Trachea & Bronchi both c. High cholesterol
Ref : Harrison 17th/ 1588. d . Octreotide is used
RefHarrison's 18th ed. ch. 263, Harrison 19 th p 1718
199. All are seen in ARDS, except: [AIIMS May 95]
a. Pulmonary edema 205. Low glucose in pleural effusion is seen in all except:
b. Decreased tidal volume. a. Rheumatoid arthritis [ NBE Pattern 2014 - 15]
c. Hypercapnia b. Empyema
d . Decreased compliance c. Malignant pleural effusion
Ref : Harrison 18th/ p 2205, 2206, 2207; d. Dressler’s syndrome
Harrison 19th p 1736 -38 RefCMDT 2014 p. 314, Harrison 19th p 1716 - 18
200. Acute Lung injury is characterized by all , Except: 206. Pleural fluid low in glucose is seen in All except:
a. Alveolar infiltrates [PCI Dec 04 ] a. CHF [ NBE Pattern 2014 -15]
b. Hypoxemia b. Tuberculosis
c. Pulmonary shunting c. Mesothelioma
d. Pa 02/ F10z < 200 mm of Hg d. Empyema
e. None of the above Ref : Harrison 18th/ 2207. RefCMDT 2014 p. 314, Harrison 19 th pi 725 -26
Pleural Effusion 207 . Transudative type of pleural effusion is a feature of:
a. Variceal sclerotherapy [ NBE Pattern 2014-15]
201 . Causes of pleural effusion are all except: b. Coronary artery bypass
a. Para [Manipal 2014} c. Peritoneal dialysis
b. Pneumocystic carini d. Radiation
c. TB RefCMDT 2014 p. 313, Harrison 19 th p 1716
d. Meig’s syndrome
Ref: George Mathews pg. 131 Harrison 19 th p 308e - 14f

I Ans. 196. b. Respiratory acid.. 197. d. FEV


1/ FVC increases.. 198. a. Small airways
L 200. d. Pa02/ FI 02 < 200.. 201. b. Pneumocystic
carini 202. b. Lymph Node Biopsy
199. c. Hypercapnia
203. d. Effusion sugar is less...
116 204. c. High cholesterol 205. d. Dressler’s syndrome
206. a. CHF 207. c. Peritoneal dialysis
Respiratory System

208. Which of the following conditions may lead to c. Bronchial adenoma


exudative pleural effusions: [ NBE Pattern 2014-15] d. Tuberculosis
a. Cirrhosis RefCMDT 2014 p. 316, Harrison 19th p 1717 -1718
b. Nephrotic syndrome
c. Congestive heart failure 215. All the following are true about bronchopulmonary
d. Bronchogenic carcinoma aspergillosis except: [Al 1995]
a. Central bronchiectasis b. Pleural effusion
RefCMDT 2014 p. 313, Harrison 19th p 1716 c. Asthma d. Eosinophilia
209. Bilateral pleural effusion is seen in:
Ref : Harrison 18th / p 2119, 2120, Harrison 19th p 1346 - 47
a. Nephrotic syndrome [NBE Pattern 2014-15]
b. Constrictive pericarditis
c. Congestive cardiac failure Miscellaneous
d. All of the above
RefCMDT 2014 p. 313, Harrison 19th p 1716 -18 216. Confirmatory test for tuberculosis: [Manipal 2014}
210. Pleural effusion in rheumatoid arthritis is typically a. Sputum for MB b. PCRTest
associated with the following features except: c. X-ray Chest d. Gram Stain
a. Glucose > 620 mg/ dl [NBE Pattern 2014-15 ] Ref: Harrison 19th p 1107

Respiratoy
b. Protein > 3 gm/dl 217. Mechanism of action of carbon- monoxide poisoning:
c .Pleural fluid protein to serum protein ratio of > 0.5 a. CO + Hemoglobin — > Carboxyhemoglobin 1 f ||02
d. Pleural fluid LDH to serum LDH ratio of > 0.6 Hb [ Manipal 2014 }
Ref:CMDT 2014 p. 314, Harrison 19th p 1713 -14 b. CO + Hemoglobin — > J, Cardiac output
.
211 All of the following are causes of hemorrhagic pleural c. CO + Hemoglobin — > i bronco constriction
effusion except: [NBE Pattern 2014-15] d. None of the above
a. Pulmonary embolism Ref: Kumar & Clark pg 981 Harrison 19th p 248
b. Rheumatoid arthritis 218. DOC for thromboembolism: [Manipal 2014}
c. Pancreatitis a. Streptokinase
d. TB
RefCMDT 2014 p. 313, Harrison 19th p 1713 -14, 2138
212. Transudative pleural effusion is present in all except:
a. Meig’s syndrome
b. CCF
[NBE Pattern 2014-15]
b. Low molecular weight heparin
c. Urokinase
d. Warfarin
Ref: Kumar & Clark pg 468 Harrison 19th p 739
219. "Morning dip" in peak expiratory flows is a feature of:
System
c. Nephrotic syndrome a .Nocturnal asthma [ Manipal 2014 }
d. Chronic liver disease b. Emphysema
RefCMDT 2014 p. 313, Harrison 19th p 1716 c. Cardiac asthma
213. A high amylase level in pleural fluid suggests a d. Brochiactasis
diagnosis of: [NBE Pattern 2014-15] Ref: [http:// www.chestjournatore Harrison 19th p 1348
a .Tuberculosis 220. Pulmonary embolism may present as: [ Manipal 2014 }
b. Malignancy a .Blood tinged sputum b. A / C breathlessness
c. Rheumatoid arthritis c .Chest pain d. All the above
d. Pulmonary infarction Ref: George Mathews Ph pg 143 Harrison 19th p 1636
RefCMDT 2014 p. 314, Harrison 19th p 1717 221. Pectus excavatum is: [ Manipal 2014}
214. Causes of haemorrhagic pleural effusion are all a. Depression of the sternum posteriorly
except:
b. Sternum depressed posteriorly and inferiorly
c. Sternum bulges forwards
a. Pulmonary infarction [NBE Pattern 2014-15] d. Sternum laterally depressed
b. Mesothelioma
Ref: Harris on 19th
p 1443

Ans. 208 . d. Bronchogenic... 209. d. All of the above 210. a. Glucose > 620 mg/dl 211 . b. Rheumatoid arthritis
212. a. Meig’s syndrome 213. b. Malignancy 214. c. Bronchial adenoma 215. b. Pleural effusion
216. a . Sputum for MB

Q
217. a. CO + Hemoglobin... 218 . b. Low molecular wei... 219. a . Nocturnal asthma
220. d. All the above 221. a. Depression of the sternum posteriorly
Marwah's Internal Medicine MCQs ( Based on Harrison's 19th )

222 . Decreased 0 Z carrying capacity and Normal P02 is a c. Chronic bronchitis


feature of: [ Manipal 2014} d. Bronchiectasis
a. Anemic hypoxia
b. Histotoxic hypoxia Ref: Harrison's 18th pg 2087 Harrison 19th p
1676
c. Stagnant hypoxia , 230. Residual volume of lung in an average adult male is
jHI d. Hypoxic hypoxia a- 30 L [ Manipal 2014}
Ufl Ref: Harshmohan pg 330 Harrison 19th p 249 b. 0.9 L
c. 1.2 L
IH 223 . 133 . Handerson Hasselbach equation is useful for:
a. Electrolytes d. 1.9 L
[ Manipal 2014}
b. Acid base
c. Fluid
-
Ref: Davidson, 21st pg 651 Harrison 19th p 306 e -l 2, 306 e -2f
231. A young woman presents to the emergency
d. Enzymatic reaction
department with central cyanosis because her friends
Ref: Vasudevan -Biochem pg 320 told her she looks bluish, she is asymptomatic. What
224. Asbestos causes all the following except: could be the cause of bluish discoloration of tongue?
a. Pleural fibroma [ Manipal 2014} a. CO poisoning [JIPMER 2014}
b. Gastric carcinoma b. Severe anemia
c. Mesothelioma c. Drinking water contaminated with nitrates

System d. Bronchogenic carcinoma


Ref PARK 17th pg 577 Harrison 19th p 1689
I 225. Acute severe asthma indicated by all except:
a. Pulsus par doxtis
b. Pulse rate >110 per min .
[ Manipal 2014}
d. Lead poisoning

232 . Conglomerate bodies found in:


a. Sarcoidosis
b. Labor pneumonia
Ref : Harrison 19th p 248
[JIPMER 2014}

Respiratoy
c. A normal or low C 02 tension c. Silicosis
d . Silent chest d. Hypersensitivity pneumonitis
Ref Kumar and Clark pg 884 Harrison 19th p 1679 Ref Harrison 19th p 1689, 1690
{ 226 . The following tests may be useful in the assessment of 233. A young man presents to the emergency department
a patient with sarcoidosis: [ Manipal 2014} with shortness of breath and chest pain. His is 120.
a. Chest X-ray BP is 80 / 50, JVP elevated and trachea shifted to left.
b. ACE His oxygen saturation is 70% in spite of supplemental
c. Conjunctiva biopsy
oxygen. What is the next step of management?
d. Serum calcium
e. All the above Ref Harrison 19th p 2205 [JIPMER 2014}
a. Insert a large bore needle on right side
: 227 . Sarcoidosis:
[ Manipal 2014 } b. Insert a large bore needle on left side
a. Is associated with HLA B1 c. Arrange for an urgent chest x- ray and meanwhile
b. is commoner in elderly people give high doses of inhaled oxygen
c. Causes caseating granuloma d. Emergency tracheostomy to secure airway
d. Produces bilateral hilar lymphadenopathies in the
Ref : Harrison 19th p 96 t, 98
absence of pulmonary symptoms.
Ref Harrison 19th p 2205 234. In an emphysematous patient with bullous lesions,
which is the best investigation to measure lung
228. Chloride receptor defect is responsible for: volume:
a. Cystic fibrosis [ Manipal 2014} a. Body Plethysmography
b. Wilson’s disease b. Gas dilution
c. Alpha -1 antitryron deficiency c. Transdiaphragmatic pressure
d. Heinochromatosis d. DLCO
-
Ref Harrison' s 18th pg 2149 Harrison 19th p 145e 5, 429 F, Ref Harrison's 18/ e p2139, 17/ e pi 587, Harrison 19th p 2081
435, 1698 F
229 . Increased lung compliance is seen in:
a. Emphysema [Manipal 2014}
b. Bronchial asthma

Ans . 222. a. Anemic hypoxia 223. b. Acid base 224. a. Pleural fibroma 225. c. A normal or low C02. ..
226. d . Serum calcium .
227. d. Produces bilateral hilar...228 a. Cystic fibrosis 229. a. Emphysema
230. c. 1.2 L 231. c. Drinking water... 232. c. Silicosis 233. a. Insert a large bore needle
118
234. a . Body Plethysmography
Respiratory System

235. A patient comes with sudden respiratory distress, on b. Pulmonary embolism


examination, bilateral basal crepts are present over c. Massive fluid deficit
chest suggestive of pulmonary edema with normal d. Myocardial depression due to anaesthetic agents
alveolar wedge pressure. The likely cause is: Ref:Harrison's 17thed. p. 1561.
a. Narcotic overdose [AIIMSJune 2000]
242. Most common cause for acute mediastinitis is:
b. Congestive heart failure
a. Esophageal perforation [ NBE Pattern 2014-15 ]
c. Myocardial infarction
b. Cervical spondylitis
d. Cardiogenic shock
c. Osteomyelitis of sternum
Ref: Harrison 19th p 473e-l d. Osteomyelitis of clavicle
236. In primary pulmonary hypertension basic Ref: Harriso n's 18th
ed. ch. 145 p. 945, Harrison 19th p 1719
abnormality in gene lies in: [ AIIMS May 07] 243. The most common posterior mediastinal mass:
a. Bone morphogenic protein receptor II a. Neurogenic tumor [ NBE Pattern 2014-15]
b. Endothelin b. Lymph nodes
c. Homebox gene c. Neurogenic parasitic cyst
d. PAX - 11 d. Teratoma
Ref: Harrison's 18/ e p2077, 17 / e pl 577, Harrison 19th p 1657 RefCMDT 2014 p. 292, Harrison 19th p 1664 -65
237. A 29 year old anxious lady presents with a history of 244. Most common cause of obstructive sleep apnea
progressive breathlessness and exercise intolerance a. Craniofacial abnormalities [NBE Pattern 2014-15]
73
ft
since four months. Her FVC is 90% and FEV1 / FVC is b. Hypothyroidism v)*
86%. Oxygen saturation after exercise was observed c. Alcoholism "2."
to drop from 92 % to 86%. What is the likely diagnosis: d. Acromegaly H
a. Primary alveolar hypoventilation [ AI 2010] RefCMDT 2014 p. 319, Harrison 19th p 1724 -25 w
b. Primary pulmonary hypertension
c .Anxiety disorder
245. First treatment for anaphylactic shock is: o
a. Subcutaneous adrenaline [ NBE Pattern 2014 -15] "I
d. Interstitial lung disease
b. IV Corticosteroid
Ref: Harrison's 18/ e p2077, 17/ e pl 587-1589, c. Theophylline tn
Harrison 19 th p 2161
d. Anti-histaminic vs
238. Conglomerate Nodules in X-ray seen in? [JIPMER 2014]
RefCMDT 2014 p. 488, Harrison 19th p 1749 - 50 o
a. Hypersensitivity Pneumonitis
246. The following are located in anterior mediastinum
b. Sarcoidosis
c.
except: [ NBE Pattern 2014-15] 3
Silicosis a. Thymoma b. Neurofibroma
d. Lobar Pneumonia c. Teratoma d. Pericardial cyst
Ref:Harriso n 18 th
edn, ch 256, Harrison 19 th p 1689 - 90 RefCMDT 2014 ch. 9, p. 291, Harrison 19th p 1664 -65
239. Highest likelihood of 02 delivery to lungs? 247. Asbestosis causes all except? [ NBE Pattern 2014-15]
a. Face mask [JIPMER 2014] a. Shaggy heart borders
b. Nasal cannula b. Honeycombing
c. Non-rebreathing mask c. Hilar lymphadenopathy
d. Oxygen hood .
d Basal peribronchial fibrosis
Refsynopsis of pediatric emergency, 4th edn.
RefCMDT 2014 p. 309, Harrison 19th p 1688 -89
240 . Asbestos causes all except? [NEET Pattern 2015-16 ] 248. The diffusion capacity of lung [ DL] is decreased in all
a. Mesothelioma b.Pleural effusion of the following conditions except:
c. Bronchial cancer d.Atelectasis a. Interstitial lung disease [ NBE Pattern 2014-15]
Ref.-Harrison's 18th edn, ch 256, Harrison 19th p 1689 - 90 b. Goodpasture's syndrome
241. A patient undergoing surgery suddenly develops c. Pneumocystis Jiroveci
hypotension. The monitor shows that the end- tidal
d. Primary pulmonary hypertension
carbon dioxide has decreased abruptly by 15 mm Hg.
What is the probable diagnosis? [NBE Pattern 2014-15] Ref:Harriso n' s 18th
ed. ch. 261 p. 2169, Harrison 19th p 1715, 1710
a. Hypothermia

Ans. 235. a. Narcotic over.. 236. a. Bone morphog... 237. b. Primary pulmonary... 238. c. Silicosis
239. c. Non -rebreathing .. 240 . d. Atelectasis 241. d. Myocardial depress ... 242. a . Esophageal perforation
243. a . Neurogenic tumor 244 . a . Craniofacial abnorm.. 245. a . Subcutaneous adren.. 246. b. Neurofibroma
247 . c. Hilar lymphaden... 248 . b. Goodpasture’s syndrome

1
L Marwah 's Internal Medicine MCQs ( Based

249. Which one of the following is not correct regarding


on Harrison's 19th )

255. All show wide alveolar-arterial 02 gradient


except -
silicosis? [ NBE Pattern 2014-15]

a
a. Bronchiectasis [NBE Pattern 2014 -15]
a. Egg shell calcification is seen on chest X- ray b. ARDS [acute respiratory distress syndrome]
b. It is more marked in the lower zone
c. May lead to progressive massive fibrosis c. Interstitial fibrosis
d . The disease may progress even after the exposure d. Central hypoventilation
has ceased Ref:Harrison's 18th ed. ch. 262 p. 2171.
RefCMDT 2014 p. 309, Harrison 19 th p 1689- 90 256. Which of the following is true regarding rheumatoid
250. Paradoxical breathing is characteristic of: arthritis? [NBE Pattern 2014-15]
a. Pneumonia [ NBE Pattern 2014-15] a. Typically involves small joints asymmetrically
b. Pneumothorax b. Causes pleural effusion with low sugar
c. Atelectasis
d. Flail chest Ref:Harrison's 17th ed. p. 1569. c. Mononeuritis multiplex
d. Enthesopathy prominent
251. Rib notching is found in all the following except:
a. Neurofibromatosis [NBE Pattern 2014-15] Ref: Harrison's 18th ed . ch. 263, Harrison 19th p 2136
b. Coarctation of aorta 257 . A patient with blood chemistry of pH 7.3, C02 of 60
£ c. Taussig Bing operation and HC03 of 28 mEq / dl are indicative of:

->
4
V» d . Hypo-parathyroidism Ref:CMDT-2014 p. 1135.
C/i I 252 . In bronchial asthma following pulmonary function
» abnormalities are present except:
a.
b.
[NBE Pattern 2014 -15]
Partially compensated respiratory acidosis
Uncompensated respiratory acidosis
C/i a. Decreased FEV1
c.Fully compensated respiratory alkalosis
b. Decreased maximum expiratory flow rate
& c. Increased residual volume d.Metabolic alkalosis with respiratory alkalosis

s d. Decreased diffusion capacity


RefCMDT 2014 p. 258, Harrison 19 th p 1679 -80
Ref Harrison 19 th p 322 -23
258. Pulmonary fibrosis is not seen in:
2 253. A factory worker was found unresponsive in his a. Asbestosis [ NBE Pattern 2014-15]

|
a>
workplace. He is afebrile anicteric, tachypneic
drowsy pale discoloration with clear lung field and
b. Rheumatoid arthritis
c. Systemic sclerosis
a£ hyperdynamic cardiovascular findings. His ABG with
100% oxygen after intubation was
d. Shrinking lung syndrome
RefCMDT 2014 p. 308, Harrison 19 th p 621, 1713- 14
pH = 7.30 [ NBE Pattern 2014-15]
p02 = 80 mm Hg 259. A patient presents with breathlessness. He has
pC02 = 30 mm Hg bilateral basal crepitations, lung function tests
Sa 02 = 95% reveal decrease in total lung capacity [TLC] and vital
What is the most likely diagnosis? capacity [VC] with normal FEV1/ VC ratio. The most
a. Adult Respiratory distress syndrome likely diagnosis is:
b. Carbon monoxide poisoning
c. Organo-phosphorus poisoning a. Chronic bronchitis [ NBE Pattern 2014 -15]
d. Cyanide poisoning b. Idiopathic pulmonary fibrosis
Ref:Harrison' s 18th ed. ch. E-52 p. 52-56, Harrison 19th p 248
c. Cystic fibrosis
d . Allergic bronchopulmonary aspergillosis
254. Bronchiectasis sicca is seen with:
a. TB [NBE Pattern 2014-15]
.
RefCMDT 2014 ch 9, Harrison 19th p 1711
b. Pertussis 260. All the following are features of interstitial lung

I c. Cystic fibrosis
d. Kartaneger syndrome
RefCMDT 2014 p. 279, Harrison 19 th p 1694 - 95
disease except:
a. Exertional dyspnoea
b. Cyanosis
c. Digital clubbing
[NBE Pattern 2014 -15]

I 120
Ans. 249. d. The disease...
253. b. Carbon monox..
257. a. Partially com...
250. d. Flail chest
254. a. TB
258. d. Shrinking lung syn...
d. Coarse crepitations
RefiCMDT 2014 p. 292, Harrison 19 th pi 70S

251. d . Hypo - parathyroidism 252. d . Decreased diffusion...


255. d . Central hypoventi... 256. a. Typically involves small..
259. b. Idiopathic pulmonary.. 260. d. Coarse crepitations
Respiratory System

261. Most common symptom in cystic fibrosis:


a. Lung infections
b. Meconium ileus
[NBE Pattern 2014 -15]
268. True about bronchopulmonary aspergillosis are A / E:
a.
Central bronchiectasis
Pleural effusion
b.
[ NBE Pattern 2014 -15] I
c. Malabsorbtion c.
Asthma
d . Infertility
d.
Eosinophilia
RefCMDT 2014 p. 267; nelson, p. 400,
Harrison 19th p 1697 - 98, 1686 RefCMDT 2014 p. 266 , Harrison 19th p 1695
269. Which of the following is not a feature of Kartagener's
.
262 An infant has a positive newborn screening test for syndrome: [ NBE Pattern 2014 -15]
cystic fibrosis. What cut off of sweat chloride confirms
cystic fibrosis? [ NBE Pattern 2014-15]
a . Bronchiectasis b. Pancreatic insufficiency
c. Sinusitis d. Situs inversus
a. Sweat Chloride > 30 b. Sweat Chloride > 40
c. Sweat Chloride > 50 d. Sweat Chloride > 60
[ Ref : Ch, 258, Harrison' s 18th ed.
RefCMDT 2014 p. 267, Harrison 19th p 1698- 99 270. Ultrastructural abnormalities reported in defective,
.
263 Residual volume is best measured by? cilia immotile cilia syndrome are:
a . Body plethysmography [ NBE Pattern 2014 -15] a . Dyne in arm deficiency [ NBE Pattern 2014- 15]
b. Helium dilution method b. Absence of radial spokes
c. Spirometry c. Absence of central microtubule

Respiratoy
d. All of above d. All of the above Ref:Harrison's 17th ed. p. 1542.
Ref Respiratory Medicine 2014, p. 959 - 971, .
271 Tension pneumothorax results in all except:
Harrison 19th p 306 e a. Respiratory alkalosis
.
264 Heerfordt’s syndrome consists of fever, parotid b. Decreased cardiac output
enlargement, facial palsy and: [ NBE Pattern 2014 -15] c. Decreased venous return
a. Arthralgia d. Absent breath sounds
b. Bilateral hilar lymphadenopathy RefCMDT 2014 p. 316, Harrison 19th pi 719, 1750 I
c. Erythema nodosum 272. Which of the following collagen vascular disease is |
d . Anterior uveitis not commonly associated with pulmonary fibrosis:

.
Ref.Harrison’ s: sarcoidosis ch. 329.
=
265 The ABG report: pH 7.45, PC 02 30 mmHg, P 02 =
100 mm Hg. The patient has got partially compensated :
a. Metabolic acidosis
b. Metabolic alkalosis
c. Respiratory acidosis
[AIPG 2009]
=
a. SLE
b. Scleroderma
c. Dermatomyositis
d . Rheumatoid arthritis
[ NBE Pattern 2014 -15]

RefCMDT 2014 p. 831, Harrison 19th p 1467,1911


273. The antibiotic commonly used for chemical
System
d. Respiratory alkalosis pleurodesis is: [ NBE Pattern 2014 -15]
Ref:ARMS pediatric ABG manual, Oxford Handbook of Medicine a. Amoxicillin b. Doxycycline
5th ed. p. 684, Harrison 19th p 322 c. Co-trimoxazole d. Rifabutin
266. Recognized features of asbestosis do not include: RefCh. 263 Harrison's 18th ed.
a. Calcification of pleura [ NBE Pattern 2014 -15]
274. Prolonged hyperventilation may lead to all except:
b. Egg shell calcification of hilar lymph nodes a. Paraesthesia
c. Clubbing of fingers b.
Alkalosis
[ NBE Pattern 2014 15] -
d. Restrictive pattern of ventilatory defect shown by c. Tetany
pulmonary function d . Somnoloscence
RefCMDT 2014 p. 309, Harrison 19 th p 1689
267. What is true regarding byssinosis:
RefCMDT 2014 p. 318, Harrison 19 th p 1722 23
275. Most common lesion in middle mediastinum:
-
a. Dyspnea resolves after cessation of exposure a. Congenital cyst [ NBE Pattern 2014 - 15]
b. Similar to chronic bronchitis and emphysema b. Lipoma
c. Present as mediastinal fibrosis[ NBEPattern 2014-15] c. Aneurysm
d. Eosinophils are prominent in BAL d . Neurogenic tumours
RefCMDT 2014 p. 311, Harrison 19th p 1688, 1691 Ref : Harrison 19th p 1664-65; 1719

Ans . 261. a. Lung infections 262. d . Sweat Chloride > 60 263. a. Body plethy.. 264. d . Anterior uveitis
265. c. Respiratory aci... 266. b. Egg shell calcification.. 267. a. Dyspnea resolves... 268. b. Pleural effusion
269 . b. Pancreatic insuf ... 270. d. All of the above 271. a. Respiratory alkalosis 272. a. SLE
273. b. Doxycycline 274. d. Somnoloscence 275. c. Aneurysm
- Marwah's Internal Medicine MCQs ( Based on Harrison's 19th )

276. In Kartagener's syndrome all are seen except:


a. Cystic fibrosis [ NBE Pattern 2014 -15]
283. All the following lung volumes can be measured by a

a
simple spirometer except: [ NBE Pattern 2014-15 ]
b. Dextrocardia a. Vital capacity
c. Sinusitis b. Residual volume
d. Absence of cilia Ref:Harrison's 17th ed. p. 1542. c. Tidal volume
277. Cystic fibrosis characteristically has following d . Forced vital capacity
features except: [ NBE Pattern 2014-15] Ref:Harrison's 18th ed. ch. 251 p. 2086, Harrison 19th p 306 e
a. Skin frosting 284. The CFTR gene associated with cystic fibrosis is
b. Diabetes mellitus located on chromosome: [NBE Pattern 2014 -15]
c. Chronic constipation a. 5 b. 12
d. Bronchiectasis c. 4 d. 7
RefCMDT 2014 p. 267; Nelson's pediatrics p. 400, RefCMDT-2014 p. 267, Harrison 19th p 1697 98, 435 -
Harrison 19th p 269, 1697 285. True in mucoviscidosis are A / E :[NBE Pattern 2014 - 15]
278. P02 decreases on exercise in all except: a. Meconium ileus b. Steatorrhoea
a - COPD
b. Interstitial Fibrosing alveolitis
[ NBE Pattern 2014-15] c. Sinusitis d. Glomerulonephritis
RefCMDT 2014 p. 267.
c. CCF

System d . Bronchiectasis
RefCMDT 2014 p. 398.
279. All of the following conditions may predispose to
pulmonary embolism except: [ NBE Pattern 2014 -15]
a. Protein S deficiency
286. Kartagener's syndrome is not associated with:
a. Situs inversus
b. Subluxation of lens
c. Bronchiectasis
d. Sinusitis
[ NBE Pattern 2014-15]

RefCh. 258, Harrison's 18th ed.

Respiratoy
b. Malignancy 287. All of the following diseases are associated with
c. Obesity peripheral blood eosinohilia except:
d. Progesterone therapy a. Allergic Bronchopulmonary Aspergillus [ABPA]
RefCMDT 2014 p. 297, Harrison 19th p 1631-32 b. Loeffer's syndrome [ NBE Pattern 2014-15]
c. Pulmonary eosionophilic granuloma
280. A 32 week new born baby presents with RR-86 / min,
d . Churg's strauss syndrome
grunting present with no nasal flaring, abdomen
RefCMDT 2014 p. 296.
behind in movement than chest, minimum intercostal
retraction & no Xiphisternal retraction. What is the 288. Cotton dust is associated with: [ NBE Pattern 2014 -15]
Silverman scoring? [ NBE Pattern 2014-15] a. Byssinosis b. Asbestosis
a. 1 b. 4 c. Bagassosis d. Silicosis
c. 3 d. 6 RefCMDT 2014 p. 311, Harrison 19th p 1688, 1691
Ref: Maternity and Pediatric Nursing By Susan Scott Ricci, Terri 289. The major diagnostic criteria for Allergic
Kyle, p. 729 -30; OP Ghai. 7/ e, p. 143].
Bronchopulmonary Aspergillosis would include all the
281. Which is not a side effect of high dose inhaled following except:
beclomethasone dipropionate: [NBE Pattern 2014 -15] a. Peripheral eosinophilia [NBE Pattern 2014 -15]
a. Dysphonia b. Central bronchiectasis
b Thin skin c. Bronchial asthma
c. Atrophic rhinitis d. Culture of A. Fumigatus from the sputum
d . Pituitary adrenal suppression RefCMDT 2014 p. 266 71:
RefCMDT-2014 ch. 9.
290. The following does not occur with asbestosis:
282. The drug varenicline is used in: [NBE Pattern 2014 -15]
a . Pulmonary Hemosiderosis a. Atelectasis [ NBE Pattern 2014 -15]
b. Sleep apnoea b. Pneumoconiosis
c. Anti -trypsin deficiency c. Pleural mesothelioma
d. Nicotine dependence d. Pleural calcification
Ref Harrison 19th p 507, 1705, 2732 RefCMDT 2014 p. 309, Harrison 19th p 1689 - 90

Ans. 276. a. Cystic fibrosis 277. c. Chronic constipation 278. c. CCF 279. d. Progesterone therapy
280. b. 4 281. c. Atrophic rhinitis 282. d . Nicotine dependence 283. b. Residual volume
284. d. 7 285. d. Glomerulonephritis 286. b. Subluxation of lens 287. c. Pulmonary eosionophilic..
288. a. Byssinosis 289. d. Culture of A. Fumigatus from the sputum 290. a. Atelectasis
Respiratory System

291. Diagnostic features of Allergic Bronchopulmonary shortness of breath for last one month. Her pulmonary
Aspergillosis [ABPA] include of all the following functions tests [ PFT] are as follows: [AIPG 2009]
except: [NBE Pattern 2014-15] PFT Observed Predicted
a. Changing pulmonary infiltrates FVC 2.63 2.82
b. Peripheral eosinophilia
FEV1% 88% 80%
c. Serum precipitins against Aspergillus fumigatus
d. Occurrence in patients with old cavitatory lesions DLCO 5.26 16.3
Ref:CMDT 2014 p. 266, Harrison 19th p 1681, 86
Diagnosis is ?
292. In cystic fibrosis the most frequent pulmonary a. Plum artery Hypertension
pathogen is: [NBE Pattern 2014-15]
b. Interstitial lung disease
a . Pseudomonas b. Enterococci
c. Obstructive lung disease
c. Staphylococci d. Klebsiella
d. Scleroderma crisis
RefFISHMAN pulmonary diseases 2008 edn, ch. 400, p. 866,
Ref : Harrison 19th p 1658
Harrison 19th p 1697
293. SARS causative agent? [ NBE Pattern 2014 -15] 299. A patient presents with decreased vital capacity
a . Corona-virus b. Picorna -virus and total lung volume. What is the most probable
c. Myxovirus d . Paramyxovirus diagnosis? [AI 2007] |Bpj
Ref: Harrison’ s 17th ed. ch. 179, Harrison 19th p 1204 a. Bronchiectasis b. Sarcoidosis
c. Cystic fibrosis d. Asthma Jj£
-
294. Alveolar arterial tension gradient increases in all
Ref : Harrison 19th p 2205-06 .
except: [ NBE Pattern 2014-15]
a. Diffusion defects 300. All are decreased in infiltrative lung disease, except:
b. Hypoventilation a. Vital capacity [AIIMS Feb
c. R- L shunt b. Alveolar arterial difference in Pa02 M
d . Ventilation perfusion abnormality c. Total lung capacity
, Ref:Harrison's 18th ed. ch. 262 p. 2171. d. Lung compliance
Ref.: Harrison 17th/ 1588, 1590, 1591.
295. FEV1/ FVC ratio is decreased in all except: C/i
a. Bronchiectasis [ NBE Pattern 2014 - 15] 301. In an emphysematous patient with bullous lesions,
b. Emphysema which is the best investigation to measure lung o
c. Chronic bronchitis
d. Interstitial lung disease
volume:
a . Body Plethysmography [ AIIMS Nov 08]
;
H
3
b. Gas dilution
RefCMDT 2014 p. 292, Harrison 19th pi 709-11
c. Transdiaphragmatic pressure
296. 30 yrs old worker in chemical plant presents with d. DLco
stupor, cyanosis. Blood drawn for sampling shows Ref : Harrison's 17th/ l 587; Harrison 19 th p 1664-65
brownish red color. Diagnosis: [ NBE Pattern 2014 -15] 302. All are correct regarding sarcoidosis except:
a. Carbon monoxide poisoning a. Often cavitate [AIIMS Dec 98] jf
b. Organophosphate poisoning b. Spontaneous remission is usual
c. Meth -hemoglobinaemia c. Tuberculine test is negative
d. G -6- P-deficiency d . B / L hilar lymphadenopathy.
RefCMDT-2014 P. 500. Ref : Harrison 18th/ 2807, Harrison 19th p 2206 , 09

.
291 The organism most frequently related to mediastinal 303. In primary pulmonary hypertension basic
fibrosis is: abnormality in gene lies in: [AIIMS May 07]
[NBE Pattern 2014-15]
a. Bone morphogenic protein receptor II
a . Actinomyces b. Histoplasma
b. Endothelin
c. Hansen bacillus d . Staphylococcus c. Homebox gene
RefCMDT 2014 ch. 36 p. 1531. d. PAX - 11
298. A 28 years old woman having limited cutaneous Ref : Robbins 7th / 743,7 4 Harrison 18th/ p 2077,
^
Harrison 19th p 1655, 1658
scleroderma for the last 10 years complains of

.
Ans. 291 . d. Occurrence in .. 292. c Staphylococci
.
299. b. Sarcoidosis 300. b. Alveolar arterial diff ...
-
293. a. Corona virus
295. d . Interstitial lung .. 296. c. Meth -hemoglobinaemia 297. b. Histoplasma
301. a. Body Plethysmogra
294. b. Hypoventilation
298. a . Plum artery Hypertension
302. a. Often cavitate
I
303. a . Bone morphogenic protein receptor II

\
I Marwah's Internal Medicine MCQs ( Based on Harrison's 19th)

304. Pulmonary hypertension may occur in all of the a. Sniff test is positive
following conditions except- [AIIMS Nov 06] b. Causes normocapnic failure
a. Toxic oil syndrome
b. Progressive systemic sclerosis
c. Diaphragmatic pacing is useful if any nerve is intact

3
d. None of the above Ref : Harrison 17th / 221, 1663.
c. Sickle cell anemia
d. Argemone mexicana poisoning 310. Most common cause of Mediastinitis is: [AIIMS 97]
Ref.: Harrison 18th / p 2082, Harrison 19th p 1658 a. Tracheal rupture b. Esophageal rupture
c. Drugs d. Idiopathic
305. Chronic Cor pulmonale is seen in all except:
a. Pulmonary embolization [ PGI- Dec- 04]
Ref : Harrison 18th / p 2182, Harrison 19th p 1719
b. COPD 311. All of the following are true about Kartagener 's
c .
Cystic fibrosis syndrome, Except [PGI-Dec-04]
d. Primary pulmonary hypertension a . Dextrocardia b. Infertility
Ref : Harrison 18th / p 1914, Harrison 19th p 1505 -06 c. Mental retardation d. Bronchiectasis
306. The most common cause for chronic cor pulmonale is Ref : Robbins 7th / 727.
a. Recurrent pulmonary embolization [PGlDec 04] 312. Causes of pulmonary renal syndrome: [ PGI ] une 07]
b. COPD a. Leptospirosis b. Hanta virus
c. Cystic fibrosis c. Paraquat poisoning d. All of the above
S
<
d. Bronchial Asthma
Ref : Acute Renal failure in Practice’ lst/ 345; Oxford Textbook of
D e.
-
4 >
V3
Airway foreign body
Ref : Rubin's pathology 5th / 231, Harrison 19th p 1505 - 06
clinical Nephrology 3 rd / 582.

>> 307. All of the following show low glucose in pleural fluid,
313. 3.5 kg term male baby, born of uncomplicated
pregnancy, developed respiratory distress at birth,
EXCEPT- [AIIMS Dec 94]
>s a. Empyema
not responded to surfactant, ECHO finding revealed
nothing abnormal, X- ray showed ground glass
b. Malignant pleural effusion
O . appearance and culture negative. Apgars 4 and 5 at
+» c Rheumatoid arthritis
c« d. Dressler's syndrome 1 and 5 minutes. History of one month female sibling

-a
s Ref : Harrison 18th / p 2178, Harrison 19th p 1716 -19
died before. What is the diagnosis?
.
a TAPVC
[AIIMS Nov 07]

t/5 b. Meconium aspiration


308. A high amylase level in pleural fluid suggests a
a> c. Neonatal pulmonary alveolar proteinosis
diagnosis of [AIIMS May 03]
Qtf d. Diffuse herpes simplex infection
a. Tuberculosis
b. Malignancy Ref : Nelson, Pediatrics 18th ed. Ch 403, Harrison 19th p 1658
c. Rheumatoid arthritis 314. Measurements of intravacular pressure by a
d. Pulmonary infarction pulmonary artery catheter should be done: [A12002 ]
Ref : Harrison 18th / p 2178, Harrison 19th p 1717, 510, 572 a. At end expiration
b. At peak of inspiration
309. All are true about bilateral diaphragmatic paralysis,
except : [AIIMS May 95]
c .During mid inspiration
d. During mid expiration
Ref : Ch - 260, Harrison 18th .
RESPIRATORY SYSTEM UPDATES

Gold Criteria for Severity of Airflow Obstruction in Copd


GOLD stage Severity Spirometry
, ,
I
I Mild FEV / FVC <0.7 and FEV > 80% predicated
II Moderate FEV,/ FVC <0.7 and FEV, > 50% but < 80% predicated
III Severe FEV,/ FVC <0.7 and FEV, > 30% but <50% predicated

I IV Very severe FEV,/ FVC <0.7 and FEV, 30% predicated

Ans. 304. d. Argemone mexi... 305. a. Pulmonary embolization 306. a. Pulmonary em...
308. b. Malignancy 309. b. Causes normocapnic.. 310. b. Esophageal rupture
307. d. Dressler ’s syndrome
311. c. Mental retardation
312. d. All of the above 313. c. Neonatal pulmonary... 314. a. At end expiration
Respiratory System

RESPIRATORY SYSTEM ADDITIONAL UPDATES I


Diagnostic Value Of Bronchoalveolar Lavage In Interstitial Lung Disease
Pulmonary Langerhans cells histiocytosis

Organizing pneumonia

Diffuse alveolar bleeding


Opportunistic infections
Diffuse alveolar damage, drug toxicity
Asbestos-related pulmonary disease
Lipoidosis
Increased CD1+ Langerhans cells, electron microscopy demonstrating
Birbeck granule in macrophage.
Foamy macrophages; mixed pattern of increased cells characteristic;
decreased CD4:CD8 ratio
Hemosiderin-laden macrophages, red blood cells
Pneumocystis carinii, fungi, cytomegalovirus- transformed cells
Atypical hyperplastic type II Pneumocytes
Duct particle, ferruginous bodies
Accumulation of specific lipo-pigment in alveolar macrophages
<
Lymphagitic carcinomatosis, alveolar cells carcinoma
pulmonary lymphoma
Malignant cells po
fD
Eosinophilic lung disease Eosinophilis >25% C/i
Silicosis T3
Dust particles by polarized light microscopy
Lipoid pneumonia Fat globules in macrophages
Sarcoidosis Lymphocytosis, CD4: CD8 ratio > 3.5 most specific of diagnosis
Hypersensitivity pneumonitis Marked lymphocytosis (> 50%)
Condition Bronchoalveolar Lavage Finding
Alveolar proteinosis Milky effluent, foamy macrophages and lipo-proteinaceous intra-alveolar
material (periodic acid-Schiff stain-positive)
Berylliosis Positive lymphocyte transformation test to beryllium

3
Obstructive Sleep Apnea /Hypopnea Syndrome ( OSAHS ): Quantification and Severity Scale

> Apnea -Hypopnea index (AHI):a number of apneas plus hypopneas per hour of sleep
> Mild OSAHS; AHI of 5-14 events /h
> Moderate OSAHS: AHI or 15-29-events /h
> Severe OSAHS : AHI of >30 events/h
> Respiratory disturbance index (RDI): Number of apneas plus hypopneas plus RERAs per hour of sleep
“ Each level of AHI can be further quantified by level of sleepiness and associated hypoxema.

Diagnositc Criteria for ARDS


Severity: Oxygenation Onset Chest Absence of Left
Radiograph Atrial Hypertension
Mild: 200 mmHg Acute Bilateral alveolar or PCWP < 18 mmHg or no
<Pao2 /Fio < 300 mmHg interstitial infiltrates
2 clinical evidence or increased
Moderate: 100 mmHg
left atrial pressure

I
<Pao2 /Fio < 200 mmHg
2
Severe: Pao2/ Fio2
< 100 mmHg
Marwah s Internal Medic ine MCQs ( Base d on Harris on's 19
,h
)
'

RESPIRATORY SYSTEM ADDITIONAL UPDATES

a Respiratory Event Definitions

>

>

>
Respiratory effort - related arousal (RERA): A partially obstructed breath that does not meet the criteria for hypopneua but
provides evidence of increasing ispiratory effort (usually through pleural pressure monitoring ) punctuated by an arousal
Apnea: Cessation of airflow for > 10sec during sleep. Accompanied by


Persistent respiratory effort (obstructive apnea)
Absence of respiratory effort (central apneas)
Flow-limited breath: A partially obstructed breath, typically within a hypopopnea or RERA , identified by a flattened or "scooped-
out” inspiratory flow shape

> Hypopnea: A > 30% reduction in airflow for at least 10 sec during sleep that is accompanied by either a > 3% desaturation or an
arousa

£
O)
Side Effects of Continuous Positive Airway Pressur E (CPAP) and Their Treatments
Side Effect Treatment

£
o
Nasal congestion
Bruised nasal ridge
Provide heated humidification, administer saline/steroid nasal sprays
Change mask interface, provide protective padding
M Difficulty exhaling Temporarily reduce pressure, provide bi-level positive airway pressure
CG
Claustrophobia Change mask interface ( e.g. , to nasal prongs) promote habituation (i.e. practice breathing

a
C/5
on CPAP while awake)

0>
06

Diagnostic Criteria of Acute Eosinophilic Pneumonia Evidence-Based Recommendations for ARDS Therapies
1. Absence of parasitic, fungal, or other infection Treatment ventilation
2. Failure to relapse after discontinuation of corticosteroids > High -frequency ventilation (Proved to be Least Effective )
3. Diffuse pulmonary infiltrates on chest x-ray > Minimized left atrial filling pressure
4. Bronchoalveolar lavage eosinophilia >25% > Recruitment maneuvers
5. Hypoxemic respiratory failure > Low tidal volume ( BEST )
6. Quick clinical response to corticosteroids > High -PEEP or “ open lung"
7. Acute febrile illness with respiratory manifestations of <1
> Prone position
month in duration
Contraindications for Noninvasive Ventilation
8. Absence of drugs known to cause pulmonary eosinophilia
inability to clear secretions

I
Severe encephalopathy
Cardiac or respiratory arrest
Hemodynamic instability
Facial surgery or trauma

\k Severe gastrointestinal bleed


Unstable angina and myocardial infarction
High — risk aspiration and /or inability to protect
Upper airway obstruction
airways

126
CHAPTER Hepatology
5
HEPATOLOGY UPDATES
> Higher rates of response are achieved in persons infected with HCV geno
type 1 when one of two first-generation direct-
acting antiviral agents-telaprevir and boceprevir, when are NS3/4A serine
protease inhibitors approved by the FDA in 2011-
is added to peginterferon plus ribavirin.
> Patients infected with HCV genotype 2 or 3 (without cirrhosis and with low level
s of viremia) may be treated for 24 weeks with
peginterferon plus ribavirin and require a ribavirin total daily dose of only 800 mg for
. the patients who clear the virus within 4 weeks
(rapid virologic response), a total treatment duration of only 16 weeks may
be sufficient, if the baseline HCV RNA level is 400,000
international units.

Most Recent Q's 2014- 15 5. What is the treatment of choice for alcoholic liver
disease with high discriminant score?
1 . In a child surgery was done for EHBO with hepatojeju - ( NBE Pattern 2014)
.
nal anastomosis Post -operative bilirubin level after 2 A. Infliximab B. Etanercept
weeks was 6 mg/ dl from a pre-operative level 12mg
dl. The reason for this could be?
/ C .Prednisolone D. Liver transplantation
(AllMS May 2015)
a. Normal lowering of bilirubin takes time Ref Harrison 19th p 2059
b. Delta bilirubin
c . Anastomotic stricture Hepatitis
d. Mistake in lab technique Ref: Harrison 19th p 280
2 . A person js HBsAg positive, but Anti- HBc Ab is nega ¬
6 . In a 3 year old child, most common cause of hepatitis
B is:
.
tive What should be the next step? (AllMS May 2015)
a. Pin prick
(Rajasthan 2009)
a. Repeat test after 6 months
b. Check HBeAg, if positive start interferon b. Saliva exchange
c .Check HBV DNA load c. Perinatal
d. Reassure patient that he does not have any disease d. Blood transfusion

Ref: Harrison 19th p 2007 Ref: Harrison's 18/ e p956, 2546, Harrison 19th p 2016
3 . Universal precautions should be practiced while 7 . A 50 year old lady presented with history of pain up¬
handling body fluids of any hepatitis B positive case. per abdomen, nausea, and decreased appetite for
But certain body secretions are exempted from this . .
5 days She had undergone cholecystectomy 2 years
Which among the following is least infectious of body .
back Her bilirubin was 10 mg/ dL, SGPT 900 IU L
/
fluids in a Hepatitis B patient ? ( NBE Pattern 2014) SGOT 700 1U/ L and serum alkaline phosphatase was
a. Urine b. Semen .
280 IU/ L What is the most likely diagnosis:
c .
Saliva .
d Vaginal fluid a. Acute pancreatitis (AllMS Nov 05)
Ref Harrison 19th p 2005f b. Acute cholangitis
.
4 A patient presents with complaints of yellowish dis¬
c. Acute viral hepatitis
d. Posterior penetration of peptic ulcer
.
coloration of selers On examination, icterus present.
His total bilirubin level is 3mg/ dL (direct bilirubin is Ref Harrison's 18/ e p2549, Harrison 19 th p 2015
.
0.3 mg/ dL) Which among the following is the most
8. All of the following are correctly matched, except:
likely diagnosis? ( NBE Pattern 2014 )
a. Obstructive jaundice -
a. ' LKM1 Autoimmune Hepatitis ( AllMS Nov 2010)
b. Rotor syndrome b. LKM 2 - Drug Induced Hepatitis
.
c Gilbert's syndrome c. LKM1 - Chronic Hepatitis C
d. Dubin Johnson syndrome d. LKM 2 - Chronic Hepatitis D

Ref Harrison 19 th p 2002 Ref Harrison's 18/ e p2567, Harrison 19th p 2033
1. b . Delta bilirubin
5. C. Prednisolone
2 . c. Check HBV DNA load 3 . a . Urine .
4. c Gilbert ’s syndrome
6 . d. Blood transfusion 7. c. Acute viral hepatitis .
8. d LKM2 - Chronic Hepatitis D
CHAPTER Hepatology
5 —
> Higher rates of response are achieved in persons infected with HCV genotype 1 when one of two first-generation direct-
acting antiviral agents-telaprevir and boceprevir, when are NS3 /4A serine protease inhibitors approved by the FDA in 2011-
is added to peginterferon plus ribavirin.

> Patients infected with HCV genotype 2 or 3 (without cirrhosis and with low levels of viremia) may be treated for 24 weeks with
peginterferon plus ribavirin and require a ribavirin total daily dose of only 800 mg. for the patients who clear the virus within 4 weeks
(rapid virologic response), a total treatment duration of only 16 weeks may be sufficient, if the baseline HCV RNA level is 400,000
international units.

Most Recent Q’s 2014- 15 5. What is the treatment of choice for alcoholic liver
disease with high discriminant score?
1. In a child surgery was done for EHBO with hepatojeju- (NBE Pattern 2014)
nal anastomosis. Post- operative bilirubin level after 2 A. Infliximab B. Etanercept
weeks was 6 mg/ dl from a pre- operative level 12mg/ C. Prednisolone D. Liver transplantation
dl. The reason for this could be? (.ARMS May 2015) Ref Harrison 19th p 2059
a. Normal lowering of bilirubin takes time
b. Delta bilirubin Hepatitis
c. Anastomotic stricture
d. Mistake in lab technique Ref: Harrison 19th p 280 6. In a 3 year old child, most common cause of hepatitis
2. A person is HBsAg positive, but Anti- HBc Ab is nega ¬
B is: (Rajasthan 2009)
tive. What should be the next step? (ARMS May 2015) a . Pin prick
a. Repeat test after 6 months b. Saliva exchange
b. Check HBeAg, if positive start interferon c. Perinatal
c. Check HBV DNA load d. Blood transfusion
d. Reassure patient that he does not have any disease
Ref: Harrison 19th p 2007 Ref Harrison's 18/ e p956, 2546, Harrison 19th p 2016
7. A 50 year old lady presented with history of pain up ¬

3. Universal precautions should be practiced while


per abdomen, nausea, and decreased appetite for
handling body fluids of any hepatitis B positive case.
5 days. She had undergone cholecystectomy 2 years
But certain body secretions are exempted from this.
Which among the following is least infectious of body back. Her bilirubin was 10 mg/ dL, SGPT 900 IU/ L
fluids in a Hepatitis B patient? ( NBE Pattern 2014) SGOT 700 1U /L and serum alkaline phosphatase was
a. Urine b . Semen 280 IU/ L. What is the most likely diagnosis:
c. Saliva d. Vaginal fluid a. Acute pancreatitis (AIIMS Nov 05)
b. Acute cholangitis
Ref Harrison 19th p 2005f
c. Acute viral hepatitis
4. A patient presents with complaints of yellowish dis ¬ d. Posterior penetration of peptic ulcer
coloration of selers. On examination, icterus present.
His total bilirubin level is 3mg/ dL (direct bilirubin is Ref: Harrison's 18/ e p2549, Harrison 19th p 2015
0.3 mg/ dL). Which among the following is the most
8. All of the following are correctly matched, except:
likely diagnosis? (NBE Pattern 2014)
a. a. ' LKM1 - Autoimmune Hepatitis (ARMSNov 2010)
Obstructive jaundice
b. Rotor syndrome b. LKM 2 - Drug Induced Hepatitis
c. Gilbert’s syndrome c. LKM1 - Chronic Hepatitis C
d. Dubin Johnson syndrome d . LKM 2 - Chronic Hepatitis D
Ref Harrison 19th p 2002 Ref: Harrison’s 18 / e p2567, Harrison 19th p 2033
Ans. 1. b. Delta bilirubin 27 c. Check HBV DNA load 3. a . Urine 4. c. Gilbert’s syndrome
5. C. Prednisolone 6. d. Blood transfusion 7. c. Acute viral hepatitis 8. d. LKM2 - Chronic Hepatitis D
Marwah's Internal Medicine MCQs ( Based on Harrison's 19th )

9 The following is a marker of acute hepatitis B infec- 16. Drug of choice for hepatitis B: ( NBE Pattern 2014 -15)
tiom ( AIMS Nov 07) a. Entecavir b. Sofosbuvir
a. DNA polymerase c. Simeprevir d. Tenofovir
b. Hepatitis core antigen Ref Harrison's 18th ch. 306, Harrison 19 th p 2033

3 10 .
c.
d.
Anti HBs
IgG to core antigen
Ref: Harrison's 18/ e p2550, Harrison 19 th p 135 e-2t
Reverse transcriptase of hepatitis B virus is coded on
the following gene:
a. C gene b. S gene
d . Xgene
( AI 2000 )
17. A patient has anti HBs without any other without any
other antigen or antibody against HBV. This indicates:
a . Vaccinated
b. Chronic infection
c. Persistent carrier
d. Acute infection
( NBE Pattern 2014 -15)

Ref: Harrison's 18th ed. ch. 304, Harrison 19 th p 2033


c. Pgene
Ref: Harrison' s 18/ e p2540, Harrison 19 th p 1247 18. HbsAg Carrier state is not associated with:
a. Down's syndrome ( NBE Pattern 2014 -15)
11. Hepatitis C virus is associated with: ( AI 2000 ) b. Chronic renal failure
a . Anti LKM antibody c. Poly-arteritis nodosa
b. Scleroderma d. Infectious mononucleosis
c. Cryoglobulinemia Ref CMDT 2014, ch. 16, pg. 674, Harrison 19 th p 2031
d . Polyarteritis nodosa Most common route of transmission of hepatitis C:
Ref Harrison's 18/ e p2550, Harrison 19 th p 2015 a. I .V. drug abuse ( NBE Pattern 2014-15)

Hepatolgy
12. " Pelioses hepatis ” occurs commonly with use of: b. Sexual contact
c. Factor 8 concentrate
a. Danazol ( MP PG 2008 )
d. Feco-oral route
b. Doxycycline
Ref CMDT 2014, ch. 16, pg. 676, Harrison 19 th p 2041
c. Disulfiram
d. Diazepam 20. Incubation period of hepatitis B is:
Ref: Harrison's 17 / e pi 950, Harrison 19 th p 283 a. 6 weeks to 6 months ( NBE Pattern 2014-15)
b. 6 days to 6 weeks
13. All the following are used for treatment of chronic c. 6 months to 6 years
Hepatitis B except? ( AIIMS Nov. 14 ) d . More than 6 years
a. Entecavir b. Telbivudine Ref: CMDT 2014, ch. 16, pg. 667, Harrison 19 th p 2031
c. Zidovudine d . Lamivudine
21. Maddrey discriminant score is used for determining
14. Not seen in association with Hepatitis C virus: mortality due to: ( NBE Pattern 2014-15)
a. Lichen planus ( NBE Pattern 2014 -15) a. Alcoholic hepatitis
b. Cryoglobulinemia b. Viral hepatitis
c. Porphyria cutanea tarda c. Cryptogenic hepatitis
d. PAN d. Hepatic encephalopathy
Ref CM DT 2014 2014, ch. 16, pg. 681.
Ref CMDT 2014, ch. 16, pg. 670, Harrison 19 th p 2044
22. Co-infection is essential for disease presentation in:
15. A 30- year-old man presented with nausea, fever and a. Hepatitis A ( NBE Pattern 2014-15)
.
jaundice of 5 days duration The biochemical tests b. Hepatitis B
revealed a bilirubin of 6.7 mg / dl, unconjugated 5.0 c. Non-A Non - B hepatitis
mg / dl with SGOT /SGPT (AST /ALT) of 1230 /900 IU / d. Delta hepatitis
.
ml The serological tests showed presence of HbsAg Ref CMDT 2014, ch. 16, pg. 672, Harrison 19 th p 2031
IgM anti HBc and HbeAg. The most likely diagnosis is:
( NBE Pattern 2014-15) 23. Best site of giving hepatitis B vaccine is:
a. Chronic hepatitis B infection with high infectivity a. Subcutaneous ( NBE Pattern 2014 -15)
b. Acute hepatitis B infection with high infectivity b. Intradermal
c. Chronic hepatitis b infection with low infectivity c. Intra- muscular deltoid
d . Intra- muscular gluteal
d. Acute hepatitis B infection with low infectivity
Ref Harrison's 18th ch. 304, Harrison 19 th p 2031
Ref: CMDT 2014, ch. 16, pg. 667, Harrison 19 th p 2081

Ans. 9. a. DNA polymerase 10. c. P gene 11. a. Anti LKM antibody 12. a. Danazol
13. c. Zidovudine 14. d. PAN 15. b. Acute hepatitis B... 16. a. Entecavir
17. a. Vaccinated 18. d . Infectious mon... 19. a. I.V. drug abuse 20. a. 6 weeks to 6 months
128
21. a. Alcoholic hepatitis 22. d. Delta hepatitis 23. -
c. Intra muscular deltoid
Hepatology

24. Following liver transplantation , recurrence of prima ¬ 31. Which statement is wrong regarding Hepatitis B?
ry disease in the liver most likely occurs in: a. It is due to RNA virus ( NBE Pattern 2014 - 15)
a . Wilson's disease ( NBE Pattern 2014 15)- b. Blood is the main source of infection
b. Autoimmune hepatitis c. Chronicity is present
c. Alpha - l -antitrypsin deficiency d. It may turn into hepatocellular carcinoma
d. Primary biliary cirrhosis
Ref: CMDT 2014, ch. 16 , pg. 667
Ref: H 18th ed. ch. 306 and 310., Harrison 19th p 207
25 . Which one of the following is not characteristic of 32. Most common subtype of hepatitis B in India:
Hepatitis B? - ( NBE Pattern 2014 -15) ( NBE Pattern 2014-15 )
a. Incubation period is 2-6 months a. Adr b. Adw
b. Mode of infection is parenteral c. Ayr d. Ayw
c. High fever is rare Ref: Harrison 19th p 2033
d. Anicteric hepatitis is not seen
Ref: CMDT 2014, ch. 16, pg. 667, Harrison 19th 2035 33. Not transmitted by blood transfusion:
a. Hepatitis A ( NBE Pattern 2014 -15)
26. Patient comes for blood donation but he has Hbs Ag
and HbeAg positive, and serum transminases level is b. Hepatitis B
normal. What would be the next line of management: c. Hepatitis C
a . Treat with interferon ( NBE Pattern 2014 -15) d. Hepatitis E
b. HBV DNA estimation Ref: Harrison 19th p 2030
c. Liver biopsy

Hepatolgy
d. Observation 34. Fulminant hepatitis is commonest seen with?
Ref: Harrison's 18th ed. Table 306 -4, Harrison 19th p 2035 ( NBE Pattern 2014-15)
a. HAV b. HBV
27 . -
A 55 year -old male patient was diagnosed to have c. HCV d. HEV
chronic hepatitis C. He responded to treatment with
interferon. However, after one year of follow up he Ref : Harrison's 18th ed. ch. 304, Harrison 19th p 2030
showed a relapse of disease. Which of the following 35. Which is not transmitted via breast milk?
would be the next most appropriate choice? a. Hepatitis A ( NBE Pattern 2014 -15)
( NBE Pattern 2014 15) - b. Hepatitis B
a. Ribavirin and pegylated interferon
b. Lamivudine and interferon c. Hepatitis C
c. Nevirapine and lamivudine d. Hepatitis D Ref: Harrison 19th p 2030
d. Indinavir and ribavirin 36. Mallory hyaline changes seen in A / E:
.
Ref: Harrison's 18th ed. ch 306, Harrison 19th p 2059 a. Wilson's disease ( NBE Pattern 2014 -15)
28. A HBs Ag carrier mother with anti Hbe Antibody b. Indian childhood cirrhosis
positive in blood. The chances of hepatitis in newborn c. Primary biliary cirrhosis
is: ( NBE Pattern 2014 - 15) d. Hepatitis E
a. 100% b. 90%
Ref: CM DT 2014, 2014, ch. 16, pg. 681
c. 20% d . 0%
Ref: H 18th ch. 304, Harrison 19th p, Harrison 19th p 2035 37. Insulin resistance in liver disease is due to:
a. Steatosis ( AIMS NOV 2012)
29 . Most common genotype of hepatitis B in cirrhosis in
b. Hepatocyte damage
North India: ( NBE Pattern 2014-15)
a. Genotype A b. Genotype B c. Decreased release of insulin
c. Genotype C d. Genotype D d. Decreased release of C - peptide
Ref: Harrison 19th p 2033 Ref: Comprehensive clinical hepatology, volume 1 , pg. 202 .
30. Which one of the following markers in the blood is the 38 . The marker of hepatitis B in the window period is:
most reliable indicator of recent hepatitis B infection? ( NBE Pattern 2014-15)
a. HbsAg ( NBE Pattern 2014 - 15) a. HBsAg b. Anti-HBs Ag
b. IgG anti- HBs
c. IgM Anti- HBcAg d. IgG Anti- HbcAg
c. IgM anti - HBc
d. IgM anti - Hbe Ref : Harrison 19th p 2033 Ref: H 18th Table 304-5 See question 27, Harrison 19th p 2033

Ans. 24.
28.
b. Autoimmune hep...
c. 20%
25. d. Anicteric hepatitis...
29. d. Genotype D
26.
30.
d. Observation
c. IgM anti -HBc
27. a. Ribavirin and pegylated ...
31. a. It is due to RNA virus
I

*
32. d . Ayw 33. a & d. Hepatitis A... 34. b. HBV 35. c. Hepatitis C
36. d . Hepatitis E 37. b. Hepatocyte damage 38. -
c. IgM Anti HBcAg
th
Marwah 's Internal Medicine MCQs ( Based on Harrison 's 19 )

39. Which is best in evaluating alcoholic hepatitis? 46. A 45 day old infant developed icterus and two days
a. Carbohydrate deficient transferrin later symptoms and signs of acute liver failure ap ¬
b. 5- nucleotidase ( NBE Pattern 2014- 15) peared. Child was found to be positive for HbsAg. The
c. SGPT raised mother was also HbsAg carrier. The mother’s hepati ¬
d . MCHC Ref: CMDT 2014, ch. 16, pg 680 . tis B serological profile is likely to be:
a. HbsAg positive only ( NBE Pattern 2014-15 )
Ul 40. Earliest manifestation of hepatitis b infection:
b. HbsAg and HbeAg positivity
M [ NBE Pattern 2014-15) c. HbsAg and anti Hbe antibody positivity
a . HbeAg b HbsAb IgM d. Mother infected with mutant HBV
c. Anti -Hbc IgM d. Anti - HbeAb IgM
Ref: Harrison's 18a ed. ch. 304, Harrison 19th p 2006
Ref: ch. 304: Harrison's 18th ed., Harrison 19th p 2033
47. Vertical transmission of hepatitis C is:
41. Which one of the following is transmitted non- paren - ( NBE Pattern 2014 -15)
terally: ( NBE Pattern 2014-15) a. 5% b. 10%
a . HBV b. HCV c. 25% d . 50%
c. HDV d . HEV Ref: CMDT 2014 2014, ch. 16, pg. 806, Harrison 19th p 2009
Ref: CMDT 2014, ch. 16, pg. 672, Harrison 19th p 2030
48. The likelihood of becoming an HbsAg carrier after
42. A 30-year-old patient presents with H / O antibodies acute HBV infection is high in: ( NBE Pattern 2014-15)
to HCV for 6 months duration and his AST / ALT is nor ¬ a. Neonates
mal. There is no symptom or stigmata of liver disease. b. Chronic hemodialysis patients
The most appropriate approach: c. Persons with HIV infection

Hepatolgy
( NBE Pattern 2014-15) d . Persons with Down's syndrome
a . Re-assure the patient e. All of the above
b. Repeat titre every 3 years Ref: Harrison's 18th ed. ch. 304, Harrison 19th p 2006
c. Repeat enzymes yearly
49. HbsAg positive mother gives birth to live baby, plan
d . Do liver biopsy and start antiviral drugs accord ¬

of action for prevention of hepatitis B transmission


ingly. baby would consist of: ( NBE Pattern 2014 -15)
Ref: Harrison’s 18* ed. ch. 306, Harrison 19th p 2030 a . Give hepatitis B vaccine to baby
43. The commonest hepatotropic virus causing increased b. Give hepatitis B vaccine and immunoglobulin to
chronic carrier state is: ( NBE Pattern 2014-15) baby
a . HEV b. HAV c. No intervention require if baby is asymptomatic
c. HBV d . HCV d. Prevent breastfeeding
Ref: CMDT 2014, ch. 16, pg. 667 . .
Ref: Harrison's 18th ed ch 304.
50. True regarding, hepatitis B is? ( NBE Pattern 2014-15)
44. A patient presenting with jaundice is found to have 20
a. Circular single stranded
mg % of bilirubin, of which 9.3 is direct reacting & 85 b. Circular double stranded
U / L of alkaline phosphatase, the likely diagnosis is: c. Circular Partial single stranded and partial double
a . Post hepatic obstructive lesion stranded
b. Haemolytic jaundice ( NBE Pattern 2014-15) d . Linear double stranded plus
c. Infective hepatitis Ref: Table 304.1 Harrison's 18‘" ed., Harrison 19th p 2206, 2053
d. Pre hepatic obstructive jaundice
Ref: www.ncbi.nim.nih.gov, 51. Acute viral hepatitis is diagnosed by:
a. HbsAg ( NBE Pattern 2014 -15)
45. - -
A 35 year old male patient presented with history of b. HbeAg
jaundice for 15 days. The onset was preceded by a pro ¬ c. HbsAg + IG M anti- HBc antibody
dromal illness. His serum tested positive for HbsAg. A d . HbsAg + HbeAg
clinical diagnosis of acute hepatitis B was made. What Ref: Harrison's 18* ed. Table 304-5.
should be the next best confirmatory investigation?
a . Anti - HbeAg antibody ( NBE Pattern 2014-15) 52. Indicators of active multiplication of hepatitis B virus
b. Hbe antigen is: ( NBE Pattern 2014-15)
c. Anti-Hbe IgM antibody a. HbsAg b. HbeAg
d. Anti - HbsAg antibody c. HbeAg d. Anti Hbs
Ref: Harrison's 18“ ed. ch. 304, Harrison 19th p 2006 .
Ref: Harrison's 18th ed. ch 304, Harrison 19th p 2006

L 130
Ans. 39.
43.
47.
a. Carbohydrate defi... 40. c. Anti Hbc lg
c. HBV
a. 5%
44.
-
c. Infective hepatitis
48. e. All of the above
41. d. HEV
45. b. Hbe antigen
49. b. Give hepatitis B...
42. d . Do liver ...
46. b. HbsAg and HbeAg ...
..
50. c. Circular Partial .
51. c. HbsAg + IG M... 52. c. HbeAg
Hepatology

53. The most common type of hepatitis


( NBE
associa
Pattern
ted with
2014-15)
61. Occult hepatitis B is:
a.
(NBE Pattern 2014-15 )
HBV DNA <104 copies /ml with HBsAg negative
1
blood transfusion :
b. HBV DNA <104 copies /ml with HBsAg positive

i
a. Hepatitis C
c. HBV DNA <104 copies / ml with HBeAg negative
b. Hepatitis B
d. HBV DNA <104 copies /ml with HBeAg positive
c. Hepatitis A
d. Hepatiti s D Ref: Harrison 19th p 2006 Ref Harrison' s 18 .
th ed ch. 304, Harrison 19th p 2035

virus 62. HCV is associated with: (NBE Pattern 2014-15)


.
54 All of the following are correct about Hepatiti s A
a. Autoimmune cirrhosis
except: (NBE Pattern 2014-15)
a. Faeco-oral transmission b. LKM antibody
b. Short incubation period c. Antimitochondrial antibody
c. Single stranded RNA d. None
th ed ch. 306, Harrison 19th p 2044
d. 3 to 4% carrier state Ref Harrison' s 18 .
Ref: CMDT 2014, ch. 16, pg. 665, Harrison 19th p 2004
63. Extrahepatic manifestations of HCV are A /E:
a. Lichenoid eruptions (NBE Pattern 2014-15) |
55. Antigen which does not appear in blood in hepatitis
B: (NBE Pattern 2014-15) b. Celiac disease
a. HbcAg b. HbeAg c. Glomerulonephritis
c. HbsAg d. None of the above d. Cryoglobulinemia
e. Arthritis
Ref Harrison's Iff ed. ch. 303, Harrison 19th p 2033
* ed. ch. 304-306, Harrison 19th p 2044

Hepatolgy
' th
Ref: Harrison s 18
56 . The most common presentation of hepatitis A is:
a. Asymptomatic (NBE Pattern 2014-15) Early diagnosis of acute hepatitis - B infection is made
b. Fulminant hepatitis by: (NBE Pattern 2014-15)
c. Chronic carrier state a. Presence of HbeAg in serum
d. Transient illness with jaundice b. Presence of IgM anti HBc in serum
Ref CMDT 2014, ch. 16, pg. 665. c. Presence of HbsAg in serum
d. Presence of IgG anti HBc in serum
57 . In a patient with fulminant liver failure after viral
Ref: Harrison ' s 18 th ed. ch. 304, Harrison 19th p 2060
hepatitis, which of the following will be unchanged ?
a. Serum albumin (NBE Pattern 2014-15 ) 65 . A young pt presents with jaundice. Total bilirubin is
b. PT 21, direct is 9.6, alkaline phosphatase 84 KA units .
c. Serum bilirubin Diagnosis is: (AI 2001 )
d. Serum ammonia a. Hemolytic jaundice
Ref CMDT 2014, 2014, ch. 16, pg 674, .
Harrison 19th p 2054 b. Viral hepatitis
c. Chronic active hepatitis
58. Most common type of hepatitis in the world:
d. Obstructive jaundice
(NBE Pattern 2014-15)
a. Hepatitis A .
b Hepatitis B Ref : Harrisons 18th / p 325, Harrison 19th p 283
c. Hepatitis C d . Hepatitis D 66. The commonest hepatotropic virus progressing to
Ref Harrison 19th p 2023, 2008 chronicity is : ( AllMS May 01 )
a. HEV
59 . Severity in acute hepatitis is best estimated by:
b. HAV
a . Serum bilirubin (NBE Pattern 2014-15)
c . HBV
b. Prothrombin time
d. HCV
c. a -glutaryl transferase
d .Alkaline phosphatase Ref : Harrison 18th / p 2553, 2546, Harrison 19th p 2041
Ref CMDT 2014, ch. 16, pg. 670, Harrison 19th p 366e-lf 67. Non-parenteral hepatitis is : (AI 2000)
60.
a. Hepatitis E
Dane particle pertains to: (NBE Pattern 2014-15)
a . HAV b. HBV b .
Hep B
c. NANB d. None of the above c .
Hep C
d. Hep D
Ref .
CMDT 2014, ch. 16, pg 675, Harrison 19th p 2035
Ref : Harrison 18th / p 2546, Harrison 19th p 2041
Ans. 53. b. Hepatitis B 54. d. 3 to 4% carrier state 55. a. HbeAg .
56. d. Transient illness with ja ..
57. a. Serum albumin 58. a. Hepatitis A 59. b. Prothrombin time 60. b. HBV
61.
65.
a. HBV DNA <104...
d. Obstructive...
62.
66.
b. LKM antibody
d. HCV
63.
67.
b. Celiac disease
a. Hepatitis E
64. b. Presence of IgM anti...

B
Marwah 's Internal Medicine MCQs ( Based on Harrison 's 19th
)

68. Early diagnosis of acute hepatitis- B infection is made 73. All of the following are seen in active chronic
hepatitis
by [AllMS Nov 03) BexcePt: ( AIIMS Nov 07 )
a. Presence of Hbe Ag in serum a. IgM against core antigen
b. Presence of IgM anti- HBc in serum b. Total core antibody
c. Presence of Hbs Ag in serum c. HbeAg
5 d. Presence of IgG anti - HBc in serum
Ref.:Harrison 18th/ p 2539, 2, Harrison 19th p 215e-4t
d. HbsAg
Ref.: Harrison’s 18th/ 1832, Harrison 19th p 2006
69 .Which of the following markers in the blood is the 74. A patient is found to be positive only for Anti HBsAg.
most reliable indicator of recent hepatitis B- infec¬ All other viral markers are negative.The likely diag-
tion? (AIMS May 03) nosisis : ( PCI 2009 )
a . HBsAg b. IgG anti - HBs a. Vaccination
c. IgM anti - HBc d. IgM anti - Hbe b. Chronic hepatitis B
Ref : H 18th/ 2538, 2539, 2540, 2541, Harrison 19th p 215e-4t c. Acute hepatitis B
d. Fulminant hepatitis B
70 . A 35-year -old male patient presented with history
Ref : Harrison 18th/ p 2550, Harrison 19th p 2006
of jaundice for 15 days. The onset was preceded by
a prodromal illness. His serum tested positive for 75. All of the following should be included during pre ¬
HBsAg. A clinical diagnosis of acute Hepatitis B was liminary evaluation of a case of suspected Acute viral
made. What should be the next best confirmatory in ¬ hepatitis except: [AIIMSJune 2000 )
a. Hbs Ag b. IgM anti HBc

Hepatolgy
vestigation ( AllMS May 03 )
a . Anti - HBeAg antibody c. Anti- HCV d. IgM anti HBe.
b. HBe antigen Ref : H 18th/ 2539, 2540, 2550, 2551, Harrison 19th p 2008
c. Anti - HBe IgM antibody 76. Hepatitis B infectivity is indicated by: [AI 1994 )
d . HBV DNA by PCR a . Anti - HBsAg
Ref : Harrison 18th/ p 2550, 2551, Harrison 19th p 215e-4 t b. HBsAg + HBeAg
c. Anti - HBsAg - Anti - HBc
71. -
A thirty year man presented with nausea, fever and d. Anti- HBeAg + Anti - Hbs Ag
jaundice of 5 days duration. The biochemical tests
Ref : Harrison 18th/ p 2550, Harrison 19th p 2006
revealed a bilirubin of 6.7 mg / dl (conjugated 5.0 mg /
dl) with SCOT / SGPT (AST / ALT) of 1230 /900 IU / ml. 77. Hera lal a 35 year old man was found + ve for HBsAg &
The serological tests showed presence of HBs Ag. IgM HBeAg, accidentally during screening of blood dona ¬
anti - HBc and Hbe Ag. The most likely diagnosis is: tion. On Lab examination SGOT & SGPT are Normal.
( AIMS Nov 02) What should you do next : (AI 2002 )
a . Chronic hepatitis B infection with high infectivity. a. Liver biopsy b. Interferon therapy
c. Observation d. HBV- DNA estimation
b. Acute hepatitis B infection with high infectivity.
c. Chronic hepatitis infection with low infectivity. Ref :H 18th/ 2540, 2541, 2550, 2551. Harrison 19th p 2007
d. Acute hepatitis B infection with low infectivity. 78. A blood donor is not considered for safe transfusion,
Ref.:H 18th/ p 2540, 2550, 2551, Harrison 19th p 215e-4 t if he has : (AI 2000 )
a. Anti HBs Ag + ve
72. A patient is found to be positive for HBs Ag on routine b. Anti HBs Ag and HBc Ag + ve
laboratory evaluation. Other serological tests for hep ¬
c. Hbs Ag + ve, & IgM anti HBc + ve
atitis are unremarkable. He is clinically asymptom ¬ d . Anti HBe + ve
atic and liver enzymes are within the normal range.
Ref : Harrison 18th/ p 2540, 2541, 2550, Harrison 19th p 2007
Which of the following best describes his diagnosis:
a. Inactive HBV carrier ( AI 2010) 79. Reserve transcriptase of hepatitis B virus is coded on
b. Acute Hepatitis B the following gene : (AI 2000 )
c. Chronic Hepatitis B a. C gene b. S gene
d . Active HBV carrier c. P gene d . X gene
Ref : Harrison 18th/ p 2550, 2551, Harrison 19th p 2038 f Ref.:H 18th/ p 2538, 2539, 2540, Harrison 19th p 2006

Ans. .
68. b. Presence of IgM .. 69. c. IgM anti - HBc 70. None 71. b. Acute hepatitis B...
72. a. Inactive HBV... 73. a. IgM against core antigen 74. a. Vaccination 75. d. IgM anti HBe.
132
76. b. HBsAg + HBeAg -
77. d. HBV DNA estimation 78. c. Hbs Ag + ve , & IgM... 79. c. P gene
Hepatology

80. Which one of the following pairs regarding Hepatitis 88. Features of Alcoholic hepatitis include all of the fol ¬

sI
B is not correctly matched ( ARMS Nov 2010 ) lowing except: ( AI 1991 )
a. Acute Viral Hepatitis B - Supportive care a. Elevated bilirubin
b. Acute Viral Hepatitis B - Antiviral therapy b. Prolonged prothrombin time
c. Chronic Viral Hepatitis B - Supportive care c. Elevated serum albumin
d . Chronic Viral Hepatitis B - Antiviral therapy d. Anemia Ref.: Harrison 18 th / p 2590, 2530.
Ref.:Harrison 18th/ p 2554, Harrison 19 th p 2006 ( AI 1MS May 07 )
89. Ratio of AST / ALT > 1 is present in
81. Interferon treatment is recommended in chronic a. Non alcoholic steatohepatitis
hepatitis B in patients with: ( PGIJune 07 ) b. Alcoholic hepatitis
a. T HBV DNA and Normal ALT c. Wilson's disease
b. T HBV DNA and T ALT
d. All of the above
c. T HBV DNA and compensated cirrhosis
d . T HBV DNA and decompensated cirrhosis 90. Which is not true about alcoholic hepatitis :
Ref.: Harrison 18th/ p 258, Harrison 19th p 2007 ( AIIMS May 95)
a . Gamma glutamyl transferase is raised
82. Chronic liver disease is most commonly caused by :
(A12000 ) b. SGPT is raised > SGOT
a . Hepatitis B b. Hepatitis A c. SGOT is raised > SGPT
c. Hepatitis C d . Hepatitis E d . Alkaline phosphatase is raised
Ref : Harrison 18th/ p 2546, Harrison 19th p 2084 Ref : Harrison 18th/ p 2529 .

Hepatolgy
83 . Hepatic C is associated with all except: ( PGIJune 08 ) Liver Failure
a. PAN
b. Dermatomyositis like syndrome 91. Which of the following statement about ascites is true?
c. Lichen Planus ( PGI June 05)
d . Psoriasis Ref : Harrison 19 th p 2006 t a . Hemorrhagic ascites is diagnosed when RBC count
84. Sustained Response to antiviral therapy ( 1 FN + Riba - > l,000 / mm 3
varin) in hepatitis C is indicated by ( PGI Dec 04 ) b. SBP is diagnosed when neutrophil count > 500 / mm 3
a. High HCV- RNA b. Cirrhosis c. Large volume paracentesis is indicated in SBP
c. Age > 40 years d . Genotype I d . USG can detect as little as 100 mL of peritoneal
e. Female sex fluid
Ref : H 18th/ p 258, Harrison 19 th p 2006 t e. Norfloxacin is the drug of choice in SBP
85 . During an epidemic of hepatitis E, fatality is maximum Ref: Harrison’ s 18/ e p2601, Harrison 19 th p 288
in (AI 2000 ) 92 . In patients with acute liver failure, the best prognostic
a. Pregnant women b. Infants indicator is: ( AIIMS May 01 )
c. Malnourished male d . Adolescents a. Serum albumin
Ref : H 18th/ p 2552, 2553, Harrison 19th p 1991 b. Serum alpha -fetoprotein
c. Serum bilirubin
86 . Characteristic Auto antibodies of Autoimmune Hepa ¬
d. Factor V estimation
titis include all of the following, Except:
a. Antinuclear Antibodies [ANA) ( PGI Dec 06 ) Ref: CMDT 2003/ 637; 09/ 592, Harrison 19 th p 739
b. Anti SLA 93. Increased LDH is an important marker for:
c. AntiLKMl ( DNB 2011 )
d . ANCA a. Bulky disease b. Lymphoma
RefcHarrison 18th/ 2586, Harrison 19 th p 2050 c. Liver metastasis d. Lung metastasis
87 . Granulomatous hepatitis may be seen with : Ref: Harrison 19 th p 433e - 2t
a. Carbamazepine ( AI 1996 ) 94. Hepatomegaly is a feature of all of the following, ex¬
b. Allopurinol cept: ( AI 2009 )
c. Phenylbutazone a. Von Girke's Disease b. Hurler's Disease
d . All of the above c. Nieman Pick Disease d. Hepatic porphyrias'
Ref : Harrison 17th 1739, 1759, Harrison 19 th p 1857
-
Ref: Harrison' s 18/ e p3172 3179, Harrison 19 th p 1992

Ans . 80 . b . Acute Viral Hep.. . 81 . b.|HBV DNA and j ALT 82. c . Hepatitis C 83 . b. Dermatomy. . .
84. e . Female sex 85. a . Pregnant women 86 . d . ANCA 87 . d . All of the above
88 . c . Elevated serum . .. 89 . b . Alcoholic hepatitis
... 90. b. SGPT is raised > SGOT 91 . d . USG can detect as little . ..
92. d . Factor V estimation 93. a . Bulky disease 133
94. d . Hepatic porphyrias ’
i

Marwah 's Internal Medicine MCQs ( Based on Harrison 's 19th )

95. Milan criteria is used for: ( NBE Pattern 2014 -15) 103. Incorrect about liver transplantation:
a. Liver transplantation ( NBE Pattern 2014- 15)
b. GERD staging a. University of winconsin solution
c. Cirrhosis staging b. Done if MELD score > 14
d . Hepatic encephalopathy staging c. HLA matching not required

^
l
II 96 .
J
a. Normal GFR
b.
Ref: CMDT 2014, ch. 16, pg. 691, 1605
Not a feature of hepatorenal syndrome:

Normal urinary sediments


( NBE Pattern 2014 -15)
d . Recurrence with Wilson disease
Ref: CMDT 2014, ch. 16, pg. 691, Harrison 19th p 2042
104. In modified Pugh's classification score of 8, what to do?
a. Conservative management ( NBE Pattern 2014 15)
b. Orthotopic liver transplant
-
c. Low Na + in urine
d. Normal renal biopsy c. Sclerotherapy
Ref: Harrison's IS"’ ch. 308, Harrison 19 th p 1809 d . Shunt surgery
Ref: Harrison's 18thed . ch. 301, Harrison 19 thpi 994
97 . M . E. L. D score includes all except:
( NBE Pattern 2014 - 15) 105 . On a epidemic of hepatitis; fulminant hepatic failure
a. Bilirubin b. INR is seen in: -
( NBE Pattern 2014 15)
c. Serum creatinine d. Serum albumin a. Malnourished child
Ref CMDT 2014 2014, ch. 16 , pg. 692, Harrison 19 th p 1995 b. Pregnant female
c. Old age
98. CAGE scale is used in: ( NBE Pattern 2014-15) d. Child < 15 year of age.

Hepatolgy
a. Alcohol Abuse b . Depression Ref: Harrison's 18th ed . ch. 304, Harrison 19th p 2018
c. Suicidal intention d. Coma
.
Ref CMDT 2014, ch 25, pg. 1080, Harrison 19th p 1992 106 . Hand signs of liver cell failure are all except?

99. All of the following are indications for liver transplan ¬


a. Palmar erythema -
( NBE Pattern 2014 15 )
b. Clubbing
tation except: ( NBE Pattern 2014 -15) c. Duputyren contracture
a. Hepatocellular carcinoma d. Splinter hemorrhages
b. Criggler najar syndrome Ref CMDT, ch. 16, pg. 689, Harrison 19 th p 739
c. Gilbert disease
107 . First line of treatment in Ascitis is: ( PGI June 96 )
d . Biliary atresia
a. Salt Restriction
Ref Harrison's 18 ed. ch. 310, Harrison 19th p 2072
“ b. Diuretics
100. Most likely precipitating cause for acute hepato -cellu- c. Paracentesis
lar failure is: ( NBE Pattern 2014 -15) d. Shunt Ref : Harrison 18th / p 2600 .
a . Oral lactulose
108. In child with acute liver failure, the most important
b. Large IV albumin infusion prognostic factor for death is - ( AllMS 06 )
c. Large carbohydrate meal a. Increasing transaminase
d . Upper G 1 bleeding b. Increasing bilirubin
Ref Harrison' s 18th ed. ch. 308, Harrison 19th p 739 c. Increasing prothrombin time
g are true of Reye 's syndrome d. Increasing serum ammonia
101. All of the followin
except: ( NBE Pattern 2014-15) Ref. Harrison 18th / p 2530.
a. It frequently complicates viral infections 109. Which of the following statement is incorrect with
b. Prothrombin time is prolonged regard to Hepatorenal syndrome in a patient with
c. Disease may by precipitated by salicylates cirrhosis ( A12003)
d. Deep jaundice is present
a. Createnine clearance < 40 ml / min
.
Ref: CMDT 2014, ch 32, pg. 1395, Harrison 19th p 1184 b. Urinary sodium < lOmq / L
102. Palmar erythema is seen in: ( NBE Pattern 2014-15) c. Urine osmolality lower than plasma osmolality
a. CCF b. ARF d. No sustained improvement in renal function after
c. CRF d. Hepatic failure volume expansion.
Ref CMDT 2014, ch. 16, pg. 686 Ref : Harrison 19 h / 2530, 2531 .

I Ans. 95. a. Liver trans...


99. c. Gilbert disease
96.
100 .
a. Normal GFR
d . Upper Gl bleeding
97
101
. d. Serum albumin 98. a. Alcohol Abuse
. d. Deep jaundice is pre... 102. d . Hepatic failure
134
103. d. Recurrence. .. 104. .
b. Orthotopic liver.. 105. b. Pregnant female 106. d . Splinter hemorrhages
. c. Urine osmolality lower...
107. a . Salt Restriction 108. c. Increasing prothrom... 109
Hepatology

I
Cirrhosis 117. The most common symptom of primary biliary cir ¬

rhosis is: (NBE Pattern 2014-15 )


a. Pruritus b. Weakness
110. Stigmata of chronic liver disease include all of the fol ¬

lowing, except: (DNB 2010) c. Fever d. Pain abdomen


a. Parmar erythema b. Spinder naevi Ref: CMDT 2014, ch. 16, pg. 692, Harrison 19th p 366e-3f !
c . Testicular atrophy d. Subcutaneous nodules
118. Normal liver microscopy is a feature of:
Ref: Internet, Harrison 19th p 364 a. Wilson's disease ( NBE Pattern 2014-15) I
.
111 A 12 years old girl with tremors and emotional liabil¬ b. Dubin-Johnson syndrome
ity has golden brown discolouration in Descemet's c. Gilbert's syndrome
.
membrane The most likely diagnosis is: (AI 04) d. Criggler -Najjar
a. Fabry's disease
Ref Harrison ' s 18 th
ed. ch. 303, Harrison 19th p 2002
b. Wilson's disease
c. Glycogen storage disease 119. Budd- Chiari syndrome is commonly due to:
d. Acute rheumatic fever (NBE Pattern 2014-15)
Ref Harrison's 18/ e p3188, Harrison 19th p 2062 a. Hepatic venous out flow obstruction
.
112 Micro -vesicular fatty liver in Reye's syndrome is? b. Portal Cavernoma
(AlIMS Nov. 14) c. Left Sided portal hypertension
a. Due to defect in beta oxidation of fatty acids d. IVC thrombosis
b. Due to defect in oxidative phosphorylation Ref: CMDT 2014, ch. 16, pg. 697, Harrison 19th p 673

Hepatolgy
c .
Due to defect in fatty acid synthesis
120. The following features differentiate Rotor syndrome
d. Defective synthesis of acyl COA synthesis
from Dubin Johnson's syndrome except: (APPG 2014)
Ref : Nelson 18th Ch 598.2, Harrison 19th p 1164 a. Liver patients with Rotor syndrome has no increased
113. Vitamin K is given to patient of jaundice but PT re ¬
pigmentation and appears normal
.
mains unchanged The probable cause is? b. In Rotor syndrome Gall bladder is usually visualized
(NEET 2014-15) on cholecystography
a. Obstructive jaundice b. Cirrhosis c. Total urinary coproporphyrin is substantially in
.
¬

c Hemolytic jaundice d. Biliary atresia


creased in Rotor syndrome
Ref : Harrison 18th Ch 308, Harrison 19th p 2058 d. Fraction of corpophyrin I in urine is elevated usu ¬

114. The following is the least likely manifestation of acute ally more than 80% of the total in Rotor syndrome
Budd- Chiari syndrome: (NBE Pattern 2014-15)
Ref Harrison's 18thed. ch. 303, Harrison 19th p 2004
a. Enlarged tender liver
b. Ascites 121. Which of the following finding is not suggestive of
c. Jaundice intrinsic hepatic fibrosis: (NBE Pattern 2014-15)
d. Venous collaterals a. Bulging flanks
Ref CMDT 2014, ch. 16, pg. 697, Harrison 19th p 673 b. Collateral flow toward umbilicus
.
115 Spider naevi can occur in: (NBE Pattern 2014-15)
c .Everted umbilicus
a. Rheumatoid arthritis
d. Venous hum
b. Cirrhosis of the liver Ref: Harrison's 18th ed. ch. 308, pg. 3;014, Harrison 19th p 286
c .
Pregnancy .
122 A 40 year old lady has ALP of 550, SGOT of 75, total
d. All of the above
serum Bilirubin = 6.5mg% and conjugated serum
Ref : Harrison's 18th
.
ed ch. 301, Harrison 19th p 2059 bilirubin of 4.3 mg%. The diagnosis of patient is:
116. In Budd- Chiari syndrome there is obstruction to : ( A1IMS NOV 2013 )
a. Inferior vena cava (NBE Pattern 2014-15) a. Dubin Johnson syndrome
b. Pulmonary artery b. Obstructive jaundice
c. Larger hepatic veins
c. Viral hepatitis
d. Portal vein
d. Cholelithiasis
Ref CMDT 2014, ch. 16, pg. 697, Harrison 19th p 673 Harrison's 18th ed. ch. 303, Harrison 19th p 283
Ref
Ans . 110. d. Subcutaneous ... 111. b .
Wilson ’s disease 112. a . Due to defect in beta... 113. b. Cirrhosis
114. d. Venous collaterals 115. d. All of the above 116. c. Larger hepatic veins 117. a . Pruritus
c - Gi|bert’s syn... 119. a . Hepatic venous... 120. d. Fraction... 121. b. Collateral flow toward...
122. b. Obstructive jaundice
p
\
Marwah's Internal Medicine MCQs ( Based on Harrison's 19th)

.
123 Cirrhosis of liver with portal hypertension occurs in .
130 Consider the following statements: Ascites in cirrho ¬

all except: (NBE Pattern 2014 -15) sis of liver is due to (NBE Pattern 2014-15)
a. Cystic fibrosis 1. Portal hypertension
b .
Alpha 1anti-trypsin deficiency 2 .Hypoalbuminaemia
c. Wilson's disease 3. Inappropriate ADH secretion
d. Schistosomiasis 4. Secondary hyper-aldosteronism
a. 1,2 and 3 are correct
Ref: CMDT 2014, ch. 16, pg. 684, Harrison 19th p 2063
b. 1,2 and 4 are correct
.
124 All of the following are true regarding chronic active c. 2,3,4 are correct
hepatitis, except: (NBE Pattern 2014-15) d. 1,3 and 4 are correct'
a .
Common in females
Ref Harrison' s 18 th
. .
ed ch 308
b. Progression to cirrhosis is not seen
c .
Remission with steroids .
131 Which finding suggests a SVC obstruction versus asci ¬

tes? (NBE Pattern 2014-15)


d. May associate with autoimmune disease
a. Bulging flanks
Ref CMDT 2014, ch. 16, pg. 806, Harrison 19th p 2030 b. Collateral flow towards umbilicus
.
125 Best Treatment of refractory ascites is: c. Everted umbilicus
a .
AV shunt (NBE Pattern 2014-15) .
d Pulsatile liver .
Ref Harrison’s, ch 44: lTh ed , .
b .
TIPS .
132 Purtscher’s retinopathy results from:
c .
Frusemide with paracentesis a. Air embolism (NBE Pattern 2014-15)

SB
o
d. Distal splenorenal shunt
Ref Harrison ’s 18th
ed. ch. 308, Harrison 19th p 288/ 2065
b. Chronic Pancreatitis
c .Parasitic infections
d. Compressive chest injury
"o 126. Low serum copper due to ATP 7 A gene is due to?
a. Dubin-Johnson’s syndrome ( NBE Pattern 2014 -15) Ref Harrison' s 18 th
. .
ed ch 313,
CC b. Wilson disease 133. Following is test marker of alcohol induced liver injury:
a) c .
Menke disease (NBE Pattern 2014-15)
0 d. Gilbert's disease a. Gamma Glutamyl Transferase
X Ref Nelson 18 ed., Ch. 335, Harrison 19th p 96e-10

b. MCV
c. SGPT/ SGOT >2
.
127 Incorrect about Wilson disease (AIIMS NOV 2012) d. SGOT/ SGPT >2 Ref CMDT 2014, ch. 16, pg. 680,
a. Decrease in urinary copper .
134 Gamma glutamyl transferase is elevated in:
b. Decrease serum copper a. Liver abscess (NBE Pattern 2014-15)
c. Decrease in serum ceruloplasmin b. Viral hepatitis
d. Increase serum copper c. Alcoholic liver disease
.
Ref CMDT 2014, ch. 16, pg 696, Harrison 19th p 2625 d. Secondaries in liver
128. Most common cause of congestive splenomegaly is: Ref CMDT 2014, ch. 16, pg. 681
(NBE Pattern 2014-15) .
135 All of the following are noticed in cirrhosis of liver,
a. Chronic congestive cardiac failure except: (NBE Pattern 2014-15)
b. Cirrhosis a .
Raised serum albumin
c. Hepatic vein occlusion b. Excessive urobilinogenuria
d. Stenosis of splenic vein c. Prolonged prothrombin time
d. Raised serum globulin
Ref CMDT 2014, ch. 14, pg. 547
Ref CMDT 2014, ch. 16, pg. 686
.
129 Most common cause of Budd-Chiari syndrome is:
a .
Hepatic vein valves ( NBE Pattern 2014-15) 136. Friction rub may not be heard in right upper quadrant
b. Hypercoagulable state in Nephrotic syndrome in one of the following conditions:
(NBE Pattern 2014-15)
c . PNH
a. Fatty liver b. Recent biopsy
d . Polycythemia vera
c . Tumour d. Perihepatitis
Ref: CMDT 2014, ch. 16, pg. 697, Harrison 19th p 673
Ref CMDT 2014, ch. 16, pg. 684, Harrison 19th p 2053
Ans. 123. d. Schistosomiasis 124. b. Progression to cirrho... 125. b. TIPS 126. c. Menke disease
127. a. Decrease in uri... 128. b. Cirrhosis 129. d. Polycythemia vera 130. b. 1,2 and 4 are correct
131. b. Collateral flow... 132. d. Compressive chest... 133. d. SGOT/SGPT >2 134. c. Alcoholic liver disease
136
135. a. Raised serum... 136. a. Fatty liver
Hepatology

137. A 45 - year - old cirrhotic patient presented with severe 144. True about Crigler Najjar type 11 syndrome is:
haematemesis. The management of choice is: a . Diglucuronide deficiency ( PGI Dec 97 )
( NBE Pattern 2014 -15) b. Recessive trait
a. Whole blood transfusion is the best c. Kernicterus is seen
b. Colloids are preferred over crystalloids d. Phenobarbitone is not useful
c. Normal saline infusion Ref : H 18th/ p 326, 2533, 2534, Harrison 19th p 2001
d . IV fluid with diuretics
145. All of the following statements about Non Alcoholic
Ref : Harrison's 18thed. ch. 270 Fatty Liver disease are true, except: ( PGI June 01 )
138. Raised unconjugated hyperbilirubinemia is seen in: a. Common in Diabetics
a . Gilbert's Synd. ( NBE Pattern 2014-15) b. Clofibrate provides effective treatment
b. Dubin Johnson synd. c. Commonest cause of cryptogenic cirrhosis
c. Drug induced cholestasis d. Associated with elevated transminases
d. Hepatocellular necrosis Ref : H 18th/ 2604, 2605; 17th/ 1982; Robbins 7th/ 907, 908.
Ref: CMDT 2014, ch. 16, pg. 662, Harrison 19th p 2002 146. Micronodular cirrhosis is commonly seen in all ex¬
139. Gilbert's syndrome all are true except? cept: ( AIIMS Nov 07 )
a. Mild conjugated bilirubinaemia a. Chronic hepatitis B
b. Normal LFT ( NBE Pattern 2014 -15) b. Alcoholic liver disease
c. Normal liver biopsy c. Hemochromatosis
d. Ligandin defect d. Chronic extrahepatic biliary obstruction

Hepatolgy
Ref Harrison’s 18th ed. ch. 303, Harrison 19th p 2002 Ref : Pathology: Basic and systemic' by Woolfe ( 1998)/ 587,597.
140. Which one of the following is NOT true about Ascites? 147 . Which of the following is the most common presenting
( NBE Pattern 2014 -15) symptom of non - cirrhotic portal hypertension?
a. S.A.A.G > 1.1 is seen with portal hypertension a. Chronic liver failure (AI 2006 )
b. S.A.A.G < 1.1 is seen with Nephrotic syndrome b . Ascites
c. Pseudochylous ascites is seen with hypertriglyceride ¬ c. Upper gastrointestinal bleeding
mia d . Encephalopathy
d. Black ascitic fluid is seen with pancreatic necrosis Ref : Schwartz 7 th/ 1420, 1421.
Ref CMDT 2014, ch. 15, pg. 587, Harrison 19th p 288 148. A man presents with history of hemetemesis of about
141. Most common cause of non -alcoholic fatty liver dis ¬ 500 ml of blood . On examination, spleen is palpable
ease? ( NBE Pattern 2014 -15) 5 cms below the left costal margin . The most likely
a . Reye syndrome b. Syndrome-X diagnosis is: ( AI 2012)
c. Cardiac syndrome-X d. Pregnancy a. Portal Hypertension
b. Gastric ulcer
Ref: CMDT 2014, ch. 27, pg. 1196, Harrison 19 thp 2057
c. Drug induced
142 . The most frequent location for spider angiomata in d . Mallory Weiss Tear Ref : Harrison 18th / 320, 2598.
cirrhosis is: ( NBE Pattern 2014-15)
149. What is the most common cause for Budd chiari syn ¬
a. Abdomen
drome ( AIIMS Nov 2000 )
b. Back
c. Neck and shoulders
a. Right ventricular failure
b. PNH
d . Upper & lower extremities
c. Valve in hepatic veins
Ref CMDT 2014, ch. 16, pg. 689 d . Polycythemia vera
143 . A patient presents with unconjugated hyperbilirubi ¬
Ref : Robbins 6 th/ 883; 7th/ 919.
nemia and presence of urobilinogen in urine . Which
150 . In Budd Chiari syndrome, the site of venous thrombo
amongst the following is the least likely diagnosis: ¬

a. Hemolytic jaundice sis is : ( A12004 )


( A12010 )
b. Crigler Najjar syndrome a . inrranepatic inferior vena cava
c. Gilbert's syndrome .
b Infrarenal inferior vena cava
d. Dubin Johnson syndrome c. Hepatic veins
d. Portal veins Ref : Harrison 18th / 43,306 .
Ref : Harrison 18th/ p 325, 326, Harrison 19 th p 2003
Ans. 137. a . Whole blood tra.. 138. a. Gilbert’s Synd . 139. a. Mild conjugated ... 140. c. Pseudochylous...
141. b. Syndrome X- 142. c. Neck and shoulders
145. b. Clofibrate pro... 146 . a. Chronic hepatitis B
143. d . Dubin Johnson ... 144. b. Recessive trait
149. d . Polycythemia... 150. c. Hepatic veins
.
147 c. Upper gastroin.. . 148. a . Portal Hypertension
th
Marwah's Internal Medicine MCQs ( Based on Harriso n 's 19 )

.
151 All of the following are features of Wilson's disease, 157. Most common cause of obstructive jaundice in chil ¬

except - (AIIMS Dec 97 ) dren: (NBE Pattern 2014-15)

i
a. Hemolytic anemia a . Biliary atresia
b. Testicular atrophy b . Criggler najjar syndrome
c .Chorea c . Byler disease
d. Chronic active hepatitis .
d Caroli cyst
Ref: Harrison 's 18th
ed. ch. 16 pg. 663, Harrison 19th p 283
Ref.: Harrison 18th / p 3187, 3188.
158. A defect in which of the following processes give rise
152. All of the following statements about. Wilson’s disease
to bilirubinuria? ( NBE Pattern 2014-15)
are true, EXCEPT- (AIIMS May 04)
a. Conjugation of bilirubin to glucuronic acid
a It is an autosomal recessive disorder
b. Conversion of biliverdin to bilirubin
b. Serum ceruloplasmin level is < 20 mg/ dl c. Transport of conjugated bilirubin to bile cana -
c. Urinary copper excretion is <100 pg/ dl liculi
d. Zinc acetate is effective as maintenance therapy .
d Transport of unconjugated bilirubin into hepato-
Ref : Harrison 18th/ p 3188, 3189 cytes Ref: Harrison's 1 Th ed. ch. 43
.
153 A patient presents with Arthritis, hyperpigmentation 159. Investigation of choice in obstructive jaundice is:
of skin and hypogonadism, likely diagnosis is - (NBE Pattern 2014-15)
a. ERCP b. Ultrasound
a. Hemochromatosis (A! 2001)
b. Ectopic ACTH secreting tumour of the lung
c. Cholecystography d MRCP .
.

Hepatolgy
c. Wilson's disease Ref: Harrison's 18th ed. ch 311, Harrison 19th p 283
d. Rheumatoid arthritis 160. All of the following statements about primary scleros ¬

ing cholangitis are true, Except: (PCI Dec 03)


Ref : Harrison 18th / p 3164, 3165. a . Increased incidence in females
154. Earliest phenotypic manifestation of Idiopathic he ¬
b. Associated with Inflammatory bowel disease
reditary hemochromatosis is- ( AIIMS May 07) c . May involve both intra and extra hepatic ducts
a. Post prandial increase in serum iron concentration .
d ERCP is a sensitive investigation
b. Elevated serum ferritin level Ref : Harrison 18th / p 2596.
c. Slate grey pigmentation of skin
d. Increased transferrin saturation Cholelithiasis & Cholecystitis
Ref.: Harrison 18th / p 3165, 3166,
161. Which of the following is not an indication for chole ¬

Primary Biliary Cirrhosis cystectomy: (AIIMS May 05)


a. 70 year old male with symptomatic gall stone
b. 20 years old male with sickle cell anemia and
155. Obstructive jaundice is better detected by:
symptomatic gallstones
(NBE Pattern 2014-15)
c. 65 year old female with a large gallbladder polyp
a. Increased ALP d. 55 year old with an asymptomatic gallstone
b. Decreased ALP
Ref: Harrison’s 18/ e p2620, Harrison 19th p 2083
c. Increased AST
162. Medical treatment of gallstone indicated in A /E:
d. Decreased AST a. GB should be functioning (NBE Pattern 2014-15)
Ref: CMDT 2014, ch. 16, pg. 664, Harrison 19th p 283 b. Gallstone should be radiolucent
156. All of the following are true about Primary Biliary c. Gallstone should be radi- opaque
Cirrhosis except: ( NBE Pattern 2014-15) d. Patient is unfit for surgery
a. Increase 5'- nucleotidase Ref: Harrison's 18th ed. ch. 311 , Harrison 19th p 2076
b. Median age 50 163. Incorrect about Mirrizi syndrome:
c. Second most common cause of cholangitis in chil ¬
a. Jaundice (NBE Pattern 2014-15)
dren b. Cystic duct obstruction
d. PBC frequently associated with CREST syndrome c . MRCP for investigation
d. Hemobilia
Ref: CMDT 2014, ch. 16, pg. 692 366e-3f
Ref: Harrison's 18thed. ch. 311.
Ans. 151. b. Testicular atrophy 152. c. Urinary copper... 153. a. Hemochromatosis 154. d. Increased transferrin...
155. a. Increased ALP 156. c. Second most... 157. a. Biliary atresia 158. c. Transport of con...
159 . b. Ultrasound 160. a. Increased incidence...
138 161. d. 55 year old with an... 162. c. Gallstone should be...
163. d. Hemobilia
Hepatology

164. Most common cause of hemobilia is: 172. Increased B 12 level is seen in all, except: ( Al 2000 )
a. Trauma ( NBE Pattern 2014-15) a. Cirrhosis
b. Hemangioma b. Primary hepatocellular Ca
c. Rupture of hepatic artery aneurysm c. Hepatitis
d. Hepatitis d. Cholestatic jaundice
Ref: CMDT 2014, ch. 15, pg. 581, Harrison 19 th p 2084 Ref.:Harrison 189th/ 42, 305, Harrison 19 th p 284
165. Medical treatment in gallbladder stone is amenable 173. Lithogenic bile has the following properties :
for: ( NBE Pattern 2014 15) - a. Decreased Bile and 1 Cholesterol ratio ( Al 1996 )
a. Size of stone less than 10 mm b. T Bile and decreased cholesterol ratio
b. Radiopaque c. Equal bile and cholesterol
c. Calcium bilirubinate oxalate d . Cholesterol only
d. GB non -functioning Ref : Harrison 17 th / 1991 , 1992 .
Ref: Harrison' s 18th ed. ch. 311, Harrison 19th p 208 174. Stone formation in Gall bladder is enhanced by all
166. HIDA scan is useful in: ( NBE Pattern 2014-15) except ( Al 1996 )
a. Acute cholecystitis a . Clofibrate therapy
b. Meckel’s diverticulum b. Dial resection
c. Colonic angio-dysplasia c. Cholestyramine therapy
d . Diverticulitis d . Vagal stimulation Ref : Bailey 23rd/ 975.
Ref CMDT 2014, ch. 16, pg . 703, Harrison 19 th p 2080 175. Incidence of gall stone is high in: ( AIIMS Nov 93)

Hepatolgy
a. Partial hepatectomy b. Ileal resection
167 . In a patient with fever, nausea and pain in right hy- c. Jejunal resection d. Subtotal gastrectomy
pochondrium for 6 hours. Liver is not palpable. Most
Ref : Harrison's 18th / edn, Ch 311 .
probable diagnosis is: ( NBE Pattern 2014-15)
a. Viral hepatitis b. Acute cholecystitis 176. Most common site for impaction of gall stones in gall
c. Gastritis d . Pleurisy stone lieus is : (Al 1999 )
Ref CMDT 2014, ch. 16 , pg. 704, Harrison 19 th p 2080 a. 1st part of duodenum
b. llnd part of duodenum
168. Most common site of Gallstone ileus: c. Terminal ileum
( NBE Pattern 2014-15) d . Colon
a. Caecum b. 2 nd pt. Of duodenum Ref : Love & Bailey 23rd/ 1066 ; Sabiston 16 th/ 1095 .
c. Terminal ileum d . Stomach 177 . Internal fistula is most common between gall bladder
Ref CMDT 2014, ch. 16, pg . 703, and: ( ARMS June 93)
169. Investigation of choice in Hemobilia: a. Colon b. Duodenum 1st part
c. jejunum
( NBE Pattern 2014 15) - d. Transverse colon
Ref : Diseases of Gall bladder and Bile duct lst/ 188.
a. ERCP b. Angiography
c. Upper GI endoscopy d . Barium study 178. Medical treatment for gall bladder stone is amenable
for : ( Al 1998 )
Ref Harrison's 18th ed. ch. 311 . pg. 2625, Harrison 19th p 2089 a. Stones less than 15 mm in size
170. A patient presented with complaints of abdominal
b. Radioopaque stone
pain, melena, jaundice, and fever of 104°C; diagnosis
c. Calcium bilirubinate oxalate stone
will be: ( NBE Pattern 2014-15)
d . Nonfunctioning stones associated with gall bladder
a. Ca pancreas b . CBD stones
c. Hemobilia d . Hepatitis Ref : Harrison 18th / p 2621 .
179. Risk factors for malignant change in an asymptom ¬

Ref CMDT 2014, ch. 15, pg. 581, Harrison 19th p 2089 atic patient with a gall bladder polyp on ultrasound
171. 5 ’ - Nucleotidase activity is increased in : ( Al 2005) include all of the following, Except: ( Al 2009 )
a. Bone diseases. a. Age > 60 years
b. Prostate cancer. b. Rapid increase in size of polyp
c. Chronic renal failure. c. Size of polyp > 5 mm
d . Cholestatic disorder d . Associated Gall stones
Ref : Harrison 18th/ 42, 305, Harrison 19 th p 284 Ref.: Schwartz 8th / 1214, Manual of Gastroenterology 4th/ 457.

ns. 164. a. Trauma 165. a . Size of stone less... 166. a. Acute cholecystitis 167. b. Acute cholecystitis
168. c. Terminal ileum 169. b. Angiography 170. c. Hemobilia 171. d. Cholestatic disorder
172. d . Cholestatic jau ... 173. a . Decreased Bile... 174. d . Vagal stimulation 175. b. Ileal resection 139
176. c. Terminal ileum 177. b. Duodenum 1st part 178. a. Stones less than 15... 179. c. Size of polyp > 5 mm
Marwah's Internal Medic ine MCQs ( Based on 's th
Harrison 19 )

Hepatic Encephalopathy 187. Alzehiemer type II astrocyte are seen in:


a. Hepatic encephalopathy ( NBE Pattern 2014-15)
b. Alzehiemer’s
180. Which one of the following is least expected to pre ¬
c. Parkinsonism
cipitate hepatic encephalopathy in a liver cirrhosis
d. Biswanger disease
| patient? ( AP 2010 )
.
Ref: CMDT 2014, ch 16, pg. 690, Harrison 19th p 441e -42f
a. Peritoneal tap b. Antibodies treatment
c. Variceal bleed d . Hypokalemia 188. Acute hepatic encephalopathy is precipitated by:
Ref: Harrison's 18/ e p2601, Harrison 19th p 252 a. Lactulose ( NBE Pattern 2014 -15)
b. Potassium sparing diuretics
181. Number Connection test is done in? [ JIPMER 2014 ) c. Excessive use of diuretics
a. Cerebral Ataxia b. Dementia d. High protein diet
c. Parkinsonism d. Hepatic Encephalopathy .
Ref: CMDT 2014, 2014, ch 16, pg. 690, Harrison 19th p 441e -42f
Ref : Harrison 19th p 441e-42f 189. Which one of the following is not advocated in the
182 . All of the following are characterized by depletion of management of hepatic encephalopathy?
the intracellular water, except: a. Oral Lactulose ( NBE Pattern 2014 -15)
( NBE Pattern 2014 -15) b. l .V. Glucose drip
a. Massive diarrhea b. Peritonitis c. High protein diet more than 60 grams / day
d. If tests for blood in stool are positive then give co ¬
c. Pancreatitis d . Hepatic coma
lonic washout
Ref : ch 308 Harrison' s 18th, Harrison 19th p 1775 .
Ref: CMDT 2014, ch 16, pg. 690, Harrison 19th p 441e -42f

Hepatlogy
183. Which one of the following is least expected to precipi ¬ 190. Portacaval Encephalopathy is treated with:
tate hepatic encephalopathy in a liver cirrhosis patient: a. Lactulose ( NBE Pattern 2014-15)
a. Peritoneal tap ( NBE Pattern 2014 -15) b. Large amounts of proteins
b. Antibiotic treatment c. Emergency portal systemic shunt surgery
c. Variceal bleed d. Diuretics Ref : Harris on' s 18th
ed., ch 308
d. Hypokalemia 191. All the following drugs are used in hepatic encepha ¬
Ref CMDT 2014, ch. 14, pg . 690, Harrison 19th p 441e-42f lopathy except: ( NBE Pattern 2014 -15)
a. LOLA b. Rifaximin
184. Hepatic encephalopathy is aggravated by all except?
c. Lactulose d. Phenobarbitone
a. Hyperkalemia ( NBE Pattern 2014-15)
b. Anemia
Ref: CMDT 2014, ch. 16, pg . 690, Harrison 19th p 441e 42f -
c. Hypothyroidism 192. All of the following is associated with recurrent acute
d . Barbiturates pancreatitis except: ( NBE Pattern 2014 -15)
a . Hypertriglyceridemia b. Cystic fibrosis
Ref CMDT 2014, 2014, ch. 16, pg. 690, Harrison 19th p 441e -42f d . All of the above
c. Pancreatic cancer
185. Patient of acute pancreatitis developed sudden loss of Ref Harrison's 18th ed. ch. 313, Harrison 19th p 2091
vision the most likely cause is: ( NBE Pattern 2014-15)
193. Antibiotic of choice in cirrhotic patient to prevent
a. Purtscher's retinopathy encephalopathy: ( NBE Pattern 2014 -15)
b. Hyperglycemia a. Neomycin b. Ampicillin
c. Hypoxia c. Metronidazole d. Rifaximin
d. CRVO Ref CMDT 2014, 2014, ch. 16, pg. 690, Harrison 19th p 441e-42f
Ref Harrison's 18 ' ed. ch. 313, Harrison 19th p 2091
“ 194. Hepatic coma is manifested by all the following except:
186. Earliest sign in hepatic encephalopathy is: a. Respiratory alkalosis ( NBE Pattern 2014 -15)
a. Asterixis ( NBE Pattern 2014-15) b. EEG abnormalities
b. Alternate constriction and dilated pupil c. Elevation of blood ammonia
c. Constructional apraxia d . Flapping tremor
d. Psychiatric abnormalities e. Anxiety & increased activity
Ref CMDT 2014, ch. 16, pg. 690, Harrison 19th p 441e-42f Ref: ch 308 Harris on' s 18 th
ed., Harrison 19th p 1775

Ans . 180. a. Peritoneal tap 181. d. Hepatic Encephalopathy 182. d . Hepatic coma 183. b. Antibiotic treatment
I 184. a. Hyperkalemia 185. a. Purtscher ’s retinopathy 186. d. Psychi atric ..
abnor. 187. a . Hepatic enceph
191. d. Phenobarbitone
alopath y

140 188. c. Excessive use... 189. c. High protein diet more... 190. a. Lactulose
192. d. All of the above 193. d . Rifaximin 194. a. Respiratory alkalosis
Hepatology

203. All of the following are important clinical manifesta ¬


Liver & Gallbladder Cancer

195. Increase in alpha - fetoprotein is seen in:


tions of hepatocellular carcinoma except:

a. Jaundice
( All India 2011 )
b. Abdominal pain
I
( AllMSJune 2000)
c. Abdominal mass d. Ascitis
a. Hepatoblastoma b. Neuroblastoma
c. Thymoma d . Angiosarcoma Ref : Robbins 581.
Ref: Robbin's 8/ e p327; Table 7 -12, Harrison 19th p 120e-l -
204. Increase in alpha fetoprotein is seen in :
a. Hepatoblastoma ( AIIMS June 2000 )
196. Best for management of 10 cm hepatocellular carci ¬ b. Neuroblastoma
noma is:
c. Thymoma
a. Radiofrequency ablation ( NBE Pattern 2014-15)
b. Transarterial catheter embolization (T.A.C.E}
d. Angiosarcoma Ref : Harrison 18th/ p 779 .
c. Percutaneous ethanol 205. About fibrolamellar carcinoma, what is TRUE :
d. Orthoptic liver transplantation a. Diffuse in nature ( AIIMS May 94 ) I
.
Ref: CMDT 2014, ch 39, pg. 1603, Harrison 19th p 545 b. Occurs after 60 years of age
c. Cirrhosis is the most common presenting feature
197 . Most common liver tumour associated with OCP: d . Has better prognosis
a. Focal nodular hyperplasia
Ref.: Harrison 18th/ p 784.
b. Hemangioma ( NBE Pattern 2014-15)
c. Angiomyolipoma 206. All of the following are known predesposing factors
d. Hepatocellular adenoma for cholangiocarcinoma except : ( AI 1997 )

Hepatolgy
Ref: CMDT 2014, H 18th pg. 701, Harrison 19th p 365 a . CBD stones
b. Clonorchis sinensis
198. Menghini ’s needle is used for: ( NBE Pattern 2014-15)
c. Ulcerative colitis
a . Pleural aspiration b. Lumbar puncture
d. Primary sclerosing cholangitis
c. Kidney biospy d. Liver biopsy
Ref : Harrison 18th / p 784.
Ref: CMDT 2014 2014, ch. 16 , pg. 665, Harrison 19th p 336e-3f
199. Best to diagnose a liver tumour? Miscellaneous
( NBE Pattern 2014-15)
a. CT b. USG 207. The genetic defect in Dubin -Johnson syndrome is:
c. MR1
-
d . Sulphur colloid scan
Ref: CMDT 2014, H 18a pg. 701, Harrison 19th p 534
200. Most common benign tumor of the liver:
( Manipal )
a. Mutation in gene for multiple -drug resistance Pro ¬
tein 2 4
b. Mutation in UDP-glucuronyl transferase
a. Focal nodular hyperplasia ( NBE Pattern 2014 -15)
c. Mutation in chromosome 23
b. Hemangioma
d. Flash mutations
c. Angiomyolipoma
d. Hepatocellular adenoma Ref: Harrison 19th p 282/ 2003
Ref: CMDT 2014, H 18th pg . 701, Harrison 19th p 5 208. Inpatients with cirrhosis of the liver the site of ob ¬
201. Granulomatosis hepatitis is not caused by: struction in the portal system is in the: ( Manipal )
a. Blastomycosis ( NBE Pattern 2014-15 )
a. Hepatic vein
b. Metastatic carcinoma b . Post sinusoidal
c. Tuberculosis c. Extra hepatic portal vein
d. Cat scratch disease Ref: Harrison 19th p 2054 d. Sinusoids Ref: Harrison 19th p 2067
202 . All of the following are risk factors for Hepatocellular 209. In Bud Chiari syndrome, the site of venous thrombosis
carcinoma except: ( All India 2010 ) is: ( Manipal 2014 )
a. Hepatitis C infection a. Intra- hepatic inferior vena cava
b. Alcoholism b. Intra - renal inferior vena cava
c. Alfatoxins c. Hepatic veins
d. Animal fat in diet d . Portal vein
Ref : Harrison 18th / p 777. Ref: Harrison pg 1759 Harrison 19th p 399/ 673
Ans. 195. a. Hepatoblastoma 196. d . Orthoptic liver trans...
199.
197. d . Hepatocellular ade... 198. d . Liver biopsy
c. MRI 200. b. Hemangioma
203. 201 . b. Metastatic carcinoma 202. d . Animal fat in diet

1
a . Jaundice 204. a . Hepatoblastoma 205. d . Has better prognosis 206. a . CBD stones
207 . a. Mutation in gene... 208 . d . Sinusoids 209. c. Hepatic veins

i
Marwah's Internal Medicine MCQs ( Based on Harrison's 19 th
)

210. Vascular congestion is seen in all except: 217. All of the following are true of Reye's syndrome, ex-
a. Strawberry gallbladder (Manipal 2014) cePt: (Feb DP PGMEE 2009)
b. Gamma gandy bodies a. If frequently complicated viral infections
c. Nutmeg liver b. Prothrombin time is prolonged
d. Cor pulmonale c. Disease may be precipitated by salicylates
d. Deep jaundice is present
Ref: Manipal Manual of Surgery pg 350 Harrison 19th p 2083
Ref: H 18/ e pl 463, 1465, 1417, Harrison 19th p 1184
211. In patients with cirrhosis of the liver the site of ob ¬

struction in the portal system is: ( Manipal 2014) 218. Patient present with fasting sugar as 167mg/ dL, skin
a. Hepatic vein pigmentation and hypogonadism. His liver enzymes
b. Post sinusoidal showed SGOT as 678 and SGPT as 692. Most probable
c. Extra hepatic portal vein diagnosis is: (DNB 2013)
a. Alpha 1 antitrypsin deficieny
d. Sinusoids
b. Wilson's disease
Ref: Harrison 19th p 2058 -2059 c. Hemochromatosis
212. All of the following causes cirrhosis except: d. Glycogen storage disease
a. Wilson's disease (Manipal 2014) Ref: Harrison's 18 / e p3165, Harrison 19th p 2514
b. Hemochromatosis
219. CURB- 65 include all except: ( NEETPattern Question )
c. Paracetamol a. Confusion b. Urea decreased
d. Cystic fibrosis c. Respiratory rate d. Blood pressure

Hepatlogy
Ref: Harrison's 18th pg 2592, Harrison 19th p 2058-2059, 2075 Ref: Harrison's 18/ e p2134, Harrison 19th p 806 -807
213. Number connection test is done to detect: 220. Lady with cystic fibrosis with chronic pancreatitis will
a. Parkinsonism (JIPMAR 2014) have deficiency of all except: (NBE Pattern 2014-15)
b. Dementia a. Vitamin B12 b. Vitamin A
c. Cerebellar ataxia c. Pancreatic cancer d. Niacin deficiency
d. Heptic encephalopathy Ref: Harrison 's 17 th
ed. ch. 307, Harrison 19th p 1699
Ref : Harrison 19th p 567, 2338 221. False statement regarding spontaneous bacterial peri¬
tonitis is: (NBE Pattern 2014-15)
214. Which of the following is true regarding gamma glu ¬

( JIPMAR 2014 ) a. Infection is preceded by ascites


tamyl transferase?
b. Clinical features are abdominal Pain, fever, leucocy -
a. Elevated in Ca prostate with hepatic metastasis
tosis and altered mental status
b. Elevated in fatty liver
c. Ascitic fluid protein of lgm/ dl
c. Exclusively found in liver
d. Common organisms are Gram negative organisms
d. Marker of Infective hepatitis rather than cholestasis
Ref : Harrison ' s 18th
ed. ch. 308, Harrison 19th p 2065
Ref: Harrison 19th p 579
222. Ranson's criteria includes all except:
215. Which of the following is a vaccine preventable can
¬

a. Fall in hematocrit > 10% (NBE Pattern 2014-15)


cer? (JIPMAR 2014)
b. Calcium < 8 mg%
a . Hepatocellular carcinoma
c. WBC > 16,000
b. Renal cell carcinoma
d. Base deficit > 2 Ref: Harrison ' s 18th
ed. ch. 313.
c. Lymphoma
d. Kaposi sarcoma 223. Grey Turner's sign is seen in: (NBE Pattern 2014-15)
Ref : H 18th edn. Harrison 19th p 544 a . Myocarditis b. Cholecystitis
c. Pancreatitis d. Pleural effusion
216. Which of the following disease is not a cause of indi ¬

rect hyperbilirubinemia? (DP PGMEE 2009) Ref : Harrison' s 18th


ed. ch. 313.
a. Rotor's syndrome 224. Causes of acute pancreatitis are A /E:
b. Criggler najjar syndrome ( NBE Pattern 2014 -15)
c. Gilbert syndrome a. Hypocalcemia b. Valproic acid therapy
d. Hereditary spherocytosis c. Biliary tract disease d. Blunt trauma
Ref: Harrison 19th p 282 Ref: Harrison's 18th
ed. ch. 313 , Harrison 19th p 209

Ans . 210 . a. Strawberry gall... 211. d. Sinusoids 212. D. Cystic fibrosis 213. d. Heptic encephalopathy
214. a. Elevated in Ca... 215. a. Hepatocellular car... 216. a. Rotor ’s syndrome 217. d. Deep jaundice is present
218. c. Hemochromatosis 219. b. Urea decreased 220. d. Niacin deficiency 221. a. Infection is preceded...
142 224. a. Hypocalcemia
222. d. Base deficit > 2 223. c. Pancreatitis
Hepatology

225. Pencillamine is mostly used in: ( NBE Pattern 2014-15) 232 . Medical treatment of acute pancreatitis to reduce
a . Hepatolenticular disease pancreatic damage: ( NBE Pattern 2014-15 )
b. Penicillin anaphylaxis a. Glucagon b. Aprotinin
c. Haemochromatosis c. Calcitonin d . Gabexate
d. Tertiary syphilis Ref: Harrison’s 18" ed. ch. 313, Harrison 19th p 2091
Ref: CMDT 2014, ch. 16 , pg. 697.
233. A patient presented to emergency ward with massive
226. Forrest classification is used for evaluating: upper gastrointestinal bleed . On examination , he has
a. Upper GI bleeding ( NBE Pattern 2014-15) mild splenomegaly. In the absence of any other infor ¬
b. Liver transplantation mation available . Which of the following is the most
c. Lower GI bleeding appropriate therapeutic modality?
d. Familial adenomatous polyposis a. Intravenous propranolol ( NBE Pattern 2014 -15)
Ref: CMDT 2014 2014, ch. 15, pg. 580 b. Intravenous vasopressin
7.7.1. Mainly cholecystokinin is secreted by: c. Intravenous pantoprazole
a. Duodenum ( NBE Pattern 2014-15] d . Intravenous somatostatin
b. Pancreas Ref: Harrison's - 18"' ch. 41 and ch. 308
c. Gallbladder
234. Diabetic ketoacidosis mimics acute pancreatitis in all
d. Ileum
findings except: ( NBE Pattern 2014-15)
Ref: Harrison's 18th ed., Ch.313, Harrison 19th p 2053 b. Elevated lipase
a. Elevated amylase

Hepatolgy
228. The following can be associated with acute pancreati ¬ c. Abdominal pain d. Hyperglycemia
tis except: ( NBE Pattern 2014- 15) Ref: Harrison' s 18th ed. ch. 313, Harrison 19 th p 2091
a . Hyperparathyroidism
b. Cystic fibrosis 235 . Child with S.A.A.G < 1.1 gm / dl: The probable diagnosis
c. Hypercalcemia of the child is: ( NBE Pattern 2014-15)
d. Hypercholestrolemia a. Cirrhosis b. Portal hypertension
c. CHF d . Nephrotic syndrome
Ref: Harrison's 18th ed . ch. 313.
Ref: CMDT 2014, ch. 15, pg. 588
229. Which is not characteristic of portal hypertension:
a. Splenomegaly ( NBE Pattern 2014-15) 236. Major symptom of acute pancreatitis is:
b. Hypersplenism a. Abdominal bloating ( NBE Pattern 2014- 15)
c. Ascites b. Agonizing upper abdominal pain
d. Gynaecomastia c. Jaundice
Ref: CMDT 2014, ch. 16 , pg. 699, Harrison 19 th p 2063 d . Constipation
Ref: Harrison's 18th ed . ch. 313, Harrison 19th p 2091
230. Feature of Granulomatous hepatitis:
a. Jaundice is a usual feature ( NBE Pattern 2014-15) 237. Rockall score is used for prognosis of patients of:
b. Seen with sarcoidosis a. Upper GI bleeding ( NBE Pattern 2014 - 15)
c. Seen with syndrome X b. Lower GI bleeding
d. Seen after jejuno-ileal bypass surgery in obese c. Hepatic encephalopathy
d. 1 BD
patients
Ref: Gastrointestinal emergencies, 2nd pg 14
Ref: CMDT 2014, ch. 26, pg. 1141, Harrison 19th p
238. Zieve syndrome is characterized by all except?
231. Which is the diagnostic test in pancreatic insuffi ¬
a. Alcohol abuse ( NBE Pattern 2014 - 15)
ciency: ( NBE Pattern 2014-15) b. Hemolysis
a . Schilling test c. Hypertriglyceridemia
b. Serum lipase d. Pancreatic lipase deficiency
c. Serum amylase Ref: http:// www.ncbi.nlm.nih.gov/ pubmed/ 2208946 Hypertri ¬
d . Fecal Elastase estimation
glyceridemia, steatosis, and hemolysis ( Zieve syndrome ) may be
Ref: Harrison's 18 ed. ch. 313, Harrison 19 th p 269
“ associated with alcohol abuse., Harrison 19th p 2059

Ans . 225. a . Hepatolenticular... 226. a . Upper GI bleeding 227. a . Duodenum 228. d . Hypercholestrolemia
229. d . Gynaecomastia 230. b. Seen with sarcoidosis 231. d. Fecal Elastase... 232. d . Gabexate
233. d. Intravenous... 234. b. Elevated lipase 235. d. Nephrotic syndrome 236. b . Agonizing upper... 143
37. a. Upper GI bleeding 238. d . Pancreatic lipase deficiency
Marwah's Internal Medicine MCQs ( Based on Harrison's 19 th
)

.
239 True about hepatitis with (HCV) is: .
248 Criteria for severity in acute pancreatitis includes all
( NBE Pattern 2014-15] excePt: (NBE Pattern 2014-15)
a. Present with fulminant liver failure a. 3 fold increase in serum lipase
b .
Chronicity is not seen b. Serum creatinine > 2.0mg%

B
c. Genotyping helps in treatment duration c. Pa02 < 60mmHg
d. Faeco / oral transmission d. SBP < 90
Ref: CMDT 2014, ch. 16, pg. 670, Harrison 19th p 203 Ref: CMDT 2014, ch.16, pg. 711, Harrison 19th p 2093t
240. Ranson’s criteria for acute pancreatitis include all .
249 Increased amylase seen in A / e: (NBE Pattern 2014-15)
except: (NBE Pattern 2014-15)
a .
Hyperglycemia b. LDH>250U
a .
Pancreatic pseudocyst
b. Chronic pancreatitis
c .
Hypocalcemia d. Hyperamylasia
c. Perforated peptic ulcer
Ref: Harrison's 18* ed. ch. 313 Harrison 19th p 2091 d. Ruptured ectopic pregnancy
.
241 Which extraintestinal symptom of IBD worsens with Ref: Harrison' s 18aed. ch. 313, Harrison 19th p 2088
exacerbation of disease activity? (NBE Pattern 2014-15)
a. Ankylosing spondylitis .
250 Bilirubin is absent in urine because it is -
b. Arthritis a. Distributed in the body fat (AI1MS Nov 99)
c. PSC b. Conjugated with glucoron
d. Uveitis Ref: Harrison’s 17th ed ch 289 . . c. Not filtered
242. Pancreatic insufficiency is best diagnosed by:
d. Lipophilic
a. Ref: Harrison’s 18th / 325, 326, 327, Harrison 19th p 2000

Hepatolgy
Abnormal Schilling test corrected by pancreatic en ¬

zyme administration
b. Ba meal study
(NBE Pattern 2014-15) .
251 Abnormal excretory function of hepatocytes may be
assessed by: (PCIJune 07)
c. Amylase levels
a. Increased PT
d. Lipase levels
b. Increased ALT
Ref: CMDT 2014, ch. 16, pg. 716, Harrison 19th p 269 c. Increased Alkaline Phosphatase
243. Hyperglycemia occurs after what % of beta cell mass d. Increased gamma GT
is destroyed: (NBE Pattern 2014-15) Ref: Harrison 18th/ p 325, 326, 327, Harrison 19th p 1997
a. 20% b. 40%
c 60% . d. 80% .
252 Which one of the following pairs regarding Hepatitis
Ref: Harrison’s 18th ed. ch 344 . B is not correctly matched (AIIMS Nov 2010)
a. Acute Viral Hepatitis B - Supportive care
.
244 SGPT is found in: (NBE Pattern 2014-15)
b. Acute Viral Hepatitis B - Antiviral therapy
a. Cytoplasm of hepatocytes
b. Mitochondria of hepatocytes c. Chronic Viral Hepatitis B - Supportive care
c .
Nucleus of hepatocytes d. Chronic Viral Hepatitis B - Antiviral therapy
d. All of above " .
Ref: Robbins, 8 ' ed , Table 16.2 . Ref: Harrison 18th/ p 2554, Harrison 19th p 2006
.
245 Best diagnosis of pancreatic disease:
.
253 HBs Ag positive person may have all of the following
(NBE Pattern 2014-15)
associated Renal lesions, Except: ( PGI Dec 06)
a. Ultrasound b. CT scan
d. PTC a. Membranous Glomerulonephritis (MGN)
c. ERCP
. b. Membrano proliferative Glomerulnephritis (MPGN)
Ref: CMDT 2014 , ch. 15, pg 564, Harrison 19th p 2012
c. Mesangiocapillary Glomerulonephritis
246. Microvascular steatosis is seen in all except: d. Focal Segmental Glomerulosclerosis (FSGS)
a. Alcoholic liver disease (NBE Pattern 2014-15)
b. Acute fatty liver of pregnancy Ref: Harrisons 17th / 1796, Harrison 19th p 2010

!
c. Methotrexate toxicity .
254 Chylous ascites is caused by all of the following except:
d. Reye's syndrome Ref: CMDT 2014, ch. 16, pg. 684 a. Colloid carcinoma of stomach (AllMS Nov 02)
247. Serum amylase is raised in: (NBE Pattern 2014-15) b. Tuberculosis.
a. Rubella b. Measles c. Trauma.
c. Mumps d. Chickenpox d. Nephrotic syndrome.
Ref Harrison' s 18 .
th ed ch. 313, Harrison 19th p 2094t
Ref Harrison 18th / p 332 .
Ans. 239. c. Genotyping... 240. d. Hyperamylasia 241. b. Arthritis 242 . a. Abnormal Sch...
243. d. 80% 244. a . Cytoplasm of hepa... 245. b. CT scan 246. c. Methotrexate toxicity
247. c. Mumps 248. a . 3 fold increase in ser... 249. b. Chronic pancreatitis 250. c & d. Not filtered ...
251. c & d. Increased... 252. b. Acute Viral Hepatitis B... 253. d. Focal Segmental Glo...254. a. Colloid carcinoma...
Hepatology

255. A 40 year old patient, a known case of cirrhosis devel ¬ 259. Indications for Liver Transplantation include:
ops acute episode of GI bleed . Initial therapy given for a. Hemochromatosis ( pa 09 )
6 hours. Which of the following procedure is useful : b. Primary Biliary Cirrhosis

E
a . Nasogastric aspiration (AIIMS June 99 ) c. Sclerosing cholangitis with ulcerative colitis
b. Urgent endoscopy d. Sclerosing cholangitis without ulcerative colitis
c. Sedation E. All of the above Ref: Harrison's 17 th/ l 984 .
d. Ultrasound Ref: H 18 th/ p 2598, 2599, 320, 321 .
260. Quantitative assessment of liver function can be done
256. A 45 year old cirrhotic patient presented with severe by: ( PGIDecOS )
haematemesis. The management of choice is : a . Degree of T Transaminases
( AIIMS June 99 ) b. Degree of T Alkaline phosphatase
a . Whole blood transfusion is the best c. Degree of T GGT
b. Colloids are preferred over crystalloids d . Estimation of Galactose Elimination capacity
c. Normal saline infusion Ref: Textbook of Hepatology' by Rodes 3rd / 475).
d. IV fluid with diuretics
261. Which one of the following serum levels would help
Ref: Harrison 18th/ p 321 , 322.
in distinguishing an acute liver disease from chronic
257 . Which of the following is not a precipitating factor for liver disease? ( AI 2005)
hepatic encephalopathy in patients with chronic liver a. Aminotransaminase.
disease? (AIIMS May 05 ) b. Alkaline phosphatase.

Hepatolgy
a . Hypokalemia b. Hyponatremia c. Bilirubin.
c. Hypoxia d . Metabolic acidosis d. Albumin. Ref: Harrisons 16 th / 1815 .
Ref: Harrison 18th/ p 2601 .
258. Lallo, aged 54 years , who is a known diabetic patient
develops cirrhosis. There is associated skin hyperpig¬
mentation and restrictive cardiomyopathy which of
the following is the best initial test to diagnose this
case. (AIIMS Nov 2000)
a . Iron binding capacity
b. Serum ferritin
c. Serum copper
d. Serum ceruloplasmin Ref: Harrison 18th / p 3165.

Ans . 255. b. Urgent endoscopy 256. a . Whole blood trans


...
259. e. All of the above 260. d . Estimation of Gala ...
257. d . Metabolic acidosis 258. a . Iron binding capacity
261 . d . Albumin .
145
Marwah's Internal Medicine MCQs ( Based on Harrison's 19 th
)

ADDITIONAL UPDATES
Important Diagnositc Tests in Common Liver Disease

3
Nonalcoholic steato-hepatitis Ultrasound or CT evidence of fatty liver and compatible histology
Alcoholic liver disease History of excessive alcohol intake and compatible histology
Hepatitis D (delta) HBsAg and anti-HDV
Hemochromatosis Elevated iron saturation and serum ferritin; genetic testing for HFE gene mutations
Wilson’s disease Decreased serum Ceruloplasmin and increased urinary copper; increased hepatic copper level
Hepatocellular cancer Elevated a -fetoprotein level (to > 500 ng/ml); ultrasound or CT image of mass
a1 Antitrypsin disease Reduced a1 antitrypsin levels, phenotype piZZ or PiSZ
Primary Sclerosing cholangitis, P-ANCA, cholangiography
Primary biliary cirrhosis Mitochondrial antibody, elevated IgG levels
Hepatitis E Anti-HEV IgM and HEV RNA
Hepatitis C Anti-HCV and HCV RNA
Auto-immune hepatitis ANA, SMA, elevated IgG Levels

Hepatolgy

146
CHAPTER GIT

6
Treatment Options for Peptic Ulcer Disease

> Active Helicobacter pylori-associated ulcer


1. Treat with anti-H pylori regimen for 10-14 days. Treatment options:
Standard Triple Therapy
• Proton pump inhibitor orally twice daily
Clarithromycin 500 mg orally twice daily
Amoxicillin 1 g orally twice daily
Standard Quadruple Therapy
• Proton pump inhibitor orally twice daily
Bismuth subsalicylate two tablets orally four times daily
Tetracycline 500 mg orally four times daily
Metronidazole 250 mg orally four times daily or 500 mg three times daily
(OR bismuth subcitrate potassium 140 mg/metronidazole 125 mg/tetracycline 125 mg [Pyleral] three capsules orally four times
daily)
Sequential Quadruple Therapy
• Proton pump inhibitor orally twice daily

Days 1-5: amoxicillin 1 g orally twice daily


Days 6-10: clarithromycin 500 mg and metronidazole 500 mg, both orally twice daily
Proton pump inhibitor orally twice daily
Days 1-14: amoxicillin 1 g orally twice daily
Days 7-14: clarithromycin 500 mg and nitroimidazole500 mg, both orally twice daily
2. After completion of course of H pylori eradication therapy, continue treatment with proton pump inhibitoronce daily for 4-6 weeks if
ulcer is large (> 1 cm) or complicated.
3. Confirm successful eradication of H pylori with urea breath test, fecal antigen test, or endoscopy with biopsy at least 4
weeks after completion of antibiotic treatment and 1-2 weeks after proton pump inhibitor treatment.

Most Recent Q’s 2014- 15 2. Gold standard test in the diagnosis of Celiac disease
is: [ NBE Pattern 2014]
.
1 Which of the following is not an auto immune disor -
a .Detection of IgA antiendomysial antibodies
b. Small intestinal biopsy
der? [AllMS May 2015]
a. Ulcerative colitis b. Grave's disease c .Malabsorption studies
c. Rheumatoid arthritis d. SLE d. Upper GI endoscopy

Ref.: Harrison 19th edition 1949 Ref: Harrison 19th p 1940

1. a . Ulcerative colitis 2. b. Small intestinal biopsy


Marwah's Internal Medicine MCQs ( Bases on Harrison's 19th )

Motility 9. Investigation of choice for dysphagia lusoria is?


a. Barium studies [ NBE Pattern 2014 - 151
Which of the following is true regarding GERD? b. X- ray
c. CT angiography

0
[ Manipal 2014]
a. Occurs during transient relaxation of LES d . Esophageal manometry
b. Eradication of H pylori infection reduces acid reflux Ref: Harrison's 18th ed. ch. 292
c. Bicarbonate secreted by esophageal mucosa 10. Triple A syndrome is all except![NBE Pattern 2014-15]
neutralizes the acid a. Alacrymia
d . Normal esophageal mucosa on endoscopy excluded b. Addison disease
GERD c. Achlorhydria
Ref.: H 18th edn. Ch -2428 Harrison 19 th p 1906 d. Achalasia Ref: Harrison 19 th p 2324 t
A lady presented with non progressive dysphagia only 11. Zenker’s diverticulum presents with :
for solids. Barium study showed proximal esophageal a. Dysphonia [NBE Pattern 2014-15]
dilatation with distal constriction . The most likely b. Reflux esophagitis
diagnosis is: [A! 2010] c. Dysphagia
a. Peptic Stricture d. It is found in stomach Ref: Harrison 19th p 1903
b. Carcinoma Esophagus
12 . Pseudoachalasia is seen with all except?
c. Achalasia Cardia
a. Esophageal tumor [NBE Pattern 2014- 15]
d . Lower Esophageal Ring
b. Paraneoplastic
Ref : Harrison's 18/ e p2429, Harrison 19 th p 255 c. Carcinoma fundus
5. Non progressive contraction of Esophagus are : d . Rosary esophagus Ref: Harrison 19 th p 1904
a. Primary [At 2009]

GIT b.
c.
d.
Secondary
Tertiary
Quaternary Ref: Internet, Harrison 19 th p 532
13. Non progressive dysphagia in a lady with a sensation
of something stuck in the throat and worsened by
intake of cold drinks is suggestive of ?
a. Diffuse esophageal spasm [ NBE Pattern 2014 - 15]
6. Patient with pyloric stenosis secondary to peptic b. Upper esophageal web
ulcer, complains of profuse vomiting and Na + - c. Achalasia

125 meq / L, K + > 2.3 meq / L and Cl - ~ > 85 meq / L, BE - .
d Scleroderma
8meq / L should be given: [ARMSJune 2000] Ref: Harrison’s 18th ed. ch. 292, Harrison 19th p 1905
a. Half normal saline 14. True statement about a 6 cm Zenker's diverticulum is:
b. Normal saline a. It is a true diverticulum [NBE Pattern 2014-15 ]
c. K + bolus b. Occurs in the mid oesophagus
d. Hypertonic saline c. Treatment is CP myotomy
Ref: Internet, Harrison 19 th p 1918 d. It occurs in children
7. Gold standard test for achalasia cardia? Ref: Harrison 19 th p 1903
a. Esophageal Manometry [NEET 2015- 16] 15 . Dysphagia lusoria is due to: [ AIIMSNov 03]
b. Barium swallow a. Oesophageal diverticulum
c. Endoscopy b. Aneurysm of aorta
d. Endoscopic ultrasound c. Oesophageal web
Ref: Harrison 18th ch 292, Harrison 19 th p 1904 d. Compression by aberrant blood vessel
8. Most common complication of achalasia is: Ref: Bailey 24th/ 995; 23rd/ 859, Harrison 19 th p 257
-
[ NBE Pattern 2014 15] 16 . Heller's operation is done for: [A11MS Nov 93]
a. Recurrent pulmonary infections a. Achalasia cardia
b. Stricture of esophagus b. Pyloric stenosis
c. Pleurisy c. Peptic ulcer
d . Peptic ulcer .
d CA Esophagus
Ref: CMDT 2014 ch. 15, p. 604, Harrison 19 th p 1904 Ref: Harrison 18th/ p 2432, Harrison 19 th p 1904

Ans. 3. a & c. Occurs... 4. d . Lower Esophageal Ring 5. c. Tertiary .


6 a . Half normal saline
7. a. Esophageal Mano ...
8. a. Recurrent pulmonary... 9. c. CT angiography .
10 c. Achlorhydria
11. c. Dysphagia 12. d . Rosary esophagus .
13. a . Diffuse esophageal.. .
14 c. Treatment is CP myotomy
148
15. d. Compression by... 16 . a. Achalasia cardia
GIT

DIAGNOSISOF PSEUDOMEMBRANOUS COLITIS


1. Stool studies- Pathogenic strains of C difficile produce two toxins: toxin A is an enterotoxin and toxin B is a cytotoxin. Rapid
enzyme
immunoassays (ElAs) for toxins A and B have a 75-90% sensitivity with a single stool specimen; sensitivity increa
se to 90-95% with
two specimens.
2. Until recently, EIA was the preferred diagnostic test in most clinical settings because it is inexpensive, easy to use , and results are
available within 24 hours .
3. However, nucleic acid amplification test (eg , PCR assays ) that amplify the toxin B gene have a 97% sensitivity and thus are
superior to the EIA tests; these PCR assays are now preferred.
4. Alternatively, some laboratories first perform an assay for glutamate dehydrogenase (a common C difficile antigen), which has a high
sensitivity and negative predictive value (> 95%). A negative glutamate dehydrogenase assay effectively excludes infection, while a
positive assay requires confirmation with PCR or EIA to determine whether the strain that is present is toxin producing
> Immediate Treatment for PSEUDOMEMEBRANOUS COLITIS
For patients with three or more relapses, updated 2013 guidelines recommend consideration of an installation of a suspension
of fecal bacteria from a healthy donor (“ fecal microbiota transplant").
In uncontrolled case reports and case series involving several hundred patients, such “fecal transplantation" into the terminal
ileum or proximal colon (by colonoscopy) or into the duodenum and jejunum (by nasoenteric tube) results in disease remission
after a single treatment in over 90% of patients with recurrent C difficile infection.
In a 2013 randomized study, duodenal infusion of donor feces led to resolution of C difficile diarrhea in 94%, which was
dramatically higher than vancomycin treatment (31%), prompting early study termination. Despite uncertainties , fecal
transplantation should be considered in patients with refractory infection. in
¥
If possible, antibiotic therapy should be discontinued and therapy with metronidazole, vancomycin, or fidaxomicin ( a poorly
absorbable macrolide antibiotic) should be initiated. For patients with mild disease, oral metronidazole ( 500 mg orally three
times daily), vancomycin (125 mg orally four times daily), or fidaxomicin, (200 mg orally two times daily) are equally effective for
initial treatment. Vancomycin and fidaxomicin are significantly more expensive than metronidazole.
Therefore, metronidazole remains the preferred first-line therapy in patients with mild disease, except in patients who are
intolerant of metronidazole, pregnant women, and children. The duration of initial therapy is usually 10-14 days. Symptomatic
improvement occurs in most patients within 72 hours.

17 . A male aged 60 years has foul breath; He regurgitates 19. Schatazki’s ring is: [ PCI Dec 98]
food that is eaten 3 days ago. A gurgling sound is often a. Mucosal ring at squamous columnar junction
heard on swallowing : Likely diagnosis is: [Al 2001] b. Muscular ring
a. Zenker’s diverticulum
c. Dysphagia is the presenting symptom
.
b Meckel's diverticulum
d. Inflammatory stricture
c. Scleroderma
d. Achalasia cardia Ref Oxford textbook of Medicine 4th / 553, Harrison 19th p 257
Ref: Dhingra 3 rd / 330, Harrison 19th p 1903
18. All of the following statements about Zenker’s diver ¬
PUD
ticulum are true Except; [AI 2009]
a. Acquired diverticulum 20. Most sensitive test for diagnosis of H. pylori:
b. Lateral X-rays on Barium swallow are often diagnos ¬ a. Rapid urease test
tic b. Demonstration of organism in gastric biopsy
c. False Diverticulum
d. Out pouching of the anterior pharyngeal wall, just
c. Histopathology
d. Endoscopy based biopsy urease test
above the cricopharyngeus muscle
Ref: Dhingra 4th / 355, Harrison 19th p 1903 Ref Harrison's 18th edn. Harrison 19th p 1923

Ans 17 . a. Zenker ’s divert!.. 18. d. Out pouching... 19. a & c. Mucosal ring. . . 20. a. Rapid urease test

- E
Marwah's Internal Medicine MCQs ( Bases on Harrison’s 19th )

CHANGES IN DIAGNOSTIC TESTING FOR CELIAC SPRUE

3
• Serologic tests should be performed in all patients in whom there is a suspicion of celiac disease The
. recommended test is the IgA
tissue transglutaminase (IgA tTG) antibody, which has a 95% sensitivity and 95% specificity for the diagnosis of
celiac disease.
Antigliadin antibodies are not recommended because of their lower sensitivity and specificity. IgA antiendomysial
antibodies are no
longer recommended due to the lack of standardization among laboratories.
A test that measures IgG antibodies to deamidated gliadin has excellent sensitivity and specificity
and is useful in patients with IgA
deficiency and young children.
Levels of all antibodies become undetectable after 3-12 months of dietary gluten withdrawal and may be used to monitor dietary
compliance, especially in patients whose symptoms fail to resolve after institution of a gluten-free diet.

21. A posteriorly perforating ulcer in the pyloric antrum 26. Which drug is not effective against H. pylori ?
of the stomach is most likely to produce initial local ¬ a. Colloidal bismuth [NBE Pattern 2014-15]
ized peritonitis or abscess formation in the follow¬ b. Metronidazole
ing: [AI 2003] c. Amoxycilline
a. Omental bursa [lesser sac] d. Erythromycin
b. Greater sac Ref. CMDT, ch. 15, p. 613, Harrison 19th p 1039
c. Right subphrenic space
27 . All of the following are indications for surgery in a
d. Hepato renal space [ pouch of Morison]
case of duodenal ulcer except: [NBE Pattern 2014-15]
Ref: Gray's 40/ e pi 107 a. Acute perforation of ulcer
M b. Pyloric stenosis
o 22 . Patient presents with recurrent duodenal ulcer of 2.5
cm size; procedure of choice: [AI 2001 ] c. Massive haemorrhage
a. Truncal vagotomy and antrectomy d . Multiple large ulcers
b. Truncal vagotomy and gastrojejunostomy Ref. CMDT, ch. 15, p. 617, Harrison 19th p 1925
c. Highly selective vagotomy 28. True about dumping syndrome is all except :
d . Laparoscopic vagotomy and gastrojejunostomy a. Caused by early emptying of stomach
Ref: Harrison's 18/e p 2452-53, Harrison 19th p 1918 b. Medically managed fNBE Pattern 2014-15]
c. Controlled by small diets
23 . Bleeding from lesser curvature in gastric ulcer, source
d . Needs re -surgery
of bleeding is? [JIPMER 2014]
a. Right gastro-epiploic artery Ref: Harrison's 18th ed. ch. 293, p. 2453, Harrison 19 th p 1926
b. Right omento duodenal 29. A 50 -year - old lady presented with history of pain
c. Pancreatoduodenal artery upper abdomen, nausea and decreased appetite for
d. Left gastric artery 5 days. She had undergone cholecystectomy 2 years
Ref: Schwartz 9th edn p 917- 921, Harrison 19 th p 1925 back. Her bilirubin was 10 mg/ dl , SCOT 900 IU / L,
SGPT 700 IU / L and serum alkaline phosphatase was
24. Eradication of infection by anti - H .Pylori antibiotics is 280 IU / L. What is the most likely diagnosis?
best determined by [NBE Pattern 2014- 15] a. Acute pancreatitis [NBE Pattern 2014 - 15]
a. S. ELISA b. Breath urea test b. Acute cholangitis
c. Rapid urease test d . Biopsy c. Acute viral hepatitis
Ref: Harrison's 118th ed. Figure 151 -2, Harrison 19 th p 1039 d . Posterior penetration of peptic ulcer
25. H. pylori causes: [NBE Pattern 2014-15] .
Ref: CMDT, ch. 16, p 664-671, Harrison 19th p 2004, 2016
a . Type A gastritis 30. Which of the following statements about peptic ulcer
b. Type B gastritis disease is true? [NBE Pattern 2014-15]
c. Autoimmune a. Helicobacter pylori eradication increases the
d. Allergic gastritis likelihood of occurrences of complication
Ref: CMDT, ch. 15, p. 609, Harrison 19 th p 1039 b. The incidence of complication has remained
unchanged

Ans. 21. a . Omental bursa... 22. a. Truncal vagotomy... 23. d. Left gastric artery 24. b. Breath urea test
25. b. Type B gastritis 26. d. Erythromycin 27. d. Multiple large ulcers 28 . d. Needs re-surgery
29 . c . Acute viral hep... 30. b. The incidence of com...
150
GIT

c. The incidence of Helicobacter pylori infection in 38. The most common complication of vagotomy is:
India is very low a. Diarrhoea [NBE Pattern 2014-15 ]
d . Helicobacter pylori eradication does not alter the b. Dryness of mouth
recurrence ratio c. Tachycardia

by:
Ref: Harrison's 18th ed. ch. 293, Harrison 19th p 1911
31. Gastric mucosa in Meckel's diverticulum is diagnosed

a. Endoscopy
b. Occult blood in stool
c. Technetium isotope scan
d. Barium studies
[NBE Pattern 2014- 15]
39.
d. Bleaching

a. Incidence has decreased


b. Incidence has increased
c. Associated with smoking
d. Common in men
Ref: CMDT, ch. 34, p. 1615
Duodenal ulcer statistics all true except:
[NBE Pattern 2014- 15]

Ref: CMDT, ch. 15, p. 611, Harrison 19 th p 1918


E
Ref: Harrison’s 18th ed. ch. 41, p. 323, Harrison 19 th p 277, 279
40. MC age of presentation of gastric ulcer is:
32. Which indicates least chances of re-bleeding after [NBE Pattern 2014- 15]
hematemesis episode: [NBE Pattern 2014-15 ] a. 3rd decade
a. Adherent clot on ulcer b. 4lh decade
b. Clean based ulcer c. S"1 decade
c. Gastric ulcer with AV malformation d. 6th decade
d. Visible bleeding vessel Ref: Harrison 19 th p 276 Ref: Harrison's ch. 239, Harrison 19 th p 1911
33. Most common site of dieulafoys lesion is: [Newpattern] 41. Which of the following have hypergastrinemia with
a. Lesser curvature of stomach
b. Greater curvature of stomach
decrease acid output? -
[NBE Pattern 2014 15 ] I
a. Peptic ulcer disease b. ZES
c. Pylorus c. G -cell hyperplasia d. Pernicious anemia
d. Antrum

34.
Ref: CMDT, ch. 15, p. 581, Harrison 19 th p 277, 1891
Not a hormone causing early dumping?
a . VIP [NBE Pattern 2014- 15]
42 .
a.
Ref: CMDT, ch. 15, p. 610, Harrison 19 th p 568, 1927
Dumping syndrome is due to all except:
Motilin
b. Small stomach
[NBE Pattern 2014- 15] j GIT
b. Neurotensin
c. Hypertonic fluid contents in bowel
c. Motilin d. CCK
d. Neurotensin
Ref: Harrison's 18th ed. ch. 239, Harrison 19 th p 1926
Ref: Harrison's 18th ed. ch. 239, Harrison 19th p 1926
35 . Most common site of peptic ulcer:
a. l!t part of duodenum [NBE Pattern 2014-15] 43. Peptic ulcer is associated with all except one:
b. 2 nd part of duodenum a. Cirrhosis [ A11MS Feb 1997]
c. Antrum b. Zollinger Ellisons syndrome
d. Terminal ileum c. Primary hyperparathyroidism
Ref: CMDT, ch. 15, p. 611, Harrison 19 th p 1911 d. Pernicious anemia
Ref : S. Das 3rd / 818, Harrison 19 th p 1920
36 . Most common site of type 1 gastric ulcer:
a. Gastric body [NBE Pattern 2014- 15] 44. Common sites for Cushing ulcers include all of the fol ¬
b. Antrum lowing except : [A11999]
c. Pylorus d. Cardia a. Esophagus
Ref: CMDT, ch. 15, p. 611, Harrison 19 th p 1911 b. Stomach
37 . Disabling mid- epigastric pain within 10 minutes of c. 1st part of duodenum
eating food with history of weight loss. Past history d. Distal duodenum
is positive for Myocardial infarction in last year indi ¬ Ref: Harrison 18th/ p 2457 , Harrison 19 th p 1918
cates. [ New pattern] 45. Commonest site of peptic ulcer is : [A11999]
a. Gastric ulcer
b. Duodenal ulcer
a. 1st part of Duodenum
c. Abdominal angina b llnd part of duodenum
d. Acute cholecytitis with stone impaction c. Distal 1/ 3 of stomach
d.
.
Ref: Harrison's, lff ^ ed., ch 160, p. 1019
Pylorus of the stomach
Ref: Robbins 7th/ 817, Harrison 19 th p 1918
Ans . 31. c. Technetium isot... 32. b. Clean based ulcer 33. a. Lesser curvature... 34 . d. CCK
35. a . 1st part of duode... 36. a . Gastric body 37. c. Abdominal angina 38. a . Diarrhoea
39. a . Incidence has... 40 . d. 6th decade 41 . d. Pernicious anemia 42. b. Small stomach
43. d. Pernicious anemia 44. d. Distal duodenum 45. a . 1st part of Duodenum
Marwah’s Internal Medicine MCQs ( Bases on Harrison 's 19th )

46. Artery to bleed in duodenal ulcer haemorrhage: 53. Not true in type A fundal gastritis is:
a . Splenic artery [ PG1 Dec 2000] a. Low gastric PH [NBE Pattern 2014-15]
b. Gastroduodenal artery b. Hyperchlorhydria
c. Left gastric artery c. Antibody against parietal cells and presence of
d . Superior mesenteric artery autoimmunity
||
j Ref: Bailey 24th / 1026; CSDT llth/ 550, Harrison 19 th p 1925 d . Antibody against intrinsic factor
I 47. Dumping syndrome is due to: [ AI 1999] Ref CMDT, ch. 15, p. 607, Harrison 19 th p 1931
a. Diarrhoea 54. H . pylori is known to cause all of the following except .-
b. Presence of hypertonic content in small intestine a. Gastric ulcer [AI 1999]
c. Vagotomy b. Duodenal ulcer
d . Reduced gastric capacity c. Gastric lymphoma
Ref: Love & Bailey 23rd / 913, Harrison 19 th p 1926 d . Fundal atrophic gastritis
Ref Harrison 18th/ p 1262, Harrison 19 th p 1923
Gastritis
55 . A patient with H. Pylori infection is treated with
drugs. The best method to detect presence of residual
48. Prolonged intake of PPI does not cause
H .Pylori infection in this person is : [AI 2007]
a. Hypothyroidism [ AllMS Nov. 14]
b. Pelvic fracture
a. Rapid urease test
b. Urea breath test
c. Clostridium difficile infection
c. Endoscopy and biopsy
d . Increased community acquired pneumonia
d . Serum anti H.Pylori titre
Ref: Harrison 18th edn, ch 239, Harrison 19 th p 263/ 568
Ref Harrison 17th/ 947, Harrison 19 th p 1923
I 49 . Helicobacter pylori is associated with following ex¬
cept: [NBE Pattern 2014-15] Malignancy
a. Type A gastritis
b. M.A.L.Toma 56. Most important prognostic factor for colorectal carci ¬
c. Gastric adenocarcinoma noma is: [AI 2009 ]
d . Hyperchlorhydria a. Site of lesion b. Stage of lesion
Ref: CMDT, ch. 15, p. 609, Harrison 19 th p 1039 c. Age of patient d. Lymph node status
Ref Harrison's 18/ e p771, Harrison 19 th p 541
50. Phlegmonous gastritis occurs due to:
a. H. pylori [NBEPattern 2014-15] 57. Based on Epidemiological studies, which of the fol ¬
b. E.coli lowing has been found to be most protective against
c. Drugs Carcinoma Colon: [AI 2009]
d. Reflux of acid a. High fiber diet b. Low fat diet
Ref CMDT, ch. 15, p. 610, Harrison 19 th p 1930 c. Low selenium diet d. Low protein diet
Ref: Internet, Harrison 19 th p 477
51. Most common viral cause of gastritis
a. H. Pylori [ NBE Pattern 2014-15] 58. True about esophageal adenocarcinoma:
b. CMV [ PGI May 2013]
c. Hepatitis A a. Majority of cases arise in Barrett’s oesophagus
d . Enterovirus b. Common in upper part of esophagus
Ref CMDT, ch. 15, p. 610, Harrison 19 th p 1930 c. Commonly arise in the distal esophagus
d. Tobacco exposure and obesity are risk factors
52. Erosive gastritis commonly occurs at: e. Incidence is increasing
a. Cardia [NBE Pattern 2014-15] Ref Harrison’s 18/ e p764-765, Harrison 19 th p 532, 533
b. Fundus
59. Most common site of MALT? [AIIMS Nov 2014]
c. Greater curvature
d . Antrum
a. Stomach b. Ileum
c. Duodenum d . jejunum
.
Ref CMDT, ch 15, p. 607, Harrison 19th p 1930
Ref Harrison 18* edn, ch 110, Harrison 19th p 703

Ans. 46 . b. Gastroduode ... 47 . b . Presence of hypertonic . .. 48 . a . Hypothyroidism 49. a . Type A gastritis


50 . b. E.coli 51 . b. CMV 52. c . Greater curvature 53. b. Hyperchlorhydria
54. d . Fundal atrophic ... 55. b. Urea breath test 56 . b . Stage of lesion 57 . a . High fiber diet
58 . a , c , d , e. Majority... 59 . a . Stomach
GIT

What is wrong about GIST ? [AIIMS Nov 2014 ] 68. Best substitute of esophagus after esophagectomy is:
60.
a. Originate from Cajal cells a. Stomach [AI 1996]
b. Common mesenchymal tumor of GIT b. Jejunum
c. Prognosis depends on size c. Left colon
d. Associated with Aik mutation
Ref: Harrison Iff'' edn, ch 91: Harrison 19 th p 119 e- 1
61. 50 year old man became dizzy while passing stool and
noticed fresh blood in stool. Previous stool examina ¬
tion for routine screening of carcinoma colon was
normal. What is the most likely cause of bleed?
a . Early carcinoma colon [AIIMS Nov. 2014]
69 .
d. Right colon
Ref: ch 292, Harrison 18th edn, Harrison 19 th p 533
Most common site for carcinoma stomach is:
a. Antrum
b. Fundus
c. Lesser curvature
d. Greater curvature
[AIIMS June 93] E
b. Sigmoid diverticulitis Ref CMPT 11 th/ 557, Harrison 19th p 534
c. Microscopic colitis
d. Dilatation of veins of colon 70. The best prognosis in carcinoma stomach is with:
Ref Harrison's 18* edn, Ch 41 and 297, Harrison 19th p 1971 a. Superficial spreading type [ AI 1995]
b. Ulcerative type
.
62 Not a pre-malignant lesion? [ AIIMS Nov. 2014]
c. Linitis plastica type
a. Peutz Jeghers b. Crohns disease
d . Polyp
c. Ulcerative colitis d. Barret esophagus
Ref Harrison's 18th, Harrison 19th p 536 Ref: Bailey 23rd / 920, 921, Harrison 19 th p 534 \
63. All are true about Plummer-Vinson syndrome except: 71. Amongst the following, the most common site for I
a. Esophageal webs [ NBE Pattern 2014 15] - Leiomyoma is [AI 1994] l
b. Premalignant a. Stomach b. Small Intestine
c. c. Duodenum d. Colon

GIT
Common in elderly male
d. Dysphagia Ref: Bailey 21st / 892; Harrison 19 th p 536
.
Ref CMDT, ch 15, p. 600, Harrison 19 th p 237/ 532 72. Strong correlation with colorectal cancer is seen in:
64. The most prevalent esophageal cancer world wide is: a. Peutz-Jegher’s polyp [AI 2003]
a. Squamous cell ca [A11991] b. Familial polyposis coli.
b. Adenocarcinoma c. Juvenile polyposis
c. Sarcoma d. Hyperplastic polyp.
d. Adenoid cystic carcinoma Ref: Harrison 18th/ p 768, 769, 770, Harrison 19 th p 541
Ref Harrison 19 th p 532
73. In Peutz- Jeghers syndrome, polyps are seen in
65. Most common site of carcinoma esophagus is: a. Colon [AI 1995]
a. Middle l / 3 rd [AIIMS Feb 97] b. Rectum
b. Upper l /3 rd c. Small bowel
c. Lower l / 3 rd d. Stomach
d. Lower end of esophagus
Ref: Harrison 18th/ p 768, 769, 770, Harrison 19 th p 538t
Ref: Harrison 16 th/ 524, Harrison 19 th p 532
74. Most important prognostic factor for colorectal
66 . Hyperkeratosis of palm and sole is seen in:
carcinoma is : [AI 2009]
a. Carcinoma colon [AIIMS Dec 97]
b. Hepatoma
a. Site of lesion b. Stage of lesion
c. Age of patient d . Lymph node status
c. Adenocarcinoma lung
d. Ca oesophagus Ref Robbins 7th / 866; Harrison 19 th p 541
Ref: Harrison 18th / p 764, Harrison 19th p 532 75. All of the following genes may be involved in develop ¬
67. Adenocarcinoma of esophagus is commonly found in ment of carcinoma of colon Except: [AI 2009]
a. Achlasia acardia [AI 1998] a. APC
b. Barret's oesophagus b. Beta - Catenin
c. Plummer vinson syndrome c. K- ras
d. Chronic smoking d. Mismatch Repair Genes
Ref: Harrison 17th/ 1852;, Harrison 19 th p 532 Ref: Harrison 19 th p 541

Ans. 60. d. Associated... 61. b. Sigmoid diverticulitis 62. a. Peutz Jeghers 63. c. Common in elderly male
64. a . Squamous cell ca 65. c. Lower 1/3rd 66. d . Ca oesophagus 67. b. Barret’s oesophagus
68. a . Stomach 69. a. Antrum 70. a. Superficial spread ... 71. a . Stomach
72. b. Familial poly... 73. c. Small bowel 74. b . Stage of lesion 75. None
153
Marwah's Internal Medicine MCQs ( Bases on Harrison's 19th )

76. Massive bleeding per rectum in a 70 yr old patient is due flares. What should be the next line of treatment for
to : [A12010] him? [NBE Pattern 2014- 15]
a. Diverticulosis a. Methotrexate b. Azathioprine
b. Carcinoma colon c. Cyclosporine d . Cyclophosphamide

1 77 .
c. Colitis
d. Polyps
Ref: Ch 297, Harrison 18th edn, Harrison 19th p 1903

IBD

Extraintestinal manifestations of Inflammatory bow¬


el disease include all of the following, except:
83.
a. Sigmoid colon
b. Transverse colon
c. Ileum
d. Rectum
Ref: CMDT, ch. 15, p. 641
Invariably involved site in ulcerative colitis:
[NBE Pattern 2014-15]

Ref: CMDT, ch. 15, p. 646, Harrison 19th p 1952


a . Uveitis [ PCI June 05] 84. Treatment of choice in intractable ulcerative colitis:
b. Sclerosing cholangitis [ NBE Pattern 2014 15] -
c. Osteoarthritis a. Mucosal proctectomy + Ileoanal pouch anastomosis
d. Skin nodules b. Proctectomy
Ref: HI 8/ e p 2487-88, Harrison 19th p 1961 c. Colectomy with ileostomy
d. Ileorectal anastomosis
I 78. DOC of acute exacerbation of ulcerative colitis? Ref: CMDT, ch. 15, p. 649, Harrison 19th p 1962
a. Sulfasalazine [NEET 2015-16]
b. Steroids 85. Treatment of choice in ulcerative colitis is:
c. Infliximab a. 5 aminosalicylic acid [NBE Pattern 2014-15]
d . Cyclosporine b. Azathioprine
c. Metronidazole
E* Ref: Harrison' s 18th edn, ch 295: Harrison 19th p 1952
d . Salicylates Ref Harrison 19th p 1962
79. A neonate has been diagnosed with necrotizing en ¬
86. Best treatment of refractory peri-anal fistula in crohn's
terocolitis with X ray abdomen showing gas in the
disease: [NBE Pattern 2014-15]
portal vein. The correct staging of the patient is ?
a. Fistulectomy b. Infliximab
a. Stage 1 [NBE Pattern 2014-15 ]
c. Olasalazine d . Mesalamine
b. Stage 2 A
Harrison 19th p 1963
c. Stage 2 B
d. Stage 3 87. Which of the following is the established biological
Ref: Ch. 102.2 Nelson’s 18th ed., Harrison 19th p 99 therapy for Crohn's disease? [NBE Pattern 2014-15]
80. All are complications of ulcerative colitis, except: a. Anti TNF a-antibody b. IL-1 antagonist
a . Haemorrhage [ NBE Pattern 2014-15] c. IL -6 antagonist d. IL-8 antagonist
b. Stricture Ref: Harrison' s 18th ed. ch. 295, Harrison 19th p 1963
c. Malignant change 88. All are true about ulcerative colitis except:
d . Fistula a. Smoking may prevent the disease
Ref CMDT, ch. 15, p. 646, Harrison 19th p 1953 b. 1:1 male female ratio [NBE Pattern 2014-15]
81. Best screening test for Crohn's disease is: c. Presents with bloody diarrhea
a. A.S.C.A [NBE Pattern 2014-15] d. Highly Associated with infertility
b. P-ANCA Ref: CMDT, ch. 15, p. 646, Harrison 19th p 1951
c. Fecal alpha 1 anti-trypsin 89. A highly sensitive and specific marker for detecting
d. Fecal calprotectin intestinal inflammation in ulcerative colitis is?
Ref: CMDT, ch. 15, p. 642, Harrison 19th pi 954 a. CRP [ APPG 2014]
82. A 41- year -old male patient presented with recurrent b. Fecal lactoferrin
episodes of bloody diarrhea for 5 years. Despite reg¬ c. Fecal calprotectin
ular treatment with adequate doses of sulfasalazine, d. Leukocytosis
he has had several exacerbations of his disease and Ref: CMDT, ch. 15, p. 642, Harrison 19th p 1954
required several weeks of steroids for the control of

Ans . 76 . a . Diverticulosis 77 . c . Osteoarthritis 78 . b. Steroids 79. c . Stage 2 B


80 . d . Fistula 81 . a . A.S. C .A 82. b. Azathioprine 83. d . Rectum
84. a . Mucosal proctect... 85. a . 5 aminosalicylic acid 86 . b. Infliximab 87 . a . Anti TNF a -antibody
88 . d . Highly Associ . .. 89 . b. Fecal lactoferrin
GIT

colitis associated features include all , ex- 97 . The histological features of coeliac disease include all
90 . Ulcterative
cepl: [NBE Pattern 2014-15] of the following, except: [Al 2002 ]
a . Iritis b Arthritis a. Crypt hyperplasia
b. Increase in thickness of the mucosa

E
c. Urethritis d . Pyoderma
Ref: CMDT, ch. 15, p. 646, Harrison 19 th pi 952 c. Increase in intraepithelial lymphocytes
d. Increase in inflammatory cells in lamina propyria
91. Chronic inflammatory bowel disease is associated with: Ref Harrison's 18/ e p2470, Harrison 19 th p 1940
[ NBE Pattern 2014 15] -
a. Chronic hepatitis Proved association of celiac sprue is with:
b. Fibrosis a. Dermatitis herpetiformis [ PCI Dec 2000]
c. Cholangiosarcoma b. Scleroderma
d. Primary sclerosing cholangitis c. Pemphigus
Ref: CMDT, ch. 16, p. 709 d. Pemphoid
92 . Which of the finding is not a usual feature of Crohn’s Ref: H 18/ e p2471, Harrison 19 th p 1897 f
disease? [NBE Pattern 2014 15] - 99. Which of the following parastitic infestation can lead
a . Granulomas b. Pseudopolyps to malabsorption syndrome? [Al 06]
c. Skip lesion d . Right colon predominance a . Amoebiasis
Ref : CMDT, ch. 15, p. 641, Harrison 19 th p 1954 b. Ascariasis
93. Skip granulomatous lesions are seen in : [Al 1996 ] c. Hookworm infestation
a. Ulcerative colitis b. Crohn's disease d . Giardiasis Ref: Harrison 19 th p 1946 f
c. Whipple's disease d. Reiter's disease 100. Crohn’ s disease may be caused by which one of the
Ref Harrison's 18th Ch 295, Harrison 19 th p 1953 following infectious agents: [Al 2008]
94. Crohn's disease may be caused by which one of the
following infectious agents:
a . Clostridium difficle
b. Mycobacterium paratuberculosis
[Al 2008]
a. Clostridium difficle
b. Mycobacterium paratuberculosis
c. Cytomegalo virus [CMV]
d . Mycoplasma
GIT
c. Cytomegalo virus [ CMV] Ref: H 18/ e p2480-82, Harrison 19 th p 1953
d . Mycoplasma
101. Which of the following organism does not cause
Ref Harrison 18th / p 2480, Harrison 19 th p 1953
invasive diarrhea: [ DNB June 2009 ]
a. Bacillus cereus
Malabsorption b. Aeromonas sp
c. Rota virus
d. Shigella
95. A 41 year old patient presented with chronic diar ¬

rhoea for 3 months. A d -xylose absorption test was Ref: H 18/ e p311 , Harrison 19 thp 268
ordered to look for: [ARMS Nov 02] 102. Celiac disease false is: [ NEETPattern Question ]
a. Carbohydrate malabsorption due to mucosal disease a. Immunological factor is present in many cases
b. Carbohydrate malabsorption due to chronic b. Does not predispose to malignancy
pancreatitis c. Anti endomysial antibody most sensitive
c. Fat malabsorption due to mucosal disease d. Intestinal biopsy is to be done to confirm diagnosis
d. Fat malabsorption due to chronic pancreatitis
Ref: Harrison' s 18/ e p2471, Harrison 19 th p 1940
Ref Harrison' s 18/ e p2465, Harrison 19 th p 1938
103. Malabsorption syndrome features include all , except:
96. Which of the following statements about Schilling's
test are true: [ PCI 2009 ]
a. Anaemial
[NBE Pattern 2014 15 ] -
a . Abnormal in pernicious anemia
b. Normal in bacterial overgrowth syndrome b. Constipation
c. Abnormal in ileal disease c. Tetany
d. Normal in chronic pancreatitis d. Steatorrhoea
Ref: H 18/ e p2467, Harrison 19 th p 1941 t Ref: CMDT, ch. 15, p. 621, Harrison 19 th p 1946

ns . 90 . c . Urethritis 91 . d . Primary scleros... 92. b. Pseudopolyps 93 . b. Crohn ’s disease


94 . a : Clostridium difficle 95 . a . Carbohydrate malabsor... 96 . a . Abnormal in per. .. 97 . b. Increase in thickness . . .
1n, ' ®
° ermatltls her
U <! b D es not pre ...
- 99 . d . Giardiasis 100. a . Clostridium difficle 101 . a . Bacillus cereus
° 103 . b. Constipation
Marwah's Internal Medicine MCQs ( Bases on Harrison 's 19 th )

104. Short bowel syndrome after extensive resection of 111. Alpha 1 anti tryspin in stool is indicative of ?
-
intestine is mostly due to: [NBE Pattern 2014- 15]

a
a. Protein losing enteropathy [ NBE Pattern 2014 15 ]
a. Mesenteric artery occlusion b. Chronic pancreatitis
-
b. Midgut volvulus c. Acute pancreatitis
c. Extensive Crohn's disease d. Whipple disease
d. Inferior mesenteric vein occlusion Ref: CMDT, ch. 15, p. 632, Harrison 19th p 1945
Ref: CMDT, ch. 15, p. 625, Harrison 19th p 1943
112. Jejunal biopsy is diagnostic in: [NBE Pattern 2014- 15]
105 . A patient with chronic diarrhoea with normal D -xy- a. Coeliac sprue
lose & Schilling test. What could be the diagnosis: b. Tropical sprue
a. Chronic pancreatitis [NBE Pattern 2014-15] c. Whipple's disease
b. Bacterial overgrowth syndrome d. Radiation enteritis
c. Coeliac disease Ref: CMDT, ch. 15, p. 623, Harrison 19th p 1091, 1944
d. Gastric disease
Ref: CMDT, ch. 15, p. 624, Harrison 19th p 1944
- -
113. A 25 year old farmer presented with history of high
grade fever for 7 days and altered sensorium for 2
106. Anti - T.T. G antibodies are seen in: days. On examination , he was comatosed and had con ¬
a. Giardia malabsorption [NBE Pattern 2014-15] junctival hemorrhage. Urgent investigations showed
b. Gluten enteropathy a hemoglobin of 11 gm / dl, serum bilirubin 8 mg / dl
c. Lactose deficient and urea 78 mg / dl. Peripheral blood smear was nega ¬
d. Bile acid malabsorption tive for malarial parasite. What is the most likely di ¬
Ref: CMDT, ch. 15, p. 622, Harrison 19 th p 1940 agnosis? [NBE Pattern 2014-15]
a . Brucellosis
107. A 41- year - old patient presented with chronic dirar -
b. Weil 's disease
H rhea for 3 months. A d-xylose absorption test was or ¬
c. Acute viral hepatitis
3 dered to look for: [NBE Pattern 2014 15 ] - d. Q fever
a. Carbohydrate malabsorption due to mucosal disease
b. Carbohydrate malabsorption due to chronic Ref CMDT, ch. 34, p. 1477, Harrison 19th p 1140
pancreatitis 114. A 25-year- old woman presents with recurrent abdom ¬
c. Fat malabsorption due to mucosal disease inal pain and anemia. Peripheral blood smear shows
d. Fat malabsorption due to chronic pancreatitis basophilic stippling of the red blood cells. What is the
Ref: Harrison's 18th ed. ch. 46, p. 257, Harrison 19th p 1938 most likely diagnosis? [NBE Pattern 2014-15]
a. Coeliac disease b. Hookworm infestation
108. Xylose excretion test is used to assess: c. Sickle cell disease d. Lead poisoning
a . Insulinoma [ NBE Pattern 2014-15]
Ref CMDT, ch. 38, p. 1582, Harrison 19th p 2607, 472e -2t
b. Atypical carcinoid
c. ZES 115. A patient presents with lower gastrointestinal bleed .
d . Monosaccharide absorption Sigmoidoscopy shows ulcers in the sigmoid. Biopsy
Ref: HI 8th ed. ch. 294, p. 2467, Harrison 19th p 1938 from this area shows flask-shaped ulcers. Which of
the following is the most appropriate treatment:
109. Patient with congenita l lactose deficiency will experi ¬
a. Intravenous ceftriaxone [ NBE Pattern 2014-15]
ence distension , flatulence and diarrhea on ingestion b. Intravenous metronidazole
of . [NBE Pattern 2014-15]
b. Sucrose c. Intravenous steroids and sulphasalazine
a. Glucose
d. Eggs: d. Hydrocortisone enemas
c. Milk
Ref CMDT, ch. 15, p. 626, Harrison 19th p 272
.
Ref CMDT, ch 35, p. 1504, Harrison 19th p 1366
ized by all ex 116. True about tropical sprue are A / E:
110. Protein losing enteropath y is character ¬

cept [NBE Pattern 2014-15] a. Protein losing enteropathy [NBE Pattern 2014- 15]
a. Decreased serum albumin and globulin b. Steatorrhea
b . Increased lymphatic flow c. Stomatitis
c. 99 mTc-dextran radionuclide study done d. Jejunal biopsy is specific
d. Lymphangiectasia on biopsy Ref: Ch 294, Harrison's 18th edn., Harrison 19th p 1942
Ref. CMDT, ch. 15, p. 631, Harrison 19th p 1945

Ans. 104. a. Mesenteric artery...105. b. Bacterial over... 106. b. Gluten enteropathy .


107. a Carbohydrate mala...
1 108.
112.
.
d . Monosaccharide. .109. c. Milk
c. Whipple’s disease 113. b. Weil ’s disease
110. b. Increased lympha ...
114. d . Lead poisoning
. .
111 a Protein losing enteropathy
115. b. Intravenous metronidazole
156 116. d . Jejunal biopsy is specific
GIT

117. A 30 - year- old lady presents with features of malab ¬ 124. Positive D - xylose test indicates all of the following,
sorption and iron deficiency anaemia . Duodenal bi ¬ Except: [A11992]
opsy shows complete villous atrophy. Probable diag ¬ a. Pancreatic insufficiency
nosis is: [NBE Pattern 2014 15] - b. Small intestinal mucosal disease
a. Antiendomysial antibodies c. Impaired carbohydrate absorption in small intestine
b. Anti -goblet cell antibodies d . Malabsorption
c. Anti-Saccharomyces cerevisae antibodies Ref: Harrison 18th/ p 2467, 2470, Harrison 19 th p 1938
d. Antineutrophil cytoplasmic antibodies
125. Best test for Small intestine malabsorption of carbo¬
Ref: Harrison’s 18th ed. ch. 294
hydrates is : [A11997] 1
118. Jejunal biopsy is diagnostic in: [NBE Pattern 2014 15] - a. Lund meal test b. Shilling test
a. Abetalipoproteinemia c. D-Xylose test d . Follacin test
b. Giardiasis Ref Harrison 18th/ 2467, Harrison 19 th p 1946 t
c. Tropical sprue
d. Celiac sprue 126. 30 year male with chronic diarrhoea, anemia, raised
Ref: Harrison's 18th ed. part-2, p. 314, Harrison 19th p 2444 liver enzymes. Most likely associated with:
a. Antimitochondrial antibody [A1IMS May 07]
119. Most common cause of malabsorption in our country b. Anti-endomysial antibody
is:
a. Intestinal surgery
[NBE Pattern 2014 15] - c. Anti -smooth muscle antibody
d . Antinuclear antibody
b. Gastric surgery Ref Harrison 18th/ p 2470, 2471
c. Sprue
d. Intestinal parasite 127. In which of the following conditions of malabsorption, j
.
Ref CMDT, ch 15, p. 621, Harrison 19th p 1946 t
an intestinal biopsy is diagnostic : -
[ AIIMS May 05] I
a. Celiac disease b. Tropical sprue
120. In celiac sprue there is a deficiency of all except: 1
a. Vitamin A
b. Vitamin B 12
c. Folic acid
[NBE Pattern 2014 15] -
c. Whipple's disease d . Lactose intolerance
Ref Harrison 18th/ 2470, Harrison 19 th p 1946 t
128. Most common CNS manifestation of Whipple's disease
GIT
d. Iron is : [ A11999]
a. Cerebellar ataxia
Ref: CMDT, ch. 15, p. 621, Harrison 19 th p 1897 f/ 1940
b. Supranuclear ophthalmoplegia
121. Not included in armamentarium of tests for malab¬ c. Seizure
sorption syndrome [A11MS MAY 2015] d. Dementia
a. D- Xylose Ref: H 18th/ p 2474, Harrison 19 th p 1093
b. 14 C Triolein breath test
c. 13 C Trioctanoin breath test 129. Which of the following vitamin deficiencies is most
d. 13 C Triclosan breath test commonly seen in short bowel syndrome: [ Al 2012]
a. Vitamin BI 2 b. Biotin
Ref Harrison 19th p 1946 t
c. Vitamin Bt d. Vitamin K
122 . Malabsorption syndrome does not result from : Ref Harrison 19 th p / 362, 1943
a. Parasite infestation
b. Small bowel diverticulae
[NBE Pattern 2014 15] -
GERD
c. Post-gastrectomy
d. Anterior resection of colon 130. All of the following are true about Barret’ s esophagus,
Ref: CMDT, ch. 15, p. 621, Harrison 19 th p 1946 t except: [ PCI June 03]
123. Non -tropical sprue is characterized by: a. Consequence of prolonged GER
a. Elongation of intestinal villi [NBE Pattern 2014 15] b. It is premalignant
b. Currant jelly stools
- c. Lower oesophageal mucosa is replaced by intestinal
c. Hypertriglyceridemia type of epithelium
d. Poor absorption of lipids d. Associated with oesophageal varices
Ref CMDT, ch. 15, p. 621, Harrison 19 th p 1940 e. Predisposes to adenocarcinoma
Ref Harrison's 18/ e p2434, Harrison 19th p 1907

Ans . 117 . a . Antiendomysial ... 118 . a . Abetalipoproteinemia 119 . d . Intestinal parasite 120 . b. Vitamin B12
121 . d . 13 C Triclosan . . . 122. d . Anterior resection . .. 123. d . Poor absorption . . . 124. a . Pancreatic insufficiency
125. c . D -Xylose test b. Anti -endomysial anti . . . 127 .
126 . c . Whipple’s disease 128. d. Dementia
129 . a . Vitamin B 130 . d . Associated with oesophageal
12 varices
th )
Marwah's Internal Medicine MCQs ( Bas es on Har riso n's 19

131. A 70- year- old male patient presented to the emer ¬ 138. Barret's esophagus is diagnosed by: [ AIIMS Nov 06 ]
genc y depa rtme nt with pain in epig astri um and dif ¬ a. Squamous metaplasia
ficulty in breathing for 6 hours. On examination, his b. Intestinal metaplasia
heart rate was 56 per minute and the blood pressure c. Squamous dysplasia
was 106 /60 mm Hg. Chest examination was normal. d. Intestinal dysplasia
1
f
^^
• j
The patient has been taking omeprazole for gastro-
esophageal reflux disease for last 6 months. What
should be the initial investigation:
a. An ECG [NBE Pattern 2014-15]
Ref: Harrison 18th / p2434, Harrison 19th p 1907
139. Barret's esophagus can lead to:
a. Stricture
[ AIIMS June 98]
b. Reflux esophagitis
c. Peptic ulcer d. Chalasia
b. An upper GI endoscopy
c. Urgent ultrasound of the abdomen Ref: Manual ofGastro. 4th/132, Harrison 19th p 1907
d. An X-ray chest 140. What is true regarding Barret's esophagus:
Ref: Harrison's 18th ed. ch. 243 a. Seen in females [AIIMS Nov 94]
b. Premalignant condition
132. The most common cause of drug induced esophagitis c. Responds to conservative management
is [NBE Pattern 2014-15]
d. Squamous metaplasia is seen
a. Metronidazole
b. Indomethacin Ref: Harrison 18th / p 2434, Harrison 19th p 1907
c. Doxycyline Bowel Obstruction
d. Steroids
Ref: CMDT, ch. 15, p. 598, Harrison 19th p 1910 141. A man aged 60 yrs has h/ o IHD and atherosclerosis.
133. Which is true regarding Barrett’s esophagus? He presents with abdominal pain and maroon stools:
a. Squamous metaplasia of lower esophagus likely diagnosis here is: [AI 2001]

GIT b. Seen mainly in females


c. Premalignant
[NBE Pattern 2014-15]

d. Responds to conservative management


a. Acute intestinal obstruction
b. Acute mesenteric ischemia
c. Peritonitis
d. Appendicitis
Ref: CMDT, ch. 15, p. 594, Harrison 19th p 1907
134. Endoscopic mucosal resection in Barrett's esophagus
Ref: Harrison's 18/ e p2511, Harrison 19th p 1978
results in: [NBE Pattern 2014-15] 142. True about acute pancreatitis:
a. Stricture esophagus b. Peptic ulceration a. Amylase has prognostic value
c. Reflux esophagitis d. Achalasia cardia b. CT is the best imaging study for initial evaluation
CMD T, ch. 15, p. 595, Harr ison 19th p 1907 c. Nasojejunal feeding is better than total parenteral
Ref:
nutrition
135 . Whi ch of the follo wing stag ing is used for GER D?
d. ERCP is better than CT
a. Ranson [NBE Pattern 2014-15]
e. Ultrasound confirms the diagnosis in most cases
b. Gleason ' 18 e p2640, 2637, Harr ison 19th p 1899 -1900
c. Savary miller
Ref: Harr ison s /
d. Hunter scale 143. Most common cause of hematochezia in children?
Grad ing and stag ing in gast roen tero logy by tytgat p. 108 a. Rectal polyp [NBE Pattern 2014-15]
Ref :
b. Meckels diverticulum
136 Reflux esophagitis is defined as pH of esophagus to be
. c. Necrotizing enterocolitis
less than: [NBE Pattern 2014-15]
d. Acute gastritis
a. 1 b. 2
c . 3 .
d 4 Ref: Table 303.12 Nelson's 18th ed., Harrison 19th p 276, 1899
Ref: Grading & stag ing in gastr o. by tytga t p. 108, HI 9 th p 1894 144. In a case of hypertrophic pyloric stenosis, the meta ¬

bolic disturbance is [ AI 2002 ]


137. All of the following are correct statements regarding a. Respiratory alkalosis
reflux esophagitis, except: [NBE Pattern 2014-15]
b. Metabolic acidosis
a. Water brash b. Weight gain
c. Metabolic alkalosis with paradoxical aciduria
c. Mediastinitis d. Infant apnea
d. Metabolic alkalosis with alkaline urine
: CMDT, ch. 15, p. 594, Harrison 19th p 1906
Ref Ref : Bailey 23rd / 917, Nelson: 19th p 1274

Ans . 131. a . An ECG 132. c. Doxycyline 133. c. Premalignant 134. a . Stricture esophagus
135. c. Savary miller 136. d. 4 137. b. Weight gain 138. b. Intestinal metaplasia
139 . a & C. Stricture... 140. b. Premalignant condition 141. b. Acute mesenteric... 142. b & c CT is the...
158 143 . a . Rectal polyp 144 . c . Metabolic alkalosis with paradoxical aciduria
GIT

Trauma 151. Best for localization for Z- E syndrome is:


[NBE Pattern 2014-15]
a. Increase in Serum gastrin with pH < 2
145. Rockall score is used for prognosis of patients of:
[NBE Pattern 2014-15] b. Increase in serum gastrin with pH > 2

E
a. Upper GI bleeding
b. Lower GI bleeding c. BAO/ MAO ratio
c. Hepatic encephalopathy d. Octreoscan
d . IBD Ref: HI 8th ed. and CMDT, ch. 15, p. 619, Harrison 19th p 1928
Ref: Grading & staging in gastro.by tytgat p. 110, H 19 th p 276 152. Which one of the following is the most definitive test
146. Most common site of tear in Boerhaave syndrome:
a. Lower end of oesophagus [ NBE Pattern 2014-15]
in identifying Zollinger-Ellison syndrome?
a. EUS [Endoscopic ultrasoundj /WB/i Pattern 2014 15] - i
b. At Gastroesophageal junction b. Secretin injection test
c. Upper esophagus c. MRI
d . Mild oesophagus d. Basal gastric acid output
Ref: CMDT, ch. 15, p. 568, Harrison 19 th p 1910 Ref: CMDT, ch. 15, p. 619, Harrison 19 th p 568, 1927
147. Most common site for iatrogenic rupture of esopha ¬ 153. Cause of acute loss of vision in a patient of alcoholic
gus: [NBE Pattern 2014-15] pancreatitis is: [AI 95]
a. Cervical esophagus a . Purtscher's retinopathy
b. Thoracic below aortic arch b. Sudden alcohol withdrawal
c. Thoracic above aortic arch c. Acute congestive glaucoma
d. Abdominal d . CRAO
Ref CMDT, ch. 15, p. 599, Harrison 19 th p 1910 Ref Harrison 18th/ p 2642 Harrison 19 th p 40e - 4

Endocrine Related Disorder 154. Medical treatment of pancreatitis includes : \


a. Cholestyramine [AIIMS May 94]|
b. Aprotinin
148. Diabetes induced diarrhea is best managed by which c. Calcium
of the following? [NBE Pattern 2014- 15 ] d. Glucagon
a . Clonidine Ref: Harrison 18th/ p 2640, 2641, Harrison 19 th p 2091
b. Octreotide
c. Levosulpiride 155 . Most common complication of acute pancreatitis is:
d. Clindinium Ref : Harrison 19 th p 270, 272 a. Pancreatic abscess [AIIMS May 95J
b. Pseudocyst
149. Consider the following feature with reference to Zoll ¬ c. Phlegmon
inger Ellison syndrome: [NBE Pattern 2014- 15] d . Pleural effusion
1. Intractable peptic ulceration
ef: Harrison 18th/ p 2642, Harrison 19 th p 2091
2. Secretory diarrhea
3. Most common site is pancreas 156. All are true about pseudopancreatic cyst of pancreas ex¬
cept : [AI 1998]
Which of these features are present in Zollinger - Elli -
son Syndrome a . Common after acute pancreatitis
a. 1 and 3 b. Presents as an abdominal mass
b. 2 and 3 c. Serum amylase is increased
c. 1, 2 and 3 d. Most common site is in head of pancreas
d. 1 and 2 Ref: CSDT 11 th/ 635, 638, Bailey & Love's 26 th / 1133
Ref CMDT, ch. 15, p. 619, Harrison 19 th p 1927 157 . All are true regarding Zollinger Ellison syndrome, ex¬
150. hour prolonged fasting test is used for? cept: [AIIMS Dec 97]
a . Fat absorption [NBE Pattern 2014- 15] a. Diarrhoea
b. Insulinoma b. Recurrence after operation
c. Carbohydrate absorption c. Hypergastrinemia
d . Amino acid absorption d. Decreased ratio of BAO to MAO
Ref: Harrison’s 18th ed. ch. 345, Harrison 19 th p 569 Ref Harrison 18th/ p 2455, 2456, Harrison 19 th p 1927

Ans . 145. a. Upper GI bleeding 146. a. Lower end of oeso... 147. a. Cervical esophagus 148. a. Clonidine
149. d . 1 and 2 150. b. Insulinoma 151. d . Octreoscan 152. a . EUS [Endoscopic...
153. a . Purtscher’s... 154. c. Calcium 155. b. Pseudocyst 156. d . Most common site...
157. d. Decreased ratio of BAO to MAO
th
Marwah's Internal Medicine MCQs ( Bases on Harrison's 19 )

158. 164.Gold standard test for diagnosis of Insulinoma is : 164. A 65 - year - old smoker has a 9 - kg [20 - lb] weight loss
a . '72 hour' fast test [ AI 2009] over the last 3 months that is associated with epigas ¬
b. Plasma Glucose levels < 3 mmol / 1 tric pain after eating, diarrhea, and jaundice. Physical
c. Plasma Insulin levels > 6mU / ml examination reveals a palpable, non-tender gallblad ¬
d . C - peptide levels < 50 der and clay- colored stools . Laboratory studies show
Ref: Harrison 18th/ p 3066, 3067, Harrison 19th p 2338 a total bilirubin of 8.0 mg/ dl [normal is 0.0 -0.3 mg/
dl], a serum alkaline phosphatase of 450 U / L [nor ¬
IBS mal is 20- 70 U / L], a serum alanine aminotransferase
[ALT] of 150 U / L [normal is 8 - 20 U / L], and a urine
159 . Which of the following drugs is used for Irritable Bow ¬ dipstick that is positive for bilirubin [normal is nega ¬
el Syndrome of the constipating type : [AI 2012] tive] and negative for urobilinogen [normal is trace
a . Lubiprostone amounts]. The primary process that is most likely
b. Cholestyramine responsible for this patient's findings is located in
c. Alosetron [Manipal 2014]
which of the following?
d. Rifaximin a. Liver
Ref: Harrison’ s 18/ e p2500 - 01, Harrison 19th p 2146 t b. Gallbladder
I 160. Which of the following excludes a diagnosis of irri - c. Duodenum
I table bowel syndrome: [NBE Pattern 2014-15] d. Pancreas Ref: Harrison 19th p 2090
a . Relieved by defacation
b. Straining during stool passage 165. All are false about Zollinger Ellison Syndrome except:
a. Multiple ulcers may be seen [ Manipal 2014 ]
c. Passage of blood per rectum
d. Change of stool form b. Ulcers at unusual places may be seen
c. Gastrin level is increased
Ref CMDT, ch. 15, p. 633, Harrison 19th pi 965

GIT 161. Which of the following features are associated with


Irritable Bowel Syndrome:
a . Weight loss
[AI 1992]
d . All the above
Ref Harrison 19th p 568/ 1927
166. Pancreatic cholera is characterized by all except: [NRI ]
b. Anorexia [ Manipal 2014]
c. Abdominal distension a . Achiorhydria
d . Blood in stool b. Hypokalemia
Ref Harrison 18th/ p 2496, 2497, Harrison 19th p 1969 c. Glucose intolerance
d. Hypercalcemia Ref: Harrison 19th p 570
167. Osmotic and secretory diarrhoea can he differentiated
Miscellaneous by: [ Manipal 2014 ]
a. Osmolarity
162 . The short bowel syndrome is characterized by all of b . Na + and K+ quantity
the following except: [ Manipal 2014 ] c. Electrolyte balance
a. Diarrhea d. Fluid balance
b. Hypogastrinemia
c. Weight loss 168. Metabolic acidosis doesn’t occur in: [Manipal 2014]
d. Steatorrhoea a. Congenital hypertrophic pyloric stenosis
Ref: Harrison 19 th p 1943 b. Lactic acidosis
c. Methanol toxicity
163. Which of the following types of pancreatitis has the d. Diabetes Ref: George Matheit's Pg 471
best prognosis? [ Manipal 2014]
a. Alcoholic pancreatitis 169. H. pylori is associated with: [ Manipal 2014 ]
b. Gall stone pancreatitis a. 90% duodenal ulcer
c. Post operative pancreatitis b. 90% gastric ulcer
d. Idiopathic pancreatitis c. 70% duodenal ulcer
Ref: Harrison 19th p 2092 d . 20% gastric ulcer two.
Ref Kumar and Clark pg 272 Harrison 19th p 1039

Ans. 158. a. ‘72 hour ’ fast test 159. a . Lubiprostone 160. c. Passage of blood per...161. c. Abdominal distension
162. b. Hypogastrinemia 163. b. Gall stone pancreatitis 164. d . Pancreas 165. d. All the above
160
166. d. Hypercalcemia 167. a. Osmolarity 168. a. Congenital hyper... 169. a. 90% duodenal ulcer
GIT

170 Which of the following is not associated with pancre -


'
177 . Which diarrhea decrease after prolonged fasting?
atitis? [JIPMER 2014] a. Osmotic diarrhea
b. Bloody
[ NEET 2015 16 ]-
a. Raised serum amylase
b. Raised serum lipase c. Infective
c. Hypocalcemia d . Secretory
d. Hypoglycemia Ref Harrison's 18th edn, ch 338, Harrison 19th p 269/ 303
e. Leukocytosis
178. Acalculous cholecystitis can be seen in all the follow¬
Ref: Harrison' s 18th edn. Pg. 2635-2643, Harrison 19 th p 2091 ing conditions except: -
[NBE Pattern 2014 15]
b. Dengue
171. Features of secretory diarrhea include all of the fol¬ a. Enteric fever
lowing, except: [AI 2009 ] c. Leptospirosis d . Malaria
a. Stool volume > ID / day Ref HI 8th ed. [2005, Harrison 19th p, Harrison 19 th p 2081
b. Normal osmotic anion gap
c. Reduces with fasting 179. Most common cause of Acute Pancreatitis is:
d. Painless a . Trauma [ NBE Pattern 2014 15] -
b . Hyperlipidemia
Ref: Harrison' s 18/ e p312, Harrison 19 th p 269
c. Alcoholism
172 . Which of the following is not a prognostic factor for d . Viral infection
Acute Pancreatitis: [AIIMS Nov 06 ] Ref: CMDT, ch. 16, p. 711, Harrison 19 th p 2091
a. Serum Amylase b. Serum Calcium
c. Serum Glucose d . Serum AST 180. Pancreatic auto -transplantation is done for:
Ref: H 18 p2636, 2637 Harrison 19 th p 2091
a. Chronic pancreatitis
b . Carcinoma pancreas
[NBE Pattern 2014 15] -
173. Which of the following statements about Cystic fibro¬ c. Wolfram syndrome
sis [CF] is not true : [AI 2009 ] d. Nesidioblastosis
a. Autosomal Recessive Disorder
b. Abnormality in CFTR which leads to defective
Calcium Transport
c. Predisposition to pulmonary infection with
Ref: CMDT, ch. 16, p. 715, Harrison 19 th p 928
181. Paralytic ileus occurs in:
a. Hypokalaemia
[NBE Pattern 2014- 15]
b . Hypomagnaesemia
GIT
Pseudomonas c. Hypocalcaemia d . All of the above
d. Cirrhosis is an established complication of CF .
Ref CMDT, ch 15, p. 627, Harrison 19 th pi 982
Ref Harrison' s 18/ e p2147, Harrison 19 th p 1699 182. Anti - diabetic drug causing hemorrhagic pancreatitis:
a . Exenatide [NBE Pattern 2014- 15]
174. Cause of thick pancreatic secretions in cystic fibrosis:
b. Sitagliptin
a. Overproduction of mucin [ DNB 2013] c. Saxagliptin
b. Failure to clean mucin due to epithelial dysfunction
c. Defect in chloride channel leading to water reab
d . Canagliflozin
¬

sorption Ref: CMDT, ch. 27, p. 1211, Harrison 19th p 2090


d . Defect in sodium channel leading to water reabsorp ¬ 183 . A 5 year child with history of barefoot walking and
tion open air defecation presents with anemia and swell ¬
Ref: Harrison's 18/ e p2065, Harrison 19th p 1699 ing around eyes. Which of the following infestation is
175 . Drug used to relieve facial flushing associated with most likely to be present.
niacin toxicity: [ PCI May 2013] [AIIMS MAY 2014 ]
a. Fometidine b. Foxafenadine a. Round worm b. Hook worm
c. Ranitidine d. Laropiprant c. Pin worm d . Whip worm
Ref: Harrison's 18/ e p598, Harrison 19 th p 96 e - 4 Ref CMDT, ch. 35, p. 1518, Harrison 19 th p 1414
176 . Mutation of STK 11 and LKB 1 gene is associated with? 184. All of the following are true regarding Acute pancre ¬
a . Familial Adenomatous Polyposis [JIPMER 2014] atitis except: [ NBE Pattern 2014 - 15]
b. Hereditary nonpolyposis colorectal cancer a. Elevated serum amylase
c. Peutz - Jeghers syndrome b. Alcoholics are more prone
d. Neurofibromatosis c. Ranson score is used to grade severity
d. Raised serum calcium
Ref Robbins 8th edn ch 15, Harrison 19 th p 536
Ref: CMDT, ch. 16, p. 711, Harrison 19th p 2090
Ans. 170 . d . Hypoglycemia
174.
171. c. Reduces with fasting 172. a . Serum Amylase 173. b. Abnormality in CFTR ...
178.
.
b . Failure to clean . . 175. d . Laropiprant 176. a . Familial Adenomat... 177. a . Osmotic diarrhea
d . Malaria 179. c. Alcoholism 180. a . Chronic pancreatitis 181. d. All of the above
182. a. Exenatide 183. 161
b . Hook worm 184. d. Raised serum calcium
Marwah 's Internal Medicine MCQs ( Bases on Harrison's 19th )

185. All cause diarrhea except? [NBE Pattern 2014-15] 192 . A lady presented with non progressive dysphagia only
a. Diabetes for solids. Barium study showed proximal esophageal
b. Hypercalcemia dilatation with distal constriction. The most likely di ¬
c. Hyperthyroidism

\
agnosis is. [AI 2010]
d. Irritable bowel syndrome a. Peptic Stricture
Ref: CMDT, ch. 15, p. 574, Harrison 19 th p 265 b. Carcinoma Esophagus
186. Exposure to which of the following drugs is incrimi ¬ c. Achalasia Cardia
nated in IHPS [ infantile hypertrophic pyloric steno¬ d . Lower Esophageal Ring
sis]: [ NBE Pattern 2014-15] Ref: Harrison's 17th/ 239, 240, 1854, Harrison 19 th p 1904
a. Erythromycin
b. Lithium 193. Which of the following statements about Schilling's
c. Warfarin test are true: [ PCI 2009]
d. Carbimazole Ref: Nelson's 18th ed., ch. 326 a. Abnormal in pernicious anemia
187 . All of the following are true regarding a patient with b. Normal in bacterial overgrowth syndrome
acid peptic disease except: [ NBE Pattern 2014-15] c. Abnormal in ileal disease
a. Misoprostol is the drug of choice given with NSAIDs d. Normal in chronic pancreatitis
b. Dll is preventable by the use of night time H 2 block¬ Ref: Harrison 18th/ p 2467, Harrison 19th p 1941 t
ers
c. Omeprazole may help ulcers refractory to H 2 block ¬ 194. Serum amylase level is raised in all except:
ers a. Blocked salivary duct [PGIJune 98]
d. Misoprostol is DOC in a pregnant lady b. Ruptured ectopic
Ref CMDT, ch. 15, p. 613, Harrison 19 th p 1911 c. Appendicitis

GIT l 188. Which of the following is/are associated with pancre ¬


atic exocrine insufficiency;
a . Hypertriglyceridemia
b. Enterokinase deficiency
[NBE Pattern 2014-15]
d. Pancreatitis
Ref: Harrison 18th/ p 2632, Harrison 19 th p 1987

195. Which of the following parastitic infestation can lead


c. Malabsorption to malabsorption syndrome? [AI 06 ]
d . All of the above a. Amoebiasis
Ref: CMDT, ch. 15, p. 578, Harrison 19 th p 2090 b. Ascariasis
c. Hookworm infestation
189. A vasopressin analogue does not produce therapeutic d. Giardiasis
effect through vasopressin V2 receptor in which of the
Ref: CMDT 6 th/ 1479, Harrison 19 th p 1946 t
following
a. Central diabetes insipidus [NBE Pattern 2014-15] 196. A patient present with lower gastrointestinal bleed .
b. Bleeding oesophageal varices Sigmoidoscopy shows ulcers in the sigmoid. Biopsy
c. Type 1 von Willebrand 's disease from this area shows flask-shaped ulcers. Which of
d. Primary nocturnal enuresis.
the following is the most appropriate treatment :
Ref CMDT, ch. 15, p. 602, Harrison 19th p 276 , 2063 a. Intravenous ceftriaxone [AIIMS Nov. 05 ]
190. The area of colon which is least visualized by barium b. Intravenous metronidazole
studies: [NBE Pattern 2014-15] c. Intravenous steroids and sulphasalazine
a. Sigmoid b. Hepatic flexure d. Hydrocortesone enemas
c. Splenic flexure d . Caecum Ref: Harrisons 18th/ p 1686 , Harrison 19 th p 1363
Ref WWW. HopkinsMedicine.org, Harrison 19 th p 590
197. Investigation of choice for invasive amebiasis is:
191. Increased gastrin is seen in: [NBE Pattern 2014-15]
a. Zollinger - Ellison syndrome a . Indirect heamagglutination [AI 02]
b. Iron deficiency anaemia b. ELISA
c. Duodenal ulcer c. Counter immune electrophoresis
d. Gastric cancer d. Microscopy
Ref: CMDT, ch. 15, p. 619, Harrison 19 th p 568, 1928 Ref: Ghai 5th/ 221, Harrison 19 th p 1363

Ans. 185. b . Hypercalcemia 186. a. Erythromycin 187. d. Misoprostol is DOC ... 188. d. All of the above
189. b. Bleeding oeso... 190. d. Caecum ..
191. a. Zollinger -Ellison . 192. d . Lower Esophageal Ring
162
193. a & c . Abnormal... 194. c. Appendicitis 195. d . Giardiasis 196. b. Intravenous metronidazole
197. b. ELISA
GIT

198. A girl presents with complaints of malena . On ex¬


amination there are pigmented lesions involving her
mouth and lips. Two of her sisters also had similar
complaints . Which of the following is the most prob ¬
200. Raised serum amylase levels are used to diagnose
a. Autoimmune disease
b. Degenerative diseases
c. Acute cholecystitis
[AIIMS May 04 ]
I
[AIIMS Nov 2000] d . Acute pancreatitis
able diagnosis:
a . Komkhite Canada syndrome Ref: Harrison 18th/ p 2636 , Harrison 19 th p 2091
b. Puetz Jagher's syndrome 201. Which of the following statements about Cystic fibro ¬
c. Gardner’s syndrome
sis [CF] is not true: [AI 2009]
a. Autosomal Recessive Disorder
d . Turcot's syndrome
b. Abnormality in CFTR which leads to defective
Ref: Ch 91, Harrison 18th edn, Harrison 19 th p 538t Calcium Transport
199. All the following are causes of Acute Pancreatitis ex¬ c. Predisposition to pulmonary infection with
cept: Pseudomonas
[AI 1994]
d . Cirrhosis is an established complication of CF
a. Gall stones
b. Alcohol Ref: Harrison 18th/ p 2147, Harrison 19 th p 1699
202 . Hypergastrinemia with hypochlorhydria is seen in:
c. Hemochromatosis
a. Zollinger Ellison Syndrome [AI 2002]
d . Hypercalcemia b. VIPoma
Ref: Harrison 18th/ p 2635, Harrison 19 th p 2091 c. Pernicious anemia
d . Glucagonoma
Ref: Ch 293, Harrison 18th edn, Harrison 19 th p 2346 t

Recommendations for Treatment of NSAID-Related


Mucosal Injury
Clinical Setting
> Prophylactic therapy
Recommendation
Misoprostol ADDITIONAL UPDATES
GIT
PPI
Selective COX -2 inhibitor
I Sensitivity of Imaging Studies in Zollinger-Ellison
I Syndrome
> H. pylori infection Eradication if active ulcer
Sensitivity, %
present or there is a past
history of peptic ulcer disease Study Primary Gastrinoma Metastatic Gastrinoma

Active ulcer H2 receptor antagonist or EUS 80-100 N /A


> >
• NSAID discontinued PPI Octreo Scan 67-86 80-100
• NSAID continued • PPI MRI 55-70 > 85

Ans . 198 . b . Puetz Jagher ’s ... 199 . c .


Hemochromatosis 200 . d . Acute pancreatitis 201 . b . Abnormality in CFTR ...
202 . c . Pernicious anemia

163
CHAPTER Rheumatology
7
RHEUMATOLOGY UPDATES

> A primary antiphospholipid syndrome ( APS) is diagnosed in


1. Patients who have venous or arterial occlusions or recurrent fetal loss in the presence of persistent (£ 12 weeks)
2. High-titer,diagnostic antiphospholipid antibodies but no other features of SLE.
3. Diagnostic antiphospholipid antibodies are IgG or IgM anticardiolipin, or IgG or IgM antibodies to beta-2-glycoprotein I, and
lupus anticoagulant.
In < 1% of patients with antiphospholipid antibodies, a potentially devastating syndrome known as the “ catastrophic antiphospholipid
syndrome” occurs, leading to diffuse thromboses, thrombotic microangiopathy, and multiorgan system failure.
Laboratory Findings
• Three type of antiphospholipid antibody are believed to contribute to this syndrome: (1) anti-cardiolipin antibodies: (2) antibodies to
beta-2 glycoprotein ; and ( 3) a “ lupus anticoagulant ” that prolongs certain phospholipid-dependent coagulation tests.
• Antibodies to cardiolipin and to beta-2 glycoprotein are typically measured with enzyme immunoassays. Anti-cardiolipin antibodies
can produce a biologic false-positive test for syphilis (ie, a positive rapid plasma regain but negative specific anti-cardiolipin antibodies
are believed to be more pathologic than IgM.
• Presence of the lupus anticoagulant is a stronger risk factor for thrombosis or pregnancy loss than is the presence of antibodies to
either beta-2-glycoprotein I or anticardiolipin. A clue to the presence of a lupus anticoagulant, which may occur in individuals who
not have SLE , may be detecte d by a prolong ation of the partial thromb oplasti n time ( which, parado xically, is associa ted with a
do
thrombotic tendency rather than a bleeding risk).
Testing for the lupus anticoagulant involves phospholipid dependent functional assays of coagulation, such as the Russell
viper

RWT ). In the presen ce of a lupus anticoa gulant , the RWT is prolong ed and does not correct with mixing studies but
venom time (
does with the addition of excess phospholipid.

3. What is the most common electrolyte abnormality in


Most Recent Q’s 2014- 15
sarcoidosis?
a. Hypocalcemia
.
1 Normal CRP with elevated ESR seen in? b. Hypokalemia
a. RA [NEET Pattern 2015-16] c. Hypermagnesimia
b. SLE d. Hypercalcemia Ref Harrison 19th p 2683
c. Scleroderma
4. Orange tonsils are a feature of: [NBE Pattern 2014]
d. Polymyalgia rheumatica a. Refsum disease
H 18th ed.Ch 319 , 321, 323, 326 , Harrison 19th p 2124
Ref: b. Tangier's disease
2 . Best test for subacute cutaneous lupus Erythematosus: c. Scarlet fever
a. dsDNA [NBE Pattern 201 -2014] d. Kawasaki disease Ref Harrison 19th p 1443
b. ANA 5. Xerostomia is seen in which of the following diseases?
c. Anti-Ro / SS-A a. Wegeners Granulomatosis [ NBE Pattern 2014]
d. Anti-histone antibody b. Ankylosing spondylitis
Ref: H 18th ch. 319, table 319 -1, Harrison 19th p 375 c. Sjogren syndrome
d. None of the above Ref Harrison 19th p 241

Ans . 1. b. SLE 2. c. Anti-Ro/ SS-A 3. d. Hypercalcemia 4. b. Tangier ’s disease

i 5. c . Sjogren syndrome
Rheumatology

6. Which is the most common organ involved in 13. -


An 18-years old boy presents with digital gangrene
sarcoidosis? [ DNB Pattern 2014] in third and fourth fingers for last 2 weeks. On
a. Lung examination the blood pressure is 170 /110 mm of Hg
b. Eye and all peripheral pulses were palpable. Blood and i
c. Liver urine examinations were unremarkable.
d. Bone Antinuclear antibodies, antibody to double stranded I
DNA and anti- neutrophil cytoplasmic antibody were
7. Kviem's Sitzbach test is used in diagnosis of which negative.The most likely diagnosis is:
disease? [ DNB Pattern 2014] a. Wegner's granulomatosis [NBE Pattern 2014-15]
a . Tubeculosis b. Polyarteritis nodosa
b. Sarcoidosis c. Takayasu's arteritis
c. Histoplasmosis d. Systemic lupus erythematosus [SLE]
d. Lieshmaniasis Ref: CMDT 2014 ch.20, Pg. 847, Harrison 19th p 2187
8. What is the treatment of acute sarcoidosis? 14. - -
A 20 year old woman presents with bilateral
a. Infliximab [ DNB Pattern 2014] maxillary sinusitis, palpable purpura on the legs and
b. Cyclosporin hemoptysis. Radiograph of the chest shows a thin -
c. IVIg walled cavity in left lower zone. Investigations reveal
d. Prednisolone total leukocyte count 12000/ mm3, red cells casts in
the urine and serum creatinine 3 mg / dl. What is the

Rheumatolgy
Vasculitis most probable diagnosis?
a. Henoch.Schonlein purpura [NBE Pattern 2014- 15]
9. Takyasu arteritis mainly affects: b. Polyarteritis nodosa
[ DNB 2013]
c. Wegener's granulomatois
a. Pulmonary artery
d. Disseminated tuberculosis
b. Celiac artery
Ref: Harrison' s 18th ed. ch.326, Harrison 19th p 2184
c. Subclavian artery
d. SMA 15. In Takayasu's arteritis there is: [ NBE Pattern 2014-15]
Ref: Harrison's 18/ e p796, Harrison 19th p 2190 a. Intimal fibrosis b. Renal hypertension
c. Coronary aneurysm d. All of the above
10. Bilateral upper limb pulse less disease is: [ DNB 2013] Ref: Harrison's 18th ed. ch.326, Harrison 19th p 2189
a. Giant Cell Arteritis
b. Polyarteritis Nodosa
16. -
c ANCA is pathogonomic of: [NBE Pattern 2014- 15]
a. Classical PAN
c. Aortoarteritis b. Wegener's granulomatosis
d. HSP c. Crescentric nephritis
11. Vasculitis seen only in childhood? [AIIMS Nov. 2014] d. SLE
a. Kawasaki Ref: Harrison' s 18th ed. ch.326, Harrison 19th p 2184
b. HSP 17. A 35- year -old man present with episodes of vomiting,
c. Susac syndrome photophobia and unilateral Pulsatile headache.What
d. Giant cell is likely cause? [NBE Pattern 2014-15]
Ref: Nelson 18th ch 326, ch 165, H 18th, Harrison 19th p 2192 a. Cluster headache
12 . Incorrect about takayasu arteritis: b. Giant cell arteritis
a. Spares pulmonary artery [NBE Pattern 2014-15] c. Acute congestive glaucoma
b. Renovascular hypertension
d . Tension headache Ref : Harrison' s 18th ed.ch.28
18. Intravenous immunoglobulin is given in all except?
c. Blood pressure difference between left and right
a . Kawasaki disease [ NBE Pattern 2014 - 15]
limbs b. Acute ITP
d. Strongly positive mantoux c. Wegener's Granulomatosis
Ref: OP Ghai 7th ed. Pg 605, Harrison 19th p 2189 d . Myasthenic Crisis Ref: CMDT 2013 Pg. 848

6. a. Lung 7. b. Sarcoidosis 8 . d. Prednisolone 9. c. Subclavian artery


10 . c. Aortoarteritis 11. a. Kawasaki 12. a . Spares pulmonary... 13. b. Polyarteritis nodosa
14. c. Wegener ’s gran... 15. b. Renal hypertension 16. b. Wegener ’s granulo... 17. c. Acute congestive... 165
18. c. Wegener ’s Granulomatosis
I
Marwah s
' Interna l Medic ine MCQs ( Bases on Harris on's 19 th
)

19 . What is feature of temporal arteritis? 25. Consider the following statements regarding classic
a. Giant cell arteritis [NBE Pattern 2014-15] polyarteritis nodosa: [ NBE Pattern 2014-15]
b. Granulomatous vasculitis I. It is multi-system necrotising vasculitis
c. Necrotizing vasculitis II . Small & medium vessels are involved

B 20.
d. Leucocytoclastic Vasculitis
Ref: CMDT 2013 Pg. 845, Harrison' s: 18th ed.ch.326
Which of the following are true about findings of Poly ¬
arteritis nodosa?
a. There is tear in the lamina Dura
[ NBE Pattern 2014-15]

b. Micro Aneurysm formation in the large blood vessel


c. Nodules are formed in skin which are clinically
III. Pulmonary artery involvement is a characteristic

IV.
feature.
Up to 30% patient may show positive test for
Hepatitis B surface antigen
Which of these statements are correct?
a. I and II
c. I, II and IV
b . II and III
d. II, III and IV
palpable Ref: CMDT 2014 ch.20, Pg. 847, Harrison 19 th p 2187
d. Chain of beads appearance 26 . Upper respiratory tract infections are common in :
Ref: CMDT 2014 ch.20, p 847 a . Tetralogy of Fallot [ NBE Pattern 2014 -15]
b. Cystic fibrosis
21 . Giant cell arteritis causes which of the following in c. Mitral Regurgitation
the eye: [ NBE Pattern 2014 -15] d . Wegener's granulomatosis
a . Episcleritis .
Ref: Harrison's 18th ed.ch 326, Harrison 19th p 2182
b. Anterior ischemic optic neuropathy
& c. Neuroparalytic keratitis
d . Band keratitis
27 . Which of the following is an example of small - vessel
vasculitis? [NBE Pattern 2014 -15]
£o Ref : Harrison' s 18th ed.ch.14, p 112
a. Takayasu arteritis b. Microscopic polyangitis
c. Giant cell arteritis d. Polyarteritis nodosa
22. A 30 - year - old male patient presents with complaints Ref: CMDT 2013, Pg. 845, Harrison 19th p 2186

s3 of weakness in right upper and both lower limbs for


last 4 months. He developed digital infarcts involving
2nd and 3 rd fingers on right side and 5th finger on
28. Cavitating lesion in lung is seen in:
a . Wegner's granulomatosis [ NBE Pattern 2014-15]
b. PAN
O left side.On examination, BP was 160 / 140 mmHg, c. SLE
,3 all peripheral pulses were palpable and there was d . Goodpasture's syndrome
oeS asymmetrical examination showed proteinuria and .
Ref: Harrison's 18th ed.ch 326, Harrison 19th p 2182
RBC- 10 - 15/ hpf with no casts. What is the most likely 29. Hepatitis B virus is associated with:
diagnosis? [NBE Pattern 2014 -15] [ NBE Pattern 2014 -15]
a . SLE
a. Polyarteritis nodosa b. Polyarteritis nodosa
b. Systemic lupus erythematosus
c. Sjoren's syndrome
c. Wegner’s granulomatosi
d . Wegener's granulomatosis
d. Mixed cryoglobulemia
Ref: CMDT 2014 ch.20, Pg. 847 Ref: CMDT 2014 ch.20, Pg. 847, Harrison 19th p 2187
is: 30. Pulse absent in radial artery is seen in?
23. Treatment of choice in Wegner's granulomatosis [NBEPattern 2014-15]
a. Coarctation of aorta
a. Cyclosporine [NBE Pattern 2014-15] b. Aortic regurgitation
b. Cyclophosphamide c. Takayasu's arteritis
c. Steroids d. Dissection of Aorta
d. Radiotherapy Ref: Harrison's 18th ed.ch.326, Harrison 19th p 2189
Ref: CMDT 2014 ch.20, Pg. 848, Harrison 19th p 2182 All of the following are small vessels vasculitis, Except:
31.
24. True about Giant cell arteritis is all except: a . Classical PAN [ PCI Dec 2000 ]
a . High dose of steroid is drug of choice b. Wegner's granulomatosis
b. ESR is usually raised [ NBE Pattern 2014-15] c. HSP
c. Intracranial ICA is particularly susceptible d. Churg Strauss Syndrome
e. Microscopic Polyangitis
d . Mainly affects people of > 70 years
Ref: Harrison 18th edn/ p 2743, Harrison 19th p 2187
.
Ref: Harrison's 18th ch.14, Pg 112, Harrison 19th p 2189

. c. are formed ... 21 . b. Anterior ischemi c... 22. a. Polyarteritis nodosa


Ans. 19. a. Giant cell arteritis 20 Nodules
26. d. Wegener’s granulomatosis
23. b. Cyclophosphamide 24. c. Intracranial ICA is parti... 25. c. I , II and IV
30. c. Takayasu ’s arteritis
27. b. Microscopic poly... 28. a. Wegner ’s granulomatosis 29. b. Polyarteritis nodosa
166 31. a. Classical PAN
Rheumatology

32. ANCA positive vasculitis include all of the following 38. A patient presents with melaena normal renal func ¬
Except: [ PGI Dec 06] tion, hypertension and mononeuritis multiplex. The
a. Wegner's granulomatosis most probable diagnosis is: [AIIMS Nov 04 ]
b. Churgstrauss syndrome a.

E
Classical polyarteritis nodosa
c. Microscopic PAN b. Microscopic polyangiitis
d. Good pasture's syndrome c. Henoch-Schonleinpurpura
Ref: Harrison 18th edn/ p 2743, Harrison 19th p 1839 d. Buerger's disease
33. C-ANCA is associated with: [ PGI - June 08] Ref Harrison 18th/ p 2794, Harrison 19 th p 2187
a. Wegener's Granulomatosis
b. Microscopic Polyangitis 39. Biopsy in PAN shows: [PGI June 98]
c. Churg- Strauss Syndrome a. Necrotizing arteritis
d. PolyarteritisNodosa [PAN] b. Atrophy
Ref Harrison 18th edn / p 274, 2792, Harrison 19th p 2182 c. Granulomatous lesion
d. Ring lesion
34. p-ANCA is characteristic for: [AIIMS Dec 92]
a. PAN Ref Harrison 18th/ p 2794, Harrison 19th p 2187
b. Microscopic polyangitis 40. Which of the following is more frequently seen in
c. Wegener’s granulomatosis Churg Strauss Syndrome in comparison to Wegener's
d . Henoch -Schonleinpurpura Granulomatosis [ NEET / DNB Pattern Question]

Rheumatolgy
Ref Harrison 18th / p 2743, Harrison 19 th p 368e- l a. Renal involvement
35. All of the following condition are associated with b. Lower Respiratory Tract involvement
granulomatous pathology, except: [Al 2010] c. Eye involvement
a. Wegner's Granulomatosis [WG] d . Upper Respiratory Tract involvement
b. Takayasu Arteritis [TA] Ref Harrison 19th p 2186
c. Polyarteritis Nodosa [Classic PAN]
41. An elderly female presents to the emergency
d. Giant Cell Arteritis [GCA]
department with history of fever, headache and
Ref Robbins 8th/ 2nd/ 286; H 18th/ p2792, Harrison 19th / 2187
double vision. Biopsy of temporal artery revealed
36. A 25 years old female develops serous otitis media panarteritis. The most likely diagnosis is
of left ear with cough and occasional hemoptysis and a. Nonspecific Arteritis [AIIMS Nov 09]
hematuria and epistaxis for one and half months. Her b. PolyarteritisNodosa
Hemoglobin is 7 gm . B. P. > 170 / 100, ptoreinuria ++ +, c. Wegener's Granulomatosis
RA positive [+ ve] and ANCA positive [ + ve], the likely
d. Temporal Arteritis
cause is- [AIIMS June 99]
a. Wegener’s granulomatosis Ref Robbins 8th/ 512; H 18th/ 2795, 2796, Harrison 19 th p 2188
b. Rheumiatoid arthritis
42 . Which of the following is the most frequent presenting
c. Rapidly proliferative glomerulonephritis
d. Good pasteur's syndrome symptoms in patients with giant cell arteritis
a. Headache [AI 1991, 1999 ]
Ref Harrison 18th/ p 2790, 2791, Harrison 19th p 2182
b. Jaw claudication
37. An 18 year old boy presents with digital gangrene in c. Polymyalgin Rheumatica
3rd & 4th fingers for last 2 weeks. On examination d . Blindness
the BP is 170/110 mm of Hg and all peripheral pulses
were palpable. Blood & Urine examination were unre ¬ Ref Harrison 18th/ p 2795, Harrison 19 th p 2189
markable. Antinuclear antibody, Antibody to ds DNA 43. The investigation of choice for diagnosis of Giant cell
& DNA & ANCA were negative. Most likely diagnosis is: Arteritis is [AI -1990]
[AI 2004] a. Temporal Artery biopsy
a. Henoch -Schonlein purpura b. Colour Doppler of Temporal Artery
b. Polyarteritis nodosa c. CT Angiography
c. Wegener’s granulomatosis
d. MRI
d. Disseminated tuberculosis
Ref: Harrison 18th/ p 2795, 2796 , Harrison 19th p 2189
Ref Harrison 18th/ p 2794, Harrison 19th p 2187

Ans - 32. d. Good pasture’s... 33. a. Wegener ’s Gran... 34. b. Microscopic poly... 35. c. Polyarteritis Nodosa...
36. a . Wegener ’s gran... 37. b. Polyarteritis nodosa 38. a. Classical polyart...
40. b. Lower Respir... 39. a. Necrotizing arteritis
41. d. Temporal Arteritis 42. a. Headache 43. a. Temporal Artery biopsy
167
I
Marwah's Internal Medicine MCQs ( Bases on Harris on 's 19 th
)

44. Bilateral upper limb pulse less disease is? SLE


a. Giant cell Arteritis [ NEET / DNB Pattern Question]
b . Polyarteritis Nodosa 51. True abut SLE in pregnancy A / E: [JIPMER 2014 ]
HH c. Aortoarteritis a. Increased Anti Ro and La implies low risk for
d. HSP congenital heart block
b. Azathioprine can be continued in pregnancy
Ref: Harrsion 18th, Ch 326 , Harrison 19th p 1637
c. Hydroxychloroquine has to be stopped if breast¬
I 45. Reversed Coarctation is seen in: feeding is planned.
a. Giant cell Arteritis [ NEET / DNB Pattern Question] d. Disease worsens during pregnancy
b. Polyarteritis Nodosa Ref : Harrison's 18th edn. Harrison 19th p 2133
c. Takayasu Arteritis 52 . A 32 - year- old primi gravid presents with DVT. There
d. Kawasaki Disease Ref: Harrison 19th p 2190 is history of 2 episodes of DVT in the past and was
46 . Kawasaki disease is associated with all of the follow ¬ diagnosed to have lupus anticoagulant antibody. Next
step in management is: [ flPMER 2014 ]
ing features except: [AI 1996 ]
a . Aspirin alone
a. Erythema b. Aspirin and heprarin
b. Posterior cervical Lymphadenopathy c.
LMWH
c. Thrombocytopenia d . Warfarin for 6 months Ref : Harrison 19 th p 1631
d . Conjunctivitis
53. Most common Inheritance in Alport syndrome is:
Ref: Harrison 18th/ p 2800, Harrison 19 th p 2193 [ NBE Pattern 2014 ]

hReumatolgy
a. X linked dominant
47 . The treatment of choice for Kawasaki disease is: b. Autosomal dominant
a . Cyclosporine [AllMS May 2005] c. Autosomal recessive
d. X linked recessive Ref: Harrison 19 th p 2514
b. Prednisolone
c. Immunoglobulins 54. The poly-arthritic condition that is NOT common in
d . Methotrexate males: -
[ NBE Pattern 2014 15]
a . Gout b. Psoriatic arthritis
Ref: Nelsons 18th/ l 038; Harrison 19th p 2193 c. Ankylosing spondylitis
48. Which of the following is most commonly involved in d . Systemic lupus erythematosus
Hypersensitivity vasculitis: [AIIMS May 09] Ref Harrison's 18th ed.ch.319, Harrison 19 th p 2124
a. Capillaries 55. An 18 yr old boy presents with melaena, abdomal
b. Arterioles pain, numbness of both lower limbs and right upper
c. Post-capillary venules limb.O / E: pulse: 88 /min, BP: 170 / 110 mm Hg, and all
d. Medium sized arteries peripheral pulses palpabie.Renal function is normal .
Urine examination shows proteinuria and RBC with
Ref: Harrison 18th/ p 2798, Harrison 19 th p 2191
no casts. What is the most probable diagnosis?
49. A middle aged female presents with polyarthritis, a. Classical PAN [ NBE Pattern 2014]
elevated Rheumatoid factor and ANA levels. Which b. Microscopic polyangitis
of the following features will help in differentiating c. HSP
Rheumatoid arthritis from SLE [AIIMS Nov 08] d . Wegener's granulomatosis
a. Soft tissue swelling in PIP Joint Ref Harrison 19 th p 2187
b. Juxta-articular osteoporosis on X ray 56. Characteristic Cardiac lesion in SLE is:
a. Verrucous endocarditis [NBE Pattern 2014-15]
c. Articular erosions on X Ray
b. Valvular incompetence
d. Elevated ESR Ref Harrison 19th p 2145 / 2131
c. Heart block
50. Renal artery stenosis may occur in all of the following, d . Myocardial fibrosis Ref: Harrison’s 18th ed.ch.319
except : [AI 06 ] 57 . A young girl is admitted with joint pains and butterfly
a. Atherosclerosis rash and positive urine proteinuria. The best test for
b. Fibromuscular dysplasia her diagnosis is? [ NBE Pattern 2014 -15]
c. Takayasu's arteritis a. Anti ds- DNA antibody
d . Polyarteritis nodosa b. Anti - centromere antibody
Ref Harrison 18th edn/ p 2794, Harrison 19th p 2187 c. Antibodies to RNP
d . Antibodies to tRNA synthetase
Ref CMDT 2013, pg 831

Ans. 44. c. Aortoarteritis 45. c. Takayasu Arteritis .


46. c Thrombocytopenia 47. c. Immunoglobulins
48. c. Post -capillary... 49. c. Articular erosions... 50. d. Polyarteritis nodosa 51. a. Increased Anti Ro and La...
52. c. LMWH 53. a . X linked dominant 54. d. Systemic lupus... 55. a. Classical PAN
168
56. a. Verrucous end ... 57. -
a. Anti ds DNA antibody
Rheumatology

A 33 - year- old woman has experienced episodes of 66. True regarding SLE in children: [ NBE Pattern 2014-15]
58 .
fatigue, pleural effusion, pericardial effusion and car¬ a. Skin pigmentation more common
pal tunnel syndrome and macrocytic anemia . Best b. No sex difference
[NBE Pattern 2014-15]

E
test for diagnosis shall be: c. Renal involvement more common
a. Anti - beta 2 phospholipid antibodies d . Cardiac involvement more common
b. Anti -smith antibody Ref: Harrison' s, 18th ed .ch. 319
c. Antinuclear antibody
d . Assay for thyroid hormones Lupus anti coagulant is associated with all except:
a. Recurrent abortion [ NBE Pattern 2014-15]
Ref: Harrison' s 18th ed.ch.319
59. Butterfly rash in SLE involves all areas except b. Polyhydramnios
a. Cheeks [NBE Pattern 2014-15] c. Intrauterine growth retardation
b. Nasolabial fold d. Pre -eclampsia -early onset
c. Lower eylids Ref: Harrison' s 18th ed.ch.319
d. Bridge of nose
68. Most common presentation ofS. L. E
Ref: Harrison's, 18th ed., ch. 319, Harrison 19th p 2127 [ NBE Pattern 2014-15]
a. Arthralgia
60. Onion skin spleen is seen in: [NBE Pattern 2014-15]
b. Non-erosive polyarthritis
a. ITP b. Thalassemia
c. Butterfly rash
c. SLE d. Scleroderma
d . Autoimmune hemolytic anemia
Ref: Harrison's 18th ed.ch.319, Harrison 19th p 2127
61. Psychosis in SLE is caused by: [NBE Pattern 2014-15]
Ref: H 18th ed., ch. 319, Table 319 -4, Harrison 19 th p 2127 PS
69. All of the following are indicators for use of cortico ¬ S'
a. Anti-ribosomal P antibody
b. Anti-glutamate add decarboxylase antibody steroids in SLE except: [NBE Pattern 2014- 15] <x>
c. Anti-endomyosial antibody a. Neuropsychiatric lupus C
d. Anti-histone antibody b. Pericarditis 3w
Ref: H 18th ed.ch. 319, Table 319 -1, Harrison 19th p 2126 c. Endocarditis
62. Lupus anticoagulant causes A/ E:
d . Nephritic syndrome o
[NBE Pattern 2014-15] Ref Harrison's 18th ed.ch.319, Harrison 19th p 2128
a . Recurrent abortion b. Arterial thrombosis
c. Increase aPTT d. Nephritis
Ref: Harrison's 18th ed.ch.319, Harrison 19 th p 2254
70. True about drug induced SLE is except?
a. Female: Male ratio = 9 : l
b. Anti- histone Antibodies
[ NBEPattern 2014 15] - II
63 . Most common presentation of cardiac lupus?
a. Myocarditis
b. Pericarditis
c. Aortic regurgitation
d . Libman sacks endocarditis
[APPG 2014]
c. CNS involvement not common
d . Renal involvement not common
Ref: CMDT 2013 p 832, Harrison 19th p 2126
I
71. True about nervous system involvement in SLE are all
Ref: CMDT 2013. Connective tissue disorders, ch. 20,Pg 833 except: [ NBE Pattern 2014-15]
64. Most common cause of death in SLE in children a. Seizures
a. Lupus nephritis [ NBE Pattern 2014-15] b. Antibodies against aquaporin-4 antibody
b. Lupus cerebrits c. Elevated protein level in CSF
c. Libman sacks endocarditis d . Pseudo-tumor cerebri
d . Anemia and infections
Ref: Harrison' s 18th ed .ch.319, Harrison 19th p 2128
Ref: OP ghai 7th ed.Pg 603, Harrison 19 th p 2128
72 . Which of the following antibodies correlates with
65. The following are features of SLE except:
[NBE Pattern 2014-15] disease activity for S. L. E [ NBE Pattern 2014-15]
a. Sterile Vegetations on valve cusps a. Anti Smith antibody
b. Raynaud's phenomenon b. Anti dS DNA antibody
c. Atherosclerosis c. Anti Histone antibody
d . Pulmonary fibrosis d. Anti Rho
Ref: CMDT 2013 Pg. 832, Harrison 19th p 2126 Ref: CMDT 2013 Pg. 832, Harrison 19th p 2126

Ans . 58. d . Assay for thyroid ... 59. b. Nasolabial fold 60. c. SLE 61. a . Anti -ribosomal P antibody
62. d . Nephritis 63. b . Pericarditis 64. a. Lupus nephritis 65. d. Pulmonary fibrosis
66. c . Renal involve... 67. b. Polyhydramnios 68. a. Arthralgia 69. c . Endocarditis
70. a . Female: Male. .. 71. b. Antibodies against... 72. b. Anti dS DNA antibody 169
,
MCQs ( Bases on Harrison's 19 h )
Marwah's Internal Medicine

73. Bony erosion are seen in the following except: 80. Deposition of Anti ds DNA Ab in kidney, skin, choroid
a. Gout [ NBE Pattern 2014-15] plexus and joints is seen in: [Al 2007]
b. Psoriasis a. SLE
j c. SLE b. Good pasture
^: d. Osteoarthritis
Ref: Harrison's 18th ed.ch.319, Harrison 19th p 2124
c.
d.
Scleroderma
Raynauds disease
Ref: Robbins 7th / 229, Harrison 19th p 2131
II 74. Partoid gland enlargement is seen in all except:
a. Sjogren's syndrome [NBE Pattern 2014 15] - 81. Autoimmune destruction of platelet is seen in :
b. Sarcoidosis a. SLE [AIIMS May 95]
c. Chronic pancreatitis b. Rheumatoid arthritis
d. SLE c. Reiter disease
Ref: Harrision’s 18th ed.p 1991, Harrison 19th p 2131 d. Polyarteritisnodosa
Ref: Harrison 18th/ p 2728, 2730, Harrison 19th p 2131
! 75. Anti -double stranded DNA is highly specific for :
a. Systemic sclerosis [AIIMSJune 97] 82 . All of the following are known to cause Lupus like
b. S.L.E. syndrome, except [AIIMS Nov 2010]
c. Polymyositis a. INH b. Penicillin
d. Rheumatic sclerosis c. Hydralazine d. Sulphonamide
Ref: Harrison 18th/ p 2727, Harrison 19thp 2131 Ref: Harrison 18th/ p 2735, Harrison 19th p 2152

Knemaiugy
76. Best marker for drug induced lupus is : [A12007] 83. In SLE, characteristic kidney lesion is: [ PCI Dec 98]
a. Antihistone antibodies a. Mesangial proliferation
b. Anti ds DNA b. Tubular fibrin deposits
c. ANA c. Wire loop lesions
d. Anti smith Ab d. IgG deposits
Ref: Harrison 18th/ p 2726, [Table: 319.1 Ref: Harrison 18th/ p 2727, Harrison 19th p 2131
77. A 23-year old woman has experienced episodes of 84. Features of SLE include all of the following except:
myalgias, pleural effusion, pericarditis and arthral ¬ a. Recurrent abortion [AI 1998]
gias without joint deformity over course of several b. Sterility
years. The best laboratory screening test to diagnose c. Coomb’s positive hemolyticanemia
her disease would be: [A12003] d. Psychosis
a. CD4 lymphocyte count Ref: Harrison 18th/ p2730, 2734, Harrison 19th p 2131
b. Erythrocyte sedimentation rate 85. Lupus anticoagulants may cause all of the following
c. Antinuclear antibody except:
d. Assay for thyroid hormones a. Recurrent abortion [AI 1998]
Ref: H 18th/ p 2726, [ Table: 319.1, Harrison 19th p 2126 b. False + ve VDRL results
78. Joint erosions are not a feature of : [A106 ] c. Increase prothrombin time
a. Rheumatoid arthritis d. Arterial thrombosis Ref: Harrison 19th p 740
b. Psoriasis 86. Bilateral parotid gland enlargement is seen in all of
c. Multicentric reticulo-histiocytosis the following except: [Al 1998]
d. Systemic lupus erythematosus a. Sarcoidosis b. SLE
Ref: Harrison 18th/ p 2728, Harrison 19th p 2131 c. Chronic pancreatitis d. Sjogern's syndrome
79. All of the following are true about SLE except: Ref: Harrison 18th edn, Ch 319, Harrison 19th p 2131
a. Autoimmune HematolyticAnemia [ PCI Dec 06 ] 87 . Indications of steroids in SLE are all except: [A! 1998]
b. Ted ANA a. Myocarditis
-
c. Anti ds DNA b. Endocarditis
d. Raynaud’s phenomenon c. Thrombocytopenia
e. Joint deformity d. Neuropsychiatric symptoms
Ref: Harrison 18th/ p 2728, Harrison 19th p 2131 Ref : CMDT 2009/ 734, Harrison 19th p 2131

Ans. 73. c. SLE 74. d. SLE 75. b. S. L. E. 76. a. Antihistone antibodies


77. c. Antinuclear... 78. d . Systemic lupus ery... 79. e. Joint deformity 80. a. SLE
81. a. SLE 82. b. Penicillin 83. c. Wire loop lesions 84. b. Sterility
170
85. c. Increase proth... 86. b. SLE 87. b. Endocarditis
Rheumatology

88. Low doses of aspirin therapy is essentially advised for 95. Indication of poor prognosis of systemic sclerosis is:
all of the following conditions except: [A! 1997] a. Calcinosis cutis [NBE Pattern 2014 -15]
a . SLE b. Renal involvement
c. Alopecia

B
b. IUGR
d. Telangiectasia
c. Post myocardinal infarction
d. Pre ecclampsia Ref CMDT 2014 ch.20, Pg. 838, Harrison 19th p 2241
Ref: William s 20th/ 701, 849; Harrison 19th p 2131 96. In scleroderma features are all except:
a. Decrease in tone of LES [NBEPattern 2014-15]
Scleroderma b. Restrictive cardiomyopathy
c. Syndactyly
d. Halitosis
89. MCTD includes all except? [ NEET Pattern 2015-16 ]
a. SLE Ref: Harrision's 18th ed.pg 1984, Harrison 19th p 2182
b. Polymyositis 97 . Woman presented with dysphagia and stiff fingers i
c. Rheumatoid arthritis and leather like skin is diagnosed to have?
d. Systemic Sclerosis a. Buergers disease [NBEPattern 2014-15] i
Ref Harrison 18th edn, ch 323, Harrison 19th p 2154 b. Rheumatoid arthritis
c. Sceleroderma
90. All of the following are features of Scleroderma d . Osteoarthrosis
except: [ NBE Pattern 2014 - 15] Ref: CMDT 2014 ch.20, Pg. 838, Harrison 19th p 2164
a. Diffuse periosteal reaction 50
b. Esophageal dysmotility
Sclerema neonatarum affects all except:
a. Palms and soles [ NBE Pattern 2014 - 15] sr
c. Erosion of tip of phalanges b. Skin over thigh
CD
d. Lung Nodular infiltrates c. Skin over chest
C
Ref CMDT 2014 ch. 20, Harrison 19th p 2156 d . Skin over face Ref CMDT 2014 ch.20 3
91. Anti - topoisomerase 1 is marker of:
a. Systemic sclerosis
b. Classic polyarteritis nodosa
[ NBE Pattern 2014-15]
99. Not seen in scleroderma : [ NBE Pattern 2014-15]
a. Anti -scl 70 antibody b. Bi - basiliar fibrosis
c. Prayer sign d. Ischemic stroke
II
* |
c. Nephrotic syndrome Ref CMDT 2013 Pg. 838, Harrison 19th p 2164
d. Rheumatoid arthritis 100. All the following are features of Scleroderma except :
Ref CMDT 2014 ch. 20, Harrison 19th p 2163 a. Dysphagia [AI 1995]
92 . Screening test for sclerodema: [ NBE Pattern 2014 -15] b. Raynaud's phenomenon
-
a. Anti nuclear antibody
b. Ul - Ribonucleoprotein antibody
c. Anti - L.K.M antibody
c. Skin contracture
d . Calcification in all the long bones
Ref Harrison 18th / p 2766 , 2767, Harrison 19th p 1911
I
101. A 35 year old lady complains dysphagia , Raynaud's
d. Anti - topoisomerase antibody phenomenon, sclerodactyly. Investigations show
Ref CMDT 2013 Pg 838 . antinuclear antibody. The likely diagnosis is :
a. Systemic lupus erythematosis [AIIMS June 99]
93. X- ray finding of scleroderma are A/ E:
b. Systemic sclerosis
a. Dilatation due to aperistalsis of oesophagus
c. Mixed connective tissue disorder
b. Pseudo-obstruction [NBE Pattern 2014-15] d . Rheumatoid arthritis
c. Pneumatosis intestinalis Ref Harrison 18th/ p 2757, 2758, Harrison 19th p 2163
d . Subperiosteal elevation
.
Ref CMDT 2014 ch 20, Pg. 838 102 . A 14 year old girl on exposure to cold has pallor of
extremities followed by pain and cyanosis. In later
94. Recurrent aspiration pneumonia caused by: stages of life she is most prone to develop:
a . Dermatomyostis/ polymyositis [ AIIMS Nov 08]
b. Rheumatoid arthritis [ NBE Pattern 2014-15] a. SLE b. Scleroderma
c. Progressive systemic sclerosis c. Rheumatoid Arthritis
d . Systemic lupus erythrematosus d. Dermatomyositis
Ref CMDT 2014 ch.20, Pg. 838 Ref: Harrison 18th/ p 2757, 2762, 2763, Harrison 19th p 1911

Ans. 88. a. SLE 89. d . Systemic Sclerosis 90. a . Diffuse periosteal ... 91. a . Systemic sclerosis
-
92. a. Anti nuclear... 93. d . Subperiosteal elevation 94. c. Progressive systemic... 95. b. Renal involvement
96. c. Syndactyly 97. c. Sceleroderma 98. a . Palms and soles 99. d . Ischemic stroke
100 . d . Calcification in ... 101. b. Systemic sclerosis 102. b . Scleroderma
Marwah's Internal Medicine MCQs ( Bases on Harrison's 19th)

Arthritis 109. All are drugs used in treatment of acute gout except:
a. Allopurinol [NBE Pattern 2014-15 ]
b. Aspirin
103. Investigation of choice during follow up of patient
c. Colchicine
with rheumatoid arthritis: [Manipal 2014]
d. Naproxen
a. X- Ray of joints
b. ESR, Rh factor Ref: CMDT 2014 ch.20, Pg. 812, Harrison 19th p 2233
c. Blood counts 110. A patient aged 40 years having arthritis of PIP and
d. Rh factor, Anti CCP antibodies DIP along with carpo- metacarpal joint of thumb and
sparing of wrist and metacarpo-phalangeal joint, the
Ref: Harrison 19th p 2145 most likely cause is: [NBE Pattern 2014-15]
104. True abut SLE in pregnancy A / E: a. Psoriatic arthritis
a. Increased Anti Ro and La implies low risk for b. Osteoarthritis
congenital heart block c. Rheumatoid arthritis
b. Azathioprine can be continued in pregnancy d. Pseudogout
c. Hydroxychloroquine has to be stopped if breastfeed ¬
Ref Harrison's 18th ed.ch. 15 Pg. 140, Harrison 19th p 2226
ing is planned.
111. Arthritis common with uveitis is:
d. Disease worsens during pregnancy a. RA [NBE Pattern 2014-15]
Ref : Harrison's 18th edn. Harrison 19th p 2133 b. Ankylosing spondylitis

Rheumatolgy
105. An 85- year - old woman presented with bilateral os ¬
c. Still's disease
d. Reiter's disease
teoarthritis of the knees. She had no history of previ¬
ous gastrointestinal disease. Which of the following Ref CMDT 2014 ch.20, Pg. 854, Harrison 19th p 2169
is the most appropriate initial treatment for her? 112. In anklyosing spondylitis joint involvement is least
[JIPMER 2014] in? [NBE Pattern 2014-15]
a. Paracetamol a. Wrist and hand
b. Naproxen b. Sacroiliac joint
c. Celecoxib c. Acromio - clavicular joint
d. Costochondral junction
d. Dihydrocodeine
Ref : Harrison's 18th edn. Harrison 19th p 2226 Ref: Harrison's 18th ed.ch.325, Harrison 19th p 2170
113. Least common site involved in osteoarthritis is:
106. HLA B 27 maximum association is with? a. Hip joint [NBE Pattern 2014-15]
a. Ankylosing spondylitis [AllMS Nov. 14] b. Knee joint
b. Rheumatoid arthritis c. Carpometacarpal joint of thumb
c. Osteoarthritis d. Distal carpophalangeal joint
d. Juvenile rheumatoid arthritis Ref: Harrison’s 18th ed.Pg. 2040, Harrison 19th p 2230
Ref Harrison 18th edn, ch 325, Harrison 19th p 2169 114. The following statements are true regarding acute
gout except: [NBE Pattern 2014-15]
107. Tophi in gout found in all regions except:
a. Acute gout is more common in males
a. Prepatellar bursae [NBE Pattern 2014-15]
b. Serum uric acid is normal in acute gout
b. Muscle c. First metatarsophalangeal joint is most commonly
c. Helix of ear affected in acute gout
d. Synovial membrane d. Treatment with Allopurinol should be started
Ref: CMDT 2014 ch.20, p 812, Harrison 19th p 2233 immediately in the case of acute gout
108. Incorrect about diagnosis of a patient of Gouty arthri¬ Ref: CMDT 2014 ch.20, Pg. 812, Harrison 19th p 2233
tis?
115. All are radiological findings of OA except:
a. High synovial fluid protein [NBE Pattern 2014-15] a. Decreased Joint space [NBEPattern 2014-15]
b. High WBC count b. Osteophytes formation
c. Urate crystal in synovial fluid c. Subchondral sclerosis
d. Normal sugar in synovial fluid d. Deposit of Calcium salts

Ref: CMDT 2014 ch.20, Pg. 812, Harrison 19th p 2236 Ref: Harrison's 18th ed.Pg. 2040, Harrison 19th p 2230
Ans . 103. d. Rh factor, Anti... 104 . a . Increased Anti Ro and 105. a . Paracetamol 106 . a . Ankylosing spondylitis
107 . b. Muscle 108 . a. High synovial fluid... 109. a . Allopurinol 110. b. Osteoarthritis
111. b. Ankylosing spo... 112. a. Wrist and hand 113. d. Distal carpopha... 114 . d. Treatment with Alio...
115. d. Deposit of Calcium salts
Rheumatology

116. Prolonged allopurinol therapy in a patient with gout 124. HLA- DR4 is a marker of: [ NBE Pattern 2014-15]
is NOT indicated for: [NBE Pattern 2014-15] a. Rheumatoid arthritis
a. Acute gouty arthritis b. Sarcoidosis
c. Sero -negative gouty arthritis

B
b. Tophi
c. Urate nephropathy d. Psoriasis Ref CMDT 2013, Pg. 826
d. Evidence of bone / joint damage 125. Type of anemia seen in Rheumatoid arthritis:
Ref: CMDT 2014 ch.20, Pg. 812, Harrison 19th p 2233 a. Normocytic, normochromic [ NBE Pattern 2014-15 ]
117. Reactive arthritis is usually caused by: b. Hyperchromic, Normocytic
a. Shigella flexneri [NBE Pattern 2014-15] c. Hypochromic, normocytic
b. Shigella boydii d. Hypochromic, leucopenia
c. Shigela shiga Ref Harrison's 18th ed.ch.321 p 2738, Harrison 19th p 2138
d. Shigela dysentriae 126. Best treatment for acute gout with kidney
Ref CMDT 2014 Pg. 857, Harrison 19th p 2173 impairment? [NBE Pattern 2014-15]
118. Polyarticular onset JRA involves more than how many a. Allopurinol b. Febuxostat
joints: [NBE Pattern 2014 -15] c. Uricase d. Benzbromarone
a. 3 b. 4 Ref CMDT 2014 ch.20 , Pg . 812 , Harriso n 19 th p 1871 / 2235
c. 5 d. 6 127. All are true about pseudogout except?
Ref: OP Ghai 7th ed.p 600 [NBE Pattern 2014-15]
.
119 C V junction abnormalities are seen in all of the
following except:
a.
[NBE Pattern 2014-15]
Rheumatoid arthritis b. Ankylosing spondylitis
a. Calcium pyrophosphate crystals
b. Most commonly idiopathic
c. Hyperparathyroidism
-e
JO
J3
Q
c. Odontoid dysgenesis d. Basilar invagination d. Most common joint involved is DIP
3
Ref: MERCK MANUAL and Williams wilkins neurosurgery p 2732-2735
120. In rheumatoid arthritis the characteristic joint
involvement is: [NBE Pattern 2014-15]
Ref: CMDT 2014 ch.20 , Pg . 816 , Harriso n 19 th
128. In Seronegative spondyloarthritis, what will cause
maximum reduction in pain and morning stiffness?
p 2235 / 372 e-8t

Q
i
\
[ NBE Pattern 2014-15 ]
a. Spine
b. Knee
a. Aspirin
c. Corticosteroids
b. Indomethacin
d. Infliximab
-S
c. Metacarpophalangeal joint
d. Hip joint Ref: Harrison's 18th ed.ch.325, Harrison 19th p 2172

.
Ref Harrison's 18th ed.ch.319
121 Which of the following regarding rheumatic nodules
is false? [NBE Pattern 2014-15]
129. Rheumatoid arthritis commonly affects the:
a. Cervical spine
b. Thoracolumbar spine
c. Lumbar spine
[ NBE Pattern 2014-15 ]
I
a. Found over extensor surface d. Sacral spine Ref: Harrison's 18th ed.ch.321
b. Tender on palpation
130. Heberden nodes are seen in: [ NBE Pattern 2014 -15]
c. Associated with carditis
a. Rheumatoid arthritis b. Rheumatic arthritis
d. Pea size nodules Ref Harrison's 18th ed.ch.322 c. Osteoarthritis d. SLE
.
122 All of the following can be used to prevent gouty
Ref: Harrison's 18th ed.Pg. 2040, Harrison 19th p 2226f
attack except: [NBE Pattern 2014-15]
a. Allopurinol b. Aspirin 131. Caplan's syndrome is seen with 7 [ NBE Pattern 2014-15 ]
c. Probenecid d. Sulfinpyrazone a. COPD b. Pneumoconiosis
Ref: CMDT 2014 ch.20, Pg. 812 c. Pulmonary edema d. Rheumatoid arthritis

123. Which of the following is the most specific test for Ref: Harrison's 18th ed.ch.321 p 2738, Harrison 19th p 2138
rheumatoid arthritis? [NBE Pattern 2014-15] 132. All joints are involved in acute gout except:
a. Anti- MCV antibody a. MTP [ NBE Pattern 2014-15 ]
b. Anti cardiolipin antibody b. Gleno-humeral joint
c. Anti Mi - 2 antibody c. Ankle joint
d. Anti Ro antibody d. Knee joint
Ref CMDT 2013 Pg. 827 Harrison 19th p 2281 Ref: CMDT 2014 ch.20, Pg. 812, Harrison 19th p 2234

Ans . 116. a. Acute gouty... 117 . a. Shigella flexneri 118. c. 5 119 . b. Ankylosing spondylitis
120 . c. Metacarpo... 121 . b. Tender on palpation 122. b. Aspirin 123 . a. Anti- MCV antibody
124. a. Rheumatoid... 125. a. Normocytic , nor... 126. b . Febuxostat 127 . d . Most common joint...
128. d. Infliximab 129. a. Cervical spine 130. c. Osteoarthritis 131. d . Rheumatoid arthritis
~ r — 13? h ninnorhnmoraLjoint— - -

_

Marwah 's Internal Medicine MCQs ( Bases on Harrison's 19th )

133 . A patient of rheumatoid arthritis develops sudden 140. All of the following are seen in inflammatory
onset Quadriparesis increased muscle tone of limbs polyarthritis, except ’
[ARMS May 94]
with exaggerated tendon jerks and worsening of gait. a . New bone formation
Hj The investigation to be done? [ NBE Pattern 2014 -15] b. Erythema
a. Flexion and extension Cervical area X- ray of neck c. Increased ESR
b. MR1 brain d . Morning stiffness more than one hour
c. EMC and NCV Ref: Harrison 18th/ p 2818, 2819
d . Carotid angiography 141. All the following are true about Rheumatoid arthritis
Ref: CMDT 2013 Pg. 826, H 18th ed„ ch. 15, Harrison 19th p 2145 except: [AI 1994]
a. Positive for Anti - IgG antibody
\ 134. Which is the most common site of subcutaneous b. juxta-articular osteoporosis
nodules in rheumatoid arthritis?
c. Morning stiffness
[ NBE Pattern 2014-15] d. C Reactive protein indicates better prognosis
a . Elbow b. Wrist
Ref: , Harrison 19th p 2145
c. Achilles tendon d. Occiput
142 . A middle aged female presents with polyarthritis,
Ref: Harrison's 18th ed.ch.321, Harrison 19th p 2145
elevated Rheumatoid factor and ANA levels. Which
135. Which of the following is true of psoriatic arthritis? of the following features will help in differentiating
a. Involves distal joints of hand and foot Rheumatoid arthritis from SLE [AIIMS Nov 08]
^
U b. Pencil in cup deformity [ NBE Pattern 2014-15] a. Soft tissue swelling in PIP Joint
} c. Sacroilitis b. Juxta -articular osteoporosis on X ray
! d. All of the above c. Articular erosions on X Ray
Ref: Harrison’s 18th ed.ch.52, p 399, Harrison 19th p 2175 d . Elevated ESR
Ref: Harrison 18th/ p 2728, Harrison 19th p 2145/ 2131
J 136. Not seen in rheumatoid arthritis :
143. Which part of the spine is most commonly affected in
5 [ NBE Pattern 2014 -15]
a . Normal C.R.P b. Juxtaarticular osteopenia Rheumatoid arthritis:
a. Cervical
[AI 1994] [ARMS Feb 97]
c. Cervical myelopathy d. Hyperandrogenism
b. Lumbar
Ref: Harrison's 18th ed.ch.321, Harrison 19th p 2144
c. Thoracic
137 . A young male presents with joint pains and backache d. Sacral
with relief of symptoms on movement/ exercise. The Ref: Harrison 18th/ p 2738, Harrison 19th p 2145
most likely diagnosis is? [NBE Pattern 2014 -15] 144. Which of the following is the most specific test for
a. Rheumatoid arthritis
Rheumatoid Arthritis [ARMS Nov. 06]
b. Ankylosing spondylitis
a. Anti - ccp antibody
c. Poly-articular juvenile arthritis
b. Anti IgM antibody
d . Psoriatic arthropathy c. Anti IgA antibody
Ref : Harrison's 18th ed.ch.325, Harrison 19th p 2169 d. Anti IgG antibody
138. All may be true about Rheumatoid Arthritis except: Ref: Harrison 18th/ p 2825, Harrison 19th p 2145
a. Anti- MCV antibody [ NBE Pattern 2014-15] 145 . Type of anemia seen in Rheumatoid arthritis is:
b. RF positive a. Microcytic hypochromic anaemia [ARMS Dec 97]
c. Anti- CCP antibody b. Macrocytic hypochromic anaemia
- -
d. Anti Mi 2 antibody c. Normocytic hypochromic anaemia
Ref: Harrison' s 18th ed.ch.321, Harrison 19th p 2145 d . Normocytic normochromic anaemia
139. In rheumatoid arthritis pathology starts in: Ref: Harrison 18th/ p 2825, Harrison 19th p 2145
a. Articular cartilage [NBE Pattern 2014 -15] 146 . The following are rheumatoid disease modifying
b. Capsule drugs except: [AI 1995],
c. Synovium a. Chloroquine b. Gold
d. Muscle c. Penicillamine d. BAL
Ref: Harrison’s 18th ed.ch.321, Harrison 19th p 2145 Ref Harrison 18th/ p 2747, 2478, Harrison 19th p 2145
Ans. 133. a. Flexion and ext... 134. a . Elbow 135. d. All of the above 136. d . Hyperandrogenism

a 137 .
141 .
145.
b. Ankylosing...
d . C Reactive pro ...
d . Normocytic nor...
138.
142.
146 .
-
d . Anti - Mi 2 antibody
c . Articular erosions...
d . BAL
139. c. Synovium
143. a. Cervical
140. a . New bone formation
-
144. a . Anti ccp antibody
Rheumatology

147. Indication of systemic steroids in rheumatoid


arthritis is:
a. Mononeuritis multiplex
[At 1997]
Sarcoidosis

154. A woman is admitted with complaints of low-grade


I
b. Carpal tunnel syndrome fever of 6 weeks duration. Chest radiography reveals
c. Presence of deformities bihilar adenopathy with clear lung fields. All of the
d. Articular cartilage involvement following investigations will be useful in differential
Ref: Harrison 18th edn, Ch 321, Harrison 19th p 2145 diagnosis except: [NBE Pattern 2014-15]
a. CD4/ CD8
148. Hemophilia with Rheumatoid arthritis, analgesic of b. Serum ACE levels
choice is: [PG1 Dec 98] c. CECT of chest
a. Ibuprofen
d. Gallium scan
b. Asprin
c. Acetaminophen
Ref: Harrison's 18th ed. ch. 329, Harrison 19th p 2236
d. Phenylbutazone 155. Sarcoidosis is characterized by all except:
a. Cavity [NBE Pattern 2014-15]
Ref: Turek’s 6th/164, Harrison 19th p 91 b. Panda sign
149. Which of the following is not true about JRA7/A / 2009] c. Hilar lymphadenopathy
a. Fever d. Egg shell calcification
b. Rheumatoid nodules Ref Harrison's 18th ed.ch.329, Harrison 19thp 2205

Rheumatolgy
c. Uveitis 156. The most common cause of sudden death in \
d. Raynaud's phenomenon Ref: CSDT 16th / 828 sarcoidosis is: [ NBE Pattern 2014-15] §
a. Pneumonia b. Cor pulmonale
150. All are seronegative [spondyloepiphyseal] arthritis
c. Arrythmias d. Liver failure
with ocular manifestations, except- [AIIMS Nov 01]
a. Ankylospondilitis Ref: Harrison's 18th ed.ch.329, Harrison 19th p 2208
b. Ritter's disease 157. In sarcoidosis: [NBE Pattern 2014-15]
c. Rheumatoid arthritis a. Causes large cavitatory lesions
b. Spontaneous remission may occur
d. Psoriatic arthritis Ref Harrison 19th p 2145 c. Tuberculin test is negative
151. Treatment of choice in seronegative spondylarthritis d. Caseation and necrosis may occur
is: [ARMS Nov 93] Ref Harrison's 18th ed.ch.329, Harrison 19th p 2205
a. Phenylbutazone b. Aspirin
158. Sarcoidosis is least likely to be associated with:
c. Indomethacin d. Corticosteroid
a. Uveitis [NBE Pattern 2014-15]
Ref Harrison 18th / p 2777, 2780, Harrison 19th p 2175 b. Pericardial effusion
152. Which of the organism most commonly causes c. Erythema nodosum
reactive arthritis? ]AIIMS Nov 08] d. Lymphadenopathy
a. Ureaplasma urealyticum Ref: Harrison's 18th ed.ch.329, Harrison 19th p 2207
b. Group A beta hemolytic streptococci 159. Following cranial nerve is involved in patients with
c. Borrelia burgdorferi sarcoidosis: [NBE Pattern 2014-15]
d. Chlamydia a. I cranial nerve b. II cranial nerve
Ref Harrison 18th/ p 2778, 2779, Harrison 19th p 2175 c . Ill cranial nerve d. IV cranial nerve
153. A patient presents with Arthritis, hyperpigmentation .
Ref: Harrison's 18th ed ch.329, Harrison 19th p 2209
of skin and hypogonadism, likely diagnosis is- 160. The primary involvement of which organ is so far not
a. Hemochromatosis [AI 2001 ] reported to be affected by sarcoidosis:
b. Ectopic ACTH secreting tumour of the lung a- Heart [ NBE Pattern 2014-15]
c. Wilson's disease b. Adrenals
d. Rheumatoid arthritis c. Kidney
d. Brain
Ref Harrisons 18th / p 3165, 3166, Harrison 19th p 2516
Ref Harrison's 18th ed.ch.329, Harrison 19th p 2010

Ans . 147. a & b. Monon... 148. c . Acetaminophen 149. d. Raynaud’s phenomenon 150. c. Rheumatoid arthritis
151. c. Indomethacin 152. d. Chlamydia 153. a. Hemochromatosis 154. c. CECT of chest
155. a. Cavity 156. c. Arrythmias 157. b. Spontaneous remis... 158 . b. Pericardial effusion
159. b. II cranial nerve 160 . b. Adrenals

I
Marwah's Internal Medicine MCQs ( Bases on Harrison's 19th )

161. All of the following are features of sarcoidosis 168. An 18 - year-old boy presents with digital gangrene
except: [NBE Pattern 2014-15] in third and fourth fingers for last 2 weeks. On
a. Right paratracheal lymphadenopathy examination the blood pressure is 170 / 110 mm of Hg
|jMj b. Cardiomyopathy and all peripheral pulses were palpable. Blood and
c. Hypercalcemia urine examinations were unremarkable.
Antinuclear antibodies, antibody to double stranded
d . Malabsorption syndrome
DNA and antineutrophil cytoplasmic antibody were
Ref: Harrison's 18th ed.ch.329 Pg. 2805, Harrison 19th p 2211 negative. The most likely diagnosis is: [ Manipal 2014]
J
162 . True about sarcoidosis is: [NBE Pattern 2014 -15]
a. Serum amyloid A is used as marker for sarcoidosis
a . Wegener's granulomatosis
b. Polyarteritis nodosa
b. Kveim test is diagnostic c. Takayasu's arteritis
c. hypocalcemia d. Systemic lupus erythemazosus [SLE]
d. Pleural effusion is common Ref Davidson 17th pg. 921 Harrison 19th p 2187
Ref: Harrison's 18th ed.ch. 329, Harrison 19th p 2209 169. Systemic necrotizing vacuities is a feature of:
| 163. Lupus Pernio is a complication of: a. Wagener's granulomatosis [ Manipal 2014]
a. Sarcoidosis [ NBE Pattern 2014 -15] b. Takayasu's arteritis
b. Skin TB c. PAN
c. SLE complication d. Systemic sclerosis
d. DLE and SLE Ref: George Mathews pg 433 Harrison 19 th p 2187

Rheumatolgy
Ref: Harrison' s 18th ed.ch.329, Harrison 19th p 2209 170. All are true regarding drug induced lupus except:
164. Garland sign on CXR in sarcoidosis involves all except: a . Acetylation makes a change [ Manipal 2014]
a. Right paratracheal nodes b. Anti Ds DNA are rare
[ NBE Pattern 2014-15]
b. Right hilar nodes c. Can resolve spontaneously
d . Renal manifestations are common
c. Left hilar nodes
d . Left pretracheal lymph nodes Ref: Kumar and Clark pg 559 Harrison 19th p 1857
Ref: Harrison' s 18th ed.ch.329, Harrison 19 th p 2208 171. About sarcoidosis all are true except: [ Manipal 2014]
a . Bilateral lymphadenopathy is present
165. Following cranial nerve is most commonly involved in b. Increased angiotensin converting enzyme is present
patients with sarcoidosis: [AIIMS Nov 02] c. Tuberculin is strongly positive
a. II Cranial nerve d . Egg shell calcification on X-ray chest
b. Ill Cranial nerve
Ref Harrison 19 th p 2208
c. VII Cranial nerve
d . IX Cranial nerve 172 . Which is true in porphyria? [ Manipal 2014]
a . Poiphyria cutanea tarda is Autosomal recessive
Ref: Harrison 18th/ p 2809, Harrison 19th p 2212
b. Protoporphyrin oxidase is a cytosolic coenzyme
166 . All are correct regarding sarcoidosis except : c. ALA synthase requires TPP
a . Often cavitate [AIIMS Dec 98] d. Variegate protoporphyrinogen oxidase abnormality.
b. Spontaneous remission is usual Ref: Kumar and Clark pg 1121 Harrison 19th p 2526
c. Tuberculine test is negative
173 . In pseudogout which crystal deposition is seen :
d. B / L hilar lymphadenopathy. a. Calcium pyrophosphate dihydrate
Ref: Harrison 18th/ p 2807, Harrison 19th p 2212 b. Calcium phosphate [ Manipal 2014 ]
c. Calcium bilirubinate
d. Calcium hydroxide + calcium carbonate
Miscellaneous Ref: Kumar and Clark pg 554 Harrison 19th p 372e -8t
174. In giant cell arthritis: [ Manipal 2014]
167 . Drugs use in the treatment for Methotrexate nephro¬ a . Histological diagnosis is based on fragmentation of
toxicity are A/ E: [ Manipal 2014 ] the internal intimal
a . Folinic Acid b. Folic Acid b. C-reactive protein is always raised
-
c. Carboxypetidase G 2 d. Thymidine c. Giant cell is needed for diagnosis
Ref: CMDTpg 1717, 2007 Harrison 19th p 2182 d. Anaemia is a feature
Ref Harrison 19th p 1643, 2189**
..
Ans . 161. d . Malabsorption . 162. a. Serum amyloid.. 163. a. Sarcoidosis 164. d . Left pretracheal lymph...
165. c. VII Cranial nerve 166. a. Often cavitate 167. b. Folic Acid 168. b. Polyarteritis nodosa
176 169. c. PAN 170. d. Renal manifestations... 171. c. Tuberculin is strong...172.d. Variegate pro...
173. a. Calcium pyro.. . 174. c. Giant cell is needed for diagnosis
Rheumatology

follow ing biops y repo rt may be found with 180. Martel's sign is seen in:
175. The
rheumatoid arthritis: [ Manipal 2014] a. Rheumatoid arthritis
a. Posterior scleritis b. Gonococcal arthritis

Bi
b. Loss of goblet cells in the conjunctiva c. Gout
c. Episcleral necrotic tissue d. TB arthritis
d. Vasculitis Ref: Harrison 19 th p 2136 Ref: Harrison's 18th; Harrison 19th p 2233

176 . 20 - year - oldwoman presents with bilateral 181. Drug of choice for low excretors of uric acid in renal
conductive deafness, palpable purpura on the legs failure is: [ Manipal 2014]
and hemoptysis. Radiograph of the chest shows a thin - a. Bromobenzene b. Ailopurinol
walled cavity in left lower zone. Investigations reveal c. Febuxostat d. Probenecid
total leukocyte count 1 2 , 000 / mm red cell casts in Ref: Harrison's 18th pg 3182 Harrison 19th p 1811
the urine and 12,000 /mm serum creatinine 3 ing/ dL.
What is the most probable diagnosis? [ Manipal 2014]
a. Henoch-Schonlein purpura
b. Polyarteritis nodosa
182 . AH are true about systemic sclerosis except:
a. Genetic component
b. Calcinosis cutis
c. Pulmonary artery hypertension
[ Manipal 2014]
i
c. Wegener's granulomatosis
d. Disseminated tuberculosis
Ref: Harrison 19 th p 2182
177 . An 18 - year- old boy presents with digital gangrene
in third and fourth fingers for last 2 weeks. On
d. Anticentromere antibodies
Ref: Harrison's 18th pg 2757 Harrison 19 th p 1471
183. Classical skin manifestation of sarcoidosis are all
except:
J
sr
n
examination the blood pressure is 170/110 mm of
a. Erytherma nodosum
b. Lupus pernio
c
Fig and all peripheral pulses were palpable’ flood and c. Urticaria pigmentosa 3
urine examinations were unremarkable. Antinuclear d. Maculopapular rash
antibodies, antibody to double stranded DNA and Ref: Harrison 19 th p 2205
antineutrophil cytoplasmic antibody were negative.
The most likely diagnosis is: [ Manipal 2014 ] 184. A 20 - year - old woman presents with bilateral
a. Wegener’s granulomatosis conductive deafness, palpable purpura on the le and
b. Polyarteritis nodosa hemoptysis. Radiograph of the chest shows a thin -
c. Takayasu 's arteritis walled cavity in left lower zone. Investi - gations reveal
d. Systemic lupus erythematosus [SLE] total leukocyte count 12000 / mm red cell casts in the
Ref: Harrison 19 th p 2187 urine and 12, 000 / mm serum creatinine 3 ing / dL .
What is the most probable diagnosis? [ Manipal 2014]
178. Concerning Marfan's syndrome: [ Manipal 2014] a. Henoch -Schonlein purpura
a. The condition is inherited in autosomal dominant b. Polyarteritis nodosa
manner c. Wegener's granulomatosis
b. It is caused by defect in the formation of type d. Disseminated tuberculosis
collagen Ref: Harrison 19 th p 2182
c. The lens typically shows upper nasal subluxation
185. 65 - year - old lady underwent mastectomy for
d. The patients have a hi her incidence of hypermetropic
carcinoma breast. Later she developed frontal
refractive error
headache, pain in temple region and around eye.
Ref: Harrison 19 th p 2512
Her ESR is 55 mm / hour. What is the most probable
179. All are true about gout except: [Manipal 2014] diagnosis? [JIPMER 2014]
a. Is caused by purine metabolism disorder a . Cavernous sinus thrombosis
b. Causes scleritis b. Meningeal metastasis
c. Causes tophi in the extraocular muscle tendon c. Frontal sinusitis
d. Is directly related to alcohol consumption d. Giant cell arteritis
Ref: Harrison 19 th p 2233 Ref : Harrison's 18th edn, Harrison 19 th p 523

.
Ans. 175. b. Loss of goblet .. 176. c. Wegener’s granulo... 177. b. Polyarteritis nodosa 178. a . The condition is inherited ..
179. a. Is caused by pur... 180. c. Gout 181 . b. Bromobenzene 182. a Genetic component
183. c. Urticaria pig ... 184. c. Wegener ’s granulo... 185. d . Giant cell arteritis
Marwah 's Intern al Medic ine MCQs ( Bases on Harris on 's 19 th )

186. The class of lupus nephritis with worst prognosis is: 193. Woman of 30 - years with Raynaud’s phenomenon ,
a. Class 2 polyarthritis, dysphagia of 5 -years and mild
b. Class 3 Sclerodactyl, blood showing Anti -centromere
c. Class 4 antibody positive, the likely cause is:
d . Class 5 a. CREST [ NBE Pattern 2014 15]-
fM Ref: Harrison's 18th, Harrison 19 th p 1838, 2126 b. Mixed connective tissue disorder
c. SLE
I 187. All are true about Berger’s disease except: d. Rheumatoid arthritis
a. Mesangial proliferation [PG1 May 2013] Ref: Harrison's 18thed.ch.319, Harrison 19th p 2155
b. Increased polyclonal IgA
c. IgA, C3 and lgG deposites in the measangium 194. Bilateral Painless parotid enlargement is seen in all
d. Hematuria may be gross or microscopic except: [NBE Pattern 2014 15] -
e. Absence of proteinuria is pathognomic a. Mumps
b. Alcoholics
Ref: H 18/ e p2069, 2070, Harrison 19 th p 1645 - 1646, 1649
c. Sarcoidosis
188. APLAS syndrome what treatment should be given to d. Diabetes mellitus Ref: CMDT 2014 ch.32, p 1370
pregnant female to avoid abortion? [ AllMS Nov. 14 ] 195 . A patient of chronic Left lung abscess with generalized
a. 1VIG edema, hypoproteinemia, hepato-splenomegaly
b. Plasmapheresis without renal failure but reduced urine output.

Rheumatlogy
c. Aspirin + LMW heparin Diagnosis is?
d. Progesterone injections a. Amyloidosis [NBE Pattern 2014-15]
Ref: Harrison 18th edn, Reference: ch 320, Harrison 19th p 2136 b. Chronic cor-pulmonale with Rt. Heart failure
c. Bronchiectasis
189. DOC for acute attack of Hereditary angioneurotic
d. Bronchogenic carcinoma
edema? [NEETPattern 2015 16] - .
Ref: Harrison's 18th ed ch.112, Harrison 19th p 732
a. Danazol
b. Cl inhibitor concentrate 196. Incorrect about Behcet’s syndrome is:
c. Icatibant a. There is a strong association with HLA- B 7
d . methylprednisolone Ref: Harrison 19th p 1874 b. The skin may be hyperactive to minor injury such as
venipuncture [NBE Pattern 2014-15]
190. Type 5 Hypersensitivity mimics![ NEETPattern 2015-16] Inflammatory reaction around large blood vessels
a. Type 1 d . Cortiocosteroid therapy is of definite value
b. Type 2
Ref: HI 8th ed.ch.327, Pg . 2801, Harrison 19th p 2194
c. Type 3
197 . All are true about Henoch Schonlein purpura except:
d . Type 4 Ref: Harrison 19th p 232
a. Raised IgA [ NBE Pattern 2014-15]
191. Which of the following disorders is least likely b. Hematochezia
associated with progression to lymphoma? c. Thrombocytopenia
a. Sjogren’s syndrome [ NBE Pattern 2014 15] - d . joint pain
b. Ataxia telangiectasia Ref: CMDT 2014 ch.20, Pg . 852, Harrison 19th p 2190
c. Severe combined immunodeficiency 198. Neuropathic joint is seen in all except:
d . Lynch II syndrome a. Diabetes mellitus [NBEPattern 2014-15]
Ref: CMDT 2013, p 831, Harrison 19 th p 563 b. Tabes dorsalis
192. A young female is suffering from , recurrent abortions c. Syringomyelia
and thrombosis of deep veins, thrombocytopenia, d. Frederich's ataxia
and a recent Ml . The most likely diagnosis is: Ref: Harrison's 18th ed.ch. 336 , Harrison 19th p 2629
[NBE Pattern 2014- 15]
199. Most common cause of unilateral Hilar lymphadenop -
a. Catastrophic anti - phosphilpid antibody syndrome
b. Primary anti-phospholipid antibody syndrome
athy:
a. Histoplasmosis b. Sarcoidosis
-
[ NBE Pattern 2014 15]

c. Tip
c. Aspergillosis d. Tuberculosis
d. Protein C deficiency Ref: Harrison's 18th ed. ch.165, Harrison 19th p 1102
.
Ref : Harrison's 18th ed ch 319, Harrison 19th p 2135

.. 188. c. Aspirin + LMW... 189. b. C1 inhibitor concentrate


I Ans . 186. c. Class 4
190. b. Type 2
194. a. Mumps
187.
191.
195.
e. Absence of protein
c. Severe combined...
a. Amyloidosis
192. b. Primary anti - phos... 193. a. CREST
196. a. There is a strong ass...197. c. Thrombocytopenia
1 7»
198. d. Frederich ’s ataxia 199. d . Tuberculosis
Rheumatology

200. Keratoderma - Blenorrhagicum is pathognomonic of: 207 . A young man develops tiny linear wheals on exposure
[ NBE Pattern 2014-15] to sun and exercise since 6 years. Most likely diagnosis
a. Behcet's disease is: [ NBE Pattern 2014-15]
b. Reiter's disease a. Cholingeric urticaria
c. Lyme's disease b. Dermatographism
d. Glucagonoma c. Idiopathic chronic urticaria
Ref: CMDT 2014 ch.20, Harrison 19th p 2174 d. Pressure urticaria
201. Recurrent oro -genital ulceration with arthritis is Ref: Harrison's 18th ed.ch.317, Pg. 2711
seen in: [ NBE Pattern 2014-15]
208. Which of the following diabetes is associated with
a. Behcet's syndrome b. Gonorrhoea
c. Reiter's syndrome d. Syphilis HLA? [ NBE Pattern 2014 15]-
a. Type I
Ref: Harrison's 18th ed.ch.327, Pg . 2801, Harrison 19th p 2194
b. Type II
202 . Following is characteristic neurologic finding in c. Fibro-calcific Diabetes mellitus
primary amyloidosis: [NBE Pattern 2014-15] d. MODY Ref: CMDT 2013, Pg. 1193
a. Peripheral motor and sensory neuropathy
b. Peripheral neuropathy associated with cerebral 209. The most common primary immunodeficiency is:
manifestation [NBE Pattern 2014- 15]
c. Guillain -Barre type of syndrome a. Common variable immunodeficiency

Rheumatolgy
d. Spinal cord compression in thoracic region b. Isolated IgA immunodeficiency
c. Wiskott-Aldrich syndrome
.
Ref: CMDT 2014 ch 13, Pg. 530
d. AIDS Ref: CMDT 2013, p 868
203 . Regarding Henoch Schonlein purpura all are true
except: [NBE Pattern 2014-15] 210. Antinuclear antibodies are seen in A/ E:
a. Associated with glomerulonephritis a. Systemic sclerosis [ NBE Pattern 2014-15]
b. Non- Palpable purpura b. Morphea
c. Decreased complement c. Pemphigus vulgaris
d. Normal platelet count d. SLE Ref: Harrison's 18th ed., Pg. 1961
Ref: Harrison's 18th ed.ch.326, Harrison 19 th p 2190
211. Earliest valvular lesion in acute rheumatic fever:
204. Not Seen in Behcet's syndrome is:
a. Mitral stenosis [NBE Pattern 2014- 15]
a. Pyoderma gangrenosum [NBE Pattern 2014-15]
b. Mitral insufficiency
b. Thrombophlebitis
c. Aortic stenosis
c. Gians penis Apthous ulceration d. Aortic regurgitation
d . Panuveitis
Ref Harrison's 18th ed.ch.322 p 2752
.
Ref: Harrison's 18th ed ch.327, Harrison 19th p 2194
205. About fibromyalgia all are true except? 212 . The most characteristic murmur of rheumatic carditis
a Associated with EEG abnormalities [NBE Pattern 2014-15] is: -
[ NBE Pattern 2014 15]
b. More common in males than females a. Apical high-pitched early diastolic murmur
c. Associated with low free cortisol levels b. Apical high - pitched holosystolic murmur
d . Associated with decreased blood flow to brain c. Apical low-pitched mid -diastolic murmur
Ref: CMDT 2013, Pg. 822, Harrison 19 th p 2238 d. Systolic ejection murmur along the left stenal border
206. Photosensitivity is a feature of porphyria. All the Ref Harrison's 18th ed. ch.322, Harrison 19th p 2150
following enzyme deficiencies have photo - sensitivity 213. In acute intermittent Porphyria, the metabolite which
except? is elevated is: [NBE Pattern 2014-15]
a. HMB synthase deficiency [ NBE Pattern 2014-15] a. Uroblinogen
b. Uroporphyrinogen decarboxylase deficiency b. Porphoblinogen
c. Protoporphyrinogen oxidase deficiency c. Coroporphyrin
d . Coproporphyrinogen oxidase deficiency d. Protoporphyrin
Ref: Harrisons's 18th ed.P- 3168 Ref Harrison's 18th ed.ch.358

Ans . 200. b. Reiter’s disease


204. c. Gians penis...
201. a . Behcet’s syndrome 202. a. Peripheral motor... -
203. b. Non Palpable purpura
205. b. More common in mal... 206. a. HMB synthase... 207. a. Cholingeric urticaria
208. a . Type I 209. a. Common variable... 210 . c. Pemphigus vulgaris 211. b. Mitral insufficiency
212. c. Apical low-pit... 179
213. b. Porphoblinogen

1
I
Marwah's Internal Medicine MCQs ( Bases on Harrison's 19"’ )

214. Consider the following dinicopathological changes in 222 . A lady with recurrent abortions and isolated
the Rheumatic Carditis: [NBE Pattern 2014-15] prolongation of APTT. Which one of the following
i. Mitral regurgitation ii. Aortic regurgitation gives positive result? [ NBE Pattern 2014-15]
iii. ASLO rise iv. Left atrial enlargement a. Prothrombin time
b. Dilute Russell Viper Venom Time test
The correct chronological sequence of these events is
b. i, ii, iii, iv c. Bleeding time
a . iii, i, iv, ii
d. iii, iv, i, ii
d. Clot solubility test
c. iv, ii, i, iii
Ref: Harrison's 18th ed., Ch 322
Ref CMDT 2013/ p836, H 18th / p2736,2737, H 19th/ p 2791
215. Lardaceous spleen is seen in: [ NBE Pattern 2014-15] 223. A patient suffering from lung abscess develops
a. Alcoholic hepatitis b. Chronic active hepatitis anasarca. The probable reason is:
c. Focal amyloidosis d. Diffuse amyloidosis a. Cor-pulmonale [NBE Pattern 2014-15]
Ref: Robbins 8th ed. Pg. 166 -172 b. Good Pasture syndrome
216. The defective platelet function is seen in all except: c. Pyelonephritis
a. SLE [ NBE Pattern 2014-15] d. Acute renal failure
b. Acute lymphoctic leukemia Ref CMDT 2014 ch.10, Pg. 425, Harrison 19th p 2632
c. Myelofibrosis 224. Kawasaki's disease has the following features except:
d. Henoch-Scholein purpura a. Coronary artery aneurysm [ NBE Pattern 2014-15]
Ref Harrison's 18th ed. ch.14 b. Conjunctival suffusion

Rheumatlogy
217. Not associated with thymoma? [ NBE Pattern 2014-15] c. Thrombocytopenia
a. Red cell aplasia d. Desquamation of the skin of fingers and toes
b. Myasthenia gravis Ref: CMDT 2014 ch.32, Pg. 1417, Harrison 19 th p 2179
c. Hypergammaglobulinemia 225. The presence of small sized platelets on the peripheral
d. Compression of the mediastinum smear is characteristic of: [ NBE Pattern 2014-15]
Ref: Harrison' s 18th ed. ch.e20, Pg. 839 a. Idiopathic thrombocytopenic purpura
218. HLA- B*1502 is a genetic marker for: b. Bernard Soulier syndrome
a. Systemic lupus erythematosus c. Disseminated intravascular cogaulation
b. Polyarteritis nodosa [NBE Pattern 2014-15] d. Wiskott Aldrich syndrome
c. Steven Johnson syndrome Ref Harrison's 18th ed. ch.316, Pg. 270, Harrison 19 th p 89e-3f
d. Seronegative spondy-arthritis syndrome 226. A boy of 16 years suddenly develops recurrent early
Ref: Harrison' s 18th ed. Pg. 840 -844 morning jerky movements of hands. ASO titer is non-
219. The most common cardiac involvement in rheumatoid detectable. Most likely cause is:
arthritis: [ NBE Pattern 2014-15] a. Syndenham's Chorea [NBE Pattern 2014-15]
a. Cardiomyopathy b. Pericarditis b. Huntington chorea
c. Myocarditis d. Endocarditis c. Febrile convulsions
Ref Harrison's 18th ed. ch.321 d . Myoclonic epilepsy
Ref Harrison' s 18th ed. ch.369, Harrison 19 th p 2544
220. Tietze's syndrome usually develops at
[ NBE Pattern 2014-15] 227 . Features of rheumatic carditis are all except?
costal cartilage:
a. Aschoff nodule [ NBE Pattern 2014- 15]
a . First and Second ribs b. Commisural involvement
b . Second to fifth ribs
c. PR segment depression
c. Sixth to Eighth
Ref: Harrison's 18th ed. Pg 2060 . d . Fourth heart sound heard
d . All seven ribs
.
Ref: Harrison’s 18th ed ch.322, Harrison 19 th p 2150-51
221. True in amyloidosis is all except:[ NBE Pattern 2014 - 15]
228. Treatment of Kawasaki disease in children is?
a . Multiple myeloma AL type- a. Oral steroids [ NBE Pattern 2014-15]
b. Secondary amyloidosis -AA type
b. IV steroids
c. Renal amyloidosis is commonly present with hyper ¬

tension
c. IV Immunoglobuin
d. Mild proteinuria
d . Mycophenolate mefentil
Ref CMDT 2014 ch.13, Pg . 530, Harrison 19 th p 732 Ref: Harrison' s 18th ed. ch.326, Pg. 2900; Harrison 19th p 2193

Ans. 214. None 215. a. iii , i , iv, ii 216. d . Henoch -Scholein ... 217. c. Hypergammaglobulinemia
218. c. Steven John ... 219. b. Pericarditis 220. b. Second to fifth ribs 221. c. Renal amyloidosis ...
222. b. Dilute Russell... 223. a. Cor- pulmonale 224. c. Thrombocytopenia 225. d . Wiskott Aldrich syndrome
180
226 . ..
d . Myoclonic epil . 227. d . Fourth heart sound ... 228. c. IV Immunoglobuin
Rheumatology

229. Charcot’s joint in diabetes affects commonly: 237 . Recurrent giardiasis is a feature of:
a. Shoulder joint
b. Knee joint
c. Hip joint
[ NBE Pattern 2014- 15] a. C 3 deficiency
b. Cl inhibitor deficiency
c. Di Georges syndrome
[ NBE Pattern 2014 -15]
I
d. Tarsal joint
Ref: CMDT 2014 ch.27, Harrison 19th p 2243
230. Ataxia telangiectasia is characterized by all of the
following except?
a. Autosomal dominant inheritance
b. Oculocutaneous telangiectasia
c. Cerebellar ataxia
[ NBE Pattern 2014 - 15]
d. Common variable immunodeficiency
Ref CMDT 2013 Pg . 868, Harrison 19th p 270
238. Charcot's joints in diabetes mellitus include typically:
a. Interphalangeal joints
b. Tarsometarsai joint
c. Cuneonavicular
d. Talocrural
[ NBE Pattern 2014 15] - B
d. Thymic hypoplasia
Ref: CMDT 2013 Pg. 826 , Harrison 19th p 2109 Ref: Harrison's 18th ed.ch.344, Pg. 968, Harrison 19th p 2244
231. Schober's sign is used to evaluate: 239. Clinical manifestation of Felty’s syndrome are all
a. Flexion of lumbar spine except: [ NBE Pattern 2014-15]
[ NBE Pattern 2014-15]
b. Chest expansion a. Rheumatoid arthritis
c. Pain with motion of hip b. Splenomegaly
d. Neck pain and stiffness c. Neutropenia

Rheumatolgy
Ref Harrison's 18th ed. ch.325, Harrison 19th p 2170 d. Nephropathy
232. Tufting of distal phalanx is characteristic of:
.
Ref: Harrison' s 18th ed. ch 321, Harrison 19th p 372e-15t
a. Psoriatic arthropathy [NBE Pattern 2014-15] 240. A 70 - year -old male presents with left sided headache
b. Gout and generalized aches and pains of three months
c. Hyperparathyroidism duration. The referring doctor has highlighted the
d . Hypoparthyroidism remarkably elevated ESR and alkaline phosphatase.
Ref Harrison's 18th ed. ch.353, Harrison 19th p 1815 The most likely diagnosis is? [ NBE Pattern 2014-15]
233. Heerfordt’s syndrome consists of fever, parotid a . Multiple myeloma
nlargement, facial palsy and: [ Bihar PG 2014] b. Disseminated carcinoma prostate
a. Arthralgia b. Bilateral hilaradenopathy c. Paget's disease
c. Erythema nodosum d. Anterior uveitis d. Temporal arteritis
Ref Harrison: sarcoidosis, ch. 329 Ref CMDT 2014 ch.26, Pg. 1152, Harrison 19th p 426e-2
234. The organism causing osteomyelitis in sickle cell 241. Important feature in Henoch Schonlein purpura?
anemia: [NBE Pattern 2014-15] . a. Raised IgA [NBE Pattern 2014 -15]
a. Salmonella b. Staphylococcus b. Membranous glomerulonephritis
c. H. influenzaed . E. coli c. Absent radial pulse
.
Ref: CMDT 2014 ch 12, Pg. 503, Harrison 19th p 840t d. Aneurysm of branching point
235. Lilac coloured [heliotrope] pigmentation over the Ref Harrison' s 18th ed. ch.326 , Harrison 19th p 2190
face is characteristic of: -
[NBE Pattern 2014 15] 242. Cortiocosteroid are used in all except:
a. Dermatomyositis a. Lupus nephritis [ NBEPattern 2014-15]
b. Polymyositis b. Lupus cerebritis
c. Systemic lupus erythematosus c. Libman sacks endocarditis
d. Systemic sclerosis d . Lupus pernio
Ref: CMDT 2013 Pg. 840, Harrison 19th p 2202 Ref Harrison's 18th ed. ch.319, Harrison 19th p 2129
236. Good's syndrome is: [NBE Pattern 2014-15] 243. The most reliable investigation in amyloid disease is:
a. Thymic hyperplasia with myasthenia gravis a. Rectal biopsy [ NBE Pattern 2014- 15]
b. Thymic hypoplasia b. Immunoglobulin assay
.
c Thymoma with hypo-gammaglobulinemia c. Ultrasound
d . Thymoma with anti -basement membrane anti ¬
d . Abdominal fat pad biopsy
bodies
Ref: Harrison's 18th ed. ch.e20, Pg. 839, Harrison 19th 1839 Ref: Harrison's 18th ed.ch.112, Harrison 19th p 723

Ans. 229. d . Tarsal joint 230. a . Autosomal dominant... 231. a. Flexion of lumbar... 232. c. Hyperparathyroidism
233. d . Anterior uveitis 234. a. Salmonella 235. a. Dermatomyositis 236. c. Thymoma with hypo...
237. d. Common vari... 238. b. Tarsometarsai joint 239. d . Nephropathy 240. c. Paget’s disease 181
241. a. Raised IgA 242. c. Libman sacks... 243. d . Abdominal fat pad biopsy
I

— —
th
MCQs ( Bases on Harrison ’s 19 )


Marwah 's Internal Medicine
I L -

-
252. Intra cardiac calcifications indicate:
! 244. Best drug for bradykinin mediated Angioedema: a SBE [NBE Pattern 2014-15]
a. Icatibant [NBE Pattern 2014 -15]
b. Rheumatic valves
b. Levocetrizine c. Old MI
WKM d . Chronic pericarditis
|
u d. Hydrocortisone Ref: Harrison' s 18th ed.ch.322, Harrison 19 th p 2138
Ref: Harrison' s 18th ed.ch.317, Harrison 19 th p 2117
253. Sicca syndrome is associated with all except:
I 245 . HLA DRw 52 is associated with : [ NBE Pattern 2014 - 15] a. Rheumatoid arthritis [NBE Pattern 2014 - 15]
a. SLE b. Scleroderma b. Midline granuloma
c. Sjogren d. Behcet c. Sarcoidosis
Ref: CMDT 2013 Pg. 842, Harrison 19th p 1857 d. Chronic active hepatitis
Ref: Harrison's 18th ed. p 1991, Harrison 19 th p 2167 t
246. Negatively Birefringent crystals in urine is seen with:
a. Phosphaturia [ NBE Pattern 2014-15] 254. A 24 year old male presents with abdominal pain,
b. Uricosuria rashes, palpable purpura and, arthritis. The most
c. Cystinuria probable diagnosis is. [AI 08]
d . Struvite stones a. Henoch Schonlein Purpura [HSP]
Ref: CMDT 2014 ch.20, Pg. 812, Harrison 19th p 431 e-5 b. Sweet syndrome
c. Meningococcemia
247 . Amyloidosis occurs in all except: [NBE Pattern 2014 -15] d . Hemochromatosis
a. Tuberculosis b. Chronic bronchitis Ref: Harrison 18th Ch 326, Harrison 19 th p 1839
c. Lung abscess d . Bronchiectasis
255. A 5 - year- old child presents with non - blanching
o Ref: Harrison's 18th ed. ch.165
purpura over the buttocks and lower limbs along with
P8 248. Which of the following can be used for confirmation colicky abdominal pain. Further evaluation revealed

s3 of anaphyalxis:
a. IgE levels
[ NBE Pattern 2014 - 15]
b. Basophil count
deposition of IgA immune complexes. The most likely
diagnosis is:
a. Henoch Shonlein Purpura [AIIMS 2011]
0) c. Eosinophil count d. Serum tryptase
b. Kawasaki Disease
XI Ref: CMDT 2013 Pg. 866
c. Wegner’s Granulomatosis
OC 249. Steroids are given in rheumatic fever when there is: d . Takayasu Disease
a . Carditis [ NBE Pattern 2014 -15] Ref: Harrison 18th edn, Ch 326, Harrison 19 th p 1839
b. Subcutaneous nodules
256. All of the following are true about HSP, Except:
c. Chorea
d . All of the above
a. Palpable Purpura [ PCI Dec 2010]
b. Kidney's commonly affected
Ref: Harrison' s 18th ed. ch.322, Harrison 19 thp 2153 c. ANCA Negative
d. Thrombocytopenia
250. A Carey- Coomb's murmur heard in a child with
multiple joint pains is suggestive of: Ref: Harrison 18th edn, Ch 326, Harrison 19 th p 1839
a. Infective endocarditis [ NBE Pattern 2014-15] 257. One of the following is a characteristic of Henoch -
b. Rheumatoid arthritis Schonlein Purpura: [A12002]
c. Rheumatic fever a. Blood in stool
d. Libman-Sacs endocarditis b. Thrombocytopenia
Ref: Harrison' s 18th ed.ch.321 p 2752, Harrison 19 th p 2153 c. Intracranial hemorrhage
d. Susceptibility to infection
251. Most common lymphoma associated with Sicca Ref: Harrison 18th edn, Ch 326 , Harrison 19th p 1839
syndrome is: [NBE Pattern 2014-15]
a. MALToma 258. Henoch - Schonlein Purpura is characterized by the
b. Burkitt lymphoma deposition of the following immunoglobulin around
c. DLBCL the vessels : [AIMS Nov. 05]
d. Lymphoplasmacytic lymphoma a
c
--
IgM
IgA
b. IgG
d . IgE
Ref: CMDT 2014 ch.324, Harrison 19th p 2167 t
.
Ref Harrison 18th/ p 2797, Harrison 19th p 1839

L 182
Ans . 244.
248.
252.
a . Icatibant
d. Serum tryptase
b. Rheumatic valves
245. c. Sjogren
249. a . Carditis
253. b. Midline granuloma
246.
250.
254.
b. Uricosuria
c. Rheumatic fever
a. Henoch Schonlein ...
247. b . Tuberculosis
251. a . MALToma
255. a. Henoch Shonlein Purpura
256. d. Thrombocy... 257. a . Blood in stool 258. c. IgA
Rheumatology

are features of Behyet’s syndrome c. Neutropenia


259. All the following
except : [ Ml 994 ] d. Nephropathy
a. Recurrent aphthous stomatitis Ref: Harrison 18th/ p 240, Harrison 19 th p 411 / 418
b. Multi -system involvement 265. All of the following may be associated with Sjogron
c. Seen only in the tropics syndrome, except : [ARMS Dec 95]
d. Common in youngsters a. Dry eyes
Ref: Harrison 18th/ p 2801 , Harrison 19th p 2194 b. Dry mouth
260. Recurrent Bilateral Hypopyon formation associated c. Parotid gland enlargement
with thrombophlebitis is most consistent with which d. Systemic manifestations
of the following: [AI 2009] Ref: Harrison 18th/ p 2770, 2771, Harrison 19 th p 1857
a. HLA B 27 associated uveitis 266. Sicca syndrome is associated with all Except :.
b. Behcet's syndrome a. Midline granuloma [AIIMS Feb 97]
c. Syphilis b. Chronic active hepatitis
d . Herpes Zoster c. Rheumatoid arthritis
Ref: Harrison 18th edn, Ch 327, Harrison 19 th p 2194 d. Scleroderma
261. 30 - year old Basanti presents with light brown lesions Ref: Harrison 18th / p 2770 [Table: 324.1, Harrison 19th p 2167t
involving both her cheeks. The lesions had never
267. All of the following statements about hereditary

Rheumatolgy
been erythematous. Which of the following is the hemochromatosis are true Except [AI 2008]
most probable diagnosis: .
[ARMS Nov 2000]
a. Arthropathy involving small joints of hands may be
a. SLE b. Chloasma
seen
c. Air borne contact dermatitis
b. Skin pigmentation is a frequent presentation
d. Photo sensitive reaction Ref: Behl/ 298,
c. Desferroxamine is the treatment of choice
262. All ofthefollowingstatementsaboutAntiphospholipid d. Hypogonadism may be seen
Antibody Syndrome [APLAb] are true, Except : Ref: Harrison 18th/ p 3166, Harrison 19th p 2516
a. Single titre of Anticardiolipin is diagnostic
268. All are seen in hemochromatosis except- [AI 2008]
b. Commonly presents with recurrent fetal loss
[AI 2010] a. Hypogonadism
c. May cause pulmonary hypertension
b. Arthropathy
d. Warfarin is given as treatment
c. Bronze diabetes
Ref: Harrison 18th/ p2734, Harrison 19th p 740
d. Desferrioxamine is the treatment of choice.
263 . Which of the following is recommended in a woman Ref: Harrison 18th/ p 3166, Harrison 19 th p 2516
with Antiphospholipid Antibodies and history of prior
[AI 2010] 269 . A Ten year old boy presents to the pediatric
abortions / still birth.
emergency unit with seizures. Blood pressure in
a. Aspirin only
the upper extremity measured as 200 / 140 mm Hg.
b. Aspirin + Low molecular weight Heparin
Femoral pulses were not palpable. The most likely
c. Aspirin + Low molecular weight Heparin +
diagnosis amongst the following is: [AI 2010]
Prednisolone.
a . Takayasu Aorto arteritis
d. No Treatment
b. Renal parenchymal disease
Ref: Harrison 18th/ p 2734, Harrison 19th p 2135t
c. Grandmal seizures
264. True regarding Felty’s syndrome is all, EXCEPT: d. Coarctation of Aorta
a . Splenomegaly [AIIMS Dec 97] Ref: Nelson's 18th/ 1900, 1901; Harrison 19 th p 1525
b. Rheumatoid arthritis

Ans . 259. c. Seen only in ... 260. b. Behcet’s syndrome 261. b. Chloasma 262. a . Single titre of Anticard ...
263. b. Aspirin + Low... 264. d. Nephropathy 265. d . Systemic manife... 266. a . Midline granuloma
267. c. Desferroxamine... 268. d . Desferrioxamine is the... 269. d . Coarctation of Aorta 183
! Kidney
CHAPTER
8
GFR categories ( ml / min /1.73 m 2 ) G1 Normal or High > 90
description and range

G 5 Kidney failure <15

Persistent albumuria
Prognosis of CKD by GFR and albuminuria categories:
A1 A3
KDIGO 2012
Normal to mildly increased Severely increased
<30 mg /g < 3 mg / mmol >300 mg /g > 30 mg / mmol

Most Recent Q’s 2014- 15 AKI

1. Most common nephropathy associated with malig ¬ 6. A 50 - year- old patient develops cardiogenic shock fol ¬
nancy? (AIIMS May 2015) lowing acute myocardial infarction. His urine output
a. Membranous b. MCD decreases in next few days. He has increased serum
c. IgA d. FSGS urea and creatinine. Urine analysis reveals no glucose
Ref: Harrison 19 th p 1843, or protein but numerous hyaline casts present. After
few days he develops polyuria and serum creatinine
2. What is the most common extrarenal manifestation
levels fall. Histopathology of renal biopsy in this pa ¬
of adult polycystic kidney disease? ( NBE Pattern 2014 )
tient would reveal: (JIPMER 2014 )
A. Hepatic cysts B. Berry aneurysms
a . Immune complex
C. MVP D. Colonic diverticulosis
b. Glomerular crescents
Ref Harrison 19 th p 1851 -52
c. Patchy tubular necrosis
3. The earliest change in diabetic nephropathy is glomer ¬ d. Messangial deposits
ular hyperperfusion. But the earliest clinical evidence Ref : Harrison 19 th p 1759
is microalbuminuria. Microalbuminuria is defined as
the loss of protein in urine at the rate of: 7. A 42 - year-old female presents with diazepam and al ¬
( NBE Pattern 2014 ) cohol overdose . She is comatose, temperature is 34.5
A. < 30 mg/ day B. 30 - 300 mg/ day degree Celsius, BP - 100 / 80 mm of Hg, creatinine is
C. 300 - 300 mg/ day D. > 3 gm / day 2.42, AST - 500, GGT - 35 IU. Urine dipstick showed 3 +
blood buturinanalysis was normal . USG abdomen was
Ref: Harrison 19 th p 1813
normal . What is the most likely diagnosis?
4. Chronic Graft versus Host disease (GVHD) is disease (JIPMER 2014)
occurring after how many days? ( NBE Pattern 2014) a. Hypothermia b. Dehydration
A. 1 month B. 3 month c. Rhabdomyolysis d. Paracetamol overdose
C. 4 month D. 7 days Ref : Harrison 19 th p 1804, 1805, 1806 t
Ref Harrison 19th p 139e- 4
8. Criterion for prerenal failure: (WB PG 08)
5. Which of the following is used for treatment of Hyper¬
a. Fractional excretion index > 1
viscosity in Waldenstrom's macrogiobulinemia?
b. Urine sodium concentration > 10 mmol / L
( NBE Pattern 2014 )
A. Plasmapheresis c. Serum BUN : creatinine > 20:1
B. Phlebotomy'
C. Steroids D. Hydroxyurea d . Urine osmolality < 500
Ref CMDT 2008 p788, Harrison 19 th p 1800
Ref Harrison 19th p 718
Ans. 1. a . Membranous 2. a . Hepatic cysts 3. b. 30-300 mg/ day 4. b. 3 month
5. a . Plasmapheresis 6. c. Patchy tubular necrosis 7. c. Rhabdomyolysis 8. c. Serum BUN : creatinine...
Kidney

9. In acute tubular necrosis all are true except? 16 . Chronic hemodialysis in ESRD patient is done?
a. Specific gravity of urine<1.020 ( Kerala PC 10) a. Once per week ( NEET Pattern 2015-16 )
b. Urine 0smolality> 500 b. Twice per week
c. BU creatinine ratio < 20 c. Thrice per week
d. Urine sodium < 20 mmol / L d. Daily
Ref Harrison 19th p 102 Ref Harrison's 18th ed. Ch. 281, Harrison 19th p 1822
10. Which of the following changes does not occur in ma ¬ 17 . Hemodialysis can be performed for long periods from
lignant hypertension: ( AI 2008) the same site because? ( NEET Pattern 2015-16 )
a . Peticheal Haemorrhages on cortical surface a. Arteriovenous fistula reduces bacterial contamina ¬
b. Fibrinoid necrosis of arterioles tion of site
c. Intimal concentric thickening b. Arteriovenous fistula results in arterialization of
d. Hyaline arteriosclerosis vein
Ref: Harrison's 18/ e p2047, Harrison 19th p 1612 c. Arteriovenous fistula reduces chances of graft fail ¬

11. Renal artery stenosis may occur in ail of the following ure
except: d . Arteriovenous fistula facilitates small bore needles
( A12006 )
a. Atherosclerosis for high flow rates
b. Fibromuscular dysplasia Ref Harrison's 18th edn, Ch 281, Harrison 19th p 1822
c. Takayasu's arteritis 18. The most common neurological disorder seen in CRF
d. Polyarteritis nodosa patients: ( NBE Pattern 2014 -15) \
Ref: Harrison’s 18/ e p2375, 2796, Harrison 19th p 1618 a. Dementia
12 . Plasma urea / creatinine ratio of 20:1 may be seen in: b. Peripheral neuropathy
a . Rhabdomyolysis
b. Ureteric calculi
c. Pre-renal failure
d. Chronic glomerulonephritis
( A12010)

Ref Harrison' s 18/ e p337, 2302, Harrison 19 th p 291


13. A girl aged 8 years has been admitted for dialysis. She
c. Restless leg syndrome
d . Encephalopathy
Ref Harrison's 18th ed. ch. 281, Harrison 19 th p 1811
19. Hypochromic- microcytic anemia occurs in all except:
a. Iron deficiency
b. Thalassaemia
( NBE Pattern 2014 -15)
Kidney
has serum K of 7.5 meq / 1, which is the fastest way to c. Lead poisoning
reduce the hyperkalemia? ( Al 2009) d . Chronic renal failure
a. Kayexalate enema Ref Harrison’ s 18th ed. ch. 280, Harrison 19 th p 393
b. Infusion of insulin + glucose 20 . The term end - stage renal disease ( ESRD) is considered
c. IV calcium gluconate appropriate when GFR falls to : ( NBE Pattern 2014-15)
d. IVNaHC03 a. 50% of normal b. 25% of normal
Ref Harrison s 18/ e p359, Harrison 19th p 1810 c. 10- 25% of normal d. 5-10% of normal
14. Nephrotic syndrome patient after a bout of diarrhea Ref: Harrison's 18th ed., ch. 281, Harrison 19 th p 1811
presented with acute kidney injury and serum creati ¬ 21. Patient with CRF is having a sodium level 110mEq
nine = 4.5. All are possible reasons except? dl. Till what level should serum sodium be corrected
= /
a. Renal vein thrombosis ( AIIMS NOV. 14 ) in next 24 hours? ( NBE Pattern 2014 -15 )
b. Diarrhea water depletion a. 120 mEq / dl b. 130 mEq / dl
c. Frusemide water depletion c. 140 mEq / dl d. 150 mEq / dl
d. Steroid induced diabetes
Ref Harrison's 18th ed. ch. 45, Harrison 19 th p 1811
Ref Harrison 19 th p 1841
22 . Renal papillary necrosis can be caused by:
15. Marker of acute kidney injury are all except? a. Phenacetin ( NBE Pattern 2014 -15)
a. Clusterin ( NEET 2015-16) b. Sulphonamides
b. Osteopontin
c. Gentamicin
c. Alanine aminopeptidase
d. Penicillin
d. Acid phosphatase
Ref Harrison's 18th ed. ch. 285, Harrison 19th p 477 t
Ref Harrison’s 18th ed. Ch. 279, Harrison 19th p 1799
Ans . .
9. d Urine sodium ...
13. b. Infusion of ins ...
.
17. b Arteriovenous...
21. a. 120 mEq /dl
10 . d . Hyaline arteriosclerosis 11. d . Polyarteritis nodosa
14. d . Steroid induced diabetes 15. d . Acid phosphatase
18. b. Peripheral neuropathy 19. d. Chronic renal failure
-
12. c. Pre renal failure
16. c. Thrice per week
-
20. d. 5 10% of normal
187
I t
22. a. Phenacetin
Marwah's Internal Medicine MCQs ( Based on Harrison's 19 th
)

23. Oliguric phase of ARF is characterized by A/ E: 30. All are true for transplanted kidney except:
a. Chest pain ( NBE Pattern 2014-15) ( NBE Pattern 2014-15)
b. Acidosis a. Humoral antibody responsible for rejection
c. Hypertension b. CM1 is responsible for rejection
d . Hypokalemia c. Previous blood transfusion increases chances of re ¬

* 24.
Ref: Harrison' s 18th ed. ch.279, Harrison 19 th pi 799
in renal failure, metabolic acidosis is due to?
a. Increased H* production
b. LossofHCO /
c. Decreased excretion of ammonia
( NBE Pattern 2014-15)
jection
d . HLA identity similarity seen in 1:100 people
Ref: Harrison’s 18th ch. 282, Harrison 19 th p 925
Clinical features of CRF/ uraemia appear when renal
function is reduced to:
a. 70% b. 50%
( NBE Pattern 2014-15)
d . Use of diuretics
c. 30% d. 20%
Ref: Harrison' s 18th ed. ch. 47, Harrison 19 th p 1799, 1811
Ref: Harrison's 18th ch. 280, Harrison 19 th p 1811
I 25. Diagnostic feature of CRF is: ( NBE Pattern 2014-15) ( NBE Pattern 2014-15)
R.T.A shows all except:
a. Broad casts in urine a. Urine pH always < 5.5
b. Elevated blood urea b. Anion gap normal
c. Proteinuria c. Bicarbonaturia
d . Bleeding diathesis d. Vitamin D deficiency
Ref : Harrison’ s 18th ed. ch. 280, Harrison 19 th p 1811 Ref Harrison’s ch. 284, Harrison 19 th p 332e-7 t
26 . Which one of the following studies is most sensitive 33. -
Nephro calcinosis is common in which type of renal
for detecting diabetic nephropathy in early stage? tubular acidosis: ( NBE Pattern 2014-15)

Kidney a. Microalbuminuria
b. Creatinine clearance test
c. Ultrasonography
d. Serum cretinine level
( NBE Pattern 2014-15)

Ref Harrison's 18th ed. ch. 344, 283, Harrison 19th p 2425
34.
a. Type 1
c. Type 111
b. Type II
d. Type IV
Ref Harrison' s 18th ed. ch. 284, Harrison 19 th p 332e-7 t
CRF changes are A / E:
a. Hyperkalaemia
c. Hypocalcaemia
( NBE Pattern 2014-15)
b. Hypophosphatemia
d. Hypokalemia
27 . Most common acute complication of dialysis is:
a . Hypotension ( NBE Pattern 2014-15) Ref Harrison’s 18th ed. ch.280, Harrison 19 th p 1813
b. Bleeding 35 . Amyloidosis protein associated with hemodialysis?
c. Dementia
d. Muscle cramps a. A Beta 2
( NBE Pattern 2014 15)
b. A Beta
-
Ref Harrison's 18th ed. ch. 281, Harrison 19 th p 1824 c. A transthyretin d. AL
( NBE Pattern 2014 -15) Ref Harrison’s 18th ch. 112, Harrison 19 th p 723
28. CRF shows all except:
a . Hyperphosphataemia 36. Raised PTH is found in: ( NBE Pattern 2014-15)
b. Hyperuricaemia a. Pseudopseudohypoparathyroidism
c. Decreased half -life of insulin b. Renal osteodystrophy
d . Decreased Serum vitamin D3 c. Hypercalcaemia
d. Osteogenesis imperfecta tarda
Ref Harrison's 18th ed. ch. 280, Harrison 19 th p 1811
Ref: Harrison’ s 18th ed. ch.280, 353, Harrison 19th p 291 / 2478
29. Biomarker not involved in acute kidney injury is?
(AUMS Nov 2013) 37. in which renal tubular acidosis, is hyperkalemia a
a. NGAL prominent feature: -
( NBE Pattern 2014 15)
b. KIM 1
a. Type I
b. Type II
c. Micro RNA 122
c. Type III
d . Cystatin C
d . Type IV
Ref: Harrison' s 18th ch. 279, Table 279.3, Harrison 19th p 1799
Ref: Above mentioned table, Harrison 19 th p 332e-7 t

Ans. 23. d . Hypokalemia 24. c. Decreased excre... 25. a. Broad casts in urine 26. a. Microalbuminuria
27. a. Hypotension
.
31 b. 50%
28.
32.
-
c. Decreased half life...
d. Vitamin D deficiency
29. c. Micro RNA 122
33. a. Type I
30. d. HLA identity similarity...
34. b. Hypophosphatemia
188
35. a . Aeta 2 36. b. Renal osteodystrophy 37. d . Type IV
Kidn ey

38. Most common cause of renal artery stenosis in young 45. A complication of diuretic phase of acute renal failure
boys in India: ( A1PG 2010) is: ( NBE Pattern 2014-15)
a . Atherosclerosis a. Convulsion
b. Fibromuscular hyperplasia b. Hyperkalemia
c. Neurofibroma
d. Non-specific aorto-arteritis
Ref: Harrison 19th p 1627
c. Increased sodium excretion in urine
d. Metabolic acidosis
Ref: Harrison's 18th ed., ch.279, Harrison 19 th p 1807
1
^^
39. The absolute indications for dialysis include the fol ¬ 46. Following are absolute indication for hemo dialysis- II
except:
lowing except:
a. Persistent Hyperkalaemia
-
( NBE Pattern 2014 15)
a. GI bleeding
( NBE Pattern 2014-15)

b. Congestive cardiac failure b. Convulsions


c. Pulmonary edema c. Pericarditis
d. Hyperphosphatemia d. Hyperkalemia of 6.5 mEq / L
Ref: Harrison’s 18th ed. ch. 280, 281, Harrison 19th p 1810 Ref: Harrison’s 18th ed. ch. 281, Harrison 19th p 1823
40. Monoclonal antibodies to the CD25 (lL 47. Dialysis patients are prone to develop:
-2a) receptors
are being used for the treatment of : ( Bihar PG 2014 ) ( NBE Pattern 2014 -15)
a. Haematologic neoplasm a. Lead toxicity b. Iron toxicity
b. Autoimmune diseases c. Aluminium toxicity d. Zinc toxicity
c. Bone marrow transplantation Ref: Harrison's 18th ed. ch. 281,
d. Kidney transplant rejection 48. Investigations in a patient of oliguria revealed: Urine
.
Ref: Harrison's 18th ed. ch 282, Harrison 19th p 1829 osmolality: 720 mosm / kg. Urinary sodium 10 mmoI/ L | .
41. Most common complication causing death in patients
on recurrent hemodialysis?
a. Cardiovascular
( NBE Pattern 2014 - 15 )
b. Adynamic osteomalacia
c. Dyselectrolytemia d. Encephalopathy
.
Ref Harrison's 18th ed ch 281, Harrison 19thpl 810
BUN: creatinine =20:1. The most likely diagnosis: j

a. Prerenal acute renal failure


b. Acute tubular necrosis
c. Acute cortical necrosis
d. Urinary tract obstruction
( NBE Pattern 2014-15)
ivumey
42. Papillary necrosis is most commonly seen in: Ref: Harrison's 18th ch. 279, Harrison 19th p 1807
( A1PG 2010) 49. A 28 yr old man has lenticonus and ESRD now. His
a. Diabetes mellitus b. Sickle cell anemia maternal uncle also died of similar illness. Diagnosis
c. Acute pyelonephritis d. Analgesic nephropathy is: ( AllMS May 2012) '
Ref: H 18th ch.-285,Table 285-3, Harrison 19 th p 1861
a. ARPKD b. ADPKD
c. Oxalosis d. Alport's syndrome
43. All of the following uremic manifestations improve Ref: Harrison's 18th ed. Ch. 357, Harrison 19th p 2513
with dialysis except: ( NBE Pattern 2014-15)
a. Metabolic acidosis 50. Central nervous system manifestations in chronic re¬
b. Osteodystrophy nal failure are a result of all of the following except:
.
c Asterixis a. Hyperosmolarity ( NBE Pattern 2014 - 15 )
d. Nausea, vomiting and anorexia b. Hypocalcemia
Ref: Harrison's 18th ed ch. 281, Harrison 19th p 291/ 2478 c. Acidosis
d. Hyponatremia
44. A 40 year -old man underwent kidney trans- planta ¬
- Ref: Harrison's 18th ed. ch. 280, Harrison 19th p 2049
tion for end -stage renal disease. Two months after
transplantation, he developed fever and features sug ¬ 51. The most common ocular infection after renal trans ¬
gestive of bilateral diffuse interstitial pneumonia. plantation is by:
a. Cytomegalovirus
-
( NBE Pattern 2014 15 )
Which one of the following is the most likely etiologi ¬
b. Toxoplasma
cal agent? -
( NBE Pattern 2014 15)
a. Herpes simplex virus b. Cytomegalovirus c. Herpes virus
c. Epstein Barr virus d. Legionella d. EB virus
- Ref: Harrison's 18th ed. ch. 282, Harrison 19th p 1190
. .
Ref: Harrison's 18th ed ch 282, Harrison 19th p 1190

Ans.
-
38. d. Non specific...
42 . d. Analgesic...
39. d . Hyperphosphatemia 40 . d . Kidney transplant... 41. a . Cardiovascular
45. c. Increased sodium .. .
43. b. Osteodystrophy 44. b. Cytomegalovirus
46. d. Hyperkalemia ... 47. c. Aluminium toxicity 48. a. Prerenal acute... 49. d. Alport’s syndrome 189
50. b. Hypocalcemia 51. a. Cytomegalovirus

I.
r
( th
Marwah's Internal Medicine MCQs Based on Harrison's 19 )

52. Normal sized to enlarged kidneys in a patient with 59. Convulsions are commonly precipitated in terminal
chronic renal failure is indicative of: renal failure by: ( NBE Pattern 2014-15)
a. Benign Nephrosclerosis ( NBE Pattern 2014- 15) a . Hyperkalemia
b. Chronic glomerulonephritis b. Hypokalemia
c. Chronic interstitial nephritis c. Water intoxication
d. Primary amyloidosis d . Hypermagnesemia
Ref: Harrison's 18th ch. 112, 283, Harrison 19 th p 723 Ref: Harrison' s 18th ed. ch. 279, 280, Harrison 19 th p 332e- 9
53. Chronic renal failure is often complicated by all of the 60. Amyloid kidney is characterized by all except:
following except: ( NBE Pattern 2014-15 ) a . Hypercholesterolemia ( NBE Pattern 2014-15)
a. Myopathy
b. Normal sized kidney
b. Hemolytic uremic syndrome
c. Peripheral neuropathy c. HTN
d. Ectopic calcification d. Heavy proteinuria
Ref: Harrison' s 18th ch. 280, Harrison 19 th p 1030 Ref Harrison's 18th ed. ch. 112, 283, Harrison 19 th p 723

54. True about renal tubular acidosis are A / E: 61. Anaemia of advanced renal insufficiency is best treat ¬
a. Increased urinary anion gap ed by: ( NBE Pattern 2014-15)
b. Bicarbonaturia ( NBE Pattern 2014-15) a. Blood transfusions
c. Hyperchloremia b. Recombinant human erythropoietin
d . High urinary PH c. Parenteral iron therapy
d . Folic acid supplementation
Ref Harrison’ s 18th ed ch. 284, 47, Harrison 19 th p 332e-7 t
Ref Harrison’s 18th ch. 280,

ivuiey
55 . Which of the following are true regarding congenital
nephrogenic diabetes insipidus: 62 . The differentiating factor between pre - renal and re¬
( NBE Pattern 2014-15) nal azotemia is: ( NBE Pattern 2014-15)
a. ADH receptors are not sensitive a. Sodium fraction excretion
b. It is associated with S1ADH b. Creatinine clearance
c. Serum ADH levels are normal c. Serum creatinine level
d. Urine is hyperosmolar d . Urine specific gravity
Ref Nelson ch. 559, Harrison 19 th p 304 Ref Harrison's 18th ed. ch.44, ch.279, Harrison 19 th p 1809
56. Dementia in patient of chronic renal failure with chron¬ 63 . In uraemia all are reversed by dialysis except:
ic hemodialysis is due to: ( NBE Pattern 2014-15) a . Sexual dysfunction ( NBE Pattern 2014-15)
a. Aluminium toxicity b Pericarditis
b. Uremia c. Uraemic lung
c. A Beta 2 amyloid d Neuropathy
d. A Beta amyloid deposition Ref Harrison 18th ed.Table 280 -4, Harrison 19 th p 1823
Ref Harrison's 18th ed. ch. 371, 281 Table-371.5 DDx of Dementia 64. Non -oliguric renal failure is commonly seen in:
57 . Most common cancer alter kidney transplanta - tion? a. Snakebite -
( NBE Pattern 2014 15)
a. Skin cancer ( NBE Pattern 2014-15) b. Hypovolemic shock
c. Aminoglycoside toxicity
b. Renal cell cancer
d. Multiple myeloma
c. NHL
d . Hodgkins lymphoma Ref Harrison's ch. 279, Harrison 19 th p 938t

I 58.
Ref Harrison’s 18th ed. ch. 282, Harrison 19 th p 1831
In chronic renal failure there is: ( NBE Pattern 2014 -15)
a. Decrease anion gap
65 . Presence of which of the following in the urine is
diagnostic of glomerular injury:
a . Bright red cells
b. 20% dysmorphic RBCs
( AllMS- June 99 )

I b. Normal anion gap


c. Increased anion gap
d . Metabolic alkalosis
Ref Harrison's 18th ed. ch. 280, 44, Harrison 19 th p 1807
c. 100 RBC per high power field
d. beta 2 micro globulin
Ref Harrison 18th/ p 339, 340, Harrison 19 th p 1831

I Ans . 52.
56.
60 .
d. Primary amylo..

b. Normal sized...
. 53.
a . Aluminium toxicity 57.
61.
b. Hemolytic uremic...
a . Skin cancer
b. Recombinant human...
54. a. Increased urinary...
58. c. Increased anion gap
62. a. Sodium fraction ...
55. a . ADH receptors are not...
59. c. Water intoxication
63. a . Sexual dysfunction
190
64. c. Aminoglycoside... 65. b. 20% dysmorphic RBC’s
Kidney

66. Most unlikely cause of acute tubular necrosis amongst 74. All improves after dialysis except: (AI1MS Nov 08)
the following is : (AI 1999) a. Pericarditis
a. Severe -bacterial-infection b. Peripheral neuropathy
b. Massive burn c. Metabolic acidosis
c. Severe crush injury in the foot d. Seizure

Ref
d. Rupture of aortic aneurysm
Harrison 18th / p 2294 Table: 279-1, Harrison 19th p 1803
75.
Ref Harrison 18th/ p 2310, 3311, Harrison 19th p 1810
Disease, which does not recur in the kidney after re ¬
8
67. Which of the following values are suggestive of acute nal transplant : (ARMS Feb 97)
tubular necrosis :
a. Urine osmolality> 500
b. Urine sodium> 40
(AllMS Nov. 2000J a. Alport's syndrome
b. Amyloidosis
c. Good Pasteur's syndrome
I
c. Blood urea nitrogen/ plasma creatinine> 20 d. Diabetic nephropathy
d. Urine creatinine /plasma creatinine> 40
Ref API 6th/ 666, Harrison 19th p 2513
Ref: Harrison 18th/ p 337, 2301, Harrison 19th p 1803
76. Type II RTA is associated with all of the following,
68. Plasma urea / creatinine ratio of 20:1 may be seen in : EXCEPT
a. Rhabdomyolysis (AI 2010) a. Normal Anion Gap Acidosis (AI-1990)
b. Ureteric calculi b. Hyper calciuria
c. Pre -renal failure
d Chronic Glomerulonephritis
c. Decreased urinary citrate
d. Minimum urinary pH < 5.5
Ref: Harrison 18th/ p 337, 2302, Harrison 19th p 1810
Ref: Harrisons 18th/ 280, Harrison 19th p 332e- 7t

Kidney
69. Prerenal and renal azotemia is differentiated on the
77. All are features of renal tubular acidosis type I, except:
basis of: (PCI Dec 99).
a. Stone in kidney ( AIIMS Sept 96)
a. Creatinine clearance
b. Serum creatinine level b. No anion gap
c. Sodium fraction excretion c. Hypokalemia
d. Urine bicarbonate level d. Fanconi syndrome
Ref Harrison 18th/ p 337, 2302, Harrison 19th p 1809 Ref: Harrison 18th/ p 2364, 2365, Harrison 19th p 332e-7t
70. All of the following are true about Oliguric ARF 78. Positive Urinary Anion Gap helps to establish the di¬
a. Anemia (Ai -1993) agnosis of : (AI 2009)
b. Metabolic Acidosis a. Alcoholic ketoacidosis
c. Uremia b. Diabetic ketoacidosis
d. Hypercalcemia c. Renal tubular Acidosis
d. Acidosis in Diarrhea
Ref: Harrison 18th/ 2303, 2306, Harrison 19th p 1755
71. Restless leg syndrome (RLS) is seen in: (AI 2009) Ref: Harrison 18th / 368, 369; 17th/ 292, Harrison 19th p 332e-7t
a. Hypercalcemia b. Hyperphosphatemia 79. Which of the following is the most specific and sensitive
c. Chronic renal failure d. Hyperkalemia screening test for Renovascular Hypertension.
a. HRCT (AIIMS May 01)
Ref: Harrison 18th/ 218, 2317, Harrison 19th p 192
b. CT Angiography
72. Metabolic complication in CRF include all of the fol ¬
c. Captopril enhanced radionucleatide scan
lowing except : (AI 1998) d. MRI
a. Hyperkalemia b. Hypophosphatemia
c. Hypocalcemia d. Hypokalemia Ref: Braunwald 8th /1040, Bailey & Love's p 179
80. Most common cause of renal artery stenosis in young
Ref Harrison 18th/ p 2311, Harrison 19th p 2458
adults in India is: (AIIMS Dec 97)
73. Renal osteodystropy differs from nutritional and ge ¬
a. Atherosclerosis
netic forms of osesteomalacia in having : (A12002) b. Non specific aorto - arteritis
a. Hypocalcaemia b. Hypercalcemia c. Fibro muscular dysplasia
c. Hypophostaemia d. Hyperphosphatemia d. None of the above
Ref Harrison 18th/ 280, Harrison 19th p 2460 Ref: Harrison 19th p 1618

1
Ans. 66. d. Rupture of aortic... 67. b. Urine sodium>40 68. c. Pre-renal failure 69. c. Sodium fraction excretion
70. d. Hypercalcemia 71. c. Chronic renal failure 72. b. Hypophosphatemia 73. d. Hyperphosphatemia
74. b & d. Peripheral... 75. a . Alport’s syndrome 76. c. Decreased urinary... 77. d. Fanconi syndrome
78. c. Renal tubular... 79. b. CT Angiography 80. b. Non specific aorto-arteritis
I Marwah 's Internal Medicine MCQs ( Based on Harrison's 19th )

Renal artery stenosis may occur in all of the following 87. A 25 Year old man develops hemoptysis. A few weeks
except: (AI 2006 ) later he experienced sudden onset of acute renal fail ¬
a. Atherosclerosis ure. His urine examinations showed presence of mild
b. Fibromuscular dysplasia proteinuria, hematuria, and RBCs casts. Most likely
c. Takayasu 's arteritis diagons is: ( MP PG 2010 )

8 d . Polyarteritis nodosa
Ref: CMDT’09/ 810; API 8th, 541, Harrison 19 th p 1618
a. Minimal change disease
b. Good pasture syndrome
c. Diffuse membraneous glomerulonephritis
82 . A 28 year old boy met with an accident and sustained d . IgA Glomerulonephritis
severe crush injury. He is most likely to develop: Ref Harrison's 18/ e p2342, Harrison 19 th p 377 e
a . Acute Renal Failure ( AIIMS Nov 09 ) Which of the following conditions are assoiciated with
b. Hypophosphatemia
pauci - immune crescenteric glomerulonephritis:
c. Hypercalcemia a. Henoch - Schonlein Nephritis ( AI 2009 )
d. Acute Myocardial Infarction b. Lupus Nephritis (SLE)
Ref: Harrison 19 th p 1799 c. Microscopic polyangitis
d. Nephritis in Alport's syndrome
83 . Renal damage due to amphotericin B are all, except
a. Azotemia Ref: Harrison's 18/ e p2337, Harrison 19th p 837
(.AIIMS Nov 01 )
b. Renal tubular acidosis 89. Good pasture's syndrome is characterized by all of
c. Glomerulonephritis the following, except: ( AIIMS May 08)
d . Hypokalemia a. Glomerulonephritis
b. Leucocytoclastic Vasculitis
Ref KDT 5th/ 717, Harrison 19 th p 1330
c. Diffuse alveolar haemorrhage

Kidney 84. Which of the following drugs is not a part of the 'Triple
Therapy' immunosuppression for post- renal trans ¬
plant patients?
a. Cyclosporine
b.
c.
Azathioprine
FK 506
(AI 2006 )
d. Presence of antibodies to basement membrane
Ref Harrison' s 18/ e p2342, 2169, Harrison 19 th p 377
A 25 year old boy presents with renal failure. His
uncle died of renal failure three years ago. Slit lamp
examination reveals Lenticonus / Keratoconus. The
likely diagnosis is: (AIIMS Nov 2010 )
d. Prednisolone a. Autosomal dominant polycystic kidney disease
(ADPCKD)
Glomerular Disease b. Autosomal recessive polycystic kidney disease
(ARPCKD)
c. Alport’s syndrome
85. A 30 - year - old male patient came with cough, hemop ¬ d . Denysh- Drash syndrome
tysis, fever since 15 days. After a few days the patient
Ref: Harrison' s 18/ e p2351, 3213, 1 , Harrison 19 th p 2513
developed hematuria. There were no cause other
bleeding manifestations. Most likely cause is: 91. Collapsing glomerulopathy, features:
( Manipal ) a. Tuft necrosis
a. Goodpasture syndrome b. Mesangiolysis
b . Ca. Bronchus with secondaries in Kidney c. Proliferation of parietal epithelium cells
d. Hypertrophy of visceral epithelium cells
c. Leptospirosis
d DIC Ref Harrison 19 th p 1715 Ref Harrison’ s 18/ e p2353, Harrison 19 th p 62e - 4 f
92 . Wire loop lesions are often characteristic for the fol ¬
86 . True statement regarding post streptococcal glomer-
(JIPMER 2014 ) lowing class of lupus nephritis: (AIIMS May 04)
ulonephritis is? a. Mesangial proliferative glomerulonephritis (WHO
a. Renal Biopsy is indicated in serves renal dysfunc ¬

class II)
tion b. Focal proliferative glomerulonephritis (WHO class III)
b. Microscopic hematuria resolves within 4 weeks c. Diffuse proliferative glomerulonephritis
c. Serum C3 levels are normal (WHO class IV)
d. Serum triglyceride levels are elevated d. Membranous glomerulonephritis (WHO class V)
Ref : Harrison 19 th p 1806 t, 1836 Ref: Harrison’ s 18/ e p2727, Harrison 19th p 1813

Ans . 81. d . Polyarteritis... 82. a . Acute Renal Failure 83. c. Glomerulonephritis 84. c. FK 506
85. a . Goodpasture... 86. a. Renal Biopsy is indi... 87. b. Good pasture... 88. c. Microscopic polyangitis
89. b. Leucocytoclastic... 90. c. Alport’s syndrome 91. d. Hypertrophy of vis... 92. c. Diffuse proliferative...
192
Kidney

93. Serum C 3 is persistently low in the following except: 100. Feature of RPGN are A / E: ( NBE Pattern 2014- 15)
a. Post streptococcal glomerulonephritis a. Rapid recovery b. Crescent formation
b. Membranoproliferative glomerulonephritis c. High blood pressure d. Non-selective proteinuria
c. Lupus nephritis Ref Harrison's 18th ed. ch. 283, Harrison 19 th p 1836

l
d. Glomerulonephritis related to bacterial Endocarditis
Ref: Harrison's 18/ e p2353, 2340, Harrison 19th p 1813 101. Type I membrano proliferative Glomerulo nephritis -
is commonly associated with all except:
94. Which of the following statements about Hemolytic a. SLE ( NBE Pattern 2014 -15)
Uremic Syndrome is least correct: (Al 2012) b. Persistent hepatitis C infections
a. Usually follows Hemorrhagic colitis c. Partial lipodystrophy
b. Often self -limited d. Neoplastic diseases
c. Fever is usually mild or absent
Ref Harrison's 18th ch. 283, table 283.4, Harrison 19th p 1843
d. Serotoxin has no role in pathogenesis
Ref: Harrison’ s 18/ e p 970, Harrison 19th p 1796 102. Recurrent hematuria in a deaf mute is seen in:
a. Fanconi's anemia ( NBE Pattern 2014 -15)
95. Locate the renal stone with pain radiating to medial b. Alport's syndrome
side of thigh and perineum due to slipping of stone in c. Renal cysts
males: ( AIIMS June 2000) d. Nephrotic syndrome
a. At pelvic brim Ref: Harrison's 18th ed. ch. 283, Harrison 19th p 2513
b. Intramural opening of ureter
103. Characteristic finding in AGN: ( NBE Pattern 2014-15)
c. Junction of ureter and renal pelvis
a. Red cell cast b. Hematuria
d. At crossing of gonadal vessels and ureter c. Proteinuria d. Epithelial Cells
Ref Harrison' s 18/ e p2382, 2386; Harrison 19th p 2187 .
Ref: Harrison' s 18th ed ch. 283
96 . Interstitial nephritis is common with?

a. NSAID
c. Rhabdomyolysis
( NEET Pattern 2015- 16 )
b. Black water fever
d . tumor lysis syndrome
Ref: Harrison's 18th ed. Ch. 285, Harrison 19th p 1856
104. Salt losing nephritis is:
a. Interstitial nephritis
b. Polycystic Kidney
c. Lupus nephritis
d. R. P.G.N
( NBE Pattern 2014-15)

Kidney
97. Leprosy causes: ( NBE Pattern 2014-15) Ref: Harrison's 18th ch. 283, Harrison 19th p 1856
a. Membranous GN 105. In Hepatorenal syndrome, urine shows:
b. Focal glomerulosclerosis a. Proteinuria ( NBE Pattern 2014-15)
c. Membranoproliferative GN b. Hematuria
d. Mesangioprolferative GN c. A and b
Ref: Harrison' s 18th ed. ch. 283, Harrison 19th p 1843 d . No abnormality
9B . Non - selective proteinuria is seen in: Ref: Harrison’s ch.279, Harrison 19th p 1809
( NBE Pattern 2014 -15) 106. RBC cast in the microscopic examination of the urine
a. Minimal change is an indicator of: ( NBE Pattern 2014-15)
b. Mesangio- proliferative GN a. Acute glomerulonephritis
c. Membranous glomerulonephritis b. Acute pyelonephritis
d. Focal segmental Glomerulosclerosis c. Chronic glomerulonephritis
Ref H 18th ed. ch. 283, 285, Harrison 19th p 183St, 1842 d . Nephrotic syndrome
99. Membranous GN with reduced complement level is Ref: Harrison's 18th ed ch 283, . .
seen in? ( NBE Pattern 2014-15) 107. What is the minimum number of red blood cells per
a. Hepatitis B microliter of urine required for diagnosis of hematu ¬
b. SLE ria? ( APPG 2014 )
c. Malaria a. 3 b. 5
d. Syphilis c. 8 d . 10
Ref H 18th ed ch.283 Table-283-6, 283.6 Harrison 19th p 1843 .
Ref CMDTch 22, pg . 879, Harrison 19 th p 1847

Ans . 93. a post streptococcal...94. d . Serotoxin has no role... 95. a. At pelvic brim 96. a. NSAID
97. a. Membranous GN 98. d . Focal segmental... .
99. b SLE 100 . a. Rapid recovery
101. c. Partial lipody... 102. b. Alport’s syndrome 103. a. Red cell cast 104. a. Interstitial nephritis 193
105. c. A a n d b 106. a. Acute glomerulone... 107. b. 5
Marwah's Internal Medicine MCQs ( Based on Harrison’s 19th )

108. Plasmapheresis is indicated for which of the following: c. Both rasburicase + allopurinol + hydration and K
a. Wegener's granulomatosis ( NBE Pattern 2014-15)
b. Henoch -Schonlein purpura
binder
d . Hydration alone is sufficient
c. Goodpasture's syndrome
d. Acute transplant rejection Ref: Harrison's 18th ed ch. 276, Harrison 19th p 1795

1 Ref: Harrison's 18th ed. ch. 283, Harrison 19 th p 1839


109. A young boy with skin rashes (Purpura ), acute onset,
oliguria, CNS manifestation after 5 days of diarrhea is
116. Manifestation of acute glomerulonephritis includes
each of the following except:
a. Peri-orbital edema
[NBE Pattern 2014-15)

suffering from: b. Hypertensive encephalopathy


NBE Pattern 2014-15]
a . D + HUS c. Acute renal failure
b. D- HUS
c. Aplastic anemia d. TTP d . Optic atrophy Ref Harrison’s 18th ed. ch. 283
Ref: Harrison’ s 18th ed. ch. 286, Harrison 19 th p 377 e - 4 117. A female patient presents with upper respiratory tract
110. Causes of low urinary calcium include: infection. Two days after, she develops hematuria . Pro¬
a. Renal tubular acidosis bable diagnosis: ( NBE Pattern 2014-15)
( NBE Pattern 2014-15)
b. Cushing's syndrome a . IgA nephropathy
c. Chronic glomerulonephritis/ CKD b. Wegener's granulomatosis
d. Paget's disease c. Henoch -Schnlein purpura
d. Poststreptococcal glomerulonephritis
Ref: Harrison's 18th ed . ch.280, 44
Ref: Harrison's 18th ed. ch. 283, Harrison 19 th p 1839
111. Essential feature of nephritic syndrome is:
a . Proteinuria ( NBE Pattern 2014-15) 118. Which of the following is not a feature of Hemolytic -
Uremic syndrome: ( NBE Pattern 2014-15)

Kidney
b. Hypoalbuminaemia
c. Hyperlipidaemia a. Encephalopathy b. Oliguria
d. Hematuria c. Thrombocytopenia d . Purpura
Ref Harrison' s 18th ed. ch. 283, Harrison 19 th p 1834 Ref Harrison's 18th ed. ch. 286, Harrison 19 th p 1030

112 . Minimal change disease in children can progress to: 119. Which one of the following statements regarding py ¬
a . Membranous GN ( NBE Pattern 2014-15) elonephritis is correct? ( AIPG 2009)
b. Mesangioproliferative a. Recurrent infection with the same strain usually
c. Focal GN occurring after one week of cessation of therapy.
d . Focal segmental glomerulosclerosis b. Xantho-granulomatous pyelonephritis is seen with
Ref: Nelson 18th ed. ch. 527, Harrison 19 th p 1842 diabetes mellitus
c. Emphysematous pyelonephritis is seen with stag ¬
113. Which is not a feature of hemolytic- uremic syndrome : horn calculus
a. Thrombocytosis ( NBE Pattern 2014-15) d. 10,000 viable bacteria per ml in clean voided
b. Uraemia midstream urine is of significance
c. Hematuria Ref: Harrison' s 18th ed. ch. 288, Harrison 19 th p 867
d. Segmented RBC's in peripheral smear
.
Ref: Harrison’s 18th ed ch. 286, Harrison 19thpl 030 120. Triad of glomerulonephritis pulmonary hemorrh ¬
ages and antibody to basement membrane is called:
114. Papillary necrosis is seen with all except? a. Goodpasture's syndrome ( NBE Pattern 2014-15)
a . Chronic alcoholism ( NBE Pattern 2014-15) b. Systemic Necrotising Vasculitis
b . Sickle cell anemia c. Mixed connective tissue disease
c. Analgesic nephropathy d. Diabetic
d. Medullary sponge kidney Ref: Harrison's 18th ed. ch. 283, Harrison 19 th p 1839
Ref Harrison's ch.283, Harrison 19 th p 1855
121. Chronic reflux nephropathy causes:
115. Initial treatment of tumor lysis syndrome is: a. Membranous nephropathy ( NBE Pattern 2014 -15)
( NBE Pattern 2014-15) b. Focal segmental GN
a. Rasburicase + hydration + K binder + urinary alkali ¬ c. MPGN
zation . d. Lipoid nephrosis
b. Allopurinol + Hydration + K binder + urinary alkaliza ¬ Ref: Harrison's 18th ed. ch. 283, 285, Harrison 19th p 1842
tion.

Ans. 108. c. Goodpasture’s... 109. d . TTP 110 . c. Chronic glomeru.. . 111. d . hematuria
112. d . Focal segmental... 113. a. Thrombocytosis .
114. d Medullary sponge... 115. b. Allopurinol + Hydration ...
116. d. Optic atrophy 117. a. IgA nephropathy 118. d . Purpura 119. d. 10,000 viable bacteria ...
194
120 . a . Goodpasture’s... 121. b. Focal segmental GN
Kidney

122 . Test predicting the return of renal function in a pa ¬ 130. RPGN occurs in A / E: ( AIIMS Sep 96 )
tient with tumour lysis syndrome is? a. SLE
( NBE Pattern 2014-15) b. Post streptococcal glomerulonephritis
a. Serum creatinine b. Serum phosphate c. Diabetic nephropathy
c. Serum potassium d. Serum Uric acid d. Good pastures syndromes
Ref: Harrison’s 18th ed. ch. 276, Harrison 19 th p 1795
123. The following type of glomerulonephritis should not
be treated with prednisolone : ( NBE Pattern 2014-15)
Ref Harrison 18th/ p 2338, Harrison 19th p 1836
131. The prognosis of rapidly proliferating glomerulone ¬
8
phritis (Crescentric GN) depends upon:
a . Minimal change disease a. Number of crescents (AIIMS Nov 01 )
b . Lipoid nephrosis
b. Size of crescents
c. Congenital Nephrotic Syndrome
d . Post-streptococcal GN c. Shape of crescents
d . Cellularity of crescents
Ref: Harrison’s 18th ed. ch. 283, 44, Harrison 19 th p 1837
Ref Robbins 6 th / 453, Harrison 19 th p 1836
124. A renal transplant recurrence of the disease occurs
132 . Marker for the renal vasculitis in children is : ( AI 98 )
mostly with:
a . Lupus nephritis
-
( NBE Pattern 2014 15)
a. IgA level
b. DM nephropathy b. Low complement level
c. Membranous glomerulonephritis c. Increased Antineutrophilic cytoplasmic antibody
d. Membranous proliferative glomerulonephritis titre
Ref: Harrison’s 18th ed. ch. 283, 280, Harrison 19 th p 1841 d. Increase antinuclear antibody Ref: Robbins 7th/ 990
125 . In hematuria of glomerular origin the urine is charac ¬ 133. True about Post-Streptococcal Glomerulonephritis is:

Kidney
terized by the presence of all of the following except: a . 50% of cases occur after pharyngitis ( AI 2000)
( AI 2004 ) b. Early treatment of Pharyngitis eliminates the risk of
a. Red cell casts b. Acanthocytes P.S.G.N.
c. Crenated red cells d . Dysmorphic red cells c. Glomerulonephritis, secondary to skin infection, is
Ref: Harrison 18th/ p 2340, 2301, Harrison 19 th p 294 more common in summer
d. Recurrence is seen
126. Presence of which of the following correlates best
with renal pathology: ( AIIMS June 2000 ) Ref: Harrison 18th/ 2345-46, Harrison 19 th p 1837
a. Hyaline cast b. Coarse granular cast 134. Association of deafness & nephritis is seen in :
c. Broad cast d. Epithelial cast a . Pickwickian syndrome ( AIIMS May 93)
Ref Harrison 18th/ p 339, 340, Harrison 19 th p 294 b. Alport's syndrome
c. Fabry's disease
127 . Salt losing nephritis is a feature of : ( Al 2000)
d . Lawrence Moon Biedl syndrome
a. Interstitial nephritis ( AIIMS 95)
b. Renal Amyloidosis ( AIIMS May 94) Ref: Harrison 18th/ p 2351, 3213, Harrison 19 th p 2513
c. Lupus nephritis 135. A 7 year old boy presented with generalized edema .
d. Post Streptococcal Glomerulonephritis Urine examination revealed marked albuminuria .
Ref Harrison 18th/ p 2367, Harrison 19th p 1856 Serum biochemical examinations showed hypoalbu -
128. Necrotizing pappillitis may be seen in all of the fol ¬ minaemia with hyperlipidemia. Kidney biopsy was
lowing conditions except : (AI 2002) undertaken. On light microscopic examination, the
a . Sickle cell disease kidney appeared normal . Electron microscopic ex¬
b. Tuberculous pyelonephritis amination is most likely to reveal - ( AIIMS Nov-03)
c. Diabetes mellitus a. Fusion of foot processes of the glomerular epithelial
d. Analgesic nephropathy Ref Harrison 18th/ 2367 cells
129. Post - infective glomerulonephritis present as: b. Rarefaction of glomerular basement membrane
(AI 1996 ) c. Deposition of electron dense material in the base¬
a - ARF b. Nephrotic syndrome
c. Nephritic syndrome d . Asymptomatic hematuria
ment membrane
d. Thin basement membrane
Ref : Harrison 18th/ 2345-46, Harrison 19 th p 1837 Ref: Harrison 18th/ p 2345

ns . 122. d . Serum Uric acid


126. c. Broad cast
130. c . Diabetic nep...
123.
127.
131.
-
d . Post streptococcal GN 124. d . Membranous prolifer...125. b. Acanthocytes
a . Interstitial nephritis
a . Number of crescents
128. b. Tuberculous...
132. a. IgA level
129. c Nephritic syndrome
133. c. Glomerulonephritis ...
I
134. b. Alport’s syndrome 135. a. Fusion of foot processes...

1
i
Marwah's Internal Medicine MCQs ( Based on Harrison's 19,h )

136. True about Light microscopic changes in Minimal


142. In Leprosy most common renal lesion seen is:
Change Glomerulonephritis is: ( Al 95 ) a. MGN ( Al 1999 )
a. No abnormality
b. Fusion of foot process b. MPGN

I c. Absence of Immunoglobulins c. Focal glomeruloselerosis


d. Absence of complement d . Diffuse glomerulosclerosis
Ref: Harrison 18th/ p 2345, Harrison 19th p 1842 Ref: Rooks Dermatology 5th/ 1073, Harrison 19th p 1841
137. A child presents with hematuria and nephrotic 143. Reflux Nephropathy with protenuria in the nephrotic
syn ¬ range may be seen in patients with : (AIIMS Nov 06 )
drome. A diagnosis of minimal change disease was
made. Which of the following statements about the a. Membranous glomerulonephritis
diagnosis is true: ( An Indiaj b. Focal segmental Glomerulosclerosis
a. Glomerular function is lost due to loss of polyanions c. Nodular glomerulosclerosis
around the foot processes d. Crescenteric glomerulonephritis
b. Foot processes of podocytes in the Glomerular Ref Harrison 18th/ 2345-46, Harrison 19th p 1842
membrane are normal
c. Glomerular function is lost due to deposition of IgA 144. Serum C3 is persistently low in the following except:
on the glomerular membrane a. Post streptococcal glomerulonephritis
d . Focal segmental changes are observed b. Membranoproliferative glomerulonephritis
c. Lupus nephritis ( Al / AIIMS)
Ref: Harrison 18th/ p 2345 d. Glomerulonephritis related to bacterial endocarditis
138. All are steroid resistant except: ( Al 96 ) Ref Harrison 18th/ p 2340, Harrison 19th p 1837
a. Post -streptococcal glomerulonephritis
b. Minimal change glomerulonephritis 145. Proliferative glomerular deposits in kidney are found

Kidney c. RPGN
d . Recurrent hematuria
Ref Harrison 18th/ p Ch 283, Harrison 19th p 1842
139. Wire loop lesions are often characteristic for the fol ¬
lowing class of lupus nephritis: ( AIIMS May 04 )
a. Mesangial proliferative glomerulonephritis(WHO
in
a. Amyloidosis
b. Diabetes mellitus
c. IgA nephropathy
d . Membranous glomerulonephritis
(AI 2000 )

Ref Chandrasoma Taylor 2nd/703, Harrison 19th p 1839


-

class II ) 146. All are features of haemolytic uremic syndrome, except:


b. Focal proliferative glomerulonephritis (WHO class III ) a. Hyperkalemia (AIIMS Dec 95)
c. Diffuse proliferative glomerulonephritis (WHO class b. Anaemia
IV) c. Renal microthrombi
d. Membranous glomerulonephritis ( WHO class V ) d . Neuropsychiatric disturbances
Ref: Harrison 18th/ p 2727 Harrisons 17th/ 2076, 2077 Ref Harrison 18th edn,Ch 286, Harrison 19th p 1030
140. All of the following factors are associated with adverse 147. A 20 year old male presents with features of acute
prognosis and high risk of Renal progression in Lupus renal failure 5 days after an episode of diarrhea .
Nephritis, Except: ( PGI Dec 03) Blood examination shows thrombocytopenia and Hb -
a. High levels of Anti-ds DNA 10 gm%. Likely cause is: (AIIMSJune 99)
b. Persistant proteinuria ( Nephrotic range > 3gm / day) a. Haemolytic uremic syndrome
c. Hypocomplementenemia b. Hereditary spherocytosis
d. Anti LA (SSB) c. Haemolytic crises
Ref Samter's Immunologic disease 6 th/ 497 d. Chronic glomerulonephritis
141. A patient who has been diagnosed with bronchiecta ¬ Ref Harrison 18th/ p 970, Harrison 19th p 1030
sis 5 years ago presents with edema on legs and pro ¬ 148. Renal vein thrombosis is most commonly associated
teinuria. The most likely finding in his kidney will be: with: ( Al 2001 )
(A! 2012)
a. Diabetic nephropathy
a . Minimal Change Disease
b. Membranous glomerulopathy
b. Amyloid Nephropathy
c. Minimal change disease
c. Rapidly Progressive Glomerulonephritis ( RPGN )
d. Membrano- proliferative glomerulonephritis
d. Crescenteric Glomerulonephritis
Ref: Harrison 18th / 945, Harrison 19th p 723 Ref Harrison 18th / 280

Ans . 136. a. No abnormality 137. a . Glomerular function ... 138. b. Minimal change... 139. c. Diffuse proliferative. ..
140 . d. Anti LA (SSB ) 141. b. Amyloid Nephropathy 142. b. MPGN 143. b. Focal segmental .. .
144 . a . Post streptoco... 145. c. IgA nephropathy 146. d. Neuropsychiatric... 147. a. Haemolytic uremic...
196
148. b. Membranous...
Kidney

10 year old child develops hematuria after 2 days of 155 . Nephrotic syndrome is the hall mark of the following
149. A
diarhoea . Blood film shows fragmented RBCs & throm ¬ primary kidney diseases except:
( NBE Pattern 2014-15)
bocytopenia. Ultrasound shows marked enlargement
of both kidneys . The likely diagnosis is: ( Al IMS June 99 ) a . Membranous Glomerulopathy
a. Acute pyelonephritis
b. Disseminated intravascular coagulopathy
c. Haemolytic uremic syndrome
Ref: Nelson 16 th/ 1589
b. IgA nephropathy
c. Minimal change disease
d. Focal segmental Glomerulosclerosis
8
d . Renal vein thrombosis Ref Harrison's 18th ch. 283, 44, Harrison 19th p 1841
150. A 60 yr old smoker came with a history of painless 156. Muehrcke lines in nails are seen in :
gross hematuria for one day. Most logical investiga ¬ a. Nephrotic syndrome ( NBE Pattern 2014-15)
tion would be: ( AI 2007 )
b. Barrter syndrome
a. Urine routine c. Nail patella syndrome
b. Plain X ray KUB
d. Acute tubular necrosis
c. USG KUB
d. Urine microscopy for malignant cytology Ref KD1GO 2012 guidelines pg. 128-29, Harrison 19 th p 1841
Ref: Bailey & Love 24th/ 1463, Harrison 19 th p 575 157 . Regarding complications of nephrotic syndrome in ¬
correct is: ( NBE Pattern 2014-15 )
Nephrotic Syndrome a. Volume overload state
b. Hypercoagulable state
151. Presence of which of the following correlates best c. Hyperlipidaemia
with renal pathology: ( ARMS June 2000) d. Hypocalcemia

Kidney
a. Hyaline cast Ref Harrison's 18' 11 ch. 283, Harrison 19 th p 252
b. Coarse granular cast
c. Broad cast 158. In bronchogenic carcinoma patient presenting as a
d. Epithelial cast case of nephrotic syndrome. If kidney biopsy is done
Ref H 18/ e p339, 17/ e p273, 1758, Harrison 19 th p 294 most likely lesion will be: ( NBE Pattern 2014- 15)
a. Membranous GN
152. Which of the following statements about orthrostatlc
b. Focal proliferative GN
proteinuria is true: ( PCI Dec 2000 )
c. Minimal change disease
a. Seen in recumbent position
b. Is benign d. Focal segmental glomerulosclerosis
c. Future risk of nephrotic syndrome Ref Harrison's 18th ch. 283, Harrison 19 th p 1843
d. < 300 mg / day 159. A 7 year old child is having steroid DEPENDANT Ne ¬
Ref Harrison’s 18/ e p338, Harrison 19 th p 293 phrotic syndrome . His weight is 30 kg and height of
153. Hypercoagulation in Nephrotic syndrome is caused 106 cm. He is having truncal obesity with subcapsular
by: (Al 2010) bilateral cataracts. Best drug for this patient:
a. Loss of Antithrombin III ( AIIMS May 2013)
b. Decreased Fibrinogen a. Mycophenolate
c. Decreased Metabolism of Vitamin K b. Levamisole
d . Increase in protein C c. Cyclophosphamide
Ref Harrison' s 18/ e p346, 17/ ep272, Harrison 19 th p 252 d. Azathioprine
.
154 The Recommended treatment of early diabetic ne ¬ Ref: Nelson Textbook, 18th ed„ ch. 527, Harrison 19 th p 252
phropathy manifested by microalbuminuria is: 160. Nephrotic syndrome associated with malaria is due to
a. Strict glycemic control
b. Low protein Diet
infection of ? ( NBE Pattern 2014-15)
c. Strict glycemic control and Low Protein Diet
a. P. malariae
d . Strict glycemic control, Low protein Diet and ACE b. P. ovale
Inhibitors c. P. vivax
Ref: H 18/ e p2348, 298317/ e p2288, Harrison 19 th p 1844 d . P. falciparum
Ref Net source PubMed, Harrison 19 th p 252

nS
it? a - °
' d' Renal vein -
; 150 - d . Urine microscopy.. . 151 . c . Broad cast 152. b. Is benign
a. b rSOfAn tith. ..
' "' 154' c' Strict glycemic control .. . 155. b. IgA nephropathy 156 . a . Nephrotic syndrome
is? • Volume over 158 . a . Membranous GN 159. c. Cyclophosphamide 160. a . P. malariae 197
th
Marwah s
' Inter nal Med icine MCQ s ( Base d on Harr ison's 19 )

161. An adult patient presents with normal or enlarged kid ¬ 167. The most common cause of renal scarring in a 3 year
neys with massive proteinuria. Most likely cause is: old child is: ( Al 2005)

a
a. Chronic pyelonephritis ( NBE Pattern 2014-15) a. Trauma.
b. Chronic glomerulonephritis b. Tuberculosis.
c. Amyloidosis c. Vesicoureteral reflux induced pyelonephritis.
d . Renal artery stenosis d . Interstitial nephritis.
Ref: Harrison's 18th ed. ch. 283, Harrison 19 th p723 Ref: Ghai 8/ e p506, Harrison 19 th p 867

162 . Renal vein thrombosis is associated with which un ¬ 168. Chromophobe variant of Renal cell carcinoma is as ¬
derlying disease of kidney: ( NBE Pattern 2014 15) - sociated with:
a. VHL gene mutations
( AI 2010 )
a. Chronic glomerulonephritis
b. Pyelonephritis b. Trisomy of 7 and 17 [+ 7, +17]
c. SLE c. 3 p deletions (3p-]
d. Nephrotic syndrome d . Monosomy of 1 and Y (-1, Y) -
.
Ref: Harrison' s 18th ed ch. 286, Harrison 19 th p 252 Ref: Harrison's 18/ e p793, Harrison 19 th p 102e - l 7

163. All of the following are decreased in Nephrotic syn-


169. Paraneoplastic syndrome associated with RCC are all
of the following except: (AI 2009)
drome except : (A11997 )
a. Polycythemia
a. Transferrin b. Hypercalcemia
b. Ceruloplasmin c. Malignant hypertension
c. Albumin d. Cushing syndrome
d. Fibrinogen
Ref: Harrison's 18/ e p793, Harrison 19 th p 2273

Kidney
Ref: Harrison 18th/ p 2345, Harrison 19th p 252
170. Nephrocalcinosis is seen in all except:
I 164. Hypercoagulation in Nephrotic syndrome is caused by: a. Polycystic kidney ( NBE Pattern 2014 -15)
a . Loss of Antithrombin 111 ( A12010 ) b. Hyperparathyroidism
b. Decreased Fibrinogen c. Medullary sponge kidney
c. Decreased Metabolism of Vitamin K d. Renal tubular acidosis
d . Increase in protein C Ref: Harrison's 18th ed. ch. 284, Harrison 19 th p 477 t
Ref: Harrisons 17th/ 272, Harrison 19 th p 252
171. All can be manifestation of polycystic kidney except:
165. A patient with nephrotic syndrome on longstanding a. Urine retention ( NBE Pattern 2014 -15)
corticosteroid therapy may develop all the following b. Renal hypertension
except: (AI 2002) c. Renal failure
a. Hyperglycemia d. Haematuria
b. Hypertrophy of muscle .
Ref: Harrison's 18th ed ch. 284, Harrison 19 th p 4771
c. Neuropsychiatric symptoms
172 . Polycystic kidney is not associated with cysts in:
d . Suppression of the pituitary adrenal axis ( NBE Pattern 2014-15)
Ref: KDT 5th/ 264, 265, Harrison 19 th p 252 a. Liver
b. Pancreas
c. Brain
Renal Cell Carcinoma d. Lungs
Ref: Harrison's 18th ed. ch. 284, Harrison 19th p 4771
166. True about adult polycystic kidney disease is all except:
( ARMS 2001 ) 173. Commonest Symptom of medullary sponge kidney
a. Autosomal dominant inheritance ( NBE Pattern 2014-15)
b. Hypertension is rare
disease:
c. Can be assoc iated with cysts in liver, lungs and pan ¬ a. Anuria
creas b. Anemia
d. Pyelonephritis is common c. Azotemia
d. UTI
Ref: Harrison's 18/ e p2355, 2356, Harrison 19 th p 1851 Ref: Nelson 18th ed., ch. 529 , Harrison 19th p 1855

162. d . Nephrotic syndrome 163. d . Fibrinogen 164. a . Loss of Antithrombin III


Ans . 161 . c. Amyloidosis Mono somy of 1 and Y...
165 . b . Hypertrophy.. . 166 . b . Hypertension is rare 167 . c . Vesic ouret eral reflux . . . 168 . d .
169 . d . Cushing syn . . . 170 . a . Polycystic kidney 171 . a. Urine retention 172. c . Brain
198
173. d . UTI
Kidney

174. True regarding chromosomal location of adult poly ¬ 181. Polycystic disease of the kidney may have cysts in all
of the following organs except : ( AI 2004 )
cystic kidney disease I and II respectively is:
b. Liver
a. Chr 16 and 5 ( NBE Pattern 2014 - 15) a. Lung
c. Pancreas d. Spleen
I
b. Chr 16 and 4
Ref Harrison 18th/ p 2356, Harrison 19 th p 1851
c. Chr 11 and 5
d. Chr 21 and 18
Ref: Harrison's 18th ed. ch. 284, Harrison 19 th p 1851
182 . Which of the following is the common extrarenal in ¬
volvement in autosomal dominant polycystic kidney
8
disease: ( ARMS Nov 04 )
175. Medullary cystic kidney disease is best diagnosed by:
a. Mitral valve prolapse b. Hepatic cysts
a. Radio nucleotide scanning [ NBE Pattern 2014-15) d. Colonic diverticulosis
c. Splenic cysts
b. Biopsy
c. USG
d . CT Scan
Ref: Harrison’ s 18th ed. ch. 284, Harrison 19 th p 1854
183.
Ref: Harrison 19 th p 657, Harrison 19 th p 1851
Medullary
nosed by:
cystic disease of the kidney is best diag ¬
I
a. Ultrasound (AI 2002)
176. All of the following is decreased in Nephrotic syn ¬ b. Nuclear scan
drome except ( NBE Pattern 2014-15) c. Urography
a. Transferrin b. Ceruloplasmin d . Biopsy Ref Harrison 19 th p 1854
c. Albumin d. Fibrinogen
.
Ref Harrison's 18th ed ch. 283, 44, Harrison 19 th p 252
I Miscellaneous
177 . Nephrocalcinosis is seen in all except - ( ARMS May 07)
a . Sarcoidosis 184. Electrolyte abnormality seen in chronic renal failure:

Kidney
( Manipal )
b. Distal RTA
c. Milk alkali syndrome a. Metabolic Acidosis High Anion Gap
d. Medullary cystic kidney b. Metabolic Alkalosis
c. Hypokelemia
Ref Harrison 18th/ p 2382, 2383, Harrison 19 th p 1854
d . Hypercalcemia
178. A patient is known to have calcium nephrocalcinosis Ref: Harrison 19 th p 1873
for the post 10 years. All of the following dietary rec ¬
ommendations should be suggested, Except: 185 . Hyperkalemia with acidosis is a feature of: ( Manipal] j
( A1IMS Nov 2010) a. RTA TYPE I b. RTA TYPE II
a . Protein Restriction c. RTA TYPE IV d. ARF
b. Calcium Restriction Ref: Harrison 19 th p 312
c. Salt Restriction 186. Wide anion gap is not seen in: (Manipal)
d . All of the above a. Acute renal failure b. Diarrhea
179. True about adult polycystic kidney disease is all. c. Lactic acidosis d . Diabetic ketoacidosis
except: Ref: Harrison 19 th p 317 / 64 e - 8
a . Autosomal dominant inheritance 187 . Edema in nephrotic syndrome is due to: ( Manipal)
b. Hypertension is rare ( ARMS 2001 )
a. Increased intravascular blood volume
c. Can be associated with cysts in liver, lungs and pan ¬ b. Hypoproteinemia
creas c. Lymphatic obstruction
d . Pyelonephritis is common
d. Increased colloid onchotic pressure
Ref Harrison 18th/ p 2355, 2356, Harrison 19 th p 1851
Ref: Harrison 19 th p 1841
180. Which of the following is associated with adult poly¬
188. C3 Nephritic factor is seen in which of the following
cystic kidney disease? ( ARMS 2001 )
conditions: (Manipal)
a. Berry Aneurysm in Circle of Willis
a. Membranoproliferative glomerulonephritis
b. Saccular aneurysms of aorta
b. Membranous glonerulonephritis
c. Fusiform aneurysms of aorta
d. Leutic aneurysms
c. Focal GN
*
d . IgA Nephropathy
Ref Harrison 18th/ p 2356, Harrison 19 th p 1851
Ref: Harrison 19 th p 252

Ans. 174. b. Chr 16 and 4 175. d. CT Scan 176. d . Fibrinogen 177. d. Medullary cystic kidney
178. b. Calcium Rest... 179. b. Hypertension is rare 180. ..
a . Berry Aneurysm . 181. a . Lung
182. b. Hepatic cysts 183. d . Biopsy 184. a. Metabolic Acidosis...185. c. RTA TYPE IV 199
186. b. Diarrhea 187. b. Hypoproteinemia 188. a . Membranoproliferative glomerulonephritis
Marwah 's Internal Medicine MCQ s ( Base d on Harr ison 's 19th
)

189. Drug contraindicated in chronic renal failure: 196 . A 30 - year - old man presents with generalized edema
a. Calcium carbonate with meal ( Manipal ) and hypertension. Urine examination shows subne -

a
b. Aluminum hydroxide before meal phrotic proteinuria ( < 2gm) and microscopic hema ¬
c. Magnesium hydroxide with meal turia . Serum complement levels are decreased and he
d . An of the above is positive for anti -hepatitis C antibodies. The most
Ref: Kumar anti Clark pp 649, 650 Harrison 19th p 2049 likely diagnosis is: (AI 2012)
190. Painless hematuria is a feature of: ( Manipal ) a . Poststreptococcal Glomerulonephritis ( PSGN )
a. IgA nephropathy b. Cryoglobulinemia
b. Stone in urinary bladder c. Membranoproliferative Glomerulonephritis (MPGN
c. Stone in renal bladder C> B
d. Trauma d. Focal Segmental Glomerular Sclerosis (FSGS)
Ref Kumar and Clark pg 605 Harrison 19 th p 1861 / 1839 Ref Harrison's 18/ e p2344, Harrison 19th p 1835 t
191. Hyperuricemia is not caused by: ( Manipal ) 197. Nephrocalcinosis is seen in all except: ( A1IMS May 07)
a. Von Gierke’s disease a. Sarcoidosis
b. Lesh nyhan syndrome b. Distal RTA
c. Renal failure c. Milk alkali syndrome
d. Gilbert's disease Ref Harrison 19 th p 2002 d. Medullary cystic kidney
192 . Bence Jones proteins are seen in which condition: Ref Harrison's 18/ e p2382, 2382, Harrison 19 th p 1850
a. Waldenstrom's macroglobulinemia ( Manipal )
198. Important clues of diagnosis chronic renal failure
b. Malignant melanoma (UP 2009)
(CRF) except:
c. Multiple myelonta
a. Bilateral contracted kidney
d. Chronic renal failure

Kidney
b. Massive proteinuria
Ref George Mathews pg 45 Harrison 19th p 714
c. Osteodystrophy
193. A 40 - year - old man presented with painless haematu - d. Hypertension
ria . Bimanual examination revealed a ballottable mass Ref: Harison's 18/ e p33517/ep269 , Harrison 19 th p 291 e- 6
over the right flank. Subsequently right nephrectomy
was done and the mass was seen to be composed of 199. Necrotizing pappillitis may be seen in all of the
cells with clear cytoplasm. Areas of hemorrhage and following conditions except: ( AI 2002)
mass necrosis were frequent cytogenetic analysis of a. Sickle cell disease
this mass is likely to reveal the abnormality of: b. Tuberculous pyelonephritis
a. Chromosome 1 ( Manipal ) c. Diabetes mellitus
b. Chromosome 3 d. Analgesic nephropathy
c. Chromosome 11 Ref Harrison's 18/ e p2367, Harrison 19 th p 867
d. Chromosome 17 Ref Harrison 19 th p 294
200. A child presented with intermittent episodes of left
194. Which of the following is a feature of contrast induced sided flank pain . Ultrasonography reveals a large hy ¬
nephropathy? ( Manipal )
dronephrosis with dilated renal pelvis and cortical
a. Increased bilirubin thinning with a normal ureter. Kidney differential
b. Increased creatinine function was observed to be 19% which of the follow ¬
c. Decreased creatinine ing is the best management: (AI 2010)
d. Decreased bilirubin a. Nephrectomy
Ref Harrison 19th p 440e -2, 1661, 1803, 1820 b. Pyeloplasty
195 . An 70 - year old
- male patie nt with unc ontr olled hype r ¬
c. External Drainage
tension has seru m crea tinin e of 4.5 , mild prot einu ria . d. Endopylostomy
Renal ultrasou nd show s left kidn ey 9 cm and right 7
10 cm ) . Ther e was no obst ruc ¬ 201. Which of the following drugs is not a part of the
cm in length (normal
tion Wha
. t is the next inve stiga tion of choi ce ? Triple Therapy’ immunosuppression for post - renal
a. 1VP (JIPMER 2014) transplant patients? ( AI 2006 )
b. MR angiography a. Cyclosporine
c. Isotope renogram b. Azathioprine
d. Retrograde pyelography c. FK 506
Ref : Harrison 19 th p 1620 d. Prednisolone

191. d. Gilbert’s disease 192. c. Multiple myelonta


Ans . 189. b . Aluminum hydro.. 190 . a . IgA nephropathy
195. c. Isotope renogram 196. b & c. Cryoglobul ...
193. b. Chromosome 3 194. b. Increased creatinine
199. b. Tuberculous... 200. b. Pyeloplasty
197. d. Medullary cystic... 198. d . Hypertension
200
201. d . Prednisolone
Kidney

child with bigger limbs (history suggestive of hemi - 209. A 65 year old male smoker presents with gross total

I
202 . A
hypertrophy) and abdominal mass is suggestive of: painless hematuria. The most likely diagnosis is:
a. Aldosteronoma ( ARMS May 2013 ) a. Carcinoma of urinary bladder ( Al 2003)
b. Neuroblastoma b. Benign prostatic hyperplasia
c. Carcinoma prostate
c. Rhabdomyosarcoma
d. Wilms tumor
Ref: Nelson Textbook, 18th ed., ch. 492, Harrison 19 th p 101 e-6 t
d. Cystolithiasis
Ref Bailey 23rd/ 1229, Harrison 19 th p 575
8
203. The most common histological variant of renal cell 210. Renal tubular acidosis are A/ E: ( NBE Pattern 2014-15)
carcinoma is: [AllMS Nov 2005) a. Impaired acid production
a. Clear cell type b. Chromophobe type b. impaired bicarbonate resorbtion
c. Papillary type d. Tubular type c. Inability to acidify urine
RefiHarrison 17th/ 592, Harrison 19th p 579 d. Nephrolithiasis
204. Classic triad in Renal cell carcinoma includes all of Ref: Harrison’s 18th ch. 284, 47
the following, except: ( Al 1991 ) 211. The following are the complication of haemodialysis
a . Hematuria except: ( NBE Pattern 2014-15)
b. Hypertension a. Hypotension
c. Flank mass b. Peritonitis
d. Abdominal Pain c. Hypertension
RefHarrison 18th/ p 793, Harrison 19 th p 579 d. Bleeding tendency
205. All can be seen in hypernephroma, except: Ref: Harrison’s 18th ed. ch.281, Harrison 19 th p 1822

Kidney
a. Polycythemia ( ARMS Nov 93) 212 . All of following are features of Bartter's syndrome
b. Renal vein thrombosis except: ( NBE Pattern 2014-15)
c. Hypertension a. Hypertension
d. Hematuria b. Periodic paralysis
Ref: Harrison 18th/ p 793, 2382, Harrison 19 th p 579 c. Alkalosis
206 . All are associated with Wilm’s tumor except: d. Polyuria
a. Anirida [ARMS Feb 1997) Ref: Harrison’s 18th ed. ch. 284, Harrison 19 th p 306
b. Male pseudohermaphrodite
213 . All of the following may be associated with massive
c. Arthogryposis multiplex congenital
proteinuria except: ( NBE Pattern 2014-15)
d. Hemihypertrophy
a. Amyloidosis b. Renal vein thrombosis
Ref: Robbin's 7th/ 604, 605; Harrison 19 th p 101 e -6 t c. Polycystic kidneys d . Microscopic polyangitis
207 . The most important determinant of prognosis in Ref: Harrison's 18th ed. ch. 284, 283, Harrison 19th p 1851
Wilm’s tumor: ( AI 2006 )
214. The most likely diagnosis in the case of a patient with
a. Stage of disease
b. Loss of heterozygosity of chromsome lp multiple pulmonary cavities, hematuria and red cell
c. Histology casts is: -
( NBE Pattern 2014 15)
d. Age less than one year at presentation a . Anti -GBM disease
b. Churg-Strauss
RefiDevita 6 th/ 2177, Harrison 19 th p 101 e -6 t
c. Systemic lupus erythematosus
208. Which of the following statements is incorrect with d . Wegner's granulomatosis
regard to Hepatorenal syndrome in a patient with Ref: Harrison’ s 18th ed. ch. 283, Harrison 19 th p 2185
cirrhosis ( Al 2003)
a . Createnine clearance < 40 ml / min 215. Not seen with uremic lung? ( NBE Pattern 2014-15)
b. Urinary sodium < lOmq / L a . alveolar injury
c. Urine osmolality lower than plasma osmolality b. Pulmonary edema
d . No sustained improvement in renal function after c. Interstitial fibrosis
volume expansion. d . Fibrinous exudate in alveoli
Ref: Harrison 19 th p 1809 Ref: Harrison’ s 18th ed. ch. 280., Harrison 19 th p 1796

Ans. 202. d . Wilms tumor 203. a . Clear cell type


206. c. Arthogryposis ...
210. a. Impaired acid ...
207.
211.
a & c. Stage...
b. Peritonitis
204. b. Hypertension
..
208. c. Urine osmolality.
212. a. Hypertension
205. None
209. a. Carcinoma... I
213. c & d. Polycystic... 201
214. d. Wegner’s granu ... 215. c. Interstitial fibrosis

I
Marwah's Internal Medicine MCQs ( Based on Harrison's 19th )

216. The drugs used in chronic kidney disease could be all, 223. The difference between sodium and chloride is low,
except: ( Bihar PG 2014 ) the metabolic disorder in the patient would be?
a. Ramipril a. Metabolic acidosis ( NBE Pattern 2014- 15)
b. Ibuprofen b. Metabolic alkalosis

3 c. Caicitriol
d . Torsemide
Ref: Goodman and Gillman - ch. 34, Harrison 19th p 1816
217. Gitelman's Syndrome incorrect is: (NBEPattern 2014-15)
a. Hypokalemic metabolic alkalosis
b. Mimics thiazide diuretics
c. Respiratory acidosis
d. Respiratory alkalosis
Ref: Harrison’s 18th ed. ch. 47, Harrison 19th p 322
224. Gitelman's Syndrome differs from Bartter's Syndrome
in all except:
a. Less common
b. Later age of presentation
(APPG 2014 )

c. Hypercalciuria
d. Generally milder clinical course c. Prominent neuromuscular signs and symptoms
d. Generally milder clinical course
Ref: Harrison's 18th ed. ch. 284, Harrison 19th p 306
Ref: Harrison' s 18th ed. ch. 284, Harrison 19th p 306
218. The accurate diagnostic aid in renal artery stenosis is:
a. Selective renal angiography 225. Worsening of kidney function on contrast
b. Ultrasound ( NBE Pattern 2014-15) nephropathy is best evaluated with? ( AI1MS Nov 2013)
a High serum creatinine
c. CT scan
b Low serum creatinine
d . IVU
c. High serum bilirubin
Ref: Harrison's 18th ed. ch. 286, Harrison 19th p 1627 d. Low serum bilirubin
219. Mosaic pattern of cement line is characteristically

Kidney
Ref: Harrison’s 18th ed. ch. 279, Harrison 19th p 289
seen in: ( NBE Pattern 2014-15) 226. Which is characteristic feature of malignant
a. Hyperparathyroid hypertension in kidney? ( NBE Pattern 2014-15)
b. Paget's disease of bone a. Hyaline necrosis
c. Renal osteodystrophy b. Fibrinoid necrosis
d. Osteomalacia c. Medical wall hyperplasia
Ref: Harrison' s 18th ed. ch. 355, Harrison 19th p 426 e-3 d . Micro-aneurysm
220. Recurrence of lesions is seen after renal transplant in Ref: Harrison's 18th ed. ch. 286, 283, Harrison 19th p 1615
all except: -
( NBE Pattern 2014 15) .
227 Dent’s disease is characterised by all except?
a. SLE a. Chloride channel defect (AIPG 2011 )
b. Diabetic nephropathy b. Males are affected
c. Alport’s syndrome c. Nephrolithiasis
d. Goodpasture's syndrome d . Defect in limb of Loop of Henle
Ref: Harrison's 18th ed. ch. 283, 280, Harrison 19th p 2513 Ref: Harrison' s 18th ed. ch.284
221. Renal artery stenosis is associated with: .
228 Osmotic demyelination syndrome develops due to
a . High renin Hypertension ( NBE Pattern 2014 -15) rapid correction of hyponatremia at a level exceeding?
b. Normal renin hypertension a. 0.5mEq / hr ( NBE Pattern 2014 -15)
c. Low renin hypertension b. 2 mEq / hr
d . Fibrinoid necrosis of vessels c. 5mEq / hr
d . lOmEq / hr
Ref: Harrison' s 18th ch. 283, 284, Harrison 19th p 1627
Ref Harrison's 18th ed. ch.45, Harrison 19th p 300
222. Iliac horns on X- ray pelvis is seen in:
a. Alport's syndrome ( NBE Pattern 2014 -15) 229. Anuria is defined as urine output less than?

| b. Nail - Patella syndrome


c. Ehlers-danlos syndrome
d. Marfan's syndrome
a. 4 ml / hr
b. 8ml / hr
c. 12 ml / hr
d. 16 ml / hr
( NBE Pattern 2014 -15)

Ref: Harrison 19th p 292


Ref: Harrison's 18th ed. ch. 283, Harrison 19th p 1847

I Ans. 216.
220.
b. Ibuprofen
.
c. Alport’s syn ..
217. c. Hypercalciuria 218. a. Selective renal...
-
219. b. Paget’s disease of bone
221. a . High renin Hypertension 222. b. Nail Patella syndrome 223. b. Metabolic alkalosis
227. d. Defect in limb of Loop...
224. a . Less common 225. a High serum creatinine 226. b. Fibrinoid necrosis
202
228. a. 0.5mEq / hr 229. a. 4 ml / hr
Kidney

presen ted with urinary infection 237 . Rate of correction of hypernatremia is:
230. A 4- year - old girl ( NBE Pattern 2014 -15)
urine dilatat ion of left ureter a. 0.5 mEq / dl
with E. coli , pus cells in ,
b. ImEq / dl
hydro - ureter; mictur ating cysto - urethr ogram
with c. 2 mEq / dl
fdling defect in bladd er, likely diagno sis is:
shows d. 3mEq / dl
a. Sacrococcygeal teratoma ( NBE Pattern 2014-15) Ref: CMDT 2013, pg. 876 , Harrison 19 th p 2279
b. Ureterocele treatm ent for hyper kalem ia with ECG changes :
238. Best
c. VUR . Calciu m chlorid e ( A1 PG 2011 )
a
d pu.V Ref Nelson ch. 540
b. Insulin drip
231. The kidney in sickle cell anemi a is chara cteriz ed by: c. K* Binding Resin Enema
a. Pyuria ( NBE Pattern 2014 -15) d. Soda bicarbonate
b. Inability to concentrate urine Ref: Harris on ' s 18th ed . ch. 45 , Harris on 19 th p 312

239. In a child non - functio ning kidney is best diagn osed


c. Decrease in glomerular filtration ,
d . Inability to acidify the urine by . ( AI 2005 )

Ref: Harrison' s 18th ch. 285, Harrison 19 th p 636 a. Ultrasonography.


b. IVU.
232 . Dialysis disequilibrium occurs due to: c. DTPA renogram .
a. Cerebral edema ( NBE Pattern 2014 -15) d. Creatinine clearance.
b. Hypertension 240. Fractional excretion of sodium < 1 is seen in
c. Alumunium toxicity a. Prerenal azotemia ( AIIMS Nov- 07)
d. A Beta 2 amyloid deposition b. Acute tubular necrosis
Ref: Harrison' s 18th ch. 281, Harrison 19 th p 250 c. Acute ureteral obstruction
233. Neonate with recurrent seizures with hypocalcemia,
recurrent infections. Diagnosis is?
a . Di George syndrome
b. Barter syndrome
c. Gitelman syndrome
( NBE Pattern 2014 -15)
d . Interstitial nephritis
RefHarrisons 16th/ 1649, Harrison 19 th p 1809
241. Which of the following changes does not occur in
malignant hypertension :
a. Peticheal Haemorrhages on cortical surface
b. Fibrinoid necrosis of arterioles
( AI 2008) Kidney
d. Idiopathic hyercalciuria c. Intimal concentric thickening
Ref: Nelson 18th ed. ch. 430, Harrison 19th p 2113 d . Hyaline arteriosclerosis
234. Hypercalciuria can be treated with: Ref: Robbins 7th / 1006, 1007, 1008
a. Torsemide ( NBE Pattern 2014 -15 ) 242. Commonest histological finding in Benign
b. Acetazolamide Hypertension is: ( AI 2009)
c. Frusemide a. Proliferative endarteritis
b. Necrotizing arteriolitis
d . Indapamide
c. Hyaline arteriosclerosis
Ref: Harrison’s 18th ed. Goodman and Gillman 12th ed. 284 d. Cystic Medial Necrosis
235. All are true regarding Bartter syndrome except? Ref: Robbins 7th / 1006 , 1007, 1008
a . Hypokalemic alkalosis ( NBE Pattern 2014 -15) 243 . The most common cause of renal scarring in a 3 year
b. Hypomagnesuria old child is: ( A12005)
c. Congenital SN hearing defect a. Trauma.
d . Associated with Barttin mutation b. Tuberculosis.
Ref: Harrison's 18th ed. ch.284, Harrison 19 th p 306 c. Vesicoureteral reflux induced pyelonephritis.
d. Interstitial nephritis.
236. A 35 year old, 70 kg male with sodium =160meq / dl.
Ref: Harrison 19th p 1859
Calculate the free water deficit for this patient:
244. All of the following are true about Rhabdomyolysis,
a. 5 L ( NBE Pattern 2014 -15) except: ( pci Dec 04 )
b. 10 L a. Hyperuricemia
c. 15 b. Hyperphosphatemia
d. 20 L Ref: CMDTpg. 875, c. Hypercalcemia
d. Creatine kinase Ref: Harrison 19th p 1810
Ans. 230. b. Ureterocele
234. d. Indapamide
238. a . Calcium chloride
231. b. Inability to con...
.
235 b. Hypomagnesuria
239. c. DTPA renogram .
232 . a. Cerebral edema
236. a. 5L
240 . a. Prerenal azotemia
233. a . Di George syndrome
237. b. 1 mEq /dl i
241. d. Hyaline arteriosclerosis 203
-
242 c Hyaline arterio... .
243 c. Vesicoureteral reflux... 244. c. Hypercalcemia
I

i Marwah's Internal Medicine MCQs ( Based on Harrison's 19th)

245. Urinary K+ excretion is increased in: (AllMS Nov -04 ) 252. All of the following statement about Renal Calculi are

I a. Bronchiectasis
b. Meningitis
c. Osteomyelitis
true, Except:
a. Cystine stones form in acidic urine
b. Struvite stones form in alkaline urine
(AI 1993)

3
d. Hepatitis Ref: Harrison 19th p 283 c. Oxalate stones are radiopaque
.
246 IOC for ureteric stone? (NEET Pattern 2015-16) d. Uric acid stones are resistant to ESWL
a. CT scan b. USG Ref: Schwartz 8th/ l 547; Bailey 24th/1319,
c. MIBG scan d. DMSA scan .
253 Stone which is resistant to lithotripsy: (AllMS May 07)
Ref: Harrison 19th Ch 342, p 1866 ) a. Calcium oxalate b. Triple phosphate stone
.
247 All are indicated in a 30 year old patient with c. Cystine stone d. Uric acid stone
increased serum cysteine and multiple renal stones RefSchwartz 8th/1547; Bailey & Love 24th/131,
except: (AllMS Nov 2012) .
254 Which of the following stones is hard to break by
a. Cysteamine ESWL: (AI 2010)
b. Increase fluid intake a. Calcium Oxalate Monohydrate
c. Alkalinisation of urine b. Calcium Oxalate Dihydrate
d. Pencillamine c. Uric acid
Ref: CMDT 2012 pg. 923 ch. 287, Harrison 19th p 1866 d. Struvite Ref: Harrison 19th p 143
248. 35 year old female with recurrent renal stone. Not .
255 All of the follwing types of Renal Stones are
advised is ? (AllMS Nov 2012) Radiopaque, Except: (AllMSJune 2000)
a. Increase water intake a. Oxalate b. Uric Acid
b. Restrict protein c. Cystine d. Mixed
c. Restrict salt Ref: Bailey and Love 24th /1317; Smith's Urology 16th/262,
G d. Restrict calcium
a 256. Renal Calculi associated with proteus infection:
Ref: H 18th ed. ch. 281 / pg. 1817, Harrison 19th p 1866 a .
Uric Acid (AI 2009)
.
249 Most common cause of calcium oxalate stones is? b. Triple Phosphate
a. Hyper-parathyrodism (NBE Pattern 2014-15 ) c. Calcium oxalate
b. Idiopathic Hypercalciuria d. Xanthine
c. Dietary intake of milk based products Ref: Bailey 24th/1316.1319; Schwartz 8th/1547
d. Renal tubular acidosis type 1 257. Ureteric colic due to stone is caused by: (AI 2008)
Ref: Harrison's' table 281 -7, Harrison 19th p 2237 a. Stretching of renal capsule due to back pressure
250. Hemolytic-uremic syndrome true is A / E: (AIPG 2011 ) b. Increased peristalsis of ureter to overcome the
a. Uremia obstruction
b. Hypofibrinogenemia c. Irritation of intramural ureter
c. Thrombocytopenia d. Extravasation of urine.
d. Positive Coomb’s test Ref: Smiths Urology 17th / 31, Bailey & Love's 25th ed p 1292
.
Ref: Harrison's 18th ed. ch 286, Harrison 19th p 657 .
258 Referred pain from ureteric colic is felt in the groin
due to involvement of the following nerve: (AI 2003)
251. A patient recently under - went a renal allograft trans ¬

plantation and on treatment with azathioprine and


a .
Subcostal
b. Iliohypogastric
prednisolone he developed, fever and cough with
c. Ilioinguinal
thick sputum and left lower lung consolidation with a
d. Genitofemoral
.
cavity The culture specimen demonstrates gram+ve

I organism with beaded string appearance. Initial treat¬


ment will include:
a. Penicillin
(NBE Pattern 2014-15)
Ref: Gray's Anatomy 38th/18, Bailey & Love's 25th ed p 1293
.
259 T / t used for lower ureteric stone is: (AllMSJune 98)
a. Endoscopic removal

I b. Erythromycin
c. Ceftazidime
d. Sulfisoxazole
Ref Harrison's 18th ed. ch. 162, 282, Harrison 19th p 932
Ans. 245. d. Hepatitis 246. a. CT scan
b. Diuretics
c. Drug dissolution
d. Laser
Ref: Ch 287, Harrison 18th edn, Bailey & Love's 25th ed p 1294
247 . a. Cysteamine 248. d. Restrict calcium
249. b. Idiopathic Hyper... 250. d. Positive Coomb’s test 251. d. Sulfisoxazole 252. d. Uric acid stones are...
253. c. Cystine stone 254. a. Calcium Oxalate... 255. b. Uric Acid 256. b. Triple Phosphate
204
257. b. Increased peri... 258 . d. Genitofemoral 259. a. Endoscopic removal
Kidney

ADDITIONAL UPDATES
Algorithm for Kidney Recipient Care

Donor cold ischemia time >24 h


Donor age > 60 years, or Anti-lymphocyte globulin
High risk
Donor age 50> years with hypertension or “ induction” therapy
Donor kidney biopsy >20% glomerulosclerosis

Donor cold ischemia time, 12 h, and Steroids


Donor physiology ideal, and Low risk Calcineurin inhibitor
Donor 15-35 years old Mycophenotic acid mofetil

ril »] »] 11[•] ? M
^* I »7A \ 14' ADDITIONAL UPDATES
a
9
The Most Common Opportunistic Infections in Renal 1
I Major Causes Of Papillary Necrosis
n
Transplant Recipients *<
Analgesic nephropathy
Early ( 1-6 months ) Prolonged NSAID use (rare)
1. Pneumocystis jiroveci Abbreviation: NSAID, nonsteroidal anti-inflammatory drug.
2. Legionella Sickle cell nephropathy-
3. Hepatitis B Diabetes with urinary tract infection
4. Cytomegalovirus
5. Listeria
6. Hepatitis C
Late (>6 months)
1. Aspergillus
2. Nocardia
3. Herpes zoster
4. Hepatitis B
5. Hepatitis C
6. BK virus (Polyoma)

205
CHAPTER Hematology
9
Most Recent Q's 2014- 15 7. Which of the following is a quantitative defect of
hemoglobin? ( DNB Pattern )
1. Best blood product to be given in a patient of multiple a. HbC
clotting factor deficiency with active bleeding? b. HbD
a. Fresh Frozen Plasma ( AIIMS May 2015) c. Sickle cell disease
b. Whole blood d. Thalassemia Ref: Harrison 19 th p 637
c. Packed RBCs
8. What is Evan’s syndrome? ( DNB Pattern )
d. Cryoprecipitate
a . ITP with cryoglobulinemia
Reference: el 38/ page 724: Harrison 19 th edition b. ITP with hemolytic anemia
2. Which of the following is false about TRALI? c. ITP with spherocytosis
a. Develops after 24 hours (AIIMS May 2015) d. ITP with myelodysplastic syndrome
b. Mostly seen after sepsis and cardiac surgeries Ref: Harrison 19 th p
c. It's a cause of non cardiogenic pulmonary edema
d . Plasma is more likely to cause it than whole blood 9. A patient presents with Hb of 8 gm%, WBC count of
Ref: ch 22: wintrobe's hematology 12th edition 2000 / mm3 and platlet count of 60000/ mm3. What is
your likely diagnosis? ( DNB Pattern )
3. Clotting factor activated on platelets a . Thalassemia
(JIPMER June 2015)
b. Sickle cell anemia
a. 8 b. 9
c. Aplastic anemia
c. 10 d. 11
d. Anemia of chronic disease
Ref: ch 20, WintrobeHematoidgy 12th Edition H - 19e: 733f
Ref: Harrison 19 th p 667
4. Girl with Von Willebrand Disease Previous history of
heavy bleeding during tooth extraction. Prophylaxis 10. What is the TIBC in Anemia of chronic disease vs Iron
before tooth extraction: (JIPMER 2014 ) deficiency anemia? ( DNB Pattern )
a. Desmopressin b. Blood transfusion a. Increased in both
c. Fresh frozen plasma d. Cryoprecipitate b. Decreased in both
Ref. Harrison 19 th p 275 - 276, 400 c. Increased in Anemia of chronic disease and
decreased in Iron on deficiency anemia
5. Pregnant lady during antenatal check up gives family d . Decreased in Anemia of chronic disease and
history of sister developing thromoembolism during increased in Iron on deficiency anemia
her second pregnancy. Investigations reveal she is
heterozygous for factor 5 leiden deficiency. Next step Ref: Harrison 19 th p 1809 / 1810
in management is: (JIPMER 2014 ) 11 . Refsum’ s disease is due to deficiency of ?
a. Warfarin a . Sphingomyelinase ( DNB Pattern )
b. Low dose aspirin b. Phytanoyl -CoA-hydroxylase
c. Low molecular weight heparin c. Alpha - galactosidase-A
d. Observation and report if any calf pain d . N -acetylglucosaminidase
Ref : Harrison 19th p 1646
Ref: Harrison 19 th p 206
6. On medical check up of a Punjabi student following
findings were seen Hb of 9.9gm / dl , RBC count of 12 . Bone scan of multiple myeloma shows: (JIPMER 2014 )
5.1million, MCV of 62.5 fl and RDW of 13.51%. What a. Diffuse increased uptake
is the most probable diagnosis? ( DNB Pattern ) b. Diffuse decreased uptake
a . HbD b . Thalassemia trait c. Hot spot
c. Iron deficiency anemia d. Cold spot Ref: Harrison 19 th p 721
d . Anemia of chronic disease
Ref: Harrison 19 th p 637
Ans . 1. a . Fresh Frozen ... 2. a. Develops after 24 hours 3. d . 11 4. a. Desmopressin
5. c. Low molecular.. 6. b. Thalassemia trait 7. d. Thalassemia 8. b. ITP with hemolytic anemia
9. c . Aplastic anemia 10 . d . Decreased in Anemia ... 11. b. Phytanoyl -CoA... 12. d . Cold spot
Hematology

Transfusion Medicine 20. Most common complication of massive transfusion:


a. Hypothermia ( NBE Pattern 2014-15) |
13 . A 30 year old male was transfused blood in operating b. Acidosis
room following which he develops tachycardia, hypo ¬ c. Hyperkalemia
tension ( BP- 70 /40 mm of Hg) and hematuria within d. Hypocalcemia Ref: Harrison 19 th p 138e -5
15 minutes. Most probable cause is;
a. ABO incompatibility (JIPMER 2014) 21. Cryoprecipitate does not contain? ( NBEPattern 2014-15)
b. Anaesthesia drug hypersensitivity a. Factor 8
c. DIC d. GVHD b. Factor 9
Ref.: Harrison's, 18the edn. Pg: 954, Harrison 19 th p 138e - l c. Von wilebrand factor
d. Fibrinogen
14. Pre - transfusion testing does not decrease the inci ¬
Ref: Hematology in clinical practice. 4“ ed ch. 37 ,
dence of Hepatitis. Possible reason could be? Or Rea ¬
son for post transfusion hepatitis even when screen ¬ 22 . Agranulocytosis is defined as neutrophil count less j
ing done before transfusion? (JIPMER 2014 ) than: ( NBE Pattern 2014-15) |
a. Most carriers do not have HBsAg
b. Post transfusion hepatitis is caused by CMV
c. Present screening test in not sensitive for HBsAg
a. 100 / pL
b. 200 / pL
c. 300 / pL
I
d . HCV not screened d . 500 / pL
Ref: Harrison' s, 18th edn. Pg. 956, Harrison 19 th p 2004 Ref: Harrison’s 18th ed. ch. 107 , Harrison 19 th p 237
15. Sugar responsible for ABO blood group expression?
a. Fucose (JIPMER 2014 ) 23 . Platelets in stored blood do not live after: A

b. Galactose a . 24 hours ( NBE Pattern 2014-15) M

c. Glucose b. 48 hours 3
d . Ribose c. 72 hours jjj?
Ref: Harrison 18th edn, ch 113, Harrison 19 th p 138e - l d. 96 hours Ref: Harrison 19 th p 138e -4 Q
16 . All are complications of blood transfusion except? 24. Which one of the following is the cause of non - Q
a . Hypomagensemia ( NEET Pattern 2015-16 ) cardiogenic pulmonary oedema seen in immunologic QfQ
b. Hypokalemia blood transfusion reaction: ( NBE Pattern 2014- 15)
c. Hypocalcemia a. Antibody to IgA in donor plasma
d. Metabolic alkalosis b. Antibody to donor leukocyte antigen
Ref: Wintrobe’s Hematology 12th edn, Harrison 19 th p 138e -5 c. Donor antibody to leukocyte of patient
d. RBC incomapatibility
17 . Blood transfusion should be completed within
hours of initiation : ( NBE Pattern 2014-15)
.
Ref: Harrison' s 18th ed , p. 955, Harrison 19 th p 138e -5
a. 1-4 hours b. 3-6 hours 25 . What is the Neutrophil count for moderate neutrope ¬
c. 4-8 hours d . 8-12 hours nia: ( NBE Pattern 2014-15)
Ref: Harrison 19 th p 138e- E a. < 500 / mm 3
18 . b. 500-1000 / mm 3
Which of the following is given to treat thrombocyto¬
penia secondary to anti - cancer therapy and is known c. >1000 / mm 3
to stimulate progenitor megakaryocytes? d . 100 / mm 3
( NBE Pattern 2014-15) Ref: Hematology in clinical practice. 4 th ed. ch. 16
a. Filgrastim b. Oprelvekin 26. A patient with previously normal hemoglobin suf ¬
c. Erythropoietin d. Anagrelide fered a sudden massive acute hemorrhage . He is most
Ref: H 18th ch. 85 a , Harrison 19 th p 103e -24 likely to show all of the following except:
19. Platelets transfusion must be completed in: a . High reticulocyte count ( NBE Pattern 2014-15)
a. 1 hour ( NBE Pattern 2014-15) b. High neutrophil count
b. 2 hour c. High packed cell volume
c. 3 hour d. 4 hour d. high MCV
Ref: Wintrobe's Hematology 12th ed. 1683, .
Ref: Harrison'sW / p 885, 886

Ans - 13. a . ABO incom ... 14. b. Post transfusion ... 15. a. Fucose 16. b. Hypokalemia
-
17. a . 1 4 hours
21. b. Factor 9
18. b . Oprelvekin 19. d . 4 hour 20 . a. Hypothermia
22. a. 100/ pL 23. c. 72 hours 24. c. Donor antibody to ...
-
5. b. 500 1000/ mm 3 26. c. High packed cell volume
Marwah's Inter nal Med icine MCQ s ( Base d on Harr ison 's 19 th
)

27. True about blood transfusion reaction: 34. Shelf life of platelets in blood bank is:
a . 5 days ( NBE Pattern 2014 -15)
( NBE Pattern 2014 -15)
a. Complement mediated severe hemolysis b. 7 days
b. Extravascular hemolysis c. 10 days
c. Transfusion should not be stopped d . 21 days Ref: Harrison 19 th p 138e-5
d. Death unlikely 35 . Storage temperature of platelets is: (NBEPattern2014-15)
Ref: Harrison's 18th p. 954 - 956 , Harrison 19 th p 138e -5 a. - 4 degrees Celsius b. + 4 degrees Celsius
c. - 20 degrees Celsius d . + 20- 24 degrees Celsius
28. Shelf life of blood in a blood bank in CPDA buffer:
Ref Harrison 19th p 138e -4
-
a 21 days
b. 30 days
( NBE Pattern 2014 -15)
36 . Which of the following is the finding in functional
c. 35 days defect in platelets? ( NBE Pattern 2014 -15)
d. 42 days Ref Harrison 19th p 685 a . Normal platelet counts and prolonged bleeding time
29. Complication of blood transfusion can be all except: b. Normal platelet count and bleeding time
a. Hyperkalemia ( NBE Pattern 2014 -15) c. Prolonged bleeding time, prothrombin time and
b. Citrate toxicity PTT
c. Metabolic acidosis d . Thrombocytopenia and prolonged bleeding time
d . Hypothermia Ref Harrison's 18th b/ e p. 970- 971; Harrison 19 th p 138e - 4
Ref Harrison's 18th ed. ch. 113, Harrison 19th p 138e-5
Transfusing blood after prolonged storage could lead
5oft 30. The following is not true of platelet transfusion:
a. Useful in ITP ( NBE Pattern 2014-15)
37.
to:
a. Citrate intoxication
( NBE Pattern 2014-15)

b. Used in D.I .C.


o c. Effective for 9-10 days
b. Potassium intoxication
c. Circulatory overload
d. Effect decrease with repeated usage
d. Haemorrhagic diathesis
£a> Ref Harrison' s 18th ed. p. 953, Harrison 19 th p 138e - l
Ref Harrison's 18thp. 956, Harrison 19th p 138- e
31. Most common complication of blood transfusion: 38. Acute renal failure in a patient who received
X a. Transfusion associated hepatitis incompatible blood transfusion with hemolytic
b. Hyperkalemia (NBE Pattern 2014-15) reaction is best prevented by: ( NBE Pattern 2014-15)
c. Hemolysis a. 20 % Mannitol
d . Febrile non hemolytic transfusion reaction b. IV fluids with K + supplementation
Ref H 18th ed. ch. 113 Table. 113.3, Harrison 19 th p 138e c. Alkalinizing the urine
d. Stopping blood transfusion
32 . Bone marrow transplantation indicated in all except: Ref Harrison's 18th ed. ch. 113, Harrison 19 th p 138e -5
a. Osteopetrosis ( NBE Pattern 2014 -15)
b. Mucopolysaccharidosis 39. Incorrect about platelet transfusion:
c. Hemochromatosis a . Platelets carry the Rh antigenNBE Pattern 2014-15)
d . Beta -Thalassemia b. Platelets bear the antigens of ABO system
Ref Harrison' s 18th
ed. ch. 114, Harrison 19th p 139e- l c. Six units for healthy adult
d. Transfuse to attain count of 50 X 109 / L
33 . There are more than 400 blood groups, we consider Ref: Harrison 19 th p 138e
only ABO blood group, because: ( NBE Pattern 2014 -15)

I a. Presence of antibodies in the serum when the RBC


lacks the corresponding antigen
b. A & B antigens are secreted by cells which are not in
40. Blood transfusion may be indicated in:
a. Acute pulmonary oedema ( NBE Pattern 2014 -15)
b. Epistaxis with hypertension

I
the circulation only. c. Congestive cardiac failure
c. Soluble blood group antigens cannot block binding d. Acute leukaemia
of organisms to polysaccharides. Ref: Harrison's 18th ed. ch. 109 , Harrison 19 th p 138e-5
d . These antigens are lipoproteins
Ref : Ch. 113, Harison's 18th ed., Harrison 19th p 138e - l

I Ans. 27.
31.
35.
.
a Complement...
d. Febrile non hem...
d . + 20 -24 degrees...
28.
32.
36.
c. 35 days
c. Hemochromatosis
a. Normal platelet...
29. c. Metabolic acidosis
33. a. Presence of anti... 34. a. 5 days
37. b. Potassium intoxication 38. d. Stopping blood...
-
30. c. Effective for 9 10 days

208 39. a. Platelets carry... 40 . d . Acute leukaemia


Hematology
f
41. Platelet transfusion is not indicated in: ( A11996 ) 48. Rarest type of von wilebrand disease:
a. Dilutional Thrombocytopenia a . vWD type 1 ( NBE Pattern 2014 -15 ) I
b.
c.
Immunogenic Thrombocytopenia
Aplastic Anemia
b. vWD type 2 A
c. vWDtype 2 N i
I
Ref: Harrison 18th/ p 968, d. vWD type 3

9
d. DIC
Ref: H 18th ed. ch. 115 ch. 31, Harrison 19th p 732
42. MC blood transfusion reaction is- ( AI 2008)
a. Febrile nonhemolytic transfusion reaction 49. A Patient has ecchymosis and petechiae ail over
b. Hemolysis the body with no hepato-splenomegaly. All are true
c. Transmission of infections except? ( NBE Pattern 2014-15)
d . Electrolyte imbalance Ref Harrison 18th/ p 953, a. Increased megakaryocytes in bone narrow.
b. Bleeding into the joints
43. All of the following infections may be transmitted via
c. Decreased platelet in blood
blood transfusion, except: (AI 2002)
d. Disease resolves itself in 80% of Patients in 2-6
a . Parvo B-19
weeks. Ref: Harrison's 18th ed. p. 968,
b. Dengue virus
c. Cytomegalo virus 50. Most common cause of DIC is: ( NBE Pattern 2014-15)
d. Hepatitis G virus Ref: Harrison' s 18th edn. ch. 113, a . Sepsis
b. Placenta previa
c. Abruption placentae
Bleeding Disorders d . Snake bite

Hematolgy
Ref: Harrison's 18thed. ch. 116 , Harrison 19 th p 736
44. Which statement is most appropriate for DIC:
a. Increase fibrinogen (JIPMER 2014 ) 51. A 45 year old lady with normal PT and increased
b. Increased D dimer aPTT. About 2 year back, she was operated for cho ¬
c. Resolves if causative factor removed lecystectomy & did not have any bleeding episode.
d . Absence of bleeding abnormalities rules out DIC What is next investigation for clinical diagnosis?
Ref : Harrison 19 th p 736 a. Factor VIII assay ( NBE Pattern 2014 -15)
b. Dilute russel viper venom assay
45. A chronic alcoholic presenting with bleeding gums c. Platelet aggregation test
and petechiae is more likely to have deficiency of: d . Ristocetin Cofactor assay
a. Vitamin B 12 (JIPMER 2014 )
Ref: Harrison's 18th ch. 58 & ch. 319 , Harrison 19th p 2131
b. Vitamin C
c. Thiamine 52. Most common inherited bleeding disorder:
d. Pyridoxine a. Von wilebrand disease ( NBE Pattern 2014-15)
Ref : Harrison’s, 18th edn, Harrison 19th p 96 e-lt, 96 e-5 b. Bernard soulier
c. Glanzmann thrombasthenia
46. -
Girl with von willebrand disease. Prophylactic action d. ITP acute
to be taken before wisdom tooth extraction? Ref H 18th ch. 115 ch. 31 , Harrison 19th p 731
a. Epsilon amino-caproic acid (JIPMER 2014 )
b. Ciyoprecipitate 53. Seen in D.I.C all except: ( NBE Pattern 2014-15)
c. Desmopressin a. Hyperfibrinogenimia
d. Abciximab b. Increase fibrin degradation products
Ref: Harrison 18th edn, ch 115, Harrison 19th p 731/ 732 c. Prolonged PT
d . Increased APTT
47. Pregnant lady on her ANC has family history of throm ¬
Ref: Harrison's 18th ed. ch. 116 , Harrison 19th p 736
boembolism (factor 51eiden deficiency) in heterozy¬
gous state. So next management is? (JIPMER 2014 ) 54. Which one of the following is not used in DIC ?
a . Warfarin a. Heparin ( NBE Pattern 2014- 15)
b. Heparin b. Epsilon amino caproic acid
c. Aspirin c. Blood transfusion
d. Observation and calf pain report ASAP d. I.V. fluids
Ref: Harrison's 18th edn, Ch 262, Ref Harrison's 18th ed. ch. 116 , Harrison 19th p 736

Ans - 41.
45.
b. Immunogenic...
b. Vitamin C
42. a . Febrile non ... 43. b. Dengue virus 44. b. Increased D dimer
49 . b. Bleeding into...
46.
50.
-
a. Epsilon amino caproic... 47. d. Observation and calf ... 48. d . vWD type 3
a. Sepsis 51. b. Dilute russel viper... 52. a. Von wilebrand disease
53. a. Hyperfibrino... 54. d. I .V. fluids
\
k
Marwah'

55.
s Interna l Medicin e MCQs ( Based on Harriso n's

In Hemophilia B what is most common cause of death?


19 th
)

62. Apheresis is: (NBE Pattern 2014-15)


I a. Hemorrhage (NBE Pattern 2014-15) a. Selective separation of components of blood
I b. HIV,HBV, HCV due to transfusions b. Preventing blood transfusion infections (HIV, HBV)
c. Transfusion reactions c. Separation of platelets from plasma
K . d. Deep vein thrombosis d. Isolating organisms from mixed culture
I Ref: Harrison's 18 th
.
ed. ch 116 , Harrison 19th p 733 Ref Harrison's 18 th
ed. ch. 113, Harrison 19th p 138e -2
K 56. Anti- factor VIII antibodies are seen in: 63 . Most common cause of febrile non haemolytic trans ¬

a. Postpartum (NBE Pattern 2014-15) fusion reaction? (NBE Pattern 2014-15)


b. Hemophilia who have received infusion of plas- a. ABO mismatch b. Rh mismatch
maconcentrates c. HLA mismatch d. All of the above
c . Both A&B
Ref Harrison 19th p 138e- 4/ 5
d. None
64. Bleeding crisis in acute Idiopathic thrombo - cytopenic
Ref: Hematology in clinical practice. 4 ed. ch. 32
th
Purpura is managed by all except:
57. In Beta thalassemia, the most common gene mutation a. RhIG (NBE Pattern 2014-15)
is: (NBE Pattern 2014-15) b Prednisolone
a. Intron 1inversion c. Intravenous immunoglobulin
b. Intron 22 d. Eltrombopag
c. 619 bp deletion
Ref: Harrison 's 18 . .
th ed ch 115, Harrison 19th p 728
d. 3.7 bp deletion
.

Hematolgy
Harrison's 18th ed. ch. 104, 106 , Harrison 19th p 637 65 Apixaban is a new drug that acts by:
Ref a. Inhibiting TNF alpha (NBE Pattern 2014-15)
58. Anticoagulant of choice for coagulation testing is? b. Inhibiting coagulation factor Xa
a. Trisodium citrate 3.2% (NBE Pattern 2014-15) c. Inhibiting platelet aggregation
b. EDTA d. Activating plasminogen
c. Heparin Harrison's 18th
ed. ch. 118, Harrison 19 th p 756
Ref:
d. Sodium oxalate
Ref: Wintrobe 11th ed. p. number 28., Harrison 19th p 749
66 . The coagulation profile in a 13 - year old girl with
Menorrhagia having von Willebrands disease is:
59. A 2 5 - year - old female presented with history of recur¬
(NBE Pattern 2014-15)
rent abortions. The most relevant investi- gation to
a. Isolated prolonged PTT with a normal PT
identify the cause is: (NBE Pattern 2014-15)
b. Isolated prolonged PT with a normal PTT
a. Bleeding time c. Prolongation of both PT and PTT
b. Prothrombin time d. Prolongation of thrombin time
c. Dilute russel viper venom time
Ref: Harrison' s 18th
/ p.971, 972, Harrison 19th p 732
d. Clot solubility test
67. Bleeding tendencies are common in:
Ref .
Harrison's 18th ed. ch. 58 and ch 319, Harrison 19th p 740 a. Chronic myeloid leukaemia (NBE Pattern 2014 -15)
60. Feature of hemorrhagic disease of new born is: b. Chronic lymphocytic leukaemia
a. Prolonged prothrombin time (NBE Pattern 2014-15) c. Acute monocytic leukaemia
b. Defective platelet count d. None of the above
c. Prolonged bleeding time Ref https:/ / www.orpha.net/ data/ patho/ GB/ uk-AMLM5.pdf
d. Prolonged thrombin time 68. All of the following are true about Warfarin therapeu ¬

' th ed. p. 980


Ref: Harrison s 18 tic usage except (NBE Pattern 2014-15)
a. The INR is maintained between 2 &3
61. Not an Indicator of mismatched Blood transfusion in
b. It is very useful in the prophylaxis of thrombo ¬

patient under general anesthesia:


(NBE Pattern 2014-15) embolism
a. Fever with chills
c. It’s effect is monitored by observing the clotting
b. Hypotension
time
c. Excessive oozing of blood from surgical site
d. Used in DVT
d. Passage of black urine
Hematol ogy : 11 th ed
. p. 1701 Ref: Harrison' s 18 th
ed. ch. 118, Harrison 19th p 756
Ref: Wintrobe

Ans. 55. a . Hemorrhage 56. c. Both A &B 57. a. Intron 1 inversion 58 . a . Trisodium citrate 3.2%
59. c. Dilute russel... 60. a. Prolonged prothrombin... 61. a. Fever with chills 62. a . Selective separation...
63. c. HLA mismatch 64. d. Eltrombopag 65. b. Inhibiting coagulation... 66. a . Isolated prolonged...
210 c. It’s effect is monitored...
67 . c . Acute monocytic... 68 .
Hematology

69. Wrong about FFP: ( NBE Pattern 2014 15) - 77. The commonest mode of inheritance of Von Wille ¬
a. Stored at minus 18 degrees brand’s disease : ( AllMS Dec 97)
b. Given for reversal of warfarin toxicity a. Codominant
c. ABO matching is mandatory before transfusion b. Autosomal recessive

70.
d. Deficient in factor 5 and 8
Ref: Hematology in clinical practice. 4“ ed. ch. 37and ch. 33
Bleeding time is increased in all except:
a. Thrombocytopenia
b. Thrombasthenia
c. Renal failure
d. Acquired hemophilia
( NBE Pattern 2014-15]
78.
c. Autosomal dominant
d. X-linked recessive Ref: Harrison 18th/ p 971,
The presence of small sized platelets on the peripheral
smear is characteristic of:
a. Idiopathic thrombocytopenic purpura
b. Bernard soulier syndrome
c. Disseminated intravascular coagulation
d . Wiskott Aldrich syndrome
( ARMS Nov. 03)
E
Ref: Harrison' s 18‘h ch. 116, Harrison 19 th p 138e - 3 Ref: Harrison 18th/ p 2702, 2703,
71. von Willebrand’ s disease is usually inherited as 79. Thrombocytopenia occurs in all except: ( A! 2001 )
a. Autosomal dominant ( NBE Pattern 2014 - 15] a. Henoch schonlein purpura
b. Autosomal recessive b. TTP c. DIC
c. X-linked recessive d . Wiskott Aldrich syndrome
d . Multicentric Ref: Harrison 19 th p 731 Ref Harrison 18th/ p 2797,
72. Which one of the following platelet counts is usually 80. Autoimmune destruction of platelet is seen in :

Hematolgy
associated with increased incidence of spontaneous a. SLE (AIIMS May 95)
bleeding: ( NBE Pattern 2014-15) b. Rheumatoid arthritis
a. Greater than 80,000 / mm 3 c. Reiter disease
b. 40, 000 / mm 3 d. Polyarteritis nodosa Ref: Harrison 18th edn. ch. 319,
c. 20,000 mm 3
Palpable purpura is seen in ( ARMS May 94 )
d . Less than 20,000 mm 3 Ref Harrison 's 18thp. 461
a. Idiopathic thrombocytopenic purpura ( ITP
73 . All are true about warfarin, except: b. Scurvy
( NBE Pattern 2014-15) c. Henoch Sconlein Purpura
a. It causes inhibition of vitamin K dependent clotting d. Monoclonal cryoglobulinemia
factors Ref Harrison 18th/ p 422, 2797,
b. Its half -life is 36 hours
82 . Predisposing factor for arterial thrombosis:
c. It can cross placenta a. AT III deficiency ( PGI Dec. 04 )
d. Its dose is increased in liver disease b. Protein S deficiency
Ref: Katzung 1 l' h/ 595-596 , Harrison 19 th p 7 E c. Protein C deficiency
74. Feature of hemorrhagic disease of new born is - d. Homocystenemia Ref: Harrison 18th edn. ch. 117,
a. Prolonged prothrombin time ( A1IMSJune 97) 83. Cryoprecipitate contains all except - ( AIIMS Nov 07)
b. Defective platelet count a. Factor VIII b. Factor IX
c. Prolonged bleeding time c. Fibrinogen d. VWF
d. Prolonged thrombin time Ref Ghai 6 th/ 31;
Ref: Harrisons 18th edn. ch 113,.
75 . A patient is on aspirin, what will be the finding?
a. Prolonged BT (AI 2007) Anemia
b. Prolonged PT
c. Prolonged APTT 84. 45 - year - old lady diagnosed to have anemia . Hb - 7.8,
d. Prolonged CT Ref: Robbins 8th ch. 12, MCV - 72 and Serum ferritin - 8. After 1 moth of iron
76 .
therapy with adequate dose, Hb is still 8. Most prob ¬
All are true about hemophilia, except (ARMS May 94 )
able cause for treatment failure is. (JIPMER 2014)
a . Increased Bleeding time (BT)
a. Non-compliance
b. Decreased factor VIII
b. Acquired sideroblastic anemia
c. Decreased factor IX
d . Increased partial thromboplastin time ( PTT) c. Inadequate intake of iron
d. Folate deficiencies
Ref Harrison 18th / p 974;
Ref : Harrison's 18th, Harrison 19 th p 1809, 393
ns. 69. d . Deficient in factor... 70. d . Acquired hemophilia 71. a. Autosomal dominant 72. d. Less than 20 , 000 mm 3
73. d . Its dose is ... 74. a . Prolonged prothrombin ... 75. a. Prolonged BT 76. a. Increased Bleeding time...
77. c. Autosomal... .
78. d . Wiskott Aldrich .. 79. a. Henoch schonlein ... 80. a. SLE
c ~ Henoch Sconlein ... 82. 211
d . Homocystenemia 83. b. Factor IX 84. -
a. Non compliance
Marwah's Internal Medicine MCQs ( Based on Harrison's 19th )

85. Earliest change in Iron deficiency anaemia is: 91. All of the following are features of hemolytic anemia
a . Erythroid hyperplasia ( yp 2009 ) except: ( PGI June 04)
b.

i
Subjective feeling of the body and increased appetite a. Decreased Rbc life span
c. Lymphoid erythroplasia b. Decreased haptoglobin
d. Reticulocytosis c. Unconjugated hyperbilirubinemia
Ref : Harrison 19 th p 1809, 1810 d . Bilirubin in urine
e. Altered erythroid and myeloid ratio
86. Which of the following is not a characteristic of Ref : Harrison' s 18 / e p873, Table 106 -2 , Harrison 19 th p 393
Fanconi's anemia ? ( Feb DP PGMEE 2009 ) 92. Hemolysis in G 6 PD may be caused by all except:
a. Hematologic abnormalities in infancy ( AI 2008)
a. Primaquine
b. Pancytopenia b. Chloroquine
c. Skeletal anomalies c. Pyrimethamine
d. Chromosome fragility d. Quinine
Ref : Harrison' s 18/e p496 , 889 , Harrison 19th p 434 Ref: Harrison' s 18 / e p878, Harrison 19 th p 656
87. Macrocytosis without myeloblastic changes occur 93. Coomb's + ve Hemolytic Anaemia is seen in except:
with: ( MP PG 2008) a. Alcoholic cirrhosis (AI 2000 )
a. Pernicious anemia b. Chronic active hepatitis
b. Aplastic anemia c. Primary biliary cirrhosis
d . Primary sclerosing cholangitis
c. Sideroplastic anemia

Hematolgy
d. Sickle cell anemia Ref Harrison' s 18 / e p881, Harrison 19th p 393
Ref Harrison' s 18 / e p867 , 17 / e p649 , Harrison 19 th p 81 e - 3f 94. -
The differential diagnosis of micro angiopathic ane¬
mia includes all except: ( PGI Dec 05)
88. - -
A child has Hb 6.5 gm %, MCV-65, MCH 15 and proto ¬ a. Sepsis
porphyria with red cell distribution width much less b. Hemolytic uremic syndrome
is most likely to be suffering from: c. MI
a. Thalassemia ( ARMS June 2000 ) d. Eclampsia
b. Iron deficiency anaemia e. Scleroderma
c. Porphyria
Ref Harrison' s 18 / e p883, 17 / e p658 , Harrison 19 th p 1810
d. Megaloblastic anaemia
95. Microangiopathic hemolytic anemia is seen in all
Ref : Harrison' s 18 / e p847 , 17 / e p631, 632, Harrison 19 th p 625
except: ( PGI June 01, 00 )
89. All of the following are causes of iron deficiency ane¬ a. HUS
mia, except: ( PGI June 01 ) b. ITP
c. Malignant hypertension
a. Chronic renal failure
d. Prosthetic valves
b. Celiac Sprue
e. TTP
c. Hookworms
Ref Harrison' s 18 / e p883 , 17 / e p658, Harrison 19 th p 393
d. Carcinoma colon
Ref : Harrison' s 18 / e p846 , Table 103-2 , Harrison 19 th p 1807 96. Tactoids are seen in: ( J & K 2010 )
a. Multiple myeloma
90. Which of the following is true about oral therapy for b. Thalassemia
iron deficiency anemia: ( PGI Dec 05) c. G6 PD deficiency
a. In 300 mg elemental iron given 100 mg get absorbed d. Sickle cell anemia
b . Reticulocytosis appears in 1 to 2 weeks and then Ref Harrison's 18/ e p854, Harrison 19 th p 634
peaks in 3- 4 weeks 97. Thrombocytosis may be seen in all of the following
c. Hemoglobin levels are usually corrected in six except: (J & K 2011 )
months of initiating therapy a. Haemolytic anaemia
d. Decrease in absorption with improvement of b. Polycythemia rubra vera
symptoms c. HIV infection
e. Stop the Rx after normalizing the Hb d. Myelofibrosis
Ref Harrison' s 18/e p848, 17/ e p632 , Harrison 19 th p 1810 Ref Harrison' s 18/e p970, Harrison 19th p 393

Ans. 85. d . Reticulocytosis 86. a . Hematologic abnorm... 87. a. Pernicious anemia 88. b. Iron deficiency anaemia
89. a. Chronic renal... .
90 . d. Decrease in absorption .. 91 . d . Bilirubin in urine 92. c. Pyrimethamine
93. a. Alcoholic cirrhosis 94. None 95. b. ITP 96. d. Sickle cell anemia
212 97. a . Haemolytic anaemia
Hematology

Plasmap heresis is indicate d in each of the following 105. Iron requirement is determined from the equation:
98 ( NBE Pattern 2014-15)
except:
a. 2.3 x wt (kg) x Hb deficit (g / dl) + 500
a. Hemolytic-uremic syndrome
b. Good Pasteur's syndrome
b. 3.3 x wt. ( kg ) x Rb deficit (g / dl) + 1000
c. 4 xwt. ( kg) x Hb deficit (g / dl) + 1000 ; gr * '

c. Polycythemia vera
-
d. Landry Guillain Barre syndrome
, 2342 , 900 , Harriso n 19th p 673
d. 4.3 x wt. ( kg) x Hb deficit (g / dl) + 1500
Ref: ch. 103, Harriso n' s Iff 1 ed., Harrison 19th p 628
'
Ref : Harriso n' s 18le p 2353
99. Secondary Polycythemia may be seen in: ( DNB 2012 ) in all of the
106. Plasma ferritin levels may be reduced
a. Cor pulmonale followin g conditio ns , except : ( NBE Pattern 2014 - 15 )
b. Congestive cardiac failure a. Iron deficiency b. Vitamin C deficiency
c. Acyanotic congenital heart disease c. Liver disease d. Hypothyroidism
d.
Ref: Harrison’s Iff * / e p. 845, 846 table ( 103-2,
All of the above
Ref : Harrison' s 18 / e p456 , Harrison 19th p 1506
100. Increased LDH is seen in: ( Rajasthan 2009 ) 107. Which of the following types of anemia is associated
a . AML b. PAN with a Raised MCV and Normal MCHC?
c. CML d. All of the above a. Sideroblastic anemia ( NBE Pattern 2014 -15 )

Ref : Harrison' s 18 / e p907-910, Harrison 19th p 678 b. Vitamin B12 and Folic acid deficiency
c. Beta thalassemia
101. Screening tests for B cell defects: ( DP PGMEE 2009 ) d . Iron deficiency anemia

Hematlogy
a. Isohemagglutinin titers
Ref: Hematology in clinical practice. 4* ed. ch. 8, Harrison 19th p 643
b. CD 4 levels
c. NItroblue tetrazolium dye test 108. Megalocytic anaemia is caused by all, except:
d . Candida albicans intradermal skin test a. Goat milk ingestion ( NBE Pattern 2014-15 )
Ref: Harrison' s 18 /e p2703, 17 /e p2035, Harrison 19th p 372e 6 t - b. Type A gastritis
c. Antimetabolites
Iron Deficiency Anemia d. Lead poisoning
.
Ref: Harrison's Iff* ed ch. 103, Harrison 19th p 643
102. All the following are suggestive of iron -deficiency 109 Iron is mainly absorbed in the: ( NBE Pattern 2014-15)
.
anemia except: ( NBE Pattern 2014-15) b. Duodenum
a. Stomach
a. Koilonychia c. Jejunum d. Ileum
b. Pica
Ref : Ch.103 Harrison's Iffh ed., Harrison 19th p 643
c. Decreased serum ferritin
d. Decreased total iron - binding capacity (TiBC) 110. Macrocytic anemia occurs in: ( NBE Pattern 2014-15)
. .
Ref: Harrison’s 18 ed ch 103, Harrison 19th p 627 a. Hypothyroidism
“ b. CRF
103. A patient with microcytic hypochromic anemia, c. Anemia of chronic disease
Hb-9%, serum iron is 20 g/ dl, ferritin level 800 ng / d. Vitamin-C deficiency
ml, transferrin percentage saturation is 64. What is
possible diagnosis: “ .
Ref: Harrison' s 18 ed. ch. 105; ch 341 , Harrison 19th p 643
a. Atransferrinemia ( NBE Pattern 2014-15) 111. Oral iron supplements used for iron deficiency ane¬
b. Iron deficiency anemia mia : ( NBE Pattern 2014-15)
c. DMT 1 mutation a. Tolerable dose will deliver 40 to 60 mg of iron per
d. Hemochromatosis day
Ref: Manual of clinical hematology 3rd ed p. 33 . b. Mass of total salt is important in determining daily
104. Filgrastim is used for the treatment of: dose
a. Neutropenia ( NBE Pattern 2014-15) c. Treatment should be stopped as soon as normal
b. Anemia hemoglobin level is reached
c. Polycythemia d. Desired rate of hemoglobin improvement is 0.5 mg
d. Neutrophilia per day
Ref: Katzung’s 11th 580-581, Harrison 19th p 103e-24 Ref: Katzung 10"' / 530;KDT 6th/ 585-86, Harrison 19th p 398
Ans. 98 .
102.
106 .
110 .
c. Polycythemia vera 99 .
d . Decreased total .. . 103.
c. Liver disease
a . Hypothyroidism 111 .
107 .
a . Cor pulmonale
a . Atransferrinemia
100. a . AML
104. a . Neutropenia
b. Vitamin B12 and Folic... 108 . d . Lead poisoning
a . Tolerable dose will ...
101 . a . Isohemagglutinin titers
105. a . 2.3 xwt...
109 . b. Duodenum J
\
Internal Medicine MCQs ( Based on Harrison's 19 < h
)
Marwah 's

112. A 30 - yrs old female, RBC count 4.5 million, MCV 55fl, 118. AH of the following are features of iron deficiency
TLC 7000, no history of blood transfusion. Diagnosis

a
anemia, except: ( AllMS MAY 2013)
is ? ( NBE Pattern 2014-15) a. Increased RDW
a. Iron deficiency anemia b. Decreased Serum Iron
b. Thalassemia major c. Decreased TIBC
c. Thalassemia minor d . Decreased Serum Ferritin
d. Megaloblastic anemia Ref Harrisons 18th edn. Ch. 109 ,
Ref: Harrison' s 18th ed. ch. 104, Harrison 19 th p 637
119 . The earliest sign of iron deficiency anaemia :
113. Which one of the following is true regarding Thalas ¬ a. Increase in iron binding capacity (AIIMS Feb 1997)
semia Major? ( NBE Pattern 2014-15) b. Decrease in serum ferritin level
a. Normocytic normochromic anemia
c. Decrease in serum iron level
b. Enlargement of medullary cavity
d . All the above Ref: Ch 109 Harrison 18th/ edn ,
c. Iron deficiency anemia
d. Megaloblastic anemia 120. Most sensitive and specific test for diagnosis of iron
Ref Harrison's 18 h ed. ch. 104, Harrison 19 th p 633, 637 deficiency is: (AI 2003, AI 2001 )
‘ a. Serum iron levels
114. All of the following are characteristic features of
b. Serum ferritin levels
treatment of iron deficiency anemia with oral iron
c. Serum transferrin receptor population
supplements, except: ( NBE Pattern 2014 -15)
a. Bioavailability is enhanced with vitamin C d . Transferrin saturation

Hematolgy
b. The proportion of iron absorbed reduces as hemo¬ Ref: Harrison 18th/ p 846
globin improves 121. Iron overload occurs in all, except : (AIIMS Sept 96 )
c. The reticulocyte count should begin to increase in a. Thalassemia

response to treatment

two weeks and peak in 4 weeks this suggests good b. Myelodysplastic syndrome
c. Polycythemia vera
d . The treatment should be discontinued immediately d. Sideroblastic anaemia
once hemoglobin normalizes to prevent side effects Ref: Harrison 18th/ p 846, 900, 3162,
of iron.
122 . Seen in chronic inflammatory anemia is: ( PCI Dec 99)
Ref Harrison' s 18th ed. ch. 103, Harrison 19 th p 628 a. Serum iron I, S. ferritin T and transferrin i
115 . All of the following cause Microcytic Hypochromic b. Serum iron S. ferritin f and transferrin f
anemia except: (AI 1995/ AIIMS 1997) c. Serum iron j, S. ferritin j and transferrin f
a . Lead poisoning d . Serum iron S. ferritin [ and transferrin J.
b. Thalassemia Ref: Harrison 18th/ p 849,
c. Iron deficiency anemia
123. Haptaglobin levels are decreased in: ( AI 1996 )
d. Fanconi’s anemia Ref: Harrison 18th/p 847, 848,
a. A mismatched transfusion reactions
116. Anemia of Chronic disease can be differentiated from b. Thalassemia
Iron deficiency anemia by: ( PCI 2009 ) c. G 6 PD deficiency
a . T TIBC d . All of the above Ref: Harrison 18th edn. ch. 106 ,
b. 1 TIBC
124. Reticulocytosis is NOT a feature of: ( AIIMS Dec 94 )
c. T S.ferritn a . Paroxysmal nocturnal hemoglobinuria
d . 1 Fe store in marrow
b. Following acute bleeding
e. i Ferritin Ref Harrison 18th/ p 848,
c. Hereditary spherocytosis
117 . Elevated serum ferritin, serum iron and percent d. Anemia in CRF
transferrin saturation are most consistent with the

!
Ref: Harrison 18th edn. ch. 106 ,
diagnosis of : (AI 2004)
a. Iron deficiency anemia 125. Osmotic fragility is increased in: ( PGIJune 98)
b . Anemia of chronic disease a. Sickle cell anemia
c. Hemochromatosis b. Thalassemia
d. Lead poisoning c. Hereditary spherocytosis
Ref: Harrisons 17th edn / 2432, 2433, 631 , d. Chronic lead poisoning Ref: Harrison 18th/ p 875;

Ans. 112. c. Thalassemia... 113. b. Enlargement of med ... 114. d. The treatment... 115. d. Fanconi’s anemia
116. b & C **TIBC... 117. c. Hemochromatosis .
118. c Decreased TIBC 119. b. Decrease in serum...
214
120. b. Serum ferritin... 121. c. Polycythemia vera 122. a. Serum iron .. . 123. d . All of the above
124. d. Anemia in CRF 125. c. Hereditary spherocytosis
Hematology

Hemolytic Anemia
-
133. Warm -antibody immune hemolytic anemia is seen in
all except: ( NBE Pattern 2014 15 ) - |
a. SLE
with all of tile
126' Hemolytic anemia are associated
( NBE Pattern 2014-15)
-
b. a Methyldopa ingestion
following, except: c.Quinidine
a. Increased indirect bilirubin in the serum d. Infectious mononucleosis
b. Decreased red cell survival Ref: Harrison's 18"' ed. ch. 106 , Harrison 19th p 658
c. Increased number of reticulocytes
d. Increased fecal Urobilinogen 134. Anemia of chronic disease is associated with:
e. Increased urine Urobilinogen
( NBE Pattern 2014 15) -
. a. Increased Fe, decreased Transferrin, increased
Ref: Harrison's 18thed. ch 106 , Harrison 19 th p 658/ 1649
Ferritin
127. 2 P.N.H is associated with all of the following condition, b. Decreased Fe, decreased Transferrin, increased
except: ( NBE Pattern 2014-15) Ferritin
a. Aplastic anemia c. Decreased Fe, increased Transferrin , decreased
b. Increased LAP scores Ferritin
c. Venous thrombosis d . Increased Fe, increased Transferrin, decreased
d. Iron deficiency anemia Ferritin
,
Ref: Harrison s 18 h ed. ch. 103, Harrison 19 th p 393
Ref: Harrison's 18th ed. p. 884, Harrison 19 th p 662 '

135. Which of the following causes of Anemia is associated

Hematolgy
128. All are true about G6 PD except: ( NBE Pattern 2014-15)
with a Hypoplastic marrow? ( NBE Pattern 2014- 15)
a. Bite cells
a. Fanconi’s Anemia
b. Intravascular hemolysis
b. Paroxysmal Nocturnal Hemoglobinuria
c. Favism
c. Hypersplenism d. Myelofibrosis
d. Confers protection against plasmodium vivax
Ref. Harrison’s 18"’ ed. p. 887, Harrison 19 th p 434 / 664
Ref: Harrison’ s 18th ed. p. 878-880 , Harrison 19 th p 656
136. Pernicious anemia is associated with all of the
129. Reticulocyte count in Hemolytic jaundice is greater following, except: ( NBE Pattern 2014-15)
than? ( NBE Pattern 2014 - 15) a. Macrocythaemia
a. 0.5% b. 1% b. Vitamin B ) 2 deficiency
c. 1.5% d . 2.5% c. Weakness, numbness & tingling of extremities
.
Ref: Harrison's 18th ed ch. 57 d. Increased reticulocyte count
130. Coombs negative hemolytic anemia is seen in: Ref: Harrison’ s 18th ed. ch. 105 , Harrison 19 th p 2346 t
( NBE Pattern 2014-15) 137. Which of the following statements about paroxysmal
a. Micro-angiopathic hemolytic anemia cold hemoglobinuria is NOT true?
b. SLE c. CLL ( NBE Pattern 2014- 15)
d . Rh incompatibility a. Chronic autoimmune form responds well to
Ref: Harrison's 18th ed. ch. 106 , Harrison 19 th p 658 splenectomy
131. Coomb's positive hemolytic anemia associated with: b. Results from formation of Donath - Landsteiner
a. TTP ( NBE Pattern 2014-15) antibody
b. PAN c. Attacks are associated with hemoglobinuria
c. SLE d . Can occur secondary to syphilis
d. HUS Ref: Harrison' s 18» ed. p. 882, 883, Harrison 19 th p 662
Ref: API, 18?/ ed. 810 , Harrison 19 th p 658/ 138e - 4 138. All of the following are true regarding BI2 deficiency,
132. Spur cell anemia is seen in: ( NBE Pattern 2014-15) except: ( NBE Pattern 2014-15)
a. Drug induced anemia a. Pernicious anemia
b. Hepatocellular disease b. Sub-acute combined degeneration of cord
c. Renal disease c. Carpal tunnel syndrome
d. Alcoholism d. Infertility
Ref: Harrison's 18th ed. ch. 103, Harrison 19 th p 396 f Ref: Ch. 105, Harrison's 18‘h ed., Harrison 19 th p 643
Ans - 126. d . Increased fecal... 127.
b. Increased LAP scores 128. d . Confers protection ... 129. d. 2.5%

d
130 . a . Micro-angio... 131. c. SLE 132. b. Hepatocellular..
134. b. Decreased Fe... 135. a. Fanconi’s Anemia 133. d . Infectious mononucleosis
136. d . Increased ... 137. a . Chronic autoimmune...
' 38. c. Carpal tunnel syndrome
i Marwah's Internal Medicine MCQs ( Based on Harriso n's 19 th
)

.
I 139. All of the following are true about Congenital Haemo 147 The following protein defects can cause hereditiary
¬

lytic anemia, except: ( NBE Pattern 2014-15) spherocytosis except : (AI 07)

z
a. Increased fragility a .
Anykyrin
b. Splenomegaly b. Palladin
c. Splenectomy is useful c. Glycophorin C
d. Positive direct Coomb's test d. Anion transport protein
Ref: Harrison's 18thed. ch. 106, Ref: Harrison 18th edn. ch. 106,
140. Which one of the following is true regarding Thalas ¬

.
148 Splenectomy is most useful in : (AI 1998)
semia Major ? (NBE Pattern 2014-15)
a. Normocytic normochromic anemia
a .
Thrombocytopenia
b. Hereditary spherocytosis
b . Enlargement of medullary cavity
.
c. H S. purpura
c. Iron deficiency anemia
d. Megaloblastic anemia d. Sickle cell anemia Ref: Harrison 18th /876;
Ref: Harrison's 18th
ed. ch. 104 , Harrison 19th p 633, 637 149. The mother has sickle cell disease; Father is normal;
Chances of children having sickle cell disease and
141. The gold standard test for the diagnosis of Paroxysmal
sickle cell trait respectively are: (AI 2001)
Nocturnal Hemoglobinuria (PNH) is:
a. Flow cytometry (NBE Pattern 2014-15)
a. 0 and 100%
b. Sucrose hemolysis test b. 25 and 25%
c. HAM test c. 50 and 50%
Ref: Harrison 18th/ p 855,

Hematolgy
d. None d. 10 and 50%
Ref: Harrison' s 18 th
ed. ch. 106, Harrison 19th p 660 .
150 Microangiopathic Hemolytic anemia is seen in all
Except: (ARMS Nov 2009)
142. Spherocytosis is associated commonly with:
a. Hypernatraemia (NBE Pattern 2014 -15) a. TTP
b. Hyperkalaemia b. Metallic heart valve
c. Hyponatraemia d. Hypokalaemia c. Microscopic polyangitis
' 18 th
.
ed. ch 106, Harrison 19th p 281, 81e-3f d. Anti-phospholipid syndrome
Ref: Harrison s
143. Not present in Sideroblastic anaemia is: (AIIMSFeb 97)
Ref: Rubin's Pathology 8th.edn ch 12/ p471,
a. Microcytic anaemia .
151 PNH is associated with all of the following conditions,
b. Decreased transferrin saturation except : (AI 2002 )
c . Sideroblast cells in blood smear film a. Aplastic anemia
d. Ineffective erythropoiesis Ref: Harrison 18th / p 848, b. Increased LAP scores
c. Venous thrombosis
144. Macrocytic anemia may be seen with all of the
following conditions except: (AI 1998, AIIMS 1996) d. Iron deficiency anemia
a. Liver disease Ref: Harrison 18th /883, 884;
b. Copper deficiency
c. Thiamine deficiency .
152 All of the following are true about PNH, Except:
d. Orotic aciduria Ref: Harrison 18th / p 862 , a. Hypocellular marrow (PGI Dec 2000)
b. Budd- chiari syndrome
145. All are seen in Hemolytic anemia except: (AI 1996) c. Thrombosis
a. Hemosiderinuria d. LAP score low
b. Reticulocytosis
c.
d.
Spherocytosis
Increased haptoglobin Ref: Harrison 18th/p 873,
.
153 All of the following statements about Fanconi's
anemia are true, Except. (AI 2010 )
146. All are features of hemolytic anaemia except: a. Autosomal dominant inheritance
a. Thrombocytopenia (ARMS Dec 95) b. Hypocellular Bone Marrow
b. Hemosiderinuria c. Congenital Anomalies
c. Decreased haptoglobin d. Usually normocytic / macrocytic cell morphology
d. Raised indirect bilirubin
Ref: Harrison's 18th edn. ch. 107,
Ref: Harrison 18th edn. ch. 106,
Ans. 139. d. Positive direct... 140. b. Enlargement... 141. a. Flow cytometry 142. b. Hyperkalaemia
143. b. Decreased... 144. b. Copper deficiency 145. d . Increased... 146. a. Thrombocytopenia
147. c. Glycophorin C 148. b. Hereditary spherocy... 149. a. 0 and 100% 150. b. Metallic heart valve
216
151. b. Increased LAP... 152. a. Hypocellular marrow 153. a. Autosomal dominant inheritance
Hematology

Haemoglobinopathies 161. X- ray skull characteristically shows " Hair standing on


end" appearance in one of the following disease :
a. Still’s disease ( NBE Pattern 2014 -15 )
154. Persistent priapism is due to: ( NBE Pattern 2014 - 15)
b. Scurvy
a. Sickle cell anaemia c. Thalassemia major
b. Hairy cell leukaemia
c. Paraphimosis
d . Urethral stenosis
d. Cirrhosis of liver
Ref Harrison's 18lh ed. ch. 104, Harrison 19 th p 638
162 . Hypochromic microcytic anemia is seen in all , except:
9
Ref: Harrison’ s 18"' ed. ch. 104, Harrison 19 th p 635 ( NBE Pattern 2014 - 15 )
a. Thalassaemia
155. Regarding to Thalassemia minor the following is
incorrect: ( NBE Pattern 2014-15)
a. Hypochromic microcytic cells
b. Raised HbA 2
b. Tropical Sprue
c. Ancylostomiasis
d . Chronic inflammatory disease
Ref Harrison's 18‘" ed. ch. 103, Harrison 19 th p 673
I
c. Severe anemia 163. A young female has the following lab values: Hemo- ]
d. RBC count increased globin = 9.8 gm%, MCV = 70 serum iron = 60, serum fer¬
Ref: Harrison' s 18th ed. ch. 104, Harrison 19 th p 638 ritin = 100, the diagnosis is:
a. Thalassemia trait ( NBE Pattern 2014-15 )
156. Leukoerythroblastic picture may be seen in all of the
b. Chronic iron deficiency anemia
following except: ( NBE Pattern 2014-15)
c. Megaloblastic anemia
a. Myelofibrosis b. Metastatic carcinoma

Hematolgy
d. Anaemia of chronic infection
c. Gaucher’s disease d . Thalassemia
Ref: Harrison's 18"' / 849, Harrison 19 th p 637
Ref Harrison's 18 ed. p. 900, 901 , Harrison 19 th p 638
“ 164. A 20 year old female presenting with anemia, mild
157 . Basic defect in HbS is: ( NBE Pattern 2014 15) - jaundice for 2 years, peripheral smear showing
a. Altered function spherocytes, the best investigation to be done is:
b. Altered solubility ( NBE Pattern 2014-15)
c. Altered stability a. Reticulocyte count
d. Altered 02 binding capacity b. Osmotic fragility test c. Coomb’s test
Ref Harrison's 18th ed. pg. 854 d. Bone marrow aspiration
Ref Nelson 17th/ p. 1620 , Harrison 19 th p 658
158. Anti - D ( Rho) Ig is used for the prevention of:
a. Sickle cell disease ( NBE Pattern 2014- 15) 165. HbH is characterized by: ( NBE Pattern 2014 -15)
b. Hemorrhagic disease of newborn a. Deletion of three alpha chain genes
c. Paroxysmal haemoglobinuria b. Deletion of three alpha chains and one beta chain
d . Hemolytic disease of newborn
genes
, c. Deletion of two alpha and two beta chain genes
Ref Harrison’s 18 h ed. ch. 113, Harrison 19 th p 649 d . Deletion of four alpha chain genes
159. Low serum haptogloblin in hemolysis masked by: Ref Harrison' s 18thp. 859 Harrison 19 th p 638
a . Liver disease. ( NBE Pattern 2014-15) 166. Not useful for treatment of hemophilia B?
b. Pregnancy. a. FFP ( NBE Pattern 2014 - 15 )
c. Bile duct obstruction b. Cryoprecipitate
d. Malnutrition c. Factor 9 concentrates
Ref Hematology in clinical practice. 4th ed. ch. 11 d. Prothrombin complex concentrates
160. Sickle cell anemia is usually associated with all, . .
Ref Harrison's 18th ed ch 116 , Harrison 19 th p 733, 450 t
except:
a. Shortened RBC life span
( NBE Pattern 2014 15)- 167. The most appropriate drug used for chelation therapy
in beta thalassemia major is: ( NBE Pattern 2014 - 15)
b. Normal reticulocyte count a. Oral desferrioxamine
c. Abnormality in Hemoglobin b. Oral deferiprone
d. Small sized spleen c. Intramuscular EDTA
Ref Harrison’s 18th ed. ch. 104, Harrison 19 th p 635
d. Oral succimer
Ref KDT' s 6 th / 868, Harrison 19 th p 638
ns - 154 . a . Sickle cell . .. 155. c . Severe anemia 156 . d . Thalassemia 157 . b. Altered solubility
158 . d . Hemolytic ... 159 . c . Bile duct obstruction 160 . b. Normal ... 161 . c. Thalassemia major
*;• b. Tropical Sprue 163. d. Anaemia of chronic... 164 . c . Coomb’s test 165. a . Deletion of three
166. b. Cryoprecipitate 167 . b. Oral deferiprone
I

L
th
Marwah's Internal Medicine MCQs ( Based on Harriso n's 19 )

168. A child has Hb - 6.5 gm %, MCV-65, MCH -15 and 175. Crew haircut appearance in X- rays skull and Gandy

I
p - v : .
protoporphyria with red cell distribution width much
less is most likely to be suffering from :
a . Thalass emia ( AIIMS June 2000 )
gamma bodies are seen in:
a. G 6- PD deficiency
b. Hodgkin's lymphoma
(AIIMS Nov 93 )

b. Iron deficiency anaemia c. Hereditary spherocytosis


U 5J c. Porphyria d. Sickle cell anaemia
yfl d . Megaloblastic anaemia 176. All of the following are true about p thalassemia trait,
Ref: Harrison 18th/ p 847-848, Except: ( PGI June 2008)
169. Anemia of chronic disease is characterized by all, a . Microcytic hypochromic picture
except (AIIMS May 94 ) b. ted HbA 2
a. Decreased serum iron c. ted HbF
b. Increased total iron binding capacity (TIBC) d. Patient requires blood transfusion
c. Increased serum ferritin Ref Harrison 18th/ p 859,
d. Increased macrophage iron in bone marrow 177. True about p- thalassemia trait: ( PGIJune 06 )
Ref: Harrison 18th/ p 849 , a. Increased HbF b. Increased HbA 2
170. Megaloblastic anemia in blind loop syndrome is due c. Microcytosis d . Severe anemia
to: (AI 99 ) Ref: Harrison 18th/859;
a. Vitamin B12 malabsorption 178. The peripheral blood smear of a patient shows
b. Bacterial overgrowth features of thalassemia, also presented with anaemia.
Cft c. Frequent diarrhoea
Decrease iron intake
Family history is also + ve. The investigation done to
o d.
-
Ref: Harrison 16th/ 1773 74; Love&Bailey 23rd/ 1031,
establish the diagnosis is: ( AIIMS 1999 )
a. ESR estimation
o
& .
171 Which doesn't cause hemolysis in G 6 PD deficiency: b. Blood spherocyte estimation
a. Oestrogen ( PGI Dec 99 ) c. Bone marrow aspiration
Sa b. Salicylates
c. Primaquine
d. Hb -electrophoresis
Ref Harrison 18th/ p 854, 859 ,
>
X d. Nitrofurantoin 179. Haemoglobin F is raised in: ( AIIMS Dec 97)
Ref Harrison 18th/ p 879 , a. Juvenile chronic myeloid leukemia
172. Which of the following is not seen in a chronic case of b. Hereditary spherocytosis
Sickle cell anemia : (AI 1996 ) c. Congenital red cell aplasia
a. Hepatomegaly d. Mysthania gravis Ref Ghai 6th/ 569;
b. Pulmonary hypertension 180. A 20 years old female presenting with anemia, mild
c. Cardiomegaly jaundice for 2 years, peripheral smear showing
d. Splenomegaly spherocytes, the best investigation to be done is:
Ref: Harrison 18th/ p 855, ( AIPGMEE 08 )
173. All are true for sickle cell anemia, except
a. Reticulocyte count
b. Osmotic fragility test
a. Pulmonary arterial hypertension ( AIIMS May 94 )
c. Coombs test
b. Fish vertebra
d. Bone marrow aspiration
c. Leukopenia
d. Increased size of heart Ref Harrison 18th/ p 875„
^
I Ref: Harrison 18th/ p 855, 181. A 23 Year old female presents with anemia and
jaundice for 2 years. Peripheral smear shows
r .
174 Commonest acute presentation of sickle cell anaemia
spherocytes. The best investigation to be done is :
is: ( AIIMS Dec 98)
^1I }
a. Priapism
b. Bone pain
a . Reticulocyte Count
b. Osmotic Fragility Testing
c. Coombs Test
( AIIMS Nov. 2006 )

\ c. Fever
d. Splenomegaly. d. Bone Marrow Aspiration
Ref Harrison 18th/ p 851,

I Ans. 168.
172.
b. Iron deficiency...
d . Splenomegaly
169.
173.
Ref Harrison 18th/ p 855,

...
b. Increased total
c. Leukopenia
170. b. Bacterial overgrowth 171. a. Oestrogen
174. b. Bone pain 175. d. Sickle cell anaemia
ZUi
176. d. Patient requires... 177. c. Microcytosis 178. d. Hb electrophoresis 179. a. Juvenile chronic...
-
180. c. Coombs test 181. c. Coombs Test
Hematology

Myeloproliferative Disorder 189. A patient presented with splenomegaly, anemia


& shows reticulocytosis & increased bone narrow
cellularity. The diagnosis is: ( NBE Pattern 2014-15)
182. Polycythaemia is commonly seen in:
a. Pernicious anemia
a. Congestive cardiac failure ( NBE Pattern 2014-15] b. Hemolytic anemia
b. Hereditary spherocytosis
c. Myelofibrosis
c. Chronic corpulmonale
Uncomplicated ASD d. Hairy cell leukemia
d. |
Ref: Harrison's 18* ed. ch. 106 , Harrison 19th p 658
. .
Ref: Harrison’s 18th ed ch 108,
183. Polycythemia vera is absolute venous hematocrit of >: Aplastic and Myelodysplastic Disorders
a. 45% ( NBE Pattern 2014 -15)
b. 55% 190. Pancytopenia with cellular marrow is seen in all j
c. 65% except: ( NBE Pattern 2014- 15)
d . 70% a. Megaloblastic anemia
.
Ref Harrison's 18* ed p. 899, Harrison 19 th p 674 b. Myelodysplasia
184. Most common myeloproliferative disorder: c. Paroxysmal Nocturnal hemoglobinuria
a. Polycyathemia vera ( NBE Pattern 2014 -15) d . G 6 PD deficiency
b. CML -
Ref H 18th ed. Table 107 1, p. 887 , Harrison 19 th p 656
c. Chronic eosinophilic leukemia 191. A patient aged 65 years, is diagnosed to have severe

Hematolgy
d. Myelofibrosis aplastic anemia . HLA compatible sibling is available.
Ref Harrison's 18“ ' ed. ch. 108 , Harrison 19 th p 673 The best option of treatment is: ( NBE Pattern 2014- 15)
.
185 Isolated deletion of which chromosome causes
a. Anti -thymocyte globulin followed by cyclosporine
myelodysplastic syndrome: ( NBE Pattern 2014-15) b. A conventional bone marrow transplantation from
the HLA identical sibling
a. 2 q
c. A non-myeloablative bone marrow transplant-ation
b. 5q
from the HLA identical sibling
c. 8q
d. Cyclosporine
d . llq
Ref Harrison' s 18th ed. p.: 891, 892, 893, Harrison 19 th p 667
Ref Harrison's 18“ ' ed. p. 895, Harrison 19 th p 670
192. Which is best to prevent rejection after bone marrow
186. Which one of the following is not commonly seen in
transplantation in aplastic anemia?
polycythemia vera? ( NBE Pattern 2014 -15)
a. Anti - thymocyte globulin + cyclosporine
a. Thrombosis b. Prednisolone ( NBE Pattern 2014- 15)
b. Hyperuricemia
c. Cyclosporine
c. Prone for acute leukemia d . Tacrolimus plus prednisolone
d . Spontaneous severe infection
Ref Harrison' s 18th ed. ch. 107, Harrison 19 th p 667
Ref Harrison's 1 <? ed. ch. 108, Harrison 19 th p 673
' '

193. All cause aplastic anemia except:


187. All of the following are true regarding myelo-fibrosis a. Hepatitis B ( NBE Pattern 2014-15)
except: ( NBE Pattern 2014-15)
b. Hepatitis C
a. Tear drop poikilocytes c. Hepatitis D
b. Giant abnormal platelets d. Human Parvo Virus B 19
c. Leucoerythroblastic blood picture
d. Absent spleen . .
Ref Harrison's 18* ed ch 107, Harrison 19 th p 667
.
Ref Harrison's 18“ ’ ed ch. 108, Harrison 19 th p 675 194. Pancytopenia is a feature of: ( NBE Pattern 2014- 15)
188. Myeloproliferative diseases include all except: a. Dengue fever
a. Myelofibrosis ( NBE Pattern 2014-15) b. Enteric fever
b. Chronic neutrophilic leukemia c. SLE
c. Acute myelogenous leukemia d. All of the above
d. Systemic mastocytosis Ref: Harrison's 18th ed. ch. 107, Harrison 19 th p 2132
.
Ref Harrison's IS" ed ch. 108, Harrison 19 th p 672
1

Ans . 182. c. Chronic corpul... 183. b. 55% 184. a. Polycyathemia vera 185. b . 5q
186. d . Spontaneous.. 187. d . Absent spleen 188. c. Acute myelogenous... 189. b. Hemolytic anemia I
-
190. d . G 6 PD deficien .. 191. a . Anti thymocyte globu
194. c. SLE
-
192. a. Anti thymocyte glob.. 193. d . Human Parvo V 219

\
i
th
Marwah 's Internal Medicine MCQs ( Based on 's
Harrison 19 )

195. Pancytopenia may occur in the following except: 202 . Most common cause of kidney involvement in
a. Megaloblastic anemia ( NBE Pattern 2014-15) multiple myeloma: ( NBE Pattern 2014-15)
b. Severe iron -deficiency anemia a. Hypercalcemia
c. Hypoplastic anaemia b. Amyloid deposition
d . Paroxysmal nocturnal haemoglobinuria c. Tubular proteinuria
w| Ref: Harrison’ s 18th ed. ch. 107, Harrison 19 th p 662 d. Hyperviscosity
li 196. Which of the following is NOT seen in Paroxysmal Ref Wintrobe Hematology: p. 5184, Harrison 19 th p 721
Nocturnal Hemoglobinuria: ( AIIMS Nov. 2000 ) 203 . Pop corn variant of Reed -Sternberg cell is seen in:
a. Thrombosis a. Follicular center lymphoma ( NBE Pattern 2014-15)
b. Hemosiderinuria b. Lymphocyte depleted Hodgkin's disease
c. Decreased LDH c. Nodular sclerosis Hodgkin's disease
d. Thrombocytopenia Ref: Harrison 18th/ p 883, 884, d. Lymphocyte predominant Hodgkin's disease
Ref: Robbins 7th ed. p. 686 -68
j 197. Pancytopenia with cellular marrow is seen in:
a. PNH ( AI 2007 ) 204. What is the most effective treatment for chronic
b. G 6 PD deficiency myeloid leukaemia? ( NBE Pattern 2014-15)
c. Acquired aplastic anemia a. Allogeneic bone marrow transplantation
d. Thalassemia b. Heterogeneic bone marrow transplantation
Ref Harrison 18th/ p 883, table 107.1, c. Chemotherapy
d . Hydroxyurea & interferon
198. While handling a febrile neutropenic patient all are

Hematolgy
Ref Harrison's 18th ed. ch. 109 , Harrison 19 th p 694
essential except : (AI 1998 )
205. Which of the following is associated with Bence jones
a . Repeated hand washing of hospital person
myeloma? ( NBE Pattern 2014-15)
b. White cell infusion
a. y chain disease b. a chain disease
c. Prophylactic antibiotic d. mu chain disease
c. A chain disease
d . Colony stimulating factor for macrophages
Ref: Harrison's 18th ed. p. 940 ch. 11 , Harrison 19 th p 721
Ref Harrison 18th/ p 476 Read text below,
206. Immunoglobulin not affected in Multiple myeloma:
Leukemia and Lymphoma a. IgG ( NBE Pattern 2014-15)
b. IgA
199. Most common lymph nodes involved in hodgkin's c. IgM
lymphomas is: ( DNB 2013) d. IgD
a. Inguinal Ref Harrison’ s 18th ed. ch. Ill , Harrison 19 th p 722
b. Cervical 207 . Radiation exposure does not cause:
c. Axillary a. ALL ( NBE Pattern 2014-15)
d. Subclavian b. AML
Ref Harrison’s 18/ e p 934, Harrison 19 th p 697 / 1189 c. CML
d . CLL
200. Which of the following is the least common feature of Ref Harrison’ s 18th ed. ch. 109, Harrison 19 th p 701
Multiple Myeloma? ( NBE Pattern 2014-15)
a . Bone pain 208. Bence Jones proteinuria is best detected by?
b. Normocytic Normochromic Anemia a. Dipstick method ( NBE Pattern 2014-15)
b. Sulfosalicylic acid
c. Susceptibility to bacterial Infection
c. Heat test
d . Hyperviscosity syndrome
d. Electrophoresis
Ref Harrison’s 18th ed. p 938, 939, Harrison 19 th p 721
.
Ref: Harrison’s 18th ed. ch I l l , Harrison 19 th p 721
201. Vitamin Bi 2 level in chronic myeloid leukemia is:
209. Arsenic is used in treatment of: ( NBE Pattern 2014 -15)
a . Elevated ( NBE Pattern 2014-15)
a. Acute Promyelocytic leukemia
b. Decreased (slightly)
b. A.L.L
c. Normal c. CML
d. Markedly decreased d. Transient myeloproliferative disorder
Ref Harrison' s 18th ed. ch. 109 , Harrison 19 th p 687 Ref:Harrison's 18th ed. p. 912 ch. 109 , Harrison 19th p 686

Ans. 195.
199. b. Cervical
-
b. Severe iron defic.. 196.
200.
c. Decreased LDH
.
d . Hyperviscosity..
197.
201.
a. PNH
a. Elevated
198. b. White cell infusion
202. c. Tubular proteinuria
220 203. d . Lymphocyte pre.. 204. a. Allogeneic bone 205. c. A chain disease 206. c. IgM
207. d. CLL 208. d . Electrophoresis 209. a. Acute Promye
Hematology

210. Which of the following statements on lymphoma is 217. Which of the following is the most common organ of
not True? ( NBE Pattern 2014 -15) origin causing cancer related death in female < 20
a. A single classification system of Hodgkin's disease is years? ( NBE Pattern 2014-15)

I
almost universally accepted a. Breast
b. HD tends to remain localized to a single group of b. Cervix
lymph nodes and spreads by contiguity c. Bone marrow
Ref table 81.2, Harrison's 18 ' ed.,
c. Several types of Non - Hodgkin's lymphoma may have
a leukemic phase
d. Lung

d. In general follicular NHL has worse prognosis 218. True about polycythemia rubra vera is ail except:
a. Bleeding ( NBE Pattern 2014-15)
compared to diffuse NHL
b. Thrombosis
Ref: Harrison’s 18th ed. p. 926 - 929 , Harrison 19 th p 701
211. in Ritcher’ s transformation CLL transforms into:
a . Large B cell Lymphoma
b. Anaplastic carcinoma
( NBE Pattern 2014 -15)
c.
d.
led ESR
Increased erythropoietin
Ref Harison 18** ed. p. 898- 899, Harrison 19 th p 678
I
c. Burkitt lymphoma 219. Most common cause of death in multiple myeloma is: $
a. Infection ( NBE Pattern 2014 -15] j]
d. Lymphoproliferative lymphoma
b. Bleeding
Ref: Harrison 19 th p 703
c. CHF
212. Reedsternberg cells are: ( NBE Pattern 2014 -15) d. Kidney failure
a. B cells b . T cells Ref: Wintrobe Hematology 11th ed. p. 5185, Harrison 19 th p 721

Hematolgy
c. Natural killer cells d. All of above .
Ref Harrison's 18‘h ed. ch. 110 , Harrison 19 th p 709f 220. Maximum ESR is seen in: ( NBE Pattern 2014 -15)
a . CHF
213. Most common type of acute myeloid leukemia: b. Polycythemia vera
a. M 2 ( NBE Pattern 2014-15) c. Multiple myeloma
b. M 3 d. Sickle cell anemia Ref: Harrison 19 th p 721
c. M 4
d. M 5 221. Kappa light chains in urine are seen in:
Ref Harrison's 18lh ed. ch. 109 , Harrison 19th p 678 a. Mu chain disease ( NBE Pattern 2014 -15)
b. Seligman disease
214. Which one of the following laboratory tests differenti ¬ c. Franklin disease
ates leukamoid reaction from chronic myeloid leuke ¬ d. Waldenstrom macroglobulinemia
mia? ( NBE Pattern 2014-15)
Ref Hematology in clinical practice. 4th ed. Harrison's 18th ed.
a . LAP (leukocyte alkaline phosphatase]
b. LCA (leukocyte common antigen) 222. A 2 -year old child comes with ear discharge, sebor ¬
c. MPO (myelo-peroxidase) rheic dermatitis, polyuria and hepatos-plenomegaly.
d. TRAP (tartrate resistant alkaline phosphatase) Which of the following is the most likely diagnosis:
Ref Harrison's 18 ed. p. 477
“ ( NBE Pattern 2014 -15)
a. Leukemia
215. DIC is seen in the following type of AML:
a. M 3 ( NBE Pattern 2014 -15) b. Lymphoma
b. M 4 c. Langerhan's cell histiocytosis
c. M 2 d. Germ cell tumor
d. M 5 Ref O.P. Ghai 7th ed. p. 595, Harrison 19 th p 135 e - 9 / 1713
.
Ref Harrison's 18th ed. ch 109; Harrison 19th p 678 223 . Gum hypertrophy is seen in which type of AML:
216 . All of the following are true about multiple myeloma a. Myelogenous leukaemia ( NBE Pattern 2014 - 15)
except: ( NBE Pattern 2014 -15) b. Myelomonocytic leukaemia
a. Lytic bone lesions b. Back pain c. Megakaryocytic leukaemia
c. Polycythemia d . Viscosity of blood d. Erthroleukemia
Ref Harrison's 18°' ed. ch. Ill , Harrison 19 th p 721 Ref: Harrison’s 18th ed. ch. 109, Harrison 19 th p 682

Ans. 210. d . In general fo... 211. a . Large B cell... 212. a. B cells 213. a. M 2
214. a . LAP ( leukocyte. 215. a. M 3 216. c. Polycythemia 217. c. Bone marrow
218. d . Increased eryt.. 219. a . Infection 220. c. Multiple myeloma 221. a . Mu chain disease
222. c. Langerhan’s ce.. 223. b. Myelomonocytic leukaemia
I Marwah's Internal Medicine MCQs ( Based on Harrison's 19 th
)

.
I 224. Pawn ball megakaryocytes are characteristic of:
a .Myelodysplastic syndrome (NBE Pattern 2014-15)
b. idiopathic thrombocytopenic purpura
c. Thrombotic thrombocytopenic purpura
231 Classical 'Rain drop' lesions seen in:
a .
Burkin's lymphoma
b. Hodgkin's lymphoma
c .
Multiple myeloma
(NBE Pattern 2014-15)

3 d. Chloramphenicol toxicity d. Haemophilia Ref: Harrison 19th p 721


Ref: Hematology in clinical practice. 4°' ed. Harrison 19th p 669 .
232 Cis- Retinoic acid syndrome is used for?
225. 80 year old, asymptomatic man present with a Total
a .
JMML (NBE Pattern 2014-15)
b. M3 AML
leucocyte count of 1lakh, with 80% lymphocytes and
c. M4 AML
20% PMCs. What is the most probable diagnosis? 495.5 Nelson's 18th
ed
a- HIV
d. CML Ref :
(NBE Pattern 2014-15)
b. CML .
233 'ABVD' regimen is used in: (NBEPattern 2014-15)
c. CLL a .
Chronic lymphocytic leukemia
d. TB b. Acute lymphoblastic leukemia
Ref: Harrison's 18* ed. ch. 109, Harrison 19th p 703 c .
N.H.L
d. Hodgkin's disease
226. All of the following are WHO classified Multiplicities ' 18th
ed. ch. 110, Harrison 19th p 697
Ref : Harrison s
Syndromes except: (NBE Pattern 2014-15)
a. CML 234. Polycythaemia vera is associated with all except:
a. Refractory anemia with excess blasts a .
Increased red cell mass (NBE Pattern 2014-15)
c. Refractory anemia with ringed sideroblasts b. Leukocytosis

1
— X
d. Refractory anemia
Ref : Harrison' s 18 th ed. ch. 107, Harrison 19th p 693
c. Splenomegaly
d. Decreased platelet count
Ref: Harrison's 18 th
ed. ch. 108, Harrison 19th p 674
2 .
221 Most common symptom of multiple myeloma:
a. Bone pain (NBE Pattern 2014-15) 235. Erythropoietin is increased in all of the following

s4> b. Anemia
c. Hypercalcemia
conditions except:
a. Hepatocellular carcinoma
b. Renal cell carcinoma
(NBE Pattern 2014-15)

d. Bleeding
X Wintrobe's hematology 11 th
ed. p. 5180, Harrison 19th p 721
c .
Cerebellar Hemangioblastoma
Ref: d. Pancreatic carcinoma
228. Which of the following infection has highest chances Ref: Harrison' s 18 th ed. . 899
p
of transmission by blood transfusion? 236. Commonest site of lytic lesion in multiple myeloma is:
a. HIV (NBE Pattern 2014-15)
b. HBV
a .
Vertebral column (NBE Pattern 2014-15)
b. Femur
c. HTLV-1 c. Clavicle
d. HCV d. Pelvis
Ref: Harrison 19th p 138e-3 Ref: Harrison’s 18th
ed. p. 937- 942 Harrison 19th p 721
229. Treatment of choice of CNS leukemia is: .
237 Not seen in Polycythemia Vera: (NBE Pattern 2014-15)
a. Intra-thecal methotrexate (NBE Pattern 2014-15) a .
Budd chiari syndrome
b. Vincristine and predinisolone b. Hypertension
c .
Intrathecal vincristine
th ed. p. 590
c .
Erythromelalgia
d. Prednisolone Ref: Ghai 7 d. Infections

I 230. According to FAB classification, promyelocytic blood


picture belongs to which type of AML?
a. MO (NBE Pattern 2014-15)
Ref: Harrison's 18th
ed. ch. 108, Harrison 19 th p 672
238. DIC is most likely characterized by: (NBEPattern2014-15)
a. Significant numbers of schistocytes

I b. Ml
c. M 2
d. M3
Ref : Harrison's 18th ed. ch. 109, Harrison 19th p 683
b. A brisk reticulocytosis
c

Ref:
.
Decreased coagulation factor levels
d. Significant thrombocytopenia
Harrison' s 18 th
ed. p. 969 and 979, Harrison 19th p 736

I Ans. 224. a. Myelodysplastic..


228. b. HBV
225. c. CLL
229. a. Intra-thecal methotre..
226.
230.
a. CML
d . M3
227. b. Anemia
231. c. Multiple myeloma
232. a . JMML 233. d. Hodgkin’s disease 234. d. Decreased platelet... 235. d. Pancreatic carcinoma
222 236. a. Vertebral column 237. d. Infections 238. c. Decreased coagulation factor levels
Hematology

239. All of the following are features of juvenile CML 246. All of the following are the causes of relative polycy¬
except: ( NBE Pattern 2014-15) themia except: ( A12005)
a. Thrombocytopenia a. Dehydration.
b. Dengue haemorrhagic fever.

1
b. Fetal Hb is increased
c. Philadelphia chromosome is positive c. Gaisbock syndrome.
d. Lymphadenopathy Ref: Harrison 19th p 687 d . High altitude. Ref: Harrison 18th/ p 456 , 898,

240. A peripheral smear with increased neutrophils, baso¬ 247. A 59-year -old male came with Hb 18.0 gm / dl on three
phils, eosinophils, and platelets is highly suggestive occasions. The resident doctor wants to exclude Poly¬
of: ( NBE Pattern 2014 -15) cythemia Vera. Which of the following is the most rel ¬
evant investigation: ( AIIMS May 04 )
a. Actute myeloid leukemia
b. Acute lymphoblastic leukemia
a. Hematocrit
b. Total leukocyte count
c. Chronic myelogenous leukemia
c. Red cell mass
d . Myelodysplastic syndrome
d. Reticulocyte count Ref: Harrison 18th/ p 899;
Ref: Harrison' s 18th ed. p. 914 - 915, Harrison 19 th p 687
248. Which of the following is not commonly seen in j
241. Essential WHO criteria for polycythemia vera:
( NBE Pattern 2014 -15)
Polycythemia Vera? ( A12002 ) |
a. Thrombosis
a. Tyrosine kinase JAK 2 mutation b. Hyperuricemia
b. Low levels of erythropoietin levels c. Prone for acute leukemia

Hematolgy
c. Thrombocytosis d. Spontaneous severe infection
d . Increased MCV Ref: Harrison 19 th p 672 Ref: Harrison 18th/ p 898, 899 ,
242 . In lymphoplasmacytoid lymphoma which of the fol ¬ 249. In Polycythemia vera, all the following are seen except:
lowing monoclonal immunoglobulin is seen: a. Thrombocytopenia ( AIIMS May 01 )
a. IgA ( NBE Pattern 2014 -15) b. Increased GI bleed
b. IgD c. Thrombosis
c. IgG d. Transient visual loss
d . IgM Ref: Harrison 18th/ p 899, 900 ,
..
Ref: Harrison's 18th ed p 942 ch. Ill , Harrison 19th p 706
250. Which is not seen in polycythemia vera:(AllMS Feb 97)
243 . Cold haemagglutinin is associated with - ( AI 97 ) a. Increased Vit BI 2 binding capacity
a. Anti IgM b. Increase erythropoietin level
b. Anti IgG c. Increase RBC count
c. Anti IgA d. Ocular congestion
d. Donath landsteiner antibody Ref: Harrison 18th/ p 899 ,
Ref Harrison 18th/ p 882 , 251. A pt. being investigated for anemia has a dry marrow
244. Autoimmune hemolytic anemia is seen in tap; peripheral smear reveals tear drop cells; likely
a. Sickle cell anemia ( AIIMS May 94 ) diagnosis is:
b. Chronic lymphocytic leukemia (CLL) a . Leukemia ( AI 2001 )
c. Acute myelocytic leukemia [AML) b. Lymphoma
d. Multiple myeloma c. Myelofibrosis
d. Polycythemia rubra vera
Ref Harrison 18th/ p 882;
Ref: Harrison 18th/ p 901 ,
245. Coomb's + ve Hemolytic Anaemia is seen in except :
a. Alcoholic cirrhosis ( A12000 ) 252 . Most common type of Hodgkin's lymphoma is:
b. Chronic active hepatitis
a. Nodular sclerosis ( AI 1996 )
b. Lymphocyte predominance
c. Primary biliary cirrhosis
c. Mixed cellularity
d. Primary sclerosing cholangitis
d. Lymphocyte depletion
Ref Harrison 18th edn. ch. 106 , Ref: Harrison 18th/ p 934 ,

Ans. 239. c. Philadelphia.. 240. c. Chronic myelogenous... 241. a. Tyrosine kinase JA... 242. d. IgM
243. a. Anti IgM 244. b. Chronic lymphocytic.. 245. a. Alcoholic cirrhosis 246. d . High altitude.
247. c. Red cell mass 248. d. Spontaneous severe... 249. a. Thrombocytopenia 250. b. Increase erythropoietin ...
251. c. Myelofibrosis 252. a. Nodular sclerosis
Marwah's Internal Medicin e MCQs ( Based on Harriso n's 19 th
)

253. Classical markers for Hodgkin's disease is- 261. Which of the following is a good prognostic factor in
a . CD 15 and CD 30 ( AlPGMEE 08) ALL:
a. High WBC count
(PCI 2008)

a
b. CD 15 and CD 22
c. CD 15 and CD 20 b. Male sex
d. CD 20 and CD 30 .
Ref: Robbins 8th edn/ p448, c . Age < 2 years
d. Hyperdiploidy RefWintrobe ’s 11th/2145, 2142,
254. All of the following are good prognostic features of
262. Treatment of choice in intracranial ALL is:
Hodgkin's disease except: (Al 04)
a. Intrathecal methotrexate (PGI June 99)
a. Haemoglobin>10 gm/ dl
b. Vincristine and prednisolone
b. WBC count<15000 / mm3 c. Intrathecal vincristine
c. Absolute lymphocyte count<600/ gl d. Prednisolon
d. Age < 45 yrs .
Ref ' Robbins 8th edn ch. 12, 263. Which of the following is the least common feature of
255. The classification proposed by the International Lym ¬
Multiple Myeloma: (Al 2012)
phoma Study Group for non-Hodgkin’s lymphoma is a. Bone pain
known as: ( A12005 ) b. Normocytic Normochromic Anemia
a. Kiel classification. c. Susceptibility to bacterial infection
b. REAL classification. d. Hyperviscosity syndrome
c. WHO classification. Ref: Harrison 18th/ 938, 939;
d. Rappaport classification. Ref: Robbins 7th/ 668; 264. All of the following are minor criteria for multiple
Myeloma, Except (ARMS Nov 2010)

Hematlogy
256. Burkitt’s Lymphoma is associated with: (AI 2010)
a. Plasmacytosis 20%
a. t (8:14)
b. Multiple lytic lesions
b. t (11:14)
c. Plasmacytoma on tissue biopsy
c. t (15:17) d. Monoclonal Ig spike < 2g/ dl for IgA and < 3.5 for IgG
.
d t (14:18) Ref: Harrison 18th / p 931, 923, Ref: William's Hematology 7th/ 1507,
257. 'Which of the following is NOT used in treatment of 265. Commonest site of lytic lesion, in multiple myeloma is
hairy cell leukemia: (AIIMS Feb 97) a. Vertebral column (A11MS Dec 97)
a. Steroid b. Femur
b. Pentostatin c. Clavicle
c. Splenectomy d. Pelvis Ref: Robbins 8th edn/ p 455,
d. Alpha -interferon RefHarrison 18th / p 931, 932, 266. Raised serum alkaline phosphatase is seen in all,
except (ARMS Dec 94)
258. Chronic Lymphocytic Leukemia (CLL) is associated a. Paget's disease
with: (A12011) b. Multiple myeloma
a. Individuals > 50 years of age c. Osteomalacia
b. Mature small lymphocytes in peripheral smear d. Hyperthyroidism Ref: Harrison 18th/ p 940, 2529;
c . Hepatosplenomegaly and lymphadenopathy
267. An 80 year old asymptomatic woman was detected to
d. All of the above have a monoclonal spike on serum electrophoresis
Ref: Harrison 18th / 926; 2007 / 138; (IgG levels 1.5 g/ dl). Bone marrow revealed plasma
cells of 8%. The most likely diagnosis is: (Al 2004)
259. All of the following are features of juvenile CML except: a. Multiple myeloma
a. Thrombocytopenia (Al 94)
b. Indolent myeloma
b. Fetal Hb is increased c . Monoclonal gammopathy of unknown significance
c. Philadelphia chromosome is positive d. Waldenstorm's macroglobulinemia
d. Lymphadenopathy Ref: Ghai 5th / 465, Ref: Harrison 18th/ p 940;
260. Marker specific for myeloid lineage- (ARMS Nov 95) 268. Franklin's disease is associated with: (ARMS May 95)
a. CD33 a .
Gamma heavy chain disease
b. CD14

I
b. Multiple myeloma
c . CD13 c. Alpha heavy chain disease
.
d MPO Ref: Ch 109, Harrison '
s 18th edn, d. Waldenstorm’s macroglobulinemia
Ref: Harrison 18th / p 943„
Ans. 253. a . CD 15 and CD 30 254. c . Absolute lympho... 255. b. REAL classification. 256. a . t ( 8:14)
257. a . Steroid 258. d. All of the above 259. c. Philadelphia chrom... 260 . d. MPO
261. d. Hyperdiploidy 262 . a . Intrathecal methotrexate 263 . d. Hyperviscosity .. . 264. c. Plasmacytoma on tissue..
224 265. a . Vertebral column 266. b. Multiple myeloma 267. c. Monoclonal gam... 268. a . Gamma heavy chain di...
Hematology

269. Idiopa thic throm bocyto penic purpura is associated 276. Example for autosomal recessive disorder A/E:
with all of the following Except: a. Hereditary Spherocytosis ( Manipal )
a. Splenomegaly ( NEET/ DNB Pattern Question ) b. Beta Thalassemia
b. Mucosal bleeding c. Albinism
c. Thrombocytopenia
d . Increased megakaryocytes
Ref: Robbins 7th / 651,652; Ghai 6 th/ 324 ,
270. All of the following statements about Thrombotic
thrombocytopenic purpura (TTP are true, Except :

a.
( NEET/ DNB Pattern Question )
Microangiopathic Hemolytic Anemia
d. Wilson's deficiency
Ref: Kumar Sr Clark pg 177 Harrison 19 th p 436 t
277 . Oral anticoagulant dosage is monitored by:
a. Prothrombin time
b. Bleeding time
c. Clotting time
d. Clot retraction time
( Manipal ) l
b. Thrombocytopenia Ref: KD Tripathi pg 566 Harrison 19 th p 402 f
c. Normal complement levels
278. The Phidelphia chromosome most commonly associ ¬
d. Grossly abnormal coagulation tests
ated with : ( Manipal )
Ref: Harrison 18th/ p 969 , a. ALL
271. False statement regarding DIC is: (Al 2001 ) b. CML
a. Thrombocytopenia c. AML
b. Decreased fibrinogen d. All the above Ref : Kumar and Clark pg 490
c. Decreased PTT

Hematolgy
279. Autoimmune haemolytic anemia is a feature of:
d. Increased PT Ref Harrison 18th/ p 979 , a. ALL ( Manipal )
272. The most sensitive test for DIC is: (AI 2001 ) b. CML
a. Serum fibrinogen levels c. Burkett 's Lymphoma
b. Serum levels of fibrin degradation products ( FDP) d. Lymphoma
c. Prolonged PT and PTT Ref George Mathews pg 22 Harrison 19 th p 1809
d . Thrombocytopenia
280. Dohle bodies are inclusions bodies seen in: ( NRI )
Ref Harrisons 18th edn. ch. 116 ,
a. Monocytes ( Manipal )
273. Causes of Deep venous thrombosis include all of the b. Eosinophils
following, Except. (AI 2009 ) c. Basophiles
a. Diabetes Mellitus d. Neutrophils
b. Oral contraceptives
281. Cold antibodies of IgG type is seen in: ( NRI ) ( Manipal )
c. Paroxysmal Nocturnal Hemoglobinuria ( PNH )
d. Prolonged surgery a. Syphilis
b. Donath- Landsteiner antibody
Ref Harrison 18th/ p 883, 987, c. Lymphoma
274. Most common cause of Budd Chiari syndrome is: d. Infectious mononucleosis
a. Idiopathic ( AIIMS May 94 ) Ref: Harrison 19th p 138e -2 / 659
b. Valves in hepatic veins
c. Hepatocellular carcinoma 282. Abelson murine leukaemia virus implicated in:
d . Thrombosis of hepatic veins a. AML ( Manipal )
b. CML
Ref Harrisons 17th/ 1950; Harrisons 18th edn. ch. 308 ,
c. ALL
Miscellaneous d. CLL
Ref: CMDT 2001 pg 528 Harrison 19 th p 693
27 S. Chediak Higashi syndrome is: ( Manipal ) 283 . Aplastic crisis in sickle cell anemia is due to
a. Autosomal recessive infection. ( Manipal )
b. Large granular lymphocytes a . Parvovirus
c. Impair NK cell function b. Adenovirus
d. All the above c. Influenza virus
Ref: Kumar & Clark pg 209 Harrison 19th p 419 d. Papilloma virus
Ref Harrison 15 th pg 1117 Harrison 19th p 634
s. 269. a. Splenomegaly
" 273. a . Diabetes Mellits
270. .
d. Grossly abnormal .. 271. c. Decreased PTT 272. b. Serum levels of ...
274. d . Thrombosis of hepatic 275. d . All the above 276. a. Hereditary Spherocytosis
277. a. Prothrombin time 278. d. All the above
281. 279. d . Lymphoma 280. d . Neutrophils
-
b. Donath Landst... 282. .
b CML 283. a . Parvovirus 225
p

I Marwah's Internal Medicine MCQs

284. Von Willebrand's disease is


( Based on Harrison

type of
s 19
,
'h
)

291. Erythropoietin is increased in all except:

I inheritance
a.
b.
.
Autosomal dominant
Autosomal recess
(Manipal ) a.
b.
c.
CML
MDS
Essential thrombocytosis
(Manipal )

B
c. X-linked recessive d. Polycythemia vera
d. X-linked dominant Ref: Harrison' s 18 th pg 898 Harrison 19 th p 672

th
Ref: Kumar and Clark pg 177 Harrison 19 p 731
(Manipal)
292. Splenomegaly is a feature of all except:
285. About 'Acute intermittent porphyria' which of the a. Aplastic anemia
following statements is false: (Manipal ) b. CML
a. Dark coloured urine c. Thalessemia
b. Skin rashes d. Hypersplenism
c. Neurological manifestation ' 18 th pg 467 Harrison 19th p 663

d. Abdominal pain
Ref: Harrison s

th 293. Pancytopenia is seen in all except: (Manipal)


Ref: Kumar and Clark pg 1120 Harrison 19 p 2526
a .
Wiskott-Aldrich syndrome
286. Which is a false statement about purpura? (Manipal ) b. Fanconi's anaemia
a. Palpable c .
Scimartzmann Diamond syndrome
b. More than 3mm d. Kostmann's syndrome
c. Accumulation of RBC in dermis th
p 662, 663t
Ref: Harrison s 18 th pg
' 889 Harrison 19
d. Blanch on pressure 1
Ref: Harrison ' th
s 15 pg 95 Harrison 19th
p 780t 294. Most common Ig, defect seen in multiple myeloma:
a. Ig (Manipal)
j 287. Splenomegaly is not found in: (Manipal )
o b. IgM

4
c
1 a. Hereditary spherocytosis
b. Thalassemia
c.
c. IgA
d. IgE
£
0)
d .
Tropical splenomegaly
Late stage of sickle cell disease Ref : Harrison' s 18 th Harrison 19th p 712

Mathews pg 19 Harrison 19th


p 2242 295. Priapism is seen in all except: (Manipal)
X Ref: George
a. Sickle cell anaemia
.
288 Drugs used in CML are A / E: (Manipal )
b. Megaloblastic anaemia
a. Busulphan
c. Scorpion sting
b. Alfa -interferon
d. Spinal injury
c. Melphalan
d. Hydroxyurea Ref: Harrison' s 18th pg 375, 855 Harrison 19th
p 325, 634
th
Ref: Harrison 15th pp 713 , 714 Harrison 19 p 687
296. Most common type of porphyria is:( NBE Pattern 2014)
289. Which is the most common cytogenetic abnormality in a. Porphyria Cutanea Tarda
adult Myelodysplastic syndrome (MDS) ? (Manipal ) b. Acuta Intermittent Porphyria
a. Trisomy 8
c. Erythopoietic porphyria
b 20q . d. Hereditary Coproporphyia
c. 5q
d. Monosomy 7 Ref: Harrison 19th
p 415, 669 Ref: Harrison 19th p 367e-3
. .
I 290 With regard to sickle cell disease: (Manipal) 291 Which of the following haematological condition pre ¬

a. 25% of the black population have sickle cell disease dispose to renal papillary necrosis?
b. The most severe from of sickle cell retinopathy is a. Hereditary Spherocytosis (NBE Pattern 2014)
associated with SS disease b. Acute Myeloid Leukemia

I c. Optic disc is the first site of neovascularzation in


patient with severe retinal ischemia
d. None of the above
Ref: Harrison 19th
p 634
c. Sickle cell anaemia
d. Hemophilia
Ref: Harrison 19th p 634

I Ans. 284. a . Autosomal dom...


288 . c. Melphalan
285. b. Skin rashes 286. d. Blanch on pressure 1 287. d. Late stage of sickle cell .
290. d. None of the above 291. d. Polycythemia vera
289. a. Trisomy 8
292. a. Aplastic anemia 293. a. Wiskott-Aldrich syn... 294. a. Ig 295. b. Megaloblastic anaemia
22 (y
296. a. Porphyria.... 297. c. Sickle cell anaemia
Hematology

298 Regarding maturation of T and B cells which is true: 305. Increased ESR is seen in all, except:
(JIPMER 2014 ) a. Rheumatic fever (NBE Pattern 2014-15)
a. Co expressed CD4 and CD8 T Cells are eliminated b. Multiple myeloma
through negative selection c. Sickle cell anaemia
b. Both B and T cells operate through negative selection d . Tuberculosis
c. Paracortical part has abundant B and T cells " . .
Ref Harrison's 18 ' ed ch 104, Harrison 19th p 635
d. Thymic T Cells with self reacting antigens are
.
306 False statement regarding DIC is:
clonally deleted
a. Thrombocytopenia (NBE Pattern 2014-15)
.
299 Plasmodium falciparum infect which RBC: b. Decreased fibrinogen
a. Mature (NEET Pattern Question] c .
Decreased aPTT
b. Immature d. Increased PT
c .
Nucleated Ref Harrison' s 18 ed. p - 979, Harrison 19th p 736
"
d. All
.
307 Reticulocytosis is seen in all except:
Ref: Harrison's 18/ e pi 689, Harrison 19th p 14Se-2t a .P.N.H (NBE Pattern 2014-15)
.
300 Difference between cardiogenic and hypovolemic b. Hemolysis
shock is: (NEETPattern Question) c. Nutritional anemia
a. Reduced cardiac output d. Dyserythropoietic syndrome
b. Reduced systemic vascular resistance Ref Harrison’s 18 ' / p. 449, 450, 452
"
c. Rales
.
308 Fetal hemoglobin achieves adult values by

Hematolgy
d. Tachycardia
(NBE Pattern 2014-15)
Ref: Harrison's 18/ e p2233, Harrison 19th p 1504, 1662 a. 6 months of age
.
301 Most common heavy chain disease is: b .12 months of age
a. Franklin disease (NBE Pattern 2014-15) c .24 months of age
b. Seligmann disease d. 36 months of age
c .
Mu heavy chain disease Ref Wintrobe Hematology 11 th ed. p. 442
d. Waldenstrom cryoglobulinemia
-
309. Anti coagulant of choice for heparin induced throm ¬

.
Ref Harrison’s 18th ed ch. Ill , Harrison 19th p 718 bocytopenia is: (NBE Pattern 2014-15)
.
302 Recent oral direct thrombin inhibitor which can be a. Lepirudin b. Aprotinin
used for prevention of stroke \s:(NBE Pattern 2014-15) .
c Abciximab d. Plasminogen
a. Dabigatrin b. Ximelagatron Ref: Harrison's 18 ed. table 118-4, Harrison 19th p 749
"
c. Lepirudin d. Saxagliptin 310. Disseminated intravascular coagulation can occur in
Ref CMDT 2012/ 537 all of the following except: (NBE Pattern 2014-15)
.
303 Incorrect about osmotic fragility test is: a. Snake bite b. Placenta praevia
( NBE Pattern 2014-15) c . Falciparum malaria d. Haemophilia
a. Increased in hereditary spherocytosis ,
Ref Harrison's 18 h ed. ch. 116, Harrison 19th p 736
b. Decreased in thalassemia
c. Hemolysis on exposure to hypertonic saline
.
311 All the following cause prolongation of the activated
partial thromboplastin time that does not correct
enviornment
with a 1:1mixture with pooled plasma except:
d . Osmotic fragility decreased in iron deficiency a. Lupus anticoagulant (NBE Pattern 2014-15 )
Ref .
Hematology in clinical practice 4* ed ch. 11; Harrison's 184 b. Factor VIII inhibitor
.
ed. ch 106, Harrison 19th p 651 c. Heparin
304. All of the following conditions may be associated with d. Factor VII deficiency
a thymoma, except: ( NBE Pattern 2014-15) Ref Harrison's lm ed. ch. 116, 118
a. Erythrocytosis
b. Hypogammaglobulinemia
.
312 Which of the following drugs does not act by blocking
c. Gp Ilb/IIIa receptors? (NBE Pattern 2014-15)
Myasthenia gravis
d. Pure red blood cell aplasia
a. Abciximab b Eptifibatide .
Ref Harrison's 18th ed. ch. 20, Harrison 19th p 668
c .Tirofiban d. Clopidogrel
Ref KDT 6" 610 , Harrison 19th p 747
Ans . 298. d. Thymic T...
302. a . Dabigatrin
306. c . Decreased aPTT
310. d. Haemophilia
299.
303.
307 .
d. All
c. Hemolysis on exposure
c. Nutritional anemia
300. c. Rales
304. a. Erythrocytosis
308. a. 6 months of age
301. b. Seligmann disease
305. c. Sickle cell anaemia
309. a. Lepirudin
I
311. d. Factor VII deficiency 312. d. Clopidogrel 227
i

th
Marwah 's Internal Medicine MCQs ( Based on Harrison’s 19 )

c. Bleeding episodes in patients with Las may be


313 . Thrombotic event is seen in all of following except:
a . PNH ( NBE Pattern 2014 15] - d.
severe and life threatening
Female patients may experience recurrent midtri ¬
b. DIC
c. ITP mester abortions
d. Heparin induced thrombocytopenia Ref: Harrison's 18th ed. ch. 118, Harrison 19 th p 740
Ref: Harrison's 18 ed . ch. 115, Harrison 19th p 728
“ 320. The presence of small sized platelets on the peripheral
314. Purpura fulminans is seen in: ( NBE Pattern 2014-15) smear is characteristic of: ( NBE Pattern 2014-15)
a . Protein C deficiency a. Idiopathic thrombocytopenic purpura
b. Protein S deficiency b. Bernard soulier syndrome
c. AT III deficiency c. Disseminated intravascular coagulation
d. Factor 5 leiden mutation d. Wiskott Aldrich syndrome
,
Ref: Harrison’s 18 h ed. ch. 116, Harrison 19 th p 737 Ref: Harrison's 18 ed. ch. 316 , Harrison 19 th p 2110

315. Idiopathic Thrombocytopenic purpura is associated 321. Which of the following drugs is not recommended in
with: septic shock? ( NBE Pattern 2014- 15)
a. Small megakaryoblasts ( NBE Pattern 2014-15 ) a. Normal saline
b. Non palpable purpura b. Activated protein C
c. Massive splenomegaly c. Steroids
d . Evan syndrome d. Rituximab

Hematolgy
Ref : Harrison' s 18th ed. ch. 115, Harrison 19th p 728 Ref: Harrison's 1 Th/ 1700 - 1701 , Harrison 19th p 1750
316. Auto - splenectomy is associated with: 322. Direct Coomb's test detects the presence of antibodies
a. Systemic lupus erythematosus on the surface of erythrocytes by using:
b. Trauma ( NBE Pattern 2014 -15) a . Sensitization of red cells with the antibody globulin
c. Sarcoidosis b. Anti - human globulin antiserum
d . ITP Ref: Harrison 19th p 2134 c. Incomplete antibody ( NBE Pattern 2014-15)
d. Non-agglutinating antibody
317. Which of the following statements about erythropoi ¬
Ref: H 18th ed. ch. 106 , Harrison 19th p 138e - 4, 138e - 4 f
etin is FALSE? ( NBE Pattern 2014-15 )
a . It is used for the treatment of anemia due to chronic 323 . Hemoglobin with zeta 2 and gamma 2 chains are seen
renal failure in which of the following: ( NBE Pattern 2014-15)
b. It results in decrease in reticulocyte count a. Gower 1
c. It decrease the requirement of blood transfusions b. Gower II
d . It can cause hypertension c. Portland
d. Fetal Gb
Ref: Katzung 10th/ 537-538;KDT’s 6 th/ 592
Ref: Harrison's 18th ed. ch. 104, Harrison 19 th p 632
318. True about Rh factor is ( NBE Pattern 2014 -15)
324. Anemia in chronic renal failure (CRF) is due to:
a . There are no natural anti - Rh antibodies in serum
b. Seen only in females
a. Decreased erythropoietin production
b. Iron deficiency ( PGlJuneOl )
c. Approximately 15 % of Indians are Rh positive c. Hypoplastic bone marrow
d. D is the least powerful Rh antigen d. Decreased Vit B12
Ref: Harrison's 18th ed. ch. 113, e.

I
Decreased folate levels Ref Harrison 18th/ p 850 ,
319. All of the following statements regarding the lupus 325. Thiamine deficiency is known to occur in all of the
anticoagulant (LA) are true except: following except: ( AI 2003)

I
( NBE Pattern 2014 -15) a. Food Faddist
a. Las typically prolong the activated partial thrombo¬ b. Homocystinemia
plastin time c. Chronic alcoholic
b. A 1:1 mixing study will not correct in the presence d . Chronic heart failure patient on diuretics
of Las Ref: Harrison 18th/ p 594, 595, 3218

I Ans. 313. c. ITP 314. a. Protein C deficiency 315. b. Non palpable purpura 316. a. Systemic lupus eryth ...
317. b. It results in de... 318. a. There are no natural 319. c. Bleeding episodes.. 320. d. Wiskott Aldrich syndrome
228 321. d . Rituximab 322. b. Anti -human globulin ... 323. c. Portland 324. a. Decreased erythropoietin ...
325. b. Homocystinemia
>
CHM T £ R Fluid & Electrolytes
10
Most Recent Q’s 2014 -15 7. Multi organ failure is defined as failure of minimum
how many organs? ( DNB Pattern 2014 )
a. 2 b. 3
1. Investigation inHypokalemia + Hypomagnesemia + d. 5 Ref Harrison 19 th p
(Jipmer June 2015)
c. 4
Hypertension:
a. Renin aldosterone ratio 8. SIRS criteria includes? ( DNB Pattern 2014 )
b. ACTH stimulation test a. Heart rate less than 30
c. 24 hour Catecholamines b. Increased BP
d. Octreo-scan Ref: 2320t c. Decreased platelets
d. Leukocytosis >12 ,000 / pL Ref: Harrison 19 th p
2. 16 year old boy admitted with snake bite with oligu -
rai, serum creatinine: 4.5 mg/ dl, sodium bicarbonate: Hypokalemia
10 meq / I. ECG given in the image shows:
a. Hyperkalemia ( NBE Pattern 2014 )
9. 1 year old male child is having a Heart Rate 40 / min,
b. Hypokalemia
BP 90 / 60. His serum Potassium = 6.5 what is the next
c. Hypomagnesemia best management? ( NEET Pattern 2015-16 )
d . Hyponatremia Ref: Harrison 19th p 310
a . Ipratropium b. Adrenaline
3. Which among the following is a wrong statement c. Sodium bicarbonate d. Calcium chloride
about Bartter syndrome? ( NBE Pattern 2014 ) Ref Harrison's 18th edn., ch 45, Harrison 19 th p 312
a. Hyperkalemic alkalosis
10 . Hypokalemia causes death due to:
b. Hypercalciuria
a. Respiratory insufficiency ( NBE Pattern 2014-15)
c. Hypokalemia
b. Torsades de pointes
d. Polydypsia Ref Harrison 19 th p 306
c. Systolic arrest
4. Ringer lactate, also known as Hartmann’s solution is d . Diastolic arrest
considered the ideal fluid in management of ADD with Ref Harrison’ s 18th ed. p. 282, ch. 46 , Harrison 19 th p 307
server dehydration, because it can correct metabolic
acidosis. The potassium content in 1 litre of Ringer 11. In hypokalemia ail ECG changes are seen except:
lactate solution is: ( NBE Pattern 2014 ) a. Decreased T wave amplitude ( NBE Pattern 2014-15)
a. 6 mEq b. 8mEq b. Normal ST segment
c. 4mEq d. 28mEq c. Wide qRS complex
d. Prolonged PR interval
Ref: Harrison 19th p 1749
Ref Harrison's 18th ed. ch. 45 , Harrison 19 th p 307
5. Which of the following is not a criterion for diagnos ¬
ing metabolic syndrome? ( NBE Pattern 2014 ) 12. Decreased dietary intake of potassium is incriminat ¬
a. LDL cholesterol greater than 100 mg/ dL ed in leading to all except? ( NBE Pattern 2014 - 15)
b. Fasting serum glucose greater than 110 mg / dL a. Hypertension b. Stroke
c. Systolic BP of 130 mm Hg or greater, or diastolic BP c. CHF d. Diabetes mellitus
of 85 mm Hg or greater Ref Harrison's 18th ed. ch. 45, Harrison 19 th p 304
d . Triglycerides > 150 mg/ dl 13. Most rapid way of lowering potassium is:
Ref: Harrison 19 th p 2454 a. Albuterol ( NBE Pattern 2014-15)
6. Flat T wave and prolonged QT interval on ECG as b. Insulin drip
shown in picture is due to? ( DNB Pattern 2014 ) c. Calcium gluconate
a. Hyperkalemia b. Hypokalemia d. Sodium bicarbonate
c. Hypercalcemia d. Hypocalcaemia Ref Harrison's 18th ed. ch. 45, Harrison 19 th p 312

Ans - 1 . a . Renin aldosterone ... 2. a . Hyperkalemia 3 . a . Hyperkalemic alkalosis 4 . c . 4mEq


5 . a . LDLcholesterol gre . . . 6 . d . Hypocalcaemia 7. a. 2 8 . d . Leukocytosis >12,000/ pL
. d . Calcium chloride 10. a . Respiratory insufficiency ; b .Torsades de pointes 11 . b. Normal ST segment
' * d . Diabetes mellitus 13. b . Insulin drip
Complete Review of Medicine after Edition of Harrison's

14. TTKG in hypokalemia is: ( NBE Pattern 2014 -15) 22. Hyperkalemia is caused due to all of the following ex¬
a. < 3- 4 b. > 6- 7 cept: ( NBE Pattern 2014 -15)
c. > 9-10 d. > 10-15 a. Alkalosis
.
Ref: Harrison's 18th ed ch. 45, Harrison 19th p 311 b. Acute renal failure
15. c. Addisons's disease
I A patient on amphotericin B has weakness and
d. Excess hemolysis
cramps. Serum potassium 2.3 mEq / dl. Calculate the
=
total parenteral potassium supplementation to be Ref Harrison's 18th ed. ch. 47, Harrison 19th p 309-310
given to the patient over the next 24 hours? 23. Hyperkalemia presents with all except?
a -40 mEq ( AIMS Nov. 2012) a. Ascending paralysis ( NBE Pattern 2014 -15)
b. 80 mEq b. Hemodynamic collapse
c. 100 mEq c. Ileus
d. 140 mEq d. Slow idioventricular rhythms
Ref: Harrison' s 18th ed. ch. 45, Harrison 19th p 307 Ref Harrison's 18th ed. ch. 45, Harrison 19th p 310
16. Hypokalemia is seen in all except? 24. Cardiac rhythm seen with hyperkalemia is all except?
a . Barter syndrome ( NBE Pattern 2014 -15) a. Sinus arrest ( NBE Pattern 2014 - 15)
b. Hypokalemic periodic paralysis b. Sinus bradycardia

Ecrolytes
c. 21 hydroxylase deficiency c. Ventricular fibrillation
d. Reninoma QG cell tumour) d . Torsades de pointes
Ref: Harrison’s 17th ed. p. 283, Harrison 19th p 307 Ref Harrison's 18th ed. ch. 45, Harrison 19th p 310
I 17. Hypokalemic, metabolic acidosis can occur with excess 25. Which of the following genetic abnormalities is asso¬
fluid loss from: ( NBE Pattern 2014 -15) ciated with the development of hyperkalemia :
a . Stomach b. Ileum a . lib - hydroxylase deficiency ( NBE Pattern 2014- 15)
c. Colon d. Pancreas b. Liddle's syndrome
Ref: Harrison's 18th ed. ch. 47 , Harrison 19th p 305 .
c Gitelman's syndrome
d . Autosomal dominant polycystic kidney disease
& Hyperkalemia Ref H 18th ed. ch. 284, Harrison 19th Table 63-5, p 309

Fluid 18. All are used for management of hyperkalemia except?


a. Calcium gluconate
b. Insulin plus dextrose
c. Beta 2 antagonist d. Dialysis
.
( AIIMS NOV. 2014)

Ref Harrison 18th edn , ch 45, Harrison 19th p 312


26.
Hyponatremia

Which of the following is not an important cause of


hyponatremia?
a. Gastric fistula
( Manipal )

19. Not recommended in management of Hyperkalemia: b. Excessive vomiting


a. Albuterol ( PGI 2014-15) c. Excessive sweating
b. Loop diuretics d. Prolonged Ryle's tube aspiration
c. Peritoneal dialysis Ref: Harrison pg 2749 Harrison 19th p 300
d . 50% dextrose in water 27. Poorly controlled diabetes with blood sugar of 450
Ref Harrison’s 18th ed. ch. 45, Harrison 19 th p 312 mg% is associated with: ( NBE Pattern 2014 -15)
20. The Maximum concentration of potassium delivered a. Hyponatremia b. Hypernatremia
via central vein is? ( NBE Pattern 2014 - 15) c . Hypokalemia d. Hypomagnesemia
a. 20 mmol / L b. 40 mmol /L Ref Harrison's 17th ed. p. 277, Harrison 19th p 299
c. 60 mmol / L d. lOOmmol / L
28. What is the of correction of sodium deficit?
Ref Harrison' s 17th ed. p. no 283, ch. 46 , Harrison 19th p 307 ( NBE Pattern 2014 - 15)
a . 0.5 mmol / hour
21. Sine wave pattern on ECG is done when serum potas ¬ b. 1 mmol / hour
sium exceeds mEq / dl? ( NBE Pattern 2014 15) - c. 1.5 mmol / hour
a. > 6 mEq / dl b. > 7 mEq / dl d . 2.0 mmol / hour
c. >8 mEq / dl d. >10 mEq > dl
Ref: Harrison's 18th ed . ch. 45, Harrison 19th p 302
Ref : Harrison' s 18th ed. ch. 45, Harrison 19th p 310

Ans . 14. -
a. < 3 4 15. d. 140 mEq 16. c. 21 hydroxylase defic... 17. c. Colon
18. c. Beta 2 antagonist 19. d. 50% dextrose in water 20. c. 60mmol / L 21. c. >8 mEq /dl
22. a. Alkalosis 23. c. Ileus 24. d . Torsades de pointes 25. d. Autosomal domi...
230
26. d . Prolonged Ryle’s... 27. a . Hyponatremia 28. a. 0.5 mmol / hour
Fluid & Electrolytes

common electrolyte abnormality in hospitalised 36 . Which of the following is associated with hyponatre ¬

29 Most (NBEPattern 2014-15)


( NBE Pattern 2014-15) mia and low osmolality:
patient ?
a. Hyperlipedemia b. Hyperproteinemia
a. Hyponatremia d. CHF
c. 1VIG therapy
b. Hypernatremia
c. Hypokalemia Ref: Harrison's 18th ed. ch. 45 , Harrison 19th p 299
d. Hyperkalemia 37 . Pseudohyponatremia is seen in? (NBE Pattern 2014-15)
Harrison’s 18th ed. ch. 45 , Harrison 19th p 298 a. Hyperlipidemia b. SIADH
Ref:
d. CKD
c. CHF
30. Maximum loss of sodium in a child occurs in:
(NBE Pattern 2014-15) Ref: Harrison's 18th ed. ch. 45 , Harrison 19th p 299
a. Gastric juice
b. Ileal fluid Hypernatremia
c. Non cholera Diarrhoea
d. Cholera 38. All are used for management for hypernatremia except?
Ref: OP Ghai 7th ed. p. 48: table 4.1 ( NBE Pattern 2014-15)
a. 5 % dextrose in water
31. Tolvaptan is useful for ? (NBE Pattern 2014-15)
b. N/ 2 in 5% dextrose

Fluid
a. Hyponatremia
b Hypernatremia
c . Nil by mouth
d. Indomethacin
c. Nephrogenic diabetes insipidus
d. Decreased water clearance
Ref: Harrison's 17* ed. p. 280, Harrison 19th p 304
39. Most common cause of hypernatremia?
Ref: Harrison's 18th ed. ch. 45 , Harrison 19th p 302 (NBE Pattern 2014-15) &

Eecrolytes
32. Acute hyponatremia becomes symptomatic at: a. Primary hypodipsia b. Carcinoid syndrome
(NBE Pattern 2014-15) c. Renal losses .
d Sweating
a. < 135 mEq
Ref: Harrison’s 17th ed. p. 279, Harrison 19th p 303
b. <125 mEq
c. < 120 mEq 40. Doughy skin and woody induration of tongue is seen
d. <110 mEq in? (NBE Pattern 2014-15)
a. Hypernatremia b. Hyponatremia
Ref: Harrison’s 18th ed. ch. 45 , Harrison 19th p c. Hypokalemia d. Hyperkalemia
33. Hyponatremia is seen in: (NBE Pattern 2014-15)
a. Hyperthyroidism
Ref: OP ghai 7th ed. p. 52, Harrison 19th p 303
b. Hypothyroidism 41. All are features of hypernatremia except?
c. Diabetes insipidus (NBE Pattern 2014-15)
d. increased insensible losses a. Convulsions
b. Elevated intracranial tension
Ref: Harrison's 18th ed. ch. 45 , Harrison 19th p 300 c. Periodic paralysis
34. Incorrect about cerebral salt wasting syndrome? d. Doughy skin
(NBE Pattern 2014-15)
Ref: Harrison's 18th ed. ch. 45, Harrison 19th p 303
a. Urine sodium> 20mEq/ dl
b. Hyponatremia 42. Hypernatremia causes all EXCEPT:
c. Fludrocortisone is used (NBE Pattern 2014-15)
d. Expansion of plasma volume a .
Seizure b. Thrombosis

Ref: Harrison's 18th ed. ch. 45 , Harrison 19th p 299


c .
Brain hemorrhage d. Cerebral edema

35. A 70 kg adult male presents with serum sodium of 110


Ref: Harrison's 18th ed. ch. 45, Harrison 19th p 303
meq / dl. Calculate correction required in 24 hours? 43. Not a cause of hypernatremia: (NBE Pattern 2014-15)
a. 100 mEq (NBE Pattern 2014-15) a. Primary hypodipsia
b. 200 mEq b. Decreased insensible losses
c. 300 mEq c .Nephrogenic diabetes insipidus
.
d 400 mEq d. Carcinoid syndrome
Ref: Harrison's 18th ed. ch. 45 , Harrison 19th p 302 .
Ref: Harrison's 18th ed. ch 45, Harrison 19th p 303
Ans 29. a. Hyponatremia 30. b. Ileal fluid 31. a. Hyponatremia 32. b. <125 mEq
33. b. Hypothyroidism 34. d. Expansion of pla... 35. d. 400 mEq 36. d. CHF
37 . a. Hyperlipidemia 38. c. Nil by mouth 39. c. Renal losses 40. a. Hypernatremia
41. a . Convulsions 231
42. b. Thrombosis 43. b.Decreased insensible losses
Complete Review of Medicine after Edition of Harrison's

44. Brain tumour causing hypernatremia in children? 51. Incorrect about Hypomagnesemia?
a. Medulloblastoma ( NBE Pattern 2014-15) a. Tetany -
( NBE Pattern 2014 15)
b. Cerebellar astrocytoma b . Causes resistance to PTH
c. Craniophyrangioma c. Arrhythmia

3 45.
d. Brain stem glioma Ref: O.P Ghai 7th ed. p. 52

Hypocalcemia

Incorrect about hypocalcemia : ( NBE Pattern 2014 -15)


a . Shortening of QT interval
b. Seizures
52.
d . Intramuscular magnesium sulphate

Hypermagnesemia
.
Ref: Harrison's 18th ed. ch 352, Harrison 19 th p 2462

All are features of hypermagnesemia except?


a. Hypotension ( NBE Pattern 2014-15)
c. Larygospasm b. Ileus
d. Di George syndrome c. Tetany
Ref : Harrison's 18th ed., Harrison 19 th p 314 - 15 d. Decreased DTR
46. Why is tetany seen with hyperventilation : Ref: Harrison's 17th ed. p. 2373, Harrison 19 th p 2462
a . Metabolic alkalosis ( NBE Pattern 2014-15)
53. All are indicators of hypermagnesemia except?

Electroys
b. Respiratory alkalosis ( NBE Pattern 2014-15)
a. Decreased DTR
c. Metabolic acidosis
b. Decreased BP
d. Respiratory acidosis
c. Heart block
Ref: Harrison's 18th ed. ch. 46 , Harrison 19 th p 323-324 d . 'Main d'accoucheur'
Hypercalcemia Ref: Harrison's 18th ed. Ch. 352 , Harrison 19 th p 2463
54. Not seen in Hypermagnesemia: ( NBE Pattern 2014-15)
47 . Prolonged immobilization leads to? a. Bradycardia b . QT prolongation
a . Hypercalcemia ( NBE Pattern 2014-15) c. Dilated pupils d. T wave inversion
& b. Hypocalcemia
c. Hyperkalemia
Ref Harrison's 18th ed. p. 2373, Harrison 19 th p 2463

Fluid d . Hypokalemia 55. All are useful for management of Hypermagnesemia


except? ( NBE Pattern 2014-15)
Ref : Harrison 19 th p 313
a. Magnesium free enema
48. Not useful for acute Hypercalcemia? b. IVF
a. Normal saline ( NBE Pattern 2014 -15) c. Calcium gluconate
b. Calcitonin d . Amiloride
c. Furosemide Ref: Harrison's 18th ed. p. 2373, Harrison 19 th p 2463
d. Bisphosphonates
Ref William textbook of endocrinology 12th ed./ p. 1272 56. Heart block is seen in: ( NBE Pattern 2014-15)
a . Hypermagensemia b. Hypomagnesemia
Hypomagnesemia c. Hypernatremia d . Hypocalcemia
Ref Harrison's 17th ed. p. 2373, Harrison 19 th p 2463
49. Heart block is seen in: ( NBE Pattern 2014-15)
a. Hypermagnesemia b. Hyperphosphatemia Acid Base Disorders
c. Both d. None
Ref Harrison' s 17th ed. p. 2373, Harrison 19 th p 2461 , 2463 57. Which metabolic derangement is seen in pregnancy?
a. Metabolic acidosis ( NEET Pattern 2015-16 )
50. Not a cause of Hypomagnesemia: ( NBE Pattern 2014-15) b. Metabolic alkalosis
a. Gitelman syndrome b. Re -feeding syndrome c. Respiratory acidosis
c. Hyperaldosteronism d. Rhabdomyolysis d. Respiratory alkalosis
Ref Harrison 19 th Table 423-4, 2462 Ref: Harrison 18th edn., ch 45, Harrison 19 th p 323

Ans. 44. c. Craniophyrangioma 45. a . Shortening of QT interval 46. b. Respiratory alkalosis 47. a. Hypercalcemia
48. c. Furosemide 49. d. None 50. d . Rhabdomyolysis 51 . d . Intramuscular magnesium ..
232 52. c. Tetany 53. d. ‘Main d’ accoucheur’ 54. d . T wave inversion 55. d. Amiloride
56. a. Hypermagnesemia 57. d . Respiratory alkalosis
Fluid & Electrolytes

c. Respiratory alkalosis with compensatory metabolic


58. pH 7.31 pC02 33 mm Hg and HC03 16 mEq / dl:
= = =
a. Metabolic acidosis ( NBE Pattern 2014 -15) acidosis
b. Metabolic alkalosis d . Metabolic acidosis with respiratory acidosis
c. Respiratory acidosis -
Ref: H 17th ed. p. 288-289 , Harrison 19th Table 66 2, p 316
d. Respiratory alkalosis
65. All are useful for treatment of metabolic alkalosis ex¬
Ref: Harrison's 18th ed. ch. 47, Harrison 19th p 317
cept? ( NBE Pattern 2014-15)
59. = = =
pH 7.55, pC02 38, HC03 33, what is the primary
( NBE Pattern 2014-15)
a . Sodium chloride
abnormality:
b. Potassium chloride c. Hydrochloric acid
a. Metabolic acidosis b. Metabolic alkalosis
d . Ammonium chloride
c. Respiratory acidosis d . Respiratory alkalosis
Ref: Harrison's 18th ed. ch. 47 , Harrison 19th p 322
Ref: Harrison's 18th ed. ch. 47 , Harrison 19th p 322
66. Most common acid base disturbance in critically ill
60. Hyperchloremic metabolic acidosis is seen in all ex¬
( NBE Pattern 2014- 15)
cept? ( NBE Pattern 2014 15) - patients?
a. Chronic respiratory alkalosis
a. RTA 1
b. Diarrhea b. Chronic respiratory acidosis

Fluid
c. Uraemia c. Metabolic acidosis
d. Gitelman syndrome d. Metabolic alkalosis
Ref Harrison's 18th ed. ch. 47, Harrison 19th p 321 Ref: Harrison's 17th ed. p. 295, Harrison 19th p 323
61. Best for management of respiratory alkalosis? 67. pH = 7.46 , pC02 = 57 mm Hg and HC03 = 42 mEq:
a. Rebreathing in paper bag ( NBE Pattern 2014-15) ( NBE Pattern 2014-15) &

Electroyes
b. IPPV a. Metabolic alkalosis with compensatory respiratory
c. Normal saline acidosis
d. Acetazolamide
b. Metabolic acidosis with compensatory respiratory
Ref: Harrison's 18th ed. ch. 47 , Harrison 19th p 324 alkalosis
62. A 70 year old man with history of CHF presents with c. Respiratory acidosis with compensatory metabolic
increased shortness of breath and leg swelling. alkalosis
d. Respiratory alkalosis with compensatory metabolic
=
ABG: pH 7.24, pC02 60 mmHg, P02 52, HC03 27 = = acidosis
a . Metabolic acidosis ( NBE Pattern 2014 -15)
b. Metabolic alkalosis Ref: H 17th ed. p. 288-289 , Harrison 19th p 316 , Table 66 -2
c. Respiratory acidosis 68. 29 year old female with history of Sjogren’s syndrome
d. Respiratory alkalosis presents with a 2 day episode of watery diarrhea 2
Ref: Harrison's 18th ed. ch. 47, Harrison 19th p 323 days ago. Physical examination is unremarkable. Be ¬
cause of her history, the physician decides to check
63. A plasma HC03 concentration of 15 mEq / L and a plas ¬
"

her urine electrolytes. Urine chemistry:


ma pC02 of 30 mmHg with a pH of 7.5 represents:
a. Simple metabolic acidosis ( NBE Pattern 2014-15) K = 31, Na 100, Cl 105. Her current diagnosis is ?
= =
b. Compensated metabolic acidosis a. Renal tubular acidosis ( NBE Pattern 2014-15)
c. Simple respiratory alkalosis b. Hypochloremic Metabolic alkalosis
d . Compensated respiratory alkalosis c. Malignant hypertension
Ref: Harrison’s 18th ed. ch. 47, Harrison 19th p 323 d . Respiratory alkalosis
Ref: Harrison' s 18th ed. ch. 47, Harrison 19 th p 323, 320, 64e-6
64. pH = 7.30, p C02 = 38, HC03 = 18-.( NBE Pattern 2014-15) 69.
a. Metabolic acidosis with compensatory respiratory pH = 7.27, HC03 = 10 mEq /dl PC02 = 23 mm Hg:
alkalosis a. Metabolic acidosis ( NBE Pattern 2014-15)
b. Respiratory acidosis with compensatory metabolic b. Metabolic alkalosis
alkalosis c. Respiratory acidosis d . Respiratory alkalosis
Ref: Harrison' s 18th ed. ch. 47, Harrison 19th p 317
ns - 58. a . Metabolic acidosis 59. b. Metabolic alkalosis 60. d. Gitelman syndrome 61 . a . Rebreathing in paper bag
62. c. Respiratory acidosis63. d. Compensated respiratory. 64. d. Metabolic
,
acidosis... 65. d. Ammonium chloride
66. a. Chronic respiratory. 67. a . Metabolic
,
alkalosis with... 68. a. Renal tubular acidosis 69. a . Metabolic acidosis
233

I
Complete Review of Medicine after Edition of Harrison's

70. All are causes of increased anion gap except? 76 . The enteric fluid with an electrolyte ( Na *, K*, Cl ) con ¬
a. Diabetic ketoacidosis ( NBE Pattern 2014-15) tent similar to that of Ringer’s lactate is:
b. Renal tubular acidosis a. Saliva ( NBE Pattern 2014 -15)
c. Diabetic nephropathy b. Contents of small intestine
d . Starvation c. Contents of right colon
d. Pancreatic secretions
VJI Ref: Harrison's 18th ed. ch 47 , Harrison 19 th p 317
Ref Harrrison 18th ed. ch 47 , Harrison 19 th p 305
H ?!- Reduced anion gap is seen in? ( NBE Pattern 2014-15)
a. Nephrotic syndrome Water Balance
b. Fistula
c. Fanconi syndrome 77 . Fever increases water losses by ml / day per
d . Renal tubular acidosis Type 4 degree Celsius: ( NBE Pattern 2014-15 )
Ref: Harrison' s 18th ed. ch 47, Harrison 19 th p 317 a. 100 b. 200
c. 400 d . 800
72 . Increased anion gap is seen in all except? Ref: Harrison's 17th ed. p. 437,
a - Fistula ( NBE Pattern 2014 -15)
Calculate water deficit for 50 kg male with Na = 160
b. Starvation 78.

Elecroytes
c. Ethylene glycol poisoning meq: ( NBE Pattern 2014-15)
d. Phenformin a. 2.9 L hypertonic saline
b. 2.9 L half normal saline
Ref Harrison' s 18th ed. ch. 47 , Harrison 19 th p 317
c. 1.5 L 5% dextrose in water
73. Metabolic acidosis with a normal anion gap is found d. 1.5 L 10% dextrose in water
in a patient with: ( NBE Pattern 2014 -15) Ref: H 18th ed. p. 280 , Harrison 19 th Table 63 -3, p 302
a. Alcohol intoxication
79. Which one of the following is the major determinant
b. Aspirin ingestion of plasma osmolality: ( NBE Pattern 2014-15)
c. Shock a. Serum sodium b. Serum potassium
& d. Small bowel fistula c. Blood glucose d. Blood urea nitrogen
Ref: Harrison's 18th ed. ch 47, Harrison 19 th p 317 Ref: Harrison's 18th ed. ch. 45, Harrison 19th p 294

Fluid 74. A 30 years old HIV positive man is admitted with con ¬
fusion . Physical examination shows a blood pressure
of 140 / 70 with no orthostasis, normal jugular venous
pressure, and no edema. Serum chemistries are no ¬
table for sodium 120 mmol / L , K +
=4.2 mmol / L, bicar¬
80. Correct about osmotic demyelination syndrome?

a.
b.
c.
Characterised by focal seizures
( NBE Pattern 2014-15)
More common with acute hyponatremia

Seen due to slow correction of hyponatremia


bonate = 24 mmol / L, and uric acid 1.5 mg/ dL. The d. Lesions occur in pontine and extrapontine areas
most likely diagnosis is: ( NBE Pattern 2014 -15) Ref Harrison' s 18th ed. ch. 45, Harrison 19 th p 301
a. Hepatic cirrhosis
81. Insensible losses of water per day are:
b. Cerebral toxoplasmosis with SIADH
a. 400 ml per day ( NBE Pattern 2014 -15)
c. Addison's disease
b. 600 ml per day
d. Significant gastrointestinal fluid loss
c. 800 ml per day
Ref : Harrison's 18th ed., Harrison 19 th p 1401 -2
d. 1500 ml per day
75. Which one of the following statements about electro ¬ Ref: Harrison’s 17th ed. p. 437 , Harrison 19 th p 298
lytes in colon is correct: ( NBE Pattern 2014-15)
a. Absorption of Na + in the colon is under hormonal 82. What is the minimum fluid urine output for neutral
(aldosterone) control solute balance? ( NBE Pattern 2014-15)
b. Bile acids enhance absorption of water from the a. 300 ml b. 400 ml
colon c. 500ml d . 750 ml
c. Net absorption of HC 03 occurs in the colon

Ref Harrison' s 17th ed. p. 275
d. Net absorption of K + occurs in the colon
Ref: NMS physiology by john bullock: ch. 44, p. 550

Ans. 70. b. Renal tubular acid...71. a . Nephrotic syndrome 72. a. Fistula 73. d. Small bowel fistula
74. .
b. Cerebral toxoplas... 75. a. Absorption of Na + in .. 76. b. Contents of small int.. 77. b. 200
78. b. 2.9 L half normal... 79. a . Serum sodium 80. d. Lesions occur in po.. 81. c. 800 ml per day
234
82. c. 500 ml
Fluid & Electrolytes

91. Most serious side effect with sodium polystyrene sul ¬


Miscellaneous fonate: ( NBE Pattern 2014 -15)
a . Intestinal perforation
83. Drug of choice for hyperkalemia: ( Manipal ) b. Rebound hyperkalemia
a . 10% KCL / IV c. Cardiac arrest
b. 10% Calcium gluconate / IV d. Respiratory arrest
c. 10% NaHC03 / IV Ref: Harrison's 18th ed. ch. 45, Harrison 19th p 312
d. Ringer lactate
Ref: Kumar & Clark pg 686 Harrison 19th p 64e-3 TTKG > 8 is seen in all except: ( NBE Pattern 2014 -15)
a. Diabetes mellitus
84. IV bicarbonate is useful in: ( Manipal) b. Acute glomerulonephritis
a. Metabolic acidosis c. Adrenocortical insufficiency
b. Respiratory acidosis d. Cushing syndrome
c. Metabolic alkalosis Ref Harrison's 18th ed. ch. 45, Harrison 19th p 311
d. Respiratory alkalosis
Ref: Manipal Manual of Surgery pg 127 93. Jansen disease is? ( NBE Pattern 2014 - 15)
a . Defect of PTH receptor
85. Normal anion gap is a feature of: (Manipal) b. Defect of GH receptor

Fluid
a. Lactic acidosis c. Defect of GHRH receptor
b. Diarrhea d . Defect of ADH receptor
c. Metabolic acidosis
Ref: Harrison's 18th ed. ch. 353, Harrison 19th p 2475
d. Respiratory acidosis Ref: CMDT 2001 pg 887
94. Neonate is posted for Congenital Hypertrophic Py¬
86 . Which is a feature of Hypokalemia is:
a. Vomiting b. Diarrhoea
( Manipal)
-
loric Stenosis, Pre op work up shows serum calcium = &

Electroys
6 mg%. What should be done next? ( NBEPattern 2014-15)
c. Diuretics d . All the above
a. Serum protein b. Oxygen saturation
Ref Harrison 19th p 307 c. USG abdomen d . Serum bilirubin
87 . Not a feature of ‘Heat stroke': ( Manipal) Ref Harrison's 18th ed. ch. 46 , Harrison 19th p 314
a. High temperature b. High pulse
Tetany may be a feature of the following except:
c. Fever d . Sweating
( NBE Pattern 2014-15)
Ref CMDT 2001 pg 1545 Harrison 19th p 124 a . Hyperventilation
88. Which of the following is not an important cause of b . Hypokalaemic alkalosis
hyponatremia? ( Manipal) c. Hypomagnesaemia
a. Gastric fistula d . Hyponatremia
b. Excessive vomiting Ref H 18th ed. ch. 353, Harrison 19th p 314 Table 65-2
c. Excessive sweating
96. Percentage of magnesium absorbed in TAL?
d. Prolonged Ryle's tube aspiration
( NBE Pattern 2014-15)
Ref Harrison 19th p 298, 2280
a. 20% b. 40%
89 . Hypernatremic hypovolemia causes all except: c. 60% d . 80%
a . Cerebral haemorrhage ( Manipal) Ref Harrison's 17th ed. pg. 2373, Harrison 19th p 2461
b. Cerebral thrombosis
c. Pontine myelinolysis 97. Tumor induced osteomalacia is caused due to?
d. Seizures ( NBE Pattern 2014-15)
Ref Harrison's 18th pg 349 Harrison 19th p 303 a - FGF- 23 b. PTH
c. Calcitonin d. All of above
90. 3.1 year old male child is having a Heart Rate 40 / min,
Ref Harrison's 17th ed. p. 2370 , Harrison 19 th p 2459, 2460
BP 90 / 60. His serum Potassium = 6.5 what is the next
best management?
a. Ipratropium
-
( NEET Pattern 2015 16 ) 98. Blast crisis leads to: ( NBE Pattern 2014-15)
b. Adrenaline a. Hyperphosphatemia b. Hypophosphatemia
c. Sodium bicarbonate d. Calcium chloride c. Hyperkalemia d. Hypokalemia
Ref: Harrison's 18th edn., ch 45, Harrison 19th p 312 Ref Harrison' s 18th ed. table 346 -1,

Ans. 83. b . 10% Calcium glue... 84.


87. d . Sweating 88.
91. a . Intestinal perfora.. 92.
95. d. Hyponatremia
a . Metabolic acidosis
d . Prolonged Ryle’s tube...
d . Cushing syndrome
c. 60 %
85. b. Diarrhea
89. b. Cerebral thrombosis
93. a. Defect of PTH receptor
86.
90.
94.
d . All the above
d. Calcium chloride
a. Serum protein 235
1
96. -
97. a. FGF 23 98. b. Hypophosphatemia
I
l Complete Review of Medicine after Edition of Harrison’s

99. Hyperphosphatemia is treated by?


( NBE Pattern 2014-15)
104. Shohl's solution is: ( NBE Pattern 2014-15)

a
a . Sodium citrate b. Potassium binding resin
a. Calcitonin b. Sevelamer c. Lugol iodine d . Radio-iodine
c. Magnesium hydroxide d . Diuretics .
Ref: Harrison's 18th ed ch 47 , Harrison 19 th p 318, 320
Ref: Harrison’s 17th ed. p. 2372, Harrison 19 th p 2461
105. Urinary anion gap is increased in :
100. Hypophosphatemia is seen in: ( NBE Pattern 2014 -15) ( NBE Pattern 2014 -15)
a. Blast crisis a. Diarrhea
b. Tumor lysis syndrome b. Renal tubular acidosis
c. Addisonian crisis c. Ureterosigmoidostomy
d. Myxedema coma d. Water intoxication
Ref Harrison's 18th ed. table 346 -1 , Harrison 19 th p 2462 Ref : Oski essential pediatrics, p. 504, 2nd ed
101. Type B lactic acidosis occurs due to? 106 . High urinary chloride is seen in all except?
( NBE Pattern 2014-15) ( NBE Pattern 2014-15)
a -
CHF b. Cyanide poisoning a. Barter syndrome b. Gitelman syndrome
c. Short gut syndrome d. Diabetes mellitus c. Vomiting d . Thiazide
V) Ref: Harrison' s 18th ed. ch. 47, Harrison 19 th p 318 Ref: Harrison's 18th ed. ch.45, Harrison 19 th p 306 , 307
CD
( NBE Pattern 2014 -15) 107. In metabolic acidosis caused by diabetic ketoacidosis,
& 102 . Fomepizole is used for:
a . Ethylene glycol poisoning which of the following would be greater than normal:
o b. Ethyl alcohol poisoning ( NBE Pattern 2014-15)
U c. Ether poisoning a. Concentration of plasma HC 03

M
U d . Type A Lactic acidosis b. Anion gap
a
mm Ref: Harrison's 18th ed. ch. 47 , Harrison 19 th p 319 c. Arterial pC02
d. All of the above
w 103. All are included in diagnostic criteria for SIADH in Ref: Harrison's 18th ed. ch. 47 , Harrison 19 th p 317
& children except? ( NBE Pattern 2014 -15)
108. Extracellular bicarbonate ions serve as an effective
a. Normal creatinine clearance buffer for all of the following EXCEPT:
b. Urine osmolality > 100 mEq / dl ( NBE Pattern 2014-15)
c. Serum osmolality < 280 mosm a . Sulphuric acid b. Phosphoric acid
E d . Urine sodium < 25 mEq / dl c. Lactic acid d. Carbonic acid
Ref: Nelson 18th ed. table 52.3, Harrison 19th p 301 Ref: Guyton, 9th ed. P. 387-389

ADDITIONAL UPDATES

SBP < lOOmmHG w th Shock

Low Output Cardiogenic Shock SBP 70-100 No Shock Dobutamine

SBP > 100 Nitroglycerin

I Ans. 99. b. Sevelamer 100. a. Blast crisis 101. d. Diabetes mellitus 102. a . Ethylene glycol
105. b. Renal tubular acidosis 106. c. Vomiting
poisoning
103. d. Urine sodium ... 104. a. Sodium citrate
236 107. b. Anion gap 108. d. Carbonic acid
Fluid & Electrolytes

ADDITIONAL UPDATES
Prevention and Treatment of Tumor Lysis Syndrome I
1
Hydration by administration of normal or /
1
2 normal saline at 3000 mL /m2
per day

Keep urine pH at 7.0 or greater by administration of sodium bicarbonate


2
Administer allopurinol at 300 mg/m per day

If, after 24-48 h

£
Serum uric acid >8 mg/ dL
Serum creatinine >1.6 mg/dL
''
Serum uric acid < 8.0 mg/dL

Correct treatable renal failure ( obstruction)


Start rasburicase 0.2 mg/kg daily

11

Serum uric acid >8 mg/dL


Serum creatinine <1.6 mg/dL
Urine pH > 7.0

Fluid
&

Electroyes
Serum creatinine >1.6 mg/dL

r Start chemotherapy
' Discontinue bicarbonate administration
Delay chemotherapy if feasible or start Monitor serum chemistry every 6-12 h
hemodialysis ± chemotherapy

If serum potassium > 6 me/L


Serum uric acid >10 mg/dL
Serum creatinine > 10 mg/dL
Serum phosphate > 10 mg/dL or increasing
Symptomatic hypocalcemia present

'i
Begin hemodialysis
I
CHAPTER Nutrition
11
Most Recent Q's 2014- 15 8. Features of vitamin B12 deficiency are all except:
a. Pernicious anemia ( NBE Pattern 2014-15)
1. Death in PEM is due to all except: ( NBE Pattern 2014-15) b. Demyelinating disease
a. Hypothermia b. CCF c. Infertility
c. Worm infestation d . Electrolyte imbalance d. All the above
Ref: Nelson 18th ed. ch. 43 Ref: Park 17th pg 421 Harrison 19 th p 96 e - lt, 463t, 2607
2. Daughter of a red green colour blind father marries 9. Roti eating coolie comes with complaints of diarrhoea,
a normal man. What is the chance of having a normal on examination he has dermatitis of neck and exposed
child? part of body. The probable diagnosis is deficiency of:
( NBE Pattern 2014 )
a. 0 ( Manipal 2014 )
b. 25
c. 50 d . 100 a. Thiamine b. Niacin
c. Carbohydrate d. Riboflavin
Ref: Harrison 19 th p 197, 2629
Ref: Park 17th pg 420 Harrison 19 th p 96 e - 4
3. Diarrhoea, Dementia and Dermatitis are seen in defi ¬
10. The following are true about thiamine deficiency:
ciency of: ( NBE Pattern 2014-15)
(Manipal 2014 )
a. Niacin b. Thiamine
a. External ophthalmoplegia is a typical feature
c. Riboflavin d. Folk acid
b. Failure of the pupils to respond to light
Ref Harrison 19 th p 96 e - 4 c. The condition is found in patients with gastric
4. Thiamine deficiency causes: ( NBE Pattern 2014-15) carcinoma
a. Wernicke - Korsakoff syndrome d. Post- mortem reveals perivascular haemorr-hages,
b . Pellagra in the region of the fourth ventricle and aqueduct
c. Scurvy Ref Harrison 19 th p 96 e- 3
d. Night blindness Ref Harrison 19 th p 461 / 96 e- l 11. Bulls eye retinopathy is seen in toxicity of:
5. Differential diagnosis of scurvy: ( NBE Pattern 2014-15) ( Manipal 2014 )
a. Rickets a. Chloroquine
b. Hyper vitaminosis A b. Mefloquine
c. Hypervitaminosis D c. Primaquine
d . Pellagra Ref: Harrison 19 th p 461 d. Quinine
disease. Ref CMDT, 2013, pg 197 Harrison 19 th p 614 t, 619
6. X-linked dominant disease is
a. Vit D resistant rickets ( NBE Pattern 2014-15) 12. Not a complication of total parenteral nutrition:
b. Colourblindness a. Hyperammonemia ( Manipal 2014 )
c. Hemophilia b. Hypercholestrolemia
d. Gaucher's disease c. Neutrophil dysfunction
d . Hyperphosphatemia Ref: Harrison 19 th p 2460
Ref Kumar and Clark pg 178 Harrison 19 th p 2465
13. All are features of Wernicke encephalopathy except?:
7. Myotonic dystrophy is type of inheri ¬

( NBE Pattern 2014-15)


a. Ataxia ( Manipal 2014 )
tance.
b. External ophthalmoplegia
a. Autosomal dominant
c. Memory loss
b. Autosomal recessive
d. Tinnitus
c. X-linked recessive
d. X- linked dominant Ref: Kumar and Clark pg 1224 Ref Harrison 19 th p 41 e- IV, 96 e- 3, 209, 1783

Ans. 1. c. Worm infestation 2. c. 50 3. a. Niacin 4. a. Wernicke-Korsakoff ...


5. d . Pellagra 6. a. Vit D resistant rickets 7. a. Autosomal dominant 8. d . All the above
9. b Niacin 10. b. Failure of the pupils... 11. a . Chloroquine 12. d . Hyperphosphatemia
13. d . Tinnitus
Nutrition

14. Which vitamin is not associated with toxicity: c. Zinc


a. Pantothenic acid ( NBE Pattern 2014-15 ) d. Copper
b. Pyridoxine Ref: Harrison's 18th ed. ch. 74, Harrison 19th p 96 e -10
c. Vitamin A
d. Vitamin C 22 . Not improving with thiamine replacement:
a. Pulmonary edema ( NBE Pattern 2014 -15)
Ref: Harrison's 18th ed. ch. 74, Harrison 19th p 96e 5 - b. Psychosis
15. Most specific screening test for vitamin D deficiency
c. Peripheral neuritis
rickets:
a. Serum vitamin 25 hydroxy-vitamin D3 levels
-
( NBE Pattern 2014 15)
d. Opthalmoplegia
b. X-ray wrist joint Ref: Harrison's 18th ed. ch. 74, Harrison 19th p 96 e-3

16.
c. Serum calcium levels
d. Serum alkaline phosphatase
Ref: Harrison' s 18th ed. ch. 352, Harrison 19th p 2465,2485
Manifestations of Vitamin E deficiency are all except:
23. Which is not a cause of vitamin E deficiency?
a . Dietary deficiency
b.
c.
Celiac disease
Abetalipoproteinemia
( NBE Pattern 2014-15)
i
a. Hemolytic anemia ( NBE Pattern 2014-15) d. Cholestasis
b. Posterior column abnormalities Ref: Harrison's 17th ed. pg. 448, Harrison 19th p 96 e-lt
c. Cerebellar ataxia
d. Autonomic dysfunction 24. Significant weight loss is ( NBE Pattern 2014 15)-
a . 5% over 6-12 months
-
Ref: HI 8th ed. ch. 74, Harrison 19th p 96 e-lt, 96 e 8, 462e-19
b. 10% over 6-12 months
17. Vitamin E toxicity is associated with : c. 5 % over 6 weeks 2!
a. Ataxia ( NBE Pattern 2014-15)
b . Peripheral neuritis
d . 10% over 6 weeks ;
£2
Ref: Harrison's 18th ed. ch. 80, Harrison 19th p 274
c. Loss of vibrations
d. Exfoliative dermatitis 25. Calcium requirement of a growing school age child is rt
Ref: Harrison's 18th ed. ch. 74, Harrison 19th p 96 e -8 about: ( NBE Pattern 2014-15) Q
18. Hyperhomocystenemia can be treated with : a. 10 gm / day
b. 0.1 gm / day
a. Vitamin B 6 ( NBE Pattern 2014 - 15)
b. Vitamin B12 c. 1.0 gm / day
c. Folic acid d . 2.0 gm / day
d . All of the above Ref: Harrison 19th p 95e-3t, 2496 97 -
Ref: Harrison's 18th ed. ch. 74, Harrison 19th p 434e-4 26. Flushing with niacin is reduced by:
19. Calculate resting energy expenditure for a 70 kg sed ¬ a. Laropiprant ( NBE Pattern 2014-15)
entary man: ( NBE Pattern 2014-15) b. Premedication with aspirin
a. 1900 Kcal b. 2200 Kcal c. Tachyphylaxis
c. 2500 Kcal d . 2900 Kcal d. All of above
Ref . H 18th ed. Harrison 19th p 95e-l , 415e-2, 415e 5 - Ref: Harrison' s 18th ed. ch. 74, Harrison 19th p 96 e-4
20. Vitamin D resistant rickets has all the following char ¬ 27. Not recommended in coronary artery disease pa ¬
acteristics except: ( NBE Pattern 2014-15) tients: ( NBE Pattern 2014-15)
a. Hypophosphatemia a. Daily exercise
b. X- linked inheritance
b. Potassium
c. Reduced calcium absorption from gut
c. Vitamin E
d. Renal stones
d. Statins
Ref: Harrison's 18th ed. ch. 352, Harrison 19th p 2465
21. Mineral which activates action of insulin:
.
Ref Harrison's 17th ed. pg. 448, Harrison 19th p 96 e-3f, 96e-8
a . Selenium ( NBE Pattern 2014-15)
b. Chromium

Ans. 14. a.
18. d .
22. b.
26. d .
Pantothenic acid
All of the above
Psychosis
All of above
15. a.
19. a.
23. a.
27. c.
Serum vitamin 25 hydro... 16. d . Autonomic dysfunction 17. d. Exfoliative dermatitis
1900 Kcal
Dietary deficiency
Vitamin E
20. c. Reduced calcium..
-
21. b. Chromium
24. a. 5% over 6 12 months 25. c. 1.0 gm /day
d
i
th )
Marwah's Internal Medicine MCQs ( Based on Harriso n 's 19

28. Which vitamin's deficiency can cause lactic acidosis? 35. Kwashiorkor is characterized by all except:
a. Niacin ( NBE Pattern 2014-15) a. Wound dehiscence ( NBE Pattern 2014 -15)
b. Pantothenic acid b. Lymphopenia
c. Anergy

B
c. Thiamine
d. Riboflavin d. Low creatinine height index
Ref: Harrison' s 18th ed. ch. 74, Harrison 19th p 96 e- l Ref: Harrison's 18th ed. ch. 75, Harrison 19 thp 460, 463t

29. Dietary deficiency of which vitamin does not exist: 36. All are associated with development of obesity ex¬
cept: ( NBE Pattern 2014-15)
a . Vitamin B6 ( NBE Pattern 2014-15)
b. Thiamine a. Carpenter syndrome
b. Ahlstrom
c. Vitamin E
c. Laurence moon Biedl syndrome
d. Vitamin D
d. Turner syndrome
Ref Harrison’s 18th ed. ch. 74, Harrison 19th p 96e- lt
Ref: Harrison's 18th ed. ch. 77 and Nelson'18th ed. ch. 80
30. Severe malnutrition in adults is defined as BMI less
37 . All are features of scurvy except: ( NBE Pattern 2014-15)
than: ( NBE Pattern 2014 -15)
a. Cork screw hair
a. 18.5 b. 16
b. Follicular hyperkeratosis
c. 13 d . 11
c. Cellophane skin
Ref: Harrison' s 18th ed. ch. 76 , Harrison 19th p 459 d. Sub periosteal bleeding
31. Deficiency of which vitamin can lead to Methyl malo - Ref: Harrison’s 18th ed., Harrison 19th p 96e - lt, 461, 732
nic aciduria: ( NBE Pattern 2014-15)

Nutrion
38. Maximum half- life is of: ( NBEPattern 2014 -15)
a. Pyridoxal Phosphate
a. Albumin
b. Vitamin C
b. Transthyretin
c. Vitamin Blz
c. Retinol binding protein
d. Folic acid
Ref: H 18th ed. Harrison 19th p 96e-lt, 96e-8, 462e- 19 d. Transferrin
Ref: Harrison's 18th ed. ch. 75, Harrison 19th p 464t
32. All are diagnostic criteria for Kwashiorkor except:
a. Easily pluckable hair ( NBE Pattern 2014 -15) 39. . Not a cause of Beri -Beri: ( NBEPattern 2014 -15)
a. Malignancy b. Diuretic use
b. Decubitus ulcers
c. Dietary deficiency d. ATT
c. Serum albumin < 2.8 g/ dl
d. Triceps fold thickness < 3 mm Ref: Harrison's 18 ed. ch. 74, Harrison 19th p 96 e- lt, 2690

Ref: Harrison' s 18th ed. ch. 75, Harrison 19th p 460, 4631 40. Vitamin deficiency associated with coronary artery
disease? ( NBE Pattern 2014-15)
33. Which is not a cause of obesity associated hyperten ¬
( NBE Pattern 2014-15) a. Vitamin A b. Vitamin D
sion
c. Vitamin E d. Vitamin K
a. Insulin mediated salt retention
b. increased peripheral resistance Ref Harrison's 18th ed. ch. 74, Harrison 19th p 96e-7, 2463
c. Non -alcoholic hepatic steatosis 41. Main clinical distinction between anorexia nervosa
d. Obesity hypoventilation syndrome
Ref: Harrison's 18th ed. ch. 77,
and bulimia nervosa:
a. Binge eating
-
( NBE Pattern 2014 15)

Harrison 19th p 415e-6 , 1612, 1622 b. Electrolyte abnormalities


inhibito ry effect on wound c. Self- induced vomiting
34 . The Vitami n which has
( NBE Pattern 2014-15) d. Underweight
healing is:
b. Vitamin - E Ref: Harrison’ s 18th ed. ch. 76, Harrison 19 th p 2719-20
a. Vitamin -A
c. Vitamin - C d . Vitamin B-complex 42. Which vitamin toxicity is associated with excessive
Ref: Harrison 19th p 96e-8 sweating: ( NBE Pattern 2014-15)
a. Choline b. Biotin
c. Folic acid d . Vitamin B
Ref: Harrison's 18th ed. ch. 74, Harrison 19thp 95e -2t, 96e-5

Ans . 28. c. Thiamine 29. c. Vitamin E .


30. b 16 31. c. Vitamin B12
32. d. Triceps fold thickn...33. c. - ..
Non alcoholic hepatic. .
34. b Vitamin E
.
38. a Albumin
- 35. d . Low creatinine height index
36. d. Turner syndrome 37. c. Cellophane skin 39. d . ATT
240 40. b. Vitamin D 41. d . Underweight .
42. a Choline
Nutrition

43. Dermatitis may be a clinical manifestation of defi ¬ 50. Poor prognostic factors in a child with Kwashiorkor
ciency states of all of following nutrients except: include all of the following except: (NBEPattern 2014-15)
a. Biotin (NBE Pattern 2014-15) a. Hypothermia b. Hepatomegaly
b. Niacin c. Purpura d. Dermatosis
c. Pyridoxine Ref: Harrison's 18th ed. ch. 75, Harrison 19th p 460t I
d. Thiamine
51. Not a component of involuntary weight loss
Ref: Harrison's 18th ed. ch. 74, Harrison 19th p 96e- lt, 373 a. Osteopenia (NBE Pattern 2014 -15 ) |
44. All are false statement about pyridoxine except: b. Sarcopenia
(NBE Pattern 2014-15) c. Cachexia
a. Deficiency can cause hemolytic anemia d. Anorexia
b. Deficiency can cause sideroblastic anemia
c. RDA for pyridoxine 5 mg.
Ref: Harrison's 18th ed. ch. 80, Harrison 19th p 275-76
d. Can cause burning foot syndrome 52. Vitamin E deficiency presents with all except?
Ref: Harrison's 18th ed. ch. 74, Harrison 19th p 96e-lt, 360 a. Ataxic gait (NBE Pattern 2014-15)
b. Peripheral neuropathy
45 . Low serum copper is seen in all except?
a. Aceruloplasminemia (NBE Pattern 2014-15)
c . Opthalmoplegia
d. Peri-follicular rash
b. Menke's disease
c. Wilson disease Ref: Harrison's 17th pg. 448, Harrison 19th p 96e- lt, 96e-8
d. Primary biliary cirrhosis 53. Cause of recent onset genu valgum and metaphyseal <

Nutrion
Ref Harrison 19th p 96e-10, 2690- 91 widening in a child with serum calcium 11.0 mEq /L jj
46. One of the following is not reported to be a clinical serum parathormone increased and phosphate 2.8 I
manifestation of zinc deficiency state in children: mEq /L is:
a. Dwarfism and hypogonadism (NBEPattern 2014-15) a. Nutritional rickets (NBE Pattern 2014-15)
b. Liver and spleen enlargement b. Azotemic osteodystrophy
c. Impaired cell mediated immunity c. Hypophosphatemic rickets
d. Macrocytic anemia d. Primary hyper-parathyroidism
Ref: : Nelson 18th ed. ch 670, Harrison 19th p 96 e- 9, 461 Ref: : Nelson 18th ed. ch. 48, Harrison 19th p 332e-7t, 2458
47. Vitamin D resistant rickets occurs due to all except: 54. Maximum dose of vitamin A in pregnancy:
a. 1hydroxylase deficiency (NBE Pattern 2014-15) a. 30pg (NBE Pattern 2014-15)
b. Renal tubular acidosis b. 300ug
c. Fanconi syndrome c. 3mg
d. Drugs d. 30mg
Ref Harrison's 18th ed. ch. 352, Harrison 19th p 2465 Ref: Harrison's 18th ed. ch. 74, Harrison 19th p 256e-12, 96e-7
48. Not seen with vitamin C toxicity: (NBE Pattern 2014-15) 55. Photosensitivity is seen with deficiency of which vita ¬

a. Abdominal pain b. Kidney stones min: (NBE Pattern 2014-15)


c. Hemolysis d. Thrombosis a. Niacin b. Pyridoxine
c. Folic acid d. Vitamin B12
Ref Harrison’s 18th ed. ch. 74, Harrison 19th p 96e-5
49. Most common problem with parenteral nutrition is:
Ref: Harrison's 18th ed. ch. 75, Harrison 19th p 96e-4
56. Not a treatment for porphyria: (NBE Pattern 2014-15)
a. Fluid overload (NBE Pattern 2014 -15)
a. Hemin b. Beta carotene
b. Hyerinsulinemia
c. Hydroxychloroquine d. Insulin
c. Hyperglycemia
d. Hypertriglyceridemia Ref: Nelson 18th ed. ch. 91, Harrison 19th p 367e-3, 2530-31
Ref Harrison's 18th ed. ch. 76, Harrison 19th p 98e-6 -8, 98e- 7t

Ans. 43. d. Thiamine 44. b. Deficiency can cause ...


47. d. Drugs 48. d . Thro... 49 . a . Fluid 45 . d. Primary biliary cirrh.. 46. d. Macrocytic anemia
52. d . overload 50. b. Hepatomegaly 51 . a . Osteopenia
Peri-follicular rash 53. c. Hypoph
56. d .
osphatemic ric.. 54. c. 3mg 55. a . Niacin
Insulin

I
th )
Marwah's Internal Medicine MCQs ( Based on Harrison's 19

57. Vitamin D deficiency rickets is confirmed by demon ¬


59. Cobalt is a part of which vitamin: (NBE Pattern 2014-15)
stration of: ( NBE Pattern 2014 15)- a. Retinol b. Pyridoxine

a
a . Epiphyseal changes in X-ray c. Cyanocobalamin d . Vitamin C
b. Hypocalcaemia and hypo- phosphatemia -
Ref: Harrison's 18th ed. ch. 74, Harrison 19th p 1691, 263e 3t
c. Raised serum alkaline phosphatase
d . Healing with physiologic doses of vitamin D3
Ref: : Nelson 18th ed. ch. 48, Harrison 19th p 2465-66 , 2485
58. Most common cause of death in Menke's disease is?
a. Cardiac rupture ( NBE Pattern 2014 -15)
b. Diaphragmatic paralysis
c. Laryngospasm
d . Cyano -methemoglobinemia
Ref . Harrison' s 17th ed. pg. 449, Harrison 19th p 96e-10

Nutrion

\
I Ans. 57. d . Healing with physiologic doses of vitamin D 3 58. a . Cardiac rupture 59. c. Cyanocobalamin

242
CHAPTER Tumours

12
7. Best prognosis in H.C.C is seen in:
Carcinogens (NBE Pattern 2014-15)
a. Fibrolamellar hepatoma
b. Hepatoblastoma
1. Streptococcus bovis infection is associated with: c. Angiosarcoma
a. CLL (NBE Pattern 2014-15)
d. Cholangiosarcoma
b. Hairy cell leukemia Harri son ’s 18 th ed., p 784 ,
c. Colorectal cancer
Ref:

d. Multiple myeloma 8. Which is a hormone dependent liver tumor:


Harri son' 18th ed., Table 86 -3 , Harri son 19th p 104 -3, 486 a. Hepatoma (NBE Pattern 2014-15)
Ref: s
b. Hemangioma
2. Most significant risk factor for cancer is:
(NBE Pattern 2014-15] c. Hepatocellular carcinoma
a. Age
d. Hemangiopericytoma
b. Smoking ., ch. ,
Ref: Harri son' s 18th ed 92
c. Viruses
d. Physical inactivity
CNS Tumours
Ref: Harrison' s 18th ed., ch. 81, Harrison 19th p 467
Liver Cancer 9. Most common site of metastasis of Neuro-blastoma?
a. Lung (NBE Pattern 2014-15)
b. Liver
3. Which of the following is effective against both HCC
and RCC : (NBE Pattern 2014-15)
c. Bones
d. Bone marrow
a. Erlotinib .
b Sorafenib
Nelso n 18th ed., ch. 498: Neur oblas toma , Neslo n 19th p 1754
c. Bortezomib d. Cetuxtimab Ref:
10. Not a clinical feature of Neuroblastoma:
Ref: Harrison's 18th ed., table 84-2 (NBE Pattern 2014-15)
a. Paraplegia
4. Polychronotropism is seen with:(NBE Pattern 2014-15) b. Raccoon eyes
a. Urothelial tumors b. G.I.S.T c . Opsoclonus
c. Pancreatic cancer d. Hepatocellular cancer d. Recurrent syncopal attacks
Ref: Harrison's 17th ed., p 589, .
Ref: OP Ghai 7th ed , p 590 -91 , Harrison 19th p 618
5. Most common liver tumour: (NBE Pattern 2014-15) 11. A 2 year old baby with hypertension and increased
a. Metastasis urinary HVA is suggestive of: (NBE Pattern 2014-15)
b. Hemangioma a. 11 hydroxylase deficiency
c. Hepatocellular carcinoma b. Neuroblastoma
d. Focal nodular hyperplasia c. Wilms tumor
Ref: Robbins 8th ed., p 663-666 d. Phaeochromocytoma

6. The most common cause of hepatocellular carcinoma


.
Ref: Nelson 18th ed , ch. 498, Neuroblastoma,
in India is: (NBE Pattern 2014-15) 12. Gold standard test for neuroblastoma:
a. Hepatitis A a. Immunohistochemistry (NBE Pattern 2014-15)
b. Hepatitis B b. N-myc oncogene
c. Non A Non B hepatitis c. Serum ferritin
d. Alcoholic cirrhosis d. MIBGscan
Ref: Harrison’s 18th ed., ch. 92, P. 777, Harrison 19th p 544 Ref: OP ghai 7th ed., p 570, Neslon 19th p 1754

1. c. Colorectal cancer 2. a . Age 3. b. Sorafenib 4 . a. Urothelial tumors


5. a . Metastasis 6. b. Hepatitis B 7. a . Fibrolamellar hepatoma 8. a. Hepatoma
9. c. Bones 10. d. Recurrent syncopal... 11. b. Neuroblastoma 12. a. Immunohistochemistry
Marwah's Internal Medicine MCQs ( Based on Harrison's 19th )

13 . Most common site of Neuroblastoma? b. 5% of patients with lung cancer present with,
a. Adrenal gland ( NBE Pattern 2014- 15) or develop complications of non -metastatic
b. Retroperitoneum paraneoplastic syndromes
c. Pelvis
c. Syndrome of Inappropriate Anti Diuretic Hormone
d . Cervical area
(SIADH ) is associated with hypernatraemia
{ Ref: 0 P Ghai 7 th ed > P 590 Neslon 19th p 1754 d . 80-90 % of small cell carcinomas have spread
BHH 14. Not associated with neuroblastoma: beyond the thorax at the time of diagnosis
a. Neurofibromatosis ( NBE Pattern 2014 -15) Ref: Harrison's 18th ed., ch. 89 , Harrison 19 th p 508, 11
b. Hischsprung's disease
c. Heterochromia 20. Hyperglycaemia is seen in which type of bronchogen ¬
d. Defect on chromosome 2 ic carcinoma: ( NBE Pattern 2014 15) -
a. Squamous cell carcinoma
Ref: OP Ghai 7th ed., p. 90, b. Oat cell carcinoma
c. Adenocarcinoma
Lung Cancer d. Large cell carcinoma
Ref: Harrison's 18th ed., ch. 89 , Harrison 19 th p 511
15. Clubbing is least common in : ( NBE Pattern 2014 -15)
21. The most radiosensitive lung cancer is:
a . Squamous cell carcinoma
a. Squamous cell carcinoma ( NBE Pattern 2014-15)
b. Adenocarcinoma b. Small cell anaplastic carcinoma
c. Small cell carcinoma of lung c. Adenocarcinoma
d. Giant cell carcinoma

Tmours
d. Mesothelioma
Ref: Harrison’s 18th ed., ch. 89 , Harrison 19 th p 511 Ref: Harrison' s 18th ed., ch. 89 , Harrison 19 th p 522

16. Pancoast tumour causes: ( NBE Pattern 2014 -15) 22. The most common type of lung carcinoma in non -
smokers is: ( NBE Pattern 2014 -15)
a. Increased sweating
a . Squamous cell carcinoma
b . Wasting of muscles of hand
b. Adenocarcinoma
c. Destruction of lower ribs c. Small cell carcinoma
d. Bony metastasis d. Large cell carcinoma
Ref Ch. 89, Harrison’s 18th ed.,, Harrison 19 th p 510- 11 Ref Harrison's 18th ch. 89, p. 738 ,
17 . Most common primary lung cancer in children: 23 . Most common cancer associated with asbestosis:
a. Metastasis ( NBE Pattern 2014 -15) a. Adenocarcinoma ( NBE Pattern 2014 -15)
b. Bronchial adenoma b. Mesothelioma
c. Squamous cell carcinoma
c. Carcinoids
d. Bronchial adenoma
d. Pulmonary blastoma .
Ref: Harrison's 18th ed., ch 256 , Harrison 19 th p 1689
Ref: Nelson 18th ed., ch. 408
24. Which one of the following types of bronchogenic car ¬
18. Which of the following statement about lung carcino ¬ cinoma is most likely to cavitate:
ma is true? ( NBE Pattern 2014-15) a. Adenocarcinoma ( NBE Pattern 2014 -15)
a. Squamous cell variant accounts for 70% of all lung b. Bronchoalveolar carcinoma
cancers c. Oat - cell carcinoma
b. Oat cell variant typically present with cavitation d . Squamous cell carcinoma
c. Oat cell variant presents an endobronchial growth Ref: BAILEY AND LOVE 26 th ed., p. 861 and DeVita 8th ed.,ch. 37
d. Adenoca rcinoma variant is typically central in
25. Following are increased in small cell carcinoma of
location lung except ( NBE Pattern 2014-15)
Ref H 18th ed., ch. 89 , Harrison 19 th p 508 a. ACTH
ng lung can b. Growth hormone
19. Which one of the followin g is true regardi ¬
c. ANF
cer? ( NBE Pattern 2014-15 )
d. AVP
a . Adenocarcinomas tend to grow quickly .
Ref: Harrison’ s 18th ed., ch. 100, p 738, Harrison 19 th p 511

. c. Small cell carcinom a ... 16. b. Wasting of muscles of ..


Ans. 13 a Adrenal gland 14. d . Defect on chromoso me 2 15
18. c. Oat cell variant pres... 19. d. 80 -90 % of small 20. b. Oat cell carcinoma
17. a . Metastasis 24. d. Squamous cell carcinoma
21. b. Small cell anapla.. 22. b. Adenocarcinoma 23. a . Adenocarcinoma
244
25. b. Growth hormone
Tumours

26 . A 70 - year - old smoker male presents with breathless¬ c. Amoebic abscess


ness, facial swelling. Incorrect is? ( NBEPattern 2014-15) d. Ca lung
a. Dilated veins have blood flow away from umbilicus Ref Harrison' s 18th ed., ch. 89
b. Worse on bending forward
c. Non - pulsatile elevated JVP 33 . Most common symptom of lung cancer?
a . Weight loss ( NBE Pattern 2014 -15)
d. Glossal edema
b. Hemoptysis
Ref: H 18th ed., p. 2266 , Harrison 19th pi 787 -88
c. Chest pain
27 . A 55 - year-old man, a chronic smoker is brought to d . Clubbing
emergency with history of polyuria, polydipsia, nau¬ Ref: Harrison' s 18th ed., Harrison 19th p 510; Table 107 -4
sea and altered sensorium for last two days. He had
been diagnosed as having squamous cell carcinoma of
lung two months prior to this. On examination, he was G. I. Tumours
lethargic and confused. An ECG was normal except for
a narrowed QT interval . Which one of the following is
34. A young male presented with asymptomatic lymph
the most likely metabolic abnormality?
node enlargement in supraclavicular region . The bi ¬
a . Hypernatremia ( NBE Pattern 2014 -15)
b. Hypercalcemia opsy shows squamous cells, the likely site of carci ¬
c. Hypokalemia noma will be :
d. Hyponatremia a . CA stomach ( NBE Pattern 2014-15)
Ref: Harrison's 18th ed., p. 743, Harrison 19th p 2469/ 247 b. CA colon
c. CA Breast

Tumors
28. Popcorn calcification is seen in:( NBE Pattern 2014-15)
d. CA Lung
a . Pulmonary hamartoma b. Pulmonary hemorrhage
Ref: CMDT-2014, ch. 39, p. 1597
c. Pulmonary teratoma d. Pulmonary embolism
Ref Ch. 89: Harrison's 18th ed., Harrison 19th p 515 35. Which is rarest site of spread of colorectal cancer?
a. Liver ( NBE Pattern 2014 -15) !
29. 5 year survival rate for small cell cancer lung is:
b. Lung
a. 1% ( NBE Pattern 2014 -15)
c. Brain
b. 5%
d. Peritoneum
c. 10%
d. 25% Ref Harrison's 17th ed., p 577 , Harrison 19 th p 541
Ref: BAILEY AND LOVE 26 th ed., p. 861 , 36 . Aspirin decreases the risk of development of which of
30. Most common carcinoma of lung is: the following: ( NBE Pattern 2014-15)
a. Squamous cell carcinoma ( NBE Pattern 2014 -15) a. Colorectal cancer
b. Adenocarcinoma b. Stomach cancer
c. Small cell carcinoma c. Carcinoid
d. Large cell carcinoma d . MALToma
Ref: Harrison's 18th ed., ch. 89, p 738, Ref: Harrison' s 18th ed., ch. 91
31. Bronchial adenoma commonly present as: 37 . Most common site of spread of colorectal cancer is?
a. Recurrent hemoptysis ( NBE Pattern 2014-15] a - Liver ( NBE Pattern 2014-15)
b. Chronic Cough b. Lung
c. Dyspnea c. Brain
d. Chest pain d. Peritoneum
Ref: Harrison's 18th ed., ch. 89 Ref: Harrison's 17th ed., p 577
38. Peutzjehgers syndrome is associated with:
32 . Well - defined rounded opacity in the lung with coarse
a . Ovarian tumor ( NBE Pattern 2014-15)
irregular calcification is a feature of:
( NBE Pattern 2014 -15) b. Brain tumor
a. Hamartoma c. Osteoma
b. Hydatid cyst
d. Lung cancer
Ref: Harrison's 18th ed., table 91 - 5, ch. 91 , Harrison 19th p 538t

Ans . 26. a . Dilated veins have 27. b. Hypercalcemia 28. a. Pulmonary hamartoma 29. b. 5%
30. b . Adenocarcinoma 31. b. Chronic Cough 32. a. Hamartoma .
33 a . Weight loss
34. d. CA Lung 35. b. Lung 36. a. Colorectal cancer 37. a. Liver
38. a . Ovarian tumor 245
Marwah 's Internal Medicine MCQs ( Based on Harrison’s 19th )

39. Not seen with stomach cancer? ( NBE Pattern 2014 15) - 46 . Percentage of blockage of esophagus by cancer to
a. Acanthosis Nigricans cause symptoms: ( NBE Pattern 2014-15)
b. Superficial migratory thrombophlebitis a . 30% b. 40 %
OBB c. Microangiopathic hemolytic anemia c. 50 % d. 60 %
d. Polycythemia Ref: Harrrison's 18th ed., ch. 91 and 17th ed., p. 571 ,
Wjm Rtf: H 18Mi ed., ch. 91 , Harrison 19 th p 534
47. Not seen with carcinoid is? ( NBE Pattern 2014 -15)
H 40. Most common site of spread of gastric tumor? a . Peyronie disease b. Mitral valve involvement
a - Liver ( NBE Pattern 2014-15)
c. Cushing disease d. Hypertension
b. Ovary
c. Peritoneum Ref: Harrison's 17th ed., p 2351-52 and Harrison’s 18th ed., ch.
d. Blumer shelf 350 , Harrison 19th p 564
Ref: H 18th ed., ch. 91 , Harrison 19 th p 534 48. Flushing is most common seen with?
41. Enteropathy type T cell lymphoma is associated with: a . Appendiceal carinoid ( NBE Pattern 2014 -15)
a. M .A.L.Toma
b. Celiac Sprue
( NBE Pattern 2014 15) - b. Midgut carcinoid
c. Hindgut carcinoid
c. Menetrier disease d . Gastric carcinoid
d . Crohn's disease Ref Harrison's 17th ed., p 2351 and Harrison's 18th ed., ch. 350 ,
Ref: Harrison's 18th ed., Less common hematological malignancy, Harrison 19 th p 563
42 . All are risk factors for development of colorectal can ¬ 49. Most common cardiac lesion in carcinoid syndrome
cers except? ( NBE Pattern 2014 15) - is? ( NBE Pattern 2014 -15)

Tmours
a. Intake of vegetable fat a . Tricuspid regurgitation
b. Polyposis coli b. Pulmonic stenosis
c. 1 BD c. Mitral regurgitation
d. Lynch syndrome d . Aortic stenosis
e. All
Ref: Harrison's 17th ed., p 2351 and Harrison's 18th ed., ch. 350 ,
Ref: Harrison's 18th, table 91 - 4 , Harrison 19 th p 538 Harrison 19 th p 564
43. Tumor exhibiting regression to anti - microbials? 50. Not used for diagnosis of carcinoid syndrome ?
a. Kaposi sarcoma ( NBE Pattern 2014 -15)
( NBE Pattern 2014-15)
b. M .A. L.T.oma
a. Somatostatin receptor scintigraphy
c. G.I .S.T
d. Zollinger Ellison syndrome b. Chromogranin A
c. Neuron specific enolase
Ref: Harrison's 18th ed., ch. 91 and 17th ed., p. 573,
d. Synaptophysin
44. Incorrect about colorectal cancer? (NBEPattern 2014-15) ( Ref : Harrison's 18th ed., ch 350 , Harrison 19 th p 566
a . Right sided lesions associated with occult bleeding
b. Left sided lesions associated with tenesmus 51. Most common site of carcinoid tumor leading to carci ¬
c. Apple core appearance on barium study noid syndrome? ( NBE Pattern 2014-15)
d. Haem-occult is best screening tool a. Appendicular carcinoid
Ref : Harr ison ’ s 17 th ed., p 576 , Harr ison 19 th p 541 b. Metastatic Midgut carcinoid
t? c. Midgut carcinoid
45. Com plica tions of esop hage al canc er are all excep
d. Bronchial carcinoid
a. Trach ea -esop hage al fistul a ( NBE Patte rn 2014 -15)
Ref: Harrison's 17th ed., p. 2351 and Harrison' s 18th ed., ch. 350
b. Hypercalcemia

I c.
d.
Aspiration pneumonia
Gastric tetany
Ref Harrison's 18th ed., ch. 91 Harrison 19 th p 583
52. Drug of choice for carcinoid crisis? (NBEPattern 2014-15)
a. Methysergide
c. Octreotide
b. Cyproheptadine
d. Kentaserin

I Ref: Harrison's 17th ed., p 2353 and Harrison's 18th ed., ch. 350,
Harrison 19 th p 566

t Ans. 39.
43.
47.
d. Polycythemia
b. M.A . L.T.oma
d . Hypertension
40. a. Liver
44. d . Haem -
occul t is
48. b. Midgut carcinoid
best...
41. b. Celiac Sprue
45. d. Gastric tetany
42 . a. Intake of vegetable tat
46. d. 60%
49. a. Tricuspid regurgitation 50. d. Synaptophysin
246 b. Metastatic Midgut... 52. c. Octreotide
51.
Tumours

c. Hemangioma
53. Most common symptom of carcinoid syndrome is:
a. Diarrhea ( NBE Pattern 2014 -15) d. Fibrous histiocytoma
Ref: Harrison's 18th ed., ch. 98 ,
b. Flushing

B
c. Pellagra 60. Rhabdomyosarcoma with the worst prognosis is:
d . Bronchospasm a. Alveolar type ( NBE Pattern 2014 15) -
Ref: Harrison's 17th ed., p. 2351 and Harrison's 18th ed., ch. 350 , b. Botyroid type
Harrison 19 th p 564 c. Pleomorphic type
54. All are correct about atypical carcinoid except? d. Embryonal type
a . High urinary 5HTP ( NBE Pattern 2014-15) Ref : Nelson 18th ed., ch. 500
b. high urinary serotonin
61. Rhabdomyosarcoma in children is most commonly
c. Absent dopa decarboxylase ( NBE Pattern 2014-15)
present at which site:
d. High urinary 5- HIAA b. Extremities
a. Head and neck
Ref: Harrison's 17th ed., p 2352 and Harrison’s 18th ed., ch. 350 , c. Genitourinary d. Sacral area
Harrison 19 th p 566
Ref: Harrison's 18th ed., ch. 500
Soft Tissue Tumours 62 . Most common extra-nodal site for lymphoma is?
a . Stomach ( NBE Pattern 2014-15)
55 . Most common malignant tumor of heart in adults? b. Colon
a. Angiosarcoma
b. Rhabdomyoma
-
( NBE Pattern 2014 15) c. Intestine

Tumors
d . Waldeyer ring
c. Lipoma
Ref: Harrison's 18th ed., ch. 91 and 17th ed., p. 573 \
d. Paraganglioma
Ref: Harrison's 18th ed., ch. 240 , Harrison 19 th p 289e 63 . Most common cause of dry tap on bone marrow ex- 5
animation is: ( NBE Pattern 2014 -15) \
56. All of the following statements about atrial myxomas a. Myelofibrosis
are true, except: ( NBE Pattern 2014 -15) b. Hairy cell leukemia
a. Most common site is left atrium c. Cancer infiltration into bone marrow
b. More common in females d . Chronic myeloid leukemia
c. Tumor plop in systole Ref: Harrison's 18th ed., ch. 107
d. Raynaud phenomenon 64. All of the following are features of tumor lysis syn¬
Ref: Harrison's 18th ed., ch. 240, Harrison 19 th p 289 e drome except: ( NBE Pattern 2014-15)
a. Hyperuricemia b. Hyperkalemia
57 . Most common tumor of heart valves:
c. Hyperphosphatemia d. Hypercalcemia
a. Papillary fibroelastoma ( NBE Pattern 2014-15)
Ref: Harrison' s 18th ed., ch. 276, p 2274, Harrison 19 th p 1795
b. Malignant fibrous histiocytoma
c. Angiosarcoma 65 . Most common cause of tumor lysis syndrome in chil ¬
d . Rhabdomyosarcoma dren? ( NBE Pattern 2014-15)
Ref: Harrison' s 18th ed., ch. 240 , Harrison 19 th p 289e a. Burkitt lymphoma
b. Acute leukemia
58. Most common soft tissue tumor in children? c. CLL
a. Rhabdo- myosarcoma ( NBE Pattern 2014 -15) d . Neuroblastoma
b. Lipoma Ref: O.P Ghai, 7th ed., p. 597 , Harrison 19 th p 1795
c. Hemangioma
d. Fibrous histiocytoma Paraneoplastic Syndrome
Ref: Nelson 18th ed., ch. 500 ,
59. Most common malignant soft tissue tumor in adults: 66. Most common MALIGNANT tumor of heart in chil -
a . Rhabdo-myosarcoma ( NBE Pattern 2014-15) drenis: ( NBE Pattern 2014-15)
b. Lipoma a. Rhabdomyoma b. Rhabdomyosarcoma
c. Angiosarcoma d. Lipo
Ref: Nelson 18th ed., ch. 441
Ans . 53. a . Diarrhea 54. d. High urinary 5-HIAA 55. a. Angiosarcoma 56. c. Tumor plop in systole
57. a. Papillary fibro... 58. a . Rhabdo-myosarcoma 59. d . Fibrous histiocytoma 60. a. Alveolar type
61. a . Head and neck 62. a . Stomach 63. c. Cancer infiltration... 64. d . Hypercalcemia
65. a . Burkitt lymphoma 66. b. Rhabdomyosarcoma 247
th )
Marwah's Internal Medicine MCQs ( Based on Harrison's 19

67. All of the following Para - neoplastic syndrome are c. Paraneoplastic cerebellar degeneration
seen in Ca lung except: ( NBE Pattern 2014-15) d. Lambert Eaton syndrome
a . Hypertrophic osteodystrophy Ref Harrison 19 th p 614; Table 122 -2
b. Myasthenia gravis
74. Paraneoplastic syndrome associated with RCC are all
c. Cushing's syndrome ( NBE Pattern 2014-15)
of the following except:
d. Hypoglycemia
a. Polycythemia
Ref: Harrison’s 18th ed., ch. 89, p. 743, 744, b. Hypercalcemia
Harrison 19 th p 511 / 609 c. Malignant hypertension
68. Lambert Eaton syndrome is associated with ? d. Cushing syndrome
a. Anti P / Q antibodies ( NBE Pattern 2014 - 15) Ref Harrison's 18th ed., ch. 94 Harrison 19 th p 578 / 614
b. Anti - HU antibodies ( NBE Pattern 2014 -15)
75. Aflatoxin causes:
c. Anti - Jo-1 antibody
a . Lathyrism
d . Anti GQ1 B antiboy
b. Hepato cellular carcinoma
Ref: Harrison' s 18th ed., ch. 89, p. 743, Harrison 19 th p 2703 c. Cholangiocarcinoma
69. ANF is produced by: ( NBE Pattern 2014-15) d . Botulism
a . Squamous cell carcinoma Ref: Harrison's 18th ed., ch. 92, P. 777,
b. Adenocarcinoma
76. Opsoclonus is seen with all except?
c. Alveolar cell carcinoma ( NBE Pattern 2014-15)
a. Neuroblastoma
d . Oat cell carcinoma b. Lung cancer

Tmours
Ref Harrison' s 18th ed., ch. 100, p. 738, c. Brain cancer
70. True statement about pancoast tumour is: d . Breast cancer
a. Affects left lower lobe ( NBE Pattern 2014 -15) Ref: Harrison's 18th ed., ch. 100 , Harrison 19 th p 618
b. Causes radicular pain in upper limb Lynch syndrome is characterized by:
77.
c. Produces recurrent laryngeal nerve palsy
d. Always associated with adenocarcinoma ( NBE Pattern 2014-15)
a. One or more case in family of colorectal cancer < 50
Ref: Harrison' s 18th ed., ch. 89, p. 742,
years
Harrison 19 th p 510 -511 b. 2 or more cases in family of polyps
71. All are paraneoplastic syndromes except: c. Cancer arising in recto-sigmoid junction
a. Cerebellar degeneration ( NBE Pattern 2014 -15) d. Defect on chromosome 10
b. Progressive multifocal leukoencephalopathy Ref H 18th ed., ch. 91 , Harrison 19 th p 538t
c. Amyotropic lateral sclerosis 78. Pellagra is seen with: ( NBE Pattern 2014 -15)
d . Opsoclonus myoclonus a. Carcinoid
Ref Harrison's 18th ed., ch. 101, p. 833, Harrison 19 th p 614 b. Phaeochromocytoma
72 . Horner's syndrome is associated with long standing c. Peutzjegher syndrome
d . Cronhkhite Canada syndrome
pain in shoulder and arm . This presentation is mim ¬
icked by all except? ( NBE Pattern 2014 -15) Ref: Harrison’s 17th ed., p 2351 and Harrison’ s 18th ed., ch. 350
a. Chronic stable angina
b. Coronary ostial occlusion
Paediatric -malignancy
c. Cervical spondylosis d. Cervical rib
.
Ref Harrison's 18th ed., ch 89, p. 750, 3471 ,
79. Most common benign tumor of heart in children is:
73. Ampiphysin antibodies are seen in? a. Rhabdomyoma ( NBE Pattern 2014 -15)
a . Stiff person syndrome ( NBE Pattern 2014-15) b. Rhabdomyosarcoma
b. Opsoclonus
c. Angiosarcoma
d. Lipoma Ref: Nelson 18th ed., ch. 441

Ans. 67. d. Hypoglycemia 68. a. Anti P /Q antibodies 69. d . Oat cell carcinoma 70. b. Causes radicular pain. ..
. .
71 b. Progressive multif .. 72. b. Coronary ostial occlusion 73. a. Stiff person syndrome 74. None
248 ..
75. b. Hepato cellular. .
76. c Brain cancer .
77. a. One or more case in .. 78. a . Carcinoid
79. a . Rhabdomyoma
Tumours

80. Incorrect about Wilm's tumor: ( NBE Pattern 2014-15) c. Carcinoma gut
a. Associated with hemi-hypertrophy d. Malignant melanoma
b. Hematuria Ref: Harrison's 17th ed., p 2609

Q
c. Embolise to lungs and heart
d. Most common malignant tumor of abdomen of Miscellaneous
childhood
Ref: Nelson 18th ed., ch. 499 , 88. Indicators of internal malignancy are all except:
a. Acanthosis nigricans ( Manipal)
81. Most common intra -abdominal mass in neonates is?
b. Generalised pruritis
( NBE Pattern 2014 -15)
c. Deep thrombophlebitis
a . Multi -cystic dysplastic kidney
d. Dermatomyositis
b. Wilm's tumor Ref: Kumar and Clark pg 1301
c. Hydronephrosis
d. Rhabdomyosarcoma 89. All are seen in tumor lysis syndrome except:
a. Hypercalcemia ( Manipal)
Ref Nelson textbook 18 th ed., ch. 537
b. Hypocalcemia
82. Most common symptom of Wilm's tumour? c. Hyperphosphatemia
a. Abdominal pain ( NBE Pattern 2014-15) d. Hyperkalemia
b. Hypertension
Ref: Harrison's 18th pg 2274 Harrison 19 th p 1804, 310
c. Hematuria
d. Fever 90. Southern blot" technique is useful on:

Tumors
Ref OP ghai 7th ed., p 570, Neslon 19th p 1758 a. DNA ( Manipal)
b. Proteins
83 . Brain tumor showing diffuse infiltration of brain c. RNA
without focal mass: ( NBE Pattern 2014 15) - d. None of the above Ref: Kumar & Clark pg 168
a. Glioblastoma multiforme
b. Gliomatosis cerebri 91. "Gene Therapy" is: ( Manipal )
c. PNET a. Delivery of new genes
d. Meningioma b. Treatment of AIDS
Ref Harrison's 17th ed., p 2604, c. Delivery of nucleic acid to alter a pathological
process
84. Cowden syndrome is associated with: d. Delivery of new chromosomes
a . Medulloblastoma ( NBE Pattern 2014-15)
Ref: Harrison 19 th p 214e- 9
b. Astrocytoma
c. Dysplastic gangliocytoma 92 . All of the following cancers are linked with HIV ex¬
d. PNET cept:
Ref Nelson 18th ed., p 2129, table 497-1, Harrison 19 th p 2344
a. Kaposi sarcoma
b. Urogenital cancer
( NBE Pattern 2014 15)-
85. Most common part of spine involved in malignant spi ¬ c. Non Hodgkin lymphoma
nal cord compression: ( NBE Pattern 2014 15) - d. Adult T cell leukemia
a. Cervical b. Thoracic ,
Ref: Harrison' s 18 h ed., ch. 81 , Harrison 19 th p 1268, 70
c. Lumbar d. Sacral
93. Most common cause of death in cancer is?
Ref Harrison's 17th ed., p. 2609 ,
86. Chang classification is used for?( NBE Pattern 2014 -15)
a. Bleeding
b. Infection
( NBE Pattern 2014 15) -
a. PNET b. Astrocytoma c. Respiratory failure d. Renal failure
c. Pinealoma d . Craniophyrangioma Ref: Harrison' s 18th ed., ch. 81 , Harrison 19 th p 484
Ref Nelson textbook of pediatrics: 18th ed., p. 2134-35,
94. Which is the most common tumor leading to death in
87 . Most common cause of Carcinomatous meningitis:
a. Carcinoma breast ( NBE Pattern 2014-15)
adults?
a. Lung cancer
( NBE Pattern 2014 15)
b. Prostate cancer
-
b. Carcinoma lung c. Colorectal cancer d. Leukemia
Ref: H 18th ed., table 81-1 , Harrison 19 th p 470, 471; Table 99-2

Ans . 80 . d. Most common... 81 . a . Multi-cystic dysplastic... 82. a. Abdominal pain 83 . b. Gliomatosis cerebri
84. c. Dysplastic gang... 85. b. Thoracic 86 . a. PNET 87 . a . Carcinoma breast
88. c. Deep thrombophl... 89 . a. Hypercalcemia 90 . a. DNA 91 . c . Delivery of nucleic acid ...
92. d . Adult T cell leukemia 93 . b. Infection 94. a . Lung cancer 249
I
I
I
Marwah 's Internal Medicine

95. All are seen in Carney's triad except?


,
MCQs ( Based on Harrison’s 19 h )

c. Bellini tumour d. Oncocytoma


I a. Atrial myxoma ( NBE Pattern 2014-15) Ref: Devita, Heilman & Rosenberg's Cancer: Principles & Practice
b. GIST of Oncology, 8th ed.,ch. 40 - Cancers of the Genitourinary System,
c. Pulmonary chordoma Section 3: Cancer of the Kidney,
d. Paraganglioma
102 . Triad of renal cell carcinoma includes all except?
WJM Ref: Harrison's 18th eci., ch. 240 , Harrison 19th p 2266 , 2314 a . Hematuria ( NBE Pattern 2014-15)
Hfinl 96. Incorrect about LAMB syndrome: b. Abdominal pain
a. Lentigines ( NBE Pattern 2014 -15 ) c. Abdominal mass d . Fever
b. Atrial Myxoma Ref Harrison' s 17th ed., p. 592 , Harrison 19 th p 578
c. Myaesthenic syndrome
d. Blue Nevi 103 . Papillary carcinoma of kidney is associated with de ¬
Ref: Harrison’s 18th ed., ch. 240 , Harrison 19th p 2265t fect on : ( NBE Pattern 2014 -15)
a . Chromosome 3 b. Chromosome 7
97 . Most common site of atrial myxoma is: c. Chromosome 11 d. Chromosome 21
-
a. Inter atrial septum ( NBEPattern 2014-15) Ref: Harrison's 18th ed., p. 793, Harrison 19 th p 578
b. Right atrium
104. The most common presenting feature of bladder car ¬
-
c. Inter ventricular septum
cinoma is: ( NBE Pattern 2014-15)
d . Left Ventricle
a. Painless profuse intermittent haematuria
Ref Harrison' s 18th ed., ch. 240 , Harrison 19 th p 289e b. Clot colic .
c Dysuria
98. Ectopic ACTH syndrome is seen most commonly with: d . Sterile acid pyuria
V)
U a. Renal cell carcinoma ( NBE Pattern 2014 -15 ) Ref Harrison's 18th ed., p.791, Harrison 19 th p 576
3 b. Lymphoma 105. Most common benign solid tumor of kidney is:
O c. Bronchogenic carcinoma
s3 d . Pituitary adenoma
Ref: Harrison's 18th ed., p. 2945 Harrison 19 th p 511 / 609
a. Oncocytoma
b. Bellini tumor
c. Chromophobe adenoma
( NBE Pattern 2014-15)

H 99. Which one of the following statements is true of sys¬ d. Papillary carcinoma
temic treatment of NSCLC? ( NBE Pattern 2014-15) Ref Devita, Heilman & Rosenberg' s Cancer: Principles & Practice
a. Chemotherapy has no effect on survival in advanced of Oncology, 8th ed., ch. 40 - Cancers of the Genitourinary System,
Section 3: Cancer of the Kidney, Harrison 19 th p 578
NSCLC
b. Adjuvant platinum-based chemotherapy confers no 106 . Interleukin involved in pathogenesis of Stauffer syn¬
survival benefit drome? ( NBE Pattern 2014 -15)
c. Epidermal Growth Factor Receptor Tyrosine Kinase a. IL-1
Inhibitors (EGFR-TKI) such as Erlotinib have proven b. IL 2 -
active in previously treated patients with advanced c. IL-6 -
d. IL 11
NSCLC Ref: Practical Urology: Essential Principles and Practice: Essential
d. EGFR-TKI have a higher chance of response in male Principles By Christopher R Chappie,William D. Steers: page 234
smokers
Ref Harrison 19 th p 516 - 17
107. Hypoglycemia is seen with : ( NBE Pattern 2014 15) -
a. b. HCC

I
RCC
100. Most aggressive kidney tumour is? c. Glucagonoma d. Breast cancer
a. Bellini tumor ( NBE Pattern 2014 -15) Ref: Harrison' s 18th ed., ch. 100 ,

I
b. Clear cell tumor 108. Isaac syndrome is characterised by?
c. Chromophobe adenoma d . Papillary carcinoma ( NBE Pattern 2014 15) -
Ref: Harrison' s 18th ed., p. 592 a. Peripheral nerve excitability
101. Most common type of renal cell cancer in dialysis pa ¬ b. Opsoclonus
( NBE Pattern 2014 -15) c. Encephalomyelitis

I tient is?
a. Clear cell carcinoma b. Papillary carcinoma d. Limbic encephalitis
Ref: Harrison's 18th ed., ch. 101 , Harrison 19th p 619

Ans. 95. a . Atrial myxoma -


96. c. Myaesthenic syndrome 97. a. Inter atrial septum 98. c . Bronchogenic carcinoma
102. d . Fever
99. c. Epidermal Gr... 100. a . Bellini tumor 101 . b. Papillary carcinoma
o 250
103. b. Chromosome 7 104. a. Painless profuse ... 105. a. Oncocytoma 106 . c. IL-6
107. b. HCC 108. a. Peripheral nerve excitability
1.

2.
C.HkP VtR
13
FISH" Technique
'

in genetics is used for: ( NBE Pattern )


a. Detection of microdeletion
b . Identification of unknown chromosomal material
c. Interphase analysis
d. All the above
All are X-linked recessive disorders except:
a. Hemophilia-A
b. G 6PD deficiency
( NBE Pattern )
— —
8.

9.
a. Cystic fibrosis
b. Phenylketonuria
c. Alpha-1 anti-trypsin deficiency
d. Familial hypercholesterolemia
Genetics

Which of the following is an autosomal dominant


disorder?

( NBE Pattern )

Ref Harrison 19th p 436 t, 437 - 438


Features of Laurence- Moon - Biedl syndrome include:
( NBE Pattern )

c. Muscular dystrophies
d. Incontinenta pigmenti a. Hypogonadism b. Obesity
c. Polydactyly d . All the Above
3. True regarding autosomal dominant disorder:
( NBE Pattern ) Ref: Harrison 19th p 415e-5 t
a. Common in males
b. 50% chances of offsprings getting the defect 10. Angiokeratoma is seen in : ( NBE Pattern )
c. Common in females a. Fabry's disease
d . 50% chances of being a carrier b. Pompe's disease
Ref: Harrison 19 th p 436 t c. Hurler's disease
d . Gaucher's disease
4. Genetic counselling is a useful tool in: ( NBE Pattern )
Ref: H 18th
pg 2349, 3191 Harrison 19 th
p 364 t , 365, 432e - l
a. HIV b. Diabetes mellitus
c. Sickle cell disease d . Hernia 11. Kinky hair disease is a disorder where an affected
Ref: Harrison 19 th p 2242 child has peculiar white stubby hair, does not grow,
brain degenration is seen and dies by age of two years.
5. In Turner's syndrome, all the following are true Mrs A is hesitant about having children because her
except: ( NBE Pattern )
two sisters had sons who had died from kinky hair
a. Amenorrhea
disease. Her mother's brother also died of the same
b. Mental retardation
condition. Which of the following is the possible mode
c. Short stature
of inheritance in her family? (AI 2004 )
d. Coarctation of aorta
a. X-linked recessive
Ref Harrison 19 th p 2351
b. X-linked dominant
6. RNA is extracted by which procedures: ( NBE Pattern ) c. Autosomal recessive
a. Northern blot b. Southern blot d. Autosomal dominant
c. Eastern blot d . Western blot Ref: Harrison's 18/ e p501, Harrison 19 th p 377 e-2
Ref: Kumar and Clark pg 168
12. A 35 year old female with a previous history of birth
7. Not a X- linked inheritance: ( NBE Pattern ) of a child with Down's syndrome, presents with 18
a. Fabry's disease weeks Amenorrhoea . The Investigation of choice rule
b. G 6 PD deficiency out Down's syndrome in the present pregnancy is:
c. Myotonic dystrophy a. Triple test (AI 2000 )
d. Hemophilia- B b. Amniocentesis
Ref Kumar and Clark pg 178 Harrison 19 th p 462e - 9 c. Cordocentesis
d . Chorionic villous biopsy
Ref Harrison's 18/ e p511, 521, Harrison 19 th p 2600

Ans . 1 . d . All the above 2. d . Incontinenta pigmenti 3 . b. 50% chances of offsp... 4. c . Sickle cell disease
5 . b . Mental retardation 6 . a . Northern blot
9 . d . All the Above
7 . c. Myotonic dystrophy 8. c. Alpha -1 anti -trypsin ..
10 . a . Fabry ’s disease 11 . a . X -linked recessive 12. b . Amniocentesis
I
Marwah 's Internal Medicine MCQs ( Based on Harrison's 19 th
)

13. All of the following may occur in Noonan's syndrome 20. Most common mitochondrial inheritance disease?
except: ( A! 2003) a . MELAS ( NBE Pattern 2014-15)
a. Hypertrophic cardiomyopathy b. CPEO
b. Cryptoorchidism c. Lebers hereditary optic neuropathy
c. Infertility in females d . Myoclonic epilepsy and ragged red fibers
d. Autosomal dominant transmission Ref : Harrison 's 18th edn, ch ed.18, Harrison 19th p 85e 4 -
Ref: Nelson 16 / e pi 756 ; Harrison 19th p 436 t/ 1654 21. True statements regarding the mitochondrial genes
14. In Klippel - Feil syndrome, the patient has all of the are: ( NBE Pattern 2014 -15)
following clinical features except: (AI 2005) a. Paternal transmission
a. Low hair line. b. Maternal transmission
b. Bilateral Neck webbing. c. Mendelian inheritance
c. Bilateral shortness of sterno mastoid muscles. d. Mitochondrial myopathy
d. Gross limitations of neck movements. Ref : Harrisons-18th ed, pg:182, Harrison 19th p 437, 85e
Ref: OP Ghai 6/ e p599, Harrison 19th p 1651 22. All of the following are chromosomal breakage
syndromes except: ( NBE Pattern 2014-15)
15. Screening by using maternal serum alpha fetoproteins b. Ehler - Danlos syndrome
a . Fanconi's anemia
helps to detect all of the following except: ( AI 2004 )
c. Bloom's syndrome d. Ataxia telangiectasia
a. Neural tube defects
Ref : ( Robbins 7th ed. pg 155-6, Table 5.5174,
b. Duodenal Artesia
c. Talipes equinovarus Harrison 19th p 2510
d. Omphalocele Ref: Dutta 7/ e pl 06 23. The mother has sickle cell disease; Father is normal;

Gnetics
Chances of children having sickle cell disease and
16. In gene therapy which is used as vehicle?
sickle cell trait respectively are: ( NBE Pattern 2014 -15)
a . E.coli ( Rajasthan 2009 )
b. 25 and 25%
a. 0 and 100%
b. Corynebacterium
c. 50 and 50% d . 10 and 50%
c. Adenovirus
Ref : Robbins- 9th ed., ch. 5, pg:141 , Harrison 19th p 438
d. Pneumococcus
Ref: Harrison's 18/ e p548, 1502, Harrison 19th p 91e-lt 24. The genetics involved in Down syndrome is:
a . Mitotic non-disjunction ( NBE Pattern 2014 -15)
17. Which of the following is autosomal dominant b. Meiotic non - disjunction
disease? ( Maharashtra 2011 ) c. Mosacism d. Monosomy
a. Wilson's disease
b. Hemochromatosis
Ref : Robbins: 9th ed, ch 5, pg, 161, Harrison 19th p 83e l -
c. MEN 2 25. Most common type of MODY is associated with defect
on ( NBE Pattern 2014 -15)
d. All of the above
a. Chromosome 12
Ref: H 18/ e p3162, 3075, 3188, 17/ e p2362, Harrison 19th p 437 b. Chromosome 13
18. Which of the following is /are an example / examples c. Chromosome 17 d. Chromosome 22
of non- Mendelian inheritance?
a. Genomic imprinting
( DP PGMEE 2009 ) Ref : Harrison 18th edn ch 61, Harrison 19th p 441, Table 82 5 -
26. All are true about Mitochondrial DNA ( mt DNA) -
b. Uniparental disomy
c. Mitochondrial inheritance except: ( NBE Pattern 2014 15) -
d. All of the above a. Maternal inheritance
Ref: Harrison's 18 / e p499, 501, 17/ e p398- 400, b. Heteroplasmy
c. Leber hereditary optic neuropathy is the prototype
19. Two siblings with Osteogenesis Imperfecta have d. Nemaline myopathy results due to mutations in mt -
normal parents. The mode of inheritance is explained
by which of the following? ( NBE Pattern 2014 -15) DNA
a. Anticipation Ref : Harrisons 17th ed. pg 2688, Robbins 8th 171,
b. Genomic imprinting Harrison 19th p 451, 85e
c. Germ-line Mosaicism
d. New mutation
Ref : Robbins 8th ed. P. 173, Harrison 19th p 439

Ans . 13. c. Infertility in females 14. c. Bilateral shortness of ster... 15 . c. Talipes equinovarus 16 . c. Adenovirus
17 . c . MEN 2 18 . d . All of the above 19 . c. Germ - line Mosaicism 20 . a . MELAS
252 21 . b. Maternal transm ... 22. b. Ehler- Danlos syndrome 23. a. 0 and 100% -
24. b. Meiotic non disjunction
25 . a . Chromosome 12 26 . d . Nemaline myopathy results due to mutations in mt - DNA
Genetics

27. The chances of having an unaffected baby, when both 34. Interaction of gene with environment is known as
parents have achondroplasia, are: ( NBE Pattern 2014-15) a. Epistasis ( NBE Pattern 2014-15)
a. 0% b. 25% b. Phenocopy
c. 50% d. 100% c. Transmission Dys-equilibrium effect
Ref : Harrison' s 17th ed, pg. 375, Harrison 19th p 437 d. Polymorphism
Ref : Nelson 18th edn, ch 82
28. All of the following are characterized by 'trinucleotide
repeats' affecting the non - coding regions except 35. The following diseases have defect in DNA repair
a. Friedreich's ataxia ( NBE Pattern 2014 -15) mechanism except for: ( NBE Pattern 2014 -15)
b. Fragile X syndrome a. Xeroderma Pigmentosum
c. Huntington's disease b. Fanconi syndrome
d. Myotonic dystrophy c. Huntington's disease
Ref : Robbins: 9th ed. ch., 5, pg 168, Harrison 19th p 434, 444e- 9 d. Hereditary non polyposis colon cancer
Ref : Harrisons-18th ed., / Ch. 61, pg:495&503,
29. The gene that regulates normal morphogenesis
during development is: ( NBE Pattern 2014-15)
Harrison 19th p 2621 22 -
a. FMR-1 gene 36. Most common sex chromosomal aneuploidy in males?
b. Homeobox gene a XX males ( NBE Pattern 2014-15)
c. P-16 b Praderwili syndrome
d. PTEN Ref: Robbins- 9ed., pg: 456 c Klinefelter syndrome
d Fragile X syndrome

Gentics
30. Father has a blood group B; Mother has AB; Children
are not likely to have the following blood group: Ref : Nelson 18th edn, ch 584, Harrison 19th p 2350
a. 0 ( NBE Pattern 2014-15) 37. Gene for folate carrier protein is located on?
b. A a. Chromosome 10 ( NBE Pattern 2014-15)
c. B b. Chromosomes 5
d. AB Ref:Harrisons-18th ed., ch 61, pg: 500 c. Chromosome 21
31. Incorrectly matched is: ( NBE Pattern 2014-15) d. Chromosome 9
a. SCA type 3: CAG repeats Ref : Harrison 17th ed. pg. 644, 385; Nelson 19th edn
b. Myotonic dystrophy: CTG 38. The approximate number of genes contained in the
c. Fragile X syndrome:CGG human genome is: ( NBE Pattern 2014 -15)
d. Frederich ataxia: GAC a. 20,000 b. 25,000
Ref : Nelson 18th ed., ch. 80 and table 373-2 Harrison's 18th ed., c. 40,000 d. 1,00,000
Harrison 19th p 451e-2t .
Ref : Nelson 18th ed., ch , 79 and Robbins- 9ed/ pg : 1
32 . Karyotyping is done for: ( NBE Pattern 2014-15) 39. In Prader Willi syndrome, which of the following
a. Chromosomal disorders leads to obesity? ( NBE Pattern 2014-15)
b. Autosomal recessive disorders a. Neuropeptide Y increase
c. Autosomal dominant disorders b. Neuropeptide Y decrease
d. Linkage disorders c. Ghrelin decrease
( Ref: Robbin's 9th ed, pg, 158 ch, 5 d. Ghrelin increase
33 . An albino girl gets married to a normal boy, what are Ref : Robbins &cotran: 9th, ch. 9, pg. 446, Nelson 19th Ch-74
the chances of their having an affected child and what 40. Thalassemia occurs due to which mutation ?
are the chances of their children being carriers? a. Missense ( NBE Pattern 2014-15)
a. None affected, all carriers ( NBE Pattern 2014-15) b. Splicing
b. All normal c. 50% carriers c. Transition
d. 50% affected, 50% carriers
d. Frame-shift
( Ref: Harrison 17th Ed. P 2332 Ref:Robbins 9th ed., ch 5,pg, 140,
e. Truncation

Ans. 27. b. 25% 28. c. Huntington ’s disease 29. b . Homeobox gene 30. a . O
31. d . Frederich ataxia:.. 32. a. Chromosomal disor.. 33. a. None affected , all... 34. a. Epistasis
35. c. Huntington ’s d... 36. c. Klinefelter syndrome 38. b. 25,000
37. c. Chromosome 21 253
39. d. Ghrelin increase 40. d . Frame-shift

'
I
F

Marwah 's Internal Medicine MCQs ( Based on Harrison's 19 th )

41. In Marfan’s syndrome there is defect in protein 48. Which of the following is an autosomal recessive
a. Collagen ( NBE Pattern 2014 - 15) condition? ( NBE Pattern 2014 -15)
b. Elastin a. Ataxia telangectasia
c. Fibrillin d . All of the above b. Peutzjeghers syndrome
l Robbins: 9th ed. ch., 5, pg 144, Harrison 19 th p 2513 c. Neurofibromatosis
42 . Maternal disomy of chromosome 15 is seen in d Tuberous sclerosis
a . Prader -Willi syndrome Ref : Robbins Basic pathology 8th Ed. chap -6,
( NBE Pattern 2014-15)
b. Klinefelter's syndrome Harrison 19 th p 2109
c. Angelman syndrome 49. Which of the following is not associated with Down’s
d. Turner's syndrome syndrome? ( NBE Pattern 2014-15)
Ref : Robbins: 9th ed, ch.5, pg. 172, Nelson 19th p Table 76-13, page 409 a. Trisomy 21
43. b. Mosaic 21
Dominant negative inheritance is seen in :
a. EhlerDanlos syndrome c. Translocation t (14,21), t ( 21, 21)
( NBE Pattern 2014-15)
d . Deletion of 21
b. Marfan's syndrome
Ref : Nelson 18th edn, ch 81, Nelson p 400-401
c. Hunter syndrome
d. Osteogenesis imperfecta 50. Increasing severity of mental retardation in male
Ref : Robbins 7th ed. pg. 151, Harrison 19th p 432e-2t members over generations is a result of:
( NBE Pattern 2014-15)
44. Gene therapy is used for: ( NBE Pattern 2014-15) a. Mitochondrial DNA mutation

Gentics
a . Cystic fibrosis b. Sickle cell anemia b. Frameshift mutation
c. Thalassemia d. All of the above c. Y linked disorder
Ref: Harrison:18th, 547-551 d . Trinucleotide repeat mutation
45. NARP syndrome is seen in : ( NBE Pattern 2014 -15) Ref : Robbins- 9th ed.,ch 5, pg:168, Nelson p 391
a. Mitochondrial diseases 51. True statements about a- 1 anti - trypsin deficiency are
b . Glycogen storage diseases all except: ( NBE Pattern 2014-15)
c. Lysosomal storage diseases a . Autosomal dominant disease
d . Lipid storage diseases b. Emphysema
c. Fibrosis of portal tract
Ref : Harrisons-18th ed. chap: 61/ P:501, Harrison 19 th p 85e-6
d. Diastase resistant positive hepatocytes
46. Which of the following is an autosomal dominant Ref: Robbins - 9th ed/ Pg:850, Harrison 19 th p 367e-2
metabolic disorder? ( NBE Pattern 2014 -15)
52 . Catastrophic variant of EhlerDanlos syndrome is:
a. Cystic fibrosis a. I ( NBE Pattern 2014-15)
b. Phenylketonuria b. II
c. a-1 antitrypsin deficiency c. Ill
d. Familial hypercholesterolemia d . IV
Ref : Robbins Basic pathology 8th ed., ch. 7, Ref : Robbins : 9th ed., ch. 5, pg 145, Harrison 19 th p 2510
Harrison 19th p 2440 53. Y- chromosome is? ( NBE Pattern 2014-15)
47. Males are more commonly affected than females in a. Telocentric b. Metacentric
which of the following genetic disorders? c. Sub-metacentric d. Acrocentric
a . Autosomal Recessive Disorder ( NBEPattern 2014-15) Ref : Nelson 17th ed., pg 382
b. Autosomal Dominant Disorder 54. BRCA1 gene is located on? ( NBE Pattern 2014-15)
c. X-linked Recessive Disorder a . Chromosome 13 b. Chromosome 11
d. X- linked Dominant Disorder c. Chromosome 17 d . Chromosome 22
Ref : Robbins 9th ed. pg: 1054, Harrison 19 th p 523
Ref : Robbins: 9 th ed, pg . 169

Ans . 41. c. Fibrillin 42. a . Prader-Willi syndrome 43. c. Hunter syndrome 44. d . All of the above
45. a. Mitochondrial... 46. d . Familial hypercholeste... 47. c. X-linked Recessive... 48. a. Ataxia telangectasia
49. d . Deletion of 21 50. d . Trinucleotide repeat... 51. a . Autosomal dominant... 52. d. IV
254 53. d . Acrocentric 54. c. Chromosome 17
”,

Infection
CHAPTER
14
amon gst the follo wing is a caus e of massive
7. Which
Most Recent Q's 2014- 15 splenomegal y? ( DNB Pattern 2014 )
a. Multiple sclerosis
1. Which of the following statements is true regarding b. Kala c. Thalassaemia
H1N1 ? (AllMS May 2015 ) Infectious mononucleosis
d.
a. Pregnant woman with sore throat can be started Ref: Harrison 19th p 245 e -2
immediately on oseltamivir without diagnostic infec tion does the rash first
8. In which of the Rick ettsi al
testing under category B
b. People on long term steroids can't receive appears on extremities then to trunk?
n typhu s ( DNB Patte rn 2014 )
Oseltamivir a. India
b. Epidemic typhus
c. Category B concerns with immunosuppressed
patients c. Endemic typhus
d. Rocky mountain spotted fever
d. Category B patients have to undergo immediate
testing Ref: Harrison 19th p 782
1211 , 1213, Harri son 19th editi on and issues in
Ref: Page
Viral Infections
pediatric health and medicine ch 43 p 3027
2. Which among the following carries the highest risk of 9. Characteristic feature of measles: (Manipal)
transmission of HIV? (NBE Pattern 2014)
a. koplik's spot
a. Blood transfusion b. SSPE
b. Needle stick injury C. Rashes
c. Sexual contact D. Photophobia Ref Harrison 19th p 1295
d. Oral sex Ref: Harrison 19 th p 1275t
10. SSPE is a complication of: (Manipal )
3. Most common organism implicated in Spontaneous a. Measles
Bacterial Peritonitis in adults is: (NBE Pattern 2014) b. Mumps
a. E. Coli c. Rubella
b. Staph, aureus d.
c. Strep, viridians
Varicella Ref: Harrison 19th p 1295
d. Enterococcus Ref Harrison 19th p 2065 -2066 11. A young male with history of diarrhea for month and
fever, was suspected to have HIV. Blood tests positive
4. Which HIV subtype is most common in India ? for HIV. A diagnosis of AIDS will be made by the
a. A ( DNB Pattern 2014)
presence of which of the following? (Jipmer 2014)
b. B a. Oral candidiasis
c. C
b. Pulmonary tuberculosis
d. M Ref: Harrison 19th p 1260 c. Lyme's disease
5. Best confirmatory serological test for syphilis is? d. Acute watery diarrhea with severe dehydration
a. VDRL ( DNB Pattern 2014 )
Ref : http:/ / www.cdc.gov/ mmwr / preview/ mmwrhtml/ rr5710a2.
b. TPPA htm Harrison 19th p 1102
c. TPHA
d. FTA -ABS Ref Harrison 19 th p 1134 12. Most common pulmonary manifestation in AIDS:
a. TB (NBE Pattern 2014-15)
6. Vaccine against capsulated organism given how many b. Pneumonia
weeks before splenectomy? ( DNB Pattern 2014 )
c. Bronchiectasis
a. 1 week b. 2 week
d. Mycobacterial avium intercellulare
c. 4 week d. 6 week
Ref: Harr ison's 18 th
ed. ch. 189, Harrison 19th p 1250
Ref: Harrison 19th p 412
Ans. 1. a. Pregnant woman... 2. a . Blood transfusion 3 . a . E . Coli 4. c. C
5. d. FTA- ABS 6. b. 2 week 7. b . Kala 8. d . Rocky mountain spotted...
9. a . koplik ’ s spot 10. a. Measles 11. b. Pulmonary tuberculos.. 12. b. Pneumonia
Marwah's Internal Medicine MCQs ( Based on Harrison’s 19th)

13. Kaposi sarcoma is commonly seen in: b. Pneumonia c. Weight loss


a. Upper limbs (NBE Pattern 2014-15) d. Myelopathy
b. Lower limbs
c. Head and neck
Ref: Harrison's 18th ed. Table 189-9, Harrison 19th p 1249
|
d. Trunk 20. HIV RNA by PCR can detect as low as:
. (NBE Pattern 2014-15 )
Ref: Harrison's 18th ed ch. 189, Harrison 19th p 1268
a. 30 copies viral RNA /ml of blood
HB 14. Side- effect of zidovidine in AIDS patient is:
b. 40 copies viral RNA /ml of blood
a. Megaloblastic anemia (NBE Pattern 2014-15) c. 50 copies of viral RNA / ml of blood
b. Hyperuricaemia
d. 60 copies of viral RNA / ml of blood
c. Nephrocalcinosis
d. Pancreatitis
Ref: Harrison's 18th ed. Table 189-7, Harrison 19th p 1248
21. Most common cause of pleural effusion in AIDS
Ref: CMDT-14/ ch. 31/ p.l 341, Harrison 19th p 1273 (NBE Pattern 2014-15)
patients?
15. Consider the following statements regarding HIV: a. Kaposi Sarcoma
(NBE Pattern 2014-15) b. TB
1. The risk of infection is 10 times more with solid c. Pneumocystis Jiroveci
needle than hollow needle. d. mycoplasma
2. The mode of transmission is through infected blood
3. CD 4 cell amount is directly related to viral load
Ref: Harrison's 18th ed. ch. 189, Harrison 19th p 1269
4. Post exposure prophylaxis should start within one 22. Most common cause of death in measles?
hour of contamination a. Encephalitis (NBE Pattern 2014-15)

Infection
Which of these statements are correct? b. Meningitis
a. 1& 2 b. 2 only c. Dehydration
c. 3&4 d. 2,3 & 4 d. Pneumonia
Ref Harrison's-18th ed / p:1515, Harrison 19th p 1222 Ref: Harrison's 18th ed. ch. 192/ p. 1601
16. All are seen with Pneumocystis carini in AIDS except: 23. Most common extra - cutaneous manifestation of
a. Pneumonia (NBE Pattern 2014-15) chicken pox is: (NBE Pattern 2014-15)
b. Otic polypoid mass a. CNS involvement
c. Ophthalmic choroid lesion b. Varicella pneumonia
d. Meningitis c. Congenital varicella

Ref Harrison's 18th ed. ch. 189, Harrison 19th p 1253


d. Reye syndrome Ref: Harrison's 18th ed. ch. 180
24. EB virus is associated with all except:
17. Most common cause of diarrhea in AIDS patients:
(NBE Pattern 2014-15)
a. Salmonella typhimurium (NBE Pattern 2014-15)
b. Hepatitis
a. Bell's palsy
b. Cryptosporidium c. Guillain-Barre syndrome
c. Candida d. Laennec's cirrhosis
d. Isophora
Ref CMDT-14 ch. 32 p. 1362, Harrison 19th p 1188
Ref Harrison's 18th ed. ch. 189, Harrison 19th p 1257
25. Skeleton changes in syphilis are all except?
18. Incorrect about HIV associated nephropathy? a. Saber shin (NBE Pattern 2014-15)
a. Proteinuria (NBE Pattern 2014-15) b. Olympian brow c. Scaphoid scapula
b. Shrunken kidneys d. Thickening of acromio - clavicular joint
c. 15% cases show mesangial proliferation
Ref: Harrison's-18ed., / p: 1384 and ch. 215, Nelson 18th ed.
d. Develops when CD4 < 200
26. Volcano ulcers in esophagus are seen in:
Ref Harrison's 18 th ed. ch. 189, Harrison 19th p 1259
a. Herpetic esophagitis (NBE Pattern 2014-15)
19. All are seen in acute HIV syndrome except: b. Candida esophagitis
a. Diarrhoea (NBE Pattern 2014-15) c. Apthous ulcer in crohn
d. HIV esophagitis
Ref : Mayo Board review -5th ed. by Stephen C. Hauser
Ans. 13. b. Lower limbs 14. a. Megaloblastic anemia 15. b. 2 only 16. d. Meningitis
17. b. Cryptosporidium 18. b. Shrunken kidneys 19. b. Pneumonia 20. b. 40 copies viral RNA/ml of...
21. a . Kaposi Sarcoma 22. d. Pneumonia 23. a. CNS involvement 24. d. Laennec ’s cirrhosis
256 25. d . Thickening of... 26. a . Herpetic esophagitis
Infection

27. Most common cause of death in EBV? b. Resistance of penicillins


a . Splenic abscess ( NBE Pattern 2014-15) c. Infection with chlamydia
d. Ureaplasma urealyticum infection
b. Meningo-encephalitis
Ref: Harrison 19 th p 1003
c. Fulminant hepatitis
d. Auto- immune haemolytic anemia 34. Typhoid is treated by all except? ( NEETPattern 2015-16 )
a. Erythromycin b. Ceftriaxone
Ref: Harrison's 18th ed. ch. 181, Harrison 19 th p 1188
c. Amikacin d. Ciprofloxacin
28. Pontiac fever is caused by: [NBE Pattern 2014-15) Ref: Harrison's 18th edn and WHO guidelines, ch 153,
a. Marbug virus b. Legionella Harrison 19th p 1052, Table 190 - 1
c. Tuberculosis bacilli d . Sindbis virus 35. Drug treatment is given for how many days in
Ref: Harrison’s-18e/ ch:147/ p:1236, Harrison 19 th p 1015 pneumococcal meningitis? ( NEET Pattern 2015-16 )
a. 5 days b. 7 days
Bacterial Infection c. 14 days d. 21 days
Ref: Nelson 18th edn , Ch 602, Harrison 19th p 889
29. Relative Bradycardia is seen in all except: ( Manipal)
a. Malaria
36. Most common presentation of extra - pulmonary TB :
a. Tubercular lymphadenitis ( NBE Pattern 2014 -15)
b. Brucellosis
b. Peritoneal TB
c. Malignant lymphoma
c. Pericardial TB
d. Bacterial pneumonias Ref: Harrison 19 th p 1066
d. Tubercular meningitis
In the management of enteric fever, all are done .

Infectio
30. Ref: Harrison's 18th ed. ch 165, Harrison 19th p 1109
except: ( Manipal )
a . Oral vaccine can be given along with antibiotics 37. Site of Gohn focus in congenital TB: (NBEPattern 2014-15)
b. Ciprofloxacin a . Lung b. Liver
c. Monocef c. GIT d. Brain
d. Typhim VI Cannot be used in carrier Ref Nelson 18th ed. ch. 212
Ref: Harrison 19 th p 1052t 38. Pathognomonic of miliary TB: ( NBE Pattern 2014-15)
31. All are true about meningococcal meningitis except: a. Snow storm appearance in CXR
a. DIC ( Manipal ) b. Choroid tubercles on fundus examination
b. Absence of meningeal signs indicates poor prognosis c. Pericardial TB
c. Meningococcemia is associated with petechial d. Peritoneal TB
rashes .
Ref: Harrison's 18th ed ch. 165, Harrison 19 th p 1112
d. Sensitivity to light (photophobia)
39. The most common cause of seizures in a patient of
Ref: Harrison 19th p 995 AIDS is: ( NBE Pattern 2014-15)
32 . ‘Mitsuda Reaction' indicates: ( Manipal ) a. Toxoplasmosis
a. Patient has leprosy b. Cryptococcal meningitis
b. Patient doesn't have leprosy c. Progressive multifocal leucoencephalopathy
c. CMI is intact d. CNS lymphoma
d. Patient had leprosy in the past
Ref: Harrison's 18th ed. ch. 189/ p:1560, Harrison 19 th p 1265
Ref: Ananthanarayan pg 347
40. Incorrect about Tuberculous meningitis:
33 . A 16 - year- boy had urethral discharge 1 week after
having unprotected intercourse. Gram staining of a. Normal CSF sugar ( NBE Pattern 2014 -15)
the discharge reveals numerous neutrophils with few b. Cerebral infarction
intracellular gram negative diplococci. He was treated c. Communicating hydrocephalus
with 250 mg intramuscular injection of ceftriaxone. 7 d . Tuberculoma resolves after treatment
days later, he presents with same complaints. What is Ref: Nelson 18th ed. ch. 212, Harrison 19 th p 1111
the most probable cause? (Jipmer 2014 )
a . Reinfection with gonorrhea

Ans . 27. -
b. Meningo encep... 28. b. Legionella
b. Absence of .. .
32. c CMI is intact
29. b. Brucellosis 30. a. Oral vaccine can be...
31. 33. a. Reinfection with g... 34. a. Erythromycin
35. c. 14 day 36. a. Tubercular lymphadenitis 37. b. Liver 38. b. Choroid tubercles on .... 257
39. a. Toxoplasmosis 40. d . Tuberculoma resolves after treatment
th )
Marwah s Internal Medicine
' MCQs ( Based on Harrison 's 19

41. Gene responsible for resistance to rifampicin: 48. Latex agglutination test in CSF is done for detection
a. RpoBgene ( NBE Pattern 2014 -15) of: ( NBE Pattern 2014-15)
b. Kat G gene a . Cryptococcus
b. E.Coli
BJHH c. Rpm B gene
d . Emb B gene Ref: icmr.nic.in/ ijmr/ 2003/ 0204. pdf
c. Tuberculosis
d . Coxsackie
42. All are indications for stopping offending ATT drug Ref: CMDT-14 ch. 30 p. 1287, Harrison 19 th p 1339
jlllll permanently except? ( NBE Pattern 2014-15)
49. Which among the following statements is true about
a. Gout toxoplasma: ( NBE Pattern 2014 -15)
b. Autoimmune Thrombocytopenia a . In adults it is mostly asymptomatic
c. Optic neuritis b. IgG antibodies in the newborn is confirmatory of
d . Hepatitis diagnosis
Ref: Harrison' s 18th ed. ch. 165, Harrison 19 th p 1117 c. Toxoplasmic encephalitis can occur in
immunocompetent
j 43. Interferon gamma release assay measures IFN release
d . Most infections are anthropozoonotic
against which M . TB antigen: ( NBE Pattern 2014 -15)
a. ESAT 6 b. ESAT 7 Ref: Harrison' s-18th ed./ ch:214/ p:1725, Harrison 19th p 1401
c. CF 11 d. CF 12 50. There is no correlation between X- ray appearance
Ref: Harrison's p. 1015, Harrison’s 17th ed., Harrison 19th p 1115 and clinical state of the patient in, pneumonia:
a. Mycoplasma ( NBE Pattern 2014 -15)
I 44. MDR TB must be treated for at least:
b. Freidlanders
a . 12 months ( NBE Pattern 2014-15)

Ifection
c. Pneumococcal
b. 18months d. Staphylococcal
c. 20 months .
Ref: Harrison's 18th ed ch. 175, Harrison 19th p 1163
d . 36 months
51. Best test for determining eradication of H . Pylori
Ref : Harrison's 18th ed. ch. 165, Harrison 19 th p 1116 infection: ( NBE Pattern 2014 -15)
45. Earliest and often the only presentation of TB kidneys a. Urease test
is: ( NBE Pattern 2014 -15) b. Breath urea test
a . Increased frequency c. Tissue biopsy
d. Serum ELISA
b. Colicky pain
d . Renal calculi
Ref: Harrison 18/ chap:151/ P:1263, Harrison 19 th p 1040
c. Hematuria
Ref: Harrison's 18th ed. ch. 165 p. 1628, Harrison 19 th p 1110 52. CLO test is used for: ( NBE Pattern 2014-15)
a. H . pylori b. Brucella
46. Relative risk of developing TB in patients already c. Gonorrhoea d . EBOLA
infected with TB bacillus is highest in: Ref: Harrisons-18e/ ch:151/ p:1263, Harrison 19 th p 1040
a . Diabetes ( NBE Pattern 2014-15)
53 . Valve affected in infective endocarditis due to septic
b. Recent infection ( NBE Pattern 2014 -15)
abortion?
c. Post transplantation a. Mitral b. Tricuspid
d. Malnutrition c. Pulmonary d. Aortic
Ref: Harrison' s 18th ed. ch. 165, Harrison 19 th p 1105 Ref: CMDT-14/ ch. 33/ p: 1435, Harrison 19 th p 819
47 . Treatment of Latent TB infection in Tuberculin
54. Bacterial endocarditis is most commonly seen in:
positive, HIV positive patients: ( NBE Pattern 2014 -15)
a. VSD ( NBE Pattern 2014 -15)
a. INH biweekly for 9 months
b. 2 ( HRZ) 3 + 4 ( HR) 3 b. PDA
c. Rifampicin biweekly for 6 months c. ASD
d. Pyrazinamide daily for 6 months d. AS
Ref: Harrison's 18th ed. ch. 165, Ref: CMDT-14/ ch:33/ p: 1435
Harrison 19 th Table 202 -3, p 1116 )

Ans. 41. a. Rpo B gene 42. d. Hepatitis 43. a. ESAT 6 44. c. 20 months
45. .
a. Increased fre. . 46. c. Post transplantation 47. a. INH biweekly for 9... 48. a. Cryptococcus
49 . a . In adults it is mos... 50. a. Mycoplasma 51. b. Breath urea test 52. a . H.pylori
258 53. b. Tricuspid 54. a. VSD
Infection

55. All are useful for management of severe Clostridium 61. Romana's sign is seen in: -
( NBE Pattern 2014 15)
difficile treatment except: ( NBE Pattern 2014-15 ) a. Toxoplasma
b. Trypanosoma cruzi
a. intravenous Metronidazole
c. Loa loa
b. Neomycin enema
d. Wuchereria
c. Fecal transplant
d. Tigecycline Ref Harrison's-18th ed/ ch.:213/ p:1717, Harrison 19th p 1395

Ref: Harrison's 18th ed. Harrison 19th p 860, Table 161 -2 62 . Incorrect about prosthetic valve endocarditis is:
a. Embolism ( NBE Pattern 2014-15)
56. Which of the following is not associated with
b. Mitral valve mostly involved
Streptococcus? ( NBE Pattern 2014-15)
c. C.O. N.S < 1 year
a. Rheumatic fever
d . Strept. Viridans > lyear
b. Scarlet fever
c. Acute GN Ref: CMDT-14/ ch.:33/ p:1435, Harrison 19th p 817
d. Scalded skin syndrome 63. Most common presentation of gonorrhoeal infections
Ref: CMDT-14/ ch. 33/ p:1419, Harrison 19th p 959 is: ( NBE Pattern 2014-15)
a . Urethritis b. Prostatitis
57 . All can be seen as Post-Diarrhoea complications
c. Orchitis d. Epididymitis
except: ( NBE Pattern 2014-15]
Ref: Harrison‘s:18e/ chap:144/ p:1503, Harrison 19th p 870
a. HUS
b. Nephritic syndrome 64. Bull neck in diphtheria is due to: ( NBEPattern 2014-15)

Infectio
c. Reactive arthritis a . Retropharyngial abscess
d. Gullian Barre syndrome b. Laryngeal oedema
-
Ref Harrison's 18th ed Table 128-2:, Harrison 19th p 855, Table 160 2 c. Cellulitis
d. Lymphadenopathy
58. Most common cause of death in diphtheria is due to? Ref: Harrison's-18th ed./ ch. 138/ p:1189, Harrison 19 th p 978
a. Airway compromise ( NBE Pattern 2014-15]
b. Toxic cardiomyopathy
65. DOC for treatment of SSPE: ( NBE Pattern 2014- 15)
a. Abacavir
c. Sepsis
b. Isoprinosine
d. Descending polyneuropathy
c. Glatiramer
Ref Harrison' s-18th ed., ch. 138, Harrison 19th p 978
d. Interferon
59. Among the toxins produced by Clostridium botulinum, Ref: Nelson 18th ed. ch. 243
the most potent is: ( NBE Pattern 2014-15]
66. Purpura Fulminans is caused by all except:
a. A
b. C a. N. Meningitidis ( NBE Pattern 2014-15)
c. D b. H . Influenzae
d. F c. Strep. Pneumoniae
Ref Harrison 18th ed. ch. 142, Harrison 19th p 987 d . Staph. Aureus
Ref Harrison’s 18th ed. table 121.1, Harrison 19 th p 782
60. Changing character of a murmur in a patient with
joint pain and embolic phenomenon indicates the 67 . Which about Legionnaires pneumonia is not true:
diagnosis of: ( NBE Pattern 2014- 15)
a. Mitral stenosis ( NBE Pattern 2014-15] a. Erythromycin is the drug of choice
b. SABE b. Seen mainly in immune-suppressed patients
c. Rheumatiod arthritis c. Smoking and alcohol are risk factors
d. Aortic regurgitation d. Common in children and adults
Ref CMDT- 14/ ch:33/ p:1435, Harrison 19th p 819 Ref CMDT- 14/ ch:33/ p:1443, Harrison 19th p 1017

Ans. 55. b. Neomycin enema 56. d. Scalded skin syndrome 57. b. Nephritic syndrome 58. b. Toxic cardiomyopathy
5®*

63.
-
a A
a . Urethritis
60 . b . SABE
64. d . Lymphadenopathy
61 . b. Trypanosoma cruzi 62. b. Mitral valve mostly in...
65. b. Isoprinosine 66. d. Staph . Aureus 259
67. a . Erythromycin is the drug of choice
th )
Marwah 's Internal Medicine MC Qs ( Based on Harriso n 's 19

68. -
A 60 year- old man has developed atypical pneumonia. c. Glutamate dehydrogenase immunoassay
Which of the following statements regarding d. Sigmoidoscop
pneumonia is correct: ( NBE Pattern 2014-15) Ref Harrison’ s-18e/ ch:129/ p:1091, Harrison 19 th p 859
a. Urine test for legionella antigen is 90% sensitive 75. Treponema pallidum crosses placenta:
b. A negative blood culture would rule out the a. After 36 weeks ( NBE Pattern 2014-15)
diagnosis of infecting organism b. After 28 weeks
c. Absence of rigors excludes a diagnosis of c. After 2 nd trimester
pneumococcal pneumonia d. At any stage of pregnancy
d. Drug of choice is levofloxacin Ref Harrison’s-18e/ ch: Infectious diseases/ p:1384
Ref: Harrison 18th ed. ch. 257, Harrison 19 th p 1017 76 . Neisseria infection is characteristic of deficiency of:
69. All of the following are associated with HUS except: a. C 2 ( NBE Pattern 2014 -15)
a. Thrombocytopenia ( NBE Pattern 2014 -15) b. C 3 c. C4
d. C5 Ref Harrison's-18e/ ch:144/ p:1501
b. Oliguria
of Wh ippl e’s disease is:
c. Pain 77. Neurolo gica l man ifest atio n
d . Purpura a. Seizures ( NBE Pattern 2014 -15 )
j .
Ref: Harrison's 18th ed. ch. 276 p 2275, Harrison 19th p 1863 b. Cerebellar ataxia
The following statements are correct for Helicobacter c. Focal neurological deficits
pylori except: ( NBE Pattern 2014-15) d. Encephalopathy
a. It shows positive urease test Ref: CMDT-14/ ch:15/ p:623, Harrison 19 th p 1945
b. It is spiral gram negative flagellate
78. In giardiasis malabsorption is due to all except:
c. It can invade tissue to a great depth ( NBE Pattern 2014-15)
a. Mucosal injury
d . It is linked with duodenal ulcer b. Bacterial overgrowth
Ref: CMDT-14/ ch. 15/ p: 609, Harrison 19th p 1039 c. Lactose intolerance
Left sided endocarditis is associated with: d . Hypogammaglobulinaemia
a. Endocardial cushion defect ( NBE Pattern 2014-15) .
Ref Harrison' s 18th ed / ch 215/ p:1730, Harrison 19th p 1406

I
b. 0. Secondum ASD
79. Indications to eradicate H . pylori all except:
c. 0. Primum ASD a. Low grade B cell lymphoma ( NBE Pattern 2014 -15)
d. P.F.O b. Acute gastritis
Ref CMDT-14/ ch:33/ p:l 43 c. Duodenal ulcer
d. Family history of gastric cancer
Differential diagnosis of Botulism are all except?
a. GB syndrome ( NBE Pattern 2014-15 ) Ref Harrison’s-18e/ ch:151 / p: 1264, Harrison 19 th p 1041
b. Myasthenia gravis Protozoal infection
c. Lambert Eaton syndrome
d. Clostridial myonecrosis 80. All are true about Dengue hemorrhagic fever except?
Ref Harr ison s- /
18e part - 8/ p.120 2 , Har riso n 19 th p 988 a. Lamivudine is drug of choice ( AIIMS NOV. 2014 )
b. Malnutrition is protective
Prophylactic antibiotic therapy not given for infective c. Transmitted by Aedes
end ocar ditis are A / E: ( NBE Patt ern 2014 -15)
d . Causative agent belongs to Flaviviradae group
a. Permanent transvenous pacemaker Ref Arthropods by J . Goddard, 2ndEdn.,P 62
b. Atrial septal defect
c. Ventricle septal defect 81. Which is not a feature of cerebral malaria?
a. Focal neurologic deficit ( NBE Pattern 2014 -15)
d. Coronary artery bypass graft
b. Retinal haemorrhages
Ref CMDT-14/ ch:33/ p:1437, Harrison 19 th p 826
c. Extensor plantar reflex
"In pseudomembranous colitis" the investigation of d . Absent abdominal reflex
( NBE Pattern 2014 -15) .
choice is? Ref: Harrison's 18th ed . ch. 210 p 1692, Harrison 19th p 1372
a. Stool culture b. CT scan abdomen

Ans. 68. a. Urine test for leg... 69. .


c Pain 70. c. It can invade tissue.. . 71. d. P.F.O
74. a. Stool culture 75. d . At any stage of pregnancy
72. d. Clostridial myon ... 73. d. Coronary artery bypass
77. a. Seizures 78. a. Mucosal injury 79. b. Acute gastritis
76. d. C5
260 81. a . Focal neurologic deficit
80. a. Lamivudine is..
Infection

regar ding falcip arum mala ria are all 88. True statement about Neuro -cysticercosis is:
state ment s
82. The
except: ( NBE Pattern 2014 -15) a. Usually presents with seizures ( NBEPattern 2014 15)-
a. Haemoglobinuria and renal failure b. Albendazole is more effective than praziquantel

*
b. Hypoglycemia c. Usually presents with 6th nerve palsy and
c. Cerebral malaria hemiparesis
d . Adequately prevented with chloroquine therapy d. High dose steroid are given for hydrocephalus
Ref: CMDT-14 ch. 35 p. 1491, Harrison 19 th p 1378 -
Ref: CMDT 14/ ch. 35/ p:1515, Harrison 19 th p 1432

83 . Which of the following is not a finding in kala azar? 89. Eosinophilic meningo encephalitis is caused by:
a . Pancytopenia ( NBE Pattern 2014-15) a. Acanthamoeba -
( NBE Pattern 2014 15)
b. Oral miltefosine b. Naegleria
c. Hypopigmented macules c. Angiostrongylus
d. Tropical splenomegaly syndrome d. Toxoplasma
Ref: Harrison's 18th ed. ch. 212, Harrison 19 th p 1374 - 75 Ref H - /
18 p:1738 , Harr ison 19 th Table 165 - 2 , p 908

84. The following is true about malaria: 90. Stage III of Lyme’ s disease is characterized by:
a . P. falciparum can cause relapse (NBEPattern 2014-15) a. Meningoencephalitis ( NBE Pattern 2014-15)
-
b. P. vivax can be detected by HRP 2 Dipstick b. Myocarditis
c. P. vivax causes enlargement of affected RBC c. Arthritis
d. LDH card test quantitates the falciparum parasitemia d. Nephritis
-
Ref CMDT 14, ch -34., p:1479, Harrison 19th p 1151

Infection
Ref Harrison' s 18th ed. ch. 210 p. 1698, Harrison 19 th p 1376
91. Not major criteria for diagnosis of neurocyst- f
Helmenthic Infection ( NBE Pattern 2014 15) §-
icercosis?
a. Detection of cysticerci antigen by immunoblot
85. In AIDS patient presenting with fever, cough a
diagnosis of pneumocystis pneumonia is best b. Detection of cysticerci antigen by ELISA
established by: ( NBE Pattern 2014 - 15) c. Cystic lesion in brain parenchyma by MRI
a . CT scan chest d . Calcified lesion in brain parenchyma by CT
b . Bronchoalveolar lavage Ref: Harrison's 18th antigen ch. 220, Harrison 19 th p 1431
c. Staining of intra - nuclear inclusion with Silver
92 . A 30 - year -old man, presented with subcutaneous
staining itchy nodules over the left iliac crest. On examination,
d . Aspiration and Culture they are firm, non - tender and mobile. Skin snips
Ref Harrison's 18th ed. ch.l 89/ p:1547, Harrison 19 th p 1253 contain microfilaria and adult worms of:
86 . True about Cryptococcus Neoformans is A / E: a. Loa Loa ( NBE Pattern 2014 -15)
-
( NBE Pattern 2014 15) b. Onchocerca volvulus
a . Capsule excludes India ink particles c. Brugia malayi
b. Common in immunocompromised patient d . Mansonella perstans
c. Anti -capsular antibody prevents recurrence Ref CMDT-14/ ch:35/ p:1526, Harrison 19 th 1421
d . Strongly positive Mucicarmine stain of the organism
in tissue is useful 93. Rhinocerebral mucormycosis is present with:
Ref: CMDT-14 ch. 36 p. 1535, Harrison 19th p 1340 a. Broad spectrum antibiotic use ( NBEPattern 2014-15)
b. Pregnancy
87 . Best management after human bite: c. Diabetic ketoacidosis
a. Ampicillin plus sulbactam ( NBE Pattern 2014 -15) d. Renal tubular acidosis
b. Clindamycin plus TMP-SMX
Ref H 18th ed. p. 1030 and p. 1661, Harrison 19 th p 2429
c. Fluroquinolone
d. Doxycycline
Ref Harrison's 18th ed. ch. e24 a

Ans. 82. d . Adequately preve.. 83. d. Tropical splenomegaly... 84. c. P. vivax causes ...
85. b. Bronchoalveolar lavage
86. c . Anti -capsular 87. a. Ampicillin plus sulb... 88. b. Albendazole is more... 89. c. Angiostrongylus
90. c. Arthritis .
91. b. Detection of cysticerci.. 92. b. Onchocerca volvulus 93. c. Diabetic ketoacidosis 261
Marwah's Internal Medicine MCQs ( Based on Harrison's 19th )

Fungal Infection b. Secondary syphilis


c. Tertiary syphilis
d. Quaternary syphilis
94. Drug of choice for madura mycosis is: [ DNB 2013)
Ref Kumar & Clark pg 125 Harrison 19 th p 1134
I a. Imipenem
rj Hi b. Dapsone 102. Most important drug for PERISISTORS of TB:
c. Itraconazole a. Rifampicin [ Manipal )
d. Amikacin Ref: Harrison 19th p 1349 b. Pyrazinamide
c. INH
I 95. Healing with calcification is a feature of:
d . Ethambutol Ref Harrison 19 th p 1107
a . Cryptococcosis ( NBE Pattern 2014-15)
103. Drug of choice in LEPROSY PERISISTORS: [ Manipal )
b. Mucormycosis
c. Aspergillosis a. Rifampicin
b. Dapsone
d. Histoplasmosis
c. Clofozamine
Ref: Harrison' s 18th ed. ch. 199 p. 1641, Harrison 19 th pi 333 d. None of the above Ref: Harrison 19 th p 1126
I 96. Best test for mycoplasma infection is? 104. Drug effective against pseudomons: [ Manipal )
a. PCR of respiratory secretions [NBEPattern 2014-15) a . Vancomycin b. Azithromycin
b. Gram stain of transtracheal aspirate c. Cefdroxil d . Tobramycin
c. Culture of transtracheal aspirate Ref Harrison's 15th pg 967 Harrison 19th p 1042
d. CXR
105. Drug of choice for toxoplasma infections: [ Manipal )
Ref: Harrison's 18th ed. ch. 175, Harrison 19 th p 1164

Ifection
a . Tinidazole
97 . Aspergilloma is commonly a complication of: b. Co -trimoxazole
a. TB [ NBE Pattern 2014-15) c. Penicillin
d. Ciprofloxacin Ref Harrison 19th p 1398
b. Bronchogenic carcinoma
c. Cystic fibrosis 106 . Drug which is immunosuppressive [ Manipal )
d . Wegener’s granulomatiosis a. Steroid with cyclophosphamide
Ref H Table 204 -1, Harrison' s 18th ed. and CMDT-2014 p. 1537 b. Ribavirin and Zidovudine
c. Vidarabine and Foscarnet
98. Lucio phenomenon is treated with: [NBEPattern 2014-15) d. Amphotericin - B and Nystatin
a. Steroids b. Lenalidomide
Ref Kumar & Clark pg 212 Harrison 19 th p 1828
c. Clofazimine d. Exchange transfusion
107 . Nosocomial infections are commonly acquired via:
Ref Harrison' s 18th ed. ch. 166, Harrison 19 th p 1125
a. URTI [ Manipal )
Miscellaneous b. Direct contact
c. Aspiration
[ Manipal ) d. Contaminated food and water
99. All are "HUMAN" Prion' diseases except:
Ref Harrison’ s 15th pg 854 Harrison 19 th p 973
a. Chronic Wasting Disease ( CWD)
b. Fatal familial insomnia 108. True about lung abscess: [ Manipal )
c. Creutzfeldt-Jakob disease a. Foul smelling sputum
d. Kuru Ref Harrison 19 th p 444e -8f b. Clubbing
[ Manipal ) c. High lever
100. Not a risk factor for HIV transmission :
a. Sharing of toilet seat d. All the above Ref: Harrison 19 th p 815
b. Unprotected sexual intercourse 109. In H . Infleunza meningitis most commonly associated
c. Transfusion of infected blood complication is: [ Manipal )
d. Mother to child transmission a . Otitis media and sinusitis
Ref Harrison 19 th p 1260 b. Bronchopneumonia
c. Orchitis
101. Neurological and cardiac manifestations of syphilis
[ Manipal ) d. Sub ducal effusion
are seen in:
a. Primary syphilis Ref CMDT 2001 pg 1368 Harrison 19 th p 230 / 953

Ans. 94. c. Itraconazole 95. d. Histoplasmosis 96. a. PCR of respiratory... 97. d . Wegener ’s granuloma...
98. d . Exchange trans... 99. a . Chronic Wasting Di.. 100 . a . Sharing of toilet seat 101 . c. Tertiary syphilis
262
102. a . Rifampicin 103. d . None of the above 104. d . Tobramycin -
105. b. Co trimoxazole
.
106. a . Steroid with cycl. . 107. b. Direct contact 108. d. All the above 109. a. Otitis media and sinusitis
Infection

118. All can be used in chloroquine resistant malar


ia
gono cocca l infect ion is: (Manipai )
110. DOC for resista nt
except: ( Manip ai )
a. Erythromycin a. Mefloquin
b. Benzathine pencillin .
b Primaquine
c. Ciprofloxacin c. Quinine
d. Ceftriaxone
on 19 th p 1008 d. Halofantrine
Kumar and Clark pg 123 Harris
Ref:
Ref: KB Tripat hi 5 th pp 742 - 4 Harris on 19 th p 14 - 15 , 1379
syndro me does not cause : ( Manip ai )
111. Toxic shock Immu nolog ical mucoc utaneo us syndr ome is similar
b. Fever 119.
a. Shock d by ( Manip ai )
d. Rash to infection cause :
c. Uremia
19th p 780t / 963 a. Staph aureus
Ref: Kumar & Clark pg 66 Harris on
b. Pneumococcus
112. Multi drug regimen is given in TB because: (Manipai) c. Coxsackie
a. Less toxicity d. Herpes
b. Less cost ' s 18 th pg 1456 Harris on 19 tli p 214 e -6
Ref Harrison
c . Less time of treatment
120. Infectious mono nucle osis is chara cteriz ed by all
d. Prevents resistance (Manipai)
Harris on 19th p 205 e -2t except:
Ref a. Atypical lymphocytes
113 . About leprom atous lepros y true is all excep t. ( Manip ai ) b. Increased B cells
a. Indefinite margins are there c. Absolute lymphocytosis
b. Central clearing is not present d. Splenomegaly

Infection
c. Raised edges are seen Harriso n's 18th pg 1467 Harrison 19th p 128t, 152, 1187t
Ref:
d. Sensation is lost late
121. Urine of a UT1 patient has a pH of more than 8, the
Ref Harrison 19th p 1126 probable organism is: (Manipai)
114. AIDS does not follow CDC criteria when associated a. E coli b. Proteus
With: (Manipai ) c. Klebsiella d. Candida
a. Cyrptococcal Ref: Harriso n' s 18 th pg 2349 , 3191 Harris on 19 th p 861
b. Kaposi's sarcoma
122. Artemesinin is best used for: (Manipai )
c. Oral candidiasis
d. Disseminated MAI infection a. Relapse of malaria
b. Cerebral malaria
Ref Harrison 19th p 16 e - lf c. Chronic malaria
115. In an HIV + person AIDS develops when CD4 + count is: d. Recrudescent malaria
a. <200 (Manipai)
b. 50 -100
Ref CMDT 2013, pg 1492 Harrison 19th p 246e- l -2t, 1380t
c .
500 123. A middle aged woman presents with black nasal
discharge, right globe fixation and proptosis.
d. 1000
Investigations reveal elevated blood sugar and
Ref Park 17th pg 264 Harrison 19th p 1221 positive urinary ketones. What is the appropriate
116. Most severe complication of LGV is: (Manipai) therapy for this condition? (Manipai )
a. Urethral stricture a. Amphotericin
b. Rectal stricture b. Itraconazole
c .
Inguinal swelling c. Broad spectrum antibiotic
d. Severe spiking fever d. Ketoconazole
Ref CMDT 2001 pg 1388 Harrison 19th p 1170
124. I.R.I.S is: ( NBE Pattern 2014-15)
117. Lepromatous leprosy all are true except: (Manipai )
a. Immune reconstitution idiopathic syndrome
a. Margins arc ill defined
b. Immune reconstitution immunological syndrome
b. Sensations are lost late
c. Immune reconstitution inflammatory syndrome
c .
Elevated margins
d. Asymmetric distribution d. Inflammatory reconstitution immune syndrome
Ref Park 17th pg 245 Harrison 19th p 1122 Ref Harrison’s 18th ed. ch. 165, Harrison 19th p 1261

Ans . 110 . d . Ceftriaxone 111. c. Uremia 112. d. Prevents resistance 113 . d. Sensation is lost late
114. d. Disseminated ... 115. a. <200 116 . b. Rectal stricture 117. d. Asymmetric distribution
118 . b . Primaquine 119. d. Herpes 120. b. Increased B cells 121. b. Proteus
122. b. Cerebral malaria 263
123. a. Amphotericin 124. c. Immune reconstitution inflammatory syndrome

I
I
I
Marwah's Internal Medicine

125. Interferon is not used in:


a. CML
b. Polymyositis
c. Hairy cell leukaemia
MCQs ( Based on Harrison’s 19th )

( NBE Pattern 2014- 15) 131. Correct about CURB 65 score: ( NBE Pattern 2014 -15)
a. Patient with score of 0 has 1.5% chances of dying
b. Urea < 7mmol / L
c. BP > 90 / 60 mm Hg

B
d. Chronic hepatitis c infection d. Coma
Ref: CMDT-14/ ch:20/ P:842, Harrison 19 th p 2200 Ref: Harrison 18th ed. ch. 257, Harrison 19 th p 807
126. An 8- year old patient presents with fever, sore 132 . A 15 - year - old boy presents to you with history of
throat and lymphadenopathy. On examination fever, altered sensorium and purpuric rash for two
hepatospienomegaly was found. Peripheral blood days. On examination, the patient is found stuporous.
film shows 20% atypical lymphoctytosis, the most He has BP of 90/60 mm Hg and extensive palpable
likely diagnosis is: ( NBE Pattern 2014 15)- purpura over the legs. Which of the following would
a. CLL be the most appropriate initial choice of antibiotic?
b. Infectious mononucleosis ( NBE Pattern 2014-15)
c. ALL
a. Vancomycin b. Penicillin G
d . CML
c. Ciprofloxacin d. Ceftriaxone
Ref: CMDT- 14 / ch. 32/ p. 1361, Harrison 19 th p 1187
Ref: Harrison's 18th ed. ch. 143
127 . While discharging a patient of meningitis due to H .
influenzae the essential step you will do: 133 . Rademecker complex in EEG is seen in :
a. EEG -
( NBE Pattern 2014 15) a . SSPE ( NBE Pattern 2014 15) -
b. Assess developmental milestones b. vC]D
ao c. Bilateral evoked auditory response c. cCJD
d . Refer for physiotherapy d. Kuru Ref: Nelson 18th ed. ch. 283
PJ* Ref: H 8e/ ch:145/ p:1229, Nelson 18th ed., Harrison 19th p 1011
CJ 134. Regarding amoebic lever abscess which of the
128. All of the following are morphological neurological
complication of measles except: ( NBE Pattern 2014 -15)
following are true: -
( NBE Pattern 2014 15)
a a. Transverse myelitis
a . Multiple abscess are more common
b. Most of the patients have a history of dysentery at
b. Encephalitis the time of presentation
c. Optic neuritis
c. May rupture into the pleural cavity
d. SSPE
d . For asymptomatic luminal carriers diloxanide
Ref: Harrison’ s 18th ed. ch. 192 p. 1603, Harrison 19 th p 1298 furoate is the drug of choice
129. Fagetsignis: ( NBE Pattern 2014 -15) e. Mostly involve the right lobe of liver
a . Tachycardia with hypertension Ref: CMDT-14 ch. 35 p. 1503, Harrison 19 th p 1365
b. Bradycardia with hyperthermia 135. Dawson disease is: ( NBEPattern 2014 -15)
c. Tachycardia with hyperthermia a. SSPE
d . Bradycardia with hypothermia b. Acute disseminated encephalomyelitis
Ref: Family Medicine Board Review 2014, A. Baltimore, p. 6 c. Neuromyelitis optica
d . Paralysis agitans
130. Drug of choice in type I lepra reaction with severe
neuritis is: ( NBE Pattern 2014 -15) Ref: Nelson 18th ed. ch. 283,
a. Systemic steroid b. Clofazimine
B c. Thalidomide d . Chloroquine
Ref: H -18e/ ch: Infectious diseases/ p:1366 , Harrison 19th p 1125

Ans. 125. b. Polymyositis 126. b. Infectious mononucleo 127. c. Bilateral evoked audit... 128. c. Optic neuritis
129. b. Bradycardia wi... 130. a. Systemic steroid 131. a. Patient with score of ... 132. d. Ceftriaxone
264 133. a . SSPE 134. c. May rupture into the... 135. a . SSPE

You might also like